1501

You might also like

Download as pdf or txt
Download as pdf or txt
You are on page 1of 569

l

CONTENTS

t
xl

t'
) 1. Normal Growth and Disorder of Growth & Development'.................. ........................-...011

Ir
NMR}V{AL GROWTH
ANP DISORDER
OF GROWTH 6
PEVELOPMEI\TT

o Growth follows a qig44-shaped curve. The fetus grows_@l1in the EIJ half ,qf-ggstation, thereafter the rate of growth
is slowed down till the baby is born.
o lntheeailypostnatallteriodvelocityofgrowthishighduringthefirstfewmonths@Psc'u).Asecondphaseofaccelerated
growth is during puberty.
o The brain enlarges rapidly during the latter m-onJlrs oJfetal life and early months of pqstqptallife. At birth, thehead
size_is about 65 to 70 percent ofthe expected head size in adults. It reaches g0percent ofthe adult head size by the
07' IGnta 07).
age of 2 y-qqr-q(comed

o Maximumgrowthof lymphoidtissueoccursbetweentheage of 4-Syeans0io-oss).


Weight !.
o The average birth weight ofneonates is about 3 Kg.
o During first few days aftg: birt-h, the newbor4 loses extracell.!'ar fluid equivalent tg about 10% of the body weight.
Most full term infants regain their birth weight by the age of 10 days. l
o Subsequently, they gain weight at a rate of approfmatelyls t9 3O gm per day for the 41qt : months of life.
Thereafter they gain about 400 gm of weight every month for the remaining part of first year@rs).

Birth 3Kg

)
t year 9 Kg {3 x birth weight)6't$s

3 years 15 kg {5xbirth welghlSt**ttt

7 year' 21kg(7 x birth weight)


cl,i,@i:p@:Nat Wlhand oisorder of Growth & Development

e A child gains about 2 kg every year between the age of 3 and 7 years and 3 kg per
year after that till the pupertal
groMh spurt begins.

Height
r Theaveragebirthheightis50cm(ArMsse).Theheightgaininfirstyearisaround25cm@NB13t(so%ofbirthheight@
ot)).

e The height of a child becomes double the birth height at around 4Yz years(NEEr, uPsces) and becomes triple by 12
years of age.
c After the age of 4 years, a child grows at rate of 6 cmlyear4t ez)
.

Birth 50 cm
,':,:.3,mohths "., 60cni .:.

9 months /u cm
.,. /eal..
1 ,:.: 75.im'.::
2years 90 cm
' 4r4 y,ears too
'ma"rtr
For clinical significance upper segment and lower segment should be measured separately. The lower segment extends
from the symphysis pubis to the heels. The lower segment grows rapidly after birth as compared to upper segment
giving rise to the gradual reduction in the upper segment/lower segment ratio with the progression of age.

At birth 1.7 :1.0


: 3 yeai5j:::,...,:i,:.1 i..3ir1;ifi,errl:'.'r..;, :,:::.'..'

At 7 years 1.0 : 1.0


TheieafGr,,. 1.O:1.1. ,.':": '
Note : Height before 2 years of age is measured as length on an infantometer. After 2 years ofage, height (standing
height) is measured.

trdeniifr the inrtrumenf given in the image:

a) Beamscale
b) Infantometer
c) Dynamometer
d) Osteometric board

Ans. is'b' i.e., Infantometer


r It is an infantometer for measuring the length of an infant.
Head cincunrference
o Head circumference is measured from the occipital protuberance to the supraorbital ridge of forehead(pcr04)
which
is the maximum occipito frontal diameter of skull. The head circumference in utero grows by
0.5 cm in first 2 weeks,
0.75 cmin 3'd week and after that 1 cm/week till birth.
o The average head circumference at birth is 35 cm, which becomes 40 cm at 3 months, 43 cm at 6 months(ups c ss),
46
cm at 1 year,48 cmat2 years and 52 cmat12years.
o Headcircumferenceisimportanttoaddtothediagnosisofreurologicaldysfunctionandmicrocephaly.Microcephaly
is defined as a head circumference that measures more than three standard deviation below the mean for age(DNB
rr . llacrocephaly
is defined as head circumference that measure more than two standared deviation greater than
mean tbr age.
CHAp r s a r Nor,mal Grawth and DisorderatG.rg;wth.$,Dqelgpment

Chest circumfenence
o The circumference of chest is about 3 cm less than head circumference at birth.
o The circumference of head and chest are almost same by the age of 9 months to 7 yea/K"'" 06).
Thereafter the chest
circumference exceeds the head circumference.
o The chest circumference is usually measured at the level of nipples.

lVlid-Arm circumference (MAC)


o Mid-arm circumference is measured between acromian and olecranon,i.e. in the middle of upper arm.It is measured
e6' Ar 13).
bY Shakir's taPe@PG

o The shakir's tape is divide lnto three coloured zones, the green zone is above 1-3.5 cm is normal; yellow-orange zone
is between 12.5 - 13.5 cm and represents border line malnutrition; and red zone is below 12.5 cm which represents
severe malnutrition.
o During
l-,!years of age, mid-upper arm circumference remains reasonably static(Pcre6) between15-17 cm among
health children because fat of early infancy is gradually replaced by muscles.
o Kanawati index(ar ra) is calculated by multiplying MAC and head circumference(Ar "). It is used to diagnose
malnutrition. Normal value is > 0-32. Value less than 0.32 indicates malnutrition.

DEVE!.OPIVI ENTAL MI LESTON E5

o Children accomplish maturation of different biological functions (level of development or milestone) at an anticipated
age, with a margin of few months on either sides. Thus, a childt growth (normal or abnormal) can be assessed by
evaluation of these milestones. Important milestones are :-

* 1 month r ln ventral suspension, starts lifting his head.


r In prone position lifts the chinup momentarily in midline.
r Turns head to soundrPc'o',.
r Regards a lighted torch or the face of mother.
r On pulling the child to sit, head lags behind and back is rounded before 4 weeks of age.

a j months r In ventral suspension Iifts his head above horizontal plance {Neck holding)rA' tsPc'or.
r ln prone position lifts his head and upper part of forearms.
r Head Cc,nttillAtoa, Pcto3,ol).

r Starts cooing
r Recognizes mother
r Can follow an object upto l$Oo?Gto3'ot)
r On pulling the child to sit, head lags partially (between 2-3 months). After 3 months head control develops.

* 5 months r Gains control of head in supine position.


r Sits with stltpport(Atls,Pctos)
r Reaches out to an object and holds it with both hands (bidextrous grasp).
r Turns from back to side (Rolls over(Dlv8 '5))
r Transfer the objects from one hand to another (between 5-7 months){rcto&os,04,AtozAttnsut)
':i:aa!rr:.:1,i:r.:i:@r::1-;#5si-

7 months s Hofdsthe obJects with crude grasp from palm (palmar grasp)(Pclos)
* Pivots
* Shows strangers anxietyrPc'07')
* Resista if a toy is pulled frorn his hand.
* gabbleiPc! sl, 02, AuMs 00)
^!

9months * Stand with supportrA/ 66,


T Makes early stepping rnovCrnentswhea feet are placed on the surface of the table.
f, Develops Fincer gtasp6,t3,07,A'ilrse1PctaToo withthe index finger and thumb apposition;
t Develops finer and more Cooridinated hand skills -+ Can scoop on a pellet crudely with his palm.
I Waves bye-bye
I Produces bisyllable sound like baba, mama. :

I Standswithoutsupport@6l8",,:. :

; Tries to feed hirnself with a spoon but spills the contents.


s Tries to remove his eoat and attempts to wear his socks or shoes without suecess.
* Does rnimicry. :

l; Plays a simple ball gamswo'icaoet.


)
T
s Tries to scribble spontaneously (between 12-24 months).

* 15 fionths : Can take several,steps sideways arnd,takes r! steps


::Ie'r'u'rv rqI\!J ua few backward.
illuJ vs!^rYor u' '
f Creeping upstairs
: Can'walk to the,toilet (b€tween 1 5-1 6 months),
u Learns',to feed hiriself With spoon without spllling ltsaontents(Ntusos,DPGoe,PGto4.
) Makesi towei'of 3 cubes. '

* Vocabulary of 4 to 6 words
; Enjoy polysyllabic jargoning

k
Cnap rn rt Normal Grotdlr ondDisordbr of Grulxil,

o 2 yeors walking up and downstairsfiEEr'AilMse4'e3'PGto3), one step at a time i.e., brings both his fuet up on one step
before climbing to the next step (but not by alternate feet).
I Can turn pages of book one at a time?G'o3).
n Draws a vertical or horizontal line.
rt ls able to wear socks or shoes.
r Kicks ball on request
Jumps of the floor with both feet.
n Builds tower of 7 cubes.
I Makes simple r"ni"na", and uses pronounsrD'v8' 2'PGtto'AilMsss,s4).
T ls trainable for toilet and verbalizes toilet needs.
I Can unscrew lids and turn door knobs(Pcr 'o.,.
t Points 3-4 body parts.
't SofionthE '* €Jimbistairs:*ItErnatirig.feelafEs0e.estoa;,,,,,..,
:.a, * MakEStOWerof9ttlb€s{fft{5oel., "'
.a Refers ta self ai,!l'qnd kistiltsfull *amdAillloe-): : ':

a 3yeors I Rides tricy(le{Pcr04'


n Stands momentarily on one foot(aEEr'AilMss8).
Draws a rirrlat*"', o''-*t "tt.
r Can dress or undress himself completely ond succesfully buckles his shoeslAn$sos,DPGos).
I Builds tower of 10 cubes.
& KnowS his gender an d age4rmsos'orcos'PGt04).
r Repeats a sentence of 6 syllables.
x Has a vocabulary of 250 words.
I Counts 3 objects correctly.
n Can withhold and postpone bowel movement.
I Handedness is established(At 1s,rN01t.

*, .'4'yeqr| r. Hopi on oEC foqt{aruie8l;


x Goes down stairs with'alternating fe€t|,rtMieej,
a Draws a cross (plus sign)
,.,. ,' Draws a rectanglerP6"or
,.. *. ldentifi es colots tPGt o)
t

':r':'.r.1* Co,nts4.:B.ennies aic.urately, :ii' :,:,:.. .i r.,,.: :,:,'::. . ::

t Can identi$ Ieft and,right {left.rightrdis{rim.iiritiot)rryJ!4,: , r. .:':i ' .:,,


*, (an use sentence of sii'iriordi(NEtvt' , , ," : ':

11tr'
can tell astory/Petoa).: ''-
s 5 years I Draws a tilted corss (multiplication sign)
t Draws a trianglelAt 't DNB t3' NEEI)
-
5 SkiP5{nrrusrar
a Name 4 colors
t Counts 10 pennies correctly
T Can use sentence of 1 0 words.

I MPORTANT MI LEsTCIN E TUAGES I

A. S'TTINIG

The age of the given child is:

a) 4 weeks
b) 3 months
c) 5 months
d) 9 months

Ano" is ? i.e.,4 weeks


* )
The child in question showing head lag when pulling to sit Age is less than 4 weeks. In newborn, the head
completely lags behing and back is rounded when pull to sit.
C rr.q. p r n n r Narmal Growth and Disorder of Growth & Development

The milestone in t&e given figure develops at:

a) 6 weeks
b) 3 months
c) 6 months
d) 9 months

Ans. is'b' i.e., 3 months


. is showing partial head lag on pulling to sit t occurs at 2-3 months. After 3 months head control devel-
llt-"r"
ops.

The age ofthe given chind:

a) 2 months
b) 3 months
c) 4 months
d) None

"&-xrs. is k'i.e.,4 months


r There is no head lag while pulling to sit ) Age is 4 months.

The given milestone develops at which age:

a) 2 months
b) 3 months
c) 5 months
d) 9 months

Als. is'c' i.e., 5 months


* Child in given figure lifting the head up when pulled up to sit ) Head control in supine position develops at 5

months.

Tlrc age of t&e giveu cleild is:

a) 2 months
b) 3 months
c) 4 months
d) 6 months

i? Kg,:
i€'

Ans. is t'i.e.,6 months


*l Sitting with support develops at 5-6 months.
irowthrrOr,r:

Ttree given milestone develops at which age:

a) 3 months 4'1:
rvC.
%-
b) 4 months '\e,'-
c) 6 months
d) 8 months .J"'
', i.

3€
'El
. ,,% Wiz
-t"\*ei
.*-^
i. qffi+; _>

Ans. is'd' i.e., 8 months


* Sitting without support develops at 8 monihs'
B" VETUTRAI- 5t"' 5p&ru51&I\!

Age of the giveer cldld is:

a) 4 weeks
b) 2 months
c) 3 months
d) 4 months

Ans. is'a' i.e.,4 weeks


e The child is not lifting his head in ventral suspension > Age is less than 4 weeks.

Given rnile stone develops at which age:

a) 2 months
b) 4 months
c) 6 months
d) None

Ans. is'a'i.e,, 2 months


* Child lifts the head in the horizontal plane (in the plane of the body) at 2 months (8-10 weeks).

i{ilestone in t}ee given figure suggest the age:

a) 2 months
b) 3 months
c) I months
d) None

Ans. is'-o" i.e., 3 rxlonths


o child raises his head above the horizontal plane at 3 months ( l2 weeks).
Cueprpn r Normar Grawth and Disorderof Growth & Deveropment
ry
e" p&&&'€ pas,Tt&ru

The age of given child is:

a) 2 weeks
b) 4 weeks
c) 6 weeks
d) 8 weeks

Ans, is'a'i"e., 2 wceks


* The image shows child in prone position with high pelvis and knees drawn under
upto 2 weeks.
the abdomen ) It occurs

?ke given rnilest*ne elevelopes at:

a) 2 weeks
b) 4 weeks
c) 6 weeks
d) 8 weeks

Ans" is'e'i.e., 6 weeks


r The child rs in prone position wirh flat pervis and extended hips + Age is 6 weeks.

?he age clf the ekild in frg.ure is:

a) 2 weeks
b) 4 weeks
c) 6 weeks
d) 8 weeks

Ans. is'd'i.e., 8 weeks


* In prone position face is lifted to 45. by 2 months (g weeks).

Est.i.xrxate t&e age *f given ekild:


a) I month
b) 2 months
c) 3 months
d) None

Ans. is t'i,e., 3 monlhs


a A child iifts his head and upper part of chest by 3 months.

&" STAru mtruG,S{ WeLKt pi,6

Age of the giv*n cXeild is:

a) 2 months
b7 5 months
c) , months
d.1 9 months

Ans" is ?'i,e., 9 rnonths


* A child can stand with support by 9 months.
CHlPrsn r Normal Growth and Disorder of Growth

The given milestone develops at which age:

a) 5 months
b) 9 months
c) 1 year
d) None

Ans. is t' i.e., I year


r Standing without suPpoft develops by 1 year.

The age of given child is:

a) 4 months
b) 6 months
c) 8 months
d) 10 months

Ans. is t'i.e., 10 months


o A child can crawl by the age of 10 months.

Child can do the given rmilcstone atr

a) 5 months
b) 6 months
c) 8 months
d) l0 months

Ans. is'd'i.e", 10 months


* A child walks with support by 10 months.

The given milestone develoPs at:

a) 5 months
b) 8 months
c) 10 months
d) 13 months

Ans. is'd'i.e., 13 months


r A child can walk without supPort by 13 months.
CEApTE n r' Normal Growth and Disorder of Growth & Development

T&e mitrestone ira figure devetrops at:

a) 5 months
b) 15 months
c) 30 months
d) 4 years

Ans. is'c' i.e.,30 months


r Climbing stairs with alternating leel develops at 30 months.

T&e meilestone in &gure rlevelops at:

a) 5 months
b) 15 months
c) 30 months
d) 4 years

Ams, is K'i.e.,4, years


e A child goes down stairs with alternating feet by the age of 4 years.

E. OTHER MILESTONES

?hc given rmilestone develops at:

a) 2 months
b) 5 months
c) 8 months
d) l0 months

Ans. is'b'i.e,, 3 months


o Bidextrous grasp (holding of object with both hands) after reaching to it develops at 4-5 months.

&{i}estone in figurc deve}ops at:

a) 2 months
b) 5 months
c) 9 months
d) I year

Ans. is t' i.e., 9 months


r Grasping object between thumb and index finger (Pincer grasp) develops by 9 months.
Cnep rrn t Normal Growth and Disorderof,Grdwth,&

The milestone in figure develoPs at:

a) 2 months
b) 5 months
c) 9 months
d) 1 year

Ans. is'*d i.e",2 months


o Social smile develops at 2 months.

The age ofthe given infant is:

a) 2 months
;; 3;;;;h;
c) 4 months
d) 6 months

Ans. is W i.e.,6 months


r A child enjoys watching his own image in the mirror bv 6 months.

PETUT'T!CITU ,4NB EMUPTIOIU OF TEET'H

o There are two sets of teeth : temporary and permant.


* Temporary teeth, (milk teeth or deciduous teeth) start appearing at about 6 months, are 20 in number(NnEr): being 4

incisors, 2 canines and 4 molars in each jaw. They start shedding at about 6 years, when permanent teeth start appearing.
o permanent teeth are 32 in number : being 4 incisors, 2 canines (cuspids), 4 premolars (bicuspids), and 6 molars
(tricuspids), in each jaw. 6 perm4nent molars which erupt extra in each jaw without replacing any teeth are called
superadded permanent teeth, while all other permanent teeth are successional permanent teeth.
o Eruption of temporary teeth is called primary dentition and eruption of permanent teeth is called secondary dentition.
rs)
c Temporary teeth start erupting at 6 months age@t .
r:'i. Therefore, between 6-12
o Temporary teeth start falling at,6 years when permanent teeth start appearing(oNr
e8l.
years, there is mixed dentition(o""o
o First temporary tooth to appear (in primary dentition) is lower central incisors and last temporary tooth
to

eruptis-2nd 11.olareclls,titnPetl2).Thesequenceof eruptionislowercentralincisor>uppercentralincisor>upperlateral


eGI0s,linper 02) > 2d molar. Therefore eurption of temporary teeth
incisor > lower lateral incisor > i,t molar > Canine
e0' limpet e7)
is completed by eruption of 2"d molar at 25 months (2 yeats){x*a" '

a First permanent tooth to erupt (in secondary dentition) is 1" molar(NE


Er' ArIMS e7' Iinper 02' PGI 0s) and last to erupt is 3'd

rnolarlrerutael).Lhe sequence of eruption is 1'r molar > central incisor > lateral incisor > 7't premolar > 2d premolar >
17 -25years'
canine > 2,d molar > 3,d molar.Eruption of permanent teeth is completed by eruption of 3'd molar between
a EruptionofteethmaybedelayedinhypothyroidismNEE.'PGI0s'AIIM'eT),hlpopituitarism(ArrMseT),Rickets(xrrr),Down's
AilMS s7),malnutrition and cleidocranial dysplasia.
syndrome(NEEr'
e Permanent teeth appear first in lower jaw.
e Permanent teeth appear a few month earlier in females'
o Third molar appears first in lefi side of lowet jaw and then on right side.
r t Normal Grawth and Disorder of Growth & Development

SHOMT STATIJRE

* If the height of the child is below the 3rd percentile or less than 2 S.D. from
the mean, he or she is considered to be
short stature.

a Causes ofshort stature:

A) Proportionate short stature


Normal variant l) Familial
2) Constitutional delay in growth
Prenatal causes 'I
) lntrauterine growth retardation (l UGp;rccrozl

2) lntrauterine infections
3) Genetic disroders (chromosomal and metabolic)
Posnatal causes 1) Nutritional dwarfism (due to malnutrition)
2) Chronic visceral disease

;; ;;;;;;0",o",, *ro"pituitrism, Juven ire DM)


4) Psychosocial (emotional deprivation, nrafe rnal deprivatislleetuty
$| Dlip-roBorttoiiite,*ortstatuie,' :.:'' -:: :::':: .. l
I

t , ,Wlh.shnrt lirr&s:,.Achon:dronlasia;,hypoehondro;litas;a; chonaioe.itoa l dyjplaiia,.defoiirtiiies


:' due'tolic&ets,,+'nd,.osteogeaesis impti64til. hypot.IittioldJimipn,orl. ...,-'
; ..Wlth:ihort,itrutk i Spondylo:epithyseal dystiasia,,.mucopfy-s rrid*is; muc,olipjdosis;.caries
:'.. spinei hemiv€r:tebrae.

A ehild is with dwarfisrn has following picture what can be the


cause of dwarlism:

a) Hypothyroidism
b) Achandroplasia :-
c) Mucopolysaccharidosis
d) CFI deficiency

Ans. is'd'i.e., GH deficiency


e The givr" pi.tu,. i, ,io*irrg proportionate short stature. Among the given
options, only GH deficiency (hypo-
pituitarism) causes proportionate short stature.

Constitutiomal delay in growth


o It is the most comrnon cause of short oz)
s[nfu1s(aro& in mid childhood period but the ultimate height is normal.
o Their birth weight and height are normal. Strong family
history of parents having short stature in childhood
with
delay in onset ofpuberty is usually present.
e The average growth velocity@Nn n' AI 11' 06)
is normal and ultimate growth
Ttotential is adequate.
The bone age is less than chronological age@NB 13,A1 11,06),
".b.
.-,* Upper segment and lower segement ratio
is normal.
a IGF-[ levels tend to be low for chronological age but within the normal range
for bone age@Ns t3). Growth hormone
and gonadotropin level after puberty are within normal range.
Cnaprun t Narmal Gra$rth.arrd
Differentia I d ia g n os is

Short stature

Normal groMh velocity Altered growth velocity


t
Normal variant short stature
+
Short stature other than normal variant
(see above table)

Bone age < chronological age Bone = chronological age


J +
Constitutional delay Familial short stature

Other important educational facts about short stature


A) Features of short stature due to growth hormone deficiency
1. Ratio of upper to lower segment is normal@EEr).
2. Bone age or epiphyseal developmgnt in less than chronological age by about 2 years@EEr).
3. Children\ are normal inheight and weight at birth@EEr).
4. Delay in growth is usually observed after the age of one year@EEr).

B) Bone age is less than chronological age


c Constitution delay in puberty.
c Marked delayed in hypothyroidism and Ltypopituitarism.
c Moderate delay in malnutrition and chronic illnesses.

C) Bone age is advanced than height age


, ; Down's syndrome t Intrauterine infections o Turner syndrome

D) Infantile type body porportion


r Achandroplasia(Pcl0r) r |uvenile myxedema (hypothyroidism(Pcr0') o Cretinism

E) Reverse infantile body proportion


o Eunuchoidism c Homocystinuria t Frolich's syndrome
o Marfan\syndrome t Klinefelter\ syndrome

BTHAVIORAL AND PSYCHOLOGICAL PROBLEMS

Enuresis
o Enuresis is defined as the voluntary or involuntary repeated discharge of urine into clothes or bed after a

developmental age when bladder control should be established (mostly mental age of 5 years).
r Diagosis of enuresis requires voiding of urine twice a week for 3 consecutive months or clinically significant distress
in child's life as a result of wetting.
r Enuresis is more common in males than females.

o Enuresis in classified into :-


1) Primary (persistent) enuresis
r It is the most common type (90%), in which child has never been dry at night. It is further divided into :-
l) Nocturnal enuresis: It is the most common type of eil)resis and is more common in boys.
ii) Diurnal: This is more common in girls.
2) Secondary(regressive) enuresis
r It is less common (10%). Child has been continent for at least 6 months and then wetting.
C,Ar:'p,ia*,l":',Niirmal Orowth and Dlsorder of Growth & Development

Tneatmemt
a No treatment is given to children below 6 years because of high spontaneous cure rate(Arr3). After 6 years, treatment
include :-
i) Behavioral therapy
x It is the treatment of choice, Most ffictive treatment is bed alarm and motivational therapy with least
chances of recurrence(Ar08). Motivational therapy includes _

a) Keep diary of wet & dry night. d) Restrict fluid at bed time.
b) Void urine before going to bed. e) Avoid punishment.
c) Change wet cloth & bedding. f) Positive reinforcement(ArI5).
x Consistent dry bed training with positive reinforcement has a success rate of B5o/o and bed and pad alarm
systems have a success rate of approximately 75o7o\r ts;
with relapse rate that are lower than those with
pharmacotherapy.

ii) Pharmacologicaltreatment
x It is used when non-pharmacological (behavioral) therapy fails. DesmopressinKarnee) is the drug of choice.
Other drugs used are impramine(uPor) and oxybutinin.
Thumb srickimg
o Thumb sucking is normal in infancy and toddlerhood, i.e. upto 4-5 years@e oz, 0s' ripmer el) .It is a
behavioral problem,
which may be self soothing, i.e. a source of pleasure(uP 07' 0s' lipnet e1).
o Presistent thumb sucking in adolescence is more common in girls and may be sign of insec rtrityop 07,0s, ripnet sl)
.
o Presistent thumb sucking in older children can cause dental malali gnment(up07,0s,lipnerei). For lessening the
effect on
dentition, thumb sucking should be discontinued before 8 years of age(ez.ussr, 7s,PGI7e).
s Treatment is not required till the age of 4-5 years. After that behavioral therapy in the form of
Ttositive reinforcement
(giving reward for not sucking the thumb) is treatment of choice. Negative reinforcemenf (application
of noxious
agents to thumb) is second line of treatment.
Breath ho!ding spells
* Breath holding spell is a paroxysmal event occuring in 0.1% - 57o of healthy children from the age of 6 months
to 6
szl
featS1IIMs .
* The name for this behaviour maybe misnomer in that it connotes prolonged inspiration. Infact, breath-holding
occurs
during expiration and is reflexive (not volitional) in nature.
e There are two major types of breath holding spells :-
A) Cyanotic form (cyanotic spells)
x This is more common and is provoked in response to frustration and anger precipitated by upsetting or scolding
infant/child.
x Cyanotic spells are dre to central sympathetic overactivity.
x Clinical features include generalized cyanosis(A[Ms e7), apnea, forced expiration, opisthotonus, shrill cry and
bradycardia. Seizures may occur due to cerebral hlpoxia, but antiepileptics are not required 6IIMI s7).

x The only treatment is support and reassurance to parents.


B) Pallid form (Pallid spells)
x These are initiated bypainfut experience, e.g falling and striking the head.
x Pallid spells are due to excessive central parasympethetic activity.
x Clinical features include pallor, apnea, loss ofconsciousness, hypotonia, seizures and bradycarda.
x Treatment includes support and reassurance of parents. AtropinerA IIMS s7) may be used in refractory cases.
e Iron supplementation may be useful in some children with breath holding spell5(oxr r:;.
Learnimg disability
a Learning disability (LD) is defined as a disorder in one or more of the basic psychological processes involved in
understanding or in using language, spoken or written, which may manifest itself in an imperfect ability to listen,
speak, read, write, spell or to do mathematical calculations. Learning disability is suspected when there is unexpected
underachievement in adequate educational settings.
Cnapr,rt r Normal

o The DSM-IV defines three academic skill specific learning disorders:-


t Readingdisorder
t Mathematic disorder
t Disorder in written exPression
o Another subtlping is language-based LD commonly known as dyslexia (problem in reading, spelling and written
expression) and nonyerbal LD (deficrts in arthmetic and spacial cognition). Dyslexia is the most common type of
os). fhs
learning disability. Children with dyslexia have problem in reailing, spelling and written expression\lrMs
important clues to dyslexia in school age children include delayed language, trouble rhymic words, mispronunciation,
hesitation, work-finding difficulties and poor spellings with presence of letter reversal. Reading is slow, inaccurated
and labored.
Attention deficit hyperactivity disorder (ADHD)
o ADHD is one of the most common psychiatric problem in school age population. It is three times more common in
boys than girls.
13'04'AttMs08'oz)
r ADHD is characterized by an age inappropriate hyperactivitytdtts'04'ArrMs08'0'), impulsiveness\l and
01' AIIMS 08' 02).
inattentionat 1s'
Hyperactivity is usually the first symptom to be noticed.
o The disorder is divided into three classes :-
r Class I (most common) -+ AII three symptoms are present i.e., hyperactivity, impulsiveness and inattention.
r Class II + First two symptoms are present, i.e., hyperactivity and impulsiveness.
r Class III (Least common) -+ Only inattention is present.
ts' oe' ilIMs 0s' 02)
impulsivenes sal 13' 04' AIIMS 08' 02)
inattentivenessar
o Child with ADHD presents with hyperac tivity,at , arrd
08),
)3,04, AilMs 08,02), restlessness 6IIM; 0s)
, decreased self control, distractability{t't
ss)
, desira to play outsiile(ArrMs difficulty
os)
in concentration, difficulty in playing(AzMs 0s),
dimculty in sitting, inability to wait for his turn while playing(efl'us
and frequently interferes others.

r Treatment options are :-


1) Non-pharmacological : Psychosocial and behaviorol therapyarnrs o<).

2) Pharmacological treatment: Psychostimulant ilrugs are most efiective treatmenf. Methylphenidate(Pcro8) is the
drug of choice. Amphetamines(Pcro8) and atomoxetine(Pcr08) are alternatives.

Pervasive development disorders


r Pervasive development disorders include several clinically similar conditions which are characterized by three s)'mptom
clustors: -
1) Impairment of social interaction : - e.g., Lack of social smile or eye contact.
2) Impairment of communication (language and non-verbal) : - Language problems, delayed or absent speech.
3) Restricted repititiye and styerotyped behavior interests and. activities: - e.g., Sterotlpy.

Autism
r Autism is a pervasive development disorder (PDD) which is characterized by: -
i) Marked impairment in social interaction@Gror'00)
r There is no social smile(MP06) or eye contact. These children prefer to play alone with inanimate objects and do
not like to mingle with others(ArrMs0r) (do not make friends). There is lack of attachement to parents and absence
ofseparation anxiety.
ii) Impairment in communication?Gr0l'0o'Attus06'01) (language well as non-verbal communication) as

r Delayed or absence speecharMs06), lack of verbal or facial response to sounds or voices, abnormal speech
patterns, echolalia, perseveration and prominent reversal (referring to selfas 'you' and to others as'I').

iii) Sterotyped and restricted repetitive behavior, interests and activities


r Mannerism, Sterotype behavior (head banging, body-spinning, rocking, clapping, watching own handarMs0').
iv) Other features
r Mental retardation(MP 06) (it 50 -7 5o/o patient)
r Epilepsy (in 25-35o/o)
Ap r E R I Normal Growth dnd Disorder of Growth & Development
r Hyperkinesis
r Inability to concentrate 6trMs06).

o Onset of symptoms of infantile autism (usual form of autims) is before 2.5 - 3years@P06). If onset of symptoms
are
afier 3 years, it is referred to as childhood autism.
r Autism is more common in boys and among low socio-economic groups(pcr04).
I Presently, the cause of infantile autism seems to be predominantly biologi calecl0l).

Other pervasive development disorders


c Two other pervasive development disorders are : -

Rett's syndrome
r Age of onset is around 5 months.
: Development may proceed normally until 1 yr of age, when regression of language and motor milestones become
apparent.
r This is the characteristic features, that they begin to loose their acquired skills, e.g., cognitive and head growth
is normal during early period after which there is an arrest of growl6rez,uso,rl.
I Acquired microcephaly (Decleration of head growth due to significantly reduced brain weight(-Arus0i,).
r Most children develop peculiar sighing respirations with intermittent periods of apnea that may be associated
with cyanosis -+ Breath holding spells(ar'uso'rt.
s Austistic behaviour is a typical finding in all patienfs -+ Impaired social interaction, language and
o3).
communicationazM's
r Generalized tonic-clonic convulsions occur in the majority.
r Feeding disorder and poor weight gain are common.
r Death occurs in adolescence or in the 3.d decade.
: Cardiac arrhythmias may result in sudden, unexpected death.
Asperger syndrome
r Qualitative impairment in the development of reciprocal social interaction after the age of 3 years.
r More common in males
r Normal intelligence, The only pervasive development disorder in which intelligence is normal.
r Eccentric interests.
s No language impairments that characterize autism.
r Children with Asperger syndrome appear to be at high risk for other psychiatric disorder.

ADOLESCENCE
Adolescence
r Adolescence is defined as the period from the onset of puberty to the termination of physical growth and attainment
offina1 adult hight, i.e. adolescence is considered as a period oftransition from childhood to adulthood. Adolescence
is usually the period 10 to 20 yr@NB 13' AI0s)
.

Puberty
r Puberty is the biological process in which a child become adult, i.e. biological process which occurs during transition
from childhood to adulthood. Changes include appearance of secondary sex characteristics and development of
reproductive capacity.

Adolescent -) 1 0-1 9 years


x Eariy adolescent ) 1 0-1 3 years
a Middle adolescent -+ i 4-1 5 years
t Late adolescent -+ 1 6-1 9 years
t \iouth, ', , :+ t5-24yeats
t Young people -) 10-24 years
Culprpn t Narmal Growth and

In Girls
o In girls, puberty starts with onset of breast development (appearance of breast buds), called thelar che(Al 13. DNB 12,
13' AI 08' PGt ee),
between 8 and 13 years (corresponds to Tanner SMR stage - 2at 1s)) . This is followed by appearance of
pubic hair (pubarche) and axillary hair (adrenarche)Atset. Pubarche is followed by peak growth velocity, immediately
after which menarche (onset of mensturation) occurs.

Theldrche _+ Pubarche & adrenorcheote6) _.> peak growth velocity'nrn, ete6sa ) menarcheNEEr, At oo, s8)

Preadolescent Preadolescent
Sparse, lightly pigmented, straight, Breast and papilla elevated as small
medial border of labia mound; diameter of areola increased
Darker, beginning to curl, increased Breast and areola enlarged, no contour
amount separation
Coarse, curly, abundant, but less than in Areola and papilla form secondary mound
adult
Adult feminine triangle, spread to Mature, nipple projects, areola part of
medial surface of thighs general breast contour

ln Boys
o In boys,frst sign of puberty is enlargement of testis\I se),
at around 9/z years. This is followed by penis enlargement
after which pubarche (pubic hair growth) starts. Then occur peak growth velocity, adrenarche (axillary hair growth)
andfacialhair growth (beard){ectsat, in that sequence.

Testicular enlargement -+ Penis enlargement -+ Pubic hair growth -+ Peak growth velocity + Axillary hair --> facial hair.

1 None Preadolescent
2 Scanty, Iong, slightly pigmentedrntrsr Minimal change/enlarge- Enlarged scrotum(A"'), pink, tex-
ment ture altered
3 Darker, starting to curl, small amount Lengthens Lar;1er
4 Resembles adult type, but less quantity; Larger; glans and breadth Larger, scrotum dark
coarse, curly increase in size
Adult distribution, spread to medial surface Adult size Adult size
of thighs

. Avlrage birth helght in lndia :50 cm.


:li;.,..A.iightirtachildfi.diibliinaArtineighi'o:4Vzyearsof,ag€.:"',".,...,,.,,.:,"''
o Gain of height in 1't year of life: 25 cm (500/o).

ffi;*m :aiiwia*41z
y9or:,:,.6-,,,ecm per year:-r,,,,: :,,,., :,,::: i,
; Body weight is tripted (3 times of birth weight) at : 1 yea r of age.
y ;.;.; y.w*tgfii iiuiaru$les 14t e;of b;ii{Ew;ight} iit : 2 yea,i.':of age,','
* Body weight is 5 times the birth weight at:3 years of age.
. Weight gain during 2'd year of life:3 kg.
. Head circumference is measured from: Occipital protuberance to supraorbital ridge of forehead.
Curprrn r Normal Growth ond Disorder of Growth& Development
I
o Maximum growth of lymphoid tissue occurs : Between 4-B years.
. lmportant causes of proportionate short stafure : Familial, constitutional delay, IUGR, malnutrition. hypopituitrism (GH deficiency).
c lmportant causes of disproportionate short stature : Hypothyroidism, achondroplasia, mucopolysaccha ridosis.
o Features of constitutional delay in growth:Normal birth height, bone age < chronological age, normal growth velocity, positive family
history, IGF-1 is low for chronological age but normal for bone age.
o Features of familial short stature i low birth weight, bone age is normal for chronological age, positive family history, normal growth
velocity.
& Short ststare due to GH deficiency: Normal body porportion, bone age < chronological age, normal birth height and weight, altered
growth velocity.
@ Most common cause of short stature: Constitutional delay in growth.
* lnfantile body proportion is seen in: Achondroplasia juvenile myxedema (hypothyroidism), cretinism.
* Reverse infantile body proportion is seen in: Marfan's syndrome, Homocystinuria, Klinefelter syndrome, Frolich's syndrome, Eunuchoidism.
w Difficutty in expressing in writting, reoding problems, mathematic mistakes, problem in spelling: Specific learning disability.
w Characteristic triad of ADHD : Hyperactivity (first symptom), impulsiveness, inattention.
a Other features of ADHD: Disturbing others, difficulty in playing, does not listen to teaching, restlessness, uncontrolled desire to play
outside.
a Not a feature of ADHD : Mental retardation.

o Drugs used for ADHD: Methylphenidate (DOC), amphetamines, atomoxetine.


e lmportant features of autism: Impaired communication/social interaction, stereotype repetitive behavior, impaired imagination,
delayed speech, low socio-econom.ic aroup, onset before 3 years.
* Not o feature of autism :Vision problem.
w Features of Rett's syndrome: Repetitive hand wringing movement with autistic behavior, breath holding spells, microcephaly.
w Lower recunence of nocturnal enuresis is seen when treated by : Behavioral therapy (bed alarm & motivational therapy).
& Treatment of choice for nocturnal enuresis in a 5 years old child: No treatment (no treatment is required up to 6 years).
w Drugs to treat nocturnal enuresis: Nasal desmopressine (drug of choice), imipramine, oxybutynin.
* True about breath holding spells: May be cyanotic or pallid, treatment is reassurance, atropine and iron may be used, antiepileptics

. i;::;::r:r"Uiioor*u,*: A source of pleasure. may be a sisn of insecurity, may cause matocctusion, no treatment is required upto
4-5 years.
@ To avoid malocclusion of teeth, thumb sucking should be terminated by:7-B years.
& Pica referes to: lngestion of non-nutritive substances.
@ Cretinism (hypothyroidism) is characterized by: Disproportionate short stature (short stature with short limbs).
w Chitdhood disorder which improves with age: Temper tantrum.
& Adolescence period extends from : 10 20 years (starts ,,.10 ,".t:,
, .
& First sign of puberty in girls:'lhelarche (development of breast buds).

@ Sequence of events at puberty in girls:fhehrche + pubarche/adrenarche -+ peak growth velocity -+ Menarche.


@ Peak growth velocity in girls is seen just after: Appearance of pubic hair (pubarche) and axillary hair (adrenarche).
w Peak growth velocity in girls is seen iust before/at the time of : Menarche.
& First siqn of puberty in boys : Enlargement of testis.
w Sequence of events at puberty in boys :Testicular enlargement -+ penis enlargement + pubic hair growth -+ peak growth velocity
-+ axillary hair -+ facial hair.

* Thelarche is: Hormone induced breast enlargement in girls.

'TI
I Disorder of Growth & Development

QUESTIONS

| ^L, Child rolls over by: (CET luly 15 pattern)


a) 3monthsb) 5 months
c) 7 months d) 8 months
NORMAT GROWTH Which about development is not
13. true: (ATIMS May 9s)
a) Pincer grasp at 3 months
l. The height of a child is doubie the birth height at the b) Sitting at 6 months
age of: (All India Dec.1t Pattern) c) Social smile at 3 months
a) 1 year b) 2 years d) 2year old can use pleurals
c) 4 years d) 6 years t^ Pincer grasp is attained at rnonths:
Z. Average gain nfheight in first year is ? (All India Dec.13 Pattern)
(CET Nov.13 Pattern) a)4 b) 10
a) 25 cm b) 50 cm c) 12 d) 18
c) /5 cm d) 100 cm 15. A 3 year old child can do all, except: (ArrMS May e3)
3" Height of child aequire 100 crn in: a) Eat with a spoon
(All India Decj5 pattern) b) climb stairs running
a) 2.5 year b) 3.5 year c) Walk properly
c) 4'5 year d) 5'5 year d) Vocabulary of20-100 words
4. Height ofchildren in 2,10 years ofage is increased t()" Child knows his/her sex by age of:
byt (Ai e7) (All India Dec15 pattern)
a) 2 cmlyear b) 4 cmlyear a) 2year b) 3 year
c) 6 cmlyear d) l0cm/year c) 4year d) 5 year
5" From 6 weeks to 12 weeks.". trnfant weight increases 17. A child speaks sentences at the age of ?
@ aft (All India DecI5 pattern) (CET Aug. 12 pattern)
il 3bild b) a0 g/d a) 6 months b) 1 year
c) s0 g/d d) 60 g/d c) 18 months d) 2 years
6. Infant bodyweight istripled byageof: 6ilMSe6) 18. A female child has recently learned to eat with
a) 5 months b) 11 months spoon without splling, to dress and undress herself
c) 2 years d) 18 months with supervion and to understand that she is a girl.
7. I{eightofnewbornquadruplesbyr These skills are first mastered between the ages of:
(All India Dec.t3 pattern) (AIIMS Nov 05, DpG 09)
a) 6 months b) 1 year
a) 2and3years b) 3and4years
c) 2 years d) 3 years
c) 4and5years d) 5and6years
lo
8. Anthropometric assessment, which does not show Purposeful movement is st*rted as:
much change in 1-4 years: (PGI Dec 96) (All India Dec15 pattern)
a) Mid arm circumference a) 6 month b) Infant
b) Skin fold thickress c) 8 month d) 9 month
c) chest circumference- Head circumference ratio z{',t. Child draws triangle at what age?
d) Height (CET Aug. 13, All India Dec.13 pattern)
a) 3 years b) 5 years
DEVELOPMENTAL MILESTON E
c) 6 years d) 7 years
21. A boy can grasp a rattle & recently he become able to
9. Sitting without support appear$ at whiclr rnonth: transfer objects, hand to hand. He can do: @Gl Nov 14)
(All India Decl5 pattern) a) Babble
a) 5months b) 6 months b) Say'mama'or'dada
c) 10 months d) 12 months c) Sit without support
10. Neck hotrding comes at lvhat a.gei (Alt hrlia Decl5 pattertr) d) Stand with support
a) 2 months b) 3 months
e) Able to walk
c) 4 months d) 5 months 22. A child climbs with alternate steps, builds a towerg:
11. Sitting in Tripod position at which month 9 cubes, tells '[' but not his name and can not say his
?
(All India Dec15 pattern)
age and sex, the probable age is: (AtrMS May 0t)
a) 5 months b) 6 months
a) 36 months b) 24 months
c) 8 months d) 9 months
c) 30 months d) 48 months
Cnlprr.n r

23. A child makes tower of 4 cubes at: b) Read a sentence


(All India Dec.lj Pattern) c) Ride a bicycle
a) lyear b) lr/z years d) Copy a triangle
c) 3 years d) 4 years 34. A child has a vocabulary of4-6 words however the
24. Which of the following cannot be done by 3 years main mode of communication and social interaction
old child: (NEET Dec.12 Pattern) continues to be non-verbal what is the most likely
a) Draw a triangle b) Draw a circle developmental age of the child: (Ar 12)
c) can arrange 9 cubes d) cango up and a) 12 months b) 15 months
down c) 18 months d) 24 months
e) Stands on one foot for 5 second 35. Object permanence milestone develop at:
25. Handedness develops by age of: (AlllndiaDecl| Pattern) (CET Nov 15 Pattern)
a) 2years b) 3 years a) 6months b) 9 months
c) 4 years d) 5 years c) 12 months d) 15 months
26. Which of the milestone develops first: (Ar 07)
a) Mirror play b)crawling ERUPTION OFTEETH
c) creeping d) Pincer grasp
27. A baby has recently developed mouthing but has not 36. The primary dentition begins to show teeth eruption
developed stranger anxiety. Likes and dislikes for byt (All India Dec.13 Pattern)

food. What is the most appropriate age of this baby: a) 6 weeks b) 12 weeks
(At 07) c) 6 months d) l2months
a) 3 months b) 5 months 37. At what age do first permanent teeth appear ?
c) 7 months d) 9 months (CET Nov. 13 Pattern)

28. A normally developing l0 months old child should a) 5 years b) 6 years


be able to do all of the following except: (Ar 06) c) 7 years d) 8 years
a) Stand alone 38. Ist permanent teeth to appear: (NEET Dec.13 Pauen)
b) Play peak to boo a) Molor b) Premolor
c) Pick up a pellet with thumb and index finger c) Incisor d)cannine
d) Build a tower of 3-4 cubes 39. Milk teeth-Total no. in hurnan being:
29. An 8 week infant can do all the following except: (All India Dec.14 Pattern)
(PGI May ru) a) 20 b) 28
a) Head control c) 32 d) 24
b) Lift its head up to horizontal line in ventral 40. Delayed dentition is seen in alllexcept:
suspension (NEET Dec.12 Pattern)
c) Follows red object upto 180 a) Down syndrome
d) Social smile b) congenital hypothyroidism
e) Turns head towards sound c) Rickets
30. A two month old child is able to: (At 04) d) All of above
a) Show a positive parachute protective reflex
b) Hold head steady in seated position MISCELLANEOUS
c) Lfft head and chest of a flat surface with extended
elbow 41. When tCF and ECF of child becomes equal to adult
d) Sustain head level with the body when placed in person: (NEET Dec.12 Pattern)
ventral a) 1 year b) 2year
suspension c) 3year d) 4year
31. A child is able to say short sentences of6 words: 42. Upper segment to lower segment ratio in 3 yr age
(NEET Dec.12 Pattern)
child is: (CET June 14 Pattern)
a) 2years b) 3 years
a) 1.2 b) 1.3
c) 4 years d) 5 years
c) 1.4 d) 1.6
32. A neonate is able to: (AlllndiaDec.ilPattern)
43. Bilabial consonants are: GEf luly 15 Pattern)
a) Fix his gaze at a object 8 to 12 inches apart a) PBSL b) PBMW
b) Focus on bright object
c) MNLO d) TLMW
c) Left his heaJ& chest on elbow '
d) Roll from side to side
33. A year 6 old child with IQ of 50. Which of the
following can the child do: @:.:uS May 07, Nov 06, At 07)
a) Identify colours
C r*e.s.i"x&?': ilormal Growth and Disorder of Growth & Developrnent

51" A 9 year old child is restless. [Ie is hlperactive and


his teacher cornplaints that he does not listen to the
DE\lELOPMENT teachings. Disturbs other students, he also shows
less interest in playing. The likely diagnosis is:
(AIIMS May 02)
SHORT STATURE
a) cerebral palsy
M. b) Attention deficit hyperactive child
A child is below the third percentile for height. His
growth velocity is normal, but chronologic age is
c) Deliruium
d) Mania
more than skeletal age. The most likely diagnosis is:
(Ar 11,06)
52. A 10 year old child is always restless, inattentive to
a) constitutional delay in growth study and always wants to play outsite. Farents are
b) Genetic short stature extremely distressed. What would you adivise:
c) Primordial dwarfism (AIIMS Nov 08)
d) Hlpopituitarism a) It is a normal behaviour
45" A ehild's growth variation io normal but trone devel-
b) Behaviour therapy
opment is not acc to the chronological age. What ls
c) It is a serious illness requires medical treatment
d) Needs change in environmenl
diagnosis ? (CET Nov. 13 Pattern)
a) Genetic
5.3" True about autisrn; (All India Dec.1j Pattern)
b) Dwarfism a) Occurs in high economic striata
c) constitutionaldelay b) Normal communication
d), Familial short stature c) Starts before2-3 years ofage
d) More common in girls
46_ The most €ommon cause of short stafure is (AI 07,08)
a) constitutional 54. A 3 year old tloy with norrnal developrnental mile-
b) Systemic diseases stones with delayed speech and diificulty in com-
c) Hypothyroidism munication and concentration. lle is not rnaking
d) Growth hormone deficiency friends. Most probable diagnosis is:
47. Which of the following is true about constitutional
a) Autism
@I.MS MaY 07' AI 07)

delay in growth? (CET Nov. 14 Pattern)


a) b) ADHD
Neonates with constitutional delay show anomalies
at birth
c) Mental retardation
d) Specific learning disability
b) IGF-1 Ievels are low for chronological age
c) Bone age is normal 55. Which of the following is not true atraut the autistic
d) IGF-I levels are low for bone age specific disorder: (All tndia Dec.14 pattern)

48. Short stature, secondary to growth hormone defi-


a) Impairedcommunication
ciency is associated withl
b) Impaired imagination
(NEET Dec.12 pattern)
a) Normal body proportion.
c) Language developmental delay
d) Vision problems
b) Low birth weighr
c) Normal epiphyseal development The following are characteristic of autism except:
d) Height age equal to skeletal age (All lndia Dec. 15 Pattern)
a) Onset after 6 years ofage
b) Repetitive behaviour
PSYCHOLOGICAL PROBLEM5 c) Delayed language development
d) Severe deficit in social interaction
49_ A 14 year old boy has dtffhculty in expressing himself
in writing and rnakes frequent spelling mistakes, 57. A two year old girl child is brocght to the out patient
does not follow instruction and cannot wait for his with features of hand wringing stereotype move-
turn while playing a garne" t{e is likely to tre sufering rtents, impaired language and communication de-
fiom: velopment, breath holding spells, poor social skills
(AIIMS Nov o5)
a) Mental retardation and deceleration of head growth after 6 mont&s of
b) Lack ofinterest in studies age. The most likely fiagnosis is: (AIIMS Nov 03)
c) Specific learning disability a) Aspergert syndrome
d) Examination anxiety b) Rettt syndrome
c) Fragile x-syndrome
50. Attention de{icit hyperactivity disorder is character-
d)colarad syndrome -
ized by: (All India Dec.13 Pattern)
a) Hyperactivity Sg, A boy does not speak at school but has normatr
speech at other placeo. IQ assessment is normal. The
b) Impulse activity
c) Poor attention diagnosis is: (All India Dec15 Pattern)

d) All of the above a) Selective mutism b) ADHD


CHAPTER:':'I

c) -{utism d) Dyslexia MISCELLANEOUS

ENURES!S 68. Arm span and height become same at what age (year):
(All India Dec15 Pattern)
59. 5 vear old child bed wetting Rx of choice: a)9 b) 11
(All lndia Dec.1j Pattern) c) 13 d) 1s
a) No treatment b) Imipramin 69. Pica refers to: (NEET Dec.12 Pattern)
c) Desmopressin d) Motivational therapy a) IU sucking
60. Lowest recurrence in nocturnal enuresis is seen b) Thumb sucking
with: 61 0B) c) Foreign object being put in the mouth
a) Bed alarms b) Desmopressin d) None ofabove
c) Imipramine d) Oxybutynin
61. Treatment ofnocturnal enuresis in a 14 year old ADOLESCENT
child is: (All India Dec15 Pattern)
a) Positive reinforcement
70. WHO defines adoleseent age between;
b) Punishment
(All India Dec.14 Pattern)
c) Bed alarm
d) Desmopressin a) 10-19years b) 10-14years
c) 10-25years d) 9-14years
62. Which of the following nasal spray is very effective
in eontrol of enuresis: (All India Dec.14 Pattern)
7r. Adolescent starts at the age of ? (CET Aug lj pattern)

a) Pitressin b) Desmopressin a) 7 years b) 10 years

c) Lipressin d) None ofthe above c) l4years d) lTyears

63. The behaviour therapeutic falls in management of


72. Peak growth velocity in adolescent girls is seen just
enuresis, The pharmacological drug of choice for
after: (AIs6)
this case is: (uP 07)
a) Appearance ofpubic & axillary hair
a) Phenltoin b) Diazepalm b) Breast enlargement

c) Imipramine d) Alprax c) Onset of menstruation


d) Enlargement of external genitalia
73- First sign ofpuberty in girlsr
BREATH HOLDING SPELLS
(CET Aug. 1i, 12 Pattern, PGI Dec 99, AI 08)

64. True regarding breath holding spells is all except:


a) Puberchy

(AIIMS Iune 97)


b) Thelarchy

a) Antiepileptic treatment is necessary


c) Growth spurt
b) Atropine is sometimes used
d) Menarche

c) Attacks ofcyanosis can occur 74. First sign of pubertal development in female isr
d) Occurs between 6 months to 5 years (All India Dec.13 Pattern)

65. T?eatment of breatlr holding spells is?


a) Breast enlargement

(CET Nov.1j Pattern)


b) Onset of mesturation

a) Pyridoxine c) Enlargement of pubic hair

b) Zinc d) Maximum growth velocity


c) Iron /5. What is the order of puberty: (Ar 2K)

d) Molybdenum a) Telarchy-pubarchy-menarchy
b) Puberchy-telarchy-menarchy
c) Puberchy-menarchy-telarchy
THUMB SUCKING d) Adrenarchy-telarchy-pubarchy

66. Which is incorrect about Thumb sucking: 1rMpERsi) 76, Order of development of secondary sexual charac-
a) can lead to malocclusion teristic in male: (PGr e6)

b) Is a source ofpleasure a) Testicular development--pubic hair--axillary hair--


c) Is a sign ofinsecurity beard
d) Must be treated vigorously in the first year b) Pubic hair--testicular development--axillary hair--
beard
67. To avoid displacement of permanent teeth, finger
c) Testicular development--beard--pubic hair-,
sucking should be terminatedbyt 1ru zo, AilMS 78, sl)
axillarlhair
a) 8 years b) 5 years d) Axillary hair--beard--pubic hair--testicular
c) 3 years d) 2 years development
Cueprrnl Normal,Grawth
c) Autism d) Dyslexia MISCELLANEOUS

ENURESIS 68. Arm span and height become same at whatage (year)t
(All lndia Dec15 Pattern)
59. 5 year old child bed wetting Rx of choice; a)9 b) 11
(All India Dec.13 Pattern) c) 13 d) 15
a) No treatment b) Imipramin 69. Pica refers to: (NEET Dec.12 pattern)
c) Desmopressin d) Motivationaltherapy a) IU sucking
60. Lowest recurrence in nocturnal enuresis is seen b) Thumb sucking
with: (Ar 08) c) Foreign object being put in the mouth
a) Bed alarms b) Desmopressin d) None ofabove
c) Imipramine d) Oxybutynin
61. Treatment ofnocturnal enuresis in a 14 year old ADOLESCENT
child is: (All lndia Dec15 Pattern)
a) Positive reinforcement
74, WHO defines adolescent age between:
b) Punishment
c) Bed alarm (All India Dec.14 Pattern)

d) Desmopressin a) 10-19years b) 10-14years

62. c) 10-25years d) 9-14years


Which of the following nasal spray is very effeetive
in control ofenuresis: (All India Dec.14 Pattern)
7L, Adolescent start$ at the age of ? (CET Aug 1j pattern)

a) Pitressin b) Desmopressin a) 7 years b) 10 years


c) Lipressin d) None ofthe above c) 14 years d) 17 years

63, The behaviour therapeutic falls in management of


72. Peak growth velocity in adolescent girls is seen iust
enuresis, The pharmacological drug ofchoice for afterl (At s6)

this case is: (uP 07)


a) Appearance ofpubic & axillary hair
a) Phenytoin b) Diazepalm b) Breast enlargement
c) Imipramine d) Alprax c) Onset of menstruation
d) Enlargement of external genitalia
73, First sign ofpuberty in girlo:
BREATH HOLDING SPELLS
(CET Aug. 13, 12 Pattern, PGI Dec 99, At 08)

64. True regarding breath holding spells is all except:


a) Puberchy

(AIIMS Iune 97)


b) Thelarchy
a) Antiepileptic treatment is necessary
c) Growth spurt
b) Atropine is sometimes used
d) Menarche
c) Attacks ofcyanosis can occur First sign of pubertal development in female isr
d) Occurs between 6 months to 5 years (All lndia Dec.13 Pattern)

55. Tleatment of breath holding spells is? a) Breast enlargement

(CET Nov.1j Pattern)


b) Onset of mesturation
a) Pyridoxine c) Enlargement of pubic hair
b) Zinc d) Maximum growth velocity
c) Iron. 75. What is the order of pubertyr (Ar 2K)
d) Molybdenum a) Telarchy-pubarchy-menarchy
b) Puberchy-telarchy-menarchy
c) Puberchy-menarchy-telarchy
THUMB SUCKING
d) Adrenarchy-telarchy-pubarchy
66. Which is incorrect about Thumb sucking: (ilMpERsl) 76, Orderofdevelopment ofsecondary sexualcharac-
a) can lead to malocclusion teristic in male: (PGr e6)

b) Is a source ofpleasure a) Testicular development--pubic hair--axillary hair--


c) Is a sign ofinsecurity beard
d) Must be treated vigorously in the first year b) Pubic hair--testicular development--axillary hair--
beard
67 . To avoid displacement of permanent teeth, finger
c) Testicular development--beard--pubic hair--
sucking should be terminated by; 1tct zs, AtMS 78,8i)
axillary'hair
a) 8 years b) 5 years
d) Axillary hair--beard--pubic hair--testicular
c) 3 years d) 2 years
development
77. fuakgowth r,elocifyin adolescentgirt is indicated 87- A newborn baby has a head circumference of 35
bp (CET Nov. 14 Pattern) cms. at birth, His optimal head circumference will
a) Breast enlargment be43 cms at: (UPSC 9e)
b) Axillary hair a) 4 months of age b) 6 months of age
\ c) Public hair c) 8 months of age d) 12 months of age
\
d) Iust before commensement of menarche 88. Ia
l healthy chil4 the head and chest circumference
a
78" I ltlhat is thelarche (NEET Dec.12 Pattern) equal each other arouad the age of: (Karnat 06)
a) Pubertal breast enlargement in boys a) 3-6months b) 6-9 months
b) Breast enlargement in pregnancy c) 9-l2months d) l2-l5months
c) Hormone related breast enlargement in girls
d) Post hormonal therapy breast enlargement in
V/eight gain in the second year of life is: (cMC 0s)
a) lkg b) 2kg
postmenopausal females
c) 3kg d) akg
pkst sign of pub€rty in feaaalel (AlI India Dec15 pattern) e) 5kg
a) Thnner stage I
A normal healthy child has a height of t{X) cm and
b) Tanner stage II
c) Pubic hair
we:ghs f 6 kg. What is his most Lkely age? (upsc 06)
a) 3 years b) 4 years
d) Axillary hair
c) 5 years d) 6 years
88" Characteristics of SMR-2 in boysdAil rn dia Dect5 pattern)
91. Shakir tape is used for: (Delhi 96)
a) Appearance ofpubic hair a) Measurement of height
b) Appearance of axillary hair b) lvleasurement of length of infant
c) Enlargement of scrotum c) Measurement of midarm circumferance
d) All of above d) Measurement of the learning ability of child
A child plays a simple ball game at: (Manipal 08)
QUESTIONS OF VARIOUS OTHER a) 52 weeks b) 36 weeks
EXAMINATIONS c) 12 weeks d) 48 weeks
W'hea achild is not able to perform the following
motor functions such as skippinS walking on heels,
GROWTH & DEYELOPMENT hoppingin place orgoing forwards in tandem gati,
his motor deyelopment is considered to be below:
81. Postnatallywheaisthe grewthvelocityma-imrrm ?
a) 3 years b) 4 years
(UPSC 06)
c) 6 years d) 8 years
a) In the firstyear oflife Child changes a rattle from one hand to another at
b) In the secondyear oflife the age of: (Comed 07)
c) In the seventhyear oflife a) 3 months b) 6 months
d) In adolescence c) 9 months d) 1 year
82. 90% ofbrain growth is achieved by the: 95. A normal infant sits brieflyleaning forward on her
(Comed 02, Kerala 04) hands, reaches for and grasps a cube aad transfer it
a) 2d year b) 3'd year from hand to hand. She babbles but cannot wave
c) 5th year d) l5ftyear bye-bye nor can she grasp objects with the finger and
83. Ihe maxirnum age for growth of lymphoid tissue: thumb. Her age is: (U\SC 2K)
a) 4 months b) 7 months
ULPMER es)
a) 3-4 years b) 5-7 years
c) 10 months d) 10 months
c) 7-11 years d) 11-i4years 96. Vocabulary of 1.5 year old child is: (AMU es)
84. Increase ia height in frst year is by: (DNB 2001)
a) 1-10 words b) 10-20 words
a) 4oo/o b) 507o c) 20-30 words d) 30-40 words
c) 600/o d) 75o/o 97. A normal child develops the abilityto use 10 words
85. Birth weight of a cbild doubles at five months of age with meaning at the age oi (Karn 11)

while the birthlength doubles at the age of: a) 12 months b) 15months


c) 18 months d) 24 months
(UPSC 98,07)
a) 1 year b) 2years 98. An infant can sit with leaning forward on his
c) 3 years d) 4 years hands. He bounces actively when made to stand.
86. The weight of the child at 3 years is usually_times He laughs aloud and becomes conoerned when the
thebirthweight (Karn
mother moyes away. What is his most likely age:
11)
a)3 b)4 (UPSC 07)

c)s d)7 a) 12 weeks b) 16 weeks


c) 22 weeks d) 28 weeks
I
'C.rrepit*.

99. A normal infant sits briefly leaning forward on her DISORDER OF GROWTH & DEVELOPMENT
hands, reaches for and grasps a cube and transfer it
from hand to hand. She babbles but canot wave bye- 103. \faking up at night, screamingwit] fear, at2.Syexrr*
bye nor can she grasp objects with the finger and ag9 is generallya manifestation of: (PGt 01,79)
thumb. Her age is: (UPSC 2K) a) Organic illness
a) 4 months b) 7 months b) Normal development pattern
c) 10 months d) 14 months c) castration anxiety
f 00. At which one of the follo-rving age period a child can d) Separation anxiety
remove front opening garment ? (coMED 06) l(}4. Cretinism is: (uP 08)
a) 24 months b) 36 months a) Disproportionatedwarfism
c) 48 months d) 60 months b) Short stature with long trunk
f 0f . Which ef the following are the first incisors to c) Short stature with short trunk
erupt in an infant: (UPSC 07) d) Long stature with long trunk
central
a) Lower b) Lower lateral tO5. Which of the following childhood disorder improves
central
c) Upper d) Upper lateral with increase in age: (MP 2K)

lA2. By-years allmillcteethareerapted: (AMC 2K) a) conduct disorder b) Emotional problems


a) 1.5 b)2 c) Temper tantrum d) Sleep disorder
c) 2.5 d):

III
af,, G rowih', & D eveI o p m e n t

ANSWERS

NORMAL GROWTH

t. Ans. is 'd' i.e., 4 years fRef: A.P. Ghai Bth/e p. 13 dr Vh/e p" 6)
r An infant usually doubles his birth height by the age of 4r/2 year s
) Ans. is 'a' i.e., 25 cm [Ref O.P. Gkai Bth/e p. 6]
o At birth, height is around 50 cm.
r At one year, height is around 75 crlrr.
r Thus gain in height in first year is 25 cm.
3. Ans. is t'
i.e.,4.5 year [Ref O.P. Ghai t"Bth/e p. d]
o A child acquire height of 100 cm by age of 4% year.

4. Ans. is t' i.e., 6 cmlye* fRef: O.P. Ghai \th/e p. 13 6 Vh/e p. 6; Nelson lyth/e p. 54, 551
'After 4 years, the child gains about 5 cm in height eyery year, until the age of 10 years" --
O.p Ghai.
'An average child gains approximately 7-8 cm in height between the age of 2-6 years and 6-7 cm
in height between
6-12 years". __ Nelson
Amongst given option, 6 cmlyear is the best answer.

5. Ans. is'a'i.e., 30 g/d lRef: Gbai Vhle p. 6l


o Average weight of New born baby is 3 kg.
o Newborn loses extracellular fluid about 10% of body weight and start gaining weight and become equal to birth weight
at day l0 oflife.
o Subsequently, they gain weight at a rate of approximately 25 to 30 gm per day for the first 3 month of life.
r Thereafter they gain about 400 gm weight every month, for remaining part of first year.
6. Ans. is 'b' i.e., 1l Months {Ref: O.P. Ghai Bth/e p. 13 b Vh/e p. 6)

3Kg

I year 9 Kg (l x birth weight)


i'Y-eil
3 years 15 kg (5 x birth weight)

7 years 21 kg (7 x birth weight)

7. Ans. is t'i.e., 2 years tRel O. p. Ghai Sthle p. 6]

8. Ans. is 'd i.e., Mid arm circumference lRef Meharbqn Singh 3d/e p. 53)
r During 1-5 years of age, the mid-upper arm circumference (MUAC) remains reasonably static between 15-17 cm
among healthy chidren because fat of early infancy is gradually replaced by muscles.

DEVELOPMENTAL M I LESTON E

9. Ans. is'b'i.e., 6 month [Rel Nelson l}thle ch. S (Table. S.l)]


'A child sits alone without support at 6-8 months"
Clinical pediatrics
'A 7-8 month old baby can sit without support"
Textbook of child care
CHeprsn'r
10. Ans. is'b'i.e., 3 month -1
lRef: Ghai 7h/e p /l
o Neck holding comes at 3 months.

ll. Ans. is'b'i.e., 6 month lRel Ghai Vh/e p 271

3 month Neck holding

12. Ans. is'b' i.e., 5 months [Rel Ghai 7h/e p.7]


o A child rolls over (turn from back to side) by 5 months of age.

13. Ans. is 'a'i.e., Pincer grasp at 3 months lRef: O.P. Ghai 8th/e p. 45,46, 49 6 7h/e p. 27; CPDT 18th/e p. 751
o Pincer grasp develops by 9 months 1 1 months)(9-
r Child can sit with support by 5 months
o Social smile will be present at 3 months (social smile develops by 2 months).
e 2 year old child makes simple sentences and can use pleurals.

t4. Ans. is 'b' i.e., l0 IRel O.P. Ghai 8th/e p. 49 & Vh/e p. 28)
15. Ans. is 'b' i.e., Climb stairs runninglRef: O.P. Ghai 9th/e p. 49, 5A, 51 /2 Vh/e p. 28, 3A; Nelson 18th/e p. 441
lmportant milestones of a 3 years child
o Rides tricycle o Knows his age and sex.
o Stands momentarily on one foot. r Repeat a sentence of6 syllables
o Draws acircle e Has a vocabulary of 250 words.
o Can dress or undress himself completely o Counts 3 objects correctly.
r Builds tower of 10 cubes o Can withhold and postpone bowel movement.

16. Ans. is 'b' i.e., 3 year lRef: O.P. Ghai 7h/e p. j}l
17. Ans. is'd'i.e., 2 years fRef: Nelson 18'h/e table 32-1, table 591-11
e At2years, a child can make simple sentences and uses pronouns.

t8. Ans. is 'd i.e., 2 and3 yeffs lRef: O.P. Ghai 8'h/e p. 50-52 6 Vh/e p. 28; Nelson 18th/e p. 49f
r A child learns to feed herself with spoon without spilling its contents by the age of 15 months.
o A child can dress or undress herselfcompletelyby the age of3 years.
r A child knows his gender by the age of 3 years.
So, the given milestones develop between the age of 15 months and 3 years.
Amongst the given options best answer is 'a' i.e., 2 and 3 years.

19. Ans. is 'a' i.e., 6 month fRef: Clinical pediatrics 2il/e p. 17)
o Purposeful movement through space starts at 6-8 months.
r During this period, children begin to move towards and away from objects or people independently.
20. Ans. is 'b' i.e., 5 years lRef Nelson 18th/e p. 49; O.P. Ghai 6th/e p. 44-461

12-24 months Tries to scribble spontaneously

A:€
3 years Draws a circle

al_gg-e.! rT::=
5 years Draws a triangle
of Growth & Development

21. Ans. is h'i.e., Babble {Ref: Nelson 18th/ep.49)


o A child is able to grasp an object (palmar grasp) at the age of 7 months.
o A child can transfer the object form one hand to another by the age of 5-7 months.
o So, the age of this child is 7 months.
r A 7 months old child can do :-
1) Holds the objects with crude grasp from palm (palmar grasp)
2) Pivots
3) Shows stranger anxiety
4) Resists if a toy is pulled from his hand
5) Babbles
About other options
o A child produces bisyllable sound (mama, dada) and stands with support by 9 months.
o A child sits without support by 8 months.

)', Ans. is t'


i.e., 30 months lRef. Nelson 18'h/e p. 49 & O.P. Ghai 8'h/e p. 50-52 6 7h/e p. 28, 301
o A child climbs stairs with alternating feet, refers to self as 'I' and builds a tower of 9 cubes by the age of 30 months.
o A child knows his gender and age by 3 years.

11 Aae . is'H i.e., l'k years {Ref: O. n Ghai */e p. 30; Nelson 18e/e p. 49)

I 12 months

18 months

. 36 months 10

24. Ano. io '{ i.e., Draws a triangle t&ef: O.p. Ghai */e p. 50, 51 6 fr/e p, 28; Nelson l}*/e p. 491
o A child can draw a triangle by 5 years.
o A child can draw a circle by 3 years.
o A child can make a tower of 9 cubes by 30 months.
o A child can go up and downstairs with one step at a time by 2 years.
r A child can stand momentarily on one foot by 3 years
ob'
L). Ans. is i.e., 3 year {Ref. Nelson 1*/e p. 50)
o Handedness develops by the age of 3 years (36 months).

26. Ans. is 'a' i.e., Mirror play fRef: O.P. Ghai 8th/e p. 49-51; Nelson 18th/e p. 491
o A child enjoys watching his own image in the mirror (mirror play) by 6 months.
o A child crawls in the bed by 8 months.
o Full creeping and crawling develop by 10 months.
o Pincer grasps develops by 9 months.
27. Ans. is 'b' i.e., 5 months {Ref: O.P. Ghai 8th/e p. 52 6 Vh/e p. 30; Nelson 18th/e p. 491
I
o Mouthing develops by 4 months, while stranger anxiety develops by 7 months.
o So, the age of this child is between 4 to 7 months.
o Amongst the given options best answer is 5 months.

28. Ans. is t' i.e., Build a tower of 3-4cubes [Rel O.P. Ghai 8th/e p. 49 dt Vh/e p. 28; Nelson 18'h/e p. 45, 49; CPDT 18th/e
p. 7s1
o A child starts trying to build a tower of cubes (2 cubes) by the age of 1 year.
About other options
o Child can stand with support by 9 months.
e Chiid can play peak-a-boo game by 10 months.
CrraPft.ri.:r
o Child can pick up a pellet with thumb and index finger (pincer grasp) by 9 months.

29. Ans. is 'l i.e., Head control fRef: Nekon 18h/ep.49l


r Head control develops by 3 months.
o In ventral suspension a child can lift his head in the horizontal plane by 2 months (8 weeks)
o A child follows object with steady movementby 2 months (8 weeks).
r Social smile develops by 2 months ( 8 weeks).
o A child turns head to sound by 1 month.

30. Ans. is'd'i.e., Sustain head level with the body when placed in ventral suipension lRef: Nekon 18e/e p.
4ej
Important rnile stones of a 2 months child.
o In ventral suspension, lifts his head in the horizontal plane (in the plane ofthe body).
e Social smile develops
o Follow object with steady movement of eye.
About other options
r Parachute reflex develops during 9th month of life --- CPDT 18'h/e p. 709
e Control of head develops by 3 months
c A child can Iift head and chest off a flat surface with extended elborv by 6 months.

31. Ans. is 'c' i.e., 4 years lR$: Nelson lSthle Ch*p. 8-9\
r word sentences = 19 month
2
o 6 word sentences = 48 month
o 10 word sentences = 60 month

32. Ans. is'lf


i.e., Focus on bright obiect [Ref Nelsoa l&h/ep. 49)
o Neonatal age is up to 1 month.
oA month old child regards a lighted torch at a distance of 20 cm.
1
o The infant fixes gaze by 8 weeks -- Meharban Singh 3'd/e p. 67
e Infant lifts his head and greater part of his chest on extended arms by 6 months.
r Rolls from back to side by 5 months.
-r-r. Ans. is 'a' i.e. Identify colours j0; Nelson l*/e p. 491
fRef. a.P. Ghai 8th/e p. 5a, 54 & Vh/e p.

o IQ is defined by as the mental age divided by the chronological age multiplied by f 00.
M0ntal Age '.,
I.Q.=---------x100
Chronological age 1

r child 6 years of age with an IQ of 50%, means that his mental age is that of
So a a3 year old child.
e Thus we have to look for the developmental milestones of a 3 year old child.
t At 3 years of age, a child can identifu two colours.
: A 3 year old child can speak a small sentence but he cannot read a sentence.
t At 3 years, a child can ride a tricycle (not a bicycle).
t A child copies a triangle at 5 years of age.
34. Ans. is'H i.e., 15 months lRef. Nelson l8thle p. 48,49)
o Vocabulary of 4-6 words in adilition to jargon is achieved by 15 months of age. However <20o/o of speech is
understood and hence main mode of communication continnues to be non-verbal,

35. Ans. is'H i.e.,9 month [Ref Nelson 18'h/e eh.81

"Object permanence, a major milestone, develops around 9 months when the infant understands that objects continue to
exist even ifthey are not seen". Smith's Anaesthesia for infants and children
--
'A major milestone is the achiyement by 9 months of object permanency (constancy), the understanding that object continue
to exist, even when not seen". Nelson
--
Note : Child fully understands object performance at 18-24 months.
htiia,b- ilaiaer of Growth & Devetopment

ERT"'PTION OF TEETH

36. Ans. is t' i.e ., 6 months lRef: Netson l9tt /e p" 4Z-Z3l
o Rule of six :-
r Primary (temporary) dentition begins --> 6 months.
r Secondary (permanent) dentition begins -+ 6 years.

2.-l Ans. is 'b' i,e., 6 years [Ref: Nelson lSth/e Chapter S)


o First primary (milk) tooth erupts at 6 months.
e First secondary (permanent) tooth appear at 6 year of age.
38" Ans. is'a'i"e., Molor
o 1't primary (temporary) teeth to erupt is mandibular (lower) central incisor.
o l't seconilary (permanent) teeth to erupt is 1.t molor.

39. Ans. is'a'i.e., 2O [Rel Nelsom tfl,/c Chap. 504)


c Primary teeth/temporary teeth/milk teeth -+ 20
o Secondary teeth/Permanent teeth -+ 32
40. Ans. is ?' i.e., All of above lRel CPDT tgth/e p. 452; Nelson tBtt/e p.42,73)
Causes of Delayed eruption
r Idiopathic (most common) r Hypopituitarism -- CPDT r Cleidocranial dysplasia
r Hlpothyroidism r Trisomy2l (Downsyndrome) --CPDT
r Hypoparathyroidism r Rickets

MISCELLANEOUS

41. Ans. is 'a'i.e., I year [Rel Nelson Chapter 52)

r By the age of 1 year, ratio of ICF to the ECF volume approaches adult level.

42. Ans" is'b'i.e., 1.3 tRdl Chhedda jd/e p. 22)

At birth 1.7 :1.A

1.3 _ 1.0

1.0 : 1.0

43. Ans. is'H i.e., pBl,Iw[Rel NCxRr sanskrit-Textbook r class lt; Nelson 18il'/e chap. 322]
t Bilabial words are pronounced or articulated with both lips, as the consonants b, p, m, and w.
: Baby starts producing bilabial words by 4th month of age.

DISORDER OF GROWTH & DEVELOPMENT

SHORT STATURE

44. Ans. is'a' i.e., Constitutional delay in growth lRe! A.p. Ghai Bth/e p. j7 b 7,/e p. 19)
r The child in this question has :

1. Height below 3rd percentile, i.e., short stature.


2. Normal growth velocity, i.e., normal variant of short stature.
3. Skeletal age is less than chronological age.

r There are two types of normal variant of short stature:


l. Constituional delay in growth
2. Familial genetic short stature
CItapTan,'r

c In constitutional delay the bone age is less than chronological age, while in familial short stature bone age is normal
for the chronological age.
o So, the most likely diagnois of this child is constitutional delay in growth.

Short stature
I

YV
Normal growth velocity Altered growth velocity
+ +
Normal variant short stature Short stature other than normal variant
'1,

+ v
Bone age < chronological age Bone = chropological age
v v
Constitutional delay Familial short stature

45. Ans. is t'i.e., Constitutional delay fRef: See above explanation)


In constitutional delay the bone age is less than chronological age, while in familial short stature bone age is normal
lor the chronological age.
So, the most likely diagnois of this child is constitutional delay in growth.

Short stature
I

Y-_}
Normal growth velocity Altered g rowth velocity
+ +
Normal variant short stature Short stature other than normal variant
'J,

v v
Bone age < chrlonological age
+ +"
Bone = ch ronolooical aoe

Constitutional delay Familial short stature

-15. Ans. is *e'i.e", Constitutional delay lRef: See above explawation)


+6. Ans. is'd i.e., Constitutional [Rel: O.P. Ghai yth/e p. 35-36 6 Vh/e p. 19-20; CPDT 18th/e p. 938]
o Constitution delay in growth is significant cause of short stature in mid childhood period but the ultimate height is
normal, \
+7. Ans. is 'tr' i.e., IGF-I levels are low for ehronologieal age lttef: Nelson {?tt'le Chopter 14, 7, Sl8;Pedistrics for
medicsl students dqniel bernstein PG 3$3, bedside clinias in paediatrics by Sibarjwn Ghaskl" p. 357)
o In constitutional growth delay, IGF-I levels tenil to be low for chronological age but within the normal range for
bone age.
r Birth weight and height are normal.
r Bone age is less than chronological age.

18. Ans. is'a'1.e",lu{orma} body proportion [ftef A.F. Ghai 8't'le p. 35-36 6 7h/e p. 474-475]

Short stature due to Human Growth hormone deficiency in characterized by -


l. Ratio of upper to lower segment is normal.
2.Bone age or epiphyseal development in less than chronological age by about 2 years.
3. Children! are normal in height and weight at birth.
4. Delay in growth is usually observed afier the age of one year.

PSYCHOLOGICAL PROBLEMS

49. Ans. is t' i.e., Specific learning disability [Ref: O.P. Ghai 8th/e p. 59-60 6 Vh/e p. j9; Nelson 1"6th/e p. 150, 151

r This child has -


i) Difficulty in expressing himself in written (problem in written expression)
ii) Makes spelling mistakes (problem in spelling).
o Both these are manifestations of dyslexia, a specific learning disability.

50. Ans" is 'd' i.e., All of the ahove fR"ef: O.P. Ghai Sthle p' 38'4A; Nelsan ISoh/e p. 148)

o ADHD is characterized by an age inappropriale hyperactivitia. impulsiveness and inattenton.


o Hyperactivity is usually the first symptom ta be noticed,

51. Ans. is 'H i.e., Attention deficit hyperactive child iRe/ 0.F. Ghai Sthle p. 59 6 Vh/e p. 381

r The child in question has following problems.


1.Child is hlperactive 3. Difficulty in playing
2. He does not listen to teachings 4. Disturbs other students

r All these are features of attention deficit hlperactivity disorder (ADHD).

52. Ans. is 'b' i.e., Behaviour therapy lRef: A.P. Gkai 9'h/e p. 59 dt(h/e p. 38; Nelson 18th/e p. 1481

o This child has -+ i) Restlessness ii) Inattentivenessiii) Uncontrolled desire to play outside
o Al1 are features of attention deficit hyperkinetic disorder.
1)Psychosocial treatment

' ff:l:'*ill1Tl'.[1f be educated with regard to the ways in


which ADHD can affect tearning, behaviour,

r Goals should be set for the family to improve the


childt interpersonal relationships, develop study
decrease disruptive behaviors. skills, and

2. Behavior therapy
r The goal of behavior therapy is to identift targeted behaviors
that cause impairment in childs life so that child
can work on progressively improving his or her
skill in these area.
3. Medications
r The drugs used for ADHD are -+ i) Methylphenidate (Doc) ii) Amphetamines iii) Atomoxetine
53. Ane ts xi i-e.,,stants before 2{ years of age{Ref GteaiStulc p-,az r{tgt? yieild py&iatry zdlo p. zzl
Autism
r Autism is aneurologic disroder characterized by _
1. Qualitative impairment in social interaction
2. Qualitative impairment in communication.
3' Restricted repetitive and streotyped patterns
ofbehaviour, interests, and activities.
o Onset of syfiI,toms is usually before 3 years of age.
o 3-5 times more common in boys, but more
severe when occurs in girls.
o More common among low socio-economic
groups.
54. An* is h' i.e.' Autism tRef. o.p. Ghai Be/e p. 61 & fl/e p. 40; High yielit
psychiatry p/e p. 22)
r Delayed speech' difficulty in communication and concentration
in a 3 year old child suggests the diagnosis of autism.
55- Am" is W i-e- Vision pr*a&lems LR€f:$_e Ghai #/e p- dt; CFU{ t#/a p_ 95, tti6 r

' by Impaired social intera ction, impaired communication,


and.restricted repetitive & stereotyped
frffi ffffi]f,t:fed
r Restricted repetitive stereotlpes pattern of behavior
is due to impaired imagination.
w- Ams" is b' i-a-, Ormet aftar 6 y*ars at rye tW O-p, Ghai #/* p-
ffi e p{e p. A{fi
57. Ans. is'b'i.e., Rett's syndrome [Ref Nelson t*/e p.
2504, 25051
r This 2 years child has following problems:_
1. Hand wringing movements.
4. Poor social skills.
2. Impaired language and communication
development. 5. Decleration of hand growth after 6 months.
3. Breath holding spells.
r All are the clinical features
of Rett syndrome.
58" Aeaa" ns?ile,Scilortivermre{iurm {eaf: M*ax n#/c p 24" SZ 31, m!
t selective mutism is defined as a
failure to speyk in specific social s_ituations, despite speaking in other situations;
typically a symptom of an undeilying anxiety d*order. it
ciildren with selective *itir* can speak normally in
certain
with their parents, but do not ,p,ik in other sociat
::::rf:;:::r:;T:"y,""',1:;'iX::;:l:Tr:::irr;:^ne settings,

ENURESIS

59-
o No treatment is given to. children below
6 years of age because ofhigh spontaneous
o After 6 years treatment include. cure rate.
i) Behaviorat therapy: This is the treatment of choice.
ii) Pharmacological treatmenf : It is used when non-pharmacological (behavioral)
therapy) fails . Desmopressin is
the drug of choice. other drugs used are impramii" '
o*yiiiinir.
""a
60. Ans. is h' i.e., Bed alarms {Ref t*/e p.
Nelson 113, I lal
o First line treatment for enuresis is behavioral
therapy. It consisls of rewarding the child for
being dry at night, child
oefore retiring and the use of conditionlrg d.',ric.s
f".g. u.a ahrm-that ring. *rr." rrr. child wets a special
:*JJr1*to
o Consistent dry bed training with positive reinforcement has a success rate of 85% and bed and pad alarm systems have
a success rate of approximately 75o/o with relapse rate that are lower than those with pharmacotherapy.
61. Ans. is'a'i.e., Positive reinforcement fRef: Ghai Vhle p. 35]
o Consistent dry bed training with positive reinforcement has a success rate of 85% and bed and pad alarm systems have
a success rate of approximately 75o/o with relapse rate that are lower than those with pharmacotherapy.

62. Ans. is 'b' i.e., Desmopressin [RS A.P. Ghqi 8e/e p. 6 Thle p. 36; Nelson l9*le p. 113, 114]
505

63. Ans. is t' i.e., Imipramine lRef: A.P Ghai Sthle p. 5a5 d* flle p. j6j
r Desmopressin is best answer.
o But it is not given in options.
r Amongst the given options only imipramine is used in enuresis.

BREATH HOLDING SPELLS

64. Ans. is 'a'i.e., Antiepileptic treatment is necessary lRef. Nelson l8.*/e p. 2476, 2477; CPDT 18th/e p" 911
o Antiepileptics are never required (please refer to text of the chapter).
e Other options are correct.
65. Ans. is t' i.e., tron [Ref Pediatrics by Lucy M. Osbom p. 758; Netsan l9hle Chapter 594f
"A subgroup of infants with breath holding spells have iron deficiency anemia. Iron therapy may treat not only the
anemia, but also th6 breath-holding spells. Pallid infantile syncope may respond to atropine sulfate, which is used on
an ongoing basis if spells are frequent, or intermittently if spells are situationally predictable (such as with venepuncture)'i

THUMB SUCKING

66. Ans.is1d'i.e.,Mustbetreatedvigorouslyinthefirstyear lRef:O.P"GhaiYe/ep.SSbZ"rcp.38;NekonlP*le


p. 1lsl
Intervention to reduce thumb sucking should not be considered until at least 4-5 yr and then only if it is causing
impairment for the child. Nelson
67. Ans. is'a'i.e., 8 years lRef : O.P. Ghai 9ft/e p. 59 6 7/e p" 38; Nekon l*le p. 15321
o The earlier the habit of digit sucking is discontinued after the eruption of the permanent maxillary incisors (7 -8 yr),
the greater the likehood that there will be lessening effect on the dentition.

MISCELLANEOUS

68. Ans. is'b'i.e., ll tRef: Textbaakaf anthraparnetry\

Arm span r
o It is the distance between the tips of middle fingers of both arms outstretched at right angles to the body, measured
across the back ofthe child.
r In under-5 children , arm span is 1 to 2 cm smaller than body length.
r During 10-12 years ofage , arm span = height.
r In adults arm span is more in adults by 2 cm.
r Abnormally large arm span is seen in patients with : (1) Arachnodactyly (Marfan syndrome) (2) Eunuchoidism
(3) Klinefelter's Syndrome (4) Coarctation of aorta.
r Armspanisshortcomparedtoheightinpatientswith:(1)Shortlimbeddwarfism(2) Cretinism(3)Achondroplasia
69. Ans. is t'i.e., Foreign object being put in the mouth lRef: Nelson lThle Chap. 22.2; AP Ghai Sele p. 58\
o Pica involves repealed or chronic ingestion of non-nutritive substances, which includes plaster, charwal, clay, wool,
ashes, patent & earth. :
o More common associated with autism :
r Family disorganisation
r Poor supervision
r Psychologic Neglect
o Geophagia (eating of earth) is associated with pregnancy and some culture.
ADOLESCENT
70. Ans. is 'a'i.e., 10-19 years lRef : O.P. Ghai 8th/e p. 63 6 Vh/e p. 42]
o Adolescence is usually the period l0 to 20 yr.
71. Ans. is'H i.e., l0 years lRef: See above explanation)

72. Ans. is'd i.e., Appearance of pubic hair and Axillary hair lRef: Nelson l},h/e p. 61]
e Thelarche -+ Pubarche -+ Peak growth velocity -+ Menarche
r Thelarche (development of breast buds) is first visible sign of puberty.
o Pubarche / Adrenarche (development of axillary and pubic hair) is the second sign. Itoccurs about 6-12 months
after thelarche
73. Ans. is'b' i.e., Thelarchy fRef: O.P. Ghai 8,h/e p. 6j dt 7/e p. ag|l
r In girls, the first visible sign of puberty is the appearance of breast buds (Thelarche), between 8-12 years of
age.
o In boys the first visible sign of puberty is testicular enlargement, beginning as early as 9lz yr.

74. Ans. is 'a' i.e., Breast enlargement lRef: Has been explainedl

75. Ans. is 'd i.e., Thelarchy-Pubarchy-Menarchy fRef: O.P. Ghai 8th/e p. 6j 6 7h/e p. 498; Nelson lSth/e p. 611

76. Ans. is'd i.e., Testicular development - pubic hair - axillaryhair - beard fRef: O.P. Ghai 8th/e p. 63 b Vh/e p.
498; Nelson 18th/e p. 51, 621
o In boys, the first visible sign of puberty and the hallmark of SMR 2 is testicular enlargement.
o The sequence of development of secondary sexual characteristic in boys is -
Testicular-enlargement -> Penis enlargement -+ Pubic hair growth -+ Peak growth velocity -+ Axillary hair -+ facial
hair.

77. Ans. is 'd' i.e., |ust before commincement of menarche [Rel Nelson lgth/e p. 60-621
Mensus typically appears around the peak in height velocity -Nelson
Menarche usually occurs afier peak velocity has been attained and as the growthrate begins to decline.
78. Ans. is t'i.e., Hormone related breast enlargement in girls
Thelarche
o Definition :- Begining of secondary (Post natal) breast development at onset of puberty in girls.
r Tanner stage2 breast development
r Usually after 8 years of age
r Because ofrising level ofestradiol
o Breast development during puberty in male termed as gynaecomastia not thelarche.
79. Ans. is 'b' i.e., Tanner stage II fRef: Ghai 7h/e p. 4971

r Thelarche- first sign of puberty in Girl around age of 10 year in Girl


r Definition :- Begining of secondary (Post natal) breast development at onset of puberty in girls.
r Tanner stage2 breast development.
r Because ofrising level ofestradiol
80. Ans. is'a' i.e., Appearance of pubic hair & t' i.e., Enlargement of scrotum lRef: Nelson lSth/e ch. 12)
o Scanty & long pubic hair appear at SMR-2. Enlargement of scrotum is there at SMR-2.
GROWTH & DEVELOPMENT

81. Ans. is'd i.e., In the first year of life lRef: O.P, Ghai 8th/e p. 9, 10 6 7h/e p. 7)
o ln the early postnatal period velocity of growth is high duringthefirstfew months,
82. Ans. is'd i.e., Zndyear fRef: O,P, Ghai 8th/e p. 10 lt 7h/e p. 4l
o The brain enlarges rapidly during the latter months of fetal life and early months of postnatal life.
o At birth, the head size is about 65 to 70 percent of the expected head size in adults.
o It reaches 90 percent ofthe adult head size bythe age of2 years.

83. Ans. is'b'i.e., 5-7 years fRef: O.P. Ghai 8th/e p, 10-11 b Vh/e p. 4)
r The growth of lymphoid tissue is most notable during mid-childhood.
o Children between 4 and 8 years of age often have hlpertrophied tonsils and large lymph nodes, which is infact a sign
of lymphoid hyperplasia.

84. Ans. is'b'i.e., 50Yo lRef: O.P. Ghai |th/e p. 13 b Vh/e p. 6l


o At birth, the average height of a child is 50 cm.
r During first year height increases about 50% of birth height (25 cm).
85. Ans. is'd' i.e., 4yet lRef: O,P, Ghai 9th/e p. 13 6 Vh/e p. 6)
Cg,Aprsn,,rt

ANSWERS OF VARIOUS OTHER EXAMINATIONS

GROWTH & DEVELOPMENT

81. Ans. is'd i.e., In the first year of life lRef: O.P, Ghai 8th/e p. 9, 10 & Vh/e p. fl
o In the early postnatal period velocity of growth is high duringthefirstfew months,
82. Ans. is '{ i.e.,Zndyear- fRef: O.P. Ghai 8th/e p. 10 dr 7tu/e p. a)
o The brain enlarges rapidly during the latter months of fetal life and early months of postnatal life.
o At birth, the head size is about 65 to 70 percent of the expected head size in adults.
o It reaches 90 percent of the adult head size by the age of 2 years.

83. Ans. is'b'i.e., 5-7 years fRef: O,P, Ghai 9th/e p. 10-11 dt 7h/e p. 4l
r The growth of lymphoid tissue is most notable during mid-childhood.
o Children between 4 and 8 years of age often have hypertrophied tonsils and large lymph nodes, which is infact a sign
of lymphoid hlperplasia.

84. Ans. is'b'i.e., 50Yo lRef: O.P. Ghai 8th/e p. lj 6 Vhle p. 6)


r At birth, the average height of a child is 50 cm.
e During flrst year height increases about 50% of birth height (25 cm).

85. Ans. is ? i.e., 4year lRef: O.P. Ghai $th/e p. 13 & Vh/e p. 6)

86. Ans. is t' i.e., 5 lRef: Child ilevelopment: Birth to adolesence, Dr Rajesh Dixit, 2006 ed. p. 38; Health, Safety
andNutritionfor the young child, Lynn R Marotz, p. 261
87. Ans. is 'b"i.e., 6 months of age lRef: O.P. Ghai $th/e p. 13 b 7h/e p. 6; Meharban Singfi 3d/e p. 521
r At 6 month of age head circumference is between 40.0-43.5cm.

88. Ans. is t' i.e., 9-12 months [Rel O.P. Ghai 8th/e p. 1j 6 7h/e p. 6; Meharban Singh 3d/e p. 52]
o The circumference of chest is about 3 cm less than head circumference at birth.
o The circumference of head and chest are almost same by the age of 9 months to I year.
r Thereafter the chest circumference exceeds the head circumference.

89. Ans. is'c'i.e., SKglRef: O.P. Ghai 9th/e p. lj 6 Vh/e p. 6l


90. Ans. is'H i.e., 4 years lRef O.P. Ghai 8th/e p. 13 & Vh/e p. 6l

91. Ans. is t'


i.e., Measurement of midarm circumferance [Ref O,P, Ghai \th/e p. 97 dt Vh/e p. 64;Nutrition &
Child d.evelapment K.E, Elizabeth IIh/e p,tual
92, Ans. is'a' i.e., 52 weeks lRef: O,P. Ghai 8th/e p. 49-50 (! Vh/e p. 301
r A child plays a simple ball game at I year (52 weeks).

93, Ans. is 'H i.e., 4years lRef: Nelson 18th/e p. 49; O,P, Ghai 9th/e p, 49)
r A child hops on one foot by 4 years and skips by 5 years.
o As this child can not hop, the age of this child is less than 4 years.

94, Ans. is 'b' i.e., 6 months lRef: O,P. Ghai 9th/e p, 49 dr Vh/e p, 27; Nelson 18th/e p, 46, 47)
o A child can transfer objects from one hand to the other by 5-7 months.

95. Ans. is'b' i.e., 7 months IRe{Nelson 18th/e p. 49; O.P. Ghai lth/e p. 49-501
96. Ans. is'b' i.e., 10-20 words lRef: O.P. Ghai 8th/e p. 53 6 Vh/e p. 301

97. Ans. is t' i.e., 18 months fRef: O.P. Ghai 8th/e p, 53 & Vh/e p. 28)
98. Ans. is'd' i.e., 28 weeks fRef: Nelson 18th/e p, 45; O,P, Ghai 8th/e p. 521
o A child bounces actively, leans forward on hands in sitting position and prefers mother by the age of 7 months (28
weeks).
:t:tr:.:,:,.:.:t.

99. Ans. is'-o" i.e., 7 months fRef: Nelson 18n/e p. 45; O.P. Ghai Sn/e p. SA, 527
r A normal infant sits leaning forward on his hands and babbles by 7 months.
o An infant transfer the object from one hand to other by 5-7 months.

100. Ans. is'H i.e.,36 months fRef: Nelson 18*/e p.491


o A child can dress or undress himself completely by 3 years.

101. Ans. is'l i.e., Lower central {Ref: Nelson 18e/e p. 47, 731
,O2. Ans. is "c' i.e.,2.5 {Ref: Nelson 18n/e p. 47, 737
r Between 20-30 months all milk (primary) teeth are erupted.

DISORDER OF GROWTH & DEVELOPMENT

103. Ans. is't i"e., Separation anxiety {Ref : Nelson 1*/e p. 1171
o Separation anxiety disorder is characterized by unrealistic and persistent worries of possible harm befalling the
affected child or his or her primary caregivers, reluctance to go to school or sleep without being hear the parents
persistent avoidance of being alone, nightmares involving themes of separation numerous somatic symptoms and
complaints of subjective distress.

104. Ans. is't i.e., Disproportionate dwarfisrn [Rel o.P. Ghai Bth/e p. 519 & Th/e p. 1g]
Note - Patient has short stature with short limbs.

105. Ans" is'C i.e., Temper tantrum lRef : O.P. Ghai 9e/e p. 5S & 7/e p. 371
o Temper tantrum reaches its peak point durin$ second and third year of life and gradually subsides in between 3 to 6
years as the child learns to control his negativism and complies to the requests ofothers.

rrr
b{&JY'KXYXON

N UTRITIONAL STATUS I N DICATORS

o Surveillance of growth and development is an important component of the routine anticipatory care of children'
o The main purpose of growth surrveillance is to identifu those children who are not growing normally.

o Nutritional status indicators for surveillance of physical growth are l-


1) Weight for age
s MeasurementofweightandrateofgaininweightarethebestsingleparametesforassessingphysicalgrowthurlMs
10,06)
.

r 8070 of the median weight for age of the reference is cut-off point below which children should b,e<6isideted
malnourished. I
/
r Low weight for age (underweight) is a combined indicator to reflect both acute and chronic mhlnutrition(ArMs
07).
B:ut it cannot diferentiate acute from chronic malnutrition. \
2) Height for age
r Height for age is a stable measuremeni of growth as opposed to body weight.
r Low height for age (stunting or dwarfism) reflects chronic (past) malnutrltionat,zArrMs,7).
r The cut-offpoint commonly taken for the diagnosis of stunting is 90% of the united states NCHS height for age.

3) Weight for height


t Weight in relation to higbt_is now considered more imltortant than weight alor'te(AllMst0'06).
s Low weight for height (wasting) indicates acute malnutritionQct 0s' ArtMS 0s) .

4) Head and chest circumference


r Chest circumference t
At birth -+ Less than 2 cm from head circumference
6-9 months -+ Two measurements become equal
> 6-9 months -+ Overtakes head circumference
I In severely malnourished children this overtaking may be delayed by 3 to 4 years.
Weight heig ht SD score - 2 to -3
(wasting) (70 - 79o/o of expected) (<7070 of expected)
.
r:.]r,,:,i
,:, .; ,5,Q,sctite -2 tq.-3- ''': aD:;i{;e.<,.ti,'',::r,
:,r..li..ijng)t.'',.:,r,, ::.,.,,:r.::.. .,
{85 .89%,oflexpectqd) (< 85olo of expected)

Reference (standard) values of growth


o For national and international comparisons and for monitoring, "reference
or standard values" of growth are essential.
r The well known reference standards are -
1) Harvard (or Boston) standards
r These are not in common use and have been replaced by WHO refrence values.
2) WHO reference values (NCHS standards)
r These are most commonly used and best availableallMss, reference values
for international use.
r These values are based on the data assembled by United States National
Centre for Health Statistics (NCHS).
t Classification of pEM is based on these standards.
3) Indian standards (ICMR)
t These are used for Indian children and based on data assembled by rndian Council of Medical Research
(lCMPStu"'oo1'

* The normal expected value is 50th percentile of the reference value.


o When weight for age is <807o of expected (expected 50th percentile
= of refrence) -+ wasting.
e When height for age is <9oo/o of expected (expected = 5oth percentii" oi ,"rr"n.")
-+ stuntin!.

CLASSIFICATION OF MALNUTRITION

lndian acaderny of pediatrics (!Ap) classification


o Indian academy of pediatrics (IAP) classification of malnutrition is
based on weight for age. IAp designates a weight of
more than 80o/o of expected for age as normal. Grades of malnutrition are :-
t Grade 1(Mild) -) 71-80o/o
t Grade 1I (Moderate) -+ 61-700/o
t Gradelll (Severe) ) 57-6lo/o(PGI ee)

s Grade IV Sery Severe) -+ ! 50o/o

Gomez classification
o It is based on weight retardation (not on height retardation)Gr ost,
o The child on the basis of his/her weight is compared with a'normal'child
of the same age.
o The'normal'reference child is the 50th centile of the Boston slsnflqyflsLroa).

Weight of child
Weight for age (o/o) :
x 100
Weight of normol child of same

9O-1100/o Normal Nutritional status


75-89o/o Mild Malnutrition (1 st degreeyAt oo)

6O-74o/o Moderate malnutrition (2nd degree)


Under 60%o Severe malnutrition (3rd degree)

r classification is easy to compute as weight is widely recorded parameter.


r classification has prognostic value for hospitalized patients (This is because
the Cut off values were set during
a study of risk of death based on weight for age at admission to a hospital
unit).
Waterlow's classifi cation
r It is based on stunting (height for age) and wasting (weight for height)roNr rsr

>m-2SD Normal Wasted


< m - 2 SD Stunted Wasted and stunted

m = mean, SD = standard deviation

Interpretation of indicators

'i/eight of the child


Weight/Height (%) = x 100
Weight of a normal child at same height

Height of the child


Height/Age (%) = x 100
Height of a normal child at same age

Itola] >95
" tfrilily:iniiraitiid:'il:".-:.:
Moderately impaired 80 - 87.5
<80

PROTEIN ENERGY MALNUTRITION (PEM)

o Protein energy malnutrition (PEM) is a spectrum of conditions ranging from mild undernutrition ot extreme forms of
malnutrition, i.e. Marasmus and Kwashiorkor.
r Most of the child suffer from mild to moderate nutritional deficit and extreme forms (marasmus and kwashiorkor)
account only for a small proportion of cases of malnutrition.

Mild to Moderate malnutrition


r If the dietry intake is deficient for a short period, the body adapts its metabolism to compensate for the deficit to some
extent. Moderately malnourished children appear more slow and less energetic.
o Growth lag is more pronounced in weight than the length. Head circumference is not reduced significantly.

Marasmus
rs).
o It is due to prolong deficiency of calories and proteins(Ar Thus there is exessive catabolism of adipose tissue and

muscle proteinarrs).
o It is characterized by gross wasting of muscle and subcutaneous tissues resulting in emaciation and marked
stunting:NEBr). There is no edema. Body weight is less then 6070 of expected.
o Fat in adipose tissues is severely depleted. However the buccal pad of fat is preserved till the malnutrition becomes
extreme because a higher proportion of saturated fatty acids is stored there and the saturated fat is the last to be
depleted.
o Skin is dry, scaly and inelastic with wrinkles. The hair is hlpopigmented. Abdomen is distended due to wasting and
hypotonia of abdominal wall muscles. i
o The child is alert but irritable.
o Child may show voracious appetite(NEEl /'pmer 11)
. .

o Marasmus represents the compensated phase of PEM.


Kwashiorkar
r Kwashiorkar represents the uncompensated phase of pEM.
r Itischaracterizedbyclassical'triad'ofedema(NEEr'AIee) (Duetohypoalbuminetniaar..,riptue,rr)),markedlyretarded
gr owth( AI 8e ), and psychomotor (mental) changs5ovrrr).

r Associated features are :-


1) Hepatomegaly(NBEr,AnMs0s'DPG0e) (due to fattyinfiltratior{Aree,tipne,1l)).
2) skin changes : Er1'thema, hlperpigmentation, fl aky paint dermatosis(,,r Nnnr). 13,

3) Hair changes: Thin, dry and brittle hair; hypopigmented, flag sign{rcrre) (alternate bands of hypo-and hyper-
Pigmentaion{errr);.
4) Infections: Diarrhoea, respiratory and skin infections.
5) Impaired (decreased) appetiteal 13).

Itffif,hi(,r*or

Sar€frl1us

Pmmiflent
bore*
&6ffsasa
lt $ub.ule,r6o&6
fat

Fo$t w{ufld

Complications of pEM
A) Changes in body composition: Increased total body water,
Reduced BMR, decreased insulin levels(Alrlfs 0s, DpG 0e),
increased cortisol and growth hormone, decreased activity
of sodium pump, reduced Bp and stroke volume/cardiac
output, decreased renal blood flow and GFR, poikilothermia,
and reduced IgA levsls{aroa.).
B) Acute complications (Mnemonic-sHIELDED) : sugar deficiency (hypoglycem
iao,cr 06,0s)); Hypothe rmiaecr 06, os).
Infection & septic shock; Electrolyte imbalance (hypokale miao,G'06,05),hlpomagnes
, iurror ou, orl; DEhydration;
and Deficiency of iron, vitamins and other micronutrients. There
may be megaloblastic anemia, which responds
well to folic acid and vitamin B,, supplemelf4fl61arer).

r Poor prognostic factors in pEM, which can lead to early


death _
1. HYPothermidxqn'ea) 4. Diarrhea 7. Cardigc falluretDPc 0, rN 8s)
2. Hypoglycemia 5, Severe anemia 8. Dehydration or overhydration
3. Systemic infections 6. Liver failure 9. Fluid & electrolyte imbalantetoPcos,rNse)

PEM and infection


r The resistance of the human organism to infections is adversely
affected in malnutrition.
r Following changes are seen :
1) The skin and mucosa do not offer effective physicql barriers against infection@pscoe).
2) Impaired chemotaxis, defective candidacidol and bactericidol capacitytuos, ort.
3) cell mediated immunity (ilelayed hypersensitivity) is impairsfl@tscoe;.
4) Circulating antibodies are usually normal or elevated. \
c The number and proportion of B lymphocytes are not altered@psc 0e)
.
o Humorql antibody response to a variety of immunizing
agents (TT 6 DT) is adequate.
I But secretory IgA is generally reduced(Are1).
Evaluation of Malnutrition
o Mid-Arm circumference (MAC): Normal value in children (1-5 years) is 16-17 cm. Value 12.5-13.5 cm represents
borderline malnutrition, whereas value < 12.5 cm represents severe malnutrition.
o Bangletest:Abangle of 4cmdiameterpassesaboveelbowinacaseof malnutrition.
o Skinfold thickness: Skin fold thickness over tricepts is evaluated. Normal value is 10 mm and value less than 6 mm
represents malnutrition.
c Various ratios: Kanawati index (MAC/head circumference6"')), Dugdale's index, Quaker arm circumference, leliffe's
ratio.

Treatment
r Treatment is done in three stages :-
A) First stage: Dehydration and infections are dealt dtringfirst 24-4Shoursby rehydration and antibiotic therapies.
B) Second stage : Dnring next 7- 10 days patient is given a diet of 75 callkglday along with antibiotics, to maintain
protein and energy need.
C) Third stage : This is aimed at restoring patient's protein and fat contents by providing diet of 175-2OO kcalikg/
day4nrvts
ss' 'u se)
in severe malnutrition and 1 50 kcal/kg/day in moderate malnutrition.
r Iron should be started when child is gaining weight once stabilization phase is over("").

'1ha.p,atii*i l*.figure ii'6ir&riag 661p:1 ' : ' , ', ,

a) Scuny
b) Rickets
c) Kwashiorkor
:.1. ,.
'

:Au*iri*&1leii.&ranhiorkor. . ., ::

':a'':Thiaiifi ag'sigron.heirr-eharaeterislicof kwashio:ko:.

The fi agtostu,'o{ the given chlldril i:

' d nickets,' ,t,',':. ,

. b) 'MarasrRus ',:
' 6) ,l(6i6sliiorkor.
r d):' Iv[3lfln syqdrome ']: ,I ' 'l

r This flaky paint dermatosis is characteristic of kwashiorkor.

BREA5T FETDING AND BBEAST MIt.K

o Breast feeding should be initiated within 30 minutes of a normal vaginal delivery and within 4 hours of delivery by
04' PGI ee).
caesarian sectionAl
r WHO recommends 'exclusivebreastfeeding'for the first six months of life(NEEr AilMs06), i.e. during first 6 months only
breast milk is given and nothing else (even water). Wealring should start by 6 months of age when complementary
food should be started in addition to breast milkarq4).
r Breast feeding should be continued with semisolid and solid foods upto two yearsatoa).
t Human breast milk is the best source of nutrition for infant with following benefits :-
1) Superiornutrition(Pcr08)
A) Carbohydrate
r Lactose is in a high concentration in breast milk which helps
in the absorption of calcium and enhances
the growth of Iactobacilli in the intestine.
B) Protein
r The protein content of breast milk is low which causes lower
solute load on the kidney. Most of the protein
is whey proteins (lactalbumin and lactoglobulin), which can
be digested easily (In contrast cow milk
contains more casein).
r Breast milk contains the ideal ratio of the amino acids cystine,
taurine and methionine to support
development of central and peripheral nervous system.
C) Fats
r Breast milk is rich in polyunsaturated fatty acids, necessary for the myelination of the nervous system
and brain growth.
r Active lipase in the breast milk promotes digestion of fats and provides
FFA. The pattern of fats facilitates
absorption of calcium.
D) Minerals
r Breast milk protects against neonatal hlpocalcemia and tetany
due to ideal calcium phosphorus ratio (2: 1)
and better calcium absorption.
r Iron of breast milk is very well absorbed -> breast feeding prevents against iron deficiency
anemia.
r Breast milk also prevents deflciencies of vitamin A, c, D, E and zinc.
r Breast milk has a water content of 88% and hence a breaslfed baby does not require
additional water in
thefirst 6 months of lift even in summer months.
r Breast milk has a low mineral and sodium content -> low osmolality presents
a low solute load to the kidney.
,\ Lower risk of infection(Pcros)
r The breast milk is clean and uncontaminated with several anti-infective factors -+
protect the baby from infection
and diarrhea.
3) Protection from allergy (atopy) {lctoa;
: Low protein contents of breast milk along with a higher concentration of secretory IgA decreases the absorption
of protein macromolecules -+ less chances of allergy and asthma.
4) Emotional bonding
r Breast feeding promotes close physical and emotional bonds
between the mother and the babv.
5) Others
r Breast feeding lowers the risk of ovarian and breast cancer
in mother.
r Breastfed babies have a higher IQ and have less chances ofdeveloping
hypertension, obesity, coronary artery
disease and diabetes in their adulthood.
I The other diseases which are less common in breastfed babies include
inflammatory bowel disease, Hodgkins
lymphoma, necrotizing enterocoritis and sudden infant death syndrome.
r Breast feeding protects against evening colical e8, AIIMIe6).
r Breast milk contains docosahexaenoic acid(ArMsrI'06), required
for the growth of nervous system(izMsir,06).
Anti-infective effect of breast milk
o Breast milk provides protection against infection because
of the following facts -
.
I The breast milk is clean and uncontaminated@cr 0s).

2. Breast milk contains several antiinfective


factors
i) Antibodies -+ secretory IgA, Igl4@eros)
ii) Lysozyme
iii) Antktaphylo co ccal factor
iv) Specific inhibitory substances against viral infections.
v) Lactoferrin -+ Inhibits growth of E. coli.
vi) Bile stimulated lipase -+ kills entamoeba histolltica and Giardia
1amblia.
vi1) Bifidusfactor ) Inhibits growth of E. coli
v1lt) Para-amino-benzoic acid (PABA) -+ Provides protection against malaria
ix) Phagocytic macrophages and lymphoid cells

3. Lower pH of the stool of breastfed infants contributes to the favorable intestinal Jlora -+ more bifidobacteria
and lactobacilli, fewer E. coli(Pcr 08) .

Some disadvantages of exclusive breast feeding


o Exclusive breast feeding may cause deficiency of vitaminBrr4uussz) (if mother is pure vegetarian), vitamin K, vitamin
D and fluoride.
r Vitamin K deficiency can cause hemorrhagic disease of newbornar es' AttMS e6)
.

e6) e6).
o There may be neonatal jaundice('{[Ms and golden colour stool(Ar e&
'{[Ms

Differing composition of breast milk


o The composition of breastmilk varies at different stages after birth to suit the needs of the baby.
o According to the postnatal period the breast milk is divided into -
1. Clostrum
r Is the milk secreted during the first three days after delivery.
r It is yellow and thick.
r It contains more antibodies andcells and high amounts of vitamins A,D,E and K.
r It contains less fat atnd stgar6o'n'st) .

2. Transitional milk
r Is secreted during the following two weelcs (after clostrum).
r The immunoglobulin and protein content decreases while the fat and sugar content increases.
3.Mature milk
r Follows transitional milk.
r It is thin and watery but contains all the nutrients essential for optimal growth of the body.
o According to feed the breast milk is divided into -
1. Fore milk
t lt is watery and is rich in protein, suga6 vitamins, minerals and water that satisfy the baby's thirst.
2.Hindmilk
r Comes towards the end of feed
: Rich in fat content(comed 07)
and provides more energy, and satisfies the baby's hunger.
r The milk of mother who delivers prematurely differs from the milk of a mother who delivers at term -
Preterm milk
r Contains more protein S, sodium, iron, immunoglobins and calories as they are needed by the preterm baby.

Storage of breast milk


o Breast milk can be stored at room temperature for 8- 70 hours@EEr) , in a refrigerator for 24 hours(up 0& PGI e8) and in a

freezer (-20" C) for 3 months.

Breast milkVs Cow milk


e7' AI 07),
o In cprnparison to cow milk, human milk contains more : Lactose (7 gllOO ml or 7o/r)(ArrMs
10' 07'
antibodies,
lactoferrin and other antiinfective factors.
o In comparison to cow milk, human milk contains less amount of : ProteinsailMs 10' 07' Ar07'PGI11) (1 gm/100 ml), salts
(sodium, chloride, potassium), fatrPct 11' At07) (3.4 gm/100 ml), and minerals(Ar07) (calcium, phosphate).
o Though, the amount of mineral and salts is less in human milk, they are in correct amount,lvhereas they are in excess
in cow milk. The coeficeint of uptake of iron in breast milk is 7|o/oqrrMs 11' 08' At 04) .
o Human milk protein constitute mainly whey protein, where as casein is the main protein in cow's milk(A[Ms r0' 07).
o Different constituents of human breast milk provides evergy in following percentage. Carbohydrate 30-40o/o, protein
oe).
7 - 1 0% and fat 4 5 - 60o/o@Gr
, Fqt content of milk: Baffalo > Gaat > Cow > Humon@NB H)
t Protein content of milk: Buffolo > Goat > Cow > Human;DNs tat
. Loctose content of milk: Humon > Buffalo > Goat > Cowt.DNB t4l
. Energy content of milk: Buffalo > Goot > Cow > HumantDNBt4)

Contraindications of breast feeding


o These are:
1) Galactosemia
2) Active untreated tuberculosis - only in initial periofltoxn tt.
3) HIV positive mother - especially in developed country.
4) Some medication

RICKETS

o Rickets is a metabolic disorder of growing children in which


there is defective minerali zation(Ar 13,pGre8)
of normally
formed osteoid (protein matrix) (Pcres).
o Most common area affected is growth plate (physis or epiphyseal
plate) of long bon-estAr ss).
o Rickets is a disease of growing skeleton.It is character izedby
following pathological changes in skeleton :-
i) Thickening of growth plate (physis).
ii) widening of growth plate (physk); clinically it presents as palpable enlargement at wrist, elbow, ankle, and also
at costochondral junction (cailed rachitic rosary).
iii) Cupping or tlaring of metaphysis@Gl s8).
iv) Fraying of metaphysis.
v) Widening of epiphysb@ct se, sst
.

vi) Softening and deformity of long bones -+ Bowing of bones@cr es)


.

Clinical features of rickets


o clinical manifestations of rickets are usually manifested
by 6 months of age. Rickets is unusual below the age of 3
months.
r Clinical features ofrickets are :-
A) Regionalmanifestations
o These are :-
i) Skull CraniotabesallMs0T'03'PGI 03) (eailiest manifestation(Alee)),
frontal and parietal bossing; widened
"
sutures; delayed closure of anteriorfontanelLrrMsoT); Caput quadratum
or hot cross_bun skull:soft skull
with ping-pong ball like feel.

I craniotabes is the softening of skull bones, which may be present in normal newborn,
in pre-
especially
mature'ItismostlyseeninJccipital andparietal bonl;.rt'*uyuisobeseen
inricketspcteT),osteogenesis
imperfecta,Pcrez', congenital syphiti5recrc;; and hydrocephflus.

ii) Chest; Rachitic rosaryallMs rr),(prominent costochondral junction);


pectus craniatum (pigeon breast);
Harrison's groove (horizontal depression along lower border
of chest corresponding to insertion of
diaphragm).
iii)Teeth : Delayed eruption; enamel hlpoplasia; dental caries.
iv) Limbs and joints : Bone pain & tenderness (most common
manifestation); coxavara; genu valg1rs,Gerss))
or genu varus ; bowing of legs (tibisltorr*s or' 03), femur, radus
& tlna; widening of wris{W ost, elbow, knee &
anue; windswept deformityNEEr); string-of-pearls deformifT (Saucage-like
enlargement of ends of phalanges
and metacarpals with constrictions at joints); double malleoli
sign(two medial malleoli are palpable instead
ofone).
v) spine Thoracic klphosis (rachitic cat back); increased lumbar lordosis;
" scoliosis (uncommon).
vi) Abdomen: Pot-belly{'trrus '3) (protruded abdomen) due to hypotonia
of abdominal muscles.

rl
B) Systemicmanifestations
o These are growth retardationect 03) ' apathy, listlessness, irritability; hlpotonia; ligament laxity; and tetany,
convulsions and laryngeal stridor when there is hypocalcemia.

The follor+ing type of skull can be seen,in -

a)."'RiCkets:, ,' ,

b) Osteogenesis imperfecta
, 1
16) ,Congenital syphilis ::
d) All of,lhe above

Ans, is'?! i.e., AII of the above


,, .. Softening of skull (craniotabe$) can be seen il rickels, osteogenesis imperfecta; hydrocephalui
and con.
genital shyphilis.

Followingdeformityis seen ln -

a)Scurly ' . ,

c) Keratomalacia
d) None

Ans. is'H i.e., Rickets


i ,,,r'Itiq,$rhdsweptldeforrni*(gerxr'val$usinonekneeandgemtvarusinotherkiiee). Itisseeninrickets.

Rachitic (RickeU) rosary


o The prominent knobs of bone at costochondral junctions of rickets patient is known as rachitic rosary or beading
of ribs. The knobs create the appearance of large beads under the skin of the rib cage, hence the name by analogy with
the beads ofa rosary.
o Differentialdiagnosisof enlargementof costochondral junction (Rosary) -+ Rickets(Pcls7'Iipner87),Scvr\y(eettz'tip^"'
s7), ez Jipner 87)
Chondrodystrophy(Pcr ,

Types and (auses of rickets


o Rickets is of 3 types:-
1) TypeI -+ Due to calcium deficiency
2) Tlpe II -+ Due to phosphate deficiency
3) Tlpe III -+ End organs are resistant to active form of VitD i.e., L,25 (OH), D3.

Serum calcium
Causes : i) Deficiency of vitamin Dro^/8,5/ i) Primaryhypophosphatemia Vitamin D dependent
r Dietary(rNe4 uPe5) rickets (x-linked), type llro,v8,5)
r Lack of sunlight ii) Fanconi syndromes(D,va?5,
r Congenital r Cystinosis
ii) Malabsorption of vitamin D r Tyrosinosis
iii) Liver disease r Lowe syndrome
iv) Anticonvulsant therapy iii) Proximal renal tubular
v) Renal osteodystrophy@nars1 aciGlOSiSrDrvsr5,
vi) Vitamin D dependent rickets type I
iv) Deficiency ofphosphate or
malabsorption

Serum markers of rickets


r Depending on the tlpe of rickets following changes are seen in serum
markers _

1. Vit D deficiency or calcium deficiency

r vit D enhances absorption of Ca*2 from gut, So its deficiency results in decreased absorption.
r Initially serum calcium level is maintained because of increased activity of parathyroid
gland in response to
calcium deficit' After sometimes' even the compensatory increase
in parathormone cannot sustain normal
calcium level --> J calcium.
r Parathormone also enhances renal excretion of phosphat e + phosphaturia@cr -+ J 06)
serum phosphate.
r Because of unavailability of calcium and phosphate, bone mineralization is decrease -+
compensatory increase
in osteoblastic activity -) Increase alkaline phosphatase.
r So, there is :-
i) Normal to decreased calcium
ii) Decreased phosphass(euoe'ee) (except in renal osteodystrophy,
where phosphate level is increased because
kidney is not able to excrete calcium).
iil) Increased parathormone
iv) Increased alkaline phosphatase@Gl 06, ee)

2. Hlpophosphatemic Rickets
r This is due to deficiency of phosph ate -) Decreased Tthosphate,
t Calcium level is normal.
r As there is no calcium deficiency, there is no compensatory hyperparathyroidism
-+ Normal parathormone.
: Due to unavailabilityof phosphate, mineralization of bone is decreased
and there is increase in compensatory
osteoblastic activity + I Alkaline phosphatase.
r
Serum alkaline phosphatase is a consistent marker and is raised in
all types of rickets(.ares).
Radiological signs of rickets
o Earliest radiological changes are seen around the wrist, i.e,
at lower end ofradius dt ulna,
o Later on, similar changes may also be seen around knee, elbow
& ankle.
o X,ray findings are - ./

I Cupping &Jlaring of metaphysis@ero*os) ' '


t Large gap between epiphysis and metaphysis because of widening of physis (growth plate).
t Fraying of metaphysis due to irregurar calcification at metaphysear margins.
t Generalized osteopenia
s Cortical thinning
t Coarse (tfuzzy trabeculation
t White line of calcification (frenkel's line) at physeal region -+ It is the first radiological sign of healing and
appears after administration of Vit D.
Diagnosis of .the given X:iay-,.

b) Bone dysplasia
c) Rickets
d) None

o Widening, cupping and fraying of metaphysis is characteristic of rickets.

Treatment of rickets
r There are two startgies for administration of vitamin D -
A) Startgy 1 (stoss regimen)
r 6 lac IU (15,000 pg or 15 mg) Vit D3 is administered every 2 weekly.
r At every follow up monitoring is done by X-rays and blood investigations.
r Once healing is started, children are further put on 400 IU br 10 pg of vitamin D, per daI.
B) Startgy 2
r 2,000 - 5,000 IU (50 - 125 pg) of vit D, is given every day for 4-6 weeks.

SCURVY

o Scurr,yis due to deficiency of vitamin C. Vitamin C is required for normal collagen synthesis. Thereforemanifestations
of scurvy are mainly due to defective collagen synthesis :-
I) Defective osteoid productional 10' NrMs eB) .
ii) Deranged capillary function predisposingfor bleeding e.g. subperiosteal bleeding@cl00), gingival bleeding{rcroor.
111) Defective endochondral ossification causing epiphyseal separation@Gr 0o) and brittleness of bone.
iv) Defective dentition and poor wound healing.

a Collagen is a component of osteoid (protein matrix).


a Collagen synthesis is deficient in scurvy; therefore, osteoid formation is defective(A,1o,NIMSsa).
a On the other hand, in rickets osteoid formation is normal, but mineralization (Calcification) of osteoid is defective.

Clinical manifestations of scurvy ,


o The usual age of manifestations of scurr,y is 6-i8 months.
o The manifestions are
1. General symptoms -) Low grade fever, irritabiliry tachlpnea, digestive disturbances, Ioss of appetite.
2. Bleedingluess) + Subperiosteal@Gt00), gingival(rcroo) and conjunctival hemorrhage, purpura & ecchymoses,
petechiae, perifollicular hemorrhage, epistaxis.
3. Defective dentition -+ Bluish purple, spongy swellings of mucous membrane (due to hemorrhage), especially
over the upper incisors.
4. Generalized tenderness(Pcle6) and pseudoparalysi5@et oe t
-+ child is dilficult to handle due to pain and generalized
tenderness. The pain results in pseudoparalysis with the hips and knees in semi-flexed position and the feet
rotated externally.

Pseudoparalysis is also seen in osteomyelitis, septic arthritis, nd congenital syphitis


5' scorbutic rosary(uPe8)-) Due to epiphyseal separation,
costochondral junctionrK,,.00) is disrupted and
sternum
is displaced backward with attached costal cartiiages.
The end of ribs becomes prominentr, rn00) -)rosary.

e, D y...............e
to m elaph ys ea I w i d e n i n g Due to epiphyseal separation
o Non-tender
r Rounded

6' !ns1ni6(ut ss)


-+ Due to defect in utilization of iron & folic acid.
7. Bones arebrittle and easilyfractured.
8. Irritability and other psychologic symptoms.
9' others -+ Arthralgia, muscle weakness, degeneration of
skeletal muscres, cardiac hlpertrophy, adrenal
and bone marrow atrophy
depletion.

Following deformity is seen in -

a) Scurvy'
b) Rickets
c) Keratomalacia
d) None

Ans. is'b' i.e., Rickets


r This is rounded beading ofribs + rachitic rosary (In scur\7, rosary is pointed
or angulated).

Following deformity is seen in

a) Sgurvy
b) Rickets
c) Keratomalacia
d) None

Ans. is'a'i.e., Scurvy


r Pointed rosary is seen in scurly, i.e. scorbutic rosary,
Radiological features of scurvy
o The tlpical radiological features occur at the ends
of long bones, partic tlarly around knee, i.e. lower
end of femur
and upper end of tibia. other commo n site is
Ttroximal end of humerus.
r Features are :-
t Ground glass appearace ofbone
t Pencil thin cortex
t white line offraenkel 6arn -+ represents well calcified cartilage in
00)
(riptner s7)
metaphysi s
t wimburger sign(ter oo) -+ white ring surrounding the epiphyseal centres ,

of ossification . winburgersign is also


called ting sign@IMHANS es)
as there is white ring surrounding the
epiphyseal centers of ossificatign.
t Zone of rarefoction under the *7i1' 1lns1i,nere7) ) represents
u lin"u. break in the bone, proximal and
parallel to
white line and epiphyseal separation may occur along this 00).
line@Gr
: Lateral part ofthis rarefaction is seen as a triangular clefect.
t Palkan spur or lateral spur -+ lateral prolongation ofwhite line.
r During healing phase, subperiosteal hemorrhages become calcified
and the affected bone assume s a dumbbell
or club shaPe(ter oo).
White line of fraenkel is seen in:
i) ScurvY iii) Plumbism v) severe protein energy malnutrition
iv) Acute leukemia vi) Congenital sYPhilis
ii) Healing rickets

Given,X.rg is.diagnoetiiof'-,

a),.RickBts,'r. . '.,' ''


b) Osteomalacia
.'C)t Scuryy.., ,:.:.:.' .....::
r,'''d),:]r{ofl€,,.,,, .,:1. ..

Ans. is'c'i.e., ScurvY


. ,,.,i ,,,_GiteiiX,ray is,showiir$,1wifr:baicq's r ris" tteiis*hite,line,of ialcifidation'eociicllng.&c .qlleoporot-
ic epiPhYsis).

Circn '&raf is diagrrostic, o-f'.

a) Rickets
b) Osteomalacia
c) Scurrry
d) None

: l.r. ,,Giten X,ray il ihowi1g$Iittdbr$Flienkel and wimburger'sign 9 Diagnosis is scu(YYl

Laboratory diagnosis
Leukocyte vitamine C
o Leukocyte concentration of vitamin 'C' is a better indicator of body store,
it may be deficient even in the absence
of clinical signs of deficiency' It is estimated by buffy coat PreParation@Gles).
Plasmavitamin C
early vitamin C deficiency'
o plasmi dscorbic and levels vary widely and cannot be relied upon to detect
Urinaryvitamin C
excretion of the vitamin after a test dose
o Saturation of the tissues with vitamin c can be estimated frorir the urinary
of ascorbic acid. Excretion of 80% of the test dose in the urine
within 3-5 hr after parenteral administration is
80% of test dose is excreted'
considered to be normal. In vitamin c deficiency, less than

VITAMIN'A'

o Important functions of vitamin A in the body are :-


t) Normat vision (regeneration of rhodopsin in dark light)
pGI 88' e7)
ii) Anti-infective activity(Arse'
lil) Anti-cancet effect
iv) Anti-oxidant PtoPertY@EEr)
v) Skeletal growth.
cod-liver oil (2d richest) and liver ox (3'd richest)'
o rhe richest sotnce of vitamin-A is halibut liver oilfollowed by
Among vegetables carrot is the richest source'
Clinical rnanifestations of vitamin A deficiency
o The most obvious symptoms of vitamin A deficiency are associated with
requirement of this vitamin for
maintenance of epithelial functions -
i) Intestine -) Pathogenic attack causes diarrhea
ii) tract -)
Respiratory Infections{Pcte6) & bronchial obstruction
iii)Urinary tract -+ UTI and caliculi
iv) Skin -) Dry, scaly, hlperkeratotic patches
v) Squamous metaplasia of renal pelves, ureter, vaginal epithelium, and pancreatic & saiivary d.ucts.
c Poor grawth(Pc1e6) because of :-
i) Above health problems in children
ii) Vitamin A helps in bone growth + deficiency may retard growth
o Other features are :-
i) Skin becomes toad like (phrynoderma). But it is due to associated deficiency of essential fatty
acids.
it) Vitamin A deficiency may rarely lead to hydrocephalus@Gl e6).
o The most characteristic and specific sign of vitamin-A deficiency are eye lesios. The term
xerophthalmia(pclo4) (dry
eye) comprises ail ocular manifestations of vitamin-A deficiencv.
r It has following stages :-
1) Nightblindness (earliest symptom)@eroo
2) Conjunctival xerosis (eailiest sign)
3) Bitot's sPot@Grol)
4) Corneal xerosis
5) Corneal ulcer (t keratomalacia.
Requirement and doses
o The daily requirement of vitamin A for infant is 300-400 s7),
children is 400-600
VgGIrMs prg and adolescents is 750
trc.
o For treatment of vitamin A deficiency, oral vitamin A is given at a dose of 50000 IU, 100000
IU and 200000 IU in
children <6 months, 6-12 months and > I yea6 respectively4r ts;, The same dose is repeated next day and 4
weeks
later.
o For prevention (vitamin-A prophylaxis programme) a total of 9 doses of vitamin-A
are given between the age of 9
months to 5 years. The first dose (1 lakh units(AzMs,s'AI0e)) is given at 9 months of age along with
measles vaccination.
The second dose (2 lakh units) is given along with DPT/OPV booster doses (at 18 months).
Subsequent doses (2 lakh
units each) are given at six months interval upto 5 years age.

MISCELLANEOUS

Hypervitaminosis-D
o The sign and symptoms of vitamin D intoxications are secondry to hypercalcemia which
is caused by -
i) Excessive bone resorption (major cause) -+ Vit D causes bone resoryfiion@Gl06) .
ii) Increased calcium absorption
r Manifestations are i-
1' GIT -+ NauseaattMss6'PGI 7e),
vomiting, onsvsxciqLrtuss6'PGr7e),poor feeding, constipation, abdomifal pain
and pancreatitis.
2. CVS -+ Hlpertension, arrhyhmias and decreased eT interval.
3. CNS -+ Lethargy, hypotonia, confusion, psychosis, disorientation, depression, hallucinations & coma.
4. Kidney -+ PolyuriaatrMss6'PGr7e),hypernatremia, dehydration, nephrolithiasis and nephrocalcinosis.
5. Metastatic calcificationAltMss6'PGI7e).
Hypervitaminosis-A
o Excess of vitamin A can lead to ruptute of lysosomal membrane.
o Acute manifestations -+ Signs and symptoms are due to raised intracranial tension (pseudotumor cerebri or benign
intracranialhypertension6"t)) + headache, nausea, vomiting, dror,r,siness, bulging fontanelles, diplopia, papilledema
and cranial nerve palsies.
o Chronic intoxication -) Anorexia, weight loss, painful extremities, dry itchy desquamating skin, alopecia, coarsening
bone abnormalities and bony swellinglPct
1s), 06)
of hair, hepatospleno megaly4l .

Zinc deficiency
e Important clinical features of zinc deficiency are :-
L. Dwarfism (growth retardation)(Pct 0s)
7. Decreased immunocomPetance

2. Diarrhea 8. Poor woundhealingleero:"uest


3. Dermatitisares) g. Hyperpigmentation(Pcr03)
03' Ar e8)
4. Hepatosplenomegaly 10. Hypogonadism@Gt
5. Iron defciency anemia 71. Perioralrash
6. Acrodermatitis enteroposlliss@t ts)
o Loss of hair and taste alterations may also be seen.

r Dose for treatment of zinc deficiency is 10 mg/d.ay for 14 days for infant 2-6 months of age and 20 mg/day for 14

days for older than 6 months@EBr).

Acrodermatitis enteroPathica
o AcrodermatitisenteropathicaisarareautosomalrecessivearMsir)disordercausedbyaninabilitytoabsorbsufficient
Zincfromthe diet.
o Initial signs and symptoms occur during the first few months of life often after weaningfrom breast to Cow's milk.

The manifestations are:-


i) Skin :- Vesciobullous, eczematous, dry, scaly or psoriasiform skin lesions, symmetrically distributed in the peri-
oral6lt), acral and perineal areas and on the cheeks, knees and elbows.
ii) Hair :- Hair often has a peculiar reddish tint and alopecia of some degree is characteristic.
iii) Ocular :- Photophobia, conjunctivitis, blepharitis, Corneal dystroply.
iv) Associated manifestations :- Chronic diarrhoea(A[Ms11), stomatitis, glossitis, Paronychia, Nail dystrophy, Growth
retardation, irritability, delayed wound healing, Bacterial & candidal infection.
rr'e3).
r Diagnosis is made by tlpical clinical findings and detection of low plasma zinc concentration(AzMs
o Oral therapy with zinc compound is the treatment of choice. Zinc therapy rapidly abolishes the manifestation of
the disease6zMs ").

..:,:,:r.: r..::;r:ii.:, .i:r::;i,i::,:


jj
.. :::".r;i,i'. :Iti:

': r::.:. ':::t:. i::;:: ----_ r. :.::: i:

| ::i::r:r;:,1ij .:r:: ji
r !--.r,,;.::. -:r:t
.r:r;!.
r.i:.r;r:r:-:j:.
,1,e.Hap,.:rpn,1 z Nitrition

* Other important features of Kwashiorkor: Hepatomegaly (fatty infiltration), flag sign on hair, flaky paint dermatosis, hypoalbumine-
mia, apathy, poor appetite, low insulin.
o Flaky pain dermatosis is characteristic of : Kwashiorkor.
o Not seen in Kwashiorkor :Vocarious appetite (it is a feature of marasmus), altertness (there is lethargy).
* I lmportant features of Marasmus : No edema, vocarious appetlte, no skin or hair,changes, no hepatomegaly;'active child,
nolmal
.a[buminandinsu[in,musclewastingandweakness.
. Most affected antibody in
PEtVt : lgAl

*Cal.bric.suppIementationrequiredinseverePEM:175.200.kcal/kg/day
,. ...:.
r Exclusive breast feeding is till : 6 months.
* Breastfeeding shoutd be initiated: Within 30 minutes,of normal vaginal delivery and'within 4 hours of delivery.by iae*rianiaitionr
o Most important protective component of breast milk against infection : lgA.
* Fatty acid in breast milk which is required for normaf g;owth of brain ; Docosahexaenoeic acid. :. :]:. ' 1
r' . I :., . :

':
* PABA in breast milk provides protection against : plasmodium.
* -, Exctusive breast fee:d:ing may.cause deficiency,of : Vitamin.Bl2, Vitami.n (, Vitamin D and fluoride ,.:.: ,.
..

c Exclusive breast feed ing can cause : Hemorrhagic d isease of newborn (d ue to deficiency of vita min K), prolongation of physiologica I
jaundice, golden colour stool.
o Not seen in exclusive breast feeding : Evening colic (breast feeding protects against evening colic). :.

o Main protein in human milk :Whey protein.


:

.x.Main.protein.inCowtmilk:Casein..,...]
o Human milk contains more (than Cow's milk) : Lactbse, antibodies, lactoferrin.
* Cowtmilkcontains'moie(thanhumanmil'ki:iProtein;saks(sodium;chloride,potassium),fat,minerals(calcium,phosphate). .i,,
o Percentage of lactose in human milk:7.0o/o(7 gm/100 ml).
a Fore-milk is rich- in : Protein, sugar. vitamins, minerals,water. :

o Hind milk is rich ," ,tr;,.;";


r clostrum contains : More vitamins but less fat and sugar (than mature milk).
o Premature milk contains : Less lactose (in comparison to term milk).
* Breast milk can be stored for : 8-10 hrs at rosm ternp€rature, 24 hours in'refriger:ato5 3 months
in freezer at -20oC.
o Breast milk is known to transmit:CMV HlV.

. Most prominently affected in Rickets : Growth plate (physis), i.e. defective mineralization of growth plate.
*,. lmportantfeaturesofrickets:Thickening&wideningof..physi5,cupping/flarlng&ftayingofmetaphysis,wideningofepiphysis.
* Earliest manifestation of Rickets : Craniotabes.
. lmportant features of rickets : Craniotabes, delayed closure of anterior fontanel, caput quadratum (hot cross-bunn skull), rachitic
' rosa$ genu valgus, bowing of tibia, double malfeoli
sign, windswept deformity, widening of wrlst, gror44h.retardation, pai*fry.
* Windswept deformiiy in children is seen in : Rickets (most common cause), physeal osteochondromatosis, herediatary epiphyseal
dysplasia.

* Most common cause of genu valgum in childlen : Rickets,


*: ', Not afeature of Riekets.:Gun5tockdefOrmity (it is seen in ma!united supracondy[arfracturgl' '' ''''' .

o craniotabes is seen in : Rickets, hydrocephalus, syphilis, osteogenesis imperfecta.

..::.,:' I
* lmportant lab findings in vitamin D dependent rickets : Normal or slightly decreased serum calcium and phosphate, raised serum
alkaline phosphatase, increase PTH,irlcreased serum bicarbonate.
lmportant labfindings in hypophosphatimic Rickets. ormalselum calcium, decreased run:l
ahosphate;jn$eased,seru.m atkaline'

a Primary defect in scurvy : Decreased osteoid (protein matrix) formation due to defect in collagen synthesis.
rl lmportant features of scurvy : Subperiosteal & gingival bleeding, generalized tendeinesi. pseudoparalysis, scorbutic rosary.
& Pseudoparalysis in an infant suggests : Scurvy.
c r'seuoopaiatysis is also seen in r septic arthritis, osteomyelitis,:congenital syphilis. , , '
a

e
a

a
a

a
a

a
a

a
a

a
a

a
a

III
QUESTIOT{S

N UTRITIONAL STATUS INDICATORS 10" Frotein requirement for 2 year otd child @er dayl
(CET luly 15 Pauern)
I. Deficit in weight for height in a 3 years old child a) 10 gm b) 15 gm
indicates- eET luly 15 pattern) c) 20 gm d) 25gm
a) Acute malnutrition 11. The amount ofcalories required at I year ofage are-
b) Chronic malnutrition (Ar e6)
c) Concomittant acute and chronic a) 900 K callday b) 1000 K call
d) Under weight duy
2. Best indicator for nutritional status for a child is - c) 1200 K callday d) 1400 Kcallday
(AIIMS May 10, Nov 06) 12. The normal calorie requirement for a 5 year old
a) Mid arm circumference child is- (PGt e3)
b) Head circumference a) 800 calories b) 1000 calories
c) Rate ofincrease ofheight and weight c) 1500 calories d) 2000 calories
d) Chest circumference
3. Acute malnutrition is manifested by - @Gr Nov t4) PROTEIN ENERGY MALNUTRITION
a) Weight for age
b) Weight for height
c) Age for height
13. In marasmus wasting is due to - (All Inriia Dec15 pattern)
a) Prolong dietery deficiency ofcalories
d) Brocas index b) Prolong dietery deficiency ofprotein
c) Ponderal index c) Excess catabolism of fat & muscle mass to provide
4. Which of the following is the besr indicator of long energy
tenn nutritional status - (CET Nov. 15 pattern) d) All of above
a) Mid arm circumference t4. Kwashiorkar is diagnosed in growth retarded chil-
b) Height for age dren along with- (NEET Dec.12 pattern)
c) Weight for age a) Edema and mental changes
d) Weight for height b) Hypopigmentation and anemia
5" Common to both acute and chronic malnutrition c) Edema and hypopigmentation
is- (AIIMS May 07) d) hepatomegaly and anemia
a) Weight for age b) Weight for height 15. Which of the following is seen in Marasrnus and not
c) Height for age d) BMr in Kwashiorkor - (All India Dec. 15 pattern)
6. Weight of child is 70% of normal - according to IAp a) Vocarious appetite
classification, categorised in - (AlltrutiaDec15 pauern) b) Fatty change in liver
a) Mild b) Moderate c) Hlpoalbuminemia
c) Severe d) Normal d) Edema
7. A 2 year old child has a weight of 6.4 kg. and has L6. Kwashiorkar is characterised by all of the following
vitamin A deficiency,What is the grade of malnutri- features except - (AI s9)
tion in this child - (AIIMS 87) a) Edema
a) First degree b) Second degree b) Patchy depigmentation of hair
c) Third degree d) Fourth degree c) Fatty liver
R The following statement about Gomez classification d) Fatty infiltration ofpancreas
is false - (Ar 08) t/. All are seen in lVlarasmus except -
a) Based on height retardation (All India Dec.14 Pattern)
b) Based on 50tl'centile Boston standards a) Hepatomegaly b) Muscle wasting
c) Between 75 a:nd 89% implies mild malnutrition c) Voracious appetite d) Weight loss
d) This classification has prognostic value for
hospitalization of children
18. Not Seen in Iftuaqhiorkar - (All rndia Dec15 pattern)
a) Apathy
9. \{aterlow elassifieation of malnutrition in ehild b) Flaky paint dermatosis
takes into account ? (CET lune 14 Pattern) c) Baggy paint appearance
a) Weight for height (wasting) d) Increased tra4saminase
b) Height for age (stunting) 19. Not seen in kwashiorkor- (Alt India Dec.t3 pauern)
c) Weight for height (wasting) and height for age a) Apathy
(stunting)
b) Flaky paint dermatosis
c) Poor appetite d) Increased albumin
d) Percent ofreference weight for age
20. All of the following conditions are obseryed inMt- d) 24
rasmus, except - (AllMS May 0s, DPG 09)
31. The current recommendation for breast feeding is
a) Hepatomegaly b) Muscle wasting that - (Ar 04)
c) Low insulin levels d) Extreme weakness a) Exclusive breast feeding should be continued till
21. l5 months old child feeding on cow milk with water 6 month of age followed by supplementation with
wih severe wasting and bipedal edema with poor additional foods
appetite - (CET luly 15 Pattern) b) Exclusive breast feeding should be continued till
a) Kwashiorkar b) Marasmus 4 month of age followed by supplementation with
c) Both d) None additional foods
22. In Kwashiorkor, which immunoglobulin is most c) Colostorum is the most suitable food for a new
afiected- (At 94) born baby but it is best avoided in first two days
a) IgD b) IgA d) The baby should be allowed to breast feed till one
c) IgE d) IgM year of age
23, A child is sufiering from severe PEM. Calories to be 32. The protective efiects of breast milk are known to be
given per kg of body weight to regain weight- associated with - (Ar 0s, DPG 09)
(AIIMS lune 99) a) IgM antibodies b) Lysozyme
a) 200 Kcal b) 150 Kcal c) Mast cells d) IgA antibodies
c) 400 Kcal d) 100 Kcal JJ. The important fatty acid present in trreast milk
24. Caloric supplementation required for a severeh which is important for gronth is - (AIIMS Nor, 1 1, Nov 06)
malnourished child @erkg-body weight) is - (Ar ee) a) Docosahexaenoeicacid
a) 100 callkg b) 125 callkg b) Palmitic acid
c) 150 callkg d) t75 callkg c) Linoleic acid
25. Ideal time to start Iron theapy in a marasmic child d) Linolenic acid
with fever and hemoglobin 7 gm%o is - 34. What is deficient in exclusively breast fed baby -
(All India Dec15 Pattern) (AIIMS Feb 97)
a) Immedietly a) Vitamin B b) Vitamin A
b) At discharge c) Vitamin C d) Proteins
c) When fever goes down 35. Exclusive milk ingestion can manifest as- (ArrMS s7)
d) At any time a) Scurr,y b) Beri-Beri
25, Flaky paint appearance of skin is seen in- c) Phryenoderma d) Night blindness
(NEET Dec.12 Pattern) 36. Which of the following will occur in an exclusively
a) Dermatitis b) Pellagra breast fed baby - (AIIMS Sep e6)
c) Marasmus d) Kwashiorkor a) Jaundice b) Scurry
27 . All of the following are characteristic features of c) Tetany d) Eczema
Kwashiorkar, except - (AIIMS May 0j) 37. Exclusive breast feeding may be associated with all
a) High blood osmolarity of the following except - (Ar e8, ArrMS 96)
b) Hypoalbuminemia a) Hemolysis due to Vit-K deficiency
c) Edema b) Evening colic
d) Fatty liver c) Golden colour stool
28. Severe acute malnutrition (Sav), true in manage- d) Prolongation ofphysiological jaundice
ment - (CET Nov 15 Pattern) 38. Breast feeding contraindication - (CET luty 15 pattern)
a) SAM with severe edema should be hospitalised a) Hep A
b) SAM wifh good appetite should be hospitalised b) Hep B
c) SAM with poor appetite should be treated in OPD c) CMV
d) Decision of transfer from in patient to out patient d) Active untreated T.B.
care depends upon specific mid-Upper arm
39. True about cow's milk are all except-
circumference value
(AIIMS May 10, May 07)
a) Cow's milk contains 80% whey protein not casein
BREAST FEEDING & BREAST MILK b) Cow milk has less carbohydrate than mothers
milk
29. Exclusive breast feeding is at least till- c) Has mo(e K* and Na* than infant formula feeds
(CET Nov.14 Pattern) d) milk
Has more protein than breast
a) 4 month b) 6 month 40. Percentage of lactose in human milk is -
c) 8 month d) 10 month (All India Dec.14 Pattern)
30. Breast feeding should begin ... hours after delivery- a) 7.2 gm b) 4.s gm
a)2 b)4 c) 8.0 gm d) 6.7 gm
ti

41. Compared with Cow's milk, mothers milkhas more- 50. True about nutritional rickets - (PGI May u)
(Ar 07) a) Craniotabes
a) Lactose b) vit D b) Multiple #
c) Proteins d) Fats c) Widening of wrist
42. Hind milk is richer in - (Comed 07) d) J Phosphate in serum
a) Carbohydrate b) protein e) GroMh retardation
c) Fat d) Minerals 51. Rickets in infant present as all except- (AtrMS May07)
43, Breast milk rtorage in a refrigeratcr io upto - a) Cranitabes
(CET Nol',.14 Pattern) b) Widened Fontanel
a) 4 hrs b) 8 hrs c) Rachitic Rosary
c) 12 hours d) 24 hrs d) Bow legs
44. Breast milk at room temperature stored for- 52- All of the following are seen in Rickets except-
(NEET Dec.12 Pattern) (AIIMS May 0j)
a) 4 hrs b) 8 hrs a) Bow legs b) Gunstock deformity
c) 12 hrs d) 24 hrs c) Pot belly d) Craniotabes
53. Most common cause of genu valgum in children is -
RICKETS a) Osteoarthritis b) Rickets
c) Paget disease d) Rheumatoidarthritis
45. Basic pathology in rickets - (AIl rndia Dec.t3 pauern) 54. Windsweptdeformityis,seen ilo,- 1Nz,ar oec.12 pattern)
a) Defective bone matrix formation a) Scurly b) Rickets
b) Defect in mineralization c) Achondroplasia d) Osteoporosis
c) Defect in osteoid formation
55. Causes of hypophosphatearic rickets - Nn -
d) All of the above
(CET luly 15 Pattern)
46. Deficient mineralisation in epiphysical growth carti- a) Vit D deficiency
lage is seen in - (Ar ee) b) CRF
a) Rickets b) Osteomalacia c) X-linkedhypophosphatemicrickets
c) Scurly d) Hyperparathyroidism d) Fanconi syndrome
47- A 2-year-old troy has vltamia D reeistant rickets. His 56. Vit-D deficiency Rickets is Characterised by -
investigations renealcd s€rum Calcium- 9 n $ dl"
@GI Nov 14)
Phosphate- 2.4 rugJ dl, alkaline phosphataoe- I 04 1
a) t ed forehead sweating
IU, nornral intact parathyroid horraone aad tricar- b) Characteristically Jed Car*
bonate 22 mfi.q/L.1&'hich of the fo0owiag is the rnost
c) Anterior fontanel widened
probable diagnosis - NEET Dec.l2 Pattern, AIIA.{S
d) ted alk. phosphatase
May{M,02)
57. il,arlier manif€statioa of Riekets is -
a) Distal renal tubular acidosis
(All lndia Dec.14 Pattern)
b) Hlpophosphatemic rickets
c) Vitamin D dependent rickets a) Craniotabes b) Rachitic rosary
d) Hypoparathyroidism c) Harrison groove d) Pigeon chest
58. Craniotabes is seen in following except- (pGt Dec 97)
48. A old boy has vitamin D refractory rickets.
2 year
Investigations show serum calcium 9 mg/dl. phos-
a) Rickets
phate2.4mg dl, alkaline phosphate 1040 tU. para-
b) Syphilis
thyroid hormone and bicarbonate levels are normal.
c) Osteogenesis imperfecta
the most probable fiagnosis is -
d) Thalassemia
(ArrMS Nov 02, 00)
a) Distal renal tubular acidosis 59" Splayiag aud Cupping of the metaphysis is seea
b) Hlpophosphatemic rickets. in - (CETNov.l4pattern)
c) Vitamin D dependent rickets a) tuckets b) Scurry
d) Proximal renal tubular acidosiS-.. c) Paget's disease d) Lead poisoning
49- A l0 year old boyhas a fracture of femur. Biochem- Rickety rosary seen in all except - (PGI Dec 97)
ical evaluation revealed Hb lf .5 gm/dl and ESR lS a) Rickets b) Scuny
mm lst hr. Serum calcium 12.8 mg/dl, serun phos, \ c) Chondrodystrophy d) Slphilis
phorus 2.3 mg/dl,, alkaline phosphate 28 KA units 61. Costochondral junction swelling are seen in-
and blood urea 32 mC/dI. Which of the following is (JIPMER 87)
the most probable fiagnosis in his case- (At 04) a) Scurvy b) Rickets
a) Nutritional rickets c) Chondrodystrophy d) Alloftheabove
b) Renal rickets 62" How to acsess Vit" D tfuerapyin Rickete is -
c) Hyperparathyroidism (All India Dec15 Pattern)
d) Skeletal dysplasia a) Sr calcium b) Sr. alkaline phosphatase

I
: \-rav wrist d) Sr. phosphatase 73. The vitamin A eupplement administered in 'Pre-
vention $. nurtntiana;l blindness in children pro-
SCURVY gra;mmc" contain - (Al0s, AIIMS Nov 0s)
a) 25,000IU/ml b) I Lakh ILr/ml
!f Primary metabolic bonc disorder in sflrrvy i$ - (AI 10) c) 3 Lakh IU/ml d) IU/ml
5 Lakh
a r Decreased mineralization 74. 2 year old boy of weight 12 kg with vitamin A defi-
b r Decreased osteoid matrix formation ciency what is oral dose of vitamin A -
. ) Increased bone resorption (All India Decl5 Pattern)
d) Decreased bone mass with normal mineralization a) 50,000 I.U. b) 1 lakh LU.
and osteoid formation c) l.5lakh I.U. d) 2lakh I.U.
b4. A six month old infant fedtatilly on cow's milk has
been brought with bl€€ding spots, anaeffiia,fcver MISCEttANEOUS
and generalised tenderness. On examinetian, there
were swelling in both the lower extremities and the 75. Allare features afYit. D intoxicatiofl,except-
blood count was normal. The most likely diagnosis (PGr 79, AilMS 86)
ts- (PGt e6)
a) Nausea and vomiting
a) Arthritis b) Poliomyelities b) Muscular weakness
c) Osteomyelities d) Scurr,y c) Anorexia
65. Pseudoparalysis in an infantis suggeetive of.- (Pqoa) d) Oliguria
a) Acute Rheumatic fever e) Metastaticcalcification
b) Vitamin 86 deflciency 76. Hypewit*minosis of which of the followingwill
c) Vitamin E deficiency cause bony abnormalities - (PGI Dec 06)
d) Vitamin C deficiency a) VitA b) VitD
6. l{inberger sign is present in - (PGt 2K) c) Vit C d) Vit E
a) Rickets b) Scurly e) Vit K
c) Secondary syphilis d) Tubercuiosis 77. Hypervitaminosis A causes - (All tndia Dects Pauern)
67. Deficiency of vit. C in infant is best estimated by Vit a) Benign cranial hlpertension
C level in - (CET lune 14 Pattern) b) Craniotabes
a) Plasma c) Liver damage
b) Urinary excretion d) All of above
c) Buffy coat estimation 78. A child with alopecia, hyperpigmentation psoriatic
d) Adrenal cortical Vit. C estimation dermatitis in genitals & mouth and hypogonadism is
68. Boy who refuses to eat fruit cornes with knee swell- likely to be sufiering from - (AI e8)
ing and hematoma, deficiency of which vitamin is a) Cu defeciency b) Iron deficiency
suspected ? (CET Nov. 12 Pattern)
c) Zn deficiency d) Mg deficiency
a) Vitamin C b) Vitamin D
79. Zincdeficiencycauses - (PGlDeclj)
c) Vitamin E d) Vitamin B,
a) Sexual infant ilism b) Loss of libido

VITAMIN A c) Poor weight gain d) Poor wound healing


80. In a child having diarrhoea with perianal moist
69. Anti infective vitamin is - (CET lune 11 Pattern) crust. The diagnosis is - (All India Dect5 Pattern)
a) Vitamin Bu b) Vitamin A a) Acrodermatitis enteropathica
c) Vitamin D d) Vitamin C b) Riboflavin dificiency
70. All of the following statement$ are true regarding c) Pellagra
Vitamin Adefrciency except - (PGte6) d) None ofabove
a) Growth retardation is common 81. Selenium defi.ciency is seen in - (NEET Dec. 12 Pattern)
b) Frequent infections can occur a) Keshan disease
c) Hydrocephalus is infrequent b) Wilson disease
d) Anterior segment of the eye is initia\ty involved c) Acrodermatitis enteropathica
71. Vitamin A deficiency is charactetizedby- (PGI May tt) d) None ofabove
a) Bitot spot b) Xerophthalmia 82. Diarrhoea in a child of 12 month, dose of Zinc is -
c) Night blindness d) Tranta's spot (CET July 15 Pattern)

72. Dailydoseofvitamin Aina6-l2months oldchild a) lmgll0-l4day


ts- (AIIMS lune 97) b) 10mglr0-t4day
a) 500 microgram b) 200 microgram c) 15mgl10-14day
c) 300 microgram d) 700 microgram d) 20mg 110-t4day
83. Elemental iron and folie acid content ofpediatric iron gZ. One year old baby exclusively breast fed ha$ a lcm.
folic acid tablet supplied under RCH programme hepatomegaly severe pallor and no spleen.The most
are- 6t 03) important investigation is - (rN s0)
a) 20 mgiron & 100 microgram folic acid a) B,, estimation
b) a0 mg iron & 100 microgram folic acid b) Serum iron estimation
c) 40 mg iron & 50 microgram folic acid c) Folic and estimation
d) 60 mg iron & 100 microgram folic acid d) Fetal hemoglobin estimation
&4, Requirement of potas*ium in child is - 93. Iron supplementation in a healthy term breast fed
(All India Dec15 pattern) -baby should be started at the age af - lcouro ool
a) 1-2 mEq/kg b) 4-7 mEq/kg a) 2 weeks b) 4 weeks
c) 10-12 mEq/kg d) 13-14 mEq/kg c) 8 weeks d) 6 weeks
94" Breast milk is maximum at - (APPGE 04)
a) l-2months b) 3-4months
c) 5-6 months d) 7-Smonths
85, The most important factar ta overcome protein gS,
Breast milk is known fo transmit -
energy malnutrition in ehildren less than 3 years is- (MAHE 07)
a) Tirberculosis b) CMV
(ArrMS 83)
a) Supply of subsidised food from ration shoo c) Varicella d) Rubella
b) Early supplementation of solids in infants 96, After premature delivery, mother,s milk is low in -
c) Immunisation to the child a) Lactose b) Fat (MH
d) Treatment of anaemia and pneumonia in infant 10)
and toddlers c) Protein d) Sodium
86, Kwashirokar in Ghanian Ga language means- 97. Lactose content of breast milk per 100 ml is - (MH t1)
(Kerala 2K)
a) 3.5 gm b) 4.5 gm
a) Condition seen in 2"d child c) 6gm d) 7gm
b) Condition seen in the displaced child 98. Which of the following is not a sign of active rick-
c) Condition seen in cousin ets?
d) Condition seen in the stepchild a) Prominent fontanelle (COMED 09)
e) Condition seen in the 3.d child b) Hot cross bun sign
87, In Kwashiorkor, the letter ,K, is post-fi.xed to de- c) Saddle nose
note- (Mh 10)
d) Caries teeth
a) Weight for height b) Skin changes 99. Definitive sign of scurvy in X-ray - jipmer u)
c) Edema d) Muscle wasting a) Ringed epiphysis
88. When can a severely malnourished child be safely b) Ground glass appearance
discharged from the hospital ? fupsc_t 0B)
c) White line in metaphysis
a) The child attains his height for age d) Thin cortex
b) The child reaches his ideal weight for height f 00. The recommended oral dose of vitamin A to be
c) The child loses edema and starts gaining weight given in a 10 month child with deficiency on each of
d) The child attains weight for his age day 1,2 and 28 is - (Karn 11)
89, The skin changes seen in proteinenergy malnutrition a) 50,000 IU b) t,oo,ooo ru
can be due to deficiency all of the following nutrients c) 2,00,000 IU d) 6,00,000 ru
except - (DELHI pG Mar.09) l0l. Phrynoderma is due to ...deficiency-
a) Zinc (Kerala 90, Karn pgi 90)
b) Trlptophan a) Vitamin D b)
89,

Niacin
c) Essential fatty acids c) Vitamin A d)Essential fatty acid
d) Pyridoxine
t02. Toad skin is seen in deficiency of vitamin -
90. Para amino benzoic acid of breast milk prevent the a)A b) B,
infection of - (up 07) c)D d) Biotin
a) Plasmodium vivax b) Kleibsella_pneumonia 103. Growth retardation, taste alteration, hepato_spleno_
c) Giardia d) E.coli megly, hypochromic microcytic anemia, loss of hain
91. The substance that has anti infective property direct- hypogonadism in aboyindicate deficiency
ly or indirectly in milk is all except-
of- (Ka,n
]TPMER es) es)
a) Lactoferin b) Lactalbumin a) Selenium b) Copper
c) Lysozyme d) Nucleotides c) Zinc d) Iron
xS4. Deficiency of which element can lead to syndrome of
gro*th failure, anaemia and hlpogonadism - 107, Child ruith frog like position and resietanc€ to move
a Calcium b) Copper (COMED 09) the Imbs - (cMC 01)
; Zinc d) Magnesium a) Scurly
tr,15. -{,bnormalities of copper metabolism are imphcated b) Rickets
in the pathogenesis of all the following except - c) Trauma
(UPSC-t o9) d) Congenital dislocations
a) Wilson's disease 108. Pseudoparalyeis is seen in - (cMC o1)
b) Monkes' Kinky-hair syndrome a) Scurr,y b) Rickets
c) Indian childhood cirrhosis c) Polio d) Osteomalacia
d) Keshan disease 109. Calorie requirement per day of.a childweighing 15
106. Dosage ofYitamin-A forchildrenbetw€en l-3 yeus- kgwouldbe - (MH 1o)
a) 1250 IU b) 1333lU (Manipal0S) a) 1150 kcal b) 1250 kcal
c) 7667 IU d) 2333 rU c) 1450 kcal d) 1550 kcal

I-I
ANSWERS
N UTBITIONAL STATUS I}I DICATOBS

1. Ans. is 'a' i.e., Acute malnutrition [Ref : O.p" Ghai Be/e p. 97 & Zh/e p. 62)
r Wasting (deficit in weight for height) -+ Acute malnutrition.
r Stunting (deficit in height for age) -+ Chronic malnutrition.
o Wasting and stunting -) Acute on chronic malnutrition.
r Underweight (low weight for age) -+ combined indicator to reflect both acute and chronic malnutrition.
) Ans, is 'c' i.e., Rate of increase of height and weight {Ref: o.p. Ghai B&/e p. 13 {t 7n/e p. 6, 6j)
a

t Weight in relation to height is now considered more important than weight alone. Ithelps to determine
whether a
child is within range of normal weight for his height.

3. Ans. is 'V i"e.,Weight for height {Ref : O.p. Ghai *h/e p. 97 & p/e p. 621

o Wasting (deficit in weight for height) usually signifies acute onset malnutrition.
. Stunting (deficit in height for age) usually signinei a chronic.orrr" of rutrrrariai""
o Wasting and stunting --> Acute on chronic malnutrition.
r Underweight {low weight for age) -+ Combined indicator to reflect both acute and chronic malnutrition.

4. Ans. is 'b' i.e., Height for age {Ref : o.p. Ghai d'/e p. 101 6 Th/e p. 62; park lgh/e p. aja)
r
Stunting (deficit in height for age) generally points towards a chronic course of malnutrition. O.p. Ghai
5. Ans. is 'i i,e., Weight tot age{Ref : hdian academy of
-
3d/e p. 12n
Ttediatui{s
r Weight for height is decreased in both acute as well as chronic malnutrition. However, it cannot
distinguish between
acute and chronic malnutrition.
r Therefore, weight for age is a common indicator to reflect both acute and chronic malnutrition.
However, it is not the
best indicator for this (as weight for age cannot differentiate acute from chronic malnutrition).
r Best indicator for this purpose( to differentiate acute from chronic malnutrition) is the
simuitaneous measurement of
'weight for height (wasting)' and 'height for age' (stunting)'which is
useful to understand the dynamics of malnurrition,
distinguishing between current ( acute) malnutrition and long term (chronic)malnutrition.

5o, for both acute and chronic malnutrition:


i) Single common indicator --+ weight for age
ii) Best indicator is simultaneous measurement for stunting (deficit in height
for age) and wasting (deficit in weight for height)

6. Ans. is 'b' i.e., Moderate fRef: Ghai Th/e p. 641

_l
Normal > BOa/o Normal
t: ,,-J=::,,
II
, 6] 70. Moderate
51 -60 Severe
-'..:..,'tlt.it;
IV < 50 V"ry r"u"r""

7" Ans. is 'c' i.e", Third degree {Ref : O.p. Ghai Be/e p. 97 6 7"/e p- 64}
r Normal expected weight at 2 years Df age is 12 kg.
r This child has a weight of 6.4 kg, i.e.53.30/o of the expected.
e So, this child is having grade III (51-610/o of expected weight) malnutrition.
8. Ans. is 'a' i.e., Based on height retardation fRef.: Park lSth/e p. 463; Op Ghai 8tu/e p. 991
o It is based on weight retardation (not on height retardation,)
9. Ans. is'c'ie.,\Seightfarheight(wastinglandheightf*r*ge(stunting)lRef:A.P,Ghai*thlep.62-64;Nelsan
1*/e p. s9!
o Waterlow's classification is based on stunting (height for age) and wasting (weight for height).
10. Ans. is 'c' i,e,,2fr WlRef: Gbai 7h/e p. 5&|

lnfant 0-6 month


?:ol/kg
6-12 month 1.r,,,,,,lrl,:6.5=/.!g.,,;,..'
Children 1-3 year z29m
4-6year : r:iri: . .30:gm:::,:r'-

7-9 year 41 9m

'l::,:,::rJ O.'l 2ty.Saf,,;


- ',..,=.-1*19.m,,,..
13-15 gm 65-70 gm

ll. Ans. is t'i.e., 1200 K cal/day lRef : PSM Park lgth/ep. 502)

lnfancy
0-6 months 118
7-12 months 108 ] K.Cal/kg/day
,:ehilirr.elL:tit..t..

1 :3,aars:::,: :t2.03 1Z4A 5,t


.1690 7.O
1950 :: : ::
'::,::::: 8.1

t2. Ans. is t'i.e., 1500 calories [Ry'; See above explanationl


o It is 1690 KCal.

PROTEIN ENERGY MALNUTRITION

t3. Ans. is id' i.e., All of above lRef: Ghai 7h/e p. 67|
o Marasmus is due to prolong deficiency of calories and protein. There is excessive catabolism of fat and muscle pro{in.

14. Ans. is '* i.e., Ederna and mental changes lRef : A. P. Ghai *th/e p- 99-]AA & Thle p. 671
r Classical triad of kwashiorkor is markedly retarded growth, psychomotor (mental) changes and edema.
15. Ans. is '{ i,e., Vocarious appetite lRef: O.P. Ghai *e/e p. gg-$0 d, 7h/e p. 6d; IAp p. }251

lncidence More common


lTid:n:" .. . Less common
Uncompensated phase
Active. Apathic
-
.:,;'O'b*-ioUS,1r,:,:f:,.. .i.' Sornetimai $dd*11,.ny,,g6;r.t qnd far.
Severe loss of subcutaneous fat Fat often retained but not firm
Pi€Sent..l. :r.:- ,. , i. :"

Low but may be masked by edema


Vocarious (good) appetite .Poor :
Diffuse pigmentations, flaky paint dermatosis

:Rd..eover.*r , '.
Mortality

16. Ans. is 't i.e., Fatty Infiltration of Pan creas {Re! O. P. Ghai 8th/e p. 99-100 & Vh/e p. 67)
17, Ans. is 'a' i.e., *trepatomegaily lRef: A. p" Ghai 8th/e p. 99 & Vhle p. 6Zl
r Hepatomegaly is seen in kwashiorkor (not in marasmus).

18. Ans. is'c' i.e., Baggy paint appearanee fRef: Gh*i Vh/e p. 6Z)
r Baggy Paint appearamce -- refer to loose skin of buttock hanging down.
t Seen in Marasmus
r Flaky paint appearance :- Refer to hyperpigmentation, depigmentation, desquemation which may be confluent seen
in kwashiorkar.
r Apathy, increase liver enzyme is also seen in kwashiorkar.

19. Ans. is 'd' i.e., Increased albumin lRef: previous explanationsl


r Albumin level is low in kwashiorkor, which results in generalized edema.
r Other options are true regarding kwashiorkor.

20- Ans. is 'a & c'i.e", Hepatornegaly & Low insulin levels {Ref : O.P. Ghai */e p. 99 & 6n/e p. 104, 105, t06l
r Hepatomegaly is not seen in marasmus and insulin level is normal.
21. Ans. is 'a' i.e., Klvashiorkar {Ref Ghai Bth/e p. 99-100; Th/e p. 6\
r Odema with muscle wasting is seen in Kwashiorkor.
22. Ans. is 'H i.e., lgL[Ref: O. P. Ghai 6,h/e p. 1071
In malnourished subiects, secretory IgA is generally reduced, Therefore infections tend to be severe and recovery
delayed. - Ghai 6th/e p. 107
23. Aas. is '* i.e.,200 Kcal {Rel API Medicine 6th/e p. 1145, O.p. Ghai Btu/e p. 103-104 & 7n/e p. 7jl
Energy requirements for -
l. Severe protein energy malnutrition -) 175 - 200 Kcal/kg/day
2. Moderate protein energy malnutrition -) 150 Kcal/kg/day.
t About 8-10o/o of total calories should be obtained from proteins ofhigher biological ualue.

24. Ans. is'd'i.e., 175 CallKg{Ref: O.P. Ghai te/e p. 103-104 6 Vh/e p. 731
?< Ans. is'c' i.e., When fever goes down [Rel Ghai Vh/e p. 68] '\
r Iron at 3 mg/kg per day should be started when child gaining weight once stabilisationlhasejsiyer.
zo. Ans. is 'd' i.e., Ift,vashiorkor {Ref : A.P. Ghai S,h/e p" 100 dr Vh/e p. 6ZJ

27. Ans. is 'f i.e., High blood osmolarity tRef O. p. Ghai Bn/e p. gg-100 {z fl/e p. 671
*d
28" Ans" is i.e", SAM with $ey€re Adema should be hospitalised leef TJpdates on the management a! savere acute
malnutrition infants nnd children, WHA protocol p. 361
c Children who are identified as having severe acute malnutrition shouldfirst be assessed with afull clinical examination to
confirm whether they have medical complications and whether they haye an appetite. Children who have appetite (pass the
appetite test) and are clinically well and alert should be treated as outpatients. Children who haye medical complications,
severe oedema (+++), or poor appetite (fail the oppetite test), or present with one or more Integrated Management of
Childhood Illness (IMCI) danger signs should, be treated as inpatients.
t Children with severe acute malnutrition who are admitted to hospital can be transferred to outpatient care when their medical
complications, including oedema, are resolving and they have a good appetite, and are clinically well and alert. The decision
to transfer children from inpatient to outpatient care should be determined by their clinical condition and not on the basis
of specific anthropometric outcomes such as a specific mid-upper arm circumference or weight-for-height/length.

BREAST FEEDING & BREAST MILK

29. Ans. is'b'i.e., 6 months fRef. O. P. Ghai 8th/e p. 150 b Vh/e p. 60)
c The WHO recommends exclusive breast feeding for the first six months of life and then breast feeding up to two
years or more.
30. Ans. is 'd i.e., 2 hours lRef, O, P, Ghai 9'h/e p. 150)
o Breast feeding should be initiated within i0 min. of a normal vaginal delivery.
o Breast feeding should be initiated within 4 hrs of delivery by caesarian section.

31. Ans. is 'a' i.e., Exclusive breast Feeding should be continued till 6 month of age followed by supplementation
with additional foods lRel O.P. Ghai 9th/e p. $a & Vh/e p, 601
o The baby should be given exclusive breast feeding not even water for first six months of life.
o Breast feeding should be given, whenever baby feels hungry (DEMAND FEEDING).
r Complementary foods (other foods in addition to breast milk) should be started after six months of age.
o Breast feeding should be continued with semisolid and solid foods upto two years of age and beyond.

32. Ans. is'd'i.e., IgA antibodies [Rel Nelson 18th/e p. 215; O,P. Ghai 8th/e p" 151 6 6th/e p. 971
Although antibodies and Lysozyme, both are important for protective effects of breast milk, antibodies play the
major role. Out of IgM and IgA antibodies, breast milk is especially rich in secretory IgA which is the single best
answer here.
33. Ans. is 'a' i.e. Docosahexaenoeis acid lRef: C.P.D.T. 18th/e p. 286; Park 20th/e p. 4631
o Up to 5-l0o/o of fatty acids in human milk are poly'unsaturated. Most of these are linoleic acid wrth smaller amounts
of linolenic acid.
o Linoleic acid -+ gives rise to Arachdonic acid
o Linolenic acid -+ gives rise to Docosahexaenoeic acid
o Docosahexaenoeic acid is found in human milk and brain lipids and is required for the development of our nervous
system and visual abilitie.s during the first six months of life.
o Lack of sufEcient Docosahexaenoeic acid may be associated wlth impaired mental and visual functioning as well as
attention defi cit hlperactivity disorder.

34. Ans. is'a' i.e., Vitamin B lRef: A. P. Ghai 6th/e p. 149, 158; Nelson 18th/e p. 215; Cloherty 4th/e p. 1131
o Deficiency of VitaminBnrtaf occur in exclusive breastfed infants of mother who is on strictvegetarian diet.
Remember following important facts -
o Milks from the mother whose diet is sufficient and properly balanced will supply all the necessary nutrients except

o The iron content of human milk is low but most normal term infants have sufficient iron stores for the first 4-6
months. Human milk iron is well absorbed. Nonetheless, by 6 months the breast-fed infant's diet should be
supplemented with iron fortifled complementary foods.
o The Vitamin K content of human milk is low and may cause hemorrhigic disease qf newborn.

35. Ans. is'None' explanationl\


\.. ____,..'
fRef: See above
o Exclusive breasfed infants may have folloWing deficiencies -
i) Vit Brz (if mother is strict vegetarian) iii) Fluoride
ii) vitD iv) VitK
r For one option, you can go ahead with option 'a', i.e. scurvy -
"Breastfed infants are protected as the breastmilk contains adequate amounts of vitamin C, except when the
mother is deficient in Vitamin C". O. P. Ghai 127
-
So, if mother has deficiency of vitamin C, exclusive breastfed infant may develop Vit C deficiency.
36. Ans. is h'i.e., faundice {Ref. Ghai 6e/e p. 97, 121)
o "tr3o/o of breast fed infants develop unconjugated hyperlbilirubinemia in the Ist week of life. The breast
fed infants
have higher, bilirubin levels than formula fed infants."
r It is due to inhibitory substances (Pregananediol and free fatty acids) in breast milk that interfere with bilirubin
conjugation.

37. Ans. is'-o" i.e., Evening colic [Ref Nekon 18h/e p. 215)
c Breast feeding protects against evening colic.
c Evening colic may be seen as a manifestation of allergy to cow's milk, but not with - Nelson 18th/e 215
breast milk.
o Haemorrhage due to vit K ileficiency may be seen. Breast milk contains very little Vit K - Dutta 4th/e p. 515
Hypoprothrombinemia, may therefore occur along with defeciency of other vit K dependent coagulation factors.
(VII, IX, X). this predisposes to haemorrhagic disease in new borns.
r There is strong association between exclusive breast feeding and neonatal jaundice. It is presumed to be due to
inhibitory substance in the breast milk, that intefere with bilirubin conjugation e.g. pregananediol and free fatty acids.
t Golden colour stool may be seen.
38. Ans, is *,* i,e,, Active untreat€d auberculosi$ {Ref: ctoherty - manual of Newatot-ogyJ
Contraindication of Breast feeding
1) Galactosemia
2) Active untreated tuberculosis - only in initial period
3) HIV positive mother - especially in developed country.
4) Some medication
39. Ans. is 'd i.e., Cow's milk contaia 80% whey protein not casein fRef: Nutrition and child develolrment, Fliz
beth F/e p. 18; Park 2ff/e p. $l
r Whey protein constitutes 80% of the protein in human milk, while the main protein in cow's milk is casein.
Comparison of human milk and cow's milk

Bacterial contamination

Protein 47o (too much)


Total 3%'(too much)
Casein

'40/o' .'::
Saturation oJ fatty acids Too much saturated
Linoleic acid (essential) Enough for qoryi"g Uraifr$ Notenough. , '

Cholesterol l
Notenough '":

Minerals {mg)
Calcium 350 (coirect amount) 1;400 (too fiuch)
Phosphate 150{correct amount) 900 (too inuch) :
tf,iiil': .:-::":,., 5in6X,,irrrounr b,ut well-absortred Smali aryouht pborly' absorbed
(enough) (not enough)

Vitamin tnouqh Extra needed

About option'C'
o Cow's milk has more K* and Na* than infant formula' CPDT 18th/e 302
-
10. Ans. is 'a' i.e., 7.2Vo lReJ. O. P. Ghai 6tt'le p. 97, 147, 149; Park 2$'t'le p. 46i, 5451
o It is 7.0 o/o.
41. Ans. is 'a' i.e., Lactose lRef. Park 20th/e p. 4631

12. Ans. is t' i.e., Fat [Rel O. P. Ghai 9n/e p. 15j 6 Vh/e p. 1251
Hind milk
r Comes towards the end of feed
t infat content and provides more energy, and satisfies the baby's hunger.
Rich
-13. Ans. is U'i.e., 24 hrs [R{: O. P. Ghai }'h/e p. 155 d" Thle p. }27)
Breast milk
o Canbestoredatroomtemperature -+ ForS-l0hours
e In a refrigerator -+ For 24hours
c In a freezer :) -200' for 3 months

44. Ans. is 'H i.e., I hrs [R{: O.P" Ghai }tt'le p. }55 d'Vhle p. 12fi

RICKETS

45. Ans. is'bn i"e., Defect in mineralization lRsf A' P" Ghai 7h/e p. 82i
r Inricketsosteoid(proteinmatrix)isformednormallybutmineralizationofthisosteoiddoesnotoccuri'e.thereis
defective mineralization.
o Rickets is a syndrome of diverse etiology characterizedby defective mineralization of bone and epiphyseal cartilage
(growth plate) of growing bones.

46. Ans. is 'a' i.e., Rickets lRef : O.P. Ghai 9'h/e p. 113 (t 7h/e p. 821

47" Ans. is'b'i.e., Hyphosphaternic rickets [Ref 0. P. Ghai 8'hle p. 11j, ]15 & 7h/e p.83; Nelsor 17'/e p. 2342]

o Clues in this question are -


1. Normal calcium 3. Low phosphate
2. Normal paratharmone 4. Elevated alkaline phosphatase

o All these suggest the diagnosis of hlpophosphatemic rickets.


r D/D of Rickets -

Normal 9-10.5 mg/dl 3-a-5mg/dl 30-120 IU 1O-55 units 21-30 rneqil

tAilSQpfresphii*ifgi;fq1lv''..:']::r:
1::r.Ni":,-'l-i','"' ]' '1:- ; f. .:; ,:':' N, :
..::, N: :ir

Vit D dependent Rickets iN/J


i(7.s-8)
N/..1 t t 1

\+
\i vI r'' ''.r+. :l
,:r,. t,.i: ',,
N
r' 'i'\ r.

Nutrit!onal rickets
Nutritional rickets -
c\r
J J
I
I t N

::d;tritbniai:ta {dtLq11t','r,.,.,':,,,,f.:,-:':"':'
: ::. j.:l::'1,.:l:':: ,.r:: . t., .,t,
-,:1

48. Ans. is 'b, i.e., Hypophosphatemic rickets lRel O.P. Ghai 8'h/e p. 113, 115 dt Vh/e p. 831

r In the patient :-
r Serum Ca2* -) Normal (Normal value -> 9-10 mg/dl)
r Serum POr2* -+ Decrease (N -+ 3-4.5 mgidl)
r Alkaline phosphatase -) increase (N + 30-120 IU or 5-15 KA
r Serum parathormone + Normal (X _+ to_ss units)
)
r Serum bicarbonate leyel _+ Normal (N _+ ZZ_ZS units)
o it is clear that it is a case of Hypophospheternic rickets.
So
c Proximal Renal tubular acidosis ulto .u,.r..,
hlpophosphetemic Rickets but the bicarbonate
in a case of proximal renal iubular acidosis. level will be reduced
r Distal renal tubular acidosis causes vit D. dependent rickets
so it is easily ruled out.
49. Ans. is t'i.e., Hyperparathyroidism
[Ref: Nelson lVh/e p. 2345]
'ff:lr#];.ji:tJi;ttt"" just looking at calciurn value. Amongst
the given options onry hyperparathyroidism

e The patient in question has an increased serum


ca**, decreased serum phosphorus and increased
phosphatase, all of which characterize hyperparathyroidism. values of alkaline

50. Ans. is 'a' i.e., Craniotabes, t, i.e., Widening of wrist, .d, i.e., JpOA, ,e, i.e.,
Ghai 9th/e p' 113 & vh/e
Growth retardation [Ref : O, p,
p' 82; welson lvh/e p. 186; Forfar and Arniel's peiatrics
r craniotabes, widening of wrist and growth retardation
6th/e p, d2 sstl
occur in rickeis.
o Serum phosphate is low
5t. Ans. is 'd' i.e., Bow legs laey o. p. Ghai Bth/e p. 113 & vh/e p. 82)
' otT:.T:;::i'ff.t;:::"formed when the chil{ starts bearing weight. rherefore
deformities of tegs are unusuat

q,)
Ans. is 'b'i.e., Gustock deformity {Ael O. p. Ghai gth/e
p. it3 6 Th/e p. 821
c Gunstock deformity is seen in malunited supraconrlylar
fracture of humerus.
Note -
r Dont get confuse between option'a'and answer.ofprevious
question. In this question the age of the chilcl
been mentioned. Bow legs can occur, once has not
the child starts ;"'il;;
53. Ans. is 'b'i.e., Rickets [ne! O.e Ghai 7/e p. 82; Maheshwari
3d/e p. 275)
r Genu valgum (also known as knock knee) is
a condition where knees are abnormally
abnormaliy divergent. approximated and ankles are
c It is caused due to softening of bones or damage
to laterar Femorar epiphysis
Out of given options -
o Most common cause in children is Rickets.
o Other diseases given in option are seen in eiderly.

54. Ans. is'b'i,e., Rickets lRef: Internetl


Windswept deformity
r A valgus deformity of one in association of varus
deformity of other ftnee is known as windswept deformity.
Causes of windswept deformity
o Rickets r physeal osteochondromatosis
o Hereditary dysplasia (epiphyseal dysplasia)
ofbone.
55. Ans. is 'a'i.e., vit D deficiency &'b, i.e., cRF
IR4 Netsqru 18th/e ch.48 (Table a)l
r vitamin D deficiency and cRF (renalosteodystrophy) cause calcium deficient (tlpe-r) rickets.
e Primary hypophosphatemic rickets (x-linkei) p*.orri .frrd.o*.
urrd cause hlpophosphatemic rickets.
56. Ans. is t'i.e., Anterior fontanel widened & d, i.e.,
ted line phosphatase [Ref : O. p. Ghai
7h/e p. 821 Sth/e p. il3 d,

5/. Ans. is 'a'i.e., Craniotabes [Rel p Ghai gth/e p. 113 ("


O. 7h/e p.\g2f_,
Craniotobes is the earliest manifestation of Rickets.
58. Ans. is ld' i.e., Thalassemia lRef : Nelson lgth/e p. 2531
Craniotabes is seen in *) r Rickets o Hldrocephalus r eslsogenesis imperfecta o Syphilis
59. Ans. is 'a' i.e., Rickets {Ref: O. p. Ghai 8*/e p. 113 6 Th/e p. 82)
60. Ans. is'd' i.e., Syphilis fRef: www.fpnotebook. coml
Rachitic (Rickety) rosary
o The prominent knobs of bone at costochondral junctions of rickets patient is known as rachitic rosary or beading
of ribs.
o The knobs create the appearance oflarge beads under the skin ofthe rib cage, hence the narne by analogy with the
beads of a rosary.
Differential diagnosis ofenlargement ofcostochondral junction (Rosary) -+ Rickets, Scurvy, Chondrodystrophy

61. Ans, is 'd' i.e., All of the above lRef : See above explanationf
62. Ans. is t' i.e., X-ray wrist lRef: Ghai 7/e p. 83)
r In nutritional rickets either single dose Vit. D. 6 iakh l/m or 60,000 IV daily for 10 days.
r It shows evidence of radiological healing within 4 weeks of therapy.
r Reduction in serum alkaline phosphatase and resolution of clinical sign occur slowly.
o Ifno healing - repeat dose.
o If no healing can be demonstrated with 2 mega dose of Vitamin-D, then labelled as refractory rickets.

SCURVY

63. Ans. is 'b' i.e., Decreased osteoid matrix fRef: O.P. Ghai 8th/e p. 120 6 Vh/e p. 82-911
o Coliagen is a component of osteoid (protein matrix).
o Collagen synthesis is deficient in scur\T; therefore, osteoid formation is defective.
e On the other hand, in rickets osteoid formation is normal, but mineralization (Calcification) of osteoid is defective.

64. Ans. is 'd' i.e., Scurvy lRef : O. P. Ghai 8th/e p. 120 6 Vh/e p. 911

o Exclusive feeding on Cow's milk in a 6 months infant with anemia, bleeding, fever and generalized tenderness suggest
the diagnosis of scurr,y.
r Cow's milk does not contain sufficient amount of Vitamin C. On the other hand breast milk contains suffrcient
Vitamin C.
65. Ans. is t'i.e., Vit C deficiency lRef : O. P. Ghai 8th/e p. 120 6 Vh/e p. 91]
Pseudoparalysis
o A voluntary restriction of motion because of pain, incordination or other cause, but not due to actual muscular
paralysis.
Causes of pseudoparalysis
o Scurry (vitamin C def,ciency) o Osteomyelitis
o Septic (arthritis) o Congenital syphilis

66. Ans. is 'b' i.e., Scurvy [Ref : See above explanation)

67. Ans. is 'c' i.e., Bufly coat estimation l&ef : Nelson lse/e p. 252)
o Leukocyte concentration of vitamin 'C' is a better indicator of body store, it may be deficient even in the absence of
clinical signs of defi ciency.
. It is estimatedby buffy coat prqvaal
68. Ans. is 'l i.e., Yitamin C [Ref: Nelson 18*/e p. 252)
o The history is suggestive of bleeding disorder and the vitamin most likely associated is Vit. C and disease is scurry.
The other hint in questions is that the boy doesrt't eat fruits. Fruits are rich sources of Vitamin C.
o Vitamin C deficiency causes scur\y.

VITAMIN A

69. Ans. is 'b' i.e., Yitamind. {Ref : O. P. Ghai {/e p. uq & 7/e p. 79; Park 1*/e p. 48sl
o Vitamin A is necessary for integrity of epithbiQ!$ides that resist invasion by pathogens.
o Vitamin A has some role in immune response.
70. Ans. is t' i.e., Anterior segment of eye is initially involved lRef : O. P. Ghai 8'h/e p. 111 6 Vh/e p. 791
o Night-biindness is the earliest symptom, which is due to inability to regenerate rhodopsin. Thvl pasterior segment is
involvecl initialiy (rhodopsin is present in ret,inaand retina
is a part of posterior segment of eye).
o Infections, poor growth and rarery hydrocephrus
occur in vitamin A deficiency.
71. Ans' is 'a' i'e" Bitot spot; 'H i.e., Xerophthalmia & 'c' i.e.,
Night blindness {Ref : o, p. Ghai gth/e p. 1 I I 6 Vh /e
p.7e1
Xerophthalmia (Dry eye)
o It is a syndrome due to def,ciency of vitamin A.
o it has foilowing stages :
l. Night blindness : Eailiest acular symptom 4. Corneql xerasis
2. Caniuctival xerasis : Eailiest ocular sign 5. Keratomalacia & corneal ulcers
3. Bitot's spot
72. Ans. is t'i.e., 300 microgram lRef O. p Ghai Bth/e p. 111 6 Vh/e p. Z9)
Daily dose of vitamin in
o Infants 300-400 microgram
o Children 400-600 microgram
o Adolescen[s 750 microgram

73. Ans' is 'H i'e" I Lakh IUlmt fRef. IAP ln/e p. 417; Nutrition & child Development KE Elezabeth
3d/e p. 941
o According to vitamin A prophyiaxis programme
children between 9 rnonths and 5 years are given 9
megadoses of
vitamin a concentrate at 6 month interval.
o The first tlvo doses are integrated with measles vaccination
and DpT Ist Booster.
o For infant the dose is I ml equivalent to I lakh IU and
in chjldren it is 2 ml i.e. 2lakh IU.
74. Ans. is 'd'i.e., 2 lakh I.IJ. [Re! Ghai Tt/e p.SA]
o For treatment of vitamin A deficiency, oral vitamin
A is given at a dose of 50000 IU, 100000 lu and 200000 IU in
children <6 months, 6-12 months and > 1 year, respectivell'.
The same dose is repeated next day and 4 weeks later.

MISCETLANEOUS

75. Ans. is t'i.e., Oliguria lRef O.p. Ghai Bth/e p. & p.


113 Zh/e 81; Nelson l1th/e p. 2621
o There is pol1'uria (not oligtiria)
76. Ans' is 'a'i.e., vit A; 'b'i.e., vit D IRel O. P. Ghai Yth/e p. 111, 113 6 7h/e p. 80; Nelson 18th/e p. 2451
o In chronic vitamin-A intoxication there are bone abnormalities
ancl bony swelling.
o In hypervitarninosis D, there is excessive bone resorption.
77. Ans' is 'a' i'e', Benign cranial hypertension &'e'i.e", Liver
darnage {Ref: Nelsatt lgil,/e ch. 45 (table-1)}
o vitamin A intoxication can cause benign intracranial
hypertension and hepatomegaly. It is not a cause of craniotabes.
78. Ans. is t'i.e., Zinc deficiency fRef: Nelson 18,h/e p. 266; O. p.
Ghai gth/e p. 122 6 Vh/e p. 921
e The patient in question ltresents with hyperpigmentation,
hypogonadism, dermatitis and alopecia, all of which
may be seen with Zinc deficiency.
79. Ans. is All tR$ Nelson 18th/e p. 266; O. p. Ghai 8th/e p. 122 6 Th/e p. 92)
o Hlpogonadism (sexual infantilism, loss of libido), p.oor
weight gain (growth retardation) and poor wound healing
are
seen in zinc deficiency.

80. Ans. is '* i.e., Acroderrnatitis enter*pathica


{Re;!: Netsan rg,h/e ch. 6z$, 46.21
81" Ans" is 'a'i.e., Keshan disease {Re_f: Netsan lVh/e Chap. 5 j; Ap Ghai gt,/e p. 122}
r Keshan disease -+ Disorder of Se metabolism
r Wilson'.s disease -+ Disorder of Cu metabolism
o Acrodermatitis enteropathica -+-Disorder of Zn metabolism
82. Ans. is'd'i.e., Z0 llrg/10-14 days
According to WHO guidlines
\.,
\i
-\
\l
t-..
'--/
./
Dose of Zinc
2 month - 6 month = 10 mg I day x 14 days
> 6 months = 20 mgl day x 14 days

i.e.,20 mg of iron and 100 microgram folic acid Nutrition & chitd development by K.E. Elizabeth
83. Ans. is 'l fRef:
3'd\ep.111,1121
o National nutritional anemia prophylaxis programme is based on daily administration of Iron & folic acid tablets to
maintain rural child health.
r For anemia prophylaxis FOLIFER tablets with 20 mg elemental iron and 100 microgram folic acid are given to children
for 3 months.
o For treatment upto 6 mg/kg elemental iron should be given for 3 months.

84" Ans. is 'd i.e.,I-2 rmEqItr{g $te}: Nets*n t9'h/e ch. 52.7}

o Recommended dietary requirement is 1-2 mEq/kg

ANSWERS OF VARIOUS OTHER EXAMINATIONS

g5. i.e., Early supplementation of solids in infants & t' i.e., Immunisation to the chil dlRef : O.P-
Ans. is
.b, Ghai
8'hle P. 108-109 dr 7h/e P. 76-771
PEM can be prevented in children by -
i) Exclusive breast feeding for first 6 months.
ii) Supplementary food after the age of 6 months'
iii) Sup;lementary food should be-complementary, so that the deficiency of a limiting amino acid is compensated bv'
a amino acid in other food.
iv) Immunization of vaccine preventable diseases'
v) Restriction of feeding in fever & diarrhea should be discouraged. \
vi) Adequate time between two pregnancies.
\)
g6. Ans. is'b'i.e., condition seen in the displaced child lRef : www.popline.orgl
r The name kwashiorkof comes from the Ga language of Accra, Ghana, and is used to describe the disease of the
displaced baby when the next one is born.

87. Ans. is t' i.e., Edema fRef: A.P. Ghai 6th/e p.I01l
gg. Ans. is t' i.e., The child loses edema and starts gaining weight [Rel: OP Ghai 9'h/e p. ru7 d" 7h/e p. 751

Criteria for discharge in PEM


i) The appetite should have retured and oral intake is adequate'
ii) Gain in body weight with loss edema.
iii) All infections, mineral and vitamin deficiencies should have been treated.
iv) Mother should have been educated regarding domiciliary care.

89. Ans. is 1f i.e., Pyridoxine fRef: Ghai 6tule p. 105, 12j, 1251

90. Ans. is'a'i.e., Plasmodium vivax [Ref O.P. Ghai 6'h/e p' 9fl
91. Ans. is 'b' i.e., Lactalbumin & t'i.e., Nucleotides lRef A.P. Ghai 8'h/e p. 151 & 6'h/e p. 147-148; SPM Park
17*le P. 373 dr 2ffh/e P. 462, 4637

92. Ans. is'H i.e., Serum iron estimation [Ref: O.P. Ghai 6h/e p. 3A] dr Thle p. 3aAl
o By the age of 6 months the breast-fed infant's diet should be supplemented with iron fortified complementary foods
because iron in breast milk is sufficient upto the age of 6 months'
o The baby in question has been exclusively breast fed upto I year, so he has developed iron deficiency anemia.

93. Ans. isNone [Rel: O.P. GhaiTh/e p. 30Al


o Correct answer is 6 months.

94. Ans. is t' i.e., 5-6 months lRef : Park 19h/e p. 4221
o Breast milk is maximum at 5-6 months of lactation --> 730 ml/day'
e It is minimum at 37-38 rnpnths of lactation ) 345 mllday'

)
\ _--,
95. Ans. is'b'i.e., CMV
[Ref Nelson lgth/e p. 215]
o Breast milk transmits (with
definitive_evidence) : (l)
o Breast milk can rarery transmit HIV (2) CMV
: (1) HTLV - tFpe 1 (2) Rubelra virus (3) HBV (4) HSV
96, Ans.is 'i i,e,,Lactose [Ref O.p, Ghai
r premature mirk
Vh/e p. t2S d" 6th/e
15g] p,
contains ress ractose in comparison
to term miik.
97. Ans is t' i.e., 7 gm [Ref Ghai 6th/e p.
97, 142, 149; park 20th/e p. 463, S4S]
98. Ans. is t'i.e., Saddle nose
[Ref Nelson t4h/e p. 186; Op Ghai
o Saddle nose
gtu/e p. 113)
is not seen in rickets_

99' Ans' is 'd i'e"Ringed epiphysis


{Ref o.p. Ghai *tu/e p. 120 6 7h/e p. 911
' :;:::rl,r'i:i?::;{'*'"rger sign):It is white ring surrounding the #;r.:, centres of ossicification.
rt is relatively
' H}t::lmlft*Tnrt" (Freaenkel) can be seen in scur\y, hearing rickets, plumbism, severe pEM,
congenitar slphilis
r Option b & d are not specific for
scurly.
100. Ans' is 'b'i'e" 1,00,000 ru
[Ref: o.p. Ghai gth/e p. 112 6 vh/e p. g0;KE Elezabeth
3th/e p. g4]
l0t. Ans. is 'd'i.e., Essential fatty acid
[Ref O.p Ghai 6h/e p. 121)
r In vitamin A deficiency there is toud iik
skin arso known as phrynoderma.
r But this is due to associated a.n.i.".foi".r.rrtiut fatty acids.
102. Ans. is 't
i.e., A fRef: O.p Ghai 6th/e p. 121
6 7tu/e p. 79]
o Here, essential fatty-acid
has not been provided as an option.
e So, amongst the given options
vitamin K is ihe best answer.
r03. Ans. is t'i.e., Zinc [Ref O.p. Ghai 8th/e p. 122 6 p.
Vh/e 92]
104. Ans. is t' i.e., Zinc [Ref Harrison lVh/e
p. 443; Op Ghai gth/e p. 122)
105. Ans. is 'd'i.e., Keshan disease
lRef : O.p. Gha_i gtu/e p. 122 6 Vh/e p. 93;
r Keshan disease is a disorder of selenium
Harrisons lVh/e p. 4491
metabolism.
o Following diseases are related
to copper metabolism :_
i)Wilson's disease
ii) Menkes kinkyhair syndrome
iii)
indian childhood cirrhosis (increased
hepatic, urinary and serum copper
concentration are characteristic
ofICC).
f 06. Ans. is'H i.e., 1333IU
[Ref O.p. Ghai 6th/e p. 120)
107. Ans. is '* i.e,,Scurvy
tRef O.p, Ghai gth/e p. 1201
r It is a case ofpseudoparalysis.

r08. Ans" is'a'i.e., Scurry


IRd O.p, Ghai gth/e p. 120 & Vh/e p. 911
109. Ans. is .b, i.e., 1250 kcal
fRef: O.p. Ghai Vh/e p. zz O On/e p. SO]

rrr
Q',, t[ A P':T'r.,,,E:r':'rR

NEWffi#Rh$ TNFANY

NORMAL NEWBORN

o From birth to under four weeks of age (< 28 days), the infant is called newborn (neonate) and that petiod.is called
neonatal period@EBr).

o Neonatal period is divided into :-


i) Early neonatal period (first'week of life, i.e. < 7 days)
ii) Late neonatal period (7h day to < 28 days).
t Term neonate (term baby)
r Any neonate born between3T and < 42 weeks (259 - 293 days) of pregnancy irrespective of the birth weight'
t Preterm baby
o Any neonate born before 37 weeks (<259 days) ofpregnancy irrespective ofthe birth weight.
t Post-termbaby
o Any neonate born at or after 42 weeks (294 days or more) of pregnancy irrespective of the birth weight"

r Features of normal'terni newborn infant are :-


t Length -+ 50 cmalt3)
r Head circumference -) 34 cm
r Chest circumference + 31 cm (3 cm less than head circumference)
r Upper segment lowet'segment ratio -) 1.7 to 1.9

t Heart rate + 120 - 140 per minute(Iipnere3)


r Respiratory rate -+a35 - 40 per minute
t ,4ttitue -) Flexion(Ar13)
t Peripheral cyanosis (acrocyanosis) may be presen{Al 13' Iipmer e3)

r Urine is passed by 24 hours after birth


r Meconium (first stool) is passed within 24hoars6"'). After that meconium stools (black tarry) can be passed
upto 3 days(/rrs). On 4th-5th days transitional stools (greenish) are passed. After 5 days regular milk stools (golden
yellow) are passed.
r The skull shows moulding with parietal bones slightly overriding the occipital and frontal bones.
r There maybe pulmonary Jlow (systolic) TnvvTnvv1itnere3), especially in low birth weight neonates.
s Liver, spleen andkidney may be palpable{x,-.sst
o There are some minor clinical problems which can occur in normal newborn
and require no treatment (NEET,DNB 12,
AI 05, pGI 08,02, 98)
.

o The problems are -

1. Milia@cro& e8)

r White dots on nose and face due to distended sebaceous glands. These diseappear spontaneously.
2. Erytherna toxi6sln(rcroa' o2' e5)
r Erythematous papules on trunk & face. They appear on 2"d & 3'd day and disappear
spontaneously.

Mongolian sPots(ecr oa ra)

r There ate 'bluish spots' most commonly


found in pre-sacral area@crh8) (mainly in lower back & butto*),bfi
may also be found over pastefior thighs{eeroat,legs@.orot), and shoulders.
r They disappear spontaneously before first birthday.

Stork bites
r Pinkish gray capillary hemangiomas on the nape of neck, upper eyelids, forehead
& nose. These disappear
spontaneously.
5. Peeling of skin
r More frequent in post-teim infonts, but can also occur in term infants.
6. Subconjuctival hemorrhages @Gr 02' s8)

r Disappear spontaneously.
7. Breast engorgment
r Due to transplacentally acquired maternal hormones. It is seen on 3d or 4s day.
r*-'
EPstein pearl@Gro8'02)
r Epithelial inclusion cysts, which appear as whitish spots. These are of two t,?es -
i) Palatal + On hard p{rte or on either side of median raphe.
ii) Prepucial --> At the rip of prepuce at 6'O'clock position.
Cnl:P.r:E,(

!{hat are these white $pots in a term newborn:


a) Lipoma
b) Vesicles
c) Epstein pearls
d) Stroke bite

Ans. is t'i.e., Epstein pearls


. Whitish ipotr on hard palate on either side of median raphe suggest the diagnosis o[Epstein pearl.

9. Pre-deciduous (natal) teeth


r Supernumerary teeth in the lower incisor position. These are shed before the primary dentition.
rz ar osl
10. Vaginal bleedilg(owr
r On 3'd to 7'h day. it is due to withdrawl of maternal hormones after birth.
11. Vaginal mucoid discharge
r Because oftransplacentally acquired estrogen on vaginal mucosa.

12. Hymenal tags(rvrrrt


r Around margins of hymen
13 Physiological phymosis

Skin changes in newborn that disapper spontaneously :MiliafctoB), erythema tox'cum(P6r08), mongolian spots(P6i '8'), stroke
bite, pustular melanosis, peeling of skin, and Harlequin color change(Pci "r.
o A new born loses upto 1 0o/o of his body weight in first few days and regiins his birth weight by age of 10 days.
a " The number, colour and consistency of stools may vary greatly in the same infant and between infants of similar age. The
color of the stoot has little signifrcance except for the presence of blood or absence of bilirubin products;'
Larynx of a neonate is at higher position (thon adults) which allows infdnt to use nasal airway to breath while suck-
ingqttMslO,

PRIMITIVE REFLEXES
o A number of primitive neonatal reflexes can be elicited in healthy term neonate.
o These disappear as the child grows -+ These reflexes are inhibited by frontal lobes as child grows.
o Absence of reflex responses indicates d),sfunction of central or peripheral motor function.
pcl D).
c Abnormal persistence of neoruatal reflexes is pathognomonic of central motor lesions@Pc 0e'

i) Rooting, sucking'and swallowing reflexes i) Symmetric tonic neck reflexutM'NtMs07)

ii) fiossed.extensior \gfleltiYsff' 4!on ;'' : ii)'Parachutereflex , " :


"
iii) M or o\ r efl elAt t tils 07' A1 07 )
iii) Landau reflex.

ivl Falmar.qraip {grasp reflex) ." ," : ,'

AtMsaT)
v) Assymetrictonic neck reflelAtaT

:ii?: ''&qtifappiiii€nee't Ag-g' ol,$appe-qra6ee{:iftef:birth) :

, Less prominent after 1 32 wks of gestation


:fotl6.vg. ' :: :: 2&32 wks of gestation 3-G fl'onthsttlfis04.
.,'0
a Palmar grasp 28 wks of gestation 2-3 months
,Aisynm*lctoaie-neik 35 wks of gestation 6:7 monthstP6'e't.
o Symmetric tonic neck ,
4-6 months 8-12 months

* ,.LaRdau , .-il 10 rnonths 24 months


. PArAChUteUtt3) 819 mgnths rem a in throug hout I ifeat t 3)
r.:iic!.o5irE .e)itenjOi;,l :: .:28r$r.k afrge-jt tio.ri 1-2 months

o Earliest reflex to disappear -+ Rooting reflex.


t Swallowing reflex is earliest to appeor _+ I 2_ I
3 weeks of gestationtAl 1,

I Moro',sreflexdevelopsby28thrveekofgestationanddisappearsat6mottthsofags{dnus'7,e5Ar07),andneverrea\rytssr.r(1'ct
e8)
' Persistence of Moro's reJlex beyond 6 manths is ahnormararrMs 07)
.

t Unilateral Moro's reJlex is seen in :


i) Erb\ palsyrtersz) (Cr-Cu lesiofl(4llrlsee))
ii) Spastic hemiplegia
lii) Shoulder dislocation@ct s7)
iv) Fracture proximal hwmerus or clayicle@cr e?).

t Exaggerated Moro',s reJlex or persistence of Moro's


reflex is seen in : brqin (cerebral) d.amage@tc as, ecroz)
"

The following figureis showing which reflex:

: a) Parachute reflex.
b) Sucking,re{lex
c) Tonicneikreflex
d) Moro's,reflex

o This'is a figure of Moro's reflex' sudden dropping of


head in relatisnito trunk causes'opening of
abduction and extension of arm; forlorved by-fleion -' hands.with
of apper . extremities.
Morgan's Neonatal Neurobehavioral assessrnent scale
r This consists of three parts : (i) Tone and motor pattern, (ii) primitive
reflexes, and (iii) Behavioral responses.

Stage I : Deep sleep, eye closed, no eye movement, regular


breathing
Stage 2: Light sleep, eye closed, rapid eye movement.
Stage 3: Drowsy or semidrowsy, eye opening and closing (eyclids fluflering).
Stage 4:Awake and alerl, eye op"r, ,,ini*ni*"r", ,.,,Jr*i;;;;";;;. "'
Stage 5: Wi<ie awake, considerable (vigrous) movement.
Stage 6 : Cryingletws tot
intense and lortd, High.motor acti\_iry

HYPOTHERMIA

r A newborn is more prone to deveiop hlpothermia


because of large surface area per unit ofbody
weight. The protective
mechanism for heat generation (thermogenesis)
in response to cold environment are :-
i) Peripheral vasoconstrictionullMs 11,06).
ii) 'Non-shiveringthermogenesis'inbrownfatbyadrenaline^rrMsit,06).Brownfatislocatedaroundo
8e), drenalglan4{eor
kidney, nape of neck(pcr ss),
and. intersca?turar and axi,ary regions.
iii) Increased heart rate to meet more O, demand.
o Newborn daes not haue shivering mechanism
for thermogenesisGr^s 11, Li6,Ar es).

o Clinicai features of hypothermia in a neonate are :_


1) Peripheral vasaconstriction : Acrocyanosis, cooi extremities, decreased peripherai perfusi.n.
ii) CNS depression : Lethargy(Pct 0, , bradycardia@Gr 06, Ar es)
, flpnea{pcr 06, a2), poo, f..airrg.
iii) Increased metabolism : Hypoglycemia@Gt 06' 02)
, hypoxiq@"' out
, metabolic acidosis(At ee)
.

iv) Incre pulrnonary artery pressure : thchlpnea, respiratory distress.


ased
v) Other:Depressed immunity, increased chances of septicemia, sclerornaule'qr, DIC (due to infection), hemorrhagic
manifestation.

LOW BIRTH WEIGHT AND PREMATURITY

o Birthweightisthesinglemastimportantdeterminantafchancesofsurvival,growthanddevelopmentafaninfant{tt
'3). Birth weight should ideally be measured within lst haur of birth. Average birth weight in lndia is 2.7- 2.9 kg (2.8
kg){Nntrt.
s Most common causes of low birth weight (LBW) in India is maternal malnutrition. Maternal nutrition has linear relation
with birth weight.
o Following terms are reiated to birth weight and maturity of a neonate at birth :-
1) Lowbirthweight
r Anyneonatelveighing/essthan2500gm(2.5kg1r*uu':'GMCETq7']'G103)atbirthirrespectiveofgestationalage(WHO
definition). But some lndian scientists consider LBW as < 2ftyleouctrozl.
2) Very low birth weight
r Any neonate weighing less than 1500 gm (1.5 kg) at birth irrespective ofgestational age.
3) Extremely low birth weight
rAny neonate weighing less than 1000 gm (1 kgy*uu't at birth irrespective of gestational age.
4) Appropriate for gestational age (appropriate for date)
r Neonate with birth weight between 10th to 90th percentile.
5) Small for gestational age (small for date)
r Neonate with birth weight less than 10th percentile(Pct00). These are the baby cailed as intrauterine growth
retardation (IUGR).
6) Largefor gestational age (largefor date)
r Neonate with birth weight more than 90th percentile.
7) Tertn neonate (termbaby)
r Neonate born between 37 and < 42 weeks (259-293 days), irrespective of birth weight.
8) Preterm baby
r Neonate born before 37 weeksql 01) (< 259 days), irrespective of birth weight.
9) Post-termbaby
r Neonate born at or after 42 weeks (> 294 days), irrespective of birth weight.
Causes of intrauterine growth retardation (smal! for date)
r Important causes of IUGR are :-
1) Maternal malnutrition: Most important cause.
2) Substance/drug intake: Smoking and tabacco(NEEr' AttMS 87), alcohol$EEr' AIIMS 87), propranolol\IlMs 87) "

3) Maternalfacfors: Short stature mother, primi or grand multipara, yound mother (< 20 years),lowpre-pregnancy
weight.
4) Maternal illness/diseases : Anemia, CRF$'rr), heart disease, malaria, pre-eclampsia, hlpertension.
5) Placentalfacfors: Abruptio placenta, excessive infarct, single umblical artery.
6) Fetal factors : First born babies, genetic/chromosomal aberrations, twin/multiple pregnancies, intrauterine
infection@at) .

Types of IUGR (smallfor date neonate)


r IUGR is classified as :-
1) SymmetricIUGR
I Reduced growth is symmetric. Head circumference, length and weight are equally affected.
2) AsymmetricIUGR.
r Reduced growth is asymmetri c with relativ e sp aring af head@EEr ) .
-t Newborn lnfanit
Predisposing factors for neonatal sepsis
1) Forearlyonsetsepsis
o Low birth weight or prematurityaleB)
t Prolonged rupture of membrans(erse)
r Foul smelling liquor
o Multiple per vaginum examination
o Maternal fever
r Di{ficult or prolonged labor
r Meconium aspiration
2) Forlate onset sepsis
r Low birth weight
o Superficial infections (pyoderma, umblical sepsis)
r Disruption of skin integrity with needle pricks and use of iv fluids.
o Lack of breast feedingler wt
o Aspiration of feeds
Clinical features of neonatal sepsis
o Early onset sepsis manifests frequently as pneumonia and less commonly as septicemia or meningitis"
e Late onset sepsis manifests as septicemia or pneumonia or meningitis.
o The most common and characteristic manifustations is an alteration in the estabilished feeding behavior -+ baby
refuses to suck arud becomes lethargic, inactive or unresponsive.

r Important laboratory finding are :-


1) Neutropenia is common (rarely neutrophilia may occur in severe neonatal infection{ttsa)1.
ii) Increased number of immature neutrophils1l e6).
111) Increased ESR and CRPute6).

Treatment of neonatal sepsis


r Antibiotic treatment for neonatal infection is :-
1) Neonatal pneumonia : Ampicillin plus gentamicinurlMseT).
2) Neonatal meningitis: (i) Ampicillin plus gentamicin pius chloramphenicol or (ii) Cefotaxime plus gentamicin, or
(iii) Ampicillin plus cefotaxime.

RESPIRATORY DISTRESS
o The principle features ofrespiratory distress in a neonate are :-
1) Taclrypnea (fast breathing): Fast breathing is defined as :-
l) Child less than 2 months of age + > 60 breaths per miflute?Gto4)
ii) Child aged 2 months upto 12 months -+ > 50 breaths per minute
iil) Child aged 12 months upto 5 years ) > 40 breaths per minute

2) Use of accessory muscles for respiratio n@NB


12' PGI 04)
r Features include use of sternocleidomastoid muscles, nasal
alar flare and tracheal tug.
3) Use of intercostal or subcostal mural"rtnN'.'' rcr 0a)
for respiration resulting in intercostal or subcostal recession-
lower chest wall indrawing.
4) Audible grunting(DNr 12'PGI01)

5) CYanosis(DNB12'PGI04)

Apnea
o Apnea malr !s defined as -
i) Cessation of respiration for 20 seconds with or without bradycardia and cyanosis(Ar 13' NEEI' DNB 12)
.

or
ii) Cessation of respiration for less than 20 seconds if it is associated with bradycardia or cyanosisGr 13' Nnnr' DNB 12)
.

i.

h
ti

I
. Apnea is more common in preterm infants.
c Apnea of prematurity ocatrs in preterm neonates in 2nd to 5th day of life and is because of immaturity of
developing brain.
o Important causes of neonatal apnea are : Hypoglycemia@Grqg),metabolic acidosis ,hypothermia@G108) andpretnaturity@er
oB)
.

Pulmonary causes of neonatal respiratory distress


o Respiratory distress syndrome (hyaline membrane disease){dtsst.
o Meconium aspiration syndrorngqt ss),

r Pneumonia.
t Transient tachypnea of newboyn(etess.
r Persistent pulmonary hypertension
o Congenital malformation -+ TER Diaphragmatic hernia(Pclo3' 0r), lobar emphysema, pulmonary hlpoplasia.
r Upper airway obustruction -+ choanal atresia, vocal cord palasy, lingual thyroid.
r Pulmonary hemorrhage
Causes of respiratory distress with mediastinal shift in neonates
r Diaphragmatic hernia
r Congenital lobar emphysema
o Congenital mediastinal mass
o Pneumothorax
o Congenital cystic adenomatoid malformation

Meconium Aspiration Syndrome


t Meconium is the first stool of an infant, composed of materials ingested during the time the infant spends in
uterus, i.e. intestinal epithelial cells, lanugo, mucus, amniotic Jluid, bile and water.
o Meconium aspiration syndrome (MAS) occurs when infants take meconium into their lungs during or before delivery.
o lt ts more common in post-term neonates@Gl0a) but may also occur in term neonates.
Consequences of meconium aspiration
o Three main problems occur due to meconium aspiration -
i) Obstructive emphysema or atelectasis(NEEr) ) when aspirated material blocks the airways.
ii) Pneumonitis and chemical pneumonia -> because of irritant property of meconium.
iii) Defective gas exchange.

Clinical maiifestations
r Respiratory distress within lst hour with tachlpnea, retraction, grunting and cyanosis.
r Overdistention of the chest.
Complications
r Respiratory distress sl,ndrome r Pneumothorax r Persistent pulmonary hlpertension

Radiographic pictures of MAS


o Hlperventilated lungs with patchy infiltration. r Flattening of diaphragm.
r Increased anteroposterior diameter of chest.

The following chest X-ray of a post-term neonat€ is seen in:

b) Transient tachypnea
'trc)l,.+keola;1protf.;i,*dsis'.',1:,,,,1.' "'',, ',',, .,r,

irrr.l:d},:rMbeqniirin.as?iratiOn.;,1r .' ,::,::1.. 'r,..:,: ., ..: :


:i.:

Ans. is'd' i.e, Meconirrrn asprration


p,11.Ttit$venehgst X-iii4,li, lho,wrry
'''1":' hff erinflation with p'iofitirution (inpost;teim nionate)+.diagnosis
:i,,.:,
'. ,- i* *6aodrt* asoitation sfiidrornsi' ' r:'" ,
' r There is herniation of abdominal contents, i.e. stomach (most of the time),
intestine, liver; into thorax through a defect
in the diaphragm.
r Females are affected more than males.
o More common on left sidearrus e7) and is poster sl4Llalanus sz)
.
o Components of CDH are :-
i) Herniation of abdominal contents into thorax
ii) Pulmonaryhypoplasia
iil) Malrotation of intestine
o Associated anomalies may be seen 30% of cases -+
cNS lesions, esophageal atresia, omphalocele and CVS lesions.
Most cases are sporadic.
r cDH is a reconized part of several chromosomal syndromes -+ Trisomy 21, 1g, 13, turner syndrome, pallister-Killian,
Fryn.

Clinical features
o CDH may present as
l. Soon (within 6 hrs) after birth (most of the cases)
or
2. After neonaral period (small group)
l) Soonafterbirth
t Respiratory distress is a cardinal sign@Nn o -> presents as tachypnea, grunting@rva re, chest
retraction,
cyanosis.
t Scaphoid abdomen@NB ls)
r Increased chest wall diameter
r Bowel sounds may be heard in the chest with decreased breath sound.
r Cardiac impulse is displaced away from the side of hernia.
2) Afterneonatalperiod
: Vomiting as a result of intestinal obstruction
r Mild respiratory symptoms
r
occasionally, incarceration of the intestine will proceed to ischemia
with sepsis and shock.
o Most common cause of death in CDH is pulmonary
complications(ArrnseL) (pulmonary hypoplasia{ArruseB)
and pul-
monary hypertension).
Diagnosis
o Prenatal ultrasound can diagnose CDH between 16 and
14 wk.
o After delivery chest X-ray and nasal gastric tube is all that
is usually required to confirm the diagnosis.
Prognosis
o Two most important prognostic
facto.rs are pulmonary hypertension(AilMs il, 0s,At oe) and pulmonary hypoplasiailrrMs
"). Other prognostic factors are : Gestational age at detectionailMs 11Ar lt),associated
anomalies, size of 11,
d.efect{ArrMs
AI11)
aldside of defect (right side has poor prognosis). Timing of surgery is not Ar
a prognostic factor*nus 11, 11).

Management
e cDH is no longer considered as surgical emergency child
is first resuscitated and stabilized before surgery.

,,,*
't
Dlttercnlial diagnosir
r Pulmonary sequestration.
o Pleuropericardial cysf |tt Ms s7 ).
r Cystoid adenomatoid malformation.

The follorring che*t )Lrayis showing:

a) Hyaline mernbrane disease


b)'Transient tachlpnea
c) Congenital di4phragmatic hernla

Aas, ts t' i.e., Congeaikl diaphragmatic hcrnia


o This chest X-ray is drowing herniation of boryel loops into left hemithorax with shift of heart and mediasti-
num to opposiie side + diJgnosis is CDH.

Hyaline membrane disease (HMD) or respiratory distres; tyndrome {RDSI


r HMD always occurs ln preterm babies often less than 34 weeks of gestation. It is the commonest cause of respiratory
distress in a preterm neonate.Incidence is highest in preterm male and white infant.
o HMD disease is due to deficiency of surfactant@Gl ee). Surfoctant start appearing in fetal lung by 20th week of
gestationAllMs e8). But mature levels appear only afier 35 weeks of gestation; therefore HMD is more common in
ee)
premature newborns. In amniotic fluiils, surfactant appears between 28-32 weelcs of gestation@Gt .

Risk factors for HMD


t Prematurity (most important)at ts' ozt r Cesarean section
r Asphyxia r Cold stress
o Acidosis o A history ofpreviously affected infants
o Maternal diabetesGt 02, PGI Be)
o Multiple bifthAl 02, PGr se)

Protective factors for HMD


r Hypertension r Maternal heroin use
r Prolonged rupture of membrane r Antenatal corticosteroid prophylaxis
Morphological changes in HMD
o Diffuse alveolar damage is the characteristic histological
manifestatio of HMD.
I In the acute stage, the lungs are heal7, firm, red and boggy with congestion, interstitial
and intra-alveolar edema.
r The alveolar walls become lined with waxy hyaline membrane. Hyaline membrane
consist of fibrin rich edema fluid
mixed with the cytoplasmic and lipid remnants of necrotic epithelial
cells(NrE").
Clinical features of HMD
o Respiratory distress occurs within the first 6 hours of life.rcr 08, AMU 88)

r Clinical features include -


t Tctchypnst@etot) r Decreased air entry r BP mav fall \
r Retraction(Pcl01) r Fatigue . oliguria I lf untreuted
r Grunting r Pollor r Ileus )
r Cyanosis r Mixed respiratory & metabolic acidosis,:Pcrol)
o FRC is smaller than closing volumeallMs 10).

X-ray appearance in HMD


r The initial roentgenogrom is occasionally normalK"oros't),withtypical
pattern developing at 6-12 hours.
i) Reticulogranulat patternullMs 12, 10, PGI e7, Kerata s4)

ii) Ground glass opacities\lrMs 12, 10' PGI e7, Keralae4)

iii) Air broncho gramArrMs 12, PGI s7' Kerata e4)


10,

iv) In severe case whiteout lunglenus n' t0, PGr e7, Keruta e4)
Prenatal diagnosis of HIUID
r Prenatal diagnosis of HMD can be made by -
1) Lecithin / sphingomyelin (L/S) ratio in amnioticfluid @EEr)

r L/S ratio > 2 indicates adequate lung maturity.

2) Shake test
r Amniotic fluid or gastric aspirate is mixed with absolute alcohol and shaken for 15 seconds and
allowed to settle' copious bubbles are formed in the presence
of adequate surfactant indicating extent of lung
maturity.

Treatmentof HMD
r Neonate should be managed in NICU (neonatal intensive care
unit).
o Oxygen should be given ,
Mild distress _+ Without ventilator
Moderate distressqllMs 10' 0e) -) Continuous positive airway pressure (CpApltenus n, os1

Severe distress -) Intermittent mandatory ventilation (IMV)


Oxygen tension is kept at 90o/o (100% oxygen is not given{AI02)
' because ofrisk ofretrolental fibroplasia).
t Intratracheal surfoctant therapy is the specific therapy. Its indications
are : -
i) All neonates less than 2g weeks irrespective of presence or absence of HMD (RDs).
ii) Neonates with > 28 week with severe respiratory distress.
Prevention of HMD
r Administration of prenatal glucocorticoids to mother at least 24 hrs prior to delivery pregnancies of less than
34 weeks gestation, reduces the risk of HMD'

olnjectionbetamethasone (l2mglmevery24hours(Nnnr)JFor2doses)ispreferred.Dexamethasone(6mglmevery
12 hours for 4 doses) i.s an alternative. Beside reducing HMD risk, corticosteroids also reduces mortalfiy(AlrMsee) and
v entri cular h emorrhage(
ut us sa) .
incidence of int er

fire followlag lLray of a pretergr newbornsuggest the diagnosis ofi


a) Meconium aspiration
b) Transient tachypnea
c) Hyaline membrane disease
d) None '.r . .,....

'Ans. i* lcii,e., Hyalinemernbrane disease


'
i.' . Il_re chestix,ray is showin$ reticulogranular pattern with increased aiibronChogram > hyaline membrane
disease.

Bronchopulmonary dysplasia {BPD}


o Bronchopulmonary dysplasia is usually defined as a need for supplemental oxygen at 36 weeks afier conception.
BPD is usually defined as a needfor supplementel oxygen at 36 wk afier conception.
e Respiratory distress persists or worsen and is characterized by hlpoxia, hlpercapnia, oxygen dependence and in severe
cases right sided heart failure.
o BPD is a result of lunginjury in infants requiring mechanical ventilation and supplemental oxygen.
o Most of the children with BPD are [,remature and have hyaline membrane disease. But, BPD can also occur in full
term newborns with meconium aspiration or persistent pulmonary hlpertension.
r The premature lung makes insufficient functional surfantant and the antioxidant defence mechanisms are not sufficiently
mature to protect the lung from the toxic oxygen metabolites generated from oxygen therapy.
r The major underlying pathogenic mechanisms causing BPD are :-
1) Oxidant damage by toxic oxygen metabolites$In&'u).
ii) Mechanical trauma\le6) z Atelecto trauma (alveolar collapse and volutrauma/barotrauma(Ale8) due to
overdistension by mechanical ventilation).
lll) Pulmonary edema due to capillar! damage@Ies'se)

Moderate Sev€re{A' 73)

Supplement 28 days) and Supplement O, (for 28 days) and Supplement O, {for 28 days) and

' . Breathing > O, and/or


3Ao/o

> 32 weeks
r Breathingroomairby56days r <300/oQrby56dayspostnatal positive pressure (PPV) by
psstnatalageoratdischarge ageoratdischarge(whichev- 56 days postnatal age or at
GA at birth
(whichevercomesfirst). ercomesfirst). whichever comes
l::i,:"tn"
=qR. tp,u,a."=q{gffi::;SlFiiiiti;i: ::.:iii:.j'r:, .::::i:...::i::.,j.,,,...rr,..ii#L't?i,t;.i:i,il:;,.::.-:iii': :ir!

Transient tachypnea of Newborn


r Transient tachlpnea of the newborn is a benign self-limiting disease occuring usually in term neonates and is due to
delayed clearance oflung fluid. It is also called respiratory distress syndrome type II.
o TTN follows -
cxapren 3,.New.,bornrr&
FI
i) Uneventful normal preterm or term vaginal
delivery
ii) Cesarean deliveryLauusoz)
I TTN is believed to be secondary to slow absorption
of fetal lung fluid resurting in decreased
and tidal volume and increased dead space pulmonary compriance
therefore arso knowi as wet rung.
o Clinical manifestations
r Early onset of tachlpnea
r Sometimes retraction or expiratory grunting
I Occasionally cyanosis
: Patients usually recover rapidly within 3 days,
t Hypoxemia, hypercapnia and acirJosis are uncommon.
r The lungs are generally clear without rales
or rhonchi and chest roentgenogram shows
-
s Prominent pulmonary vascular markings s overaeration(ArrMs5s)
t Fluid lines in the fissure t Fbt diaphragms
r Prominent inter-labarfissureLrrMs05,pcree)
t occasionally pleuralfluid.
r Distinguishig this disease from hyaline membrane
disease is difficult. The distinctive
feature of transient tachyapnea
are -
: Mild symptomology
s Sudden recovery ofthe infant
: Absence of roentgenogralthic reticulogranular
pattern or air bronchogram(pcr e8).
Pulmonar), alveolar proteinosis
o Pulmonary alveolar proteinosis disorder characte rized by the intraarveolar
is a
accumulation ofpulmonary surfactant.
' ":7^:!::r*'n-B component or swractant, which results
in imparied spread and absorption ormajor
;H.ffi:Jfl?
r Thus there is functionar deficiency
of surfactant and.failure of expansion
c usually newborn is full term and may have positive
of arveori.
famiry histovrr*,*s,, 08.07).
r chiid usually present with respiratory distress immediately
after birth which does not respond to
08,07). surfactant(Arr*s 11.

o fhere may be ground-glass appearanceon


chest x-ray.
t Histopathological examination of lung biopsy
specimen is the gold standard for diagnosis.on histopathological
examination distal air spaces are filled with a granular, eosinophillic material that stains
schiff reagent and is diastase resistant- positiv ery withperiodic - acid

RESUSCIT4T|ON OF NEWBORN

r The goal of pediatric resuscitation is to


maintain adequate oxygenation and perfusion
of blood throughout the body
while steps are taken to stabirize the chird and
estabrish rong term hemostasis.
o The most common indication for neonatal
resuscitation is asphlxia. Second most
prematurity. common indication is extreme
r The components of neonatal resuscitation
are TABC _
t T - Maintenance of tempetAture@G107): By radient heat source,
and by drying the baby with removal
t A - Airway(Pcl,7) : Establishment of open airwayby placing of wet linen.
the neonate on his back with stight
extension of neck by
':::':"f.::shoulder 3/4 or 1 inch of the mattress with the help of rolled blanket o, to*J Suction of mourh and
s B - Initiate breathinglpcr07) : Tactile stimulationis
given and when necessary,bagand
ET tube can be used. mask ventilation is given or

t C - Maintain circulation@G|07) : This is done


by chest compression@Gr o7)and/or medications.
Resuscitation protocol
r As soon as the babyis delivered, five signs
are assessed _

1. Clearance of meconium
2. Active breathing or crying
3. Good muscle tone (flexed posture and active movement of baby).
4. Pink color (look at tougue and lips)
5. Term gestation (deliverybetween3T-42 weeks of pregnancy).
r Ifall signs are positive no active resuscitation is required.
r Ifany ofthe 5 signs is absent, baby requires resuscitation.
r The baby should be placed under the heat source (radiant warmer) and subjected to a set ofintervention known as

initial steps.

lnitial steps of resustitation


1. Positioning
r The neonate should be placed on her back or side with the neck slightly extended. This can be achieved by
putting a rolled blanket or towel under the shoulders, elevating them 3/t or 1 inch off the mattress.
2. Suctioning
r The mouth and nose should be suctioned. The mouth is suctioned first{Pcl
10)
to ensure that there is nothing for
the infant to aspirate when the nose is suctioned.
r One should not insert the catheter very deep in mouth or nose for suction -+ Stimulation of posterior pharynx
during the first few minutes after birth can produce a vagal responce, causing severe bradycardia or apnea.
r Therefore, during oral suctioning, a suction tube is gently introduced into the babys mouth until the 5 cla
10)
markGGI 10) is at the babyt lip. During nasal suctioning, a suction tube is introduced upto 3 cm mark@Gl into
each nostril.

3. Dry, stimulate and reposition


r After suctioning, the baby should be dried by using pre-warmed linen to prevent heat loss.
1 A brief tactile stimulation in the form of flicking the soles or rubbing the back may be provided in case of
non-establishment of good respiratory efforts(Are7).

4. Freeflowofoxygen
r Ifthe baby continues to be depressed, provide free flow of oxygen using a facemask.

o After providing initial steps, the baby should be evaluated for three signs -
1. Respiration
2. Heart rate (HR)
3. Color
r If baby has good breathing, HR >100 and pink color, he should be given supportive care.
o If the baby is not breathing well or HR <100 then bag and maskventilation is needed.
o After the infant has received 30 seconds of ventilation with 100% oxygen by bag and mask, evaluation of heart rate
should be done -
HR >100 -+ Discontinue ventilation if spontaneous respiration is presenl(ersz).

HR 60 to 100 -+ Continue ventilation


Below 60 -+ Continue ventilation + chest compressions
o After 30 seconds of chest compressions, the heart rate is checked.
HR < 60+ Continue chest compression and bag & mask ventilation + initiate medications.
HR > 60+ Discontinue chest compression but continue bag & mask ventilation until the heart rate is above 100'

Resuscitation in newborn through meconium stained liquor


r When baby passes meconium in utero, there is a chance that the mecomium will be aspirated into infant's mouth and
potentially into the trachea and lungs.
o Appropriate steps must be taken imrii'ed irtely after delivery to reduce the risk of serious consequences resulting from
aspiration of meconium.
t Intrapartuffi nasopharyngeal suctioning just afier the delivery of head is no longer recommended ss it does not
reduce the risk of meconium aspiration syndrome and, on rare occasions, may cause nasopharyngeal trauma or a
cariliac arrhythmia.
r The first step after delivery is to identify whether
the newborn is vigrous or non-vigrous :
A) Vigrous newborn
r A newborn is classified as vigrous, if he has all the three signs are present :
1) Strong respiratory effoyl\rrus os)
2) Good muscle lon"6rlMsoe)
3) Heart rate greater than 717GIIMI0e)
r The vigrous child does not require any tracheal suctioning
and the usual
initial steps of resuscitation are provided,
i'e'' provide warmith, positioning, suctioning of mouth and
nose (not tracheal suctioning), Dry, stimulate and
02 if necessary.
B) Non-vigrous newborn
: If any of the above three signs is present, the newborn is classified
as non_vigrous.
r For non-vigrous child, the initial steps are modified: _
r) Place the baby under racliant warmer and postpone
suctioning to prevent stimulation of posterior
pharyngeal wall that can cause bradycardia.
ii) Residual meconium in the mouth and posterior pharynx
should be removed by suctioning under direct
vision using a laryngoscope.
iil) The trachea
should then be intubated and mechonium suctionecl
from the lower airway. Tracheal
suctioning is best done by apprying suction directry to
the endotrachear tube.
I After providing initial steps, the further management is
same as with resucitation for other conditions-

a Forchildren,uncuffedandstroightblade(,4//M599]endotra.n;;l*b"
a
";rt* ;;;l,lli;on,o,
ofpremature
a l:::::"j:1,::l
birth l"j_,1"_1l.,l"sutation
In cases of asphyxia, corticosteroidsr,renotuseclhtos).
,,,,6,pcles,
warmed
vvqlrrreu rrtLuu..u'5'
incubators.
a Bagandmaskventilationiscontraindicatedin:Diophragmtatichernia(AtMsoa,ss),Tracheo-esophagealfistulorucscoz),and
meco nium aspi roti on sy n d rom e4il.vs 8s).
Important druqs used for neonatal resuscitation are epinephrine
(adrenaling){A/75,, normal saline or
naloxone/4i,5) and sodabicarbonate#i rir. ringer lactate,
Dose of adrenaline inneonatal resuscitation : 0.1
to 0.3 mg/kg diluted l n. :l o,oggurrr

APGAR Scoring
o APGAR Score is a quantitative method for assessing infants respiratory,
circulatory and neurological status

. Respiratory efforl

:l*i
o Color ofthe bodytAttMseT) (Ap-
pearance) Blue or pale Body pink extremities blue pink

Flaiiid
o Reflex stimulation (Grimace)
or putting catheter in nose No response GrimaCerrr,5l Cries, coughs or sneezes

r Scoring for APGAR Score


r Total score l0
r No depression 7-t0
r Mild depression 4-6
r Severe depression 0-3
o The test is generally done at one and
five minutes after birth, and may be repeated later if the score is and remains
low' Scores 3 and below are generally regarded as critically
lo w, 4 to 6fairly low, and 7 to 10 generally
normal.
o A low scote on the one-minute test may show that
the neonate requires medical attention (e.g. resuscitation@nMs 11))
x3 Newborn lnfant

The first step after delivery is to identify whether the newborn is


vigrous or non-vigrous : -
A) Vigrous newborn
r A newborn is classified as vigrous, ifhe has a[ the three signs are present: _

l) Strongrespiratory effortGIIMSoe)
2) Good muscle toneGIIMSos)
3) Heart rate greater than 700aIIMS0s)
r The vigrous child does not require any tracheal suctioning and the
initial steps of resuscitation are provided,
usual
i'e', provide warmith, positioning, suctioning of mouth and nose (not tracheal
suctioning), Dry, stimulate and
02 if necessary.

B) Non-vigrous newborn
r If any of the above three signs is present, the newborn is classified as non-vigrous.
: For non-vigrous child, the initial steps are modified: -
i) Place the baby under radiant warmer and postpone suctioning to prevent stimulation of posterior
pharyngeal wall that can cause bradycardia.
ii) Residual meconium in the mouth and posterior pharynx should be removed
by suctioning under direct
vision using a laryngoscope.
iil) The trachea should then be intubated and mechonium suctioned
from the lower airway. Tracheal
suctioning is best done by applying suction directly to the endotracheal
tube.
r After providing initial steps, the further management is same as with resucitation for other conditions.

a For chi ld ren, u n cuffed a nd stra ight blade^rMs ss) enclotr achea l tu be is used.
a lncubators used for thermal regulation of premature neonates are convectio
nut13.06,pctsa) warmed incubators.
a ln cases ofbirth asph;n<ia, corticosteroidsare not asedal0s).
f Bagandmaskveniilaiioniscontraindicatedi; riiopnrng^oticherniaulMsM,se),Trocheo-esophagealfistutatuescoz),and
mecon i u m aspi rotion sy n d rome(At tns 8st.
lmportant drugs used for neonataf resuscitation are epinephrine
{adrenaline)(,4r,5.r, normal saline or ringer lactate,
6616;qpgFr rs,r and sodabicaylgn31g(er ;s).
Dose of adrenaline inneonatal resuscitation : 0.1 to 0.3 mg/kg
diluted l :l o,ooo(Ntr).

APGAR Scoring
r APGAR Score is a quantitative method for assessing infants respiratory,
circulatory and neurological status :

. Respiratory
iii.;:#;*irefforttil"*ai j\rr,.r.;
None

;,,,*,,,,;4,;
Slow irregular
i:,=,'r1?itf.ii-,.1-'r...;1"
Good, crying
> 100
t Color ofthe boayroiisin lg;-
pearance) Blue or pale Body pink extremities blue Pink
.-,rt*:i:t:::.j -:rr*r::=i:.: a.: -.... ..
o ii.:::r.:1::::1-1

Mrislcle tone(Affi er, (A(tivity)


Some flexion .t
Aatival, moving extremity
r Reflex stimulation (Grimace)
or putting catheter in nose No response Grimacer'ari5., Cries, coughs or sneezes

Scoring for APGAR Score


r Total score 10
r No depression = 7-10
r Mild depretsion = 4-6
r Severe depression = 0-3
The test is generally done at one and
five minutes after birth, and may be repeated later if the score is and remains
low' Scores 3 and below are generally regarded as critically low, 4 to 6 fatlylow,
and 7 to l1generally normal.
A low score on the one-minute test may show that the neonate requires medical
attention (e.g. resuscitation(ArrMs11))
but is not necessarily an indication that there will be long-term problems, particularly if there is an improvement by
the stage of the five-minute test.

c lf the Apgar scote remains below 3 at later times such as 10, 15 or 30 mintues, there is a risk that the child will suffer
longer-term neurological damage. There is also a small but significant increase of the risk of cerebral palsy.

r However, the purpose of the Apgar test is to determine quickly whether a newborn needs immediate medical care; it
was not designed to make long-term predicitions on a child's healthallMs 11) .

Assessment of respiratory depression


c The severity of respiratory distress is assessed by Silverman- Anderson score and Downes' score.
r While the Silverman Anderson Retraction Score is more suited for preterms with HMD, the Downes' score is more
comprehensive and can be applied to any gestational age and condition.

Lort.thest .,,,
Xiphoid:retraction .,. Na!-l'flaring G'i*nt. ,:.
retra(tion.
Nong None None None
,:t :.:1 , Lag oqin$pirqliqh :': Jnii'tti.bte .r.:.,i. Just visible Minimal :Stetho5c0peonly
2 See- saw Marked Marked Marked Naked ear

e A score of >6 is indicative of impending respiratory failure

Nil
0 < 60/min Nil Normal None
' \.\
:',,. .:r';;;',,,11.,14
Au,sc with',,.'
:,,r.-1 ,,.1'l :60.80/m.in ',.[n'rgorn air :Mild:?::',: ,.
stethoslo:pe
Mild,. .1.: :,::.':.

Audible with
2 >80/ min ln >4Oo/o O, Marked? Moderate
naked ear

e A score of >6 is indicative of impending respiratory failure

NEONATAL HYPERBILIRUBINEMIA AND JAUNDICE


a Bilirubin is the end product of catabolism of hemoglobit 1 gm of hemoglobin yields 35 mg of bilirubin(NEEr).
a A bilirubin level of more than 5 mg/dl manifests as clinical jaundice or icterus in neonate (in adults jaundice occurs at
2-2.5 mg/dl).
As the bilirubin levels increase, jaundice involves more distal parts of body:-

Head & Neck 5 mg/dl


Lower trun k. &, thigh. ..,,
:',
:..10mgldl
Legs 15 mg/dl
PaIm {hands) & sole {fedt} 5 1,5 mgldr*/'sr
e Hyperbilirubinemia may be of two types : (i) Unconjugated; and (ii) Conjugated.

A) UnC,onjugated hyperbilirubinemia
* Conjugated hyperbilirubinemia is seen when -
1) lncreasedproductionofbilirubinfromhemoglobin,Sothatthecapacityoflivertoconjugatebilirubinisoverwhelmed
by increased production, e.g. -
t Hemotytic on"^iotn"''tto7t -+ Hereditory spherocytosis, G6PD deficiencyulittsaT).
o lneffective erythropoiesis - + Thalassemia, Pernicious anemia.
2) Reduced hepatic uptake of bilirubin from bilirubin - albumin complex + Drugs,
3) lmpaired hepatic conjugation, e.g.
i) -) Liver of a neonate is functionally immature (, UDPG transferase)
Physiologicol joundice4ilMs0T)
ii) Breastmilkiqundiceottusair-+ Pregananedial in breast milk interferes with bilirubin conjugation.
iii) Geneticdeficiencyof UDPGtransferase-+ Crigter-Nojjatsyndrome\'ls,NEETDNBl3,Pctosa6Aufsot),Gilbertsyndrome(At
I 3, NEEI DN| | 3, pct 0g. 06, AIIMS O I )

iv) Hepatocellular disease --+ Virol or drug induced hepatitistatBs),cirrhosis.


v) Hypothyroidism(At;t'l\ 07),
cepholohematomo
B) Conjugatedhyperbilirubinemia
r Conjugated hyperbilirubinemia is seen when -
i) lmpaired secretion of conjugated bilirubin into bile -+ Dubin-johnson syndrome(At 13, NEEI. DNB 13 PGtas M AilMs o1),
Rotor
Sffidrom2tAt : 1 Nrfi' DNB t3 PLt as 06' AttMS a4.

ii) lmpaired bile flow --> Obstructive jaundice(AttMsol), primary biliary cirrhosis, Neonotal cholestosis, e.g. Extrahepotic
biliaryatresia(At13)/neonateidiopathichepatitis,Choledocolgyst(PctM,sclerosingcholangitis, Carolidiseose,Metobolic
(Tyrosinemia, Wolman disease, Nieman pick disease, Galactosemia, Fructosemia).

Tirne of appearance of jaundice


Jaundice pres€nt at birth or within 24 hours of life
a Erythroblastosis fetalis (Rh incompatibility;wanr eGI 04' 02' AI es)
-) most common cause
r ABO incompatibility
r Concealed hemorrhage
r Congenital infections -+ Rubella, slphilis, CMV, toxoplasmosis
r Red cell enzyrne defects (G6PD deficiency)
r Red cell membrane defect (Hereditary spherocytosis)
. SePsis(At e5)

Jaundice appearing on 2nd or 3rd days of life


t Physiologicalqtes) r Crigler - Najjar syndrome
\ r Early onset breast milk jaundice

Jaundice appearing frorn 3rd to 7th days of life


r Bacterial sepsis r Urinary tract infection
r Other infections _> syphilis, CMY enterovirus, toxoplasmosis r Polyclthemia
Jaundice appearing after 1st rveek of life
r AII other causes ofjaundice.

Physiological jaundice
r Most neonates develop visible jaundice due to elevation of unconjugated bilirubin concentration during their
first week. This common condition is called physiological jaundice.
r This pattern of hyperbilirubinemia has been classified into two functionally distinct periods -
1. Phase one
r Last for 5 days in term infant with peak bilirubin levels to 12 mg/dl.
r Last for 7 days in preterm infrnt with peak bilirubin levels to 15 mg/dl.

2. Phase two
r 'Ihere is decline to about 2 mgldl, which iasts for 2 weeks after rvhich adult values are attained.
Criteria for physiological jaundke
. Clinicaljaundice appears after 24hours ofage'PCI03'.
'.'. Total bilirubin rises by less than Smg/dl per day (no sudden rise)@etos).
;llt Peak bilirubin occurs at i-5 days of age, with a total bilirubin of no more than 15 mg/dl
it t Clinical jaundice is resolyed by I weeks in term infants and 2 weeks in preterm infants(eet ost,

Breast milk jaundice


o There is strong association between exclusive breastfeeding and neonatal jaundice.
o ^\ fery babies who remain on exclusive breast feed develop jaundice rn the second week of life and continue well
into the thirdmonth. This is called breastmilkiaundice.
o -\ bilirubin level of over 20 mg/dl may be attained. (It is presumed to be due to inhibitory substances in the
breastmilk that interfere with bilirubin conjugation e.g. pregananediol and free fatty acids).
. Teffiporary interruption of breastmilk feeds will dramatically reduce the serum levels of bilirubin and there may
be slight increase in bilirubin when breast feeding is resumed, but it never reaches the previous levels.

ABO hernolytic disease


o ABO hemolytic incompatibiliry It causes unconjugated neonatal hlperbilirubinemia.
disease is less common then Rh
r It occurs it neonates havingblood group A or B with mother of blood group O. |aundice appears within 72-24hosrs
of life.
o Direct agglutination test (DAT) test is positive.
to)
o Reticulocy.tosis and the presence of microspherocytesunus on smear help to confirm the diagnosis.

Congenital hyperbilirubinemias
r Important congential hlperbilirubinemias are :-
1) IJnconjugatedhyperbilirubinemia (ilefective conjugation): Gilbert syndromeql 13'NEETDNa n'PGIoe'06'AIIM\01'%),
Crigler Najiar Synilrome (type I dt II)(u t:' Nrnr'
DNB ls' PGr 0e' 06' AIIMS 01' e3)
,

2) Conjugated hyperbilirubinemia z Rotor syndromeut


13' NEEI' DNB 13' PGr 0e' 06' AttMS 01' %)
, Dubin lohnson syndrome.

Gilbertsyndrome
e7) 13' NEEI' DNB 13' PGI
e Gilbert syndrome rs an autosomal dominantattMs congeflital unconjugated hyperbilirubinemiaat
09, 06, 02, 4ttMs 0 L 97, 93).

c Allhepaticbiochemicaltestsandliverhistologyarenormal\ttMseT),butinsomepatientsincreasedlipofuscinpigment
may occur.
ls' AttMS e7),
e There is mild increase in unconjugate bilirubin(/r thus kernicterus does not occur.
e Basic defects are: - \
l) Decreased hepatic uptake of bilirubin.
ii) Decreased activity of UDPG transferaseql
ls).

e Usually no treatment is requiredarrs).


e Phenobarbitone induces the enzyme UDPG transferase; thus it can reduces the level of unconjugated bilirubin by
increasing its conjugation. Hemolysis may also occur.
e The disposition of most xenobiotics metabolizedby glucronidation appears to be normal exceptfor antitumor
0')
drug irinotecan. Thts, irinotecan toxicity can occur@G' .
o There is no association with cirrhosis\I 13).

Criggler-Najjar Syndrome (CN)


o CNsyndromeisacongenitalunconjugatedhyperbilirubinemia(Al)3'DNBI3'NEEI).Basicdefectiseitherreducedorabsent
UDPG transferase.
r There are two types of CN syndrome :-
L) Typel
r It is an autosomal recessiye disorder in which UDPG transferase is absent. Thus phenobarbitone has no effect
(as enzlT ne is absent).
s It is more severehyperbilirubinemia than type II. There may be kernicterus.
Criteria for physiological jaundice
i) Clinicaljaundice appears after 24hours ofage@Glos).

ii) Total bilirubin rises by less than Smg/dl per day (no sudden rise)(ecr 03)
.

ili) Peak bilirubin occurs at i-5 days of age, with a total bilirubin of no more than 15 mg/dl
iv) Clinical jaundice is resolved by 1 weeks in term infants and 2 weeks in preterm infants@ct ost
.

Breast milkiaundice
o There is strong association between exclusive breastfeeding and neonatal jaundice.
o A few babies who remain on exclusive breast feed develop jaundice in the seconil week of life and continue well
into the third month. This is called breastmilkiaundice.
o A bilirubin level of over 20 mg/dl may be attained. (It is presumed to be due to inhibitory substances in the
breastmilk that interfere with bilirubin conjugation e.g. pregananediol and free fatty acids).
o Temporary interruption of breastmilk feeds will ilramatically reduce the serum levels of bilirubin and there may
be slight increase in bilirubin when breast feeding is resumed, but it never reaches the previous levels.

ABO hemolytic disease


r ABO hemolltic disease is less common then Rh incompatibility. It causes unconjugated neonatal hyperbilirubinemia.
r Itoccurs inneonateshavingbloodgroupAorBwithmotherofbloodgroup O. |aundice appearswithin l2-24hotrs
of life.
e Direct agglutination test (DAT) test is positive.
to)
o Reticulocltosis and the presence of microspherocytesGltus on smear help to confirm the diagnosis.

Congenital hyperbilirubinemias
o Important congential hyperbilirubinemias are :-
l) (Jnconjugated hyperbilirubinemia (defective conjugation) t Gilbert syndrome6"i'NEEI'DNB 13'PGt0e'06'AilMS01'e3),
ts' Nnnr' DNB 13' PGI 0e' 06' AIIMI 0t' e3)
Crigler Najiar Syndrome (type I dt IISGT .

2) Conjugated hyperbilirubinemia : Rotor syndrotneat 13' NEEr' DNB 13' PGt 0e' 06' AIIMS 01' e3)
, Dubin f ohnson syndrome-

Gilhert syndrome
e7) 13' NEET' DNB 13' PGI
e Gilbert syndrome is atr autosomal dominan{or/Ms congenital unconjugated lryperbilirubinemiaAl
09,06,02, ArlMS 01,97,93)
.

e Allhepaticbiochemicaltestsandliverhistologyarenor*o1{utusez),butinsomepatientsincreasedlipofuscinpigment
may occur.
1s' ArrMseT),
o There is mild increase in unconjugate bilirubin(Ar thus kernicterus does not occur.
t Basic defects are: -
l) Decreased hepatic uptake of bilirubin.
ii) Decreased activity of UDPG transferaseql
ts).

r Usually no treatment is required'Als).


e Phenobarbitone induces the enzl,rne UDPG transferase; thus it can reduces the level of unconjugated bilirubin by
increasing its conjugation. Hemolysis may also occur.
e The disposition of most xenobiotics metabolizedby glucronidation appears to be normal exceptfor antitumor
drug irinotecan. Thus, irinotecan toxicity can occur@G'0') .
e there is no association with cirrhosis^I 13).

€ri ggler-ftlajjar Syndrome (CN)


DNB 13' NEET).
e CN syndrome is a congenital unconjugated hyperbilirubinemiaarr3' Basic defect is either reduced or absent
UDPG transferase. I
o There are two tlpes of CN syndrome :-
A) Typel
r It is an autosomal recessive disorder in which UDPG transferase is absent. Thus phenobarbitone has no effect
(as enzyme is absent).
s lt is more severehlryerbilirubinemia than tlpe mav be kernicterus.
Cnaprrn 3 llewbornlnfant ,: " .':,

I LFTs and liver histology are normal,Pcl02).


B) TypeII
r It is an autosomal dominantQcleT) disorder in which
UDPG transferase is ysflusgrrnersz;.
t Phenobarbitone can reduce unconjugated bilirubinrpprsT)
by inducing uDpG transferase.
r LFTs and liver histology are normal?Gl02) and. there
is no kernicterus@Gre7).
Dubin Jhonson syndrome (DJS)
o DfSisatypeofcongenitalconjugatedhyperbiliruginsnxirrer13,NLLI',ArrMs07).
o It is autosomal recessive.
r coniugated bilirubin is increased because of defective biliary excretion of
bilirubin glucuronides due to mutation in
canalicular multidrug resistance protein 2 (MRP 2). Liver function
tests are normal
o A cardinal feature of DIS is the accumuiation in
the lysosome of centrilobular hepatocltes of dark, coarsely
granular pigment' As a result, the liver is black in appearance.
This pigment is thought to be derived from epinephrine
metabolites that are not excreted normally.
o There is increased urinary excretion ofcoproporphyrine (normal
I coproporphyrine III is more excreted), but total
coproporphyrine level is normal.
o Gall bladder is not visualized on oral cholecystography.
After iv administration, there is reflux of conju gated.sulfubro-
mophthalein (Bromsulphalein, BSp) from liver to circulation.

Rotor syndrome
o Rotor syndrome is a type of congenital conjugated hyperbilirubinemiaut n, NEEI, AnMs 07).

o It is autosomal recessiye.
r It is due to decreasedbiliary excretion ofconjugatedbilirubinand
also due to decrease hepatic uptake & storage of
bilirubin.
o Differentiating features of rotor syndrome (from DIS) -
i) Liver is not pigmented and liver histology is normal(pcr02).
ii) coproporphlrine I is increased in urine but total coproporphyrine level
is also increased.
iii) Gall bladder is visualized
iv) There is no reflux ofconjugated BSp.
Kernicterus
r Kernicterus or bilirubin encephalopathy, is a condition caused
by bilirubin toxicity to the basal ganglion(Aree) and
various brainstem nuclei.
e It is mainly caused by unconjugated bilirubinal 12) as unconjugated bilirubin
can cross blood brain barrier.
Unconjugated bilirubin level of more than 20 mgl dltl't tst increases
the risk of kernicterus.
o Clinicaily, kernicterus is described in 3 phases, which may
progress over 24 hours to 7 days :-
l. Phase I -s poor suck, lethargy, hypotonia{o, ,i), depressed sensorium.
2. Phase II -) Fever, hypertonia progressing to opisthotonus(,Arrs).
3. Phase III -+ High pitched cry, conulsions, death.
o Long term survivors demonstrate choreoathetoid cerebral
palsyLl 13),
upward gaze palsyar 13),
sensorineural
hearing lo ss( AI 3) andmental retariation.
1

o The toxic effects ofunconjugated bilirubin (and therefor


e chances ofkernicterus) are increased by:-
1) Hypoproteinemia (t albwminl{,'ost -+ Normally unconjugated bilirubin binds to albumin
in circulation. If the
Ievel of albumin decreases, concentration of free unconjugated
bilirubin will increase.
2) Drugs (sulfunamides)Gt ut .

3) Acidosis, 4r ,o','"""-"o' l
4) Increased FFA (secondary to hypoglycemia, Oisllace bilirubin from its binding
starvation or hypothermia)Gr t'tt /I site on albumin
5) Asphyxiqal ta)
6) Hyperosmolarity\tt4 )t
Make neurones more susceptible lo
7) Prematurity{At u) I
toxic effec1 ofunconjugate bilirubin
8) Infection J
IREATM ENT OF HYPERBILIRUBINEMIA

. Treatment options of hyperbilirubinemia are :-

A) Drugs (Pharmacological therapy)


r Drugs used for hlperbilirubinemia are :-
1) Barbiturates?Gt06'02) (Phenobarbitone):Indtce UDPG transferase and enhance conjugation of bilirubin.
2) Metalloporphyrins (Tin : Sn and Zinc : Zn) ': Decrease bilirubin formation by inhibiting heme oxygenase.
3) Miscellaneous: Frequent milkfeeding charcol and agar prevent reabsorption of bilirubin from gut.
B) Phototherapy
o Phototherapy has emerged as the most widely used form of treatment for unconiugated hwerbilirubinemia.
c Phototherapy converts unconjugated bilirubin into isomers that are able to bypass the conjugating system of
liver and are excreted in the bile or urine.
o Three types of photochemical reactions occtr; in decreasing order of importance :-
1 . Structural isomerization@IlMs 0s)
r Bilirubin is converted into lumirubin, which is excreted in the bile and urine.
2. Photoisomerization
r Less toxic, polar isomers are formed which are excreted in bile. But, the photoisomer can revert back to
unconjugated bilirubin and get reabsorbed from the gut ifthe baby is not having stools.
3. Photo-oxidation
r It converts bilirubin into small polar products that are excreted in urine.
t "The most effective lights for phototherapy are those with high energy output near the maximum absorption
peak of bilirubin (450 to 460 nm). Special blue lamps with a peak output of 425 to 475 nm are the most eficient
for phototherapy."
r Complications of phototherapy are :-
1) Bronze baby syndrome@Gle8). 4) Purpuric rabh with transient porphyrinemia.
2) Loose stool. 5) lncrease environmental and body temperatureql es).

3) Retinal damage(retss). 6) Erythematous macular rash.


C) Exchange transfusion
o Exchange transfusion is indicated when bilirubin level is high and chances of brain damage are significant.

lndications to start phototherapy and exchange transfusion


1) Accordingto gestational age
o It is usually used for premature newborn.

11-14

30-31 8-1 0 t 3-'t6

34 12-14 17-19

2) According to gestational weight (at birth)


o It is used when age is not available.
1 00i-1 500 g 7-10

2001-2so0 g
10-12

3) According to age after birth


o It is used for full term (mature) neonates

,F.hii,fe&rii1y.
24 - 48hrs >15 >20
48,-72hrs:,. ,)18 >25
> 72 hrs{AEMs ee)
> 20@aM3se)
>25

NEONATAT HYPOGLYCEMIA

o Hlpoglycemia in a neonate is defined as blood sugar less


than < 40 mg/dl{cutofiot;.
o According to serum level h}polycemia is
defined as serum glucose level < 35 mg/dl between
1-3 hours of life; < a0
mg/dl between 3-24 hours of life antt < 45 mg/cll
thereafter. After neoratal period (especialy
afier 2 months@NB t2))
hypoglycemia is defined as serum glucose < 54 mg/dl{oNn u;.
t is the most common metabolic problem in
newborns. Important causes of neonatal hypoglycemia
]ruoslrcemia
A) Due to inadequate substrate enzyme
function : prematurity(euusoz),small for gestational age (SGA), smaller of
twrns, respiratory distress syndromeallMs 021 and infant
of toxeinic mother.
B) Due to hyperinsulinism : Infant of diabetic motherLllMso')
(large for dateurrMse2)),erythroblastosis
fetalis (Rft
incompatibilityqnus szt',, and perinatal asphyxis(.arus oz, sz))
.

Treatment of neonatal hypoglycemia


1) Asymptomatic with blood glucose between 2O_40 mg/dl
r Direct breast feeding is started' If neonate is unable
to suck, expressed breast milk is given. Expressed
milk should be fortified by 59 sugar per 100 ml of milk. breast

2) Symptomatic or asymptomatic with blood glucose


< 20 mg/dl
r Bolus l0% dextroseul l'))
2 mllkgis given IV Followed by continuous infusion
of 6 mg/kg/minute. If normogly,
cemia is not achieved within 24 hours, glucocorticoids
(prednisone or hydrocortisone) should
be administered.
For intractable hlpogrycemia, glucqgon, epinephrine
or diazoxide can be given.
r In hypoglycemic seizures, dose of l}o/odextrose is
4 ml/ftgrarrst

Dertrose in concentration > l2.So/o is never used


through peripheral linestiElt) as il
can cause thrombophresnis. centrar Iine is
prererre d for > r2.50/odextrose.

I
n;;;;';;i,*;*i
Neonate of diabetic rnother
r Neonates of diabetic mother have following probiems :-
1) Hyp o gly cemi aGI
t[s 06' I'Gr 07 ) (non-ketotic) (ArrMs e8' Ar e5r.
6) Polyclthemia.

2) Macrosomia(vcto:) (large for date6"Mser) )' 7) Transienttachypneaofnewborn.

3) Hyperbilirubinemia\iltts06'PGt07)' 8) CardiomegalY.

4) HYPocalcemiaGIIMS06'|'jGI07)' 9) Asymmetric septal hypertrophy.

5) HyPomagnesemia. 10) Increased risk of RDS'


07 )
r Neonates of diabetic mother are at increased risk for fetal malformatiofl(Pct
!: i:!:: :::::nl:;: li;::+::J+:,:r i+i:.:i : jiai.

:*E-.i.ri:
Cardiac malformations
c Ventricular or atrial septal defect
a Double-outlet right ventricle
o TransPosition of great vessels
* Coarctation ofthe Aorta.
c Truncus arteriosus
Congenital anomolies other than <ardiovascular system
* Neurat tubl fl2fg6l5tearazt
o Situs inversus
e HydronePhrosis
c Double ureter
s Renal agenesis and.DYsPlasia
r HoloprosencePhalY

a Caudal r,egressian syndrame {specifico'a6)}


rr'ArrMs0e)) and
(most common cause@I
Two most common causes of seizures in these neonates are hypogtycemia
t1' AtIMs0e))'
hypocalcemia (2d most common causeal
DMtAltuses.
A"s) in futrtre.
o These infants are at increased risk ofdeveloping obesity(tttuset'A1e5r and

MISCELLANEOUS

Ferinatal asPhYxia
(hlpoxia) and/or a lack of perfusion
o perinatal asphlxia is an insult to the fetus or newborn due to lack of oxygen
(ischemia). Effects of perinatal asphyxia are :-
l) CNS : Hlpoxic ischmeic encephaiopathy, infarction, cerbral edema, seizures\"Mso'), hypotonia ot hypertonia
(generalized, involving all muscles simultaneouslyallMs03)), altereil sensorium(^"Mso')'
2) CVS : Myocardial ischemia, tricuspid insufficiency' hypotension'
3) Pulmonary : Pulmonary hypertension, pulmonary hemorrhage' RDS'
4) Renal: Acute tubular or cortical necrosis '
5) Adrenal : Adrenal hemorrhage'
6) Metabolic: SIADH, Hyponatremia, Hypoglycemia, Hypocalcemia, Myoglobinuria'
7) Integument : Subcutaneous fat necrosis'
8) HematologY: DIC.
g) Gastrointestinal : Perforation, ulceration with hemorrhage, Necrosis.
CneprsR. 3 Hewbornlnfant. .-..r. 1,,. .,,,.. ;: .. '

Hypoxic ischemic encephalopathy


e Encephalopathy progress over time _
1) Birth to 12 hours -) Decreased level of conciousness, poor tone,
decreased spontaneous movement, periodic
breathing or apnea, seizuresarlMs 10).
2) 12-24 hours -) More seizuers, Apneic spells, jitteriness, weakness.
3) --) Hvpotonia, J conciousness' poor feeding, brainstem
signs (oculomotor) and pupiuary
{:r:r::^:::rs
o Hypotonia is generalized, involves both limbs
and trunk and all muscles simultaneou
sly,trusto).
Causes of respiratory depression in neonates
r Intrapartum asphixia (most common cause).
c Drugs :- Norcotic analgesics (morphine or other opioids(ArrMs10.Aro7)), anaesthetics.
o Sepsis
o Prematurity:- CNS immaturity, surfactant deficiency.
r Respiratory problems :- Diaphragmatic hernia, obstructive lesions.
r CNS abnormalities :- Malformation, trauma.
r Muscle disorders :- Myopathy, prematurity.
Erythroblastosis fetalis
r Erlthroblastosis fetalis is caused by the transplacental passage
of maternal antibody active against paternal RBC
antigens ofthe infant and is characterizedby an increased
rate ofRBC destruction.
o Although more than 60 different RBC antigens
are capable of eliciting an antibody response,
significant disease is
associated primarily D antigen of Rh group{AIIMSeT)
andwith ABo incompatibil ilyL+ilusez).
r Other rare antigens involved are _
z C or E antigen of Rh grosp{.,,nusszt
r RBC antigens - Cw, Cx, Cu, K (kell), M, Duffy, S, p, MNS, Xg, Lutheran,
Diego and Kidd.
r Anti_Lewis antibodies do not cause diseaseUrrMseT).

Hydrops fetalis
r Hydrops fetalis (fetal hydrops) is a serious fetal condition
characterized by abnormal accumulation of
more fetal compartments, including ascites, pleural effusion,
fluid in 2 or
pericardial effusion and skin edema.
o Important causes of hydrops are :_
1) Immune: Rh incompatability.
2) Non-immune: Twin-Twin transfusion, chorangioma ofplacenta, congenitalheartblockrcone(t0e),
0e)' cystichygroln4@omert
mediastinal teratoma, congenital infections (TORCH)
and. congenital ,ephrosi5ao-"aos).
Microcephaly
e Microcephaly is defined as a head circumference (occipitofrontal
circumference) that meastnes more than three
standard deviation below the meAn@NB 12) for age and sex.
r Important causes of microcephaly are :_
A) Primary (genetic)
i) Isolated: Autosomal recessive, autosomar dominant,
x-rinkbd.
ii) syndrome: Down syndrome (trisomy 21), Edward
syndrome (trisomy lg), patau syndrome (trisomy
cri-du-chat syndrome.
l3),

B) Secondary
i) Structurar defects: Neurar tube defects (anencephary,
encephalocele).
ii) Metabolic disorders: RhenylketonuriaarMse4),citrullinemia,
methylmalonic aciduria.
iii) congenital infections i RubeilqLrrMss4), cMy,HS{ toxoplasmosis, syphiris, varicelra.
iv) Teratogens: Alcohol, tobacco, cocaine, heroin.
v) others: Maternal diabetes, maternal phenylketonuria,
hypothy,oidism, hlpopituitrism, adrenal insufficiency.
C ria p t,g R :,s.:' N€nt.Or

Congenital infections causing microcephaly with intracerebral calcification


o Following congenitai infections can cause microcephaly, intracerebral calcification and hydrocephalus :

(i) Commonlyby toxoplasmosis and CMV and (ii) Less commoniy by HSV and rubella.

Hydrocephalus ++ ++ +/- (Rare)

Micibcephaly,. ':'. ++r.'l +++ r:+i

lntracranial ++ +++ +\-


edeifeatioh.. {C6rnmon) {eomman)',..:.,'(Unc0mmon} : (Rare)

o Hydrocephalus with intracerebral calcificationin a neonate with history of 'spiramycin' treatment of mother in
pregnancy suggest the diagnosis of congenital toxoplasmosis(AlrMs 11' At 0e) .

Diagnosis in the figure below is:

a) Macrocaphaly
b) Microcephaly
c) Dolchocaphaly
d) Plagiocephaly

Ans. is'H i.e., Microcephatry


r Head circumference in the given figure is 13- l4 cm (normal overage head circumference at birth is about
35 cm) + Diagnosis is microcephaly.
Hidrocephalus is the most important cause. Macrocephaly is associated with frontal bossing dolicocephalic
r shape, narrowing of biparetal diamete and a square shaped face.

Macrocephaly
o Macrocephaly is defined as an occitrtito frontal circumference greater than two standard (SD) above the mean@I
is) for age and sex (Note : Microcephaly is three standard deviation below the mean).
r Important causes are :-
l) Syndromes : Fragile-X syndrome, and neurocutaneous syndromes (Neurofibromatosis), tuberous scierosis, Sturge-
Weber.
ii) Increased CSF : Hydrocephalus, choroid plexus papillom6l{Atss).
111) Bone disease: Achondropiasia, osteogenesis imperfecta, osteopetrosis.
iv) Others: AV malformation, intracranial hemorrhage (subdural, epidural, subarachnoid), Thalassemia major,
, hlpervitaminosis-A, lead poisoning, pseudotumor cerebri, galactosemia, Canavan's leukodystrophy(AflMs08).

The diagnosis in the given figure isr

a) Microcephaly
b) Macrocephaly
c) Plagiocephaly
d) Dolichocephaly

Ans, is'H i"e., Maerocephaly


r The figure in question is showing large head with frontal bossing and square shaped face + features of
macrocephaly.

Fetalalcohol syndrome \
r High level of alcohol ingestion in pregnancy can cause damage to fetus, known a s fetal alcohol syndrome. The harmful
effects may be due to alcohol itself or due to one of its breakdown products.
r Some evidence suggests that alcclhol may impair placental transfer of essential amino acids and zinc, both necessary
for protein synthesis, which may account for IUGR.
r Characterististics of fetal alcohol syndrome include : -
1) I(JGRarIMsoe) 5) Facial abnormalities
Cnaptrn' 3''Newborn Infant
2) MicrocePhalyGrrMsoe) 6) Minor joint anomalies
3) Congenital heart defects (ASD, V51l/utus ont
7) Hlperkinetic movements
4) MentalretardationattMsoe)

l"larNeqr.rin eoNon chanEe


o This rare but dramatic vascular evenf occurs in immediate newborn period and is most common in low birth weight
infants. If reJlects an imbalance in the autonomic yascular regulatory mssl,rsni5fll1il,netss).
o When the infant is placed on the side, the body is bisected longitudinally into pale upper half and a deep red dependent
half.

Note: Ihe fefus in ichthyosis appears grostesque and the appearance is called horleguin fefus. But, it is different from Harlequin color change.

Cephalohennatoma
o Cepholhematoma is a subperiosteal hemorrhage usually involving parietal and temporal bones. It appears within
12-24 hours. It is more common in forceps delivery, vaccum extraction and prolonged labor. It is soft and fluctuant
swelling with well defined margins.
o A cepholhematoma crossing the midline indicates underlying fracture of the skuli -+
fracture is linear not clepressed.
a It disappears between 2 week to 3 months(uPsceT).

The diagnosis in the given figure is:

a) Macrocephaly
b) Microcephaly
c) Cephalohematoma
d) None rr

Ans. is ?' i.e., Cephalolleruatorna


o Swelling not crossing suture line with distinct margins suggests the diagnosis of cephalohematoma.

& Neonatal period extends upto : First 28 days of life.

murmur.
o Not a finding in a normal newborn : Central cyanosis (there is peripheral cyanosis).

* Treotmmt'ofvaginal bleedinqina4daysoldneoncte:Notreatment(it.isminorclinical'problembetween3-Tneonatal daysl


* Common sites of mongolian spots: Presacral area (lower back & buttocks), legs, shoulder, posterior thighs.
a, Tteatment of hymen tog in'a neondfe: No treatment (it is a normal finding), .

s Newborn babies are able to breath and suck at the same time because of : High position of larynx.
x Reflexes pre*enf qf &rth :rRootinglsucking/swallowing reflexes, crossed extensor, moro! reflex, Assymelrlc tonic neckrefiex, graspreflex.
a Reflexes appear after birth: Symmetric tonic neck reflex, parachute reflex, landau reflex.
x' Parachute reflex disappsars af : Never iit remains throughout:life). . ,

* Persistence of moro's reflex is abnormal beyond the age of :6 month (morot reflex disappears at 6 months).
*',Refle*wltichfieve.r.recppears:Morosreflex .' , , ', ' .,

& Persistence of moro's reflex beyond 6 months indicates: Dysfunction of CNS (e.g. brain damage).
* unilatera! morob re{lex is seen in Erb's patly {Cu-Cu damage}, spastic hemiplegia,tracture of humerus or'clavicle, shoulderdislocation.
r

* Not a mechanism of heat production in a neonote:shivering (neonates respond bytnon-shivering thermogenesis).


*.Smal|fardotebobyis:Birthweightlessthan10thpercentile'..
a Low birth weight baby :Birth weight < Z-.1=OO Om irrespective of gestational age at birth.
& txtremely law birth weighitbcby: Birth wEigftt <t OOOgm (< 1kg) irrespective of gestational age at birth. l

e Not a cause of IUGR: Maternal diabetes.


6 Treatmentfor birth asphyxia includes:Glucose, calcium gluconate, normal saline, oxygen, anticonvulsants (but-not corticosteroids).
& Bag and maskventilation is contraindicated in: Diaphragmatic hernia,TE fistula, meconium aspiration syndrome
@ Dose of adrenaline in neonatal resuscitation:0.1-0.3mg/kg diluted 1:10,000.
w APGAR score components are: Respiratory effort, heart rate, color of the body, muscle tone, reflex stimulation (grimace).
b Not a component of APGAR score : Respiratory rate
Cur prrn 3 Newborn lnfant

& One gram of hemoglobin liberates: 35 mg of bilirubin.


& lmportant congenital causes of conjugated hyperbilirubinemia: Rotor syndrome, Dubin-lohnson
syndrome.
& lmpartant cangenitol causes of unconjugated hyperbitiribinemia:Crigler-Najjar syndrome,
Rotor syndrome. :

& Jaundice appearing within 24 hours of birth: Erythroblastosis fetalis (most common cause), infections, G6pD deficiency, hereditary
spherocytosis.
& PeripheralsmeqrofneonatewithABahemolyticdiseaseshows:Microspherocyte5.
& Drug which inqeases the risk of kernicterus in a child with hyperbitirubinemia : Sulfonamide.
* Not a late feature of kernicterus: Hypotonia (it is an early feature).
@ lmportant late features of kernikterus: Choreoathetosis, sensorineural deafness, upward gaze palsy,
mental retardation.
& Kernicterus is due to:High unconjugated bilirubin (not conjugated bilirubin).
@ Drugs used in treatment of hyperbilirubinemia/kernicterus: Barbiturates, metalloporphyrins (tin, zinc).
* Mechonism mainly resposnsible for reduction of bilirubin by phototherapy:5tructural isomerization.
@ Bronz-baby syndrome is due to: Phototherapy
* A child with hyperbilirubinemia, parameters measured should be:Total bilirubin and direct (conjugated) bilirubin.
& Not a cause of neonatal hypoglycemia: post-terminfant
@ lmportant causes of neonatal hypoglycemia: Prematurity,
RDS, maternal diabetes, asphp<ia, erythroblastosis (Rh incompatibility).
& Symptomatic neonatal hypoglycema should be managed by: IOyo dextrose.
& lmportant complications of neonate of diabetic mother: Hyperbilirubinemia, hypocalcemia, hypomagnesemia, polycythemia,
HMD,
TTN, CHD.
@ Not a complication of neonate of diabetic mother:Hyperglycamia (there is hypoglycemia).
@ Most specific fetal malformation in neonate of diabetic mother:Caudal regression syndrome.
& Type of hypoglycemia in neonate of diabetic mother: Nonketotic hypoglycemia.
& Most common cause of seizures in neonates of diabetic mother : Hypog lycemia (most common) followed by hypocalcemia.
& Maximum concentration of dextrose that can be given to neonate through periphera! lines:12.5o/o.
& Neonatal hyperglycemia is defined as: Blood glucose > 1 25mg/dl orlplasma glucose > 1 5gmg/dl.
& Neonatal hypogtycemia is defined as:Blood glocose < 4omgld l; or seru m g lucose < 35 mgldl between -3
1 hou rs of life, serum glucose
< 40 mg/dl between 3-24 hours of life, serum g*ucose < 45 mg/dl after 24 hours of life.
& Hypogtycemia in infant> 2 month o/d: Serum glucose < 54 mg/dl.
& Resolution of physiological umbilical hernia occurs at:1Oth week of gestation.
& lmportant drug causing neonatal respiratory depression: Opioids (morphine).
& Tests used to differentiate maternal from fetal btood : Alkali denaturation {Apt. test), and acid denaturation (kleihouer
betke) test.
& Causesofvomitinganfirstdayoflife:Aerophagy;esophagealatresia,faultyfeedingtechnique, amnioticfluidgastritts.
@ Best way to monitor neanate's breathing and apnea in incubator for preterm nonventilated baby : lmpedence technique.
a lnflation pressure required for first'inflation in a neonate : 25-40 cm H for > 1 .5 seconds.
,o
Organs palpable in a normal neonate: Liver, kidney and spleen.
& Common rashes in a newborn: Erythema toxicum (most common), acne neonatorum, transient pustular melanosis.
& M.C cause of respiratory distress in preterm neonate: Hyaline membrane disease.
& MC cause of respiratory distress in term or post-term neonate: Meconium aspiration syndrome.
& Heart rate in a normal neonate: l ZO-l40 per minute.
I Respiratory rate in a normal neonate:35-4ber miiute.
& Best method for transport of newborn with maintainance of temperature: Kangaroo Mother Care (KMC).

xxx
QUESTIONS

NORMAL NEWBORN a) 3 months b) 6 months


c) 9 months d) Never
Neonatal period extends upto- (NEET Dec.12 Pattern) 12. Child of 9 months, which reflex is most abnormal -
a) 21 days oflife b) 30 days of life (PGI lune 98)
c) 28 days oflife d) 35 days oflife a) Asymmetric neck reflex
Normal finding in a newborn - (All India Dec.1j Pattern) b) Parachute reflex
a) Length 30 cm c) Righting reflex
b) Peripheral cyanosis d) None
c) Central cyanosis 13. A baby on examination shows unilateral moro's
d) Extension ofbody reflex with positive palmar grasp reflex. The site of
3. Meeonium can passed upto ----- days in healthy lesion is - (AIIMS Nov 99)
bady- (Atl India DeclS Pattern) a) Cr- Cn b) C-C
a)l b)3 c) Cr- T, d) C,- C,
c) s d)7 L4. Atypical moro's reflex is seen in A/E - (PGI Dec e7)

The apropriate approach to a neonate presenting a) # clavicle


with vaginal bleeding on day 4 of life is - b) Sternomastoid tumor
(CET Aug.12 Pattern, Al 05) c) Shoulderdislocation
a) Administration of vitamin K d) Brachial plexus injury
b) Investigation for bleeding disorderl2 15. Persistence of Moro's reflex is abnormal beyond the
c) No specific therapy age af - (All lndia Dec. 15 Pattern)
d) Administration of i0 ml/kg of fresh frozen plasma a) 3rd month b) 4th month
over 4 hours c) 5th month d) 6th month
5. Common sites for mongolian spot are- (PGI Nov 15) 16. Reflex which never reappears is - (PGl lune 98)
a) Face b) Neck a) Moro's b) Grasp
c) Lumbosacral'area d) Leg c) Snout d) Glabellar
e) Thigh
6. A hymenal tag in a newborn is best treated by - HYPOTHERMIA
(All lndia Dec.14 Pattern)
a) Steroids b) Surgery 17. All the following can occur in a neonate for heat
c) Leaving it alone d) None ofthe above production except - (AIIMS Nov 11, Nov 06)

7. New born babies are able to breathe and suck at the a) Shivering
same time due to - (AIIMS Nov 10) b) Breakdown of brownfat with adrenaline secretion
a) Wide short tongue b) Short soft palate c) Universal flexion like a fetus
c) High larynx d) Short pharynx d) Cutaneous vasoconstriction
18. Following features may be seen in cold injury of
PRIMITIVE RELFEXES neonates except - (Ar ee)
a) Bradycardia
8. Which of the following reflexes is not present at b) Uncontrolled shivering
birth- (AilMS May 07, At 07) c) Scleroma
a) Asymmetric Tonic Neck Reflex d) Metabolic acidosis
b) Morot Reflex
c) Symmetric tonic neck reflex LOW BIRTH WEIGHT
d) Crossed extensor reflex
9. Swallowing breathing reflex - not seen in fetus for - 19. Small for date baby is - (All India Dec. 11 Pattern)
(All lndia Dec15 Pattern) a) < 10 percentile for the gestational age
a) 14 weeks b) < 50 percentile for gestational age
b) 12 weeks c) < 2000 gm
c) 16 weeks d) < 2500 gm
d) Appear in all above period '
20" Extremely low birth weight baby-
10. Persistant Moro's reflex at 6-7 months indicates - t' (All India Dec.lj Pattern)
(PGI 02, DPG oe) a) < 2.5 kg b) <2kg
a) Normal child b) Brain damage c) < 1.5 kg d) <lkg
c) Hungry child d) Irritable child ,1 IUGR is caused by all except - (NEET Dec.12 Pattern)

1I. Parachute reflex disappear at- (All lndia Dec.13 Pattern) a) Diabetes b) Alcohol
.r;.,;. .,t ::.:
''
:iii{dit
smokrng
c) Smoking
cJ d)
Chronic renal failure c) 36
22. d) 72
For a term small for date baby, true is _ (ArrMS May 9s)
31. Most commoa cause of sepsis in Indiawithin 2
a) No nipple nodule
b) No palmar/plantar crease
months- (AIIMS Nov 09)
a) H influenza
c) Weight less than lOth percentile b) E.coli
d) Hperbilirubinemia
c) Coaguiase positive staph aureus
23. All of the fr:Ilnwing are features of prernaturity in a d) Group B streptococcus
neonate, except - (All India Dec. 15 Pattern) 32.
A 7 day old iafant develops syrnptoms of neoaatal
a) No creases on sole
septicemia. Most likely cause is _ (Ar
b) Abundantlanugo s8)
a) Local nursery environment
c) Thick ear cartilage
d) Empty scrotum
b) Infection through umblical cord
c) Exclusivelybreast fed baby
24. A premature infant is more likelythan a full term d) Infection by GIT bacteria
infant to - (AilMS 80, pG Bs) JJ. Most corn:aon caus€ of Ne{rnatal sepsis in hospital
a) Suffer fromjaundice ofhepatic origin
in India is ^
b) Maintain in normal body temperature in a cold @rrMS May 07)
environment
a) Escherichia coli

c) Excrete urine with a uniform specific gravity b) Klebsiella

d) Suffer from anaemia


c) Staph aureus
d) Listeria monoc)togen
25. Full 1erm, Srnsll for Date Babies are at high risk of_
34. In neonate with meningitis, grarn positiye cocci,
(Ar 2000)
CAMP test positive most comm{rn source of
a) Hypoglycemia
infection is -
b) Intraventricularhaemorrhage (All India Decl5 Pattern)

c) Bronchopulomonarydysplasia a) Genital tract


d) Hyperthermia b) Respiratorytract
c) Hematogenous
25. True staternent about IUGR is _ (Ar ee) d) May be any
a) Hepatomegaly is due to fatty infiltration
35. Least obseryed laboratory finding in Neonatal Sepsis
b) Head circumference is 3 cm more than chest
circumference
is- (ai 96)
c) Hyaline membrane disease is
a) t C-reactive proteln
a common cause of b) Neutrophilia
death
d) Hypothermia does not occur due to good
c) Increased ESR
d) Increased Immature Neutrophils
shivering mechanisms
35. Neonatal septicemia is most commonly caused by
27 " In asymmetrical IUGR which organ is not _

affected - (NEET Dec.12 Pattern)


(CET Nov. lt pattern)

a) Subcutaneous fat a) Group B Streptococci


b) E.coli
b) Muscle
c) Liver c) Streptococcusviridans
d) Staphyloceccus aureus
d) Brain
37. Neonatal sepsis and meaingitis most common
28. What is the ponderal index of a child with werght cause-
2000 g and he:ght 50 crn - (ArrMS Nov 10) a) Streptococcuspyogenes
@G198)
a) 1.6 b) 3.6
c) 2.2 b) Streptococcus agalactacea
d) z.e c) Enterococcusfecalis
29, A aeonate weighing 1500 grams is delivered at 33 d) Staphylococcus aureus
weelrs. Which of the following would tre most appro_
38. Most common cause of neonatal meningitis
priate method of nutrition for the baby? _
fi ul (All India Dec.14 pattern)
a) IV fluids and oral feeding
b) Orogastric feeding/alternate oral feeding a) Staphylococcus

c) Total parenteral b) E. coli


nutrition
d) IV fluids and assessment/follow_up c) H. influenze
d
d) Pneumococcus
39. Eommunity acquired Neonatal pneumonia treat_
I{EONATAL 5EP5IS ment of choice - @IIMS Feb 97)
a) Cefotaxime + Amikacin
30. Early neonatatr sepsis occurs within &ours) _
b) Ampicillin + Chloramphenicol
(CET luly t5 pattern) c) Ampicillin + Gentamicin
a)8 b) rz d) Metronidazole + Amikacin
c) Smoking d) Chronic renal failure c) 36 d) 72
22. For a tean emall for date babS true is - (ArrMS May s3) 31" Most common cause of sepsis ia India rryithin 2
a) No nipple nodule moaths- (AIIMS Nov 09)
b) No palmar/plantar crease a) H influenza
c) Weight less than 1Oth percentile b) E.coli
d) Hyperbilirubinemia c) Coaguiase positive staph aureus
23. All of the following are features of prematurityin a d) Group B streptococcus
neo'I1ate, exc€pt - (All India Dec. 15 pattern) 32" A 7 day old iafant develnps symptoms of neonatal
a) No creases on sole eepticemia" Most likely cause is - (Ar e8)
b) Abundantlanugo a) Local nursery environment
c) Thick ear cartilage b) Infection through umblical cord
d) Empty scrotum c) Exclusivelybreast fed baby
24. A premature infant is more likely than a full term d) Infection by GIT bacteria
infani to - (ArrMS 80, PG 8s) 33. lVlost common cause of Neoaatal sepsis in hospital
a) Suffer fromjaundice ofhepatic origin in India is - (AilMS May 07)
b) Maintain in normal body temperature in a cold a) Escherichia coli
environment b) Klebsiella
c) Excrete urine with a uniform specific gravity c) Staph aureus
d) Suffer from anaemia d) Listeriamonoc)togen
25. Full terrn, Small f,or Date Babies are at high risk of- 34- IR neonate with meningitis, grarn positive cocci,
(Ar 2000) CAMP test positive mo$t common source of
a) Hypoglycemia infection is - (All India Dec15 Pattern)
b) Intraventricularhaemorrhage a) Geniial tract
c) Bronchopulomonarydysplasia b) Respiratory tract
d) Hlperthermia c) Hematogenous
26, True statemeat about IUGR is - (Ar 9s) d) May be any
a) Hepatomegaly is due to fatty infiltration 35. Least obseryed laboratory finfing in Neonatal Sepsis
b) Head circumference is 3 cm more than chest is- (ai s6)
circumference a) t C-reactive protein
c) Hyaline membrane disease is a common cause of b) Neutrophilia
death c) Increased ESR
d) Hlpoihermia does not occur due to good d) Increased Immature Neutrophils
shivering mechanisms 36. Neonatal septicemia is most commonly caused by -
27. In asymmetrical IUGR which organ is not (CET Nov. 14 Pattern)
afiected - (NEET Dec.12 Pattern) a) Group B Streptococci
a) Subcutaneous fat b) E.coli
b) Muscle c) Streptococcus viridans
c) Liver d) Staphyloceccus aureus
d) Brain
5/- Neonatal sepsis and meningitis most common
28. lflhat is the ponderal index of a child with weighl cause- (PGI?B)
2000 g and height 50 cm - (AIIMS Nov 10) a) Streptococcus pyogenes
a) 1.6 b) 3.6 b) Streptococcus agalactacea
c) 2.2 d) 2.6 c) Enterococcusfecalis
29. A aeonate weighins 1500 grams is delivered at 33 d) Staphylococcusaureus
weeliis. Which of the followiag would be most appro- 38. Mnst cornmon cause of neonatal meningitis -
priate method of nutrition for the baby? (Ar 11) (All India Dec.14 Pattern)
a) IV fluids and oral feeding a) Staphylococcus
b) Orogastric feeding/alternate oral feeding b) E. coli
c) Totai parenteral nutrition c) H. influenze
d) IV fluids and assessment/follow-up d) Pneumococcus
?
39- pommunity acquired Neonatal Pneumonia treat-
NEONATAL SEPSIS ment of choice - (AtrMS Feb s7)
a) Celotaxime + Amikacin
30. [arly neonatal sepsis occurs within (hour) - b) Ampicillin + Chloramphenicol
(CET luLy 15 Pattern) c) Ampicillin + Gentamicin
a)8 b) 12 d) Metronidazole + Amikacin
RESPIRATORY DISTRESS
Most probahle diagnosis is - (CET luly 15 Pattern)

a) HMD
40. \{hich of the following d*es not indicates respirato* b) Diphragmatic hernia
neonate? (CETNou' 12Pattern) c) Pneumothorax
ry distress in
a) Wheeze d) Meconium aspiration syndrome

b) Grunt 48. A new born baby has been referred to the casualty as
c) Retraction a case of congenital diaphragmatic hernia. The first
d) Tachypnea clinical intervention is to - @IIMS May 03)

With a) Insert a central venous pressure line.


41. [n a child cessation of breathing for 2O sec'
b) Bag and mask ventilation
bradycardia is - (NEET Dec'12 Pattern)

a) Apnea c) Insert a nasogastric tube


b) Dyspnea d) Ventilate with high frequency ventilator
c) Cheyne stokes resPiration 49. A child presented with respiratory distress was
d) None brought to emergencywith bag and maskventi-
lation. Now child is intubated. Chest X-ray shows
42. Apnea of prernafurity of lasts fot ? Tcnr eug' 12 Pattern)
right- sided deviation of mediastinum with scaphoid
a) 10 sec b) 15 sec
atrdomen. His Pulse rate is increased. What is the
c) 20 sec d) 30 sec
(AltMSNov 07)
next step ?
a) Endotrachealintubation
MECONIUM ASPIRATION SYNDROME b) Put a nasogastric tube
c) Surgery
43. Meconium passage in utero leads to which of the d) Endtidal CO, to confirm intubation
following - (NEET Dec.12 Patern)
50. 2 days after birth, child developed respiratory
a) Listeriosis
distress and had scaphoid abdomen. Breath sounds
b) Obstructive emPhYsema
were decreased on the left side. After bag and mask
c) Pathological jaundice
ventilation, ET tube was put and the maximal car&-
d) Meconium ileus
ac impulse shifted to the right side. What should be
44. A 3 kg term baby delivered by caesarian section the next step in management ? (AIIMS Nov 07)
develops respiratory distress soon after birth. The a) Confirm the position of endotracheal tube
liquor was meconium stained. Breathing rate is 90/ b) Emergency surgery
min. correct statements - @GI Dec 04)
c) Naso gastric tube insertion
a) Transient tachlpnoea ofnewborn d) Chestx-ray
b) Meconium asPiration sYndrome
c) Reticulonodular shadows in X-ray chest 51. A neonate having congenital fiaphragmatic hernia
developed respiratory distress. Breath sounds were
d) I Surfactant Production
decreased on the left side. After bag and mask ven-
e) Oral feeding started earlY
tilation, ET tube was put and the maximal cardiac
45. Post term baby with tachypnea - commonest cause- impulse shifted to the right side. What should be the
(All India Dec15 Pattern) (Ar 08)
next step in management -
a) Transient tachlPnea of newborn a) Confirm the position of endotracheal tube by x-ray
b) Meconium asPiration sYndrome chlst
c) Hyaline membrane disease b) Remove tube & reattempt intubation
d) Infection c) Naso gastric tube insertion
d) Chest x-ray
CONGENITAL DIAPHRAGMATIC H ERNIA 52. A neonate having congenital diaphragmatic hernia
developed respiratory distress. Breath sounds were
46. The cause of death in congenital diaphragmatic decreased on the left side. After bag and maskven-
hernia is - (CET lune 14 Pattern)
tilation, ET tube was put and the maximal cardiac
a) SePticaemia impulse shifted to the right side. What should be the
b) Hemorrhage next step in management - (Ar 08)
c) PuimonaryhypoPlasia a) Confirm the position of endotracheal tube by x-ray
d) Intestinalobstruction chest
47. New born term baby born by vaginal delivery had' p Emergency surgery
respiratory distress, glY"L1lt! .caphoid abdomen- c) Naso gastric tube insertion
d) Chest x-ray
53. Which of the following is the least important prog-
nostic factor in congenital diaphragmatic hernia?
(AllMSNov 11' Al 11)
a) PulmonaryHypertension
c) Ultra sound
b) Delay in emergent surgery
d) Amniocentesis
c) Size ofdefect
62. Which one of the following is true of hyaline
d) Gestational age at diagnosis
membrane disease of the newborn _
54. Most irnportant prognostic factor in congenital @psc ss)
a) Prematurity provides relative protection of the
diaphragmatic hernia - (Al 11, AIIMS Nov 09) occurrence
a) Pulmonaryhlpertension b) Maternal steroid exposure increase severity of the
b) Size ofhernia disease
c) Timing of surgery c) Phosphatidyl glycerol estimation is a reliable
d) Gestational age
method of
diagnosis
HYALINE MEMBRANE DISEASE d)
Surfactant increases the surface tension ofa lveoli
53. With reference to RDS, all of the following state_
55. Most important risk factor for neonatal respiratory ments are true, except - (Ar 02)
distress syndrome- (All India Dec. lj pattern) a) Usually occurs in infants born before 34 weeks of
a) Diabetic mother gestation
b) Asphyxia b) Is more common in babies born to diabetic
c) Prematurity mothers
d) Twin pregnancy c) Leads to cyanosis
56. Deficiency of surfactant is seen in - (pGI Dec 99) d) Is treated by administering 100% oxygen
a) BPD 64. A 1.5 kg child born at 32 weeks by LSCS presents
b) Neonatal asphlxia with moderate respiratory difficulty @R70/ minutes).
c) HMD Which of the following is the appropriate manage_
d) Anencephaly ment- (AIIMS Nov 10, 09)
57" The clinical sign ofhyaline rnernbrane disease
Sener_
A) CPAP
ally first appears - (All India Dec.14 pattern) b) Mechanicalventilation
a) In the first 6 hours of life c) Warm oxygen
b) Between 12 and24 hours of life d) Surfactant and ventilation
c) Between 36 and 48 hours of life 65. Allare true regarding administration of beta-meth_
d) After 48 hours of lilfe asone to a mother with premature delivery except_
58. A 3l week pregnant lady delivers a child. In 4 hrs (AilMS ee)
haby develops dyspnoea and cyanosis. X-ray chest a) Neonatal morbidity better
shows ground glass appearance. The diagnosis _ b) Helps lung maturity
(AITMS Nov 99) c) Prevents hyperbilirubinemia
a) Meconium Aspiration d) Decreases intraventricuiar hemorrhage
b) Hyaline membrane disease 66. The dose of betamethasone in prenatal to prevent
c) Tracheo esophageal atresia respiratory distress syndrome is - (NEET Dec.t2 pauern)
d) Esophageal atresia a) 6mg
59. A 3 hour old preterm baby develop respiratory b) 12 mg every 24 hours
grunt and X-ray shows reticulonodular pattern. c) 6 mg every 12 hours
Diagnosis is- (All India Dec15 Pattern) d) 4 mg start
a) Hyaline membrane disease 67. Hyaline membrane disease of lungs is characterized
b) Transient tachlpnea of newborn bY-
c) Meconium aspiration syndrome @rrMS Nov jo)
a) FRC is smaller than closing volume
d) All of above b) FRC is greatefthan closinivolume
60. A term gestation newborn developed respiratory c) FRC is equal to closing volume
distress. Which of the following would f*oo. Respi- d) FRC is independent of closing volume
ratory distress syndrome (HMD)?
(AIIMS May 12, Nov 10) BRONCHOPULMONARY DYSPLASIA
a) Historyofreceiving antenatal corticosteroids
b) Air bronchogram on chest x-ray 68. All of following are causes of bronchopulmonary
c) Onset of distress after 6 hrs of birth dysplasia except
d) Term birth
a) Oxygen toxicity
@r s6)

61. Foetal lung maturity is assessed by - b) Theophyline use


(NEET Dec.12 Pattern) ,6) Traumatic damage to lungs
a) L/S ratio 'd) Pulmonary odema due to capillary damage
b) Billirubin content of amniotic fluid 69. "d baby is bo'rn at 2Z weeksof gestation required me*
chanical ventilation for next 4 weeks & O2 for next a) Discontinue ventilation
1 week. He maintained at room air subsequently. As b) Continue ventilation
per new Bronchopulmonary dysplasia definition, he c) Give chest compression
has which of the following- (All India Dec.13 pauern) d) ET intubation
a) Miid BPD b) Moderate BPD 76. True regarding neonatal resuscitation- (pGtNov. t4)
c) Severe BPD d) NoBPD a) Ist nasal suctioning done
b) Ist mouth suctioning done
TRANSIENT TACHYPN EA OF N EWBORN c) Max. length of nasal suctioning is upto 3 cm and
mouth suctioning is upto 5 cm
70. Transient tachypnea of new born (TTN) is commonly d) Max. length of nasal suctioning is upto 5 cm and
seen in which of the following situations - mouth Suctioning upto 3 cm.
(AIIMS May 02) 77. A baby is born with rneconium stained liquor which
a) Term delivery requiring forceps of the following is taken account of in terming a
b) Term requiringventouse
baby vigorous except - (AIIMS Not 09)
c) Elective caesarean section a) Tone b) Colour
d) Normal vaginal delivery
c) HR d) Respiration
71. Characteristics radiological feature oftransient 78. A 5 year old child is rushed to casualty reportedly
tachypnoea of newborn is - (AIIMS May 0s) electrocuted while playing in a park. The child is
a) Reticulogranular appearance apneic and is ventilated with bag and a mask. Which
b) Low volume lungs of the following will be the next step in the manage-
c) Prominent horizontal fissure ment - (AIIMS Nov 01, Nov 05)
d) Air bronchogram a) Checkpulses
72. True about transient tachlpnoea ofnewborn is - b) Start chest compressions
(PGI Dec 99) c) Intubate
a) Air bronchogram seen d) Check oxygen saturation
b) Common in preterm infants 79. Bag and maskventilation is contraindicated in -
c) Interlobar fissure effusion
(AilMS Mav o4' lune ee)
d) Respiratory distress resolves in 6-10 days a) cleft lip
b) Choanal atresia
PULMONARY ALVEOLAR PROTEINOSIS c) Diaphragmatichernia
d) Multicentric bronchogenic cyst
73. 3.5 kg term male baby, born of uncomplicated
80. Which of the following is the principal mode of heat
pregnancy, developed, respiratory distress at birth,
exchange in an infant incubator ?
not responded to surfactant, ECHO finding revealed (All India Dec.13 Pattern, AI 06, PGI 98)
nothing abnormal, X-ray showed ground glass a) Radiation b) Evaporation
appearance and culture negative. Apgars 4 and 5 at
c) Convection d) Conduction
1 and 5 minutes. History of one month female sib-
ling died before. What is the diagnosis?
81. All of the following therapies may be required in a 1
(AIIMS May
hour-old infant with severe birth asphpia except-
11, May 08, Nov 11, Nov 08, Nov 07)
a) TAPVC (At 0s)

b) Meconium aspiration
a) Glucose
c) Neonatal pulmonary alveolar prciteinosis
b) Dexamethasone
d) Diffuse herpes simplex infection
c) Calcium gluconate
d) Normal saiine

RESUSCITATION
82. Grimace with APGAR score - (All rndia Dec15 pattern)
a)0 b)1
74. Meconium aspiration is done for
c)2 d)3
3 times but no
breathing occurs. Next step in resuscitation would 83. Which is not a component of APGAR Score -
be- (At e7) (All India Dec.14 Pattern)
a) Chest compression a) Colour ofthe body
b) Orinhalation $
b) Muscle tone
c) Bag & mask intubation c) Heart ratelminutes
d) Trikling of sole d) Respiratory rate per minute

75. A 3 hours old neonate with apnea is on bag and 84. Which of the following is not true about newborn
mask ventilation for last 30 seconds, now showing assessment - (AIIMSNov 11)

spontaneous breathing with heart rate 110/min. The a) APGAR at7 minindicates neonatal mortality
next step should be - (Ar e2)
depression
b) APGAR at 1 min, indicators for neonatal
Cnaprnn 3 Newborn lnfant

resuscitation c) Biliary atresia


c) Fetus can rapidly washout CO, through placenta d) Physiological
d) Anaerobic metabolism causes acidemia
93. Neonatal |aundice first time appe&r$ in the 2nd week
85. A newborn with respiratory distress with RR 86/ not a cause iS - (NEET Dec.12 pattern)
min, nasal flaring, auditlle gruntinS abdomen a) Galactossemia
lagged behind chest respiratory movement, no lower b) Rh Incompatibility
chest or xiphoid retraction. What is silvermans c) Hlpothyroidism
score - (AIIMSNov 10) d) Breast milk |aundice
a) I b)3 94. A term neonate with unconjugated hyperbili-ru-
c) s d)6 binemia of 18 mg/dl on 20th day. All are common
86. Drugused in neonatal resuscitation- causes except - (AIIMS May 07)
(All lndia Dec15 Pattern) a) Breast milk jaundice
a) Adrenaline b) Soda bi carbonate b) Congenital cholangiopathy
c) Naloxone d) Allofabove c) G 6PD deficiency
87" Dose of i.v. adrenaline in term infant is during d) Hlpothyroidism
neonatal resuscitation - (NEET Dec.12 pattern)
a) 0.1 - 0.3 ml/kgin l:1000 CONGENITAL HYPERBILIRUBINEMIA
b) 0.3 - 0.5 ml/kg in 1:1000
c) 0.1-0.3 ml/kg in 1:10,000 95. Defective hepatic conjugation is seen in all the
d) 0.3 - 0.5 ml/kg in 1:10,000 following except - ki s6)
a) Neonatal jaundice
HYPERBILIRUBINEM!A & NEONATAL JAUN. b) Gilbert syndrome
DICE
c) Crigler-Najjar syndrome
d) Novobiocin therapy
88. Hyperbilirubinemia in newborn involving palm & 96. All are true about Gilbert's syndrorne except -
sole- (All Irulia Dec15 pattern) (AILMS Dec 97)

a) 8 mg/dl a) Mild conjugated hlperbilirubinemia


b) 10mg/dl b) Autosomal dominant
c) 14mg/dl c) Normal liver histology
d) >15 mg/dl d) Almost normal liver function tests
89. Conjugated bilirubin is increased in 97 " Gilbert syndrome- True is - (All India Decl; pattern)
?
(CET Nov.13 Pattern)
a) Reduced activity ofglucuronyl transferase
a) Rotor syndrome
b) Causes indirect hlperbilirubinemia

b) Gilbert's syndrome
c) Not required any treatment
c) Criggler Najjar syndrome I d) All oTabove
d) Criggler Najjar syndrome 2
98. True about criggler najjar type-Il syndrome is -
(All India Dec. t4 Pattern)
90. A child has bilirubin of 4 mg. Conjugated bilirutrin
a) Diglucuronide deficiency
and alkaline phosphatase are normal, trile salts and
b) Dominant trait
trile in urine are atlsent. However urobilinogen in
urine is raised. What is the likely diagnosis -
c) Kernicterus is seen
d) Phenobarbitone not useful
@IIMSNov0l)
a) obstructivejaundice 99. A case of jaundince with 50% direct bilirubin, other
b) Rotor's syndrome LFTs normal. Diagnosis is - (ArrMS Nov 0e)
c) Biliary cholestasis a) Rotor syndrome
d) Hemolytic jaundice b) Gilbert syndrome
91. A 5-years old male child presentgwith episodic
c) Glucuronyl transferase deficiency
d) Primary biliary cirrhosis
anaemia and jaundice since birth. He is least likely
to have which of the following - (AIIMS Nov 11) 100. Unconjugated hyperbilirubinemia is seen in -
a) Hereditary spherocltosis (All India Dec.13 pattern)
b) Sickle cell anemia a) Rotor syndrome
c) PNH b) Dubin-fohnson syndrome
d) G-6-PD deficiency c) Biliary atresia
d) Crigler-Najjar syndrome
92. |aundice at birth or within 24 hours of birth is com-
monly due to - (Ar e5)
101. Causes of conjugated hyperbilirubinemia is -
a) Erythroblastosis (NEET Dec.12 pattern)

b) Congenital hlperbilirubinemia a) Rotor syndrome


b) Breast milk jaundice
; i Crigler najjar
d) Gilbert syndrome UjtiyUin to start phototherapy - (AIIMS June 99)
I0l. a) 20 mgo/o
-\Iildly elevated bilirubin, aormal liver enzymes b)
are 12.5 mgoTo
Seen in- (All India Dec15 pattern) c) I8 mgTo
a) Maiaria d) t5 mgo/o
b) Thalassemia
c) G-6 pD deficiency 1l l. A 30_weeks premature infants, 900gm
weight on
the third days. The serum bilirubin"is
d) All of above 13 mgyo.
The treatment of choice is_ (Ail India Dec. js pattern)
a) Exchangetransfusion
KERNICTERUS b) Phototherapy
c) Wait and watch therapy
I 03. False about kernicteres is (All India Decl| pattern) d) Pharmacologictherapy
a) No long term effect I12. What should be measured in a newborn
b) Occurs with bilirubin more than 25 who pres-
mg% ents with hyperbilirubinemia -
c) deposition in basal ganglion a) Total & Direct bilirubin @I 2000)
d) Opisthotonus b) Total bilirubin onlv
104. fu unconjugated hyperbilirubinemia, c) Direct bilirubin only
the risk of
kernicterus increases with the os. of - d) Conjugated bilirubin only
@I 0s)
a) Ceftriaxone
b) phenobarbitone
NEONATAT HYPOGLYCEMIA
c) Ampicillin
d) Sulphonamide f13. Serum glucose levels in children >
105. The late features
2 rnonths with
ofkernicterus include all except - hlpoglycaemia is ? (CET Aug. 12 pattern)
(All India Dec.13 pattern) a) <4}mgldL b) < 4s mgtdL
a) Hlpotonia c) < 50 mg/dL
b) Sensorineural d) < s+ mg/dl
hearing loss l14. All of the following groups of newborns
c) Choreoathetosis increased risk of hypoglycemia
are at an
d) Upward gaze palsy except _

f 06. All of the following statemeats regardiag jaundice


in a) Birth asphlxia
(AIIMS Nov 02)

a newtrorn are true, except - b) Respiratory distress syndrome


@112)
a) Physiological /aundice usually peaks after 4g hours c) Maternal diabetes
b]
!*1.1 milk jaundice usually peaks after day 7 d) Post term infant
c) High levels of conjugated biliiubin may
cause 115. Alargefor gestationalagebabydelivered
Kernincterus at40
weeks was otrserved to be lethargic.
d) All of the above are true The blood sugar
was measured to be 35 mg/dl.
The management is _
(Ar 12)
TREATMTNT OF HYPERBILIRUBINEMIA a) Fortified Breast Milk
b) 10% IV Dextrose
1O7. Drugs that can be used in kernicteru s_ (pGI c) Oral Glucose Solution
06, Dec 02)
a) Barbiturates , b) Benzodiazepines d) Normal Saline
c) Phenl.toin d) lChlorpromazine 116. Child comes with blood suger 32 mg/dl
e) Carbamazepine with conyul_
sioas Treatment is-
108. Which mechanism in phototherapy ltii naia Decjs pattern)
is chiefly re_ a) 5o/o dextrose 2mllkg
sponsible for reduction in serumLillirubin _ b) t\%o dextrose 2 ml/ig bolus
(AIIMS May 0s) c) l0o/o dextrose a mVkg
a) Photo-oxidation d) 5o/o dextrose 4mllkg
b) Photo-isomerization ll7. Maximum concentration of dextrose that can be
c) Structural isomerization given through peripheral yascular
d) Conjugation line in neonate
* _

(NEET Dec.12 partern)


f09. Bronze trabysyndrome is due to _
(PGI Dec 98) a) 5 b) 10
a) Phototherapy c) 12.5 d) 2s
b) Wilson disease 118. Administration of glucose solution is prescribed for
c) Chloramphenicoltoxicity all of the following situations except J ptums
d) Hemochromatosis a) Neonates
uay ool
f 10. A full term, g0 hrs old new born baby
develops jaun_ b) Child of a diabetic mother
dice what should tre the minimum level
of serum c) History of unconsciousness
d) History of hypoglycemia
NEONATE OF MOTHER d) Trunkinvolved
128. Asphyxial injury in a term baby is characterized by
I 19. All of the following are the complications in the new all except - (AilMS Nov 03)
born of a diabetic mother except - (ArrMS May 06) a) Seizures
a) Hyperbilirubinemia b) Hlperglycemia b) Differential hypotonia (lower limbs > upper limbs)
c) Hlpocalcemia d) Hlpomagnesemia c) Altered sensorium
l2A. Which of the following malformation in a newborn d) Difficulty in clearing oral secretions
is specific for maternal insulin dependent diabetes
129. A newborn with eyes closed 6 hrs after birth lustily
mellitus? @r 06) crying, no chest retraction and movements of all
a) Transposition ofgreat arteries
four limbs. Neonatal behavioral response grading -
b) Caudal regression
(AIIMS Nov 10)
c) Holoprosencephaly a) State I
d) Meningmyelocele b) State 3
121. Infant of diabetic mother with weight 3.8 Kg c) State 5
presented with seizures after 16 hours ofbirth. d) State 6
What is the cause - (AI 11, AIIMS Noy 09)
a) Hlpoglycemia
130. A neonate requires how much pressure for first
b) Hypocalcemia inflation ? ICET Nov. t2 pattern)

c) Birth asphyxia a) 25 mm of Hg

d) Intraventricularhemorrhage b) 25 cm of HrO
c) 25 cm of Hg
d) 25 mm of HrO
MISCELLANEOUS
131. Vomiting on the first day of baby's life may be
122, Kangaroo mother care-True is - (All India Dec15 pattern)
causedbyallofthefollowingexcept - (Ate7)

a) Can also be given by fathar a) Pyloric stenosis


b) Especially for low birth weight body b) Oesophageal atersia
c) Effective thermal control c) Aerophagy
d) AII of above d) Amniotic gastritis
123. Resolution of physiological umbilical hernia occurs
132. A non ventilated preterm baby in incubator is under
observation. Which is the best way to monitor the
at which week of gestation ? (CET Aug 12 Pattern)
baby's breathing and detect apnaea ?
a) 6 weeks b) 8 weeks (Ar 07)

c) weeks a) Infrared throraric movement study


10 d) 12 weeks b) Capnography
124. Hyperglycemia in Neonate if blood sugar is above - c) Nasal digital temperature monitoring
(NEET Dec.12 Pattern)
d) Impedence technique
a) mg/dl
150
b) mg/dl
125
133. Which of the following agents is likely to cause cere-
bral calcification and hydrocephalus in a newborn
c) 180 mg/dl
whose mother has history of taking spiramycin but
d) 100 mg/dl
was not complaint with therapy - (Ar oe)
I25. A newborn was given a drug in neonatal ICU, but a) Rubella
then was found in respilatory distress. The likely b) Toxoplasmosis
drug is? (Ar 07)
c) CMV
a) Morphine
d) Herpes
b) Naloxone \
c) Salbutamol
134. A pregnant lady had no complaints but mild cervical
\
lymphadenopathy in first trimester. She was pre- \
d) Soda-bicarb
\
scribed spiramycin but she was noncompliant. Baby
L26. A new born child developed respiratory depression was born with hydrocephalous and intracerebral
in neonatal ward. Which of the following drug is the calcification. Which of these is likely cause?
cause - , (AllMS Nov 10)
(AllMS May 11)
a) Opioids a) Rubella b) Toxoplasmosis
b) Barbiturates
c) CMV d) Herpes
c) Diazepam
d) Propofol
135. Microcephaly is defined as head circumference?
(CET Aug. 12 Pattern)
127. Hypoxic Ischemic encephalopathy true is - a) < lSD for age and sex
(AIIMS Nov 10)
b) < 2SD for age and sex
a) Lower limbs affected more than upper frrnbs c) < 3SD for age and sex
b) Prox. Muscles > distal muscles d) < 4SD for age and sex
c) Seizure
136. Hydrops fetalis rnay be caused by the following, 146. Most common newborn rash which presents at 24-
except- (All India Dec.14 Patrern) 48 hours of life is - (UPSC - il oe)
a) Congenital heart block a) Erythematous papular pustular lesions
b) Cystic hygroma b) Milia
c) Congenital varicella syndrome c) Transient neonatal pustular melanosis
d) Congenitalnephrosis d) Haemangioma
137. Fetal alcohol syndrome is characterized by all ex- 147. Which one of the following medical disorders
cept- (AIIMS Nov 09) leads to delayed foetal lung maturity - (UPSC 07)
a) Microcephaly a) Heart disease b) Diabetes
b) Low intelligence c) Thalassemia minor d) Epilepsy
c) Large proportionate body 148. Most common cause of respiratory distress after
d) Septal defects ofheart birth in frrst24 hours is - (JIPMER 95)
138. Test used to differentiate maternal from fetal a) Neonatal sepsis
blood? \uMSNov to) b) Meconium aspiration
a) Osmotic fragility test c) Bacterial pneumonia
b) Water bulb test d) Air embolism
c) Apt test 149, Cephalhematomausuallydisappearswithin
d) Kleihauer Betke test
ULPMER 80, UPSC 82)
139. Macrosomia is - (All lndia DecL5 Pattern) a) 3-5 months b) 2-5 weeks
a) Large size baby c) 3-5 weeks d) 5-7 weeks
b) Big mouth 150. Which one of the following is true of Transient
c) Large head Tachlpnea of Newborn (TTNB) - (UPSC 97)
d) Large tongue a) It is the commonest respiratory disorder caused
by absence of surfactant
QUESTIONS OF VARIOUS OTHER EXAMINA. b) In premature babies, it is often fatal
TIONS c) Onset of respiratory distress is immediately after
birth and it rarely lasts beyond 48 hrs
l4O, Grasp reflex develops by - (lipmer t1)
d) It often leads to chronic lung disease
a) 20 weeks b) 24 weeks 151. 8 days old neonate with extensor posture- (rN s9)
c) 28 weeks d) 32 weeks a) Cerebral palsy
I4l. The following is of serious pathological significance b) Hlpoxic ischemic encephalopathy
in infants - (Karnotaka 89) c) Malnutrition
a) Loss of weight d) Infection
b) Palpable left kidney 152. A26year old third*gravida mother delivered a male
c) Palpable spleen baby weighing4-Zkgat 37 weeks of gestation
d) Deviation of trachea from midline through an emergency caesarean section, for
L42. Harlequins Skin change in the newborn is seen obstructed labour. The child developed respiratory
in - (lipmer 93) distress one hour after birth. He was kept nil per
a) Autonomicdysfunction orally (NPo) and given intravenous fluids. He
b) Icthyosis maintained oxygen saturation on room air. No
c) Septicemia antibiotics were given. Chest radiograph revealed
d) Polyclthemia fluid in interlobar fissure. Respiratory distress
seffled by 24 hours of life. What is the most likely
143. In a new born, what is the normal respiratory rate ?
diagnosis ? (UPSC 06)
(UPSC-II o8)
a) Transient tachlpnea of the newborn
a) 10-20 breaths/minute b) Meconium aspiration syndrome
b) 30-40breaths/minute c) Persistent fetal circulation
I
c) 40-60 breaths/minute ,
d) Hyaline membrane disease
d) 60-80 breaths/minute
- 153. The foetal length is affected if the mother has
K The newborn heart rate is about
a) 120 - 160 /min b) 160 - 180 /min
(TN 04)
undernutrition during the -
a) First trimester
(UPSC e8)

E) 180 - 200 lmin d) 200 - 220 llrlin


b) Second trimester
145. "Mkrosomid'is defined as - (MAHE0s)
c) Third trimester
a) Blrth weight below 90'h percentile d) Any time during the pregnancy
b) Sl|th weight below 10'h percentile
L54. Meconium contains all except - (PGr 88)
c) Birth weight below 20'h percentile
a) Lanugo b) Bacterial flora
d) Birth weight below 50'h percentile
c) Epithelial debris d) Bilirubin
155. Which of the following is best fur transport of t&e b) Bile salt
*ewborn with rnaintainance of warm ternperature _
c) Haemoglobin
(MH 11) d) Melanin
a) Kangaroo Mother Care (KMC)
162,. A S0-hour old fuIl-term breast-fed newborn boy
b) Transport incubator
weighi*g 3l&Ig preser*s with ctinieatrtry evident
c) Thermacol box
jaundice. Fhysical examination is qthen€i$e
d) Hot bottle
nomal. The totatr bilinrbin is l t"CI mgldl with a
156. In Rh lso Imunisation, exchaage transfusion is direct bilirubin of 0.4 mgldtr. What woutrd be &e
indicated if - (AilMS 8e) €orrect treatment - (UPSC 07)
a) Cord blood hemoglobin is less than 10 g % a) Continue breast feeds and review after 4g hours
b) Cord bilirubin is more than 5 mg. b) Stop breast feeds and review after 24 hours
c) History of previous sibling afected c) Continue breast feeds and start blue-light
d) Hydrops fetalis phototherapy
157" Indications for exchange transfusion are all except_ d) Arrange for a double,volume exchange transfusion
(lipmer 03) 163. Fetallungm*turityasses.sedbyallexcept - (Up0B)
a) Unconjugated bilirubin > 18 mg/100 ml a) Measurement of cr-feto protein
b) Cord hemoglobin < 10 mg/100 ml b) Lecithin:sphingomyelin ratio
c) Cord bilirubin < 5 mg/100 ml c) Measurements of amnotic fluid creatinine
d) Bilirubin protein ratio > 3.5 d) Phosphatidyl choline concentration in amniotic
158. |auudice in the new-b,orn is physiological when _ fluid
(UPSC e8) Itr. Respiratory rate in * 2 montt et{ to }abe} it tachy-
a) The infant is visibly jaundiced in the first 24 hours pnea is - (DPGEE 08)
ofbirth a) 40 b) so
b) The total bilirubin concentration in the serum c) 60 d) zo
increases by 1 mg/dl per day
165" The blood sugar in a neqnate shortly after hirth
c) The total bilirubin concentration is above l5 mg/dl
reaches t&e lcweet Ievel of 3* mgldl at t&e age af -
d) ]aundice persists for more than one week in a
term infant @PGEE oB)
a) t hour
159" In a neonate, jaundice appears for tle first time in b) 3 hours
the 2nd week The following is not a cause _ @pG 10) c) 6 hours
a) Galactosemia d) 8 hours
b) Rh Incompatibility
c) Hypothyroidism 166. Whi€h of r&e fullowing is NCT the correct sign cf
good attachment cf;ababyto t&e breast _ (upsc_r
d) Breast milk jaundice 0e)
a) Baby's mouth wide open
160. Which of the following is not true about late onset
b) Lower areola more visible
Hemorrhagic disea,se of newborr (IIDN)? @H j0)
a) Begins between 2-7 days of life
c) Baby's lower lip everted
b) Intracranial hemorrhage is common d) Babyt chin touching the breast
c) Biliary atresia can predispose 167. \ffhictrl is an abnorrnal finding in * nesnate ?
d) Warfarin therapy is associated (Delhi PG Feb. 09)

l6f . Whi€h of the following subtance


a) Glycosuria
is toxic to neurons_
b) Bacteriuria
iipmER es)
a) unconjugated bilirubin c) WBCs in urine
d) Hlperbiiirubinemia

IIN

T
ANSWERS
NORMAL NEWBORN

L Ans. is 'c' i,e,,28 days of life fRef: O.P, Gkai }th/e p. 124 6 7h/e p. 96]
o From birth to under four weeks ofage (< 28 days), the infant is called neonate.
2. Ans, is 'c' i.e., Central cyanosis lRef : O.P. Ghai 6th/e p. 145 & Vhle p. 11jl
r There is peripheral cyanosis (not central cyanosis).
o Length is approximately 50 cm and the attitude of body is in flexion (not extension).

J. Ans. is'H i.e., 3 days fRef: Textboak af maternal dt wamenhealth carel


r The baby may pass meconium in utero or soon after birth, but all healthy babies must eyacuate within 24 hours.
After that, in normal breastfed baby meconium stools can be passed upto 3 days and on 4th and 5th days transitional
stools are passed. After 5 days regular milk stools are passed.

s Meconium is passed within -+ 24 hours ofbirth.


t Meconium stools are passed -+ upto 3 days.
t Transition stools are passed -+ 4s &5d days.
t Regilar milk stools are passed -+ After 5 days.

4. Ans. is 'c' i.e., No specific therapy fRef: O.P. Ghai 6thle p. 1471

r Menstural-like bleeding (vaginal bleeding) may occur from the third to seventh day of life.
o This is attributed withdrawl of maternal hormones after birth.
o The bleeding would subside after 2-3 days and no therapy is required.

5. Ans. is t' i.e., Lumbosacral area; 'd' i.e., Leg & t' i.e., Thigh lRef: Netson l8'k/e p. 26621
o Mongolian spots are blue or slate - gray macular lesions which occur most commonly in pre-sacral area (mainly in
lower back dt buttoclcs) but may be found over the posterior thighs, legs, and shoulders.

6. Ans. is'c'i.e., Leaving it alone [Ref: O.P. Ghai 6'h/e p. 146, 147]
o Hymenal tags around margins of hymen is normal finding.

7. Ans. is t' i.e., High larynx lRef : Gray's 40th/e p. 584; Andrew Gr*yson p. 821
o The infant larynx differs markedly from its adult counterpart. Although it is about one - third adult size, it is
proportionately larger. Its lumen is short and funnel - shaped and disproportionately narrower than that of adult. It
lies higher in the neck than the adult larynx. At rest, the upper border ofthe infant epiglottis is at the level ofthe
second or third cervical vertebra; when the larynx is elevated, it reaches the level ofthe first cervical vertebra. This
high position enables an infant to use its nasal airway to breath while sucking.

PRIMITIVE REFLEXES

8. Ans. is t' i"e., Symmetric Tonic neck reflexes lRef : Nelson 18'h/e p. 2$9; A.P. Ghai 8'h/e p. 142 {z
7h/e p. 114; Meharban singh 3'd/e p. 71; Chedda 3'd/e p. 31)
A. Reflexes present at birth
1. Rooting, sucking & swallowing reflexes 4. Crossed extensor
2. Moro reflex 5. Assymmetric tonic neck reflex
3rfalmar grasp (grasp reflex)
B. Reflexesappearafterbirth
1. Symmetric tonic neck -+ appears at 4-6 months
2. Parachute reflex -) appears at 8-9 months
3. Landau reflex -> appears at 10 months

9. Ans. is'b' i.e., 12 weeks fRef: Nelson 18'h/e ch. 6.1)


r Swallowing reflex appears at 1.2-13 weeks of intrauterine life.
I

"rhe suck-swattow reftex,:: to appear. rt typicauy begins


to appear by 12_13 week of pregnancy,,
{,1;l;:;:!,:;::
c By 13-14 wk' breathing and swallowing
motions appear and tactile stimulation
elicits graceful movements.
t0. Ans. is 'b' i"e., Brain dama ge
[Ref : A.F. Ghai trh/e p. 145, 146 6 Th/e p. 114]

Absent
Abnormal persistence
'l +
Dysfunction of CNS or pNS
Dysfunction of CI\S

u. Ans. is ?'i.e., Never [Ref l{elson l}th/e p. 2439; Meharban Singh


3d/e p. 7l}
r Parachute reflex appears at 8-9 months
of life and remains thtroughout tifeafter
that.
12. Ans. is 'd i.e., Asfm1elfl1 neck reflex
{Ref: A.F. Ghai tth/e p. 142 dr Th/e p. 114;Nelsan tSth/e p. ZqSg;
singfi 3'd/e p. 71; Chedda 3d/e p. 31) Meharban
o Asymmetric tonic neck reflex
is prominant between 2nd and
4th months.
' ,1T,'::t:1.o.u:iliffiffirTvond
the age of 6 - e months o. u .o,r.iu,r,
tonic neck posture are abnormar and
usuary

13. ,dns" is'b'i,e., Cr-Cu[Ref: o,p. Ghai 6thle p. 146; Nelson lg,h/e p. 2439, z2l)
o Unilateral Moro,s reflex is seen
in :_
t E-rbspalsy (Cs-Cu) damage
t Fracture of humerus or clavicle.
I Spastic hemiplegia
t Shoulder dislocation
o Exaggerated Moro,s reflex is
seen in _ cerebral damage.

14. Ans' is 'b'i.e., sternomastoid tumour


[Ref : o.p. Ghai ,th/e p. 142 6 6th/e p. 146)
15. Ans" is t'i.e., 6th rnonth
lLef : Tachdjian p. 371; O,p. Ghai gth/e p. t42
"In normal infant the Moro's 6 Zhle p. t14]
reJlex begins to at rhree
when persii, t ryona 6 months it fade months o and gradually disappears at 4-6
irailotrr'iday in cNS arr,op*rn{,age months.
"Moros.it reflex disappears
by six months in normal'infant.,,
r "-'""
____l__T:llr#",
t6. Ans. is 'a'i.e., Moro,s lRef : 0,p, Gkai 6th/e
p. 146 6 Vh/e p. l l4l
r Moros reflex disappe ar at 3_6 months
and never reappears.
a GrasP reflex' snout reflex' palmomental
reflex and *.rurrg reflex are n_ormally
inhibited by frontal lobe as the child
i.l,Ilii"t'J}Iffi::
reieased from inhibition in frontar
rJo. So, ;;;;;;.
these reflexes .ui, b. ,..r, in persons
with

Newborn infant Grasp reflex, sucking reflex, snout


&
palmomental refl ex present

lnhibition by frontal lobe +


Child grows
Disappearance of these refl exes

Frontal lobe lesions


Loss of lnhibition *
Reappeara nce of these refl exes

HYPOTHERMIA

t7. Ans. is 'a'i.e., ShiveringfRef: O.p. @tai gth/e p. l4j 6 Vh/e p. t t5]
"Always remember that newborn
cannot produce heat by shivering),
xs' Ans. is'b'i.e., Uncontrolled shivering fRef: o.p. Ghai B,h/e p. 143 6 vh/e p. 118]
"N e o n at e s Re sp o n d b y N on- shiv er ing thr mo gene sis',.

LOW BIRTH WEIGHT

19. Ans. is 'd i.e., < tr 0 percentile for gestational age lRef o.p. Ghai 6th/e p, 136 d, Th/e p. 1291
o Appropriate for gestational age (appropriate for date)
Babies with birth weight ranging between 10th to 90th percentile on such a chart are considered
appropriate for date.
o Small for gestational age (small for date)
Babies with birth weight less than 10th percentile are categorized as small for dates.
r Large for gestational age (large for date)
Babies with birth weight more than 90th percentile are categorized as large for date.

20. Ans. is kg [Ref Ghai 7h/e p. 128]


1d' i.e., < 1
o Low birth weight newborn
Any neonate weighing less than 2500 gm at birth irrespective of the gestational age.
r Verylowbirth weight newborn
Any neonate weighing less than 1500 gm at birth irrespective ofthe gestational age.
r Extremely low birth weight newborn
Any neonate weighing less than 1000 gm at birth irrespective ofthe gestational age.

21, Ans. is 'a'i.e., Diabetes [Ref O.p" Ghai B,h/e p. 155 6 f,/e p. 1281
o In DM, there is large for date baby (not small for date : IUGR). Other three are causes
of IUGR.
)) Ans. is t' i.e., weight less than l0th percentile [Ref o.p. Ghai 8'h/e p. 138-140 6 Thle p. 109]
o First let me differentiate the following three terms
i) Low birth weight neonates
ii) Preterm infant (Premature infant)
iii) Small for gestational age (small for date)
i) Lowbirth weight
r
Any neonate weighing less than 2500 gms at birth irrespective of the gestational age.
ii) Preterm infant (premature infant)
r Any neonate born before 37 weeks of gestation irrespective of the birth weight.
r Because birth weight is a function of gestation, a preterm baby is expected to have less weight.
iii) Small for date (IUGR)
r Babies with a birth weight less than 10 percentile for that gestational age. These may be preterm (born before
37th week) or term (born between 37 to 42 weeks).
o So, both preterm infant and small for date neonate have low birth weight.

Low birth vyeight baby

Pre term Small for gestational age (IUGR)


+
Fullterm

r Low birth weight neonates will have following problems -


i) Problems of small for date babies
ii) Problems of preterm babies
23. Ans. is t' i.e., Thick ear cartilage lRef : O.p. Ghai Bth/e p. 138-140 6 Vh/e p. 109)
c The ears in a premature neonate are sofi and flat with ear cartilage being deficient and pliant (and not thick).
24, Ans, is 'a'i.e., Suffer from jaundice of hepatic origin fRef: O.P. Ghai 8th/e p. 156 6 Vh/e p. 129-tj0)
c The liver of premature (preterm) neonate is functionally immature that leads to hlpoglycemia, hyperbilirubinemia
(jaundice) and poor detoxification ofdrugs.
25. Ans. is'd i.e., HypoglycemialRef O.p. Ghai tth/e p. 156 6 Vh/e p. t2g_130)
o Small for date babies are prone to hlpoglycemia.
o Intraventricular hemorrhage occurs in preterm infants (not
in full term, small for date babies).
26. Ans' is'b'i'e" Head circumference is 3 cm more than chest circumference
p. 1561
fRef: o.p. Ghai gth/e p. 156 6 6th/e

IUGR is synonymous with smallfor gestationalage


infants -
Malnourished small for dates (commonest variety of IUGR
infants) some important features are -
r The difference in the head and chest circumferences is more than 3 cm.
r Internal organs like Liver, thymus and lungs are shrunken,
although pulmonary alveoli appear mature. (i.e. No
Hepatomegaly)
o Pulmonary alveoli are mature; patient is not premature
but only LBW or SFD -+ No ARDS
27. Ans. is t'
i.e., Brain I Ref : O.p Ghai Bth/e p. 156 6 Vh/e p_ 129; Nelson trth/e p.
202, 7031
IUGR is classified as
1) Symmetric IUGR
e Reduced growth is symmetric + head circumference,
length and weight equally affected.
o Has earlier onset.
o Is associated with diseases that seriousiy affect fetal
cell number, such as conditions with chromosomal,
genetic malformation, teratogenic, infectious or severe '
maternal hypertensive .tiofogi...
2) Asymmetric IUGR
o Reduced groMh is asymmetri cwrth relative sparing
of head growth.
o Has late onset.
r Is associated with poor maternal nutrition or with late onset / exacerbation
(preeclampsia, chronic hypertension).
of maternal vascular disease

28. Ans. is 'a'i.e., 1,6 fRef: O.p. Ghai Zh/e p. 109)

Weight (gm)x roO


Ponderal index
Length (cm)'

r The PI of the baby in question is -

pI='ooo*too=r.o
(s0)'

o That means the baby is having assymeric IUGR (pI <


2).
29. Ans is 'b' i.e., Orogastric [Re! Handbook of pediatric nutrition p.
50_60)
Feeding ofa preterm neonate
o The aim of nutritionai management of a preterm infant is to achieve full enteral feeding as soon as possible.
o For preterm infants of more than 34 weeks of gestation,
it is possible that breast or bottle feeding can be adopted
as
they should be able to coordinate sucking, swalowing and
bieathing.
o But for those younger infants who may have compromised
respiration or be neurologically less well developed, this
may not be advisable and nutrients may need to be passed
lnto the stomach through a fine tube either through the
mouth (oro-gastric tube) or the nose (naso-gastric tube). Naso-gastric
tube increases the airway impedance and
work ofbreathing, therefore, orogastric tube is preferred
orr". rlrogurtric tube.
o Breast milk is food of choice as it is best tolerated and includes other benefits
the
as well as nutrition.
r TPN is given to vLBW infants who cannot tolerate gastric
tube feeds, have gastric disease, enteral feeding (by tube)
is not possibie or just while enteral nutrition is being established.
other important indication for TpN is sick pre-
mature/LBW infants which include Infants with:
(i) Significant respiratory distress required assisted ventilation.
(ii) Shock requiring inotropic support.
(iii) Seizures, renal/cardiac failure
(iv) Symptomatic hlpoglycemia/hypocalcemia, electroly.te abnormalities.
(v) Surgical condition of GIT, NEC, hydrops
o Even in these sick infants, enteral feed should be started as early as possible.

<28 weeks No proper sucking efforts lntravenous fluids


No propulsive motility in the gut
9:;
*!;:jit;n
ri'.:r:r;\iiin

32-34 weeks

Alogrithm for feeding of a preterm infant

tr4week;l
J
I nititate b r eas t fe edin g
I
V
Observe iJ
1. Positioning and attachment are good.
2. Able to suck effectively and long enough
(about 10-15 min)

I Breast feeding I ---- Startfeed I


by spoon/paladai
I I

J
Observe i/
l. Accepting well without
spilling/coughing

H
2. Able to accept adequate amount
I
+
Start feed by OG/NG tube
spoon/paladai feeding I
Y
Observe if
1. Vomiting / abdominal
distension occurs
l7&-..k.1
2. The prefeed aspirate exceed
>25%o of feed volume

I
Gastric tube feeding

Coming to question
o Best answer of this question is feeding by spoon or paladia. However these are not provided in options.
o Amongst the given options orogastric tube is the answer of choice.
30. Ans. is 'd' i.e., 72 hours 7/e p. t36l
fRef: Ghai
o Early onset neonalal sepsis occurs within 72 hours of life.

31. Ans. is 'b' i.e., E.coli [Rel A.F. Ghai |th/e p. 163 6 7h/e p. 136; Care of the new born 6th/e, Meharban Singhl
New born sepsis can be classified into early onset sepsis-occurring within 72 hours and late onset sepsis occuring after
,/2 hours-
Early onset sepsis -
r It is caused by organisms prevalent in the genital tract or in the labor room and maternity operation theatre.
o In the west it is mostly caused by group B streptococcus and E.coli.
o In our country it is mostly due to gram negative organisms-E.coli, klebsiella and enterobactor sp.
r
Majority of the neonates with early onset sepsis manifest as respiratory distress due to intrauterine pneumonia.
Late onset Septicemia :
o Late onset septicemia is acquired as nosocomial infection from the nursery or lying in ward.
o The onset is delayed for 48-72 hours after birth.
r In most cases symptoms appear by the end of first week or during second week of life.
o About two third cases of late onset septicemia are caused by gram negative bacilli, Klebsiella pneumonea, entero
bacteria, E.Coli, pseudomonas aeruginosa, alkaligenese fecalis, salmonella tyhimurium, proteus, citrobacter and
serratia.
32. Ans. is 'a' i.e., Local nursery environment [R4 O.p, Ghai gth/e p. 163 & Th/e p. 1361
. Late onset sepsis is caused by organisms of the external environment of home or hospital and the infection is
transmitted most frequently by the hands of the care-provider.
33. Ans" is 'b' i.e., Klebsiella lR ef : OP Ghai 8th/e p. 163 dz Vh/e p. 1i6; Meherbsan Singfi 6th/e p. 209 and lournal of past-
graduate medicine (IPGM)I
t Most common cause of neonatal sepsis in hospitals in India is -+ Klebsiella
t Most common cause of neonatal sepsis in hospitals across the world is -+ E. coli
o Most common cause of overall neonatal sepsis -+ Group B streptococci
34. Ans. is 'a' i.e., Genital tract lR.ef: Nelson lBth/e ch. lB3l
. Gram positive CAMP positive coccus is group B streptococcus (Str. agalactiae), the most common cause of neonatal
r mepingitis.
------...:tt"habitat in human hosts is female genital tract and rectum.
35. Ans. is 'H i.e., Neutrophilia lRef: O.P. Ghai 8th/e p. 164 & Thle p. 137; Nelson t6h/e p. 54Bl

Laboratory finding in Neonatal Sepsis :

Neutropenio is more (ommon than Neutrophilia


in severe neonotal infections

An lmmature Neutrophils /Total Neutrophils ratio


neutrophils
of 0.1 6 or greater suggests bacterial infection
,ii' i::i:li+::l::::::i:l:lr: :i,iil :-:i:t:l

:1!=eti:tr:ie$as.slra[s-ai1
inflammatory response ERS is lncreased

36. Ans. is 'a' i.e., Group B streptoco ccifRef: O,p. Ghai 8th/e p. 163 6 Vh/e p. 1j6)
J/. Ans. is '-o" i.e., Streptococcus agalactiae [Rel O.P. Ghnl Sth/e p. 16j 6 Vh/e p. 136; Nelsan lrth/e p. Z4\
38. Ans. is 'b' i"e., E. coli [Rel Nelson l|th/e p. 747]
r E. coli & streptococcus agalactie (group B streptococci) are the two most common cause of neonatal sepsis and
meningitis.
39' Ans. is'c'i.e., Ampicilrin and Gentamicin
[Rl o.p. Ghai Bth/e p. 171 6 p/e p. r38;]
r Antibiotic therapy for neonatar pneumonia incrudes ampicillin prus gentamycin.

RESPIRATORY DISTRESS

40. Ans. is 'l i.e.,Wheeze {Ref: Nekon ltth/e Chapter 101.4)


o The principre features of the respiratory
distress in a neonate :-
1' Thchypnoea : Rates of > 60 breaths/min, > 50 breaths/-i,
urra > 40 breaths/min for young infants (0-59
infants (60-365 days) and childhood (1-5 years). days),
2' use of accessory muscles for respiration : Features include
use of sternocleidomastoid muscles, nasal alar flare
and tracheal tug.
3' Use ofintercostal or subcostal muscles for respiration
resulting in intercostal or subcostal recession
wall indrawing. - lower chest

4. Audible grunting
5. Cyanosis

41. Ans. is 'd i.e.,Apnea [Re! O.p. Ghai dth/e p. 169 6 Zh/e p. 116]
e Apnea may be defined as _
i) cessation of respiration for 20 seconds with or without bradycardia
and cyanosis.

ii)cessation of respiration ro. t..o.ttrrur, 20 seconds


if it is associated with bradycardia or cyanosis.
42. Ans, is 'c' i.e.,20 sec fRef: See above explanation]

MECONIUM ASPIRATION SYNDROME

43. Ans. is'b'i.e., Obstructive emphysema


{Ref: Dtttta 4th/e p.
p. 20, 2t)
511; O"p, Ghai Bth/e p. 170 6 Vh/e p. 144_145; CryDT lyth/e

Consequences of meconium aspiration


o Three main problems occur due to meconium
aspiration _
i) obstructive emphysema or atelectasis- *h.r, aspirated material
blocks the airways.
ii) Pneumonitis qnd chemical pneumonia-+ because of irritant property
of meconium.
iii) Defective gas exchange.
44. Ans. is'b'i.e., Meconium aspiration syndrome
[Ref Nelson lgth/e p. 742; O.p. Ghai gth/e p. j1y d2 Th/e p. 144_
14s)
o Meconium stained liquor (amniotic fluid)
with respiratory distress soon after birth suggests
the diagnosis ofmeconium
aspiration syndrome.
r The important clues in this question are :_
i) Baby is full term +
MAS occurs in term or post term neonates.
ii) Respiratory distress soon after birth -+ MAS presents
like this
iii) Liquor is meconium stained -) supports the diagnosis
of MAS
o In transient tachypnea of newborn and hyaline
membrane disease (Jsurfactant production)
stained with meconium. amniotic fluid will not be

45' Ans' is'b' i.e., Meconium aspiration syndrorne


Ghai vh/e p.
rRef: 142)
o Tachypnea (or respiratory distres) in post term -->
Meconium aspiration syndrome.
r Transient tachypnea occurs in preterm or term
baby and HMD occurs in preterm baby.

CONGENITAL DIAPHROGMATIC HERNIA

46. Ans. is t' i.e., Pulmonary hypoplasia


fRef: Nelson tSth/e p. 748]
o Most common cause of death in congenitar
diaphragmatic hernia is -+ Pulmonary complications
c 2d most common cause of death in congenitar
di.ph.ng-utic hernia in -+ Intestinal obstruction
47. Ans. is'o" i.e., Diaphragmatic hemia lRef; Xttetsan lghle eh.Ml.9l
o Term newborn with respiratory distress and scaphoid abdomen
suggests congenital diaphragmatic hernia.
48. Ans. is t' i.e., Insert a nasogastric tube fRef. Schwartz Vhle p. lZ20]
Management of congenital diaphragmatic hernia
o Earlier congenital diaphragmatic hernia was considered to be a surgical
emergency. It was believed that the mass
efect of herniated viscera is the major factor in cardiorespiratory compromise.
r Nowitisrecognizedthatmajorcauseofcardiorespiratorycompromiseispzlmonaryhypertensionandpulmonary
hypoplasia.
o So now the patient is initially resuscitated before surgery is performed.
This resuscitation consists of
a) Stabilization by mechanical ventilation with 100o/o O,
b) I'{asograstric suction
o ventilation prevents the development of pulmonary hypertension
and provide adequate oxygen delivery. Now
days infants are ventilated by high frequency oscillation.
e Nasogastric tube is used for suction to aspirate swallowed air and to prevent
distension of the herniated bowel
which would further compress the lung.
o So both option t' & A' are preoperative procedures in congenital diaphragmatic
hernia. The question is which
should be performed fist.
r Remember that nasogastric suction is performed first in a case of CDH unless the patient presents with
following
symptoms -+ a) Cyanosis, b) Apnea, c) Respiratory disease
o Here in the question the patient does not complain of any of the above
symptoms so nasogastric suction should
be the first priority.

49. Ans. is 'H i.e., Put a nasogastric tube {Ref. see previous expranatian}
r Mediastnal deviation, scaphoid abdomen with respirator-y distress suggest the
diagnosis of congenital diaphragmatic
hernia (CDH).
o The resuscitation of CDH patient consist of :
a) Stabilization by mechanical ventilation with 100% 02.
b) Nasogastric suction
r This child has already been intubated.
r Now nasogastric suction should be done to aspirate swallowed air and to prevent
distension of the herniated bowel
which would further compress the lung.

50. Ans. is'a'i.e., Confirm the position of endotracheal tube


[Ref Nelson lgele p" 246-z4sl
This is a case of congenital diaphragmatic hernia
The clinical features suggesting this arer
r Respiratory distress after birth with decreased air entry on the left side
r Scaphoid abdomen
r Mediastinal shift
o In the above question, mediastinal shift occurred after bag and mask
ventilation and endotracheal intubation.
o So there is a possibility of wrong endotracheal intubaiion (i.e., endotracheal
tube entered esophagus instead of
trachea).
I This caused distension of stomach with gas which resulted in mediastinal
shift. So the confirmation of the
position ofthe endotracheal tube should be done.
r If the endotracheal tube is in proper position the next immediate treatment modality in this patient would be
decompression of the gastric distension by nasogastric tube insertion.

51. Ans. is 'b'i.e., Remove tube & reattempt intubation


[Ref Read.beraw]
o Dont get confuse with answer of previous question.
o There is difference between option a ofprevious question and option'a'ofthis
question :-
Previous question - confirm the position ofendotracheal tube.
This question - confirm the position ofET tube by chest x_ray.
r Best answer would have been, confirm the position of endotracheal
tube.
o But, chest x-ray cannot distinguish between tracheal and esophageal
intubation and cannot confirm the position
ofthe ET tube in trachea. Chest x-ray is done to confirm the corrJct length oftube (not
the position).

I
Method to confirm tracheal intubation
l. Tests which suggest tracheal placement (not confirmatory)
r Symmetrical Bilateral chest movements when reservoir bag is squeezed.
r Equal breath sounds over lung fields when reservoir bag is squeezed-
r Absence of gurgle over lung fields when reservoir bag is squeezed.
r Misting of the tube (water vapour deposition) during expiration.
: Feeling of correct lung compliance and refilling of bag during expiration.
2. ConfirmatorY tests
r Capnography -+ Measures end tidal CO2 in expired gas.
r Esophageal detector device.
o None of these method has been mentioned in the options of this question.
r Amongst the given options, best course of action is to remove the tube and reattempt intubation : -
"If there is any doubt about tube placement the tube should be removed" --- Lee'

Neonatal respiratory distress


v
Signs of CDH
(Scaphoid abdomen, mediastinal shift)
v
lmmediate ET lntubation

Signs of improper positioning of tube Proper positioning of tube


+
Dysfunction gf CNS or PNS
*
Confirm the position of tube
v
Remove the tube & reattempt intubation Nasogastric tube insertion
to decompress bowel
t
Chest x-ray to confirm the Diagnosis of CDH
v
urgery as required

52. Ans. is t'i.e., Nasogastric tube insertion[Ref See above explanation)

r Here "removal of tube and reattempting intubatiori' is not provided amongst the option.
o The next best stept is to decompress the distended stomach by nasogastric tube insertion.
So, the same question has three different answers depending on the options provided :-
1) Confirm the position of ET tube (not by chest x-ray) -+ Best answer
ii) Removal of the tube and reattempting intubation -+ If previous one is not provided as an option.
111) Nasogastric tube insertion -+ lf previous two are not provided in option.

Note : In previous three explanations I have explained the management protocol of CDH in which intubation has
already been used. But you should keep in mind that nasogastric suction should be the flrst procedure except
in
Apnea, cyanosis and respiratory desease, where intubation is preferred initially.

53. Ans is'-o- i.e., Delay in emergent surgery fRef: Nelson 18th/e p. 748; Fundamentals of pediatric Surgery p.
535,5361
r Timing of surgical repair has gradually shifted from an emergency repair, to a policy of stabilization using a variety
ofventilatory strategies prior to operation.
o Current recommenJation is to adopt a conservative approach and delay surgical repair of the CDH until the infant
stablizes from a hemodynamic and respiratory point of view'

Prognostic factors in CDH


r Prognostic factors can be divided into:-
A) Primary prognostic factors (Pathophysiological)
I These are the most important prognostic factor which affect morbidity and mortility.
Method to confirm tracheal intubation
l. Tests which suggest tracheal placement (not confirmatory)
r Symmetrical Bilateral chest movements when reservoir bag is squeezed.
r Equal breath sounds over lung fields when reservoir bag is squeezed.
r Absence of gurgle over lung fields when reservoir bag is squeezed.
r Misting of the tube (water vapour deposition) during expiration.
r Feeling of correct lung compliance and refilling of bag during expiration.
2. Confirmatory tests
r Capnography -+ Measures end tidal CO2 in expired gas.
r Esophageal detector device.
. None of these method has been mentioned in the options of this question.
o Amongst the given options, best course of action is to remove the tube and reattempt intubation :

"If there is any doubt about tube placement the tube should be removed" --- Lee.

Neonatal respiatory distress

Signs of CDH
(Scaphoid abdomen, mediastinal shift)

lmmediate ET lntubation

Signs of improper positioning of tube Proper positioning of tube


+
Dysfunction of CNS or PNS
v
Confirm the position of tube
+
Remove the tube & reattempt intubation Nasogastric tube insertion
to decompress bowel

Chest x-ray to confirm the Diagnosis of CDH

Surgery as required

52. Ans. is t'i.e., Nasogastric fube inserti onfRef: See above explanation)
r Here "removal of tube and reattempting intubation" is not provided amongst the option.
o The next best stept is to decompress the distended stomach by nasogastric tube insertion.
So, the same question has three diferent answers depending on the options provided :-
1) Confirm the position of ET tube (not by chest x-ray) -+ Best answer
ii) Remoyal of the tube and reattempting intubation -+ lf previous one is not provided as an option.
111) Nasogastric tube insertion -+ If previous two are not provided in option.
Note : In previous three explanations I have explained the management protocol of CDH in which intubation has
already been used. But you should keep in mind that nasogastric suction should be the first procedure except in
Apnea, cyanosis and respiratory desease, where intubation is preferred initially.

53. Ans is '-o- i.e., Delay in emergent surgery lRef: Nelson 18th/e p. 748; Fundnmentals of pediatric Surgery p.
535, 5361
r Timing of surgical repair has gradually shifted from an emergency repair, to a policy of stabilization using a variety
ofventilatory strategies prior to operation.
r Current recommendation is to adopt a conservative approach and delay surgical repair of the CDH until the infant
stablizes from a hemodynamic and respiratory point of view.

Prognostic factors in CDH


e Prognostic factors can be divided into:-
A) Primaryprognostic factors (Pathophysiological)
r These are the most important prognostic factor which affect morbidity and mortility.
:.:'..t'.:.a.:'.::..'...,
a:':.,:.,: : :,,.1: ::::::,. :a::.,.- a . : :.). :

.1!$*aa*,!itq',
r These factors are :-
i) Pulmonary hypoplasia (most important)
ii) Pulmonary hlpertension (2nd most important)
B) Secondary or relative predictors (diagnostic/clinicai)
a These poor prognostic factors are:
a) Antinatal
i) Detection at an early gestational age (< 24 wks)
ii) Associatedextradiaphragmatic congenital anomalies
iii) Liver herniation into thorax (Liver above diaphragm)
v) Stomach herniationinto thorax (Stomach above diaphragm)
vi) Presence of poiyhydraminios
vii) Small lung to head circumference ratio (LHR ratio)
viii) Small fetal abdominal circumference (< than 5th percentile)
b) Postnatal
i) Early age (< 6 hours) ofpresentation
ii) PO, and PCO, unresponsive to ventilation
iii) Need for extracorporeal membrane oxJgenation (ECMO)
c) Side ofdefect
i) Right sided defect
54. Ans. is'a'i.e. Pulmonary hlryertension IRel Sabistan 18th/e p. 20731
"Compression of the lung results in pulmonary hypoplasia involving both lungs, with the ipsilateral lung being the
most affected. In addition to the abnormal airway development, the pulmonary yasculature is distinctly abnormal in
that the medial muscular thickness of the arterioles is excessive and extremely sensitive to the multiple local and
systemic factors known to trigger vasospasm. Thus, the two main factors that affect morbidity and mortality are
p ulm o n ar y hy p opla si a and p ulmon ar y hy p er ten si on, " - S ab i st o n 1 8th / e p 2 0 7 3

HYALI NE NIEMBRAI\IE DISEASE

55. Ans. is t' i.e., Prematurity lReJ: Nelson !.9't'/o p. 731, 7321
o HMD always occurs in preterm babies often less than 34 weeks of gestation.

55. Ans. is t' i.e., }IMD lRef : O.P. Ghai 9th/e p. 169 & 7h/e p. 143; Nelsan tVth/e p. 7jtr)
o Surfactant deficiency (decreased production and secretion) is the primary cause o/RDS.

57. Ans. is'a' i.e., In ttre flrst 6 hours of life lRef : A,P. Ghai 9th/e p. 169 6 Vh/e p. 1.43; Nelson IBth/e p. 7321
c Respiratory distress occurs within the first 6 hours of life.
58" Ans. is 'b' i"e., Hyaline membrane disease [Ref: A.P. Ghai ?th/e p. 169 6 Vh/e p. ]43; Nelson lYthle p. Z3l, Zj2l
Respiratory distress in a preterm infant and X-rays showing ground glass appearance with air bronchogram suggest
the diagnosis of neonatal respiratory distress syndrome (RDS) also called hyaline membrane disease.
"Hyaline membrane disease is the commonest couse of respiratory distress in a preterm neonate". Ghai
-
About other options
o Meconium aspiration sl, rdrome usually occurs in full-term or post-term newborn.
o ARDS is adult respiratory distress syndrome.

59. Ans. is 'a' i.e", Flyaline membrane disease {Ref: Ghai Vh/e p. l4jl
60. Ans. is 'b' i.e., Air bronchogram on chest x-ray {Ref : Nelson 18th/e p, 7fi; aP Ghai Bth/e p. 169 6 Vh/e p. 143)
The chest x-ray of an infant with RDS is characterized by atelectasis, air bronchograms, and a diffuse reticular-
granular pattern commonly referred to as 'ground glass'i The chest x-ray may progress to a complete "white out"
with severe disease.
About other options
r Antenatal corticosteroids are given in pre-term (pre-mature) pregnancies not in term pregnancies. Further, antenatal
corticosteroids are given to prevent RDS (HMD) -> after antenatal corticosteroids administration, risk of HMD is
reduced.
. Respiratory distress occurs within first 6 hours.
r HMD occurs in pre-term neonate.
a1. Ams, ic i,*., LIS uafi*r {Ref : {},ff" {itr*i Vtt'lt. y, l{t* ,*. Vt'le p, t4,ll
"d
Prenatal diagnosis o[ HMD can be made bv -
1) Lecithin / sphingomyelin (L/S) ratio in amniotic Jluid -+ LIS ratio > 2 indicates adequate lung maturity.
2) Shake test -+ Amniotic fluid or gastric aspirate is mixed with absolute alcohol and shaken for 15 seconds and
allowed to settle. Copious bubbles are formed in the presence of adequate surfactant indicating extent of lung
maturity.

62, Ans. is t' i.e., Phosphatidyl glycerol estimation is a reliable method of diagnosis [Ref : O"F. Ghai 8th/e p. 169
dz Vh/ep. 143-144)
o In HMD surfactant is reduced and its measurement can be used for diagnosis e.g. in shake test.
r Constituent of surfactant are - dipalmitoyl phosphatidylcholine (lecithin), phosphatidylglycerol, apoproteins
(surfactant proteins -+ SPA, SPB, SPC, SPD) and cholesterol.

63. Ans. is 'd' i.e., Is treated by administrating 100% orygen [Ref : A.P. Ghai 9th/e p. 169 6 Vh/e p. 143-144]
o Premature infants receiving high concentrations ofoxygen may deveiop retrolental fibroplasias. So generally oxygen
tension is kept approximately 90o/o and not 100%.
ff. Ans. is 'd i.e., CPAP lRef: Neonatology by Eval4th/e p. 591
r Specific treatment for HMD is intratracheal surfactant therapy. This therapy requires endbtracheal intubation, which
also may be necessary to achieye adequate ventilation and oxygenation.
o Less premature infants (those > 1kg or > 28-30 weeks gestation) and those with lower O.requirements (FiO2 <
40 - 50%) may respond well to supplemental A, alone or to treatment with nasal continuous positive airway
pressure (CPAP).

65. Ans. is t' i.e., Prevents hlperbilirubinernia lRef : O.P Ghai ?'h/e p. 170 & 7h/e p. 144)
Prevention of HMD
r Prenatal steroids are effective in preventing HMD.
o Steroids acts by enhancing lung maturity.

Benefits
r 50 percent reduction in the incidence of respiratory distress syndrome (RDS)
c 5O percent reduction in the incidence of intraventricular hemorrhage (lVH)
e 40 percent reduction in mortality.
lndications
r All mother at risk of preteim delivery between 24 and 34 weeks of gestation.
r Cases of preterm premature rupture of membrane at less than 32 weeks of gestation
in the absence of overt clinical chorioamnionitis

Contraindicotion
a Clinical chrioamnionitis. (Maternal hypertension and diabetes mellitus are no
contiaindications. Careful monitoring and management of hypertension and hy-
perglycemia should be ensured).

Treatment schedule
r Inj. Betamethasone 12 mg lM evety 24 hours, 2 doses (preferred).
* Inj. Dexamethasone 6 mg tM every '12 hours,4 doses (only if betamethasone can
not be arranged).
Timing of effect
r Optimal effect occurs after 24h of initiating treatment.
c The effect ofone course lasts for 7 days.

uh'
6&,
'&ns.
l$ i,e,, tr? mg eveyy 24 h*qrg {fl,ef : See oLtove cxplanation)
o/. Ans. is 'd i,e,, FRC is snialler than closing yolume lRef : Handbook of pediatric intensive care l"t/e p. 1j8|
"Hyaline membrane disease reduces compliance and causes a fall in FRC (to levels below the normal closing yolume
of the lung)". -- Handbook of pediatric intensive care
o Respiratory distress occurs within.first 6 hours.
o HMD occurs in pre-term neonate.

6t. &l*s. 1s "lt i.*,, X,15 r*tiey {t4.{"{ : t}"t!. ti&a*i ttt,/r p. I {i9 ,$, 7tt'/* yt, I 4 4}

Prenatal diagnosis of HMD can be made by -


1) Lecithin / sphingomyelin (L/S) ratio in amniotic Jluiil -+ LIS ratio > 2 indicates adequate lung maturity.
2) Shake test -+ Amniotic fluid or gastric aspirate is mixed with absolute alcohol and shaken for 15 seconds and
allowed to settle. Copious bubbles are formed in the presence of adequate surfactant indicating extent of lung
maturity.

62. Ans. is'c'i.e., Phosphatidyl glycerol estimation is a reliable method of diagnosis [Ref : O.P. Ghai *thle p. 169
dr 7h/e p. 143-1441
r In HMD surfactant is reduced and its measurement can be used for diagnosis e.g. in shake test.
o Constituent of surfactant are - dipalmitoyl phosphatidylcholine (lecithin), phosphatidylglycerol, apoproteins
(surfactant proteins -+ SPA, SPB, SPC, SPD) and cholesterol.

63. Ans. is 'd' i.e., Is treated by administrating 100% oxygen lRef : O.P. Ghai 8th/e p. 169 6 7h/e p. 14i-1441
o Premature infants receiving high concentrations of oxygen may develop retrolental fibroplasias. So generally oxygen
tension is kept approximately 90o/o and not 100%.
64. Ans. is 'a' i.e., CPAP lRef: Neonatology by Eval4th/e p. 591
r Specific treatment for HMD is intratracheal surfactant therapy. This therapy requireq endotracheal intubation, which
also may be necessary to achieve adequate ventilation and oxygenation.
o Less prcmature infants (those > 1 kg or > 28-3A weeks gestation) and those with lower O, requirements (FiO2 <
40 - 50o/o) may respond well to supplemental O, alone or to treatment with nasal continuous positive airway
pressure (CPAP).

65. Ans. is 'c' i.e., Prevents hyperbilirubinemia [Ref : O.P. Ghai 8th/e p. 170 6 Vh/e p. Ma)
Prevention of HMD
r Prenatal steroids are effective in preventing HMD.
r Steroids acts by enhancing lung maturity.

Benefits
e 50 percent reduction in the incidence of respiratory distress syndrome (RDS)
f 50 percent reduction in the incidence of intraventricular hemorrhage (lVH)
I 40 percent reduction in mortality.
lndications
o All mother at risk of preterm delivery between 24 and 34 weeks of gestation.
c Cases of preterm premature rupture of membrane at less than 32 weeks of gestation
in the absence of overt clinical chorioamnionitis

Controindicatio
q Clinical chrioamnionitis. (Maternal hypertension and diabetes mellitus are no
contraindications. Careful monitoring and management of hypertension and hy-
perglycemia should be ensured).

Treatment schedule
e lnj. Betamethasone 12 mg lM every 24 hours, 2 doses (preferred).
. lnj. Dexamethasone 6 mg lM every 12 hours, 4 doses (only if betamethasone can
not be arranged).
Timing of effect
a Optimal effect occurs aftet 24 h of initiating treatment.
a The effectofone course lasts for 7 days.

{:t{x, A&s, ,s
rt:u'
i,*,, n } lug *veyy A4, k*urs lii,.cf' ; See ll;i:,vr tt:flttuttlianl
67. Ans. is 'd i.e., FRC is smaller than closing yolume [Ref : Hand book of pediatric intensiye care l't/e p. 138f
"Hyaline membrane disease reduces compliance and causes a fall in FRC (to leyels below the normal closing yolume
of the lung)" Handbook of pediatric intensive care
--
Ans. is 'b' i.e., Theophylline use [Rel: A.P. Ghai Sthle p. 171 6 Vh/e p. 146; Nelson l1,t,/e p. 737, 73g1
Bronchopulmonary dysptasia {BPD}
o BPD is a resuit of lwng iniury in infants requiring mechanical ventilation and suppiemental crxygen.
e Nlost of the children with BPD are premflture and have hyaline membrane disease. But, BPD can also occur in fultr
term newborns with meconium aspiration or persistent pulmonary hlpertension.
Pathogenesis
o The premature lung makes insufficient ftrnctional surfantant and the antioxidant defence mechanisms are not
sufiiciently mature to protect the lung from the toxic oxygen metabolites generated from oxygen therapy.

Premature newborn
J
lmmature lung (.f surfactant)
.t
Hyaline membrane disease
Jr
Oxygen therapy to newborn

Alveolar collapse T
Generation of toxic oxygen metabolites
(atelectotraiuma)
j J
Oxidant damage to lung because in prernature
Ventiiator induceci
newborn the antioxidant defence mechanisms
overdisiension
(volutrauma) are not sufficiently mature
J
Pulmonary edema, lnflammation & hypercellularity
followed by fibrosis and repair
J
Bronchopulmonary dysplasia

tLl ,1.-:"r-*. ir 'a' i.e", 3{itr{3Y,P'* i.?.':i: \e{s*rt l#'i':* {.-i;,:::ttr ii}i.i1:

SevereG,,3,
Supplement Or(for 28 days) Supplement O, (for 28 days) Supplement O, (for 28 days)
and and and
< 32'wereks GA:at birth, r, 'Br,eatlinE room airat36weeks Breathing with < 307o O, at Breathing > 3oo/o O, and/or
36 week corected',GA of at positive pressure ventilotion
, . . ,
-,:,, ,,,,. ., iwhichever.comes first). ,
discharge (whichever come (PPvltn'r) at 36 weeks
first). corrected GA or at discharge
(whichever come first).
:.
,
..1,'

: 32 weeks GA at birth Breathing room air by 56 days < 30olo O, by 56 days postnatal Breathing > 30o/o O, and/or
postnatal age or at discharge age or at discharge (whichever positive pressure ventilation
(whichever comes fi rst). comes first). (PPV) by 56 days postnatal
age or at discharge (whichever
comes first).
GA : gestational age

r The baby in question falls in category < 32 weeks gestation age at birth.
r Sirryly looking at question, answer seems to be sever BPD as mechnnical ventilation (ptositive pressure ventilatian)
orrX:., in diagnostic criteria of sever BPD.
l::rs iar:luded
e But, this question is not as simpie as it is looking. It is tricky one.
o In baby with > 32 gestation age at birth, we will have to see the respiration at 36 weeks corrected gestational age -
r This ba\ born at 27 weeks of gestati6n and required mechanical ventilation for 4 more weeks, i.e. upto 31 weeks
was
corrected gestational age. After that he maintained at room air. Thus, at 36 weeks corrected gestaional age, baby is
breathing at room air + diagnostic criteria of mild BPD.

TRANSIENT TACHYPNEA OF NEWBORN

:0. Ans is t' i.e., Elective caesarean section lkef. O.P. Ghai */e p. 171 & 7e/e p. 145; Variaus artides af Abs & Gynaej
o In text books, both elective caesarian section and normal preterm or term vaginal delivery have been mentioned as
risk factors for transient tachypnea of newborn.
e But the best answer is caesarian section -
"Delivery by caesarian section and gestational age are the risk factors for TTN". Articles Obs & Gvnae
-
7 L. Ans. is 'c' i.e., Prominent horizontal fissures lRef : A.P. Ghai */e p. l7I & 7/e p. 146; Nek*n 18*le p" 741]
r The lungs are generally clear withoui rales or rhonchi and chest roentgenogram shows
t Prominent pulmonary vascular markings - Overaeration
t Fluid lines in the fissure : Flat diaphragms
- Prominent inter lobar fissure s Occasionally pleural Jluid.

72. Ans" is t' i"e., Interlobar fissure effusion lRef : O"F" Gk*i */e p. 171 {z Tele p. 146; Nekar' 1*/e p. 741}

PU LMONARY ALVEOLAR PROTEINOSIS

73. Ans. is t' i"e., Neonatal pulrnonary alveolar proteinosis f&ef : Nekon 18"1e p" 1821!
"Respriratory distress in an infant along with a positive family history of similaily affected newborn infants
strongly suggests, pulmonary alveolar proteinosis".

RESUSCTTATION _ _ __
74. Ans. is 'd' i.e., Trikling of sole lRef: A.P. Gb*i */e p. 128 & flle p. 98-99]
r After suctioning, the baby should be dried by using pre-warmed linen to prevent heat loss.
r A brief tactile stimulation in the form of flicking the soles or rubbing the back may be provided in case of non-
establishment of good respiratory efforts.
75. Ans. is "d i.e., Discontinue ventilation [Re, O.P" Ghai 8*/e p. 127 {r */e p- 99
}{.A"TEB

c But, this question is not as simple as it is looking' It is tricky one'


have to see the respiration at 36 weeks corrected
gestational age:'
r In baby with > 32 gestati.o., ug. at birth, we will
i'e' upto 31 weeks
r and required mechanical ventilation for 4 more weeks,
This baby was born at 27 weeks of gestation
gestaional age' baby is
that he maintalned at room air' Thus' at 36 weeks corrected
corrected gestational age. After
breathing at room air + diagnostic criteria
of mild BPD'

TRANSIENT TACHYPNEA OF NEWBORN


.c i.e., Elective caesarean section l*ef- a.P. Ghei *le p. varia*s articles af *bs & Gynce]
171 6'7ele p' 146;
T&. Ans is
preterm or term vaginal delivery have been mentioned
as
c In text books, both elective caesarian section and normal
risk factors for transient tachlpnea of newborn'
I But the best answer is caesarian section
-
& Gynae
for TTN" '
,,Delivery by caesarian section and gestational age are the riskfactors - Articles obs
.c i.e", Prorninent horizontal fissures lRef : o"P' Gkai */e p' 171 dr */e p' 3'46; Nels*zt lythle p' 741\
71" Ans. is
c The lungs are generally clear withoui rales
or rhonchi and chest roentgenogram shows
s Prominentpit*o'oiy vascular markings t
O-vera.eration
* Flat diaphragms
: Fluid linesin the fissure
z occasionally pleural fluid'
t Prominent inter lobar fissute "
72. Ans.ist'i.e.,traterlob'arfissrreeffirsion lkef :o"P"GkaiSelep'171&Velep'146;Nek*se1*lep'7a1]

Pu LMoHARY ALvEoL4ElBqIFf q09I q


: Nekon lg*le p' 1821\
73. Ans. is t' i.e., Irleonatal pulmonarY alve*lar prcteinosis f&ef
,,Respiratory distress in an infant along with a..positive family history of similarly affected
newbarn infants

stroigly suggests, pulmonary alveolar proteinosis" '

er5{"tsctTATION
i.e., Triklicg of sole lkef: A"p' *hai p' & Ple p' 9s-991
74. Ans. is ? Sele 12s

usjng pre-warmtd
r After suctioning, tnJbaUy shoul<i be dried-bJ
"i:-1-1"- Y;t^t:::,t]::l:i:

0'P" Ghai sele p" 127 {z Ple p' 99


75" Ans" is 'd i.e., Discontinse ventilatioa IReJ!
'

e But, this question is not as simple as it is looking' It is tricky one'


at 36 weeks corrected gestational age:-
o In baby with > 32 gestation age at birth, we will have to see the respiration
mechanical ventilation for 4 more weeks, i'e' upto 31 weeks
r This baby was born at 27 weeks of gestation'and required
Thus, at 36 weeks corrected gestaional age, baby is
corrected gestational age. After that he maintained at room air.
breathing at room air --> diagnostic criteria of mild
BPD'

TRANSIENT TACHYPNEA OF NEWBORN


Ghai */e p. 171 & Tele p' 146; variaus articles af cbs {t cyxae}
78. Ans is t, i.e., Elective caesarem section lRef. o.P.
term vaginal delivery have been mentioned as
o In text books, both elective caesarian section and normal preterm or
risk factors for transient tachlpnea of newborn'
r But the best answer is caesarian section -
,,Delivery by caesarian section and gestational age are the riskfactorsfor TTN"' Articles Obs & GYnae
-
i.e., Prominent horizontal fissures lR$: o'P' Ghai 8*/c
p' 171 & 7/e p" 146; Nekon 1*/e p 741"1
7L. Ans. is'c
and ches|.rolntEenogram shows
r The lungs are generally clear withoui rales or rhonchi
: Prominent pulmonary vascular markings
t Overaeration
t Fluid lines in the fissure
: Flat diaphragms
t Prominent inter lobar fissure
t Occasionally Pleural Jluid.

Yele p. {r Ple p. 145; Nels*x x&*/e p' 741}


Ans. is i.e., Interlobar fissure effirsion lRef : a.P. Ghai
.c, 171
zz.
i
PU LMO NARY ALVEO LAEIROTEI N Oqq
: Nekon 18"1e p' 1821]
73. Ans. is.c i.e., Neonatal pulmonary alvealar proteinosis lRef
,,Respiratory distress in an infant along with a.positive family history of similaily affected newborn infants
stroigly suggests, pulmonary alveolar proteinosis"'

RTSUSCITATION

74. Ans. is lf i.e-, Trikling of sote [Ref A'P' Ghai */e p" 12s e f
p' 9s-991
le

r dried-bf using pre-warmed linento


After suctioning, the baby should be ftl:1'l::-9::
establishment of good respiratory efiorts'
Ts"Ans.is.di"e.,Discontinueventilationl&efa.P.Ghai*/ep.127&fr4ep"99
Birth

o Clearance of meconium
o Active breathing or crying
o Good muscle tone Routine care
o Color -+ Pink
o Term gestation

lnitial steps
o Provide warmth
o Positioning
o Good muscle tone
o Clear airways + suctioning
o Dry stimulate -+ flicking sole or rubbing back
r Orif necessary

I
Supportive care
HR>100&Pink

Apnea or
HR < 100

Ongoing care
HR>100&Pink

76- Ans is'b' i.e., Ist mouth suctioning done & 'c' i.e.,Max. length of nasal suctioning is upto 3 cm and mouth
suctioning is upto 5 cmlRef: A.P. Ghai 8th/e p. 128 6 Vh/e p. 98-994ganual of neonatatiare p" 68, 3S5l
Suctioning during resuscitation "

r The mouth and nose should be suctioned


r The mouth is suctioned first to ensure that there is nothing for the infant to aspirate when the nose is
suctioned.
r one should not insert the catheter very deep in mouth or nose for suction -+ Stimulation of posterior
pharynx during the first few minutes after birth can produce a vagal resp once, causing severe bradycardia
or al,nea.
o Therefore, during oral suctioning, a suction tube is gently introduced into the babyt mouth
until the 5 cm mark rs
at the babyt lip.
I During nasal suctioning, a suction tube is introduced upto 3 cm mark into each nostril.
77. Ans. is 'H i.e., Colour lRef: O.P. Ghai Bth/e p. 126 & Th/e p, 100)
r A newborn is classified as vigrous, ifhe has all the three signs are present :-
1) Strongrespiratary effart
2) Good muscle tone
3) Heart rate greater than 100

L
j] ]. ]: i,:,]i,..,,ji....-:11:..,i i,i. '',: ]v

-8. Ans. is '* i.e., Check pulses lRef: O.P. Ghai 8'h/e p. 126 & Vh/e p. 1001

o After airway has been opened and two rescue breath! provided, determine the need for chest compression. For
this check the pulse in carotid (in children) or branchial artery (in infants).
r If the pulse is not palpable or heart rate is <60/min, begin chest compression.
-9. Ans. is 'c' i.e., Diaphragmatic hernia lRet O.P. Ghai 9th/e p. 129 (t Vh/e p. 100, 1537
r In diaphragmatic hernia, the abdominal contents herniating into thoracic cavity have already comprornised venti-
lation.
r Giving bag and mask ventilation to this patient will worsen the condition of the patient as air will also enter in the
stomach and further compress the lungs.
o Indications of Bag and Maskventilation.
t Bag and mask ventilation is indicated if even after tactile stimulation.
a) Theinfantis apneic or gasPing
b) Respiration is spontaneous but heart rate is below 100 beats/minute.

Remember
Bag and mask ventilation causes abdominal distention as air or oxygen not only enters the lung, but also escapes
into the stomach via esophagus. Distended stomach presses on the diaphragm and compromises ventilation. Therefore
if ventilation continues for more than two minutes, an orogastric tube should be inserted and left open to decompress
the abdomen.

80. Ans. is t' i.e., Convcetion [Ref : 0.P. Gksi $tt'le p. 128 & 7h/e p. 117)

"Conyection warmed incubators are being routinely used for thermal regulation of the premature neonate's ambient
air" - Ghai 6'h/e 154

81. Ans. is 'b- i.e., Dexamethasone ["&e/ This is an extraet of the book PAEDIATRICS pOR DACTORS; Frank Shann
and John Vince; http://www.develapmentgateway.com.au/health/paed,iatrics/PaediatricsFarDoctors-pg236-2 2"pdfl
'Corticosteroids should not be used' - Paediatrics for doctors - Frankshann 6 /ohn Vince

Management Protocol
The management protocol of babies with asphlxia:
1. Oxygen. In the absence of continuous oxygen saturation monitoring, it is reasonable to give nasopharyngeal
oxygen (0.5 litre/min) untii the baby recovers. If monitoring is available, oxygen is given as appropriate.
2. Thermal control. Baby's body temperature should be kept in the normal range of 36.5-37.20C (sometimes the babies
become hlperpyrexic).
3. Correction of shock, If peripheral perfusion is poor, it is reasonable to give 20 ml/kg of normal saline initially. If
perfusion remains poor, the use of dopamine should be considered.
4. Fluid balance. Give IV fluids at 213 maintenance. Use 107o dextrose"
5. Monitor blood glucose with dextrostix and do not let it fall below 2.2 r4mol (explains gJucose administration)
6. Prevent/control convulsions. In less severely affected babies, phenobarbitone should be given when there is
anyrspicion of actual or impending convulsions (phenobarbitone loadid! dose 20 mg/kg IM or 10mg/kg slowly
IV then 5 mg/kg daily orally).
T.Treath)tpocalcaemiaif it occurs (or more practically, if the babyhas uncontrollable fitting with anormal dextrostix).
(explains calcium gluconate administration )

Notes
l.Corticosteroids should not be used, and although many paediatricians use mannitol, there is no evidence for its
effectiveness.
2. Babies with severe asphlxia may appear to settie relatively quickly after the resuscitation - but there is likely to be
a deterioratiorr after 6-12 hours or so as cerebral oedema develops.

82, Ans. is'b' i.e., I [i?ef NeJsor l\il'le ch. 9 .31

r Grimace is given score-1 in APGA.R score.

r Respiratory effort None Slow irregular Good, crying

i,t:i.Heairrnle{p4,IsEJ1 .:,: :'.Ab;er'tt':'1 r..r,,r:. :!a. I'eO ..r:.::. 1


>:11]O:::' i::1.::
* Colorofthebody Blue or pale Body pink extremities pink
(Appearance)
blue
*, Muscle tone,{ArtiVi_ty- 1..,' ',.,., .r,:,,Flaccid .' .
Some fleiionr: Actively mwing extremity
* Reflex stimulation (Grimace) No response Grimace Cries, coughs or sneezes
or putting catheter in nose

Ans" as'W i-e.,&*",ytxxttarv rat*lr,*txtate


?1"1.
iR.e:i" *.p. Gfuai,qtie *. )2* * 7$le ;t. i. li]
: Breathing ffirt (not respiratory rTte) is used in A.pGAR score.

84. Acs is ?' i-e., APGAR at 7 rnia iadicates neonatal mortality depression
[Re! Avery,s pediatrics]
I Later times APGAR score (after 5 minutes) indicates about long term
neurological damage (not neonatal mortality)
lnterpretation of APGAR Score
t The test is generally done at one andfive minutes afier birth, and
may be repeated later if the score is and remains
low. Scores 3 and below are generally regarded as critically low, 4 to
6 fairly low; and 7 to 10 generally normal.
a A low score on the one-minute test may show that the nei,onate
requires medical attention (e.g. resuscitation)but
is not necessarily an indication that there wiil be long-term problems, particulariy
if there is an improvement by the
stage of the five-minute test- lf the Apgar score remains below
3 at liter times such as 10, 15 or 30 mintues, there
is a risk that the child will suffer longer-term neurological damage.
There is also a small but significant increase of
the risk of cerebral palsy. However, the purpose of the Apgar test i
to determine quickly whether a newborn needs
immediate medical care; it was not designed to make long-term predicitions
on a childl health.
CO2 transport across placenta
r
Coz is cleared by placenta by simple diffusion. Co, is produced abundantly
in the fetus, and the pCo2 of fetal blood
is higher than maternal blood. co, therefore diffuses from fetal blood,
through the placenta, into the maternal
circulation, and is disposed by expiration from mother,s iung.
Anaerobic metabolism causes acidemia due to lactate (lactic acid) production
o Anoxic perfusion causes an increase in glucose consumption
which is more than two fold higher than that seen in
the oxygenated perfusion, resulting finally in placental uptake of glucose
not only from the maternal but also from
the fetal circulation.
t
Lactate production i-s increased during the anoxic peifusion,whiie
the final tissue energy value lies between the
values observed for fresh tissue and for the oxygenated perfusion.
The shift to anerobic ietabolism shown by pla-
cental tissue in anoxic conditions enables continued functioning ofthe
tissue over the 2-h perfusion period but it
appear that under anoxic conditions the tissue may incur u, .rr".gy
debt not observed in oxygenated perfusions.
85. Ans. is 'c' i.e.,5 {Ref : Meharban Singh 6a/e p. 2621

0 Synchronized None None None None


1,' Lag.on'inspiration lust visible Just visible Minimal.,. S.lethosco,ire only
2 See- saw Marked Marked Marked Naked ear
r A score of >6 is indicative of impending respiratory failure. -
o Now analyzing our question data :-
l) Upper chest - Lag on inspiration present u score -_) 1
ii) Lowerchest retraction
- No score 0
iii) Xiphoid retraction
- No _) score 0
iv) Nasal flaring - present _+ score 2
v) Grunting - present _+ score 2
o So, total score is 5.

Qri Ans, is 'rj' i.e., &Yl qlt *kl*v* {Re t Netsc}y, xgtr/e p. l{}{}}
o Drug used during neonatal resuscitation : (l) Epinephrine/Adrenalin, (2)
l/S or RL, (3) Naloxoneand (a) Sodium-by-
carbonate. Dose of Adrenaiin 0.1 - 0.3 ml/kg of l: 10,000.
Ans" is k't"c", *"1-*"3 rcL/kgira l:3*,{}*Xl [fr# {,}.p"

Dose or adrenlaine -
0.1 ml/kg to 0.3 ml/kg diluted (1:10,000)
:
Routs (1) Intravenous (umbilical vein) or
(2) Endotracheal
lndication -
HR < 60/min after 30 sec. of positive pressure ventilation & chest compression

HYPERBILIRUBINEMIA

SS. A*s" is'# i"e.n >15 xxlgld{ {1}-e! &leai Tkle 1t. }'a7}

Head & Neck 5 mg/dl

s9. Ams, is 'a' i-e", K*€err c3xdreme* {R.ef: X$**isrtee's 17r'le Ck 7*b!* a3- 11

o Conjugated hyperbilirubinemia is seen when -


i) Impaired secretion of conjugated bilirubin into bile -+ Dubin-johnson synilrome, Rotor syndrome"
ii) Impaired bile flow -+ Obstructive jaundice, primary biliary cirrhosis, Neonatal cholestasis, e.g. ExtratLet:a'ric
biliary atresia / neonate idiopathic hepatitis, Choledocal cyst, Sclerosing cholangitis, Caroli disettse,
Metabolic (Tposinemia, Wolman disease, Nieman pick disease, Galactosemia, Fructosemia).

90. Ans. is 'd' i.e., Hemolytic jaundice fRef: Chatterjee Shinde 4th/e p. 593; Chanilrasoma 3th/e p. 6j5l
o-Important clues provided in question are'-
1) lncreased total bili.rubin lj
2) Normal."r;,rg"*i btlorut" so' t unconjugated bilirubin

o Amongst the given options, only hemolytic jaundice causes increased unconjugated bilirubin.
o Remaining three cause conjugated hlperbilirubinemia'
91. Ans is t' i.e., PNH tRel Readbelowl
o Causes of )aundice since birth are:-
i) Rhincompatibility(erythroblastosisfetalis)
ii) ABOincompatibility
iii) Congenital infections (TORCH)
iv) Sepsis
v) Concealedhemorrhage
vi) Red cell membrane defect (hereditary sphero-cytosis)
vii) Red cell enzyme defect (G6PD deficiency)
o So, option a & d can cause jaundice si*ce birth. ,
o In sickle cell anemia, affected infants do not develop symptoms in the first few months of life because the hemoglobin
produced by the developing fetus (fetal hemoglobin) protects the red blood celis frcrn sickling. This fetal hernogiobin
disappears after 5 month of age so that by 5 months of age, the sickling of the red blood cells is prominent and symp-
toms begin.
o PNH is manfested in adults.
o So, both PNH and sickle cell anemia does not cause jaundice since birth.
o But among these two I would prefer PNH as the answer because it is manifested in adulthood while the patient in
question is a 5-years old child.
o Sickle cell anemia slrnptoms develop at the age of 5 months and it is one of the cause of jaundice (en.wikipedia.org).
Ans. is 'l i.e., Erythroblastosis [Ref, Nelson l9th/e p. 258)
c Er1'throblastosis fetalis (due to Rh incompatibility) is the most common cause ofjaundice
presenting at birth or within
24 hours oflife.
&ytt;. i:r't3' i,r;,,b?"fu r.taxe*xrzy*aZltzigit,z,{iLr:l: :r,rt trbLt..,, 1..,-!t,tt,rt..,:,}
o laundice of Rh incompatibility appears at birth or within 24 hours.
94. Ans. is nb' i.e., congenital cholangiopathy lRef : o.p. Ghai Bth/e p. 122 6 vh/e p. 1491
This child has unconjugated hyperbilirubinemia, while congenital cholanglopathy causes
conjugated
hlperbilirubinemia.

€OI\N6ENITA!. HYPEREILIRU B!NEMIA

95" Ans. is 'd'i.e., Novobiocin therapy [Ref: Robbins Vh/e p. 887, 888]

ttr,'',i;,tl jr;;*:=;tnt $ffi - - i:i:'ij;r'i:'!B:€


''
Physiological joundice of newborn Decreased UGT activity
Breast milk jaundice @ tnhibition of UGT activity
Crigler Najjar synd rome Genetic deficiency of bilirubin UGT activity
Type I m Autosomal recessive / Absent UGT activity
Type ll w Autosomal dominant / Decreased UGT activity
Gilbert syndrome Decreased UGT,activity due to mixed,etiologies .

D iffuse he patace! lula d i seose

96" Ans. is'a' i.e., lVtrild conjugated hyperbilirubinemialRef. Robbin's vh/e p. BB8; Harrisan lVh/e p. 19291
e Gilbert syndrome is a type of congenital unconjugated hlperbilirubinemia.
x lt is autosomal dominant.
o Hepatic biochemical tests are normal
a There is normal hepatic histology, but in some patients increased lipofuscin pigment may occur.
o There is mild increase in unconjugated bilirubin.
{}'7" |z*y.. tt'ttr'i.t:., l,1l *{ atzazv*. 1}tttl: {,{*lsa.rz t8,t'le t:/t. .1.:i,11 \
o In Gilbert syndrome there is unconjugated (indirect) hyperbilirubinemia due to decreased activity of UDPG
transferase,
o usually no treatment is required. But phenobarbitone can be used to reduce the level ofunconjugated bilirubin.
{}{i. *,y'r*. it't:' s.*",13**s'tt$.**t t *ratt {llLl: {.{*ryisez* tythl*, y l,}:tr|

a lnheritance Autosomal recessive Autosomal dominant


e LFT 'Normal .' :' :, : .. Normal : '. 1

* Liver Histology Normal Normal


q Kernicterus Yes Rare...,., . ..i
s UDPG transferase Absent' Reduced
+ Response to phenobarbitone No responsaas thereis absenceof :',Deciease,bilirubin' r.,.
.'
UDPG transferase
{.
* Hemolysis Absent Absent

99. Ans. is "d i.e., Rotor sydrome


* 50% direct bilirubin means conjugated hyperbilirubinemia.
o Normally the direct (conjugated) bilirubin is iess than L5-20o/o of total bilirubin.

Total bilirubin 0.2 - 1.9 mg/dl


Direct bilirubin 0 - 0.3 mg/dt,
o So, this child has : -
0 Conjugatedhlperbilirubinemia
ii) Other LFTs normal
o Amongst the given options, Rotor slT rdrome and Primary biliary cirrhosis cause conjugated hlperbilirubinemia.
o In Primary biliary cirrhosis, other LFTs are also abnormal, e.g., SGOT and SGPT are raised.
. Now we are left with Rotor syndrome which causes conjugated hlperbilirubinemia. Other LFTs are normal.
10S" ,Ans. is 'd' i.e", Crigler-Najjar syndrorn* {Ref: Nelson trVth/e p. 20E4; Ghai 7h/e p. 32a}

101" Ans. is od i.e., &,otor syndrome {l?.ef: Nelson lVt'/e Chap" 3521

102" ,4"ns. is 'd' i.e., ,{ltr of atrove [Ref: Gkai 7h/e p. ja4]
o Mildly elevated bilirubin especially indirect and normal liver enzyme seen in hemolytic anemia.
o In above question all causes hemolytic anemia.

KERNICTERUS

103. Ams. is 'd i.e,, No nong term effect &'H i.e., Occurs with bilirubin rnore than 25 mga/o {R.ef: Read belaw}
r Long term survivors demonstrat e choreoathetoid cerebral palsy, upwaril gaze palsy, sensorineural hearirug loss and
mental retardation.
o Kernicterus occurs when bilirubin level is > 20 mg/dl or > 20 mgo/o.
"It is generally believed that unconjugated bilirubin lewl of more than 20 mg/dl may lead to kernicterus"
pediatrics
--Clinical
"Risk of kernicterus increases with bilirubin levels more than 20 mg/dl and this became the conyentional cut-of pointfor
therapeutic intervention in term neonates" Pediatric essentials
--
o Unconjugated bilirubin causes toxicity to basal ganglion and various brainstem nuclei.
o Opisthotonus is seen in phase II of kernicterus.

f 04. Ans. is'd' i.e., Sulfonamides [Rel KDT Sn/e p. 644; Goad.man Gilman 10th/e p. 117fl
The administration of sulfonamides to newborn infants, especially if premature, may lead to the displacement of
Bilirubin from plasma albumin. In newborn infants, free Bilirubin can become deposited in the basal ganglia and
subthalamic nuclei of the brain. causing an encephalopathy called kernicterus. Sulfonamides should not be given
to pregnant women near term because these drugs pass through the placenta and are secreted in milk.
Kernicterus is due to unconjugated hyperbilirubinemia.

105" Ans. is'a'i.c,, Hypotoni* ltlef: Gkai $'h/e p, 172)


o Hypotonia is an early feature (in Phase I).
f 06. Ans. is t' i.e., High levels of conjugated bilirubin may cause Kernincterus [Ref O.P. Ghai vth/e p. 173-174 6
Vhle p. 14fl
o Kerruincterus is caused by high lev els of un- conjugated bilirubin.

TREATMENT OF HYPERBILIRUBINEMIA

lO7. Ans. is 'd i.e., Barbiturate lRef O.P. Ghai 6th/e p. 172, 173 6 Vh/e p. 1501
Barbiturates (Phenobarbitone)
o It is effective only if given to mother before delivery.
o It acts by inducing the conjugation of bilirubin -+ Phenobarbitone is an enzyme inducer.

108. Ans. is t'i.e., Structural isomerization [Ry'; Manual of Neonatal Care Sthle p. 2A8l
o "Structural isomerization is the intramolecular. cyclization of bilirubin to lumirubin. It is the most important
pathway for the lowering of serum bilirubin levek! - Manual of Neonatal Care 5/e, p 208

109. Ans. is 'a' i.e., Phototherapy I Ref : O.P. Ghai thle p:-175 dz 6hle p. 17j)
Bronze baby syndrome -
r It refers to dark grayish brown discoloration of the skin in infants undergoing phototherapy. Almost all infants ob-
served with this slmdrome have had a rnixed type of hlperbilirubinemia with significant elevation of direct reacting
bilirubin and often with other evidence of obstructive liver disease.
o So, this child has : -
i) Conjugatedhlperbilirubinemia
ii) Other LFTs normal
o Amongst the given options, Rotor syndrome and Primary biliary cirrhosis cause conjugated hyperbilirubinemia,
o In Primary biliary cirrhosis, other LFTs are also abnormal, e.g., SGOT and SGPT are raised.
o Now we are left with Rotor syndrome which causes conjugated hlperbilirubinemia. Other LFTs are normal.

na'
XSl" Ams, is i"c., Rotqlr syndreir*e {{lef: Nelsaw 1.7t'/e Chap^ 3521

L&2. Ans" is 'd' i"e", AII mf above {Ref: Gkai Thle p" 3a4)
o Mildly elevated bilirubin especially indirect and normal liver enzyme seen in hemolytic anemia.
o In above question all causes hemolytic anemia.

KERNICTERUS

XS3, Ams. is 'd 2.e., N*r lmng term effect &'b' i.e., Occurs with bilirutrin more tban25 wegak {R.ef: &eadbelaw}
o Long term survivors demonstrat e choreoathetoid cerebral palsy, upward gaze palsy, sensorineural hearing loss and
mental retardation.
r Kernicterus occurs when bilirubin level is > 20 mg/dl or > 20 mgo/o.
"It is generally believed that unconjugated bilirubin level of more than 20 mg/dl may lead to kernicterus"
pediatrics
--Clinical
"Risk of kernicterus increases with bilirubin leuels more than 20 mg/dl and this became the conyentional cut-of point for
therapeutic intervention in term neonates" P ediatr ic es senti als
--
o Unconjugated bilirubin causes toxicity to basal ganglion and various brainstem nuclei.
o Opisthotonus is seen in phase II ofkernicterus.

104. Ans. is 'd' i.e., Sulfonamides [Rel KDT 5'h/e p. 644; Goodman Gilman 10th/e p. 1176]
The administration of sulfonamides to newborn infants, especially if premature, may lead to the displacement of
Bilirubin from plasma albumin. In newborn infants, free Bilirubin can become deposited in the basal ganglia anil
alo be ven
, . .: t.j.l::i::::,rri:'::l:;..:
:,, ::.r,.
i ... : .rl:ii::ii.:i .r ii:ir,i
.,' :.:ii-;. r'r,'ii..:nl,:
. ...t,::t.,:....: ):.-;;r.a.)

r So, this child has : -


i) Conjugatedhlperbilirubinemia
ii) Other LFTs normal
r Amongst the given options, Rotor syndrome and Primary biliary cirrhosis cause conjugated hyperbilirubinemia.
o In Primary biliary cirrhosis, other LFTs are also abnormal, e.g., SGOT and SGPT are raised.
o Now we are left with Rotor syndrome which causes conjugated hlperbilirubinemia. Other LFTs are normal.

nCI0, ,&ns. is 'd' i"e", Crigler-Najjar symdrorne lRef: Nelsow tr9th/e p. 20E4; Ghai 7h/e p. 32Al

101" "&ns. is 'd 1.e., Kotor syndrome {flef: Nelsow tr7t'/e Chap. 3521

1CI2" Ans. is 'd' i"e., ,&ll of above l&ef: Glaai Th/e p. N_<1)
o Mildly elevated bilirubin especially indirect and normal liver enzyme seen in hemolytic anemia.
o In above question all causes hemolytic anemia.

KERNICTERUS

1&3, Ans" is 'd i,e.,lnlo long term effect &'tr' i.e., Occurs with bilirut in more tbax 25 mgYa lRef: Read belawl
o Long term survivors demonstrat e choreoathetoid cerebral palsy, upward gaze palsy, sensorineural hearing loss and
mental retardation.
o Kernicterus occurs when bilirubin level is > 20 mgldl or > 20 mgo/o.
"It is generally believed that unconjugated bilirubin level of more than 20 mg/dl may lead to kernicterus"
pediatrics
--Clinical
"Risk of kernicterus increases with bilirubin levels more than 20 mg/dl and this became the conventional cut-of point for
therapeutic intervention in term neonates" Pediatric essentials
--
o Unconjugated bilirubin causes toxicity to basal ganglion and various brainstem nuclei.
r Opisthotonus is seen in phase II of kernicterus.
104. Ans. is 8' i.e., Sulfonamides [Rel KDT Sth/e p. 644; Goodman Gilman 10th/e p. 11761
The ailministration of sulfonamiiles to newborn infants, especially if premature, may lead to the displacement of
Bilirubin from plasma albumin. In newborn infants, free Bilirubin can become deposited in the basal ganglia and.
subthalamic nuclei of the brain. causing an encephalopathy called kerhicterus. Sulfonamides should not be given
to pregnant women near term because these drugs pass through the placenta and are secreted in milk.
Kernicterus is due to unconjugated hyperbilirubinemia.

X05. Ans. ie *d i.e., Hypotonia fllef: Ghai 6't'/e p, 172)


r Hypotonia is an early feature (in Phase I).
f 06. Ans. is t' i.e., High levels of conjugated bilirubin may cause Kernincterus [Rd O.,R Ghai \th/e p. 17j-174 d,
7h/e p. 147)
o Kernincterus is caused by high lev els of un- conjugated bilirubin.

TREATMENT OF HYPERBILIRU BINEMIA

lO7. Ans. is'a' i.e., Barbiturate lRef: O.P. Ghai 6h/e p. 172, 173 6 Vhle p. 1501
Barbiturates (Phenobarbitone)
o It is effective only if given to mother before delivery.
o It acts by inducing the conjugation of bilirubin -> Phenobarbitone is an enzyme inducer.

f 08. Ans. is t' i.e., Structural isomerization [Relr Manual of Neonatal Care Sth/e p. 2081
o "structural isomerization is the intramoleculary cyclization of bilirubin to lumirubin. It is the most important
pathway for the lowering of serum bilirubin levekl - Manual of Neonatal Care 5/e, p 208

f 09. Ans. is 'd i.e., Phototherap y I Ref : O.P. Ghai t;nt, p. vs 6 dhle p. 17j)
Bronze baby syndrome -
o It refers to dark grayish brown discoloration of the skin in infants undergoing phototherapy. A1most all infants ob-
served with this syndrome have had a mixed type of hyperbilirubinemia with significant elevation of direct reacting
bilirubin and often with other evidence of obstructive liver disease.
r l0'
Ans' is 'a' i'e', 2omg% [Ref cpDT r*/e Figure(I-3]
Nerson r8h/e p. 7641
o in full term newborn phototherapy is
indicated when total serum bilirubin
birth; (ii) > 18 mg/dl in 48_72hrs. after birth; level is : (i) > l5 mg/dl in24-4ghrs.
and (tii);_';; ;g/dl more than 72 after
hrs. after birth.
111' Ans'isk'i'e.,Exc&amg*tr&lxsfu*ic:aa
isrri e"r; Gia*i8rr*p.:"rs-:z*";;;p ,r'--ri*
' ff:::1ifffl:[1t;ii['fi];Jl,ffi1Tf to weight (< rooo gm) birirubin rever
orthis newborn (13 mgo/o or 13 ms/
rl2' Ans' is h'i.e., Totar and direct bilirubin
[Ref Nelsoa lPh p. 75sl
o Measurement of total and direct bilirubin
will make us available with all the three parameters
bilirubin levels' These will help to classify jau"ai." i.e. direct, indirect, total
into-rtr..lp..ti". type and aid in diagnosis.

ilTEoNATAL HYPOG LYCEM tA

r13' Netson textbook of pediatrics


#$:;id2;"'lfrf*f,ffjief tule p. 816, lele p. slz; hup//www.unicef.org/india/

Agre

Between 1-3 hours of life


,eetwein irza h6ris;*ife
After 24 hours
Older infant and children >2 months

ll4. Ans. is t'i.e., post term infant


[Ref Nelson 1*/e p zl5l
t Neonatal hypoglycemia is seen in preterm infants (not
post term)

l15' Ans' is'b'i'e., r0% IV Dextrose [Ref, Essence of peditrics (Elsevier, rndia)
4e/e p.44,451
t
symtomatic (Lethargy) Hypoglycemia (<4hmg/dl)
should be managed i,tith 10o/o IV Dextrose,
116" A:rs. is t' i"e." 1$% dextrqls x 4 xallk g,
iRri Nefs*zr iS&le y, l)31
o In seizures, dose of l07o dextrose is 4
ml/kg.
l1?. Ans. is'c'i.e., 12"S A,{IMSi{e*w$taansgr iwl}t **{ - *€9,7ttzg$r*rxziqe{l.Ne*qg$x.nl
{fie_f
r
Dont give > l2'5o/o dextrose infusion through peripheral
line because of risk of thrombophlebitis. (prefer
18. central line)
I Ans. is t' i.e. History of Unconsciousness
r Neonates are susceptible for developing hypoglycemia
during the first 2-4 hours of life.
' is not started wi{r,in ulnhour of brrih about
r0% of normal ,,eor,"t"s are likely to develop
dffi:t"?tfeding
o This hypoglycemia is corrected either
by starting breast-feeding or by administering
glucose.
' ,l#1,i,[:ff:etic
mother is prone ror hipoglv;mia -+ gt,r.o-r.
solution is given #iire child deverops symptoms
of
o In a child with history of hlpoglycemia,
glucose administration is indicated if the
spnptoms of hypoglycemia recur.

NEONATES DIABETIC MOTHER

I19. Ans. is 'H i.e. Hyperglycemia


[Ref O.p. Ghai 86/e p. t8t & fl/e p. 156; Nekon pediatrics lple p.
t At birth the separation of placenta suddenly interrupts 613_6141
glucose infusion into the neonate without
a proportional
t Hyaglycemia -+ which is nonketotic
t Decreased lipolysk
o In addition these infants have following
abnormalities:
r Hyperbilirubinemia r Transient tachyapnea of the newborn
r Polycythemia . incidence.i..rpi*a.y disrress sl,ndrome
r Hypocalcemia Tigl:.
r Caidiomegaly
r Hypomagnesemia r As)rmmetric septal hlpertrophy
120. Ans. is'V i.e.,Caudal Regreseion lRqf : Readbelowl
'The fetal malformations seefl in Caudal regression syndrome are the most typical seen in cases of maternal
diabetes'.- Early detection of caudal regression syndrome, D. Subtil et al.

Caredal regr€siion:
o Caudal regression syndrome (CRS) is a rare malformative syndrome seen mainly in cases of maternal diabetes with
poor metabolic control.
r This syndrome associates vertebral agenesis of variable level with genito-urinary and digestive malformations.
These abnormalities are related to the impaired development of the mid-posterior axis mesoderm

l2l. Ans. is'l i.e.,Hypa$y*wia


o Electrolyte disturbances, includinghypoglycemia andhypocalcemia, are commonly seen in association with infants
of diabetic mother and may result in early-onset seizures.
o Sorry friends, I could not find any reference which hasdirectly mentioned that hlpoglycemia is a more common cause
of seizure than hypocalcernia in infants of diabetic mother.
c I was just able to find following two statements :-
"Infants of diabetic mother are most at risk for hypoglycemia which can result in seizure". - Clinical Pediatrics
"Infant of diabetic mothers with seizures, that does not respond to glucose should have their serum calcium
measured" - Cloherty
o From these two statements, it seems to me that hlpoglycemia is a more common cause of seizures in infants of diabetic
mother.

MISCETTAHEOUS

122" Ans. is'ff i.e., .&71erf abovc lR$: Nl{F manual}


l(argaroo mother care (KMC)
r Special way for LBW body
t Benefit
1) Eftective Temperature control 5) Weight gain
2) Breast feeding 6) Apnea decreases
3) Infection control 7) Mother satisfaction
4) Parentalbonding
s Component
1) Skin to skin contact
2) Exclusivebreastfeeding
TheKtrilCprocedure /
c The baby should be placed between the mother\ breasts in an upright position.
s The head should be turned to one side and in a tlrChtly upturned position. This position helps in breathing af and
allows eye-to-eye contact between the mother and her baby.
s The legs and arms should be folded.
t Baby\ abdomen should be at the level of the mother\ upper abdomen"
: Support the baby bottorn with a sling/binder.

123. Ans.isnc'i.c.*lOweeksfR.ef:AbdaminolW,altHernias:'Principlesandmanagcm:entbyRabertBmdavip.5g4l
c During development the intestinal loops lie outside the abdominal cavity in the umbilical cord called the physiological
umbilical hernia.
r The loops go back into the abdominal cavity by 10uh week of gestation"

124. ,4ns. is "H i.e., 125 mgldl lRej': Indian Pediatrics 20{t8;45:29-38},
o No established definition of neonatai hlperglycemia and upper safe limit of blood glucose has been determined
o Various researches has suggested
Whole blood glucose > L25 mgldl
Plasma glucose > 150 mg/dl

125" Ans. is
ol i.e.,Morphi*e *
{Ref : Avery's d**ase af newborrc /e p. 442}
e Morphine and other opioids can cause neonatal respiratory depression.
126. Ans is 'a' i.e., Opioids fRef: Cloherty 6h/e p. 220, 237; Auery,s disease of the newborn gth/e p. 4421
127. Ans. is t' i.e., Seizure lRef : o.P. Ghai 8th/e p. 167 6 Vh/e p. l4t; Nelson lgth/e p. 718; CpDT lSth/e p. ill
Clinical features of hypoxic ischemic encephalopathy
r Encephalopathy progress over time -
1) Birth to 12 hours -+ Decreased level of conciousness, poor tone, decreased spontaneous movement, periodic
breathing or apnea, seizures.
2) 12-24 hours ) More seizuers, Apneic spells, jitteriness, weakness.
3) After 24 houts -+ Hypotonia, J conciousness, poor feeding, brainstem signs (oculomotor) and pupillary
disturbances.

128. Ans. is'b' i.e., Differential hypotonia (lower limbs > upper limbs) fRef. a.p. Gbai 8th/e p. 166, 167 6 vh/e p.
139-MA; Nelson 18'h/e p. ZISI
o During asphyxia the infant may remain hypotonic or change
from hypotonia to extreme hypertonia or their tone
may appear normal. These changes are seen simultaneousty and the change in muscle tone
is also of the same degree in
both the limbs.

129 ' Ans' is 'd' i.e., State 6 [Ref : Brazelton TB' The Neonatal intensive care unit network neurobehaviaral scales procedures.
Pediatrics 20a4; I 1 3:641 -67)
r Crying lustily n'ith all limbs moving (child in question), straight away goes in state 6.
."ry-
r 3&" 'A"ns' is i.e., 25 crn of F{r0 fRef: Texthoak af paediatric errxergerxcy nzedicine - peter catner*n, George
felirzek, Ia*
Everitt p. 49; obstetrics: Narmal and prabtew Ttregruarocies by Steven G.Gsbbe, R. Nieb),t p. 4g6llennifer
r Higher inflation pressure of about '25 to 40 cms of Hro and higher inflation time > 1.5 sec. may be required for first
several breaths (=5).
r The volume of first breath is between 30-67 rnl.
o Residual volume after first breath : 4-30 ml (average : 16,20 ml).
o By around 30 minutes most neonate achieve normal FRC.

l3l. Ans is 'a' i.e., Pyloric stenosis fRef: Neanatology by Meharban singh p. loSl
c A large number of normal babies vomit on first day due to irritation of stomach by swallowed
amniotic fluid
o The regurgitation / vomiting in a neonate is due to -+ faulty feeding techniques
or, aerophagy
o Eosophageal atresia may present with vomiting on the first day.
r In hypertrophic pyloric stenosis vomiting characteristically occurs after 2 weeks of age. (2-3weeks).
132. Ans. is H' i.e., Impedence technique lRef : Care of the newbarnby Meharban singh 6h/e p. 30, 2801
r The respiratory monitor based on impednace technique measures changes in the electrical
resistance during breathing.
The electrode is fixed on the chest wall to pick up signals which are displayed as respiratory
rate.
o capnography - It is a simple non invasive method to assess arterial cor.
r It is used to assess the placement of ET tube in esophagus or trachea.

133. Ans. is 'b'i.e., Toxoplasmosis [Rel Moffets 4th/e p. 46jl


r This question is straight forward; spiramycin is in pregnancy is given for toxoplasm osis. prenatally acquired
T'gondii may infect the brain and retina of the fetus and can cause chorioretinitis, intracerebral
calcifications,
and hydrocephalus,
o Cerebral calcification and hydrocephalus may also occur with congenital CMV
and HSV infection, but spiramycin is
not given for these infections.

134, Ans is'H i.e., Toxoplasmosis [Rel Williams 27dle p, SB]

135, Ans. is'c'i.e., < 3SD for age and sex {Ref: Nelsan lgth/e p. 2aA\
I Microcephaly is defined as a head circumference that measures more than three standard deviation
below the mean
for age.
.Cllap.t:Eirii

136. Ans" is t'i"e., Congenital variceltra syndrorne {Ref Nelsan tr811'/e p. 77$7

Causes of Hydrops fetalis


lmmune
e Rh incompatability
Non-immune
c Anemia o-thalassemia, G-6PD defi ciency
r Cardiacdysarrhythmias Supraventricular tachycardia , AF, congenital heart block
o Structual cardiac de- Tricuspid insufficiency, ednocardial cushion defect, cardiomyopathy, hypoplastic left heart,
fects premature closure of foramen ovale
* Vascular Chorangioma of placenta, Twin-Twin transfusion, umblical artery aneurysm, thrombosis
of renal or umblical vein or IVC
* Lymphatic Lympha n giect asia, cysti c hygroma, Noona n synd rome
r CNS Encephalocele, intracranial hemorrhage
e Thorasic Mediastinal teratoma, diaphragmatic hernia
& Teratomas Choriocarcinoma, sacrococcygeal teratoma
o Tumor & storage dis- Neuroblastoma, hepatoblastoma, Gaucher disease,
eases

Niemann-Pick disease, Mucopolysaccharidosis


e Chromosomal Trisomy 13, 1 5, 1 6, 18, 21
* Bone diseases Osteogenesis imperfecta, skeletal dysplasias
o Congenitalinfections CMV rubella,Toxoplasmosis, Syphilis, Parvovirus, Leptospirosis, chaga's disease
* Others Congenitol nephrosis, Myotonic dystrophy, lnfant of diabetic mother, Maternal therapy
with indomethacin, Hepatic fi brosis

137, Ans. is t' i.e., Large proportionate body fRef: Nelson 18th/e p. 7811
r Characterististics of fetai alcohol syndrome inciude : -
1) IUGR (not large proportionate body) 4) Mental retardation
2) Microcephaly 5) Minor joint anomalies
3) Congenitalheart defects (ASD, VSD) 6) Hlperkinetic movements
7) Facial abnormalities -+ Short palpebral fissures, epicanthal folds, maxillary hlpoplasia, micrognathia, low set
ears, smooth philthrum, thin smooth upper lip.
138. Ans. is t > d' i.e., Apt test > Kleihauer Betke test [Relr Perinatal transfusion ?ned.icine 2d/e p. 218]
j.*.*=.r*..,
i*g[t4'S-;'
sourc::J::mry. M,ut"lul or neonatll . .
Mut9rnlt
.
How jt,Wolkl :: r : '.,,. fiil6[i6g'1'%-NaOfJ,destigysiduk.llbAibut. Addingnciddestroys adult HbA but noi fetalHbF ,:,.

..:,"i, ,r:::.t '


..,:.'11,.,,,, ..ngletai:t+bf r::',:,.:r: ::,: .i::i:' .- ',
.:"'
Assessment type Qualitative Quantitative

o Both Apt test and KB test are used to differentiate maternal blood from fetal blood. So, trvo options are correct.
r However, we have to choose one option and according to me Apt test is best here because Apt test is used
specifically for differentiating maternal and fetal blood while KB test is used to quantifli fetomaternal hemorrhage.

139. Ans, is'a'i.e", Large size baby


o Macrosomia refers to newborn with excessive weight, i.e. large for date. Causes are maternal diabetes and maternal
obesity.

ANSWERS OF VARIOUS OTHER EXAMINATIONS

140. Ans. is t' i.e., 28 weeks lRef: Meharban Singh S'd/e p. 71)
136" -&ns' isne'i.e,,C<ragemitatrvaricellasy&dr${TelRef:Nels**{*il'lep'77a}
Causes of HYdroPs fetalis

lmmune
c Rh incomPatability
Non-immune
q-thalassemia, G-6PD defi ciencY
o Anemia
, congenitol heort block
c CardiacdYsarrhYthmias Supraventricular tachycardia AF,
hypoplastic left heart'
ednocardial cushion defect, cardiomyopathy,
o Structual cardiac de- Tricuspid insufficiency,
fects premature closure of foramen ovale

* Vascular Chorangiomaofplacenta,Twin.Twintransfusion,umblicalarteryaneurysm,thrombosis
of renal or umblical vein or IVC
syndrome
* Lymphatic Lymphangiect asia, cystic hygroma, Noonan

o CNS Encephalocele, intracranial hemorrhage

* Thorasic Mediastinal teratoma, diaphragmatic hernia

a Teratomas Choriocarcinoma, sacrococcygeal teratoma

* Tumor & storage dis- Neuroblastoma, hepatoblastoma' Gaucherdisease'


eases
Niemann-Pick disease, Mucopolysaccharidosis

* Chromosomal TrisomY 13, 1 5, 1 6, 18, 21

c Bone diseases Osteogenesis imperfecta, skeletal dysplasias


Leptospirosis' chaga's.disease
* Congenitalinfections CMV rubella, Toxoplasmosis, Syphilis, Parvovirus'
therapy
Congenitol nephrosis, Myotonic dystrophy' lnfant of diabetic mother' Maternal
6 Others
\ivith indomethacin, Hepatic fi brosis

lRef: Nelson 18'h/e p. 7811


137. Ans. is t' i.e., Large proportionate body
include : -
r Characterististics of fetal alcohol syndrome
4) Mental retardation
1) IUGR (notlarge proportionatebody) 5) Minor joint anomalies
2) MicrocePhalY ySD) 6) HlPerkinetic movements
3) Congenitalheart defects (ASA epicanthal foids, ma*illary hlpoplasia' micrognathia'
low set
7) Facial ab"ormalities + Short palpebral fissures'
lip'
upper
ears, smooth philthrum' thin smooth
Perinatal transfusion medieine 2d/e p' 2181
138. Ans. is'c > d i.e., Apt test > Kleihauer Betke test [Re/r

lvlolcl I lol
Source of samPle Maternal or neonatal
Adding acid destroy!'aduti HbA but'no!fetal'HbF
How it'works.., :' Adding loloNaOH destloys adult HbA but
notfetal HbF
Ouantitative
Assessment tYPe Qualitatlve
Positive neais blood ls af fetal o tigln :'' Reboited'in estimated''ml3of futal,blood..''
Results: .: .:: i:
are correct'
maternal blood from fetal blood' So, two options
r Both Apt test and KB test are used to differentiate
rHowever,wehavetochooseoneoptionandaccordinqtomeApttestisbestherebecauseApttestisused
to quantifli fetomaternal hemorrhage'
and fetal blood while KB test is o..d
specifically for differentiating maternal

139. Ares. ie 'a'i.e., I-arge slz"ebaby i diabetes


,. r- a^^ and
^-r mater..'-
"-^1
newborn with excessive weight, i.e. large for date. causes are maternal
o Macrosomia refers to
obesitY.

ANSWERS OF V
Mehatban Singh S'd/e p' 711
140. Ans. is t'i.e., 28 weeks lRef:
Aas, is'd i.e.,Loes ofweight [Ref: Netson l.*/e p. 220" 22107
r An infant should gain weight progressively, except for the first few days of lifb.
r Liver, spleen and kidney may be palpable ia normal newborn.
Nelson
o Sometimes trachea may be deviated from midline without any significance. --
142. Ans, is'l i.e",Autonomic dysfunction {Ref: Nelson t*/e p. 2262}
o If reflects an imbalance in the autonomic vascular regulatory mechanism.

143" Ans. is '8 i.e",30-40 breathc/minute {Ref : op Ghai &/e p. 145 d? p/e p. 10sl
o Heart rate in neonate -) l2O-l4O per minute.
I Respiratoryrate -+ 35 to 40 per minute.

lM. Anc, is'd i.e,,120-tfilm,-{Ref: O.p, Ghai tr/e p. 145 {r p/e p. 105)
145, Ane" ic 'V Le,,Birthweight below t0* percentile {Ref: Internetl
c lVlicrosomia means small for date baby, i.e. newborn baby with a birth weight less than 10e percentile.
l#. Ans, ie 'd i"e.,Erythcrnatouc papular pustular leeion * {Ref : Dutta #/e p. 446; Gupte S. */e p. 522)
Common neubom raches
r Erlthema toxicurn -> Most common
r Acne neonatorum
r Transient neonatal pustural melanosis.

147. Ans, ic '8 i,a., Diabetec {Ref: Has been explained)


r Maternal diabetes predisposes to HMD.

148" Ans. ic *A i.e,tvleconium aepiration


o Most common cause of respiratory distress in a term or post-term neonate -+ Meconium aspiration.
c Most common cause of respiratory distress in a preterm infant -+ hyaline membrane disease.

149, Ane. io'il i,e,,5-7 weeks{Ref A.P" Ghai *le p" 141 6 Zh/e p. fi2; Nelson 18tu/e p. 214)
r It disappears between 2 week to 3 months.

150. Anc. ie 'C i.e., Olset of reepiratory distresc is immediately after birth and it rarely laste beyond 4g hrs
{Ref O"P, Ghai */e p" tz1 & Zh/e p" t46l
151, Ano, ic 'd i.e., cerebral palsy &'U i,e.,rlypoxic lcchemic encephalop *hy {Ref: Internetl
r Normal posture of a full term neonate is flexor posture.
o Extensor postlrre may be seen when there is brain damage as in hlpoxia --> hlpoxic
ischemic encephalopathy and
cerebral palsy.

,52, Anr, ic'* i,e,,Transient tachypnea of newborn fRef: o,p, Ghai gth/e p, l7l & ztu/e p. rufl
I Respiratory distress, which resolves within 24 hours without any respiratory support and fluid in interlobar
fissure
on chest X-ray suggest the diagnosis of TTN.

153. Ane" is 'il i,e,, Any time during pregn fRef: Read below]
^cy
r Fetal growth is affected by maternal nutrition during any trimester in pregnancy.
r Fetal growth is affected even by nutritional status of mother prior to pregnancy.

154. Anc, ie 'H i,e., Bacterial flora fRef: Has been explained]
r Meconium contains -
i) Intestinal epitheliai cells iii) Mucus v) Bile
ii)Lanugo iv) Amniotic fluid vi) Water

155, Ans is '* i.e., Kangarooo Mother Care (KMC) lRef, O.P. Ghai Bth/e p. 14s, 148 6 Vhle p, 121,153; K.N.
Agrawal l,t/e p, 1A4l
"Preferably mother should accompany and baby can be transported in KMC position.
Even father can provide KMC during
transport if mother can not acompany. _ K.N. Agrawal
i.e., cord bilirubin is more than 5 mg;
156. Ans. is .d i.e., cord blood hemoglobin is less than l0 g %;'H
.c,
i.e., History of previous sibling affected [Ref lVelsolr
l*/e p- 77ll

lndications of Exchange transfusion


o Bilirubin Protein ratio > 3.5 o PrematuritY
o Cord hemoglobin < 10g/dl
e Reticuloclte count > 1570
o Cord bilirubin > 5mgld
o Previous kernicterus or severe erythroblastosis in a sibling

p'
157. Ans. is t'i.e., Cord bilirubin < 5 mll00 ml [Rel: Nelson
18e/e 7711

I Cord bilirubin 5 or more is an indication'


.H i.e., The total bilirubin concentration in the serum increases by r mgldl per day lRef': o'P' Ghai
r5g. Ans. is
8*le p. 172 & Ple p. 147; Nelson lf le p. 594\
o Rate of rise is less than Smgo/o
l*le p' V58l
159. Ans. is'b'i.e., Rh Incompatibility [Re/: O.P. Ghai 8*/e p' 174 6 ffle
p' 172; Nelson

of Newborn by Maherbarcingb 6h le p' 336; Nelsott


r 60. Ans. is ? i.e., Begins between 2-7 days of Me fRef: cate
lSele P.7731
Hemorrhagic disease of the newborn
o Early onset -) 0-24hr
r disease ->
Classical 2-7 days
r Late onset -) 1-6 months

f 61. Ans. is'd bilirubin tRef Has been explainedf


i.e., Unconjugated
hours lRef Clohetty 5*/e p' 187)
162. Ans. is'b' i.e., Stop breast feeding and review after 24
r This is a case of breast milkiaundice' may be
r Temporary interruption of breast milk feeds will dramatically reduce the serum levels of bilirubin and there
but it never reaches the previous levels'
slight increase in bilirubin when breast feeding is resumed,

f 63. Ans. is 1d i.e., Measurment of ct - fetoprotein [Ref : Nelsorc l*le p' 6951

164. Ans. is'C i.e., 60 [Ref: O.P. Ghai }e/e p' 381 & *le p' i52l

Tachypnea
o < 2 months -+ 60 or more RR/min
o 2-12 months ) 50 or more RR/min
o 12-60 months -+ 40 or more RR/min

165. Ans. is'H i.e., 3 hours [Ref Netson 18th/e p' 655]
o Blood glucose reaches its nadir in initial 2-3 hrs' of life'
; A'P' Ghai Sele p' 154 & 7e/e p' 126i
166" Ans. is'b' i.e., Lower areola more visible [Ref

Signs of good attachement of baby's mouth to nipple


1. The babY's mouth is wide oPen one'
2. Most oithe nipple and areola in the mouth, only upper areola visible, not the lower
3. The baby's chin touches the breast'
4. The ba\t lower liP is elevated'

167. Ans.is.b,i.e.,BacteriuriatRef o.P. Ghai8th1ep.468&6h/ep.443;SmithGeneralurologylvhlep'491

III
P T E'R

CAKffiXffi\{&SCq"-fX"r&K
SYSTEM

FETAL CIRCULATION

o Fetal circulation differs from adult circulation in several ways. Almost all differences are attributable to the fundamental
difference in the site of gas exchange. In the adult gas exchange occurs in the lungs, while in the
fetus placenta

provides the exchange of gases and nutrients.


Course of fetal circulation
A) Blood returing to heart
o Bloodoxygenatedintheliacentareturnsbywayoftheumblicalvein?Gt|T),whichenterthefetusattheumblicus
and course through to join the portal vein'
o tine iluctus venotts is a blpass shunt between portal vein and IVC'
o Most of the umblical venous blood shunts through the ductus venosus to the IVC.
o Blood frorr,lVC (which carries the blood drained from lower extremities, umblical veins, hepatic veins and renal
return of
veins) enters the right atrium. On reaching the right atrium, about 1/3rd of total inferior vena caval
blood enters the left atrium through formen oval. The rest two third mixes with the venous return from SVC
(which carries the blood drained from head, neck and upper limbs) to enter the right ventricles.

B) Blood coming out of the heart


1. Right side
o The right ventricle pumps out the blood into pulmonary trunk -+ very small amount of this blood enters the
pulmonary circulation because the lungs are collapsed and pulmonaryvascular resistance is high. Most of this
blood passes to desending aorta through ductus arteriosus, a shunt between pulmonary trunk and desending
aorta
Aorta. Descending aorta carries blood to umblicus through umblical artery, for oxygenation. Descending
also supplies the blood to lower lumbs, kidney and abdominal viscera.

2. Left side
o The left ventricle pumps blood into the ascending aorta for distribution in coronary circulation, head and upper limbs.

llmblical cord contains one umblical vein and two umblical arterieswEEr)
CAKMKffire$CULAK
$YSYKivg

FETAL CIRCULATION

o Fetal circulation differs ffom adult circulation in several ways. Almost all differences are attributable to the fundamental
difference in the site of gas exchange. In the adult gas exchange occurs in the lungs, while in the fetus placenta
provides the exchange ofgases and nutrients.
Course of fetal circulation
A) Blood returing to heart
o Blood oxygenated in the placenta returns by way of the umblical vein@Groz), whichenter the fetus at the umblicus
and course through to join the portal vein.
o The ductus yenous is a bypass shunt between portal vein and IVC.
o Most of the umblical venous blood shunts through the ductus venosus to the IVC.
o Blood from fVC (which carries the blood drained from lower extremities, umblical veins, hepatic veins and renal
veins) enters the right atrium. On reaching the right atrium, about 1/3rd of total inferior vena caval return of
blood enters the left atrium through formen oval. The rest two third mixes with the venous return from SVC
(which carries the blood drained from head, neck and upper limbs) to enter the right ventricles.

B) Blood coming out of the heart


1. Right side
o The right ventricle Pumps out the blood into pulmonary trunk -+ very small amount of this blood enters the
pulmonary circulation because the lungs are collapsed and pulmonaryvascular resistance is high. Most of this
blood passes to.desending aorta through ductus arteriosus, a shunt between pulmonary trunk and desending
Aorta. Descending aorta carries blood to umblicus through umblical artery, for oxygenation. Descending aorta
also supplies the blood to lower lumbs, kidney and abdominal viscera.

2. Left side
' e fhe left ventricle pumps blood into the ascending aorta for distribution in coronary circulation, head and upper limbs.

Umblical cord contains one umblicol vein and two umblical arterieswEEt)
$i a : ; a:)ir: ;.:.:::t a:i:,:..
t::,,:::::::::t: : a::::,.::,
$:P:ff,-f,:.{::

Fetal circulation

I uPPer"-""**l
timus
I I

+-Er*
syc

Pulmonary vein

RA
+LA
T
1t3
2t3

RV*+ * r-v
A

I Heart
i --------+-
I

V
P u I m o n a ry tru n k -------------**
I

I Major
---.-------}Ductus arferrosus----> Descending aorta

Lower limb, Kidney

r Blood in IVC has more oxygen saturation than blood in SVC as IVC carries the oxygenated blood of
umblical vein.
The left ventricular blood has more oxygen saturation that right ventricular blood because it
carries the blood of IVC,
while blood in right ventricle is a mixture of blood from IVC and SVC. Sq thebrain and coronary circulation
receiye
blood with higher oxygen saturation (through ascending aorta) than the lower half of boflyrtr tst.
r The left ventricular output is approximately half of right ventricular output because volume of
blood reaching in left
atrium is considerably lower than volume of blood reaching in right atrium. Infact right atrium is the major
source of
blood to left atrium.
r Aorta and pulmonary trunk are connected by ductus arteriosis, and pulmonary trunk has pressure slightly
higher or
equal to that of arota + So, blood flows from pulmonary trunk to aorta.
. The pressure in right and left ventricles are equal@r ts).

Circulatory adjustments at hirth


r These are brought about because of a shift from placental dependance for gas exchange in the fetus to pulmonary
gas exchange in neonate.
1) Pulmonarycirculation
r Immediatly after birth lungs expand due to first few breaths of neonate. This causes a fall in pulmonary
vascu-
lar resistance (O, causes pulmonary vasodilatation). This results in increased flow into pulmonary
trunk and
arteries-
: The pulmonary artery pressure falls due to lowering of pulmonary vascular resistance. The pressure
relations
between aorta and pulmonary trunk are reversed so that the blood flow through the ductus arteriosus
is reversed
-+ Instead of blood flowing frorn the pulmonary artery to aorta, the direction of flow through ductus, is from
aorta to pulmonary trunk.
r Functional closure of ductus occurs within 15 hours after birth(Arse). Increasing oxygen saturation causes
the muscle of ductus to constrict ) In full term neonates the ductus arteriosus closes within l0 to 25
days
(Anatomic closure)6t oo' DPG es).
2) System circulation and circulation through heart
Cn,eplEx 4
r Loss of placental circulation and clamping of the cord after birth results in increase in systemic vascular resist-
ance. This tends to increase the aortic blood pressure and the left ventricular systolic pressure.
r Thelossofplacentalcirculationresultsinsuddenreductionofflowthroughductusvenosuswhichclosesoff->
Flow through ductus venosus disappears by the 7th day of posnatal life(Alse).
r The loss of placental flow results in a decrease in the volume of blood returning to right atrium -+ Right atrial
pressure decreases.
: T1ae lefi atrial pressure becomes higher than right atrial pressureK"tutaoa) and the septum primum which ats as
a valve of fossa ovalis, approximates with the septum secundum to close offformen ovale.
r Functional closure of foramen ovale occurs very quickly(pcr8e).
r Over a period of months, the septum primum and septum secundum become firmly adherent resulting in
anatomical closure of the foramen ovale.
r After closure of ductus arteriosus, there is establishment of postnatal circulation: -
I The blood reaching the right atrium through IVC and SVC is emptied into the right ventricle from where it
pumped into pulmonary trunk.
r After coursing through lungs for gas exchange, it reaches the left atrium and ventricle.
r The Ieft ventricle pumps it out for distribution in the body for oxygenation of the tissues. The venous return again
comes back to right atrium through IVC and SVC.
r AII of the blood leaving the right ventricle, after coursing through lungs, reaches the left ventricle -+ The two ventricles
are connected in series and therefore, the output ofright and left ventricles are same (in contrast to fetal circulation,
where right ventricular output is more).

CONGEN ITAL HEART DISEASES


r Congenital heart disease (CHDs) are abnormalities caused by faulty development during 3 - 8 weeks of intrauterine
Iife.
r Both hereditary and environmental t'actors play a role in etiology of CHDs.
A) Hereditary factors
r A higher incidence of CHD in siblings suggests heredity as an important factor.
t The strongest familial tendency is known in ASD associated with bony abnormalities (Holt oram syndrome)
(ArrMs os)
,

i*.*:ir€;l::;r:ii# |.=i:j ',:ep.s .-. .. .:.;.,.. ..."-. ..,.Fr::#:t$ir'.:3ir#


..Syndrome, ., ,' ' . :,'::: Arsociatedcongenitalcardiovas(ulardisease ,. ',
Holt-oram syndrome ASD (most common), 1't degree heart block

Down'ssyndrome{trisomy2l} Enidocardlatcushiondefect{Atse){atypeofASD),VSDIOF
Turner syndrome Bicuspidaorticvolve (mostcommon)@NB13),asarctotionof aortq(2d MC)6,tMs08), AS, MVP

ttlaonan syndrome

William syndrome Suprava lv ular AS(il ee, Bra nch P5, hypercalcem ia

Marfan syndrame Aortic root dilatation

Edward syndrome VSD (most common), PDA

Apeftqtndrome ' VSD

Cri-du chat syndrome VSD

Osteogenesis imperfecta AR

B) Environmentalfactors
I Those who have an inherited tendency, develop CHD due to unfavorable environmental factors.
I of these the most well known is high altitude -> There is higher incidence
of pDA and ASD in children born
at a high altitude.

Phenytoin Pulmonary stenosis, Aortic stenosis, Aortic .our.t..Uon, pOn


Alcohal .
VSD, PDA, ASD, TOF
Rubello PDA (most common)NlEn, PS (2d 6s/nerr), VSD (3td
mcyNEEr)
Valproate : '. ASOVSqAS.PA,COA . -,,,
tnfant of diabetic mother TGA, VSD, COA, PpHN, Cardiomyopathy
Lithium Ebstein anoma$AilMl e),Tricuspidatiesia :
I

NADA's criteria
o The Assessment of a chird
for the presence or absence of heart disease can be done with the help of some guidelines
suggested by NADA. These guidelines are called NADAs
criteria(At )3, Arrfts e7)

* Systolic murmur, grade lll or more, * Systolic murmur < grade lll
especially with thrill
a Diastolic murmur o Abnormal 2ndheart sound
o Abnormal ECG
& Cyanosis * Abnormal X Ray
& Congestive cardiac failure o Abnormal Bp

Presence of one maior or two minor criteria are


essential for indicating the presence of heart disease,

cLASStF|CAT|ON qESOryCENtTAL HEART DTSEASES

o Congenital heart diseases are divided into two main


groups :_
1) Cyanotic
2) Acyanotic
o Each group is further subdivided accroding to :_
i) Pulmonary blood Jlow : Decreaserl (origemia), increased (prethora) or normal.
ii) Dominated (hypertrophied) ventricle: Right, or left.
A) Cvanotic CHD's
o These are characterized by presense of cyanosis.
cyanotic cHDs are :-
1) Increased pulmonary blood flow (pulmonary pletfi61a(arr:1
i) Left venh'icular or both ventricular hypertrophy: Persistant
truncus arteriosus; single ventricle;
TGA+VSD.
ii) Right ventricularhypertrophy:TGA;TAPVR (total
anomalous pulmonaryvenous return); hwoplastic
1eft heart.
2) Decreased pulmonary blood flow (pulmonary oligemia)
i) Both ventricle hypertrophy: TGA + PS; Persistent truncus
arteriosus with hlpoplastic pulmonary
artery; single ventricie with pS.
ii) Left ventriclehypertrophy : Tricuspid atresia(Al13, pcrse, Reraro r%
pulmonary atresia with hypoplastic right
ventricle.
iii) Right ventricle hypertrophy :ToF;Eisenmenger's syndrome;
Ebsteins urro..ruty.
r Of these the most well known k high altitude -+
There is higher incidence of pDA and
ASD in children bor,
at a high altitude.

Phenytoin Pulmonary stenosis, Aortic


Alcahol VSD PDA, AsD,TOF
Rubella PDA (most com6gn)tNEEr), pS (2nd mc1{nee\, VSD (3td
111syxrcrt
Valproate ASD, VSD, AS, PA, COA
lnfant of diabetic mother TGA, VSD COA, ppHN, Cardiomyopathy
Lithium Ebstein onomal/An[s s7),
Tricuspid atresia

NADA's criteria
t The Assessment of a child
for the Ttresence or absence of heart disease can be done with the herp
suggested by NADA, These guidelines are of some guidelines
called NADAs criteria,r 13,AnMse7)

Abnormal 2nd heart sound


& Abnormal ECG
& Cyanosis
Abnormal X Ray
& Congestive cardiac failure
Abnormal Bp

Presence of one maior or two minor criteria


are essential for indicating thepresence
of heart disease.

CLASSIFICATION OF CONGENITAL HEART


DISEASES
r Congenital heart diseases are divideci into two
main groups :_
1) Cyanotic
2) Acyanotic
r Each group is further subdivided accroding
to :_
i) Puimonary blood Jraw : Decreased (oligemia),
increased (plethora) or normal.
ii) Dominated (hypertrophiecl) ventricle: Right, or
left.
A) Cyanotic CHD,s
o These are characterized by presense of
cyanosis. cyanotic cHDs are
1) Increased pulmonary blood flow (pulmonary plslfrslarerr:.); :-
i) Left ventricular or both ventricular hypertrophy: persistant
truncus arteriosus; single ventricle;
TGA+VSD.
ii) (total anomalous pulmonaryvenous
rerurn);hypoplastic
*,'iT"'::.""ularhypertrophy:TGA;TAPVR
2) Decreased pulmonary blood flow (pulmonary
oligemia)
i) Both ventricle hypertrophy: TGA + PS; Persistent
truncus arteriosus with hlpoplastic pulmonary
afiery; single ventricle with pS.
ii) Lefi ventriclehypertrophy z Tricuspid atresia$I 13,pcree.Ke,,roe%
pulmonary atresia with hypoplastic right
ventricle_
iii) Right ventricle lrypertrophy : ToF; Eisenmenger,s
syndrome; Ebsteins anomaly.
B) Acyanotic CHDS

o These CHDs are characterizedby absence of cyanosis. Acyanotic CHDs are :-


1) Increased pulmonary blood flow (pulmonary plethora(.arI3))
i) Le[t ventricle or both ventricle hypertrophy. ySD(,41r3); pDA.
ii) Rightwntriclehypertrophy i ASD6IIMS03) (often RBBB); PAPVR (Partial anomalous pulmonaryvenous
return).
2) Normal pulmonary blood flow
i) Left ventricle hypertrophy: AS or AR; coarctation of aorta (COA); Mitral regurgitation (MR).
ii) Rightventriclehypertrophy: Pulmonic stenosis (PS);COA (in infants); mitral stenosis (MS).
Axis deviation on ECG
o CHDs maybe associated withleft axis deviation (LAD) or right axis deviation (RAD),depending upon the compartment
hypertrophied.
r Left ventricular hlpertrophy causes LAD, and CHDs causing LVH are : _
i) YsDrez'Mssal |-:l) Endocardial cushion ASD v) Tiuncus arteriosus vIl) Pulmonary a1resia@nMses)
ii) PDA iv) COA (in older children) vi) Tricuspi d atr e s i aaII Ms e 8)

o Right ventricular hlpertrophy causes RAD, and CHDt causing RVH are : -
i) TGA iii) ToF v) Eisenmenger\ syndrome
ti) TAPVR iv) AgPtdttusstl vi) COA (in infants)

. Most common congenital heort disease. VSD(atee)


t Mo,s;t common irforoti, ,*genital heart disease : VSD
e Most common cyanotic congenital heort diseose i TOFfl,EEII

PATHOPHYSIOLOGY OF CHDs

Acyanotic congenital heart disease


e These are divided into two types :-
A) Left to right shunts
r These are atrial septal defect (ASD), ventricular septal defect (VSD) and patent ductus arteriosus (pDA).
o Patients with Ieft to right shunts have following features : -
1. Frequent respiratory tract infection(AllMs0S) -> Because pulmonary circulation is overloaded which may
result in pulmonary edema, CHF and infection.
2. No cyanosis _) Most of the blood is in pulmonary circulation. So there is no impairment in oxygenation of
blood.
3. Precardial buldge -> Frequent chest infections with tachypnea causes the soft rib cage being drawn inward
at the diaphragmetic attachments of the ribs. This combined with cardioamegaly gives them a precordial
buldge.
4. Hyperkineticprecordium
5. Delayed diastolic murmur in tricuspid or mitral area -+ Due to increased flow through these areas depending
on the type of CHDs.
6. Cardiomegaly with plethoric pulmonary vasculature on chest X-ray -+ Due to increased flow through pul-
monary circulation (plethoric iung) there is increased volume overload to left side of heart -+ Cardiomegaly.
Left atrium is enlarged in these CHDs, except in ASD in which lefi atrium is not enlargediAr oe).
B) Obstructive lesions
o These are pulmonic stenosis (PS), aortic stenosis (AS), and coarctation of aorta (COA).
r Obstructive congenital diseases have following features :-
1' Absence offrequent chest infection{A'Ms0s)
-)purmonary circulation is not overroaded.
2' Absence of cyanosis -) No defect in pulmonary
oxygenation and there is no shunt where
blood can mix. arterial and venous

3' Absence of precardiar buldge -+ There is no chest infection or cardiomegaly.


4' Heaving (forcible) cardiac impulse -+ obstructive
lesions cause concentric hypertrophy
produce forcibie contraction. of ventricles that

5. systolic thrill due to systolic ejection murmur


-> \Arhen hl.pertrophied ventricles forcibly pump the blood
through stenotic pulmonary or aortic valve,
systolic thrill and.yr,olr. ejection murmur
are produced.
6. Absence of delayed diastolic murmur in
mitral or tricuspid area -)Blood flow through
increased. these area is not
7. Presence of delayed corresponding second
sound (A, in AS and p2 in ps) -+Because prolonged
these area results in delayed closure ofthat flow through
valve.
8. Normal sized heart and pulmonary vasculature
in chest x-ray-> There is concentric hlpertrophy,
pertrophy without dilatation. So, heart size is i.e., hy-
normal on x_ray.
Cyanotic Congenitial heart diseases
o Cyanotic CHDs are Right to Left shunts.
o These are further divided into : _
1) Cyanotic CHD with decreased pulmonary
blood flow.
rThis group includes -+ ToF, Pulmonary
atresia with intact se|ttutn, tricuspid
atresia, total anomalous pul-
monary venous return with obstruction.
r These lesions have following components
: _

a) obstruction to puimonary blood flow at tricuspid


right ventricular or pulmonary valve level.
b) A pathway by which systemic venous blood enters
the systemic circulation via a patent foramen
ASD or VSD. ovale or
r Degree of cyanosis depends on the
degree of obstruction to pulmonary
brood flow :
i) Mild obstruction -) cyanosis is precipitaed by
stress, but may be absent at rest.
ii) olood flow is dependent on patency
::,X ",Y::l':: :, 1",,i:"*,
ductus closes (10-21 days), the neonate experience
of the ductus arteriosus. when the
profound hypoxemia, cyanosis and
r These diseases do not have frequent chest infectionas pulmonary shock.
circulation is not overloaded.
2) Cyanotic CHD with increased pulrnonary
blood flow.
r This group of lesions is not associated with obstruction
to pulmonary blood flow.
: Cyanosis caused by anv of the following mechanisms.
u connection (e.g
, Transposition of great vessels6rr^sos))-+ rn
ti:'::j"1'.lll,'!Y' .,'rterial brood.retir.ni"; ff;-';il;;;;;*,.;;::H:ffffiT,:
this, aorta arises

[:Ti:J::'::'::.-""-':i:y,']li"'.*":"
the body, and oxygenated brood returning from
rung is pumped
I ruur ro Pu

back into the rungs.


ii) btoad (e.g., totat anomatous
i#,I::X,,r,::t:,:':-::,'^:.' ':o !"'*onctry.venous Ttutmonary yenous

:.::,:,:^::::'::"?: :'::,:ilo i*'a -Ir putmonarv btood rtow ,' ",;,;:;;;;;,";;:r,;;":;:il:::


cyanosi nlon,
cyanosis alone.
e
;;;;;;;;;;,";;,;,:"ff;:';:;
t Tftese diseases are chatacterized by frequent chest infections(ArrMs Bs)as
pulmonary circulation is overloaded.

ATRIAT SEPTAL DEFECT (ASD}

o ASD is an abnormal communication


between the trvo atria. There are two
types of ASD:-
1) Ostium secundum (most comm ontArls,AilMS81))
rIt is generally located at fossa avalis, but may
aiso be superior or posterior to
fossa ovalis.
2) Ostium primum (endocardial cushion defect)
r It is situated inferior to fossa ovalis. It associated with cleft in the anterior leaflet of mitral valve, with or without
a

a cleft in the septal tricuspid leaflet. It is the most common CHD in Down syndrolrrreulls).
r ASD is more common in females than males.
o lmportant syndromes associated with ASD are :-
t Holt - oram syndrome@t1s'PGI0L) t Thrombocytopenia absent radius (TAR) slfldrome(Pctot)
t Down slndromsedot) t Pierre Robin syndrome
t Ellis-van crevald syndromeecl 01 ) s Ehler Danlos syndrome
t Rubinstein - Taybi syndrome t Fetal alcohol syndrome
Hemodynamics of ASD
o There is abnormal connection between left and right atrium, which is due to defect in atrial septum.
r Blood flows from left atrium to right atrium because left atrium has slightly higher pressure than r@t atrium@I e7)

+ Left to right shunt6arMs8l).


o Because the pressure difference between two atria is small, blood passes at a narrow pressure difference -+ No shunt
e& PGr oe).
murmurar
r There is volume overload to right atrium because ofleft to right shunt-+Right atrial dilatation and hypertrophy(or00).
o During diastole large amount of blood passes from right atrium to right ventricle through tricuspid valve -+ delayed
diastolic murmur and accentuation of first heart sound.
r Volume overload to right ventricle -+ Right ventricular hlpertrophy that produces parasternal heeve('{ror, e8).
o Large blood volume passes through pulmonary valve *> Pulmonary ejection systolic murmur.
o Prolonged blood passage along pulmonary valve causes delayed closure of pulmonary valve -+ P, is delayed and
accentuated, wide split anilfixed S2are8),

o Increased blood flow through pulmonary circulation -+ Pulmonary plethora that may cause pulmonary
otr.
hYPertensisnrar
o The left atrium is not enlargedal 11' 06)
because it decompresses itself by shunting blood to right atrium at a minor
difference in pressure. The left atrium may enlarge once Eisenmanger's syndrome develops and a reversal of shunt is seen
across the defect.
Clinical manifestations of ASD
r Patients with ASD are generally asymptomatic.
o Mild effort intolerance and respiratory tract infection may occur.
: CHF is rare.
o Infective endocarditis is very rare in ASDNEID.
r Complications usually develop in 4'h decade and include pulmonary hypertension, strokeNEEr), right sided HF and
Eisemmenger's syndrome.
Physical examination
o Parasternal impulse
r Systolic thrill at 2dleft interspace.
o Accentuation of S, due to loud tricuspid component.
o Wide split andfixed S2euoe'AltMsa3).

o Ejection systolic murmur at the second anil thirdleft interspacesallMs0s).


o Delayed diastolic murmur at the lower left sternal border.
r ASD with mitral stenosis -+ Lutembacher syndrom e@NB ls' AIIMS06).
t There is riglrt axis deviation on ECG in usual osteum secundum type of ASD(PGI 0e)
. Ostium primum type of is associated
PGI0e)
with lefr axis deviation4lMso3' due to maldevelopment of anterosuperior division of left bundle branch (LBB).

Chestx-ray in ASD
o Mild to moderate cardiomegaly o Prominent pulmonary artery segement.
c Right atrial and right ventricular enlargement c Relatively small aortic shadow{NBgr)
t Plethoric lungfields. o Normal left atrium@IlMs t' 06, 01)
1
r It is situated inferior to fossa ovalis. It associated with a cleft in the anterior leaflet of mitral valve, with or without
a cleft in the septal tricuspid leaflet. It is the most common CHD in Down syndrome(Arls).
r ASD is more common in females than males.
o Important syndromes associated with ASD are :-
t Holt - otam sYndrome4l 1s'PGIol) t Thrombocytopenia absent radius (TAR) sffld.rome@cl ot)

: Down sfndrotns@etot) t Pierre Robin syndrome


t Ellis-van crevald syndromeecl 01)
s Ehler Danlos syndrome
t Rubinstein - Taybi syndrome t Fetal alcohol syndrome
Hemodynamics of ASD
o There is abnormal connection between left and right atrium, which is due to defect in atrial septum.
r Blood flows from left atrium to right atrium because lefi atriumhas slightly higher pressure than right atrium(Ale7)
-+ Left to right shunta[Ms8r).
o Because the pressure difference between two atria is small, blood passes at a narrow pressure diference -> No shunt
murmurare& PGroe).
o There is volume overload to right atrium because of left to right shunt-+Right atrial dilatation and hypertrophy(Arm).
o During diastole large amount of blood passes from right atrium to right ventricle through tricuspid valve -+ delayed
diastolic murmur and accentuation of first heart sound.
o Volume overload to right ventricle -+ Right ventricular hypertrophy that produces parasternal heeve@r01'e8).
o Large blood volume passes through pulmonary valve -> Pulmonary ejection systolic murmur.
o Prolonged blood passage along pulmonary valve causes delayed closure of pulmonary valve -> P, is delayed and
accentuated, wiile split andfixeil S2aIeB).

r Increased blood flow through pulmonary circulation -+ Pulmonary plethora that may cause pulmonary
hYPertension6ror).
o The lefi atrium is not enlarged@I 11' 06)
because it decompresses itself by shunting blood to right atrium at a minor
difference in pressure. The lefi atrium may enlarge once Eisenmanger\ syndrome develops and a reversal of shunt is seen
across the defect.

Clinical manifestations of ASD


o Patients with ASD are generally asymptomatic.
o Mild effort intolerance and respiratory tract infection may occur.
c CHF is rare.
c Infective endocarditis is very rare in ASD@Exr).
r Complications usually develop in 4s decade and include pulmonary hypertension, stroke@EEr), right sided HF and
Eisemmenger's syndrome.
Physical cxamination
o Parasternal impulse
o Systolic thrill at 2d left interspace.
r Accentuation of S, due to loud tricuspid component.
o Wide split and fixed S
r{PGr
0e' AIrMs 03)
.

o Ejection systolic murfiiur at the second and thirdleft interspacesalrMs03).


r Delayed diastolic murmur at the lower left sternal border.
r ASD with mitral stenosis -+ Lutembacher syndrom e@NB ts' AIIMS06).
t There is rigfut axk ileviation on ECG in usual osteum secundum type of Aspteet ot . Ostium primum type of is associated
withlefi axis deviationqltMso3'PGI0e) due to maldevelopment of anterosuperior division of left bundle branch (LBB).

Chest x-ray in ASD


c Mild to moderate cardiomegaly c Prominent pulmonary artery segement.
. Right atrial and right ventricular enlargement c Relatively small aortic shadow(NBBr)
o Plethoric lungfields. t Normal lefr atrium@IlMs 11' 06' ot )
r It is situated inferior to fossa ovalis. It associated with a cleft in the anterior leaflet of mitral valve, with or without
a cleft in the septaltricuspid leaflet. It is the most common CHD in Down slndromsrdrs).
o ASD is more cotwnon infemales than males.
o Important syndromes associated with ASD are :-
t Holt - oram syndrome@Ils'PGIo1) s Thrombocytopenia absent radius (TAR) syndromeecl0l)
t Down Slndromseetot) s Pierre Robin syndrome
t Ellis-van crevald syndrome@Gl 01 ) t Ehler Danlos syndrome
t Rubinstein - Taybi syndrome t Felal alcohol syndrome
Hemodynamics of ASD
o There is abnormal connection between left and right atrium, which is due to defect in atrial septum.
e7)
o Blood flows from left atrium to right atrium because lefi atrium has slightly higher pressure than right atrium@I
+ Left to right shunt6rMs8r).
o Because the pressure difference between two atria is small, blood passes at a narrow pressure difference -) No shunt
98, PCI oe).
murmur(Ar
o There is volume overload to right atrium because of left to right shunt+Right atrial dilatation and hlpertrophy{eroo).
o During diastole large amount of blood passes from right atrium to right ventricle through tricuspid valve -+ delayed
diastolic murmur and accentuation of first heart sound.
o Volume overload to right ventricle -+ Right ventricular hypertrophy that produces parasternal heeve(Aror' e8).
o Large blood volume passes through pulmonary valve -+ Pulmonary ejection systolic murmur.
o Prolonged blood passage along pulmonary valve causes delayed closure of pulmonary valve --> P, is delayed and
andfueil S2('ue8).
accentuated, wide split
o Increased blood flow through pulmonary circulation --> Pulmonary plethora that may cause pulmonary
oI).
hypertension(Ar
o The lefi atriuffi is not enlarged(At 11' 06)
because it decompresses itself by shunting blood to right atrium at a minor
difference in pressure. The lefi atrium may enlarge once Eisenmanger's syndrome develops and a reversal of shunt is seen

across the defect.


Clinical manifestations of ASD
o Patients with ASD are generally asymptornatic.
o Mild effort intolerance and respiratory tract infection may occur.
t CHF is rare.
t Infective endocarditis is very rare in ASD(dEET).

r Complications usually develop in 4th decade and include pulmonary hypertension, strokeNnnr), right sided HF and
Eisemmengert syndrome.
Physical examination
o Parasternal impulse
r Systolic thrill at 2"d left interspace.
o Accentuation of S, due to loud tricuspid component.
o Wide split andfixed Sr{PeIqe'AIIMSo3).

t Ejection systolic murmur at the second and third left intetspacesqllMs


os)
.

o Delayed diastolic murmur at the lower left sternal border.


o ASD with mitral stenosis -+ Lutembacher syndrome@NB1s'AllMs06).
t TherekrightaxisdeviationonECGinusualosteumsecundumtypeofASDecroe).Ostiumprimumtypeofisassociated
with lefi axis deviation@IlMs 03, PGI 0e) due to maldevelopment of anterosuperior division of left bundle branch (LBB).

Chestx-ray in ASD
t Mild to moderate cardiomegaly o Prominent pulmonary artery segement.
t Right atrial and right ventricular enlargement c Relatively small aortic shadow$EB?')
t Plethoriclungfields. o Normal left atrium4ilMs 1' 06' ot )
1
Fluoroscopic examination shows vigorous pulmonary pulsations due to increased pulmonary
blood flow called as ,Hilar dance slgn(et tst or Great Hilar Dance sign,.
Note r Hilar dance sign may be seen in other left-to-right shunts, but it is typically described in
relation to ASD.

VENTRICT.|LAR sEPTAL DEFECT (VSD)

o vsD is the most common congenital heart diseaseal se). 'fhere is a communication between two ventricies. VSD mav
be located at :-
i) Membraruous part of interventricular septum (called membranous VSD) : Most commot 6t t3,ArrMssz).

ii) Muscular septum (muscular VSD).


o VSD is more commofl infemales.

Hemodynamics of VSD
o A vsD results in shunting ofoxygenated blood from left to right because
left ventricle has more pressure than right
-+ Left to right shunt(Arr3).
I Blood flow from Ieft to right ventricle due to high pressure gradient -+ Pansystolic
murmur and systolic thrill(Arr3).
r Because Ieft ventricle starts contracting before Right ventricle, pansystolic murmur starts early -+ Masking
of S,.
o This pressure gradient is maintained throughout the systole pansystolic
murmur lasts long -+ Masking of Sr.
e Towards the end of systole, the declining left ventricular pressure
becomes lower than aortic --> Early closure of Ar.
o Left to right shunt occurs during systole at a time when the right ventricle
is also contracting, therefore left to right
shunt streams to pulmonatY artery more or less directly -+ No volume overload
-+ Rightventricle size remains
normal.
o Increased blood flow through pulmonary valve -+ Pulmonary ej ection
systolic murmur and delay & accentuated pr.
o Early closure of A, and delayed closure of P, cause -+ Widely split s2(Ar,3) (But
this is usually masked by pansvstolic
murmur).
o Larger volume reaches the ieft atrium + Left atrial hypertrophy(eii:r.
c Increased blood flow through mitral valve Accentuated S, (But it is masked
-) by pansystolic murmur) and delayed
diastolic murmur.

o Ejection systolic murmur of pulmonary valve can not be separated from pansystolic
murmur.
a The effect of ejection systolic murmur is a selective transmission of pansystolic
murmur to the upper left
sternal border (pulmonary valve area) --> ln this area ejection characteristic of
this murmur can be recognized
since it does not mask the aortic component of Sr. For the same reason second
heart sound (Sr) can be heard
in the pulmonary area where it is not masked by pansystoric murmur.

Clinical manifestation of VSD


r The patients become symptomatic around 6 to 10 weeks of age.
t At this time CHF may d.eyelop\ilMs01),
o Frequent symptoms -+ Palpitation, dysnea on exertion and chest infections.
o Symptoms of CHF improve and cyanosis develops once pulmonary vascular
resistance increases above systemic
. vascular resistance (Eisenmeneger physiology){Peteo.
Si$ns
o Hyperkinetic precordium with a thrill at the left sternai border.
r Pansystolic murmur at left sternal border.
Sr & S, are masked by pansystolic murmur maximum intensity of murmur is at
' 3.d, 4,h or 5th left interspace.
t S, can be make out at pulmonary area (upper left sternal border) where it is not masked by pansystolic murmur -+
S, is widely spliy'Al )3) with accentuation of pr.
r S. may be audible at apex.
r A delayed diastolic murmur in mitrai area.
r Pulmonary systolic ejection murmur in pulmonary area.
Clllprnq 4
Course and complications
o 70 to 80% of all vsDs become smaller in size rtr disappear
si*tultaneously, i.e. spontaneous crosure of vSp1o,*ssz;
Large VSD may have some complication :
i) Infective endocarditis@Glss)
ii) Pulmonaryhypertension
iii) cHF
iv) Aorticregurgitation.
o vSD is the mast cafitmofi crtngenital heart defect complicated
by infective endoc(trditis6rrMsez).
Chest x-ray in VSD
o Increased pwlmorcarl, v ast ulature (pulmonary plethorq)Grrus sz:.
o Left atrial enlargement1l lt).
o Cardiomegaly if defect is large"
o Normal to smali aorta.
e Normal size heart it. ciefect is small.

This chest X,ray is suggeeting tle diagno$is of -


a) ASD b) VSD
C) TOF d) PDA
Ans, is'b'i,e., YSD
r '',The X-rayis showing cardiornegaly involving left ventricre with increased pul_
monary vasculature (pulmonary plethora) as suggested by
the size nrra m.r"u..a
number of end on vessels in lung fields + Diagnosis is ViO.

PATENT DUCTUS ARTERIOSUS

c Patent ductus arteriosus is a communication between the pwlmonary


@rtery and aorta(N}Er).
o The ductus arteriosus is present in fetal life which connects
p,rlmonary trunk to aorta. It closes functio.gaily
and
anatomically soon after birth.
t 'Functional closure' takes place within 15 hours of birth.
Anatomic clasure' of ductus qrteriosus acctffs N-21
days
sfter birthataat.
c Prostaglandins maintain the patency of ductss{.rt oot.

a Thepersistenceaffunctionofductttsarteriosus beyond24hoursafterbirthisconsidere|aspDAinterm,eanate,i.e.if
functional closure does not take place in 24 hours afier birth, it is considerecl as pDA.
r The aortic attachment of ductus arteriosus is just distal
to subclavian artery.
o PDA is more cammon infemales than in males(Aros).

Hemodyrramics of pDA
I In PDA blood flows from aorta to pulmonary artery because
aortic pressure is higher than pulmonary artery pressure
-+ Left to right shunt.
e Pressure gradient between aorta and puhnonary artery
is maintained throughout the cardiac cycte (cluring
systole and
diastole) -> continous murmur, i.e., murmur starts in systole
after s, and reaches a peak at s2. It then dimi'ishes
and audible only a part ofdiastole.
r Larger biood volume passes through pulmonary circulation (blood
from right side of heart plus some blood from
aorta) -+ Fulmonary plethora which may cause pulmonary
h1p ertensionKetuta 00 ).
o Increased flow after passing through lung reaches the left
atrium and causes volume overload + Left atrial
dilatation
and hypertrophy.
c Increased blood volume passes from ieIl atrium to left ventricle
through mitral vaive, i.e., increasecl flow through mitral
valve -+ Accentuation of Sl and delayed diastolic
murmur.
System
r Left ventricle recieves larger amount
of blood that results in vorume
overroad -+ Left ventricre enrargement.
'"T:,;'".ff;:il:iffii*".t:#::r;.Tffs;:H":*rticvarvewhichmavcroseevenafterpurmonary
ur r* varves,' -> raradoxical spliting
A, occurs before pr). of Sr, i.e.,
{ occurs after p, (Normally
t left ventricular volume ejected
into the aorta resulrs in diratation
:fli: of the ascending aorta + Aortic ejection
" passes through normal aortic valve -+
Aortic ejecrion systoric murmur (bur
:;:il:JrTr:.Tl;o it is masked by
Clinical manifestations of pDA
c Patient may be asymptomatic
r Symptoms develop early and CHF
may develop at 6 to gth weeks of
e Common symptoms -+ Dysnea age,
on exertion, palpitation and.frequent
chest inf,ections@erataq|).
Signs
o Tachypnea & Thchycardia
o Bounding pulse with wide
pulse pressure (with elevated
systolic and lower diastolic
o Hyperkinetic cardiac impulse pressure).
o Systolic or continuous thrill.
o Accentuated S,
t split of s' (But it may be masked
by continuous murmur since
X;H:.T:, maximum intensity of conrinous
"",U1
o Continous (machinery) murmur -> Murmur starts after
S, and reaches the peak at
audible only during a part of the sr. It then diminishes and is
diastole' Murmur is best heard at second reft
below left clavicle. irntercostal space and is arso
heard
o S, may occur at apex followed
by a delayed diastolic murmur.
Differential diagnosis
o 'Small'Aor topulmonary window
defec{AIIMSll) has symptoms
similaT fs pl)1{.aaus0r). Aortopurmonary
is an abnormar communication window defect
between ascending aort) ana
mati:in, purmonary artery.
Course and comptications of pDA
c spontaneous closure of PDA does
not usually occur infull infants
because pDA in term infonts
abnormality of ductal smooth muscles resultsfo,rm structural
(In premature infants spontaneous
infants results from unresponsiveness crosure may occur because pDA in these
to oxlgen){euus t0, 06, es,Kerata 00).
o Pulmonary vascular pressure
regress slower than normal and
maximum decrease occurs around
at 1-3 weeks). 6-g weeks (normary
o Common complications are _
:
i) ctlp -> MC complication and MC cause of
deathar 13,NEE)).
ii) Infective end.ocayfllslsrNrnr) --y 2nay6 cause
of death.
iii) Aneurysm of pDA _+ Rare and possible rupture in adult life.
iv) Pulmonary hypertension.
v) Pneumonia (6
frequent chest infections)(Kerata00).
vi) Eisenmenger syndromeNEED.

Management of pDA
r NSATDs (most commonly indomethacin)^'ttusoalare
used to induce crosure of pDA,
of prostaglandins,llMs 06) (Prostaglanding as these drugsinhibit synthesis
oot).
are involved in maintainance of patency of
ductus arteriosus@rrMs 04, ss,Ar
o If indomethacin is not successful
and the duc hemodynamically significant,
without division of ductus should surgical ligation{Aros.) yri16 o,
u. p.rro.-'JJ.remains
r Left ventricle recieves larger amount of blood that results
in volume overload -+ Left ventricle enlargement.
o Extra volume passes through aortic area
cause delayed closure of aortic
valve which may close even after pulmonary
valve (normal pulmonaryvalves close
after aortic valves). -+ paradoxical spliting
of Sr, i.e. A, occurs after p, (Normally
A, occurs before pr).
t left ventricular volume ejected into the aorta
results in dilatation of the ascending
Hi: aorta + Aortic ejection

' through normal aortic valve -+ Aortic ejection


systolic murmur (but it is masked by
*|,.J:J.T;#|..]'passes
Clinical manifestations of pDA
o Patient may be asymptomatic
o Symptoms develop early and CHF may develop
at 6 to gth weeks of age.
r Common symptoms -+ Dysnea on exertion, palpitation andfrequent
chest infectior.ts(Keratao0),
Signs
r Tachypnea & Thchycardia
r Bounding pulse with wide pulse pressure (with
elevated systolic and lower diastolic
pressure).
o Hyperkinetic cardiac impulse
r Systolic or continuous thrill.
e Accentuated S,
o Narrow or paradoxical sPlit of s, (But it
may be masked by continuous murmur
since maximum intensity of continous
murmur occurs at S,).
o continous (machinery) tnurmur -+ Murmur
starts after S, and reaches the peak
at Sr. It then diminishes and is
audible only during a part of the diastole.
Murmur is best heard qt second lefi intercostal
space and is also heard
below left clavicle.
c S, may occur at apex followed by a delayed diastolic murmur.
Differential diagnosis
o 'Small' Aor topulmonary window defect(AllMs 0t)
has symptoms similay fs ppn{lrmsoi). Aortopulmonary
is an abnormar communication between ascending window defect
aori and main purmonary artery.
Course and complications of pDA
o spontaneous closure of PDA does not usually
occur infull infants because pDA in term
infants resultsfotrm structural
abnormality of ductal smooth muscles (In premature
infants spontaneous closure may occur
infants results from unresponsiveness to oxygen)@rrMs 10, 06, es, Ke;h 00). because pDA in these

t pressure regress slower than normal and


maximum decrease occurs around 6-8 weeks (normally
:Ji:T..lJascular
o Common complications are: _
i) CHF -'> MC complication and MC cause of death(Ar13,NEEI).
ii) Infective endocarditis@EED __s 2naylgcause of
death_
iii) Aneurysm of pDA -+ Rare and possible rupture
in adult life.
iv) Pulmonary hypertension.
v) Pneumonia (6 frequent chest infections)(Kerota00).
vi) Eisenmenger syndrom4NElt ).

Management of pDA
r NSATDs (most commonly indomethacin)^ilMs06)
are used to induce closure of pDA,
as these d^tgs inhibit synthesis
of prostaglandiflsallMs 06) (Prostaglanding
are involved in maintainance of patency
oo)). of ductus arteriosus'rrMs 04, es, Ar

o If indomethacin is not successful and


the ductus remains hemodynamically significant,
surgical ligation(Ar's) with or
without division of ductus should be performed.

h.

I
&, fl
fhe X-ray isrsuge*tive- of irhi{h CI{D , .
a) ' ppA,:"' 1.

b)',, :,.,, T1{PVC,,', t .

c) .''TOF ''r .

d) 'TGA,'''
Ans. is,lql i,e;;,PDi!l

r This,chest'X"r:rv is $owing enlargement of aofta with,prominent aortic knuck-


le,withrlarge main pulmonaryrartery,and increased pufmonaryvasculature -+
DiagnosisisPDA.l . -

TETRALOGY OF FALLOT
r TOF is the commonest cyanotic congenital heart disease@EEr).
o It has four components :-
06' NEEI' PGt 04)
i) VSDaI 13'

ii) Pulmonic stenosisal 13,06)


1i1) Overriding or dextroposed aorta4l 13' 06' NEEI' AITMS 07)

iv) Right ttentricular hypertrophy@I 13' 06' AIIMI 07).


o Pulmonicstenosis(rightventricularoutflowobstruction)ismainlyduetoinfundibularstenosis@Ecr,AltMsoT).Obstruction
at pulmonary valve level (vah'ular stenosis) is very uncommon (only 10%o).

'Pentology of fallot'consists of components of TOF plus atriol septal defect4t tl,07,AttMslo).

Hemodynamics of tetrology of fallot


o ThedirectionofflowisdeterminedbyseyerityofobstructiontorightventricularoutJlow(infundibularpulmonicstenosis).
o Most of the time there is significant obstruction, resulting in :-
i) Enjection systolic mufmur at pulmonary area and delay in P
26IIM|
0s)
: due to flow of blood against tesistance
during systolic.
ii) Right to left shun/AIIMS 'sl : As there is resistance at pulmonary level, blood is shunted to Ieft ventricle through
VSD.
11i) There no shunt murmur: As pressure difference across the shunt is low.
iv) CHF does not occur(NEET PGro' r As right ventricle is effectively decompressed by large VSD.
Clinical features
. DysPnea on exertion and exercise intolerance are the most common symptoms.
c Cyanosis and clubbinglPct0e"AIIMSes), Cyanosis usually develops after neonatal period and patients are rarely (if ever)
cyanotic at birth.
c Hypoxic or cyanotic spells (Tet spells),
cSquattinglPcr0e): TOF is the commonest CHD in which squatting is used.
o Normal jugular venous pulse(AIIMS 0s' e8) or slight prominence of 'a' wave
o Normal first heart sound
o Single second soundalrMs 0s' e8) (P is soft and delayed, hence generally inaudible. S, is therefore single and the audible
2
sound is Ar. Since the aorta is anteriorly displaced, A, is quite loud).
o Ej ection systolic murmur@IlMs 0s' e8),
t Parasternal impulse.

There is concentric right ventriculor hypertrophy$'Inses) without ca rdiomegolytAttMsss).


There is no CHFAfiMSes, no rect rrent chest infectionsedoe) and no cardiomegolytAttMssB).

o Chest x-ray shows normal heart sizewith oligaemic lungfields. The heart is boot shaped@NB ts) (coeur an sabot)@Elr,
Al0a).'Ihere may be right aortic
arch in 25o/o patients.
o ECG shows right axis deviation, inverted T waye and'P' pulmonale.
i;:9,r,x1gt* :A. ca r di$ 5 cu Ia r sy st e m

Management of TOF
o Management consists of :
A) Medical management : It is mainiy concerned about management of anoxic/cyanotic spells (tat spells)
correction of anemia :-
r Knee chest position r Propranoiol : 0.1 mg/kg/IV
r Humidified oxygen r Vasopressors : Methoxamine IM or IV
r MorphinerPclat) A.l to 0.2 mg/kg subcutaneous inj r Correction of Anaemia.
r Correction of acidosis with sodium bicarbonate TV
w lsoprenaline should be ovoided asitcausesperiphera! pooling of blood@nMsA).

E) Snrrgicalmanagernent:Itconsistsofanastomosingasl,stemicartervwithpuimonaryarterytoincreaseoulmonarr.
biood florv. These shunt are :,
i) Blalock - Taussig shunt -> Subclavian artery - Pulmonary Artery anastomosisur06).
ii) Modifieil Blalock-Taussig shunt: Subclavian-pulmonary artery shunt by Gore-'fer
interposition graft. It is
the most popular procedure at any age@NB 1s).

iii) Pott's shunt -+ Descending Aorta to Pulmonary Artery@rot) (no longer performed).
iv) Waterston's shunl -+ Ascending Aorta to right pulmonary Artery (no longer performed).

The X-ray is suggestive of which CHD -


a) PDA
b) TAPVC
c) TOF
dl rGA

Ans. is t'i.e., TOF


c The given chest X-ray is showing'boot shaped'heart (coeren sabot) -+
charac.
teristic of TOF.

Fallot's physiology
c Group of condition presenting with similar feature of TOF.
e These includes -
1) TOFGIi5) 4) Single ventricle with pS
2) TGA with VSD &Psrerisr 5) Corrected TGA with VSD & pS
3) TA(AI ls) 6) Double outlet right ventricle with pS

TRICUSPIS ATRESIA

c Congenital absence of tricuspid valve is called tricuspid atresia. TA has following


featgres : -
i\ Absence oftricuspid valve.
ii)Hypoplastic right ventricre@Il,s eB) witfu absence of inftow portion of RV.
o So' blood can not flow direciiy from right atrium to right ventricle and following
associatecl defect a-re neccessary fbr
survival : -
x For flow of blood from RA to LA -+ patent foramen ovale or ASD.
r
For flow of blood frorn LV to PJ -+ VSD (VSD is always muscuiar in tricuspid
atresia)
o Therefore blood flow in following direction.
Right atrium

I ThroughASD
Left atrium

I Through nomal mitral valve

Left ventricle
Pulmonary circulation rhroush *rZ\ hroush aortic vatve

Right ventricle Aorta Systemic circulation


Through pulmonary valve
-+
r Tricuspid atresia is classified according to : -
1) Presence or absence of pulmonic stenosis (PS)
2) Presence or absence of TGA (Trasnposition of great vessels).
o In 70 of cases, great arteries are normally related, i.e., pulmonary artery is connected to right ventricle and aorta is
%o

connected to left ventricle.


o In 30 % of cases TGA is present, i.e., Aorta is connected to right ventricle and pulmonary artery is connected to left
ventricle.
r In patients with normally related great arteries, VSD is small and PS is present -+ J Pulmonary blood flow and
pulmonary oligemia.
r When TGA is present, following two defect may be seen : -
i) Normal pulmonary valve (2l3rd of cases) -+ Increased pulmonary blood flow (PBF).
ii) Pulmonic stenosis (1/3rd cases) -+ Decreased PBF.
Tricuspid atresia

Normally related great vessels Transposition of great vessels


(70 o/o) (30 vo)

t +- I

Pulmonic stenosis (PS) present


Normal pulmonary valve PS Present
in all cases
+ +
+ tPrlronury blood flow Jper
JPulmonary blood flow

Haemodynamics
o The inflow portion of R.V. is hlpoplastic
r The systemic venous blood coming to R.A. exits by way of a patent foramen ovale@EBr) or an ASD.
o A VSD provides communication between L.V and outflow portion of R.V
r The L.V. thus maintains both the systemic and pulmonary circulation.
o The saturation of blood in the pulmonary artery and aorta is identical --> Because unoxygenated blood from right
atrium mixes with oxygenated blood into left atrium.
o The pulmonary blood flow is dependent on size of VSD -+ Smaller the VSD, less the pulmoanry bloodJlow@IlMsss).

Clinical features
o Child is cyanotic from birth@101' AIIMS 00)
.

o Anoxic spells and squatting may be present.


I Normal Srand single S26nMSe8'NEEr).
o Murmur (Grade II to grade IV)
Features suggestii'e of tricuspid atreria i .
a) L.V. Qpe of apical impulse
b) Prominent large'a waves
c) Enlarged liver with presystolic pulsations (a waves)
d) ECG characterised by Left axis deviation and Lefi ventricular hypertrophy\EEr. N 01. AI.MS 00, e8)
.

e) There is pulmonary oligemia on chest x-rayNEEr).


EBSTEIN ANOMALY

o Ebstein anomaly consists of downward displacement of an abnormal tricuspid valve into the right ventri-
el, 80)
cle@NB .

c Normally tricuspid valve has three leaflets -+ Anterior, posterior and septal.
r Fixed end ofthese leaflets is attached to valve ring in tricuspid area.
r In Ebstein anomaly, anterior leaflet is attached to valve ring as normal, but the other two leaflets (posterior and septal
are displaced downward and are attached to the wall of left ventricle.
c The portion of right ventricle above he tricuspid valve becomes a part of right atrium -+ atrialized right ventricle.
Hemodynamics
* The tricuspid valve anomaly results in obstruction of blood flow as well as regurgitation of blood from the righ;
ventricle into the right atrium -+ Dilatation and hypertrophy of right atrium@r 15 DNB 12) due to volume overload.
e Blood flows right atrium to left atrium through patent foramen ovale or ASD + Right to left shunt and cyanosis.
Clinical manifestations
o Cyanosis o Fatigue
o Dysnea on exertion o Paroxysmal attacks of tachycardia

Signs
o Cyanosis and clubbing
o Dominant V wave on JVP.
e Systolic thrill at the left sternal border
e S, normal
o S, wides split but variable
c Rightventricular S.
0 Right.atrial 54.
e Systolic murmur due to regurgitation at tricuspid valve.
e Delayed diastolic murmur due to obustruction at tricuspid valve like tricuspid stenosis.
c Both systolic and diastolic murmur produced at the tricuspid valve have scratchy character like pericardial
friction rib.
e Patients with Ebstein anamoly may have accessory conduction pathway causing WPW syndromerA,rs).

The chest X-ray suggests the diagnosis of-


a) PDA
b) VSD
c) Ebstein anomaly
d) ToF

Ane, !rt'i-e'r E_-bf,lianotnali '


o X-ray is showing almost 'box-shaped' heart due to enlargement of right atrium
-+ seen in Ebstein anomaly.

Tffi&&'SPOSITION OF GREAT VESSELS {TGA}

c It is the most common conot-runcal defect(Arls).


o Transposition ofgreat vessels is defined as : -
\\ Notto orismgJ,rom ,right ^'rentric\e.
ii) Pulmonary artery arisingfrom the left ventricle.
c Anatomically TGA is divided into : -
A) Complete variety
r Right atrium is connected to right ventricle and right ventricle is connected to aorta.
r Left atrium is connected to left ventricle and left ventricle is connected to pulmonary artery.
This results in complete separation of systemic and pulmonary circulation : -
i) Blood from pulmonary circulation flow in following direction

Pulmonary artery --------+ Pulmonary circulation -------) Pulmonary veln


(oxygenated blood)

Left atrium <- Left ventricle

ii) Blood from systemic circulation flow in following direction

Systemic circulation -------) SVC & IVC


| (Deoxygenated blood)
Aorta j
I

Right ventricle <- Right atrium

r As oxygenated blood is not available for systemic distribution, survival of the patient depends on the
presence of atrial, ventricular or aortopulmonary communication.

B) Physiologically correctedtype
r Right atrium is connected to an inverted morphologically left ventricle and Ieft ventricle is connected to
pulmonary arterY.
r Left atrium is connected to an inverted morphologically right ventricle and right ventricle is connected to
Aorta.
r So, these patients have normal route ofblood flow except for that, the functions ofright and left ventricles
are interchanged due to Position.
r This type of TGA is associated with other anomalies.
: The hemodynamics and clinical features depend on other anomalies because corrected TGA itself has normal
route of blood flow.

Hemodynamicsof TGA
13' PGI ee).
o ln TGA aorta lies anterior and to the right of pulmonary 4rtery6r
o In patients with TGA the oxygenated pulmonary venous blood recirculates in the lungs whereas deoxygenated
systemic venous blood recirculates in the systemic circulation -> Pulmonary artery saturation is always higher than
aortic saturation (opposite to normal person).
o Because oxygenated blood is not available for systemic circulation, survival depends on the mixing available between
the two circulation.
r So, patientswith TGA can be divided into : -
i) TGA with intact ventricular septum.
ii) TGA with VSD.

i) TGA with intact sePtum


r The mixing of blood depends on atrial communication.
r As patent foramen ovale is very small communication, mixing of oxygenated blood with deoxygenated blood is
ArrMsol'Ato4), and CHF may develop during
inadequate +
Patients are symptomatic and cyanotic at birth(Pcroz
first week.

ii)TGAwithVSD
t Presence of VSp/rctoz'se) of adequate size results in good mixing of blood.
r patients become symptomatic, at 4-8 weeks when fetal pulmonary vasculature regresses maximally -+ CHF
develops at this time.
r The failing left ventricle and large pulmonary blood flow cause increase in Ieft atrial pressure + t back
pressure -> Pulmonary venous hlpertension and pulmonary plethora'
r The presence of a large VSD equalizes the pressure in the two ventricles as well as in two great vessels.
r pulmonary artery carries large flow -+ patients with TGA and large VSD develop pulmonaryvascular obstructive
disease (Eisenmenger physiology) early in life.
eardlouasculsr System

Clinical features of TGA


r Time of presentation depends whether the ventricular septum is intact
or VSD is present.
o Patients of TGA with intact septum are cyanotic at birth, with CHF?GI a7)
occuring in first week -) because adequate
mixing does not occur.
I Patients of TGA with VSD presents :,vith mild cyanosis {tfid Ctrtrf:ecl0r)
occurs in 6-g weeks.
lmportant signs are :-
r Cyanosis o CHF o Eiection systoiic murmur
o Single S, c Normal S,

X-rays
e Cardiomegaly
o Plethoric lungfields
o "Egg on Side'(NEEr) appearance -+ right upper fields appear more plethoric
than other areas.
o Absent thymic shadow.

Management of TGA
o when TGA is suspected, an infusion of prostaglanding E,6t tst (Misopro{Alts),
riaprostil) should be iwitiated to
maintain potency af dwctus arteriosus\t 04, Arr/rs 04) to allow mixing of biood.
a Infants who remain severely hypoxic or acidotic clespite PGE infusion I .r:
should undergo atrial septostomy(lr i.e.
artilrcial ASD to allow mixing of blood. Septostomy is successful only upto
the age c;f 6 -12 weeks.
r Arterial Switch Procedure is the definitive swrgico.l
ltrocedure.In this, aorta and pulm,nary artery are divided and
reanastomosed in correct position. Two arterial switch procedures are
:-
i) Iantene arterial switch procedure (surgical treatment of choice)
ii) Mustard and senning procedure.

The X-ray is suggestive of which CHD -


a) PDA
b) TAPVC
C) TOF
d) TGA

Ans. is'd'i.e., TGA

r The chest X-ray is showing Eg3' on side appearancewhichis characteristic


of
Transposition of great artertes ffG,4).

TOTA L A NO MA LO U S PU LM O NA RY VlN q UIC9 N N ECTI O N (TA PVS)


o Total anomalous pulmonary venous connection (TAPVC) is characterized
by abnormai d.rainage of pulmonary
veins into the right heart either by direct connection into the right atrium
or into its tributaries.
r According to the site or level of connection of the pulmonary veins to the systemic venous system
TApVC has been
classified into four types :-
e Type I (mosf s6tn ','tsn(uos) :45o/o): Anomalous connection at supracardi46lsysl(eros)
(pV drains into left innominate
vein or SVC).
t Type II (25%o) : Anomalous connection at cardiac level (PV joins the coronary sinus or enter right
atrium directly).
I Type III (25%): Anomalous connection at infracardiac level (pV
drain into portal vein).
t Type IV (5o/o) :
Anomalous connection at multiple levels,
e In supracardiac TAPVC the pulmonary ueins join to
form a single trunk (common pulmonary vein) which than
drain through anomalous cannection@I e7),

Hemodynamics of TAPVC
c TAPVC results in the pulmonary venous blood reaching the right atrium which
also receives the systerrtic venous
blood(Ate7).
o This results in volume overload to right atrium -+ Right atrial enlargement.
r I-arger volume of blood reaches the right ventricle + Right ventricle enlargement.
o bloodJlow to the lefi atrium is through patentforamen ovale or ASD(AIe7).
The
o Because there is mixing of unoxygenated blood from systemic circulation and oxygenated blood from pulmonary
circulation in the right atrium -+ The oxygen saturation of blood in pulmonary artery is higher or identical to that
in AortaGIeT).

r TAPVC can be divided into : -


i) TAPVC with pulmonaryvenous obstruction
r Pulmonary venous obstruction results in pulmonary arterial hypertension.
r Because ofpulmonary venous obstruction oxygenated blood cannot reach heart -+ Cyanosis and CHF during
first week of life.
ii) TAPVC without obstruction (more common)
r In TAPVC without obstruction, pulmonary blood flow is high.
r Patients develops CHF at around 6-8 weeks when the fetal pulmonary resistance falls maximally and pulmonary
circulation reaches the maximum.
o Infracardiac type of TAPVC is always obstructive whereas cardiac and supracardiac type may be obstructive or
nonobstructiYeal e7).

X-ray findings of TAPVC


r Cardiomegaly
r Plethoric lung fields
o Snowma.n or figure of '8' configurationNEEr'
AI ee)
-+ In supracardiac TAPVC.
r Ground glass appearance of lung -+ In obstructive TAPVC.

Clinical manifestations of TAPVC


1) Nonobstructive TAPVC -+ Patients presents with mild cyanosis and CHF at 6-8 weeks.
2) Obstructive TAPVC -+ Paients presents with severe cyanosis and CHF within first week.
ECG
o Right axis deviation with right ventricular hypertrophy.
o In obstructive TAPVC'P' pulmonale.

The X-ray is suggestive of which CHD -


a) PDA
b) TAPVC
c) TOF
d) TGA

Ans. is't)'i.e., TAPVC

: ", t'
a l Figqre tof S,appearance (Snowman,sign) is characteristic of TAPVC.

EISENMENGER SYNDROME

o There are two diflerent terms : -


1) Eisenmengersyndrome
r It refers to patients with a ventricular septal defect in which blood is shunted from right to left as a result o'f the
development of pulmonary vascular resistance.
r This physiologic abnormality can also occur with ASD, atrioventricular septal defect, PDA or any other
communication between aorta and pulmonary artery.
ii) Eisenmenger complex
r It consists of pulmonary hypertension with a VSD(Pcroo.) providing the right to left shunt.
o Eisenmenger physiology is defined by an absolute elevation in pulmonary arterial resistance to greater than 12 wood
units or by a ratio of puln'lonary to systemic vascular resistance of > 1.0.
r The pulmonary hr,Dertension is the result of : -
r Early in the course it is due to markdely increased pulmonary biood flow --> Hyperkinetic pulmonary hypertension.
This form of pulmonary hypertension decreases with the administration of pulmonary vasodilators or oxygen.
t With the develapment of Eisenmenger syndrome pulmonary hypertension is due to pulmonary vascular ohstructive
disease, i.e., obstructive chunges in the pwlmonary vessels. This tlpe of pulmonary hlpertension only minimally
responsive to prilmonary vasodilators or oxygen.

Hernodynarni<s of Eisenmenger syndrome


i) If right to left shunt is VSD or PDA
r Rigtrt ventricular pressure cannot go beyond the systemic pressure because, before reaching to that high level,
Right ventricle decompresses into Ieft ventricle.
r So, there is novolume overload to right ventricle.
r But due to pulmonary hypertension there is pressrlre overload + Right ventricular concentric hypertrophy
*'ithout enlargement -+ Only a mild parasternal heave.
ii) If fight to left shwnt is ASD
. . to right to left shunt more blood flows to systemic circulation -+ More venous return to rt atrium -+ Iarger
Dure
amount of blood enters the right ventricle.
r So, there is voiume overload -+ Hlpertrophy with dilatation of right ventricle + Parasternal heave.

Cyanosis in Eisenmenger syndrome


r ln ASD and VSD, the deoxygenated blood reaches the ascending aorta and is thus distributed to the whoie systemic
cii'culation -+ Equal cyanosis of finger and toes.
r trn PDA deoxygenated blood reaches the aorta distal to left subclavian artery. So, upper extrimities receive mixed
'n1ood -+ flifferential cyanosis (Fingers remain pink whereas toes show cyanosis
and clubbing).
t Di-fferential cyanasis is diagnostic of pulmonary arterial hypertnsion with a pDA.

lmpending signs of Eisenmenger syndrome


o tlne should keep in mind that term Eisenmenger syndrome is used most frequently in relation to VSD (Eisenmenger
complex), though it can also occur in other defects also (see above).
r Eisenmenger syndrome refers to patients with a VSD in which blood is shunted from right to left as a result of
l
l

development of pulmonary vascular resistance. Initially shunt is from left to right as the systemic vascular pressure
l

is greater than pulmonary vascular pressure. With time pulmonary vascular resistance increases due to change in
pulmonary vessel wail as a result of increased flow in pulmonary vessels. When pulmonary vascular pressure exceeds
the systemic vascular resistance, reversal of shunt into right to left shunt occurs.
t Tltus, development of pulmonary hypertensian can be assurmed to be a sign of impending Eisenmenger's syndrome.
Signs of pulmonary hypertension are : -
1) Loud' P2ttrrrurs 'ot

ii) Single S2(AIrnIs


10)

11i) Parasternalheaye

iv) Pulmonary ejection click


v) Graham steel murmur (a high pitched decrescent diastolic murmur due ta pulmonary regurgitation)@ntts ro).

Clinical manifestations of Eisenmenger syndrome


. Syrnptoms -+ Cyanosis (Dillerential cynosis in PDA), fatigue, effort intolerance and dyspnea.
o Physical examination
r Cyanosis and clubbing r Parasternal impulse (more in ASD)
: Palpable P, r Wide and fi,xed splitting of S, in ASD
r Pulmonary regurgitation murmur.
r Ejection systolic murmur (pulmonic).

Chest X-ray in Eisenmenger syndrome


1a' AI as)
e Dilateil and Trominent central arteries@IlMs '
10' AI0s).
o Rapid tapering (pruning) of the petipheral pulmonary vasculature(AllMs

o The hilar (central) area suggests pulmonary plethora whereas the peripheral lung fields suggest pulmonary oligemia'

ECG
o tught ventricular hlpertrophy with right axis deviation'

e 'P'pulmonale.

COARCTAtrION OF AORTA
andposterior
r Coarctation of aorta (coA) is a constriction of aorta due to sharp indentationinvofuinganterior,lateral
wall of aorta. The medial wall is spared in the narrowing'
arteriasus
o of coarctation@t se) occur just below the origin of left subclavian occur at the arigin of ductus
9go/o

(luxtaductal coarctation).It may occur just proximal to subclavian artery also.


Mosl
o CoA is the 2nd most common congenital heart disease in Turner syndrome(AIrMses) (after bicuspid aortic valve)"
11' 08' AIIMi eB)
. other associated defects
common associated heart anomaly in coA is bicuspid aortic valve (50-7ao7o1ro' "'
VSD, ASD, TGA, hypoplastic Ieft heart, PDA, and berry aneurysm' When CoA is associated with
are aortic stenosis,
mitral valve abnormality and subaortic stenosis, it is called as shone complex'
a coarctation'
e Malesare afecter3 twice often asfemales although females withTurner syndrome frequentlyhave
e Two classic forms of coarctation of Aorta have been described :-
1) An infantileform with tubular hypoplasia of the Aortic arch\l
ee)
proximal to a PDA that is often asynaptomatic
in chiidhod. (.This is known as coarctation syndrome)'

Z) An,,adult'forru in which there is discrete ridge like in{biding of the Aorta just opposite to ciosed ductus arteriosus'

Hemodynamicsof COA during fetal life


o In fetal life, blood from right ventricle passes to puimonary artery and then to descending aorta through ductus
and left subclavian artery.
arteriosus. Blood from left ventricle passes to Aorta and then to innominate, left carotid
is no
e If constriction (coarctation) is proximal to ductus arteriosus (preductal or infantile coarctation), there
obstruction to flow because constriction is not obstructing the florv of upper limb and lower
limb.+ Collaterals are

not formed.
o Ifthe constriction is distal to tluctus arteriosus (post ductal or adult type coarctation ofaorta), it causes obstruction
circuiation + This
offlow ofdesending aorta, and parts ofbody that are suppiied by descending aorta, have impaired
stimulates formation of collaterals in fetal life'
This collateral circulation
r Collateral circulation connects the proximai and the distal aspects of the vessels over time.
will develop mainlY from the : -
r Subclavian
r Axiliary
07)
s Internal thoracic@rttts
t Superior and infetior epigastric@uMs 07)

r Intercostal arteries
Hemodynamics after birth
r After birth, ductus arteriosus closes, so that blood flow to descending aorta from right ventricle does not occur and
descending aorta must receive its total supply from left ventricle via ascending aorta.
r But there is obstruction due tc coarctation'
-+ Hypertension
r In preductal coarctation, there are no collaterals -+ Neonates become symptomatic imrnediatly
results in CHF.
r In postductal coarctation, due to Presence ofcollaterals neonates are spared
from developing hypertension & cHF.
r Due to obstruction, there is pressure overioad to ieft ventricle -+ Left ventricular
hypertrophy.
Clinical manifestations of CoA
o Symptoms of coarctation of aorta occurs due to decrease
in blood suppty distai to aortic obstruction.
o If obstruction is distal to left subclavian artery, manifestations
are due to arterial insufficiency to lorver limbs.
t Intermittent claudicationLr ss)
r pain
r Weakness of Leg
a Dyspnea on rurxninglAree)
I weak pulses and iower BP in the lower extremities -+ Delayed
or weak impatpable fetnorals compared to sh.ong
brachialarteries6I10'06,AllMS02).ftis6Irc,06,AIIMS02).
c collatetal circulation develops between precoarctation arterial branches and the post caarctation arteries through
enlarged intercostal and internal mammary arteries.
e This produces radiologicallyvisible erosiotts (notching)
ofundersurface {ir{e; ior) afribs.
. Hypertension -+ Headache, dizziness.
o If the obstruction is proximal to left subclavian artery
-+ Blood supply tr: leli arm (sr,rcoiied by left subclavian)
is interrupted whereas blood supply to right arm is normai
-+ pressure difference in two arms"
Murmur in CoA
r The blood passing through the narrowed aorta --> Midsystolic
nxurrnttrin anterior part of chest, back and spinous
process.
r when lumen becomes seYerly narrowed the blood flow through it, turns into
c coA is associated with bicuspid aortic valve in
a high velocity jet + cantinous murmur.
50-807o of patients. The bicuspid vaive produce aortic
regurgitation
-> Diastolic murmur.
Radiological finding of CoA
1) Abnormal aortic arch is the commonest finding. Site of narrowing may be seen
an indentation. classical ,3,
as
sign or 'double bulge sign' on ieft border of aortic shadow
is formeci (above by down word) : prestenotic dilatation.
coarctation itself (indentation), and poststenotic dilatation.
2) Rib notching is seen on 'inferipr' border of 3'd or 4th rib down
wardslr se, e8). rtis due to enlargement of intercostal
arteries (collateral vessels)alss). Rib notching is seen in adulthood, (not
seen until late childhood).
Causes ofdeath in CoA
o The causes of mortality in co-arctation of aorta are :-
t Hypertension
t Rupture of aorta
t Cerebral haemoruhage(ecr 00)
(due ta aneurysm rupture)
s Aortic dissectiort
t Left ventricular f,ailwrsea oo)

t P remature ath er clero si.t


os

t Infective endocarditisecl oo)

The depicted in figure is seen in


fgn
a) ASD
h) VSD
C) PDA
cl) CoA
Ans. is'd i.e., CoA

r This is tlpical 'figure of 3' sign or 'daubre bwrge' sign


-+ ch aracteristic of coA
OTHER CHDs

Atrioventricular septal defect (AVSD)


lltisalsocalledasatrioventricularcanaldefect(AVCD)orendocardialcushiondefect. septal defects
characterized by contiguous atriai and ventricular
o It covers spectrum of congenital heart malformation
a

with markedlY abnormal AV valve' right and left


atrioventricular junction as compared to separate
o The hallmark of AVSD is the presence of a common
heart'
atrioventricular junctions in the normal
o AVSD maYbe:-
|)lncompleteAVSD:ItisthesimplestformandnothingelsebutostriumprimumtypeofASD(AII5)inwhichthereis is more common
vah'ular orifices for right and Ieft ventricles' It
a common atrioventricular junction but separate
Is)'
in Down sYndrome(Ar

rThus,ostiumprimumASDisatypeofAVSDorendocardialcushiondefect.

ii)CompleteAVSD:Thereisacommonatriventricularjunctionandsinglecommonvahrrlarorifice.

Anomalous origin of coronary arteries .-- ^r,- +L^ r-r^^r supply nf


the blood ",,^^r-r ol
artery from the purmonary artery: In this anomaly
o There is anomarous origin of the reft coronary
compromised'
the left ventricular myocardium is severely
left coronary artery
arterial pressure falls, perfusion pressure to the
r Soon after birth (Neonate) as the pulmonary
becomes inadequate'
o Myocardial infarction andfibrosis may resul{Ar,l)'

Clinical manifestion by respiratory infection'


first few month of life and is often precipitated
o congestive heart faiiure becomes manifest within without cyanosls
o Recurrent attacks of discomfot, restlessness,
irritability, diaphoresis, dyspnea and pallar with or
occur - could be caused by angina pectoris'
o Cardiac enlargement is moderate to massive
o Murmurs maY be Present'

Hypolastic left heart sYndrome reft ventricre, aortic valve'


parts of the reft side of the heart (mitral valve,
o Hlpoprastic reft heart syndrome occurs when
and aorta) do not develop completely'
o The condition is present at bitth (congenital)'
hours of life' although it may take a
' Symptoms develop in firstfew
1')
o At first, o nr*birn may appear normal@'IMs

few daysto develop sllnptoms@ilttstt)' /trM.tt\ -^.-.^r:-^r-^-.r r'l+F ^rA shnckallMsli)


rSymptomsincludecyanosis,lethargy,poorpuise,respiratorydi,l,",,(ltlusll),poundingheart,CHFandshock4IIMS,.

Ductus arteriosus dePendant CHDs II) with


Truncus arteriosus(AllMs
05' 02), ToF critical Ttulmonary stenosis(A,Ms
i) pulmonary flow ductal dependence z

intactseptum . . ,._-__^L^t -^-+:- arch(AIIMS 0t'


nerh@rrns0s,02)
vah,-ular aortic stenosis, coA, interrupted aortic
ii) Systemic flow ductal dependence : congenital
t 1' 0s' 02)
hypoplastic left heart syndrotne\llMs '
11' 0s' 02)
Transposition of great vessels with intact septumullMs '
iii) oxygenation ductal dependence ;
11' 0s' 02)

pulmonary vanous return' truncus arteriosus'IlMs '


iv) Ductus independent CHDs : Total anomalous

Double aortic arch of vascular rings around


and its major branches result in the formation
o congenital abnormaiities of the aortic arch
degrees of compression'
the trachea and esophagus with varying
dorsal aoita persists distal to
r The most common cause is a double
aortic arch. Double aortic arch results when right

the origin of the right 7h cervical intersegmental


artery'
13) respiratory symptoms' e'g striclor and cough.
o vasculaili.rg *uy-.o*press trachea(DNB and produce
:. :. .. . .1,:r
., ... .::.4 . . . :.
C 41,#r.TA, +|$4f$ivasc*a r Syste m
Taussing Bing anornaly
r Double-outlet right ventricle with transposition of great arteries is
knolvn as Tausslg-Bing anomallr It also has
associated VSD.
r Surgicai correction is by: ,
A) Rastelli pracedure ar
ts) Arterial switch {iantewe) pracedure

CO}IGESTIVE }IEA3T FAILURE {CHr}

s congestive heo"tt failure is state in which the heart co?1tlot praduce tlte cardiac out required to sustain the
rnetabolic needs o;f tue body, without evoking certain compensatorv meclzanism.
o The most cofilmon cause of CHF in infants -+ Congenital heart
disease(coued0s).
I The rnost c&mmcn cause of CHF in the older children -+ R.heumatic fever(rrri, & rheurnatic heart
disease.

Timing of CHF in congenital heart diseases


'it:i.:::.;:jir:t: . .r:.r ., t:.r:i.:::
Afri*'',,,.j:i..:::i.i:i3r.:' jrr.-:ilir:iir-'-i.
: -r:: -. :a ..:ri::-::r:j::"
Blrth - 72 hours Pulmonary,, mitrai and aortic atresias.
4 days to I week Hypaplastic left and right heart syndromesiAl tr ssr,
transposition and malposition of great ateries.
1 week to 4 week Transposition and malposition complexes, endocardial fibroetastosis,
coarctation of the aorta.
J Month to 2 monlhs Transposition and matposition complexei, er+docardial cushion
defects, ventr:icutar septat defecl Fatent
ductus arteriosus' Total anomalous pulmonaiy venous connectio[ anomalous
left corohary,artery front ':.
pulmonaryartery- ' :' : ......,.,,:.,, ...'.. ':.t:'
2monthstodmonths Transpositionandmalposiiioncomplexes,ventricularseptaldefect,patentductusarteriosus,total
anomalous
pulmonary venous connection, aortic stenosis, coarctation of the
aorta.

Clirn*ilfsatlnres of CHF in infants


r In infants, heart failure mav be dificult to identifi. Pron-iinent manifestations inciude.

Clinical features of CHF in infants

:r'-1 :: .::.li:iii,l:::::i:..=1:ar::l r 1:
a Tachypnea 1S Cardiac enlargement
e Tachycardia * Galtop rhythrn (S3) t Facia'l edem;i :
a Cough * Peripherai cyanosis {t Jugar venous engrogment
o Wheezing * Small volume pulse * Edema of feettNEEt)
6 Rales in chest & Absence of weight gain o Ascites@Gt83) (in older children)
e Hoarse Cry * Ot i!!u rig{eet a1 ttwen ao1'

* PUlmonaryedern6{Pct83)

Treatmant
r Coirrmonly useri drugs for trearment of cI{F in chilclren are :-
1) ACE inhibitors (drug of cltoiee1ipn,0q))
2i Diuretics
3) Beta-blockers.
o lliuretics (especially doop diuretics) are the drug of choice to cantrol pulmonary
eclema antl systemic venous congestion,

RHEUMATIC FEVER

r ilheumatic fever is an acute immunologically mediated multisvstenr inflammatory


disease that occurs few weeks
afler an cpisode of {rrc,itp L streptococcal pharyngitis9oned{)8).
o ]lte di:,e*s* i.s immune wecliated, nat a cammunicoble fllsssss(:an*r) os).

H
As is the case of streptococcal sore throat, acute rheumatic fever most ofien occurs in children, the p*ak age
related incidence is between 5-75 years(co-"doa),
Epidemiological risk factor for rheumatic fever, include lawer standards of living especially cror,vriing, ihe diseasr: ir a.);

been more cotnmon among socially and economically disadvantageil populations9o*"d08).


Criteria to diagnose Rheumatic fever is Modified |one's criteria.

(riteria for Diagnosis of Rheumatic fever

1. Corditlsutt&s,0,el) A) Clinical
2, ArthrltiCutnslo,st) i) FevelAtoT)
3. SubcutaneousnoduledAilMs%t ii) ArthralgiarArrsr
4. Chareaatlrls 10) iii) Previous rheumatic fever or rheumatic heort disease(At 07)

5. Erythema marginatum B) Laboratory


i) Acute phase reactants; leukocytosis, elevated sedimentotiofi rate{Attrasl*)
and C reactive protein
ii) Prolonged PR interval in the electrocardiogram
Essential criteria
I Evidence of recent streptococcal infection as indicated by -
a) ln*eased antistreptolysin'O' titey'Attus 0e' e4 A' 07)
b) Positive throat culture
c) Recent scartel fever

t Two major or one major & 2 minor criteria are required in the presence of essential criteria to diagnose ecul*t
rheumaticfever.

Rheumatic carditis
o It is the most common acquired heart disease In 6hildlsn{atrs).
o It is seen in 40-600/o of patients. Usually seen within 3 weeks. It is a pancarditis@Gre8r involving all the three iayers I
Myocardium, endocardium & Pericardium.
o Rheumatic carditis is the only manifestation of acute RF that has the potential to ceuse long term disubility a*d d.c*h.

o The diagnosis of rheumatic carditis requires presence of one of the following:-


i) Organic cardiac murmurs not present previously.
ii) Congestive heart failure
111) Cardiomegaly
iv) Pericarditis
o Most common valve involved in RF is mitral valve(Pc|03) and mitral regurgitation@tl(S e4)
is the nnost comman defec{."
Carey-coombs murmu/AllMs06' PGt eT)
may be heard due to mitral regurgitationL|ltMs 06).
It is a sol (low pitctaed) miil-
06).
diastolic tnurmur$ItMs
o Aortic valve involvement (aortii,regurgitation) is seen in25o/o of cases(DNB Is). Tricuspid regurgitation is se*ln il:
r5).
10-30% cases@NB's). Pulmonary valve involvement is unusual(DNB
Arthritis in RF
o It is the mostfrequentmajor(etsa) manifestation (7570 of cases).
t Itis migratory polyarthritis(KeruIoee'PGI03), i.e., several joints are involved in quick succession and eaeh for a biir:t periorl
of time.
o Anyjointmaybeaffectedbutinvolvementoflargejointssuchasknee,ankle,elbowandwristiscommon.Small joir:'rs
ofhands and feet are not involved.
o Inflammation of any one joint subside spontaneously within a week and the entire bout of polyarthritis rarr:1y ii.is1

more than 4 weeks,


o Resolution is complete with ro residual joint damage(rN tst.
o A strikingfeature is ilramatic response to salicylatesrPcleat.
Taussing Bing anomaly
r Doubie-outlet right r.entricle with transposition of great arteries is knolvn as Taussig-Bing an<lmaly. It also has
associated YSD.
r Surgical correction is by: -
A\ Rastelli procedure ar
B) Arterial switch (lantene) pracedure

CONGISTIYE HTABT fAILUAE {C}If}


t Congestive heart failure is siate in which the heart canilot produce the cardiac out required to sustain the
metabolic needs of the body, withttut evoking certain coffipensatory mechanism.
r The most common cause of CHF in infants -+ Congenital heart disease9oned
0s).

r The rnost common cause of CHF in the older children -+ Rheumatic feveral ") & rheurnatic heart disease.

Timing of CllF in congenital heart diseaEes

Birth - 72 hours Pulmonary, mitral and aortic atresias.


4 dayr to 1 week , Hypaptaxicteft and right:heart syndreimestAiiltsdtransposiflon and malposlti,On'of.grgalat3ilq1:.;, ,r..:.:
1 week to 4 week Transposition and malposition complexes, endocardial fibroetastosis, coarctation of the aorta.
1. Month to 2 months Tra6strios,itio'n'and. mqlpqition cornplexes,.'endocardial cushion defutti, vehtrlcu[qr sep]a! dg!.gf.. pq.1qnt

ductus arterioirrt Tota*:antmalous-,pqlmonary v.qng'us -qnnection, anomdtoug',ldft tsrsne,ry'airtg*:$.fory


pulrnonaryai'tely.,',,,,
2monthsto6months Transpositionandmalpositioncompiexes,ventricularseptal defect,patentductusarteriosus,total anomalous
pulmonary venous connection, aortic stenosis, coarctation of the aorta.

Cliniralfeatures of CHF in infants


r In infants, heart faiiure may be difficult to identif,v. Prominent manifestations include.

Clinical features of CHF in infants

s Tachypnea t Cardiac enlargement a Hepatomegoly't{EEr)


, ,,: , .,,, :.,:, : i:r, ':
6 ]a6fiy6avcli3: , a Galfop'rhythm{53}' ,. F.acial,edema r:

c Cough {t Peripheral cyanosis r Jugarvenousengrogment


c Wheezing t smallvolur*e,prrlse , i,',,Edeiiia etlleelflstqa :..

o Rales in chest * Absence of weight gain c Ascitesecta3) (in older children)


r Hoarse Cry a Olig$riA{Paa3'tt?MEfr8t} :

a Pulmonaryedemaed$)

Trsatment
r Commonly useci drugs for treatment of CHF in children are :-
1) ACE inhibitors (drug of choicelipns0a))
2) Diuretics
3) Beta-blockers.
s l)ittretics (especialty doop diuretics) are the drug of choice to control pulmonary edema and systemic venous congestion.

RHEUMATIC FEVER

r Rheumatic fever is an acute irnmunologically mediated muitisystem intlammatory disease that occurs few weeks
08).
after an episode of group A streptoco ccal pharyngitis(co'ned
o Lhe disesse is imtnune mediated, not a communicable disease(co,ed98).
the penk age
o As is the case of streptococcal sore throat, acute rheumatic fever most often occurs in children,
related incidence is between 5-75 years$o*"doa) '
o Epidemiological risk factor for rheumatic fever, include lower standards o! living especially crowding,
the disease hai';
populations(conedol) '
been more commofi among socially and economically disadvantaged
o criteria to diagnose Rheumatic fever is Modified fone's criteria.
Criteria for Diagnosis of Rheumatic fever

1. CordlitistAtr[s,o,e4) A) Clinical
2, ArthrltistAttnsla,s4) i| Fevey'Atoz)
3. Subcutaneousnoduleso*Ms%) ii) ArthratgiarAt'5t
iiil. Previoi rheumatic fever or rheumatic heart disease(Atl7)
s ,o)
4. Chorea$:lr,t
5' Erythema marginatum '
;"T::::?tase reactants; teukocytosis, erevated sedimentotion ret*{Atir,s&t

and C reactive Protein


ii) Protonged PR interval in the eiectrocardiogram

Essential criteria
I Evidence of recent streptococcal infection as indicated by
-

a) lncreosed antistreptolysin 'O' titey'AttHs@' 'Ato7)

b) Positive throat culture


c) Recent scartel fever

e Two major or one major & 2 minor criteria are required in the presence of essential criteria to diagnase
acntle:

rheumaticfever.

Rheumatic carditis
r It is the most common acquired heart disease in children(Aris)'
o It is seen in 40-600/o of patients. Usually seen within 3 weeks. It is a pancarditis(Pcr
es)
involving ail the three ial''ers )

Myocardium, endocardium & Pericardium.


ttnd deti'n'
o Rheumatic carditis is the only manifestation of acute RF that has the potential to cause long term disabilitl'

I The diagnosis of rheumatic carditis requires presence of one of the following :-


i) Organic cardiac murmurs not Present previously'
ii) Congestive heart failure
i;;r) Cardiomegaly
iv) Pericarditis
e4)
yalveecl03) and mitral regurgitation$It*S is the most common defeca"
o Most common yalye involved in RF is mitral
sol (low pitched) mid-
Carey-coombs tn,rmur@lMso6,Pcle7) may be heard due to mitral regurgitation@ttMs06).It
is a
ou).
diastolic murmu/AttMs
o Aortic valve involvement (aortic regurgitation) is seen in 25o/o of cases(oN, "). Tricuspid regurgitation is seen ier

10-30o/o cases(DNB
r5). Pulmonary valve involvement is unusual(DNB's)'
Arthritis in RF
o It mostfrequentmaior@rst) manifestation (75% of cases)'
is the
period
o ltis migratory polyarthritisx<"tutaes,Pclo3), i.e., several joints are involved in quick succession and each for a brief
of time.
e Anyjointmaybeaffectedbutinvolvementoflargejointssuchasknee,ankle,elbow,andwristiscommon.Smalljoiirts
ofhands and feet are not involved.
r Inflammation of any one joint subside spontaneously within a week and the entire bout of polyarthritis rareiy last
more than 4 weelcs,
o Resolution is complete with no residualioint damage(rNte)'
o A strikingfeature is dtamatic response to salicylates(Pcl88) '
Sydnehamk chorea
o The characteristic picture of chorea is
that if ofien occur in isolation@cr'{ either
unaccompanied by other major
manifestations of RF or after a latent period of
several months, at a time when all other manifestations
subsided. of RF have
o Rapid purposeless involuntary movements
most noticeable in extremities and face.
Movements disappear during
sleep. Emotional liability is characteristic.
a Most patients recover in 6 months. sex
hormones (estrogen) or pregnancy can
cause recttrrences@cr 03) .

Subcutaneous nodute
c It is a rare manifestation of acute RF Subcutaneous
nodules are a late manifestation and appear 6 weeks after the onset
ofRF.
o It is often seen in association with carditis@cr 07. 03).
They do not appear as isolated manifestation.
o These are round' firm, painless dt nontenderecl0T'*3),rr..ly
extensor surfocs5eetoz) of tendons near the bony
-orubl subcutaneous lesions. They are found along the
prominences such as elbow, knees, wrists,
occiput and spine. They
last for a week or two and disappear spontaneously.

Erytherna nnarginatum
e Rash present on the trunk and proximal
parts of the extremities. Face is spared@cro3).
Macales or papules that extend
outwards while the skin in centre returns to
normal. Never pruritic,'' ,LvEr
never ,ruuraLe(l'
indurated. Kasn .
Rash ls evanescent migrating
from place to place. No residual scarring occurs.
Treatrnent of Rheunnatic fever
o Drug of choice for treatment o-f RF is penicillin. Erythromycin is the drug
07)
of choice in penicillin allergic patienlArrMs
"

o Drug of choice for prophylaxis of rheumatic


carditis is Benzathine
I Primary prevention :- Benzathine penicillin administared befor. penicillin :_
the
r Secondary prevention :- Benzathine penicillin is administered 9th day of streptococcal pharyngitis.
of patients who have had an episode of
Rheumatic fever this prevents further attacks.
o In penicillin aliergic patients erythromycinis
administered.
Suppnessive therapy
o corticosteroids or aspirin are given for suppressive
therapy. corticosteroids are indicated
es) in presence of carditis@Mu
with or without cHF Aspirin is preferred in absence
of carditis. Duration of therapy b l)
wssks(xuataot),

INFECTIVE ENDOCARDITIS

o Infection of the endocardial lining of the


heart is called infective endocarditis. Infection
may occur over the endocar-
or mural endocardium or the vascular endothelium.
The commonest site of infection is generally
lTril:}}i}l]es
c staphylococclts aureus is the most common causative agent in
native valve@Gr,i) and in intrauenous drug
r coagulase negative staphylococcus (Staph. epidermidis)
is the most common organism in prosthetic
abusers.

o Endocarditis is more common in previously valves.


damaged valves or in congenital heart
diseases :-
r) aigh risk lessions are : vsD@NB 12, ArrMS e8, Ar s4
prostheticvalves, ToF,rr^Sss,Arr4l pDA,
AS, andARrnrr ,r,,t"'"u'''\6t2'A)tMse8'AIe4)'11'111@Nno"nus''"'1 coA,
ii) Moderate risk lesions are : Tricuspid stenosis, tricuspid
regurgitation, pulmonic stenosis, and mitral
Infective endocarditis is rare in ASD@NB 12, ArrMS eB, Ar s4) stenosis.

YSD is the most common congenital heart disease


involved in infective endocarditis.
. r,. .':::i:i:ai'i::.::.ll:i:rl

Crr.q.pii*,

':::a'
.:.a::,

a o Roth spots
a * Oslert nodes
Night sweats o Janeway lesions
Anorexia r Splinter hemorrhages
& Fatigue * Stroke. myocotic aneurysm
t Weakness

MISCELLAI{EOUS

Hypertension
I Hypertension is defined as the arterial blood pressur e above 95th percentile with reference to age and sex.
o Between the 90th and 95th percentile it is called borderline high blood pressure.
t Mostcommoncauseofhypertensioninchildrenisrenalparendrymaldisease@l10'es'Pcre3).Amongrenalparenchyrnal
diseases chronic glomerulonephritis is the most common (2|o7o;rouo,nt,rcl0a) followed by chronic pyelonephritis(Ar
)s).

o Most common cause af renotascular hypertension in children is renal artery stenosis.


e Hypertension in newborn is most often associated with umblical artery catheterization and renal
artery thrombosis.

r::,! tdlk
1-3 months 75+5 50+5
4.'12 rn0iiths '.,'84:+.5:.,..
651..5
1 -B years 95+5 65+5
9:14 years. ,.,,105 15r. 65'+ 5

Differential cyanosis
r When one extremity is cyanotic and the other is pink, it is cailed differential cyanosis. Causes are :-
1) Upper limb : Transposition of great arteries, preductal f,64rarrs1 secondary pulmonary hypertension, Interrupted
aortic archGr's).
ii) Lower limb ' PDA(PGIee), Persistent pulmonary hypertension(A/r5), right to 1eft shunt.

* Umbilical cord contains:Two arteries and one vein.

o ln fetal circulation'. Brain and heart (coronary) circulation receive blood with higher oxygen saturation.

. Anatomic closure of ductus arteriosus occurs'. Between l0 to 25 days after birth.

e Ebstein's anomaly is seen with intake of : Lithium.

.1..:.-$ffigesia.miIialieidencyin..cHDS.i'kno,wiior.:rtolroram.s}rndrome(ASDwithbanyabnormalities).
o CHDs in congenital rubella:PDA (most common), pS, VSD, ASD (least common).

. t: .:.aa. :::_
lmportont cyanotic CHDs with pulmonary oligemia: Tricuspid atresia, TOEEisenmenger's syndrome, Ebstein anomaly.
'
i:s..;*8,.T-EBr4,

* lmportant acyanotic CHDs with pulmonary


plethora:pDA, VSD, ASD, pApVR.
* lmportont CHDs with pulmonary plethora:TGA,
TAPVR, PDA, vsD, AsD, pApvR, persistant
a Cyanotic CHD with left ventricle hypertrophv: Tricuspid atresia.
truncus arteriosus, singre ventricre.

a lmportant CHDs with right ventricular hypertrophy


(right s^
axis deviationl
wcvtullut t) -: A)u
mengerb syndrome, Ebstein,s anomalf.' ' ' (with HBBB),
A5D (wltn pApvR, P5,
RBBB), PAPV& p5, fiilSf
MS, COA (in infant), TOf;,
ToE tEisen_
terft ventricutar hvpertrophv (teft..'
,' axis .t)
deviation) :VSD, pDA, As, AR, coA,
: ::::Tr:::;:,ith MR, ricuspid atresia, putmonary
atresia.
a CommonestcyanoticCHD:TOF.
* Recurrent chest infections ore seenrn : ASD, VSD, pDA, TGA,
TAPVR.
._ i:re.cti.ons is not a reature;;,.;;,;;;;;;;.;
atresia, rricuspid atresia, ps, AS.
v 1:,,,::,:,h:t
Lyanottc LHD which does not develop cardiomegaly
or t,CHF:TOF.
4rl vt il . tva.
o Left atrium is not enlargedin: ASD.
* ASD moy be associated with: Halt-oram syndrome, TAR syndrome,
syndrome. Down syndrome,
yrrurome, tllls-van
Ellis-van crevald syndrome, EDH,
fetal alc,
alcohol
- of right atrium & rigth ventricle, pulmonary
:ffif*:: rHvpertrophy prethora, purmonary hypertension,
reft to right shunt, wide
* Not seen in ASD: Left atrial enlargement, cHF, right to left
shunt, shunt murmur, infective
* Most common ASD:Ostium secundum type. endocarditis.

a ln ASD, aorta is i Small_


ASD with mitral stenosis: Lutembacher syndrome.
True abouttl5D: Membranous most common, teft
to rigfrt shunt, pansystol,.
trophy, CHF. endocarditis common, p,
Tl::r, wide split S, , left atrial & left ventricle hyper_
i,,;;.;y;L;;"i., ;;.t; ;";;il;:";;"r,.neousty.
& lmprovement of CHF in a large vsD is due
to: Development of vascular changes ,.
Clubbing in a patient with vsD suggests:rnfective
0r,r"".,
circulation.
endocarditis.
*; Differentiating feature between ASD and
VSD on chest X-ray : Sizeof left
atrium (enlarged in VSD but not in
tn PDA connection is between: Aorta ASD).
and pulmonary artery.
& D.uctus arteriosus forms: Ligamentum
;;;r';n. ; post-natal life.
* Potency of ductus arteriosLts causes:
Machinery murmur.
* Symptoms of pDA are similar to : Aortopulmonary
window
a PDAis morecommonrn: Females.
e lmportant features of PDA: continuous (machinery)
murmur, pulmonary prethora, reft
adoxical split of S, atriar and reft ventricre enarrgemement,
. par-
* Complications of PDA:CHF'infective endocarditis, pulmonary
hypertension, chest infection, Eisenmenger
* y.*:.ro*^o.n cause of death in pDA: cHF (most common) ""'=" syndrome, aneurysm of pDA.
f",;*J;;;*.,,r. .;;":.;;;;.
e Medtators which mointain patency of ductus arteriosus: prostagla
ndins.
e Drugs used to maintain patency of pD^ : prostagrandin (misoprost,
E, rioprostir).
s Closure of PDA is stimuloted by: Prostaglandin ,-nn,o,,orr,
i.e. NSAIDs (mit commonly
indomethacin).
* Constituents of ToF:vsD'overriding/dextroposed aorta,
right ventricurar r,yp"rtropr,y,
o Constituents of pentalogy of Fallot:Components of ,r;;;;. r;""osis (infundiburar stenosis).
':,
* Most common site of pulmonia stenosis (right ventricular
plus ASD.
TOF Plus-ruul
'I:r
outflow obstruction) inroF: lnfundibular
Q tmportantreatures in roF: Ejection svstotic murmur stenosis.
s, , right to reft shunt. right ventricuiar
in 2"d intercost.,
*;.;, .r.*, ;, :;;;;;;;", ri",,r, ror"uins, normrJVp, sinsre
*
hypertrophy, origemic l;dfifi;, ;"ot shaped heari(c;;;, sabot).
Not seen in ToF : cHF,
shunt murmur, cardiomegary, recurrent chest
infections.
* Boot shapted (couer an sabot) heart is
seen in :TOF.
& Drug which should be avoided in IOF: lsoprenaline.
a Blalock-taussig shunt is between: subcravian artery and purmonary
artery.
6 Pott's shunt is between: Descending aorta and pulmonary
artery.
e Left ventricle hypertrophy with central cyanosis is seenln
Tricuspid atresia.
. :

ffi;":T::l:;;:i:l';tr:lrffi;r""sis since birth, patent foramen ovate


or A5D, sinste s,, reft ventricutar hypertrophy
with
Cneprun

True about Ebstein anomaly: Downward displacement of tricuspid valve, right atrial dilatation, systolic and diastolic murmur with
scratchy character.
CHD which may be associated with WPW syndrome: Ebstein anomely
position of aorta in transposition of great arteries (TGA) : Anterior and right to pulmonary artery.

Trueabout TGA: Cyanosis at birth, intact septum or VSD, CHF, sinq le S, , pulntonary plethora, 'Egg on side' on X-ray.
Treatment of TGA includes: PGE, Rashkind atrial septostomy, arterial switch procedures (Mustard & senning or Jatene).
I"4ost common conotruncal defect: TGA
M o st com m on type of TAPVC : Su pra-ca rd iac (Type 1 ).

tnTApVC:Oxygen saturation is higher in pulmonary artery, infracardiac type is always obstructive, pulmonary venous blood reaches
in right atrium, blood from right atrium to left atrium florvs through patent foramen ovale or ASD (but notVSD).
Figure of '8' configuration or snowman Gppearance on chest X-ray :TAPVC (su pracardiac type).

Eisenmenger complex is common in : VSD.


Sequence of events in developemtn of Eisenmenger complex in l/5D : Left to ' g ht s r l rt , rig ht ventricle hypertrophy -+ Pu lmonary
t.rypertu,lsion -+ right to lefi shunt.
lmportant signs of impending Eisenmenger's syndrome: Loud Pr, single Sr, parasternal heave, pulmonary ejection click, Grahm steel
murmur.
lmportant features of Eisenmenger syndrome: Dilation of central pulmonary artery with peripheral prunning, no dilatation of pul-
monary veins.
CHD with risk of Mt in a neonate'. Anomalous origin of coronary artery.

Coarctation of aorta {CoA) is common with:Iurner syndrome-


Coarctation is associated with : Bicuspid aortic valve (most <ommon), Aortic stenosis, ASD, VSD, MR
CoA is not associated with: Pulmonary stenosis.
Most common site of CoA: Just below origln of left subclavian artery.
Rib notching in CoA is seen at ilnferior border.
Rib notching is due to: Enlargement of collateral vessels (especially intercostal vessels).
Collateral vessels involved in CoA : lnternal thoracic, intercostal, subclavian, superior & inferior epigastric, axillary
CoA is characterized by:Weak/absent pulses in lower limb (femoral artery) compared to upper limb
(brachial artery).

Common presentation of CoA: lntermittent claudication, headache, dizziness, hypotension, dyspnea on exertion, weak pulse in
femoralvessels.
lmportant causes of death'in CoA: CHF, aortic dissection, cerebral hemorrhage, infective endocarditis, hypertension, aortic rupture'
Ductus dependent CHDs: P5 without VSD, TOF, CoA, TGV, Hypoplastic left heart, congenital aortic stenosis, Tricuspid atresia.

Ductus independent CHDs:TAPVR, Truncus arteriosus, anomalous origin of left coronary artery from pulmonary artery.
Acyanotic heart disease with eiettion systolic murmur: ASD, VSD, PDA, CoA

Right aortic arch is associated iarlth : tru ncus arteriosus, TOF-


Double aortic arch commonly present as: Compression of trachea.
Commonest cause of CHF in infants: Congenital heart disease.
Commonest cause of CHF in older children: Rheumatic heart disease
lmportant causes of CHF in first week of life: Pulmonary/mitral/aortic atresias, TGA, hyoplastic left and right heart syndromes.
Treatment of choice for CHF in children: ACE inhibitors.
Rheumaticfeveris:Not a communicable disease (it is an immune mediated damage).

True about'RF:Occurs after streptococcal pharyngitis, affect 5-15 years old in low socioeconomic status.
Major criteria in diagnosis: Carditis, arthritis, chorea, subcutaneous nodule, erythema marginatum.
RF

Minor criteria in RF diagnosis: Fever, arthralgia, previous RF, ^ ESR, leukocytosis, ^CRq prolonged PR interval on ECG.
Most common valve involved in RF: Mitral valve.
Most common valvular defectln RF: Mitral regurgitation'
Most common maior manifestation in RF: Arthritis'
Carey-Coombs mLlrmur.. Soft (low-pitched) mid-diastolic murmur in RF due to mitral regurgitation.
Sydenham's chorea of RF:Occurs in isolation; aggravated by sex hormones (estrogen) and pregnancy.
Subcutaneous nodulesin RF: Non-tender, on extensor surface, often seen in association with carditis.
I
QUESTIONS

c) Assessment of child for presence of heart disease


FETAL CIRCULATION
d) Assessment of child for degree of mental
retardation
1. Oxygenated blood to the fetus is carried by -
(PGl lune 07)

a) Umblical arterY CLASSIFICAT!ON OF CHDS


b) Umblical vein
c) SVC 9. Congenital cyanotic heart disease with pulmonary
d) PulmonarY atterY oligemia is seen with - (PGl Dec 99)

2. True about fetal circulation- (All India Dec'lj Pattern) a) ASD


a) Blood in SVC has more oxygen saturation b) VSD
b) Pressure in left ventricle is more c) Tricuspid atresia
c) Brain receives blood with low oxygen saturation d) Hypoplastic left ventrlcle
d) Heart receives blood with high oxygen saturation 10. A child with central cyanosis and enlarged left ven-
-r. Anatomical closure of ductus arteriosus occurs at - tricle the probable diagnosis is - (CET June 14Pattern)
(Atl lndia Dec.14 Pattern) a) Tricuspid atresia
a) Birth b) Eisenmengher'ssyndrome
b) 3-4 day c) Tetralogyoffallots
c) 10th day d) Anomolous pulmonarY arterY
d) 30'h day 11. Which one of the following does not produce cyano-
sis in the first year oflife - (cET Nov. 15 Pattern)
a) Atrlal septal defect
CONGENITAL HEART DISEASES b) Hypoplastic left heart sYndrome
ETIOLOGY c) Truncus arteriosus
d) Double outlet right ventricle
4. Ebstein's anomaly is seen with intake of - 12. Right axis deviation is seen in all except -
(CET Nov.14 Pattern) (AIIMS lune 98)

a) MercurY a) VSD b) TricusPid atresia


b) CopPer c) Pulmonary atresia d) ASD
c) Lithium t3. Cyanosis is seen in - QGI Nov 14)

d) Lead a) Persistent ductus arteriosus


5. Which of the following syndromes is best associated b) Tricuspid atresia
with congenital heart disease - @ltMS MaY 0s) c) Ostium primum ASD
a) Lesch-NyhansYndrome d) Eisenmenger comPlex
b) Rasumussen syndrome e) TetralogyofFallot
c) Holt Oram s1'ndrome 14. Plethoric lung field is seen in which CHD -
d) LEOPARD sYndrome (All India Dec.li Pattern)

6, Holt oram syndrome is characterised by - a) Ebstein's anomaly


(All lndia Dec15 Pattern) b) Tricuspid atresia
a) ASD c) AS
b) VSD d) VSD
c) TGA 15. Commonest type of congenital heart disease is-
d) TAPVC (Ar ee)

7. The heart lesion not found in Congenital Rubella a) ASD


infection is - (NEET Dec.l2 Pattern) b) VSD
a) ASD c) TOF
b) VSD d) PDA
c) PDA 16. Commonest tgre of cong' cyanotic heart disease is -
d) PS (NEET Dec.12 Pattern)

a) ASD
NADffS CRITERIA b) VSD
c) TOF
8. NADAs criteria are used for- (All India Dec li Pattern) d) PDA
a) Assessment of child for degree of dehydration 17. Recurrent respiratory tract infections may occur in
b) Assessment of child for degree of malnutrition all of the following excePt - (AllMS Nov 05)
a) Ventricular septai defect d) Ventricular septal defect with puimonary arterial
b) TetrologyofFailot hlpertension.
c) Transposition ofgreat arteries
d) Total anomalous venous return 26, ASD is associated ryith all except - (NEE: Dec.12 pauern)
a) Infective endocarditis
18. In which of the following conditions left atrium is
b) Stroke
not enlarged - (Ar 06) c) Arrhythemia
a) Ventricular septal defect d) Puimonaryhypertension
b) Atrial septal defect
c) Aortopulmonarywindow 27 . Hinar danee on {luorosccpy is seen in -
d) Patent ductus arteriosus (ALl India Dect5 pattern)
A) ASD
19. Which one of the following congenital heart diseases
b) VSD
has cyanosis without cardiomeglay and/ or conges_
c) PS
tive heart failure - (upsc s8) d) rn
a) Transposition ofgreat arteries
b) Fallots tetralogy
c) Congenital mitral regurgitaion VENTRICULAR SEPTAL DEFECT
d) Congenital pulmonary stenosis
28. Moot eommon type of ySD- (All India Dec.14 Pattern)
ATRIAL SEPTAL DEFECT a) Membranous b) Muscular
c) Multiple d) None
2*- Lerter*bacher symdrome - in*luda A/E - 29" True atrout VSD are a.ll exeept - (All tnrlia Dec.t3 pattern)
(cET luly 15 Pattern) a) Left to right shunt
a) Mitrar stenosis b) Pansystolic mormor
b) eso c) Reverse spliting of S ,
c) VSD d) Leti arria[ hyperrrop-hy
d) Left to right shunt 30. A child with large perirnembranous vSD has
Moot eornrnon AS& is - lAll lndia Dec j5 pattern) congestive heart failure. What may be the cause of
a) Ostium primum improvement of,cardiac failure in the patient -
b) Patent foramen ovale (AIIMS Nov 01)
c) ostium secondum a) Aorticregurgitation
d) Sinus venosus b) Vascular changes in pulmonary circulation
22_ &l*lot eomrnon ASD ie down syredrorne is _ c) Infective endocarditis
(All India Dec15 pattern) d) Closure of VSD spontaneously
a) Ostium primum 31. A 29-day old child presents with features ofconges_
b) Ostium secondum tive cardiac failure and left ventricular hypertrophy.
c) Absent atrial septum Auscultation shows a short systolic murrnur. Most
d) Sinus venosum likely diagnosis is - (AIIMS Nov 2K)
23. All of the following are true about ASD except -
a) Rheumatic fever
(Ar 01)
b) TetralogyofFallot
a) Right arrial hlpertrophy c) Transposition ofgreat arteries
b) Left atrial hypertrophy d) Ventricular septal defect
c) Right ventricular hypertrophy JZ. A patient of VSD in CCF develops clubbing with no
d) Pulmonary hypertension cyanosis diagnosis is -
@Gr eB)
24. Irr atrial septal defec{ t}ee aorta is - (NEET Dec.12 pauern) a) Right to left shunt
a) Small b) Left to right shunt
b) Normal c) Subacute bacterlal Endocarditis
c) Enlarged d) Pulm. edema
d) Aneurysmal JJ. True about VSD is all except - @ttMS lune e7)
?< A young female presents with history of dyspnoea a) Small hoie closes spontaneously
on exertion. On examination, she has wide, fixed b) Defect is usually in membranous part
split S, with ejection systolic murrnur (ilr/w) inleft c) Endocarditis is a common complication
second intercostal space. Her ECG shows left axis d) Pulmonary Oligemia in chest x-ray
deviation. The most probable diagnosis is - 34. Which of the following features on X-ray chest can
(AIIMS May 2003) differentiate an Atrial septal Defect (ADH) from e
a) Total anomalous pulmonary venous drainge. Ventricular Septal Defect (vsl) (At u)
b) Tricuspid atresia. a) Enlarged Left Atrium
c) Ostium primum atrial septal defect. b) Pulmonary Plethora
ie H,Ap!jrrR::4::,:

c) Enlarged pulmonarY ArterY b) Common heart lesion in rubella


d) Enlarged Aorta c) Treatment is closure of defect by ligation and
division of ductus
d) Hlpoxia and immaturity are important in
PATENT DUCTUS ARTERIOSUS maintaining the PatencY
44, L;rrge P{}A (Patent tluctus arteriosus) le*ds t*-
r. {n paten[ duc{els arteri*sars cc}nn*eti$n is hetweear - (NEET Dec.12 Pattern)
(liEl:T tr]ee" I 2 P*t{€rin
a) Aorta and coronarY arterY a) Endocardialvah,'ulitis
b) Aorta and Pulmonary artery b) Eisenmengersyndrome
C) CHF
c) Aorta and subclavian artery
d) Pulmonary arlery and subclavian artery d) Ali of above
r6. Ftr)A true is alX exeePt - (All India Dec15 Pattern)

a) More common in Preterm bab,v FALLOT'S TETRALOGY


b) Leftto rightshunt
c) Acyanotic congenital heart disease "45. Tl!* foll*rtimg fuatueres are trl'e i-)r t*tral&$ry'*f F*1-
lot. er*rept- (All Inditt Dec.li Pattern, Al 06)
d) More common in term babY
a) Ventricular sePtal defect
37. True about I year old child with PDA is-
(AllMS Ma1 9s)
b) Right ventricular hlPertroPhY
c) Atrial septal defect
a) Symptoms similar to Aorto pulmonary window d) Pulmonary stenosis
b) Chances ofspontaneous closure high
c) Indomethacin may help in closure 46- Essential criteria for TOF includes all except -
(AIIMS Nov 07)
d) Endocarditis is rare
a) Valvular stenosis
38. h4C eause of *treath in acluit rsith PI]-A is -
(All India Dec.13 Pattern)
b) Infundibular stenosis
a) CCF
c ) Overriding ol aorta
d) RVH
b) Infective endocarditis
c) Rupture 47 . AlI of the following are true regarding Tetrclogy of
Fallot except - (AIIMS May 0s)
d) Embolism
a) Ejection systolic murmur in second intercostal
39. Preterm batry with PDA, which is the least Iikely
(ALLMS Nov l0)
space
findings?
b) Single second heart sound
a) CO, washout c) Predominantly left to right shunt
b) Bounding pulses d) Normal jugular venous Pressure
c) Pulmonaryhemorrhage *ll*l o{ tk* toBl*llv-
d) Necrotising enterocolitis 4S. T*tr;el*g,y c.lt Fxtrlart's f r*s*mt q'i{Xr

i*g- (AIl India Dec"14 Pattern)


40. A premature infant is born with a patent ducftrs
a) Cenral cyanosis with ciubbing
arieriosus. Its closure can be stimulated by adminis-
(AIIMS May 06)
b) Cardiomegaly
tration of - c) Left ventricular hlPertroPhY
a) Prostaglandin analogue d) Normal ECG and Chest x-raY
b) Estrogen i***
c) Anti-estrogencomPounds 49. tr&hick of th* fotr}*rving does not complieate
CHF- (CET Nov 14 Pattern)
d) Prostaglandin inhibitors
a) Correlation ofaorta
at
.tt- ?ne rnost appropriate managermemt for rnaintaileing
b) Transposition ofgreat vessels
patency of <luctus arterisstl{i irr a neomate is -
(All lndia Dec. 15 Pattern)
c) Tetralogy of fallot's
d) Patent ductus arteriorus
a) Prostaglandin E,
b) Oxygen 50. In which of the following a'Coeur en Sabot' shape of
c) Nitric oxide the heart is seen - 6t o'l)

d) Indomethacin a) Tricuspid atresia


in FEA is - b) Ventricular sePtal defect
Fr*rstaglandin anaiogue usec.{
(All lndia Dec15 Pattern)
c) Transposition ofgreat arteries
d) Tetralogy ofFallot
a) Anaprastone
b) Misoprost 51. A 6 month old child with Tetralogy of Fallot devel-
ops cyanotic spell initiated by crying. Which one of
c) Danaprostone
d) PGE-2 the following drugs you would like to avoid-
(AlIMs Nov 04)
43. All of the following about PDA are true except -
(At 08) a) Sodium bicarbonate
b) Propranolol
a) More common in males
c) Phenylephrine d) Left axis deviation in ECG.
d) Isoprenaline
61. True about Ebstein anomaly is? (Alt India Dec. ts pauern)
52. Potts shunt is - (Ar 01) a) Right ventricular dilatation
a) Rt subclavian artery to rt pulmonary artery b) Rightatrialdilatation -
b) Descending aorta to left pulmonaryartery c) Left ventricular dilatation
c) Left subclavian to left pulmonary artery d) Left atrial dilatation
d) Ascending aorta to right pulmonary artery
Ebsteins anomaly is associated with -
Blalock and Thussig shunt is done between - 61 06)
1IPMER80'DNB91)
a) Aorta to pulmonary artery a) Tiicuspid atresia
b) Aorta to pulmonary vein b) Coarctation ofaorta
c) Subclavian artery to pulmonary vein c) Cardiomyopathy
d) Subclavian vein to artery d) Rightventricular hepertrophy
54. End.stage treatment of TOF is - Gf,f P1y 15 Pattern)
a) BTshunt
b) Modified BT shunt TRANSPOSITION OF GREAT VESSELS
c) Waterston shunt
d) Pott - shunt operation 63. Most common conotruncal anomaly -
55. Which of the following is a component of pentalogy (AlllndiaDecl;Pattern)
of Fallot - a) TGA
(Ar 11,07, ArrMS May 10) P
a) Atrial septal Defect (ASD) b) Tetralogyoffallot
b) Patent Ductus'Arteriosus (pDA) c) Truncus arteriosus
c) Coarctation of Aorta (COA) d) Double oulet right ventricle
d)Left Ventricular Hypertrophy (fVH) In TGA, position of aorta is- (All India Dec.13 pattern)
Tetralogy offallot is characterized by following a) Posterior and left to pulmonary artery
except - (NEET Dec.12 Pattern)
b) Posterior and right to pulmonary artery
a) AS c) Anterior and left to pulmonary artery
b) vsD d) Anterior and right to pulmonary artery
c) Over-riding of aorta 65. A neonate has central cyanosis and short systolic
d) Infundibular constriction murmur on the 2nd day of birth.The diagnosis is _
57. Fallot physiology includes all except - (AIIMS May 01)

(All India Dec15 pattern)


a) TetralogyofFallott
a) TOF b) Transposition ofgreatvessels
b) Eisenmenger complex c) Atrial septal defect
c) TGA d) Ventricular Septal defect
d) Tricuspid atresia 66. A five day old, full term male infant was severely
cyanotic at birth. Prostaglandin E was administered
initially and later ballooned atrial septosomy was
TRICUSPID ATRES!A
done which showed improvement in oxygenation.
The most likely diagnosis of this infant is -
58. A five year old child presents with left ventricular (Ar 04)

hypertrophy and central cyanosis what is the most


a) TetralogyFallot
probable diagnosis -
b) Tiansposition ofgreatvessels
(AIIMS Nov 2000)
a) Tricuspid atresia c) Truncus Arteriosus
b) Eisenmenger syndrome d) Tricuspid Atresia
c) TetrologyofFallot
d) Total anomalous,pulmonary venous drainage TAPVC
59. A patient presents with LVH. and pulmonary
complications. ECG, shows left axis deviation. Most t?. The most common type of total anomalous pulmo_
like$ diagnosis is - (Ar 01) nary venous connection is- (CET Nov. 14 Pattern)
a) TOF a) Supracardiac
b) Tricuspid atresia b) Infracardiac
C) TAPVC c) Mixed
d) TGA d) Cardiac
AII ar: true regarfing tricuspid atresia e,xcept -. 68. All of the following statements regarding total
(NEET Dec'12 Pattern) anoriralous pulmonary connection are true except -
a) .split s,
b) Patent foramen ovale a) The total pulmonary venous blood reache , rn'!""
c) Pulmonary oligemia in chest X-ray right atrium
b) Always associated with a VSD d) Noonant
c) The oxygen saturation ofthe blood in the
76" Coarctation of anrta is rnost commonly associated
pulmonary artery is higher than that in the aorta
with - (All India Dec.13 Pattern, AI 11,08)
d) Infracardiac tlpe is always obstructive
a) VSD
-: . Figure of I configuration on ehest x-ray is seen in - b) PDA
(All India Dec. 14 Pattern) c) Bicuspid aortic valve
a) TOF d) ASD
b) ASD 77. Coarctation ofaorta is associated with all, except -
c) TAPVC (AIIMS lune 98)
d) VSD a) Tlrrner's syndrome
-tl. Total anomalous pulmonary venous connection b) Bicuspid aortic valve
false statements is- (Ar e7) c) Pulmonary stenosis
a) All pulmonary viens enters by single trunk d) Atresia ofaortic arch
b) Need not always be associated with septal defects 78. Trueaboutco-arctationofaorta - (AIss)
c) Radiologicallyhas figure of8 appearance a) Most common site is distal to the origin of the left
d) Is a cyanotic heart disease
subclavian artery
b) Most common age of presentation is at 15-20 years
EISENMENGER SYNDROME c) Upper rib notching is due to erosion by dilated
collateral vessels
-1. Eisenmenger complex is common in adult in - d) Right ventricular hlpertrophy
(PGI June 2K) 79. In post ductal coarctation ofthe aorta, blood flow to
a) VSD the lower limb is maintained through which of the
b) ASD following arteries - (AIIMS Nov 07)
c) PDA a) Umblical artery and subcostal arteries
d) Cushion defect b) Thoracic and pericardiophrenic arteries
-2" A child with VSD presents with development of c) Intercostal arteries and superior epigastric artery
cyanosis because of Eisenmenger physiology. What d) Ant and post circumflex arteries
is the correct sequence of events which leads to this 80. A child presented u,ith headache, dizziness, inter-
change - (AllMS Nov 2000) mittent claudication with occasional dyspnoea. The
a) Left to right shunt, pulmonary hypertension, right most probable diagnosis in- (Alt India Dec.J4 pattern)
ventricular hypertrophy, right to left shunt a) ASD
b) Left to right shunt, right ventricular hlpertrophy, b) PDA
pulmonary hlpertension, right to left shunt. c) TOF
c) Pulmonary hypertension, right to left shunt right d) Coarctation ofaorta
ventricular hlpertrophy, left to right shunt.
81. All of the following causes death in coarctation of
d) Left to right shunt, right ventricular hypertrophy,
Aorta except - (PGI lune 2000)
right to left shunt, pulmonary hlpertension.
a) Infectiveendocarditis
73. All are signs of impending Eisenmenger except - b) CCF
Mav 10) c) Intra cranial hemorrhage
a) Increased flow murmur across tricusp i,ts d) Anterior MI
pulmonaryvalve
b) Single 52
82. A l-month-old boy is referred for failure to thrive.
On examination, he shows feature of congestive
c) Loud P2 failure. The femoral pulses are feeble as compared to
d) Graham steel murmur
branchial pulses. The most likely clinical diagnosis
74. Eisenmenger syndrome-True are A/E- (AIIltgMay 10) is - (At 06)
a) Pulmonary veins are not distended a) Congenital aortic stenosis
b) RV & LV walls come back to normal size b) Coarctation ofaorta
c) Dilatation of central pulmonary artery c) Patent ductus arteriosus
d) Peripheral pruning of pulmonary arteries d) Congenital aortoiliac disease
83. A ten year old boy presents to the pediatric emer-
COARCTATION OF AORTA gency unit with seizures. Blood pressure in the up-
per extremity measured as 200/140 mm Hg. Femoral
75. Coarctation of the aorta is common in rvhich syn- pulses were not palpable. The most likely diagnosis
drome - (NEET Dec.12 Pattern) amongst the following is - (Ar 10)
a) Downt a) Takayasuaortoarteritis
b) Turnert b) Renal parenchymal disease
c) Klinefelter's c) Grandmal seiures
d) Coarctation ofaorta b) Truncus arteriosus
c) Corrected TGA
OTHER CHD'S d) Total anomalous pulmonary venous connection
92. The most commofi presentation of doutrle aortic
84. Most cotnrnon syndrome associated with A-\r canal arch in infants is ? (CET Nov. 1j Pattern)
defect - (All Intlia Dec15 pattern) a) Murmur
a) Down syndrome b) Tracheal compression symptoms
b) Klinfelter syrdrome c) Positional hyperemia with right upper arm oedema
c) Turner syndrome d) Bleeding
d) Marfan syndrome %. Infant with double aortic arch presents with -
65. A neonate has recurrent attacks of abdominal pain, (CET luly 15 Pattern)
restless irritability and diaphoresis on feeding. Car- a) CATCH22
diac auscultation reveals a nonspecific murmur, He b) Digeorge syndrome
is trelieved to be at risk for &I.I. Likely diagnosis here c) velo-cardio-facial syndrome
ls- (At 0t) d) All of above
a) Asd 94. Brain abscess in Cyanotic heart disease is commonly
b) Vsd located in - (All tndia Dec15 pattern)
c) Tof a) Cerebellar hemisphere b) Frontal 1obe
d) Anomalous coronary artery c) Temporal lobe d) parietal lobe
86. A child is admitted on 7 days of life with severe 95. WPW syndrome is associated rvith -
respiratory distress and shock. He was discharged (All India Dec15 pattern)
2 days back healthy. What could be the probable a) Ebstein anomaly
diagnosis - (AIIMS May 11) b) TOF
a) VSD large c) VSD.
b) Hlpoplastic left heart syndrome d) TAPVC
c) Ebstein anomaly
d) AP window defect
87. The following cardiac defects are characterized by
HEART FAILURE
ductus dependent blood flow except -
96. Commonest cause of heart failure in infancy is-
(AIIMS May 11,05.02)
(Comed 08)
a) Transposition ofgreat arterics with intact septum
a) Myocarditis
b) Interrupted aortic arch
b) Rheumatic fever
c) Truncus arteriosus
c) Cardiomyopathy
d) Hypoplastic left heart syndrome
d) Congenital heart disease
88. PGEcausesworseningininfantwith? (AtrMSNov u) 97, The commonest cyanotic heart disease manifesting
a) PS without VSD
as congestive
cardiac failure during first week oflife
b) Hlpoplastic left heart syndrome ls- (All India Dec.13 Pattern)
c) Obstructive TAPVC a) Pulmonary stenosis
d) Obstruction in aorta
b) Fallott tetralogy
89. A blue new born presents with cyanosis. The X-ray c) Tricuspid atresia
chest reveal oligaemic lung field and normal sized d) Hlpoplastic left heart syndorme
heart. Most likely diagnosis is - (AtrMS Nov 0t)
9g. Congestive cardiac failure is diagnosed in an infant
a) Ebsteint anomaly
b) Pulmonary atresia
bY - (NEET Dec.t2 pauern)
a) Basal crepts
c) Transposition ofgreat arteries b) Elevated fVP
d) Tetralogyoffaliot
c) Pedal edema
90. A child after weeks ofbirth acyanotic, ejection d)
\
systolic murmur detected causes arelis - (pGr May t4)
Liver enlargement
gg. Which of the following is not a characteristic of
a) V.S.D.
right sided failure - PGI 83)
b) P.D.A. ]LPMER 80,
a) Pulmonary oedema
c) TOF b) Ascites
d) Coarctation ofaorta
e)
c) Oliguria
Tricuspid stenosis
d) Dependent oedema
91. Right aortic arch is most commonly associated with- 100, The treatment of choice for a case of congestive fail-
(Ar 08)
ure with hypertension is - (fipmer 0t)
a) Tetrologyoffallot a) ACE inhibitors

il
b) o - blockers (CET Nov 15 Pattern)
c) Cal. channel blockers a) Aortic valve b) Tricuspid valve
d) Nitrates c) Mitralvalve d) Pulmonaryvalve
tLz. Drug of choice for Rheumatic fever prophylaxis in
penieillin allergic patient - (AIIMS May 07)
RHEUMATIC FEVER a) Erythromycin b) Clindamycin
c) Vancomycin d) Gentamycin
lrDl. All are true about rheumatic fever, except - l 13. Sternids are given in rheumatic fever when there is-
(AIIMS June 99)
(All India Dec.14 Pattern)
a) Common in poor socioeconomic group
a) Carditis b) Chorea
b) Develops after streptococcal pharyngitis
c) Subcutaneous nodules d) All
c) Communicabledisease
d) Seen in 5-15 years ofchildren I14. In a patient of rheumatic carditis full dose of steroid
is given for - (Kerala 04)
n02. Which of the following is a minor criteria for diag-
a) 3 weeks b) 6 weeks
nosis of Rheumatic fever fRF) according to modified
c) 9 weeks d) 12 weeks
Jones criteria - (Ar 07)
a) ASO titre b) Carditis
c) Fever d) Subcutaneous nodules INFECTIVE ENDOCARDITIS
103. Which is not a major criteria of fones in Rheumatic
fever? (AIIMS Nov to) 1 15' Bacterial errdocarditis is most commonly caused by-
a) Pancarditis b) Arthritis (PGl Dec 03)

c) Chorea d) Elevated ESR a) o-Hemolytic Streptococci


104. Major criteria for rheumatic fever - A/E - b) p-Hemolltic Streptococci
(AlI India Decl5 Pattern)
c) Staphylococcusaureus
d) Cardiobacterium
a) Carditis b) Arthralgia
e) Staph epidermidis
c) Erlthema marginatum d) Subcutaneous nodule
I 16. Infective endocarditis least common in?
105. True about Rheumatic carditis - (cET lune 14 Pauern)
(CET Aug.12 Pattern)
a) Only endocarditis b) Only myocarditis
c) Only Pericarditis d) Pancarditis a) Sever MR b) Severe AR
c) Small VSD d) Small ASD
106. About carey commb's murmur which is false -
(AIIMS Nov 06)
IL7. A?year old known case of RHD presents with 3 wks
history of fever, hematuria and palpitation diagnosis
a) Delayed diastole murmur
(PGl Dec 99)
b) Seen in rheumatic fever 'utf
s,..pro.occal endocarditis
c) Can be associated with AR b) Collagen vascular disease
d) Low pitched murmur
c) Reactivation
107. Carey coomb murmur is seen in - (PGt Dec 97)
d) Staphylococcalendocarditis
a) Severe mitral stenosis
b) Acute rheumatic carditis
c) Pure aortic regurgitation HYPERTENSION
d) Severe pulmonary hypertension
118' Sustained severe hlpertension in children is most
108. Most common manifestation of rheumatic fever -
(All India Dec.14 Pattern)
comnionly suggestive of - (Ar es)

a) Arthritis b) Carditis a) Coarctation ofaorta


c) Chorea d) Nodules
\ b) Pheochromocytoma
c) Renal parenchymatous disease
109. True about Rheumatic fever (PGI Deco3)
d) Drug induced
a) Chorea is aggravated during pregnancy
b) Chorea & arthritis co-existing I 19. A 10 year old boy is having hypertension. There is no
other significant history and urine analysis is cause
c) Subcutaneous nodules are tender
for his hlpertension - (Ar 10)
d) Erlthema multiforme seen
a) Chronicglomerulonephritis
110. Earliest vah'ular lesion in a case of acute rheumatic b) Polycystic kidney disease
fever is - (AttMS May 9a)
c) Refluxnephropathy
a) Mitral regurgitation (MR) d) Renal Parenchyrnal disease
b) Aortic Regurgitation(AR)
c) Mitral stenosis (MS) 120. Most common cause of persistent hlpertension in
child with intrinsic renal disease is -
d) Aortic Stenosis (AS)
(All India Dec15 Pattern)
1l l. Least common valve involved in rheumatic fever- a) CGN b) Chronic
Pyelonephritis d) Endocardial hypertrophy
c) Obstructive uropathy d) Renal tumor 130. A child with tetralogy of fallot uses which of the
12L. 2 year old child presented with sudden onset of following positions - @pG j0, ArrMS 87)
altered sesorium on examination Bp was 200/100 a)
- Supine b) Prone
(PGI Dec 04) c) Squatting d) Leaning forwards
a) Renal artery stenosis
131. A taussing - Bing malformation is best treated by
b) Coarctation of Aorta
_

c) Glomerulonephritis
d) Essential hypertension
a) Diversion of VS.D. rr-urun.orffi,ij.'ffi'
operation
e) Phaeochromocltoma
b) Diversion of the septal defect
c) Reattachment and riversal of aorta and pulmonary
MISCELLANEOUS arteries
d) No corrective procedure for this anomaly
122, Most common cause of acquired heart disease in 132, Systolic rnurmur in TOF is due to ? (APPG 08)
children- (All Inrlia Dec15 pattern) a) VSD b) pulmonary stenosis
a) Acute rheuamatic fever b) Kawasaki c) ASD d) none
c) Takayasu d) Diabetes 133. A two-year old boy presented with episodes of
123. Differential cyanosis is seen in- (All India Dec.t4 pattem) becoming dusky. On exarnination, there was central
a) TGV b) TAPVC cyanosis and clubbing. There was no pallor, oedema
c) PDA d) VSD or respiratory distress. The heart was normal sized
124. Differential cyanosis occurs in which disease - with a parasternal heave" A systolic thrill was pal_
(All India Dec15 Pattern) pable over the left middle sternal border. First heart
a) Severe coarctation b) Inturrupted aortic sound was normal and only the aortic component
arch was audible in the second heart sound. Liver was not
c) PPHN d) AII of above enlarged - (UPSC 07)

125. Cardiomyopathy is not a feature of - What would be the likely diagnosis -


(Ar 07)
a) Duchenne's Muscular Dystrophy a) Congenitalmethemoglobinemia
b) Friedriech's ataxia b) Eisenmengersyndrome
c) Pompe disease c) Aortic stenosis
d) Lowes syndrome d) Tetralogy ofFallot

126. Pulsatile varicose veins in lower lirnbs is seen in - L34. In which of the following differential cyanosis
found? (UPSC-r a8)
(ArrMS 01)
a) Klippel trenaunay syndrome a) VSD with reversal of shunt
b) TR b) PDA with reversal of shunt
c) RV failure c) ASD with reversal of shunt
d) Carcinoid stenosis of tricuspid d) Tetralogy ofFailot
127. -
Umbilical cr:lrd has (NEET Dec.12 pauern)
135. Ductus arteriosus closes in response to - (upsC_II 0B)

a) 1 vein and 2 arteries b) 2 vein and 2 arteries a) Decrease in peripheral oxygen saturation
c) I vein and I artery d) 2 veins and 1 artery b) Indomethacin therapy
c) Prostaglandin E1
d) Increase in pulmonary vascular resistance

Q['ESTIONS OF VARIOUS OTHER EXAMINA- 136. A new born presents with deepening cyanosis at
TIONS birth, with congestive heart failure and normal first
heart sound. X-ray reveals cardiornegaly diagnosis
128. Children born to mothers with systemic lupus S rs - (uP oB)

erlthematosis are likely to have one of the following a) Tetralogyoffallott


anornalies - b) Ebstein anomally
Karnataka 03)
a) Atrial septal defect c) Transposition ofgreat vessels
b) Tetralogyoffallot d) Tricuspid atresia
c) Transposition ofgreat vessels 137, "Egg on side appearan€e,, is seen in - Up 08)
d) Complete heart block a) Faliot's tetralogy
I29. Most cornmon type of atrial septal defect is - b) Transposition ofgreat vessels
(uP 07'
c) Ebsteins anomaly
06)
a) ostium primum d) Ventricular septal defect
b) Ostium secondum 138. A two-month-old infant is brought to the hospital
c) Endocardial cushion defect emergency with marked respiratory distress. On
examination, the infant has cyanosis and bilateral c) Pericarditis d) Rheumatic carditis
crepitations. Heart rate is 180/min, respiratory rate L42. Dose of digoxin in a child as mg/kg is - (upsc ss)
56/min and the liver span 7,5 cm. The child has had a) 0.02-0.04 b) 0.03-0.0s
repeated episodes of fever, cough and respiralory c) 0.04-0.06 d) 0.06-0.08
distress since the time of birth. Cardiovascular
examination reveals a grade III ejection systolic
143. Pure left sided failure may be seen with -

murmur in left parasternal area and the chest X- ray


a) ASD (ArrMS 78, PGI 81)

reveals cardiomegaly with a narrow base and pletho-


b) Aortic stenosis

ric lung fields. What is the most likely diagnosis ?


c) Patent ductus arteriosus
(UPSC-r 0e)
d) Pulmonary valvular obstruction
a) Congenitalmethemoglobinemia 144. Infantile myocarditis and pericarditis is due to - (TN
b) Transposition ofgreat arteries ee)

c) Cystic fibrosis a) Coxsackie A b) Coxsackie B


d) TetralogyofFallot c) Mumps d) Pox virus

139. A newborn baby develops cyanosis on daythree of I45. Cause of death in Acute rheumatic fever is- (up 08)
life. On auscultation, there is a systolic murmur. a) Pericarditis b) Myocarditis
Echocardiography reveals a cyanotic heart disease c) Endocarditis d) Streptococcal sepsis
in the baby. Which one of the following drugs can be 146. Which of the following manifestation of rheumatic
administered to prolong the life of the baby pending fever disappears completely? (Jipmer 11)
intervention - a) Carditis b) Arthritis
(uPSC,r oe) c) Chorea d) Subcutaneous nodules
a) Indomethacin b) Ibuprofen 147. The average B .P. of a I year old child is -(pcr 78, AMC 81)
c) Prostaglandin E, d) Propanolol a) t20180 b) 7sls0
140. The clinical features associated with coarctation of c) 95150 d) 60/30
aorta in older children are the following except - 148. The most common anomaly seen in the fetus of a
a) Upper body hypertension (UPSC-II 0s) mother taking lithum carbonate is - (Up 97)
b) Prominent pulsation in neck a) Cardiac deformities
c) Fatiguableness, tiredness in leg b) Neural tube defect
d) Absence of flow murmurs over scapular region c) Limb reduction
l4l, Commonest cause of enlarged cardiac shadow in d) Genitourinarydeformities
X-ray of a child is - (Karn.2oo0)
a) PDA b) CoarctationofAorta

IIT
ANSWERS
FETAT CIRCULATION

t. Ans. is 'b' i.e., umblical vein tRef o.p, Ghai }e/e p. 402 & fr/e p. 394; Nelson t1h/e p. 1s551
t Blood oxygenated in the placenta returns by way of the umblical vein, which enter the fetus at the umblicus and
course through to join the portal vein.
,,
Ans" is'd'i.e., Heart reeeives bloodwith htgh ffiyge& satur*tiolr lR$Netsen l#bfcp. t&SS; Gkad.pfep.3%,1
o Blood in IVC has more saturation than blood in SVC as IVC carries the oxygenated blood of umblical
vein.
r The left ventricular blood has more oxygen saturation that right ventricular blood because
it carries the blood of IVC,
while blood in right ventricle is a mixture ofblood from IVC and SVC.
r So' the brain and coronary circulatiorl receive blood with higher saturation (through ascending
aorta) than the lower
halfofbody.
r The pressure in iight and left ventricles are equal.
Ans. ie t'i.e., t0th dey fxcf ple p.395; Netrsoee r*fep. ]es6]
$n, Gk*t*fep.40J 6
3"
e In full term neonates the ductus arteriosus closes within l0 to 25 days. "

ETIOLOGY

4. Ans. ie 'c' i"e", Lithium [RcS FfiyumgKpark*fe p" 3]


e Teratogenic cardiovascular defects oflithium -+ Ebsteins anomaly, Tricuspid atresia.
5. Ans. is'c'i.e., Holt Oram Syndrome [Ref O"p. GheiBelep.40I &@/ep.399]
"The strongest
familial tendency is known in ASD associated with bony abnormalties (Holt-oram syndrome),,
6. Ans" is h'i.e., ASD [Ref Nelson l*te eh^ 426.tr]
-
7. Ans. is'a' l.a, ASn [Ref, Myang K p*rk *fe p. I Z; epW t*fc p. 53, 54]
r Cardiac anomalies.in Congenital rubella (In decreasing order of frequency) -+ pDA (most common), pS,
vsa AsD
r ASD is the best allswer amongst the given options because amongst the given options, ASD is least
commonly
associated with congential rubella.

NADA'S CRlTERIA

8. Ans. is 'C i.e., Asseesment of chitd for presemee of He*rt disease lkef 6h*6 */e p. 4M & ffile p. 39g\
The Assessment of a child for the presence or absence of heart disease can be done with
the help of some guidelines
suggested by NADA. These guidelines are called NADAs uiteria.

t
CLASSIFICATION OF CHDS

9. Ans, is 'c' i,e,, Tricuspid atresia lRel A,P, Ghai $th/e p. 404 6 Vh/e p, 410-4111

Gyanotic CHD

lncreased pulmonary blood flow Decreased pulmonary blood flow

Left ventricular or Both Right ventricular Both ventricle Left ventricle Right ventricle
hypertrophy hypertrophy
ventricu lar hypertrophy

o
Y
I

Persistant truncus o TGA


Hypertrophy
+ ++J
o TGA + PS o Tricuspid atresia o TOF
Hypertrophy

arteriosus o TAPVR o Persistent truncus o Pulmonary atresia o Eisenmenger's


o Single ventricle o Hypoplastic Lt. heart with
arteriosus with hypoplastic RV syndrome
r TGA+VSD hypoplastic o Ebstein's anomaly
pulmonary artery
r Single ventricle
with PS

Note : TAPVR -+ Total anomalous pulmonaryvenous return.

10. Ans, is 'l i.e., Tricuspid a*esi^{Ref: O"P. Ghai $th/e p. 422 b Vh/e p. 410-4111
r A11the given options are cyanotic heart diseases.
. But, amongst the given options, only tricuspid atresia causes left ventricular hlpertrophy.

11. Ans. is '* i,e,, Atrial $eptal defect {Ref. Myung K Park sth/e p. 75 d" A,p- Ghai |th/e p. 404, 41i dr 7h/e p, 4021

Acyanotic GHD

Increased pulmonary blood flow

Left ventricle or both Right ventricle Left ventricle Right ventricle


ventricle hypertrophy hypertrophy hypertrophy hypertrophy
+ + + +
o VSD r ASD (often RBBB) r AS orAR rPS
o PDA e PAPVR o COA (Coarctation r COA (in infants)
r ECD of aorta) oMS
oMR

t2" Ans. is '-p" i.e., Tricuspid atresia; 'c' i.e., Pulmonary atresia fRef: Reail helow)
o Normaily, the QRS axis ranges from -30o to + 1000.
r An axis more negative than -300 is referred to as left axis deviation.
r An axis more positive than + 1000 is referred to as right axis deviation.
Causes sJ left axis deviation (LAD)
o Left ventricular hlpertrophy
r Left anterior fascicular block or hemiblock.
r Inferior wall MI.
Causesof Bightaxisdeviation (RAD) q
r Right ventricle overload (right ventricular hypertrophy)
r Left posterior fascicular block.
o Lateral wall MI
r Dextrocardia
e Left pneumothorax
Now see the explanations of previous question and you can easily solve this question.
r Left ventricular hlpertrophy causes LAD, and CHDs causing LVH are : -
i) vsD lll) Endocardial cushion ASD v) Truncus arteriosus vii) Pulmonary atresia
li) PDA iv) COA (in older children) vl) Tricuspid atresia
"$a r d i ovi sc u I ar :Sy st e m

e Right ventricular hypertrophy causes RAD, and CHD,s causing RVH


i) TGA i11) TOF v) Eisenmenger\ syndrome
ii) TAPYR iv) ASD vt) COA (in infant)
About option h'
o All patients with VSD are born with right ventricular hypertrophy, as the puimonary
vascular resistance decreases
RVH regresses and there is LVH. Hemodynamics of VSD will be expiained in respective question.
So, at birth VSD may show right axis deviation.

lJ. Ans. is'b'i.e., T'ricuspid atresia, U'i"e., Eisenrnenger complex, ?'i.e., Tetralogy of Fallot
lRef: MywngKpark
Sthle p. 75; O,F. Ghai Bthle p. 404]

Congenital Heatt diseases

Cyanotic Acyanotic
o Tetrologyoffallot e Atrial septal defect
s Transposition of great vessels e Ventricular septal defect
. Tricuspid atresia t Patent ductus arteriosus
o Ebstein's anomaly t Partial anomalous puimonary venous return
. Pulmonary atresia a Coarctation of arota
t Hypoplastic left heart syndrome t AS orAR
e Double outlet right ventricle
I Totai anomalous pulmonary Venous
return with or without obstruction
't Truncus arteriosus
Eisenmenger's syndrome

t4. Ans. is'd'i.e., VSD tfief $.F" G&ea{ &,t'k p. 4t}4 S T*ls p. 4{}s}
Appearance of chest x-ray in a patient with cHD, depends on pulmonary blood flow
:
1) Plethoric lun&fields + If the pulmonary blood flow is increased, the iung f,elds
appear plethoric throughout and
in addition cardiomegaly is present.
2)oligenic lung fields -+ If the pulmonary blood flow is diminished there is paradoxical prominence
of hilar arteries
with clear or ischemic peripheral lung fields.

* Truncus arteriosus & TAPVR e VSD


s Single ventricle' , & HyperBlastic lt. heart * ,ASD
* IGA c PDA

(yaii6tl< '"r:,,: Aiy5nn r., ,.,...t

& Tricuspid atresia & As or AR


e Eisenmenger's syndrome & Coarctationrof'aorta
Q Ebstein's anomaly &. MR or MS
& Tiuncus arterioiui with hypoplastic pulmonary artery & PS

& Single ventricle with PS

I
ob'
15. Ans. is i.e., VSD [Rel O.P, Gkai Bth/e p. 414 dz Vhle p. 4A3l
o VSD is the commonest CHD.
t ySD is the commonest acyanotic CHD.

16. Ans. is ocn i.e", TOF ffi#i {}.P. Ghai 8,t,/e {}. ,j$:t, 420 * yt'le p. ,}08; Ntls*ya l{ttil/* y:.

a Tetralogy of fallot is the commonest cyanotic congenital heart disease.


Ans. is 'H i.e., Tetralogy of Fallot lRef : O.P. Ghai 8'h/e p. a06 dt 7h/e p. 4091
e Chest infections are frequent in the following two groups of congenital cardiac anomalies.
a) Left to right shunts -+ This group includes:
s Atrial septal defect
t Ventricular septal defect
t Patent ductus arteriosus
r This group of CHD's are characterized by frequent chest infections.
r There is large flow of blood in the lungs which causes lung infections.
o Each attack of cold seerns to turn into Bronchopneumonia.
o It is not uncommon for these patients to have 6-8 attacks of pneumonia in the first year of life.
b) Cyanosis with increased pulmonary blood flow.
r A large no. of conditions result in a combination of cyanosis with increased pulmonary blood flow.
r Common to these conditions is the presence of abnormal mixing of pulmonary venous blood with the systemic
venous blood and absence of obstruction to pulmonary blood flow.
o The anomalies included in this gro'.ip are :
t Transposition of great vessels
t Total anomalous pulmonary venous connection
t Single ventricle without obstruction to pulmonary blood flow.
t Persistent truncus arteriosus
t Tricuspid atresia with absence of obstruction to pulmonary blood flow.
s Double outlet right ventricle without pulmonary stenosis.

r Clinically these patients are characterizedby


r Cyanosis
r Cardiomegaly
r Almost 80% of these patients are lost due to congestive cardiac failure or puimonary infection.

Congenital heart diseases which are not predisposed to pulmonary infection


1)Right to left shunts 2)Obstructive lesions
This group includes This group includes
o Tetralogy of Fallot o Aortic stenosis
c Ebstein's anamoly o Coarctation ofAorta
Characteristic of this group is c Pulmonary stenosis
: Presence of cyanosis Clinical characteristic of this group
: Absence of chest infections t Absence offrequent chest infections and cyanosis
s Absence of precordial bulge
t Presence of forcible or heaving cardiac impulse.

r8. Ans. is 'b' i.e., ASD [Ry'; O.P. Ghai 8th/e p. 413 & 7h/e p. 402)
o ASD is associated with an enlarged right atrium 6 right tentricle to accommodate large volume of blood. The left
atrium is not enlarged because it decompresses itself by shunting blood to the right atrium at a minor difference
in pressure. The left atrium may enlarge once Eisenmenger's syndrome develops and a reversal of shunt is seen
across the defect.
r All congenital heart diseases given in option cause left to riSht shunt.
Size of left atrium in left to right shunts
o In left to right shunt, blood enters from left side ofheart (Lt. atrium, Ltventricle or aorta) to right side ofheart (Rt
atrium, Rt ventricle or pulmonary artery) depending on the site of abnormal communication.
o This blood then flows through pulmonary circulation and again enters the left atrium.
o The path ofblood is then depending on the defect.
r In all these defects, left atrium recieves larger volume of blood which comes through pulmonary circulalion -->
Blood from right side of heart plus some blood fr3)m left side of heart because of shunt enter into pulmonary
circulation and then into left atrium. So left atrium recieye blood volume larger than normal.
o But the effect of larger volume on left atrium will be different in ASD and other left to right shunts.
1) InASD
r Left atrium recieves extra volume of blood, but fortunate for the Ieft atrium, it also has an extra exit for blood,
i.e., septal defect.
.{rrs. is 'b' i.e., Tetralogy of Fallot lRef : O.P. Ghai 8'h/e p. a06 dz Vh/e p. a09l
o Chest infections are frequent in the following twc groups of congenital cardiac anomalies.
a) Left to right shunts + This group includes:
t Atrial septal ilefect
s Ventricular septal defect
t Patent ductus artefiosus
o This group of CHD's are characterizedby frequent chest infections.
r There is large flow of tllood in the lungs which causes lung infections'
o Each attack of cold seerns to turn into Bronchopneumonia.
r It is not uncommon for these patients to have 6-8 attacks of pneumonia in the first year of life.
b) Cyanosis with increased pulmonaryblood flow.
o A large no. of conditions result in a combination of cyanosis with increased pulmonary blood flow.
o Cornmon to these conditions is the presence of abnormal mixing of pulmonary venous blood with the systemic
venous blood and absence of obstruction to pulmonary blood flow.
o The anomalies included in this group are :

t Transposition of great vessels


t Tatal anomalous pulmonary velrotts connection
t Single ventricle without obstruction to pulmonary blood Jlow.
t Persistent truncus arteriosus
t Tricuspid atresia with absence of obstruction to pulmonary blood flow.
t Double outlet right ventricle without pulmonary stenosis.
o Clinically these patients are characterizedby
r Cyanosis
r Cardiomegaly
r Almost 80% of these patients are lost due to congestive cardiac failure or puimonary infection.

Congenital heart diseases which are not predisposed to pulmonary infection


l)Right to left shunts 2)Obstructive lesions
This group includes This group includes
o Tetralogy of Fallot o Aortic stenosis
anamoly
o Ebstein's o Coarctation ofAorta
Characteristic ofthis grouP is o Pulmonary stenosis
t Presence of cyanosis Clinical characteristic of this group
t Absence of chest infections s Absence offrequent chest infections and cyanosis
t Ab sence of precordial bulge
t Presence of forcible or heaving cardiac impulse.
18. Ans. is 'b' i.e., ASD [Rel: O.P. Ghai 8'h/e p. 41j dt 7h/e p. 4021
o ASD is associated with an enlarged right atrium 6 right ventricle to accommodate large volume of blood. The left
atrium is not enlarged because it decompresses itself by shunting blooil to the right atrium at a minor difference
in pressure. The left atrium may enlarge once Eisenmenger's syndrome develops and a reversal of shunt is seen
across the defect.
o All congenital heart diseases given in option cause left to right shunt.
Size of left atrium in left to right shunts
o In left to right shunt, blood enters from left side ofheart (Lt. atrium, Lt ventricle or aorta) to right side ofheart (Rt
I
atrium, Rt ventricle or pulmonary artery) depending on the site of abnormai communication.
r This blood then flows through pulmonary circulation and again enters the left atrium.
o The path ofblood is then depending on the defect.
o In all these defects, left atrium recieves iarger voh.rme of blood which comes through pulmonary circulation ->
Blood from right side of heart plus some blood frpm left side of heart because of shunt enter into pulmonary
circulation and then into Ieft atrium. So left atrium reciete blood volume larger than normal.
o But the effect of larger volume on left atrium will be different in ASD and other ieft to right shunts.

1) ItASD
r Left atrium recieves extra volume of blood, but fortunate for the left atrium, it also has an extra exit for blood'
i.e., septal defect.
ilriiltlr: !ii...: :r'::if '

r so, larger volume of blood is decompressed through left atrium.


i) Amount, which is normally exits through AV valve (mitral valve) into
left ventricle.
ii) Extra amount, which left atrium recieves, exits through septal defect into
right atrium.
2) Other lefi to right shunts (VSD, pDA, Aortopulmonary window)
rIn these defects also, left atrium recieves extra volurne of blood, But there
is no extra exit.
rSo, left atrium can pumP out only normal amount of blood through
mitral valve, and extra volume remains in the
left atrium -+ Volume over load -+ Left atrial enlargement.

19. Ans. is 'H i.e., Fallot's Tetralogy lRef O.p. Ghai Bthle p. 420 6 Vh/e p. 4081
o Cardiomegaly and CHF do not occur in Fallott tetrology.
No Cardiomegaly
o First you should know the following facts : _
i) Pressure overload to ventricles causes concentric hlpertrophy without dilatation.
ii)Volume overload to ventricles causes hlpertrophy with dilatation
-> Eccentric hypertrophy.
o Due to pulmonary stenosis, there is pressure overload to right ventricle
-> Coicentric hipertrophy of right
ventricle without dilatation -+ No Cardiomegaly.
No CHF
o The VSD of ToF is always large enough to allow free exit to the
right to left shunt.
o Since the right ventricle is effectively decompressed by the VSD,
C-HF never occurs in TOF.

ATRIAL SEPTAL DEFECT

20. Ans. is t' i.e., vsD IR4 clinical recognition of congenital heart diseasep. %al
o Lutembacher syndrome is defined as a combination of mitral stenosis
and a left-to-right shunt at the atrial level.
t Typically, the lefi-to-right shunt is an atrial septal defect (ASD) of the ostium
,irundul* variety.
2r. Ans. is t'i.e., Osteum secondum fRef: Nelson lgth/e ch.42.11
o Osteum secundum is the most common type of ASD.
22. Ans. is 'a' i.e., Ostium primum tRef Ghai Vh/e p. 61j, a01)
o Most common type of ASD and most common cHD in Down syndrome
) osteum primum (endocardial cushion
defect).

23. Ans. is'b' i.e., Left atrial hypertrophy lRef: o.p. Ghai 8,h/e p. 413 6 vh/eq. 402; Nelson
rrth/e p. 18g3,r8s41
o In ASD there is abnormal communication belween left and right atria
which causes left to right shunt because
pressure in left atrium is higher (3-5 mm Hg) than right atrial pr.rrrrr".
o This results in : -
1) Volume overload to right atrium -+ Dilatation and hypertrophy of right atrium.
2) Volume overlead to right yentricle -+ Dilatation and hypertrophy of right ventricle.
3) Increased blood Jlow thro.ugh lungs -+ Pulmonary plethora which can cause pulmonary hypertension.

24. Ans. is 'a' i.e., small [ne7 o.p. Ghai Bth/e p. 4r3 6 Th/e p. 402; Nelson rSth/e p: 18841
Chest x-ray in ASD
o Mild to moderate cardiomegaly . Prominent pulmonary artery segement.
o Right atrial and right ventricular enlargement. o Relatively small
aortic shadow
o Plethoric lungfields.

25. Ans. is t'i.e., ostium primum atrial septal defect fRef: o.p. Ghai B*/e p. 41j dz Vh/e p. 402-40j; Nelson lgth/e p.
1883, 1884)
o wide fixed splitting o/ s, in association with ejection systolic murmur suggests the diagnosis of Atrial septal
defect.
o Ideally with hypertrophy of the right side of the heart, right axis deviation
should be present, but sometimes left
axis deviation is also seen in A.S.D.

Explanation for the presence of left axis deviation in A.S.D.


r Atrial septal defects are of two types
0
t Ostium primum atrial septal defect
t Ostium secundum atrial septal defect
o This division depends upon the ilevelopment of interatrial septum and related structures.
o Ostium secundum type atrial septal defect is the classical atrial septum defect.
o Presence of interatrial septum is the only developmental anomaly in ostium secundum type and associated with
right axis deviation.
o In contrast the ostium primum defects are characterized by presence of certain other developmental abnormalities,
besides the interatrial sePtum.
o Once such abnormality is the malfurmation or lack of development of the anterosuperior division of the lefi
bundle branch.

deviation.

-{r Ans. is'd i.e., Infective endocarditis


o ASD is usually subtle and not having significant problem except mild growth disturbance.
r Ostium secundun ASD is M.C. tipe of ASD.
o Ostium secundum ASD usually associated with mitral valve prolopse or stenosis (Lutembachert Syndrome).
o Complication usually develop in 4'hdecade and include :
1) Pulmonaryhypertension
2) Rt. sided heart failure
3) Stroke
4) Eisenmenger'ssyndrome
r Infective endocarditis is very rare in ASD and not require any antibiotic prophylaxis.

Ans. is'a' i.e., ASD iRel Tertbook of Paediatrics - by Desai p. 5181

o Hilar dance sign refers to vigorous pulmonary arterial pulsations due to increased blood flow, often seen fluoroscopicaily
in patients with congenital left-to-right shunts, especially atrial septal defects.

VENTRICULAR SEPTAL DEFECT

28. Ans. is 'd i.e., Membranou s lRef: Qhai Vh/e p. 40 j; Nelson 18th /e p. -1888, 18891

o VSD may be lecated at : (t) Membranous part of ventricular septum (most common -+ 90o/o), (ii) Muscular septum,
and (iii) May be multiple.
29. Ans. is t' i.e., Reverse spliting of S, [Rel Nelson 18th/e p. 1888, 18891

o There is wide spliting of S, (not reverse spliting)'


r Here, I would like to meniion two commonly used terms about S, : Wide spliting, and (ii) Reverse spliting.
r Normally, S, has two components : (i) Aortic component (Ar) due to closure of aortic valve; and (ii) Pulmonic
component (Pr) due to closure of pulmonary valve.
r Aortic valve closes slightly earlier than pulmonary valve. Thus there is minute split betweeq these two component
ofS,
r It this spliting gap between A, and P, is increased, it is caled wide spliting. It may be due to early closure of A, or
late closure of P, or both.
r When, spliting order is reverse, ie.e. P, occurs before Ar, it is called reverse spliting. It may occur due to delayed
closure of A, or early closure of P, or both.

30. Ans. is'b- i.e., Vascular changes in pulmonary circulationlRef: Nelson 18th/e p. 1888, 18891
o A patient with large VSD develops CHF because of the large left to right shunt. Large amount of blood at high
systolic pressure is entering into the pulmonary circulation and with lhis continued exposure of the pulmonary
vascular bed to high systolic pressure and high llow pulmonary vascular obstructive disease develops.
r The pulmonary vascular resistance gradually rises and becomes equal to the systemic resistance. When this
happens the shunt becomes bi-directional and the signs of heart failure diminish but the patient becomes cyanotic
( Ei s e nm e n e n gerqthy si ol o gy ) .

31. Ans. is t' i.e., i.e., VSD lfuef: O.P. Ghai 9th/e p. 415 6 7tu/e p. 403-404; Nelson 18th/e p. 1888, 18891

o The answer is quite obvious.


r TOF and TGA are cyanotic heart diseases and
r Rheumatic fever can be ruled out on basis of age. (Age group of rheumatic fever is 5-15 yrs.)
Ans. is t'i.e., i.e., subacute bacterial endocarditisfRef: o.p. Ghai 8th/e p. 415 6 Th/e p. 4051
r Clubbing in a patient with VSD suggest infective endocarditis.
JJ. Ans. is'd'i.e., i.e., Pulmonary oligemia in chest x-ray fRef: O.P. Ghai 8th/e p. 415, 416 6 Thle p. 40i-404; Nelson
ISth/e p. t8B9l
c Pulmonary vasculature is increased -> Pulmonary plethora (not oligemia).
34. Ans is 'a' i.e., Enlarged Left Atrium lRef: O.P. Ghai 8th/e p. 416 6 7/e p. 403, 404, 4067
o Lefi atrium is not enlarged in an ASD in contrast fo VSD where thelefi atrium is typically enlarged.

Enlorgement (Mld/*od:ratg.cardlomgqatr) (Grosl yrjjo.mesat/).


. ." ".
Flrffi6+#i, Ai.,t€iv.+.:'.,.:, 1:.'-t Enlarged lntq1ged t :.'"r.:.::, ,:iil :i'.1 ..i.,:i*#+tge*,{ ,€-'
Pulmonary Present Present Present
Vasculature
el-"l,no*)
, ll'lt"""1,tv .,,,,., .
Aorta N &<if :fu s irsA&..' "",
, .

PATENT DUCTUS ARTERIOSUS

35. Ans. is'b'i.e., i.e., Aorta and pulmonary artery lRel O.F. Ghai 9't'/e p. 117 (z 7h/e p. 405; Nelson 18'h/e p. 18911
c Patent ductus arteriosus is a communication between the pulmonary artery and aorta,
36. Ans. is'd' i.e., lVlore comrnon in term baby [Ref Nelson 18th/e ch. 426)
o It is a common problem in premature infants, where it can cause severe hemodynamic derangements and several major
sequelae
37. Ans. is "l i.e., Symptoms similarto aortopulmonarywindowlRef.Nelson 18th/e p. 1891, 1893; O.P. Ghai 8'h/e
p. 419 6 6th/e p. 4A4, 4A61
o Before comparing the symptoms of PDA dr Aortopulmonary window let me discuss the C/F of PDA.
o Patent ductus arteriosus is a communication between pulmonary arters' and Aorta after fetal life"
r PD.A, results in left to right shunt from the aorta to the pulmonary artery. The flow occ*rs brLth
during systole and diastole as a pressure gradient is present throughout the cardiac cycle between the 2 grcat
arteries. This flow of blood results in continuous murmur it is usually heard at 2'd left interspace.
r In patients u.'ith large left to right shunt a low pitched mitral mid diastolic murmur is audible at the
apex, owing to the increased volume of blood flow across the mitral valve.
o There are other conditions which can produce systolic and diastolic murmur in the pulmonary area such as
Aortop ulmo nar y w in dow de.fect.
o In Aortopulmonary window defect there is communication between the ascending aorta and main pulmonary
artery.Usually the defect is large therefore the pressure between the two great arteries tends to equalize and flolv is
established only during systole when left ventricular contraction increases the aortic pressure. So the murmur in
Aortopulmonary window defect is systolic with mid diastolic rurnble due to increased blood flow through mitral
valve.
t
Sometimes when the Aortopulmonary window defect is small, there is continwaus btood JTow fram Aorta to
pulmonary artery resulling in continuous murmur. trn this condition symptoms of Aorticopulmofiary windaw
drtct mrmics the sygnptoms of PDA lContinuous murmur, wide pulse pressure,mid diasiolic murmur).It is very l

dilficult to distingui3h between the 2 condition.


o Chances of spontaneous closure in PDA in term infant is rare (chances of spontaneous closure of PDA is more in
preterm infants.)
o If diagnosis of PDA can be made within the first - two weeks, indomethacin can be given to close the defect. (Bal
since the child is one ye6r old lndomethacin is of no help).
r In PDA there is increased risk of infective endocarditis.
Ans' is t' i.e., i.e., subacute bacterial endocarditi s
[Ref: o.p. Ghai 8,h/e p. 415 6 vh/e p. 405]
o Clubbing in a patient with VSD suggest infective endocarditis.
JJ. Ans' is 'd'i'e', i'e" Pulmonary oligemia in chest x-ray
I8'h /e p. 1 8B9l
lRef o.p. Ghai 8h/e p. 415, 416 6 Vh/e p. 403-404; Nelson

e Pulmonary vasculature is increasecl -+ purmonary prethora(not


oiigemia).
34. Ans is 'a' i.e., Enlarged Left Atriurn fRef: o.p. Ghai Bth/e p. p/e
416 6 p. 40i, 404, 406]
t Left atrium is not enlarged in an ASD in camtrast to vsD
where the left atrium is typicaily enlarged.

Ch.ambar:. :':,, .r:: .RA+ftVr:,, ,r. ,":r: ' ,,'r IAJ IY-*Y "'--. ,,
,W.+'LA+,Rt/
Enlargement lw d / m od rate ca rd i o m eg a !y)
( t I e (Gross cartiamegaly)
Pglnronary {rtery,,,, Enlarged Enfarged' ,':
El:alrged"
Pulmonary Present Present Present
Vasculature
(Pulmonary Plethora)
Aoirta,,' ' . ::'
Normal (or small)
Enlarye4 ,'

PATENT DUCTUS ARTERIOSUS

35. Ans' is 'b' i'e', i'e" Aorta and pulmon ary artery
[Ilef: a.p. Ghai gil,/e p. 4x7 6 7th/e p. 405; blelson lg,h/e p. 1g9tr)
o Patent ductus arteriosus is a communication between the pulmonary artery and
aarta,
36. Ans. is 3'i.e., &{ore cornrnon in term baby
t.el Nelson XEh/e ch. 426)
problem in premature infants, where it can cause severe
" :H;.ilI-on hemodynamic derangements and several major

37. Ans' is'a'i'e', symptoml similar to aortopulmonarywindo


w fRef; Nelson lgth/e p. lggl, 1gg3; o.p Ghai gth/e
p. 419 6 6th/e p. 4A4, 40d)
c Before cotnparing the s1tmp\sa5 of PDA 6 Aortopulntonary windaw let nle discuss
ti.re c/F ,f pDA.
o Fatent ductus arteriosus is a communication bitween pui.*orrory
,rtrry oi,d aorr, -rr-.'r# to..
I PD'A' results in left to right shunt from the aorta
to the plrlrnonary artery. The ftrow occr",rs bath
during systole and diastole as a pressure gradient is present
throughout the cardiac cycle beiwee'i the 2 grcat
arteries' This flow- of blood results in continuous murmur
it is usually heard at 2"d left interspace.
r In patients u'ith large left to right shunt a low pitched mitral miil
eiiastolic murmur is audibie at the
apex, owing to the increased volume of blood flow across
the mitral valve.
o Th"ere are other conditions which can prod.uce systolic
and cliastolic murmur in the pulmonary area such as
Ao r t o p ulrn a n ar y w in dow d efect.
.},,ffi*:l.':::-municationbetweerttheascendingaortaandmainpulmonar"1
rhe pressure between the two g..u, u,i",1",?.,J,
:3:1,:"y::l1lj,.,Li-:f::liig",l*.1":: ," .q""i,
"/ "'--'
:.:::l:.1,.j:1,'*ll,,y,;;:;;;;;";;;;;;;;.;;;;".*[i]:;""""ifJT:fi#::J,::t]ff.T1ffi:
Ao,rtoPulmonary window defect is systolic r,vith mid diastolic
rumble due to
YrrjlrlLur4r LUuLrd.Llluil trlcreases tne aorttc pressure,
the murmur in So
increasetl blood flow through rnitrai
valve.
t sometimes when the Aortopl\le1e\ary window defect is
small, there is continuous blocd
pulmonary artery resulting in continuous murturur. trn Jl'w frorn Aorta ,o
this condition symptoms <tf Aortic,pulmonary window
defect mlmics the symltoms of PDA (contir ruous murmur,
wicle pulse pi.esiure,mirt
diastolic mr.rmur) .It is very
cliIficult to distinguish between the 2 conditic.n.
o chances of spontaneous closure in PDA in term inf'ant
is rare (chances of spontaneous closure of pDA is
preterm infants.) more in
o If diagnosis of PDA can be made within the first - two
weeks, indomethacin can be given to close the
since the child is one r-ear old Indomethacin is of na hetp). defect. (Eul
r In PDA there is increased risk of infective endocarditis"
32. Ans. is t' i.e., i.e., Subacute bacterial endocarditi s
[Ref: O.p. Ghai 8'h/e p. 415 6 Vh/e p. 4A5)
o Clubbing in a patient with VSD suggest infectlve endocarditis.
JJ. Ans. is 'd'i.e., i.e., pulmonary oligemia in chest x_ray
[Ref O.P. Ghai 8th/e p. 415, 416 dz Vh/e p. 403_404; Nelson
lSth/e p, 1889)
o Pulmonary vasculature is increased -+ purmonary prethora(not
oligemia).
34. Ans is 'd i.e., Enlarged Left Atrium {Ref o.p. Ghai 8th/e p. 416
6 Thre p. 403, 404, 4a6)
o Le{t atrium is not enlarged in an ASD in contrast to vsD where the left atrium is typicaily enlarged.

tll diibei| ::...,,..; :...:. '.&Ai+:.Rll,:i'.:r ....:.-.:''r'. -':::r'


.,.:tA +,,lry,r'flV,::r:,.. .-,'. :i.::':'LU.::i' t*.ii *V,
Enlargement (M ild/mod e rate ca rd io mega ly) (Gross cartiomegaly)
P,uhbr'eii*ii& Enlarged Enbiged.
Pulmonary Present Present
Vasculature
ptethora)
!Puimongry
Ao.rta,..:,'" ...:..r'.:':' ..,::l' Normal (or sma!l) ffo.r.,-r11gff,or:srtr a:l$ . ;'',,,.. ;'Ei iloIn*6'.

PATENT DUCTUS ARTERIOSUS

Ans. is'b'i.e", i.e', Aorta and pulmonary artery


[Re! a.p. Ghsi g,t,/e p. 417 d* vh/e p. rts; Nerson rP*/e p. 1891]
c Patent ductus arteriosus t's a comruunication between the pulmonary
artery and aorta.
36" Ans. is'd'i.e., More common in term baby
fRef: Nelson lg,h/e ch. 426]
o It is a common problem in premature infants, where it
can cause severe hemodynamic derangements and
sequeiae several rnajor

37. Ans' is'a'i'e" symptoms similar to aortopulmonarywindo w fRef; Nelson lg,h/e p. 1gg1,
1g%; a.p Ghai gth/e
p. 419 & 6'h/e p. 404, 4061
o Before comparing the syntptoms of PDA 6 Aortopuhnonary
window let rlre discwss the C/F of pDA.
o Patent ductus arteriosus is a communication bitween pui*orory
artery and Aorta afterfetal life.
r PDA" results in left to right shunt from the aorta to the pulmonary artery. The iforr occurs br:th
during systole and diastole as a pressure gradient is present
throughout the cardiac cycle betneen the 2 great
arteries' This flow of blood results in continuous murrnur
it is usuaily heard at 2,d left interspace.
r In patients with large left to right shunt a low pitched mitral mi<l
diastolic murmur is audible at the
apex, owing to the increased volume of brood flow across
the mitrar val.i,e.
o There are other conditions which can produce systolic
and diastolic murmt4r in the pulmonary area suclt as
Aor to p ulmo n ar y w indow defe ct.
r In Aortopulmonary window defect there is comtnunication between the ascending
aorta and mo.in pulmonarT
artery' usually the defect is large therefore the pressure between
the two great arteries tends to equalize and flow is
established only during systole when left ventricular contraction
increases the aortic pressure. so the murmur in
Aortopulmonary window defect is systolic with mid diastolic
rumble clue to increa.*,l blood flow through mitrai

t sometimes when the Aortapulruonary window defect is


small, there is continwous blood
pulmonary artery resulting in continuous murtnur. In JTow .from Aorta ta
this condition symptoms of Aorticopulmonary window
defect mimics the symftoms of PDA (Cont.inuous mtffmur,
wi de pulse p'resiure,mid diastolic rnurmur) .It
difficult to distinguish between the 2 condition. is very
r chances of spontaneous closure in PDA in term infant
is rare (chances of spontaneous closure of pDA is
preterm inlants.) more in
o If diagnosis of PDA can be made within the first -
two weeks, indomethacin can be gir.en to close the
since the child is one year old Indomethacin is of no help). defect. (Eul
o In PDA there is increased risk ofinfective endocarditis.
Spontaneous closure of PDA
1) In premature infank
infants pDA is due
'::.'J.il:ffi;:rd;I?H.'.:"i,T::::,*"TJ,::ffiff:".," in
'lhese 'lo

2) In Full term infants


I Uniike that in prematute infants, spontaneous closure of a PDA does not usually occur in full term infants.
r This is because the PDA in term infants results from a structural abnormaliry o[the ductal smooth muscle.

38. Ans. is 'a'i.e., i.e., CCF lRef: MyungKPark Sth/e p. 176)


o Most common cause of death in PDA is CCF.
o Second most common cause of death is infective endocarditis.
39. Ans. is'a'i.e., CO, washout[Ref : Nelson 18th/e p. 795)
PDA in pleterm neonate
o In pre-vious explanations I have explained that chances of spontaneous closure of PDA are higher in premature
infants as there is no structural abnormality. However, you should keep in mind that this fact is true when we are
comparing the spontaneous closure of PDA betrveen term and pre-term neonate. Overall, the percentage of
spontaneous closure of PDA in preterm neonate is very low. To avoid confusion, I am explaining following facts in
brief :-
1) Normal neonate:- Ductus arteriosus closes functionaily within about 15 hours of birth.
2) PDA in term (mature) neonate:- The persistence of ductus arteriosus beyond 24 hours after birth is considered
as PDA in term neonate. Spontaneous closure of PDA does not usually occur because PDA resuits from
structural abnormality.
3) PDA in pre-term (pre-mature) neonate:- The ductus arteriosus in pre-term neonate is not as responsive to
increased oxygen content as it is in term neonate. However, there is no structural abnormality.
i) Chances of spontaneous closure are very less (compared to normal term neonate).
ii)Chances of spontaneous closure are more than the term-neonate with PDA. However, in premature neonate,
where hlpoxic pulmonary disease is commonplace, the ductus being sensitive to hypoxia, remains open.
o So, it is very unlikely that spontaneous closure of PDA will occur in pre-term neonate. But chances are more when
compared to term neonate with PDA (not normal term neonate in which ductus arteriosus closes within 15 hours of
birth).

Manifestations of PDA in pre-term neonate


1) Apnea for unexplained reasons in an infant recovering from RDS;
2) A hlperdynamic precorditm, bounding peripheral pulses, wide pulse pressure, and a continuous or systoiic
murmur with or without extension into diastoie or an apical diastolic murmu! multiple clicks resembling the
. shaking ofdice;
3) Carbon dioxide retention;
4) Increasing oxygen dependence;
5) X-ray evidence of cardlomegaly and increased pulmonary uascular markings; and
6) Hepatomegaly I
r Increased pulmonary blood flow and compromised ventricular function accompanying dropping pulmonary resistance
in the setting of a PDA is a significant risk factor for pulmonary hemorrhage.
o The greatest risk factor for NEC is prematurity. Any etiology which leads on to hemodynamic or hlpoxic insult
predisposes to NEC. PDA is common in preterm infants and it definitely causes hemodynamic stress and is a risk
factor for development of NEC.

40. Ans. is 'B i,e. Prostaglandin inhibitors {Ref: O.P. Ghai 8th/e p. 419 & Th/e p. 407)
Management of PDA
r After birth ductus arieriosus constricts within a few hours producing functional closure.
o Permanent anatomic closure follows in the next 10-21 days due to extensive intimal thickening.
o The mechanism producing the initial constriction is not completely understood, but the increase in arterial O,
tension plays an important role.
r One more factor which helps in closure of the ductus arteriosus is the decrease in concentration of prostaglandins
at the time of birth
r In the intrauterine period the concentration of prostaglandin PGF2 a is very high. This prostaglandin has a
vasodilatory effect which keeps the ductus open.
Spontaneous closure of PDA
l) In premature infants
infants pDA is due
'::ffi::ff:#:ilx*llHj:"ifflT:*#J,;:ffi,T:".* in
'lhese '{o

Z) InFullterminfants
r llnlikethatinprematureinfants,spontaneousclosureofaPDAdoesnotusuallyoccurinfullterminfants.
r This is because the PflA irr term iafants results from a structural abnormality of the ductai smooth muscle.

Ans. is 'a'i.e., i.e., CCF fRef: MyungKPark 5'h/e p. 1761


o Most common cause of death in PDA is CCF.
o Second most common cause of death is infective endocarditis.
i9. Ans. is 'a' i.e., CO, washout {Ref : Nelson 18th/e p. 795)
PDA in preterm neonate
o In previous explanations I have explained that chances of spontaneous closure of PDA are higher in premature
infants as there is no structural abnormality. However, you should keep in mind that this fact is true when we are
comparing the spontaneous closure of PDA betrveen term and pre-term neonate. Overall, the percentage of
spontaneous closure of PDA in preterm neonate is very low. To avoid confusion, I am explaining following facts in
brief :-
1) Normal neonate.'- Ductus arteriosus closes functionaily within about 15 hours of birth.
2) FDA in term (mature) neonate :- The persistence of ductus arteriosus beyond 24 hours after birth is considered
as PDA in term neonate. Spontaneous closure of PDA does not usually occur because PDA results from
structural abnormality.
3) PDA in pre-term (pre-mature) neonate:- The ductus arteriosus in pre-term neonate is not as responsive to
increased oxygen content as it is in term neonate. However, there is no structural abnormality.
i) Chances of spontaneous closure are very less (compared to normal term neonate).
ii)Chances of spontaneous closure are more than the term*neonate with PDA. However, in premature neonate,
where hypoxic pulmonary disease is commonpiace, the ductus being sensitive to hypoxia, remains open.
r So,it is very unlikely that spontaneous closure of PDA will occur in pre-term neonate. But chances are more when
compared to term neonate with PDA (not normal term neonate in which ductus arteriosus closes within 15 hours of
birth).

Manifestations of PDA in pre-term neonate


1) Apnea for unexplained reasons in an infant recovering from RDS;
2) A hlperdynamic precordium, bounding peripheral pulses, wide pulse pressure, and a continuous or systolic
murmur with or without extension into diastole or an apical diastolic murmur, multiple clicks resembling the
. shaking ofdice;
3) Carbon dioxide retention;
4) Increasing oxygen dependence;
5) X-ray evidence of cardiomegaly and increased pulrnonary uascular markings; and
6) Hepatomegaly I
r Increased pulmonary blood flow and compromised ventricular function accompanying dropping pulrnonary resistance
in the setting of a PDA is a significant risk factor for pulmonary hemorrhage.
o The greatest risk factor for NEC is prematurity. Any etiology which leads on to hemodynamic or hlpoxic insult
predisposes to NEC. PDA is common in preterm infants and it definitely causes hemodynamic stress and is a risk
factor for development of NEC.

40. Ans. is 'd' i.e. Prostaglandin inhibitors fRef: O.P. Ghai 8*/e p. 419 & Vh/e p. 407)

Management of PDA
o After birth ductus arieriosus constricts within a few hours producing functional closure.
. Permanent anatomic closure follows in the next 10-21 days due to extensive intimal thickening.
r The mechanism producing the initial constriction is not completely understood, but the increase in arterial O,
tension plays an important role.
r One more factor which helps in closure of the ductus arteriosus is the decrease in concentration of prostaglandins
at the time of birth
r In the intrauterine period the concentration of prostaglandin PGF2 a is very high. This prostaglandin has a
vasodilatory effect lvhich keeps the ductus open.
o The synthesis of these vasodilator is inhibited at birth
by the inhibition of the enzymecycloxygenase at birth.
o cycloxygenase is an important enzymein the synthetic
pathway oif.ortugrurai.r.
o In many premature infants the ductus fail to ciose
.porrt"n"oorly. in th... cases the closure can be produced
infusion of drugs that inhibit cyclooxygenase. br

the enzyme cycloo\ygenase.

o If indomethacin is not successful and the ductus remains


hemodynamicaliy significant, surgical ligation shoulc
be performed without hesitation.
o Recent studies from Europe indicate that ibuprofen
may be as efective as indomethacin for the medical closure
preterm PDA. oi

41. Ans. is 'd i.e., Prostaglandin E, [Rel O.p. Ghai Bh/e p. 419 & Vh/e p. 407]
o Ductus arteriosus rernains patent in the uterus in
the fetal life because of high level of vasodilators especialll
prostaglandin E, in the blood.
o Aftel birth the prostaglandin synthesis is inhibited
by inhibition of iycloxygenase at birth.
o So if the patency of ductus arteriosus has to be mainiained
-+ pririrgtoniins
r The drug which is specifically used for this is prostaglanain ni (Misoprost, should be given.
Rioprostil)
o If the ductus arteriousus has to be closed, prorloglonii,
tyntt nii innibitor - Indomethacinshould be given.
42. Ans. is'b'i.e., Misoprost [Ref Nelson l#hie ch. 4j0J
o This question is framed improperiy.

' iltifffLlll:l*il:t are not used in patent ductus arteriosus. In pDA, prostagtandin
antagonists (NSArDs e.g.
o Prostaglandin analogues are used to maintain the patency
of ductus arteriosus in certain cyanotic congenitai heart
diseases (ductus arteriosus dependant CHDs).
o Pro_staglandinX, is responsible for keeping the
ductus patent.
o PGE-1 used to keep open duct are Alprostadil
or misoprostol.
43. Ans' is b' i'e', It is more common in males than female s
lRef: Nekon rth/e p. 1g59, 1879, IBSI;
PDA is more common infemales thqn males - Nelson CpDT lgth/e p. 560, 561)

M. A.ns. is 1d'i.e., All of above tRel O.p Ghai Pth/e p. at9 6 fblcp. aiA6l
o Endocardial valvulitis (infective endocarditis), cHF
and Eisenmenger syndrome, all can develop in pDA.

FALLOT'S TETRALOGY

45. Ans. is t'i.e., ASD iRef A,.p; Ghai thle p. 420 & Zele p. 4},8;Nelsoa l8*1c,p. 19e7, WbSI
o Constituents of TOF
0
t Ventricular septal defect : Overriding or dextroposed aorta
t Pulmonic stenosis t Right ventricular hypertrophy.

46. Ans. is 'a' i.e., Valvular stenosis [Ref, A.p, Ghai Bele p. 42A 6 pk p. 408; Nekon \p/e p. 1524_i526; Harckon tp/e
p. 14631
o Right ventricular outflow obstruction is one
ofthe characteristic oftetrology offallot.
This right ventricular outflow obstruction is mainly caused
due to infundiu"I". ,t.roriu.
Pulmonary valve stenosis (alone) rareiy contributes to the
right ,r.ri.i.rrlu. outflow obstruction in tetralogy of fallot.
Hurst cardiology says
"The pulmonary valve is oJten malformed,
usually being either biscuspid or unicuspid. The
valve may contribute
bv the inhibition{ ur
: &:,:H'l:::Jl'*i'J:::,*'::::T:11"1"1_ll1l
eyz.'.'.ein,the synthe,i.
of the
ut€ cIenzymecycroxygenase at birth
.: :"r*:Y_r::::.",:.i:,,:15ilt J";h;;;;r;;,tagrandin.
fflH;i;ffiX[i1g3,li:j::::3rto.io,.,p";;;;ly
infusion of drugs that inhibit cyclooxygenase.
i;ffi;ffi the crosure can,beproduced by

the enzyme cyclooxygenase.


a

i
'i:Ti."#*i'ilffi:ffi:Tsful and the ductus remains hemodvnamicariv
significanr, surgicar rigation shourd
o Recent studies from Europe indicate that
ibuprofen may be as effective as indomethacin for the medical
pDA.
preterm closure of

41. Ans. is '* i.e., ProstaglandinB


rfRef: O.p. Ghai tth/e p. 41g 6 Vh/e p.4071
' patent in the uterus in the fetal life because
of high tevet of vasoditators.especiaily
i;';:;:;:;';::;;;tr;s
r After birth the prostaglandin synthesis is inhibited
by inhibition of cycloxygenase at birth.
o so if the patency of ductus arteriosus
o The drug which is specificalry used for
has to be maintained n prir/ogtondirs should be given.
this is prostagrandin E, (Misoprost, Rioprostir)
c If the ductus arteriousus has to be closed, prorlaglanin ,yntrrrrii iniibito, - nai*ethacinshourd
be given.
42. Ans. is'H i.e., Misoprost [Ref; Netson lPth/e ch. 4jAl
o This question is framed improperly.

' #ir:fxi:l:xJl:il* are not used in patent ductus a.teriosus. rn pDA, prostaglandin antagonisrs (NSArDs e.g.
'ft:*:'ffii.T,::ii:f,H:fi:'j#Jt[3:l.the patencv orductus arreriosus in certai, cvanotic congenitai hearr
o Prostaglandin E, is responsible for
keeping the ductus patent.
o PGE-l used to keep open duct
are atprostadit o, *iroprostol.

43' Ans' is 'd i'e" It is more common in males than


female s fRef.: Nekan r*/e p. lBSg, 1879, ISgr;
PDA is tnore commoru infemales than males -
Nelson CFDT lgth/e p. 560, 56l)
.

TGA
:i. t:li:::j:a:: : ..i::::i.:,r'i:r '.:.:::: ... ii:: I .::...1,,i::.i:;r .::rii..,:ir_. , :

:$
M, Ans. is t'i.e., AII of above fRef: O.p. Ghai Ya/e p. 419 6 Zble p,
4a6l
o Endocardial valvulitis (infective endocarditis),
cHF and Eisenmenger syndrome, all can develop pDA.
in

FALLOT'S TETRAIOGY

45. Ans. is t'i.e., ASD tRef O.E Ghai Bele p. 420


& Tble p. 4,A8; Nels*n l},hlcp. DAZ, IgASl
o Constituents of TOF
t Ventricular septal de.fect : Overriding or dextroposed aorta
t Pulmonic stenosis t Right ventricular hypertrophy.
46. Ans. is 'a' i.e., Valvular stenosis
[Ref O.p Ghai Bn/e p. 420 & Zh/e p. lp/e
p. 14631
4a8;Nekan p. 1524_1526; Harrisan lp/e
o Rigltt ventricular outflow obstruction
is one of the characteristic of tetrology
This right ventricular outflow obstruction of fallot.
is mainly caused due to infundibuiar
Pulmonary valve stenosis (alone) rarely contributes stenosis. t
to the rigii ,r.rri.i.rru. outflow obstruction in tetralogy
Hurst cardiology says of fallot.
"7he pulmonary valve is oJten
malformed, usually being either biscuspid
or unicuspid. The yalve may contribute
-

i.il: rlll,lr.l,:,,rla:..*:lrll'rijl:-:.i:l7ri:;l*:$i#l{

to pulmonary stenosis, but, only uncommonly, it is the only site oi significant obstruction to the pulmonary Jlow"
Causes of right ventricular outflow tract obstruction.
i) Infundibular stenosis (4570)
ii) Obstruction at the pulmonary level (10%)'
iii) Combination of infundibular and vahrrlar obstruction (3070).
iv) Pulmonary atresia (15%).

4- Ang is'c' i.e., Predominantly left to right shunt lRe! O.P. Ghai 8th/e p. 421 6 Vh/e p. 409; Nelson 78th/e p. 1906,
1907, 19087
o There is predominately right to left shunt (not left to right shunt)'
o There is single 2"d heart sound and ejection systolic murmur in pulmonary area.
o JVP is normal.

{8. Ans. is '* i.e,,Central cyanosis with clubbingl&ef O.P. Ghai 8th/e p. 421 & Vh/e p. 409; Nelson 18'h/e p; 1907, DaSl
r Cyanosis and clubbing are seen in TOF.
o There is right ventricular hlpertrophy, no cardiomegaly, and chest x-ray shows boot shaped heart and pulmonary
oligemia.

49. Ans. is 's" i,e.,Tetralogy of fallot {Rel O.P. Ghai yth/e p. 421, 422 6 7h/e p. 4a8-4091

r The ventricular septal defect of TOF is always large enough to allow free exit to the right to Ieft shunt.
o Since the right ventricle is effectively decompressed by VSD, Congestive cariliac failure never occurs in TOF.
r Exceptions to this rule are : -
i) Anemia iii) Systemic hypertension v) Aortic or pulmonic regurgitation
ii) Infective endocarditis iv) Myocarditis complicating TOF
Remember
Following do not occur in TOF -> o Cardiomegaly o CHF o Recurrent chest infections

50. Aas. is t' i.e., Tetralogy of Fallot lRel & Vh/e p. 4091
O.P. Ghai 8'h/e p. 422
'Coeur en Sabot' refers to the radialogical appearance ofheart in a patient with Tetralogy of Fallot.
The cardiac silhouette resembles that of a'boot' or Wooden shoe in'Coeur en Sabot'fboot shaped heartl.

51. Ans. is'd'i.e., Isoprenaline lRef: O.P. Ghai 8n/e p. 422 6 Vh/e p. 409-410, Nelson 78th/e p. 1906' 19071

o In Tetralogy of Fallot there is right outflow obstruction due to pulmonary stenosis with supravalvular pulmonary
artery obstruction.
of severe obstruction, the right ventricular pressure becomes greater than the left ventricular presence and
" In case blood starts moving to the left ventricle resulting in severe cyanosis and erythrocytosis.
the deoxygenated
o In this situation the aim is to increase blood flow towards the right heart. The methods adopted are compression of
Aorta (knee chest position) and use of vasopressor drugs like methoxamine (Phenylepinephrine is also a
vasopressor).
o Any condition or drug which causes peripheral pooling of blooil should be avoided because this will red'uce
return ofblood to heurL
, Therefore isoprenaline which has marked F, agonistic action should be avoided as it will cause vasodilatation which
leads to peripheral pooling of blood. This will be harmful in a case of Tetralogy of Fallot.

52. Ans. is'o- i.e., Descending aorta with left pulrnonary afiery fRef: O.P. Ghai 7h/e p. 410]
Blalock - Taussig shunt -+ Subclavian artery - Pulmonary Artery anastomosis.
shunt
Pott's -) Descending Aorta to Pulmonary Artery
Waterston's shunt -) Ascending Aorta to right pulmonary Artery

f,J. Ans. is 'd i.e.,Aorta to Pulmonary Artery lRef : O.P. Ghai th/e p. 422 & 7/e p. 410; Nelson l8'h/e p. 19101

c Blalock Taussig shunt consists of subclavian artery (br. of aorta) and pulmonary artery anastamosis. This inturn
actually establishes a shunt between the aorta (Via the subclavian artery) and the pulmonary artery,
54. Ans. is 'b' i.e,,Modified BT shunt lRef: Nelson 18'h/e ch. 430; Iournal af cardiac anesthesiq 2014 p. 197ll \
o Modified BT (Blalock-Taussig shunt) is the most popular procedure for TOF.
55. Ans" is 'd i.e,, ASD [Rel: Dofland's Dictionary 28h/e p. 1253, 1746; Steilman's Medical Dictionary 28'h/e p' 14521

Pentalogyoffallot consists : -
o Tetralogy of Fallot -+ VSD, Pulmonic stenosis, Overriding of aorta, Right ventricuiar hlpertrophy
e Atrial septal defect

56. Ans. is 'd i.e., AS tRel O.P. Ghai 8th/e p. 420 6 Vh/e p. 40s)
5/. Ans. is'H i.e., Eisenmenger complex fRef: Ghai Vh/e p. 412)
e TOF, TA and TGA (with VSD & PS) are included in Fallot's physioiogy.

TRICUSPID ATRESIA
\

58. Ans. is 'a' i.e., i.e., Tricuspid atresia fRef: Nelson 18th/e p. 191j; O.P. Ghai Bth/e p. 422 & Vh/e p. 410-411)
. "Left axis deviation and left ventricular hypertrophy are usually present on electrocardiogram distinguishing
tricuspid atresia from most other cyanotic heart lesions.
The combination of cyanosis and left axis deviation is highly suggestive of tricuspid atresia."
- Nelion 18th/e p. 1913
59. Ans. is '-o" i.e., Tricuspid Atresia {Ref: o.P. Ghai 8th/e p. 423 & vh/e p. 410; Nelson l9th/e p. 19131
o Among the given options, left ventricie hlpertrophy is seen only in tricuspid atresia.

60. Ans. is 'a' i.e., Split S, fRef: o.P. Ghai 8,h/e p. 423 6 7h/e p. 410)
I Atresia of the tricuspid valve results in the absence of a communication between the right atrium and right veiitricle
therefore the right ventricle is underdeveloped the inflow portion being absent.
o The only exit for systemic venous blood coming to the right atrium is by way of Atrial Septal defect or patent foramen
ovale. Through this the blood goes to left atrium from rvhere it enters left ventricle. ,
o A ventricular septal defect provides communication between the Ieft ventricle and the outflow portion of the right
ventricle. The left ventricle therefore maintain both the systemic as well as the pulmonary circulatio,n thus there is hy-
pertrophy of the left ventricle which is reflected by lefi axis deviation in ECG.
r The pulmonary blood flow ls dependent on the size of the ventricular defect, the smaller the VSD, the lesser the
pulmonary blood flow. 90o/o patients of Triscuspid Atresia have diminished pulmonary blood flow.
o Auscultatory finding in case of Tricuspid Atresia -+
r S - Normal
1
r S, - Single r Murmur grade II to grade III i VI
61. Ans. is 'H i.e., Right atrial dilatation {Ref: Ghai 7h/e p. 411; Nelson 18,h/e p. 1915)
o The tricuspid valve anomaly results in obstruction of blood flow as well as regurgitation of blood from the right ven-
tricle into the right atrium -+ Dilatation and hypertrophy of right atrium due to volume overload.
62. Ans. is 'a' i.e., Tricuspid atresia fRef: O.P. Ghai 8'h/e p. 423 6 7h/e p. 411; Nelson l9thle p. 1915)
a Amongst the given options no one is correct.
a But best option is a + though tricuspid atresia is not there, abnormality of tricuspid valve is the basic pathology in
Ebsteinb anomaly.
r Ebstein anomaly consists of downward displacement of an abnormal tricuspid valve into the right ventricle.

TRANSPOSITION OF GREAT VESSELS

63. Ans. is 'a' i.e., TGA IRel Nelson 19th/e ch. 124, Texas birh defect epidemiology b survillencel
o Conotruncal defects are abnormalities of outflow tract septation or ectomesenchymal tissue migration abnormalities.
r Most common conotructal defect is transposition of great arteries (TGA).
64. Ans. is 1d' i.e., Anterior and right to pulmonary artery fRef: Ghai Vh/e p. 413; Nelson.tSth/e p. tgtsl
o In TGA aorta lies anterior and to the right of pulmonary artery,

65. Ans. is 'b' i.e., Transposition of great vessels [Ref Nelson 18*/e p. 1907; O.E Ghai B,h/e p. 424 6 7h/e p. 4i4-413)
o Out of the four options, only 1st two are cyanotic heart disease i.e. TOF 6 TGV.
o TOF can be ruled ott as it does not cause cyanosis at birth.
Nelson says - "aften cyanosis is not present of birth, but with increr*ing hypertrophy of the right ventricular
infundibulum and patient growth, Cyanosis occurs later in the 1st year of hfe."
r Short systolic murmur is not of much help in reaching the diagnosis as it is present in both the conditions.
i . .: ,i.,it:.rr:.rti:il.r l-i , i..tr.tr..::ti a.ri.tr:j.ir'l!:i::i
irt:,,lrr'',i : :ri::1"'i:ir:l
i . .:r,:.:rr: :::i:'::i i a:::.: !..i,i :r'.i:l :irli:i,.:.:r:r',r:
::11.s...:iii::$l li:ilii:iii:,ffi
..

tr. Ans. is 'H i.e., Transposition of great vessels fRef: O.P. Ghai 8th/e p. 424 6 Vh/e p. 41i-414)
o Cyanosis at birth, PGE infusion and atrial septostomy to improve oxygen are typical of TGA.

TAPVC

b. Ans. is'a' i.e., Supracardiac lRef: Schwartz Sth/e p. 622]


o Type ITAPYC with anomalous connection at the supracardiaclevel is the most common type accountingfor about 45%
of all cases.
e3. Ans. is 'b' i.e., Always associated with VSD [Ref: O.P. Ghai 8tn/e p. 427 dr Vh/e p. 415; Nelson 18th/e p.19221
o TAPVC results in the pulmonary venous blood reaching the right atrium r,yhich also receives the systemic venous
blood.
r The blood flow to the Ieft atrium is through patent foramen ovale or ASD.
o Because there is mixing of unoxygenated blood from systemic circulation and oxygenated blood from pulmonary
circulation in the right atrium -+ The oxygen saturation of blood in pulmonary artery is higher or identical to that in
Aorta.
o Infracardiac type of TAPVC is always obstructive whereas cardiac and supracardiac type may be obstructive or
nonobstructive.

69. Ans. is t'i.e., TAPVC lRef: O.P. Ghai 9th/e p. 427 dt Vh/e p. 415; Nelson 18th/e p. 19231
o Snowman orfigure of '8' configuration -) In supracardiac TAPVC.
:0. Ans. is 'None' lRef: o.P. Ghai 8th/e p. 426, 427 dt Vh/e p. 4151
o All options are correct.
o In supracardiac TAPVC the pulmonary veins join together to form a common pulmonary vein which may drain into
the left innominate vein.
o Not always associated with septal defect -+ Rt to left shunt may occur through Patent foramen ovale.
o There is "Figure of '8' configration" on chest x-ray in supracardiac TAPVAC.
o TAPVC is a cyanotic CHD.

EISENMENGER SYNDROME

7t. Ans. is 'a' i.e., VSD [Ry': O.P. Ghai 8'h/e p. 428 6 6'h/e p. 415; Nelson 18th/e p. 1936]
o Eisenmenger complex -+ Consists of pulmonary hlpertension with a VSD providing the right to left shunt.
72. Ans. is 'b' i.e., Left to right shunt, right ventricular hypertrophy, pulmonary hypertension, right to
left shunt lRef: O.P. Ghai 8'h/e p. 428 dt 6th/e p. 416; Nelson 18th/e p. 19361
r In VSD the flow of blood is from left to right, so the patient is acyanotic. When the direction of this shunt reverses i.e.
from right to left, the patient becomes cyanotic. This is k/a Eisenmenger's physiology or syndrome.
o There is very little or no flow of blood across the VSD in utero (as in utero right and left ventricular pressure are near
equal about 70 mm of Hg).
r At birth when circulatory pattern changes, the flow across the VSD begins from left to right.
o So it is understood thal the first eyent in sequence of development of Eissenmenger syn. would be left to right shunt
and last would be right to left shunt.
o The only confusion is whether pulmonary hypertension occurs earlier or right ventricular hypertrophy.
o For this it is known that "all children are born with right ventricular hypertrophy. As such initially all patients
of VSD have right ventricular hypertrophy" - O.P. Ghai,6h/e p. 298,

73. Ans. is 'a' i.e., Increased flow murmur across tricuspid & pulryonary valve fRef: Nelson Vh/e p. 1936-19371
o Eisenmenger syndrome refers to patients with a VSD in which blood is shunted from right to left as a result of
development of pulmonary vascular resistance. Initially shunt is from left to right as the systemic vascular pressure
is greater than pulmonary vascular pressure. With time pulmonary vascular resistance increases due to change in
pulmonary vessel wall as a result of increased flow in pulmonary vessels. When pulmonary vascular pressure
exceeds the systemic vascular resistance, reversal ofshunt into right to left shunt occurs.
o This development of right to left shunt due to reversal of left to right shunt as a result of development of pulmonary
vascular resistance and pulmonary hypertension is called Eisenrflenger syndrome.
systeft

a In a normal person the pressure in the pulmonary


circuit is lower than in the systemic circuit
and in the presence ofthe defect a
left to right shunting occur
I
O This means an increased pulmonary
blood flow compared to
the systemic flow
+
Over aperiod of time the increased
pulmonary blood flow leads to
pulmonary vascular changes

ne.crotizing *i,r,
I ::::l::"::::::n:.lnd
with"resuttant I time irreiersibre changes
I ant
I
::r::::,:rru.r
oot,teratron oJ pulmonary II
of smalt putmonary arteritLs

L arterioles and c apullaries ) I


Features ofpulmonary
Development of pulmonary
Htpertension vascular disease leads to
o Parasternal heave Pulmonary vasculir resistanc e
s Loud P2 Pulmonary hypertension
o Single S 2
+
For a short time the pressure b/w
the circuits may be equal with
virtually no shunting

I fnit i, the point at which 1


t
With progressgion of the disease pressure
I Eisenmengher'ssyndrome I in the pulmonary circulation e*ceeds
I develops J systemic circulation
+
Left to right shunt change to
right to left shunt

' {;::;;!i::::7{{;;J:,:xlSx.ertension can be assumed to be a sign of impending


Eisenmenger\ syndrome.
i) Loud p2
ii) Single 32
lii) Parasternalheave
iv) pulmonary ejection click
v) Graham steel murmur (a high pitched decrescent
diastolic murmur due to pulmonary regurgitation).
74' Ans' is 'H i'e', RV & LV walls come back to normal
size {Ref: Nelson 18tu/e p. 1936-1937,
o Harrison Lp/e p. H59l
Thecharacteristic pathological pulmo.nary rascular
changes that deverop i* jriii'x
ate confined to arteries only and involve-small-pulmonarf with Eisenmenger syndrome
arterioles ani musiular arteries (veins are
r There is right ventricular-hlpertrophy (RV waris do not come not intolved).
back to normal size).
o on x-ray there are dilated and prominent
central arteries with rapid tapering (pruning)
of the peripheral vasculature.

IqARCTAT|ON OF AORTA
75. Aas. is'H i.e., Turner,s syndrome {Ref, Nelson lge/e
p- 19001
r coA may be a feature of furner syndrome, and is associated
with a bicuspid aortic valve.
76' Ans. is t'i.e., Bicuspid aortic valve
IfieJ o.p Ghai p.
8e/e 432 & F,/e p. 420)
r Bicuspid aortic valve is associated in more thanT}o/oof
patients.
It is also associated with : -+ pDA, vsD, ASD, TGA,
' ruyiritrrtx le{t heart
Ans. is'c'i.e,, Pulmonary stenosis lRef: C,P.D.T. p.
564, 5i65; Nelson Ig
ISth/e Sh/ep. D0Al
o Bicuspid aortic valve, Turner syndrome and tubular hlpoplasia of aortic arch (atresia of aortic arch) may be associated
with CoA.

:8. Ans. is 'a'i.e., Most common site is distal to the origin of left subclavian artery fRef: a.p. Ghai ttu/e p. 431 &
7le p. 419-420 ; Nelson 18th/e p. DA\, D01l
e Most common site of COA is distal to the origin of left subclavian artery.
o There is left ventricular hypertrophy and rib notching of inferior border (not upper).
o Neonates become symptomatic immediately, especially in preductal coarctation (not at 15-20 years).

:9. Ans. is'c'i.e., Incorcostal arteries and superior epigastric artery fRef: www.emedicine.com/radio/topic42.htm;
http://perfline.com/student/coq.html; Grays anatomy 38th/e Section 10 cardiovascular system|
r The coarctaion may be :
- Preductal - where is narrowing is proximal to the ductus arteriosus or ligamentum arteriosum. or
: Postductal - where the narrowing is distal to the ductus arteriosus or ligamentum arterosum.
o Collateral circulation connects the proximal and the distal aspects of the vessels over time. This collateral circulation
will develop mainly from the : -
r Subclavian, r Internal thoracic r Intercostal arteries
r Axillary, r Superior and inferior epigastric and
o Following collateral connections may be seen :
r In the anterior thoracic wall, the internal mammary arteries (arising from subclavian arteries) and the epigastric
arteries join to form collaterals which supply the abdominal wall and the lower extremities.
o Internal mammary artery gives rise to anterior intercostal arteries which forms anastomoses with the posterior
intercostal (post intercostals arise from descending aorta).
e Superior epigastric artery is terminal branch of internal mammary artery and forms anastomoses with inferior
epigastric artery which arises from external iliac artery.
o Musculophrenic artery is terminal branch bf internal mammary artery and forms anastomoses with inferior
phrenic and post intercostals.
r The Para scapular arteries (arising from subclavian artery and axillary arteries) connect with the posterior intercostal
arteries to form collaterals which supply the distal aortic compartment.
o The suprascapular artery from the subclavian; and the thoracoacromial, lateral thoracic and subscapular
arteries from the axillary; and the first and second posterior intercostal arteries from the costocervical trunk
anastomose with other po sterior intercostal arteries.
r One route of collateral formation can be subclavian artery - vertebral artery-spinal arteries - post intercostals and
lumbar artery - aorta.

80. Ans is '& i.e., Coarctation tf *a** LRd: A.n 6hai thle p. 432 & ?kle p. 420; Nelso?r I*/e p. 1900, $7tj
I Symptoms of coarctation of aorta occurs due to decrease in blood supply distal to aortic obstruction.
e If obstruction is distal to left subclavian artery, manifestations are due to arterial insufficiency to lower limbs.
t Intermittent claudication r Pain
r Weakness of Leg t Dysnea on running
r Weak pulses and lower BP in the lower extremities --> Delayed or weak impatpable femorals compared to
str ong br achi al arte rie s.
8l- Ans. is '& i.e., Anterior Nll lRef: Nelson 18b/e p. DA0, DAl; O.P Ghai */e p. 4j2 & fi/e p. a2l
o The causes of mortality in co-arctation of aorta are :
. Hypertension t
Aortic dissection s Premature athero s clero sis
s Rupture ofaorta - Left ventricular failure t Infe ctiv e en do car diti s
s Cerebralhaemorrhage (due to aneurysm rupture)

82. Ans. is'H i.e., Coarctation of aorta lRef: A.P. Ghai */e p. 32 & 7/e p. 2Al
Feeble femoral pulsations com.pareil to strong brachial pulsations suggests the diagnosis of coarctation of aorta.
Hypertension and strong pulsations in the upper extremities anll absence, maiked dimunition or delay in pulsations
in the femoral artery are characteristic in coarctation. Also failure to thrive and congestiye heart failure are
consistent with the diagnosis of congestive heart failure.

Remember:
o Differential cyanosis is a maniftstation of PDA
o Differential pulses/Blood pressure is a manifestation of Coarctation

83. Ans. is t' i.e., coarctation of aorta fRef:Nelsorc 18*/e p. 1900, 1901; o.p. Ghai 8th/e p. 432 6 Vh/e p. 420)
c Isolated upper extremity hypertension (200/140 mm Hg), together with absent/diminished
characteristic feature of coarction of aorta.
femoral pulses i-, r.

OTHER CHDS

84- Ans. is 'a'i.e., Down syndrome lRef: Nelsan Igth/e chap" 426.5]
o Ostium primum ASD (an incomplete atriventricular septal defect) or endocardial
cushion defect is common in Drr :
syndrome.

85. Ans. is 'd' i.e., Anomalous coronary artery p. 1930, 193r)


fRef: Nelson rgth/e
o There is anomalous origin of the left coronary artery from the pulmonary artery: In this
anomaly the blood sup:.
of the left ventricular myocardium is severely compromised. Soo., aft". tirth (Neonate) as the pulmonary
arte:-:-
pressure falls, perfusion pressure to the Ieft coronary artery becomes inadequate . Myocardial
infarction andfibrosr.:
may result.

86. Ans is 'b' i.e., Hypoplastic left heart syndrorne lRef Nelson Pediatrics 18th/e p. t9_26-1925)
l'lypolastic left heart syndrome
o Hlpoplastic left heart syndrome occurs when parts of the left side of the heart (mitral
valve, left ventricle, aor:::
valve, and aorta) do not develop completely. The condition is present at birth (congenital).
o Hlpopiastic left heart is a rare t)?e of congenital heart disease. It is more common in males
than in females. As rr i::,
most congenital heart defects, there is no known cause. About l0 % of patients with hlpoplastic left
heart syndron:=
also have other birth defects.
o The problem develops before birth when the left ventricle and other structures do not
grow properly, including the:
i) Aorta-the blood vessel that carries oxygen-rich blood from the lett ventricle to the"entire bodv
ii) Entrace and exit ofthe ventricle
iii) Mitrai and aortic valves
I This causes the left ventricle and aorta to be poorly developed, or hypoplastic. In most cases, the left ventricle anc
aorta are much smaller than normal.
o In patients with this condition, the Ieft side of the heart is unable to send enough blood
to the body. As a result, the
right side of the heart must maintain the circulation for both the lungs and the 6ody. The right ventricle
can suppor:
the circulation to both the lungs and the body for a while, but this extra workoad eventually causes
the right side oi
the heart to fail.
o The only possibility of survival is a connection between the right and left side of the heart,
or between the systemic
arteries and pumonary arteries (the blood vessels that carry blodd to the lungs). Babies are normally
born with two
ofthese connections:
i) Foramen ovale (a hole between the right and left atrium)
ii)Ductus arteriosus (a small vessel that connects the aorta to the pulmon ary artery)
o Both of these connections normally close on their own a few days after birth.
o In babies with hlpoplastic leIt heart syndrome, blood from the right side of the heart traveis
through the ductus
arteriosus. This is the only way for blood to get to the body. if the ductus arteriosus is ailowed to close
in a baby with
hlpoplastic left heart syndrome, the patient may quickly die because no blood will be pumped to
the body.
o Babies with known hlpoplatic left heart syndrome are usually started on a medicine
to keep the ductus arteriosu-
sopen.
o Because there is little or no flow out ofthe leftheart, blood rearning to the heart from the lungs
needs to pass through
the foramen ovale or an atrial septal defect (a hole connecting the collecting chambers on"the
left and rlght sid""s
of the heart) back to the right side of the heart. If there is no foramen ovale, or if it is too small,
the baby cluld die.
Patients with this problem have the hole between their atria opened, either with surgery
or using a thin, flexible tube
(heart catheterization).
Symptoms
o At first, a newborn with- hypoplastic left hegt may apper normal; Symptoms usually occur
in the first few hours of
life, although it may take up to afew days to develop symptoms. These symptoms may inciude:
r Bluish (cyanosis) or poor skin color
r Cold hands and feed (extremities)
I Lethargy
r Poor pulse
r Poor suckling and feeding
r Pounding heart
: Rapid breathing
r Shortness ofbreath.
t Since the systemic circulation is dependent on the patent duclrs arteriosu s the closure of ductus arterious leads to shock.
s failure usually appears within the first few days or weeks of life and include dyspnoea, hepatomegaly
Signs of heart
and low cardiac output. When PDA closes suddenly shock occurs all the peripheral pulses may be weak or absent.

Ve nt r i cul ar s eo t al d efe c t
o Thesepatients with VSD's become symptomatic around 6-10 weeks of age.
o They usually present with congestive cardiac failure.

Ebstein's anomal)t
o Ebstein\ anomaly consists of downward displacement of an abnormal tricuspid value into the right ventricle.
o Thesepatients usually present in teenage/adoEstent years.
o They may also present in infancy but they usually do not present with shock or severe hypoperfusion.

Aor ti c opulm onary w in do w defe ct


o An Aorticopulmanry window defect consists of a communication between the ascending Aorta and the main pulmonary
artery.
t In these cases present and they may present with heart failure in infancy.
minimal cyanosis is

87. Ans. is t' i.e., Truncus arteriosus lRef: V, Mohan Reddy Cardiac surgery for premature qnd low birth weight neonates.
Pediatric cardiac surgery annual of seminarsl
o Congenital heart disease in the newbo.rn can be broadly categorized by the relationship between the patients cardiac
defect and the patent ductus arteriosus; this categorization yields four distinct groups:

I. Pulmonary flow ductal dependence.


o Newborns with congenital heart disease who are dependent on the patency of their ductus for pulmonary blood
flowpresent with varying degrees ofcyanosis.
t Critical Pulmonary Valve Stenosis with lntact Ventricular Septum.
t Tricuspid Atresia:
t Tetralogy ofFallot
II. Systemic flow ductal dependence
o These newborns are dependent on their ductus arteriosus for systemic blood flow and unlike the babies who are
pulmonary flow dependent, these newborns present with severely decreased cardiac output.
o This decreased systemic flow is characterized by pallor, diminished peripheral pulses, low urine output, cool
extremities and varying degrees of metabolic acidosis.
o This includes newborns with left ventricular outflow tract obstruction at various levels.
t Congenital Valvular Aortic Stenosis
t Coarctation of the Aorta
t Interrupted Aortic Arch
t Hypoplastic Left Heart Syndrome (HLHS)
III. Oxygenation ductal dependence
r These are babies who are born with congenital heart disease that requires a patent ductus for adequate mixing of
saturated and desaturated blood.
t Transposition of the Great Vessels (TGV)
IV. Ductal independence
o These are neonates who are not dependent on a patent ductus but still require urgent operation.
t Total anomalous pulmonary venous return (TAPVR)
t Truncus Arteriosus
t Anomalous origin of the left main ,oronory artery from the pulmonary artery.

88. Ans is t' i.e., Obstructive TAPV&tRel Pediatric cariliology Sthlep. 164, 1651

r PGE 1 is used to maintain the ductus arteriosus.


o Therefore, it is obvious that PGEI will be used in heart lesions where the patency of ductus is essential, i.e., in ductus
dependent lesions.
o Ductus dependent CHDs are:
i) PS with intact ventricular septum (PS without YSD) v) Coarctation of aorta (Obstruction in aorta)
ii) Tricuspid atresia vi) Interrupted aortic arch
iii) ToF vil) Hypoplastic left heart syndrome
iv) Congenital vahrrlar aortic stenosis viii)TGV
(obstruction in aorta)
so, option a, b & d can easily be excluded as PGE is used in
r'
these CHDs (pGE does not worsen).
Following conditions are not ductus dependent. So, these
do not require maintenance of patency of ductus. Infact
these can be worsened by maintaining the patency of
ductus (by pGE 1):
i) TAPVR
ii) Truncus arteriosus
iii)
Anomalous origin of the reft main coronary artery from the pulmonary
artery.
89. Ans. is'-u" i.e., Pulmonary atresia {Ref: Nelson 18n/e p. 1912,
19027
o The findings in this newborn are -
t C.yanosisatbirth
: Oligaemic lungfields
t Normal sized heart
r Lets see each option one by one:
r Ebstein anomaly - It can be ea.siry ruled out as there is marked cardiomegaly
in it.
r Transposition of great ve ssels- There is cardiomegaly and plethorh fifi i"ii,
see in it. so ruled out.
o Now the real confusion - between pA and TOF.
r ToF - in ToF there is oligaemic lung fields and normal sized heart but usually
cyanosis is not present at hirth.
o-Nelson states - " often cyanosis is not present at
birth. but with increasing iyprrtrophy of the right ventricular
infundibulum an.d patient growth, cyanosis occurs later in the
Ist year of life. But some cases may present with
cyanosis at birth."
' r Pulmonary atresia -
o The child is cyanotic atbirth
o There is pulmonary oligaemia
o But the heart is of variable size (ftom small to enlarge)

Cyanotic heart disease

Pulmoirary plethora Pulmonary Oligemia

.
+ -------_]
Severe cyanosis at birth a Mild cyanosis at birth
r SingleSz a Widely split S,
o Cardiomegaly a Cardiomegaly
t No murmur or ejection systolic a Tricuspid flow murmur-diastolic &
murmur pulmonary ejection systolic murmur
. ECG-RVH ICGR/H
Egg shaped heart on x-ray
+ +
TGA TAPVR

. Cyanosis usually not at birth. t a


r
Cyanosis at birth Cyanosis at birth I Cyanosis at birth
NoCHF . Smilar to TOF a No heart murmur . Prominent'V' wave of JVp
r Normal sized heart except: - o LVHwithnormal r Triple or quadruple rythm
r Puimonary ejection systolic 1. LVH i
QRS axis o ECG-RBBB
murmur 2.ilominentlarge'a' . Variable size heart . SoftTRmurmur
r Single S , wave on fVP (normil to large) . Extremecardiomegaly
r ECG-RVH 3. Hepatomegaly
r Bootshapedheart
+ + + I
Y
TOF Tricirspid atresia Pulmonary atresia Ebstein anomaly
p' a15' 419' 421' 432
t' i.e', coarctation of aorta lRef: o'P' Ghoi 8h/e
lt! Ans. is 'l i.e.,v.S.D.; 'H i.e., P.D.A.;
e 7e/e P. 404, 4A6, 4A9, 42Al
rAcyanoticheartdiseasewithejectionsystolicmurmur+ASD,VSD'PDA,Coarctationofaorta

CPDT 1*/ep' 571' 58al


&tr. Ans. is'b- i.e., Truncus arteriosus lRef:
o Right aortic arch is associated with : - ,-
ii) 25% of cases of ToF'
iiio"z" of .u."s of Truncus arteriosus
.b, ef: Netsan r*/e ck*pter 432; pcdiatric cerdierrcscnter meiliei*e
Ans. is i.e., Trachear compression symtoms [R
by lcm* H. Mo$er P' 67t\

result in the formation ofvascular rings around


"tj::"r:#kff:.-r-ies of the aortic arch *a i , -uio. branches
- degrees of compression'
irr" i?".i")*d esophagus with varying

: *m:m:):;;:mff.l}:[ffiT f'ololip",,*tsdistattotheoriginoftherightn cervicatintersegmentat


arterY.
are :-
o Clinical manifestations of double aortic arch a double aortic arch' 55olo
tr'e tightness ofthe ring' About 75olowith
1. The age and type of presentation a.p."Jot' compression by an abberant innominate
with a right r. & abberant fut *uJu,ri*"urtery and ss"/";th frequent respiratory
".1 "i.t ptoms, including stridor persiitent cough and
artery present in infancywith ."pi.uto[.yn
present during
the trachea and esophagus, s1'rnptoms are frequently
,. #r"riJltj;."rar ring produces compression of
infancY.
feeding'.and flexion of the neck'
3. Chronic wheezing is exacerbated by crying'
noisy respiration'
4. Extension of the ieck tends to relieve the
5.Vomitingmayalsobeacomponent.Affectedinfantsmayhaveabrassycough,pneumonia,orsuddendeathfrom
of respiratory symptoms
teenagers or young adults with a history
a 3il:1$" present in later childhood or even as
asthma'
that mimic asthma or exercise-induced
asymptomatic during entire childhood'
7. Some u.. aiffi.Jlt.. ir.,life after being
symptoms being the presenting symptoms'
g. The later the presentation, the more cha'nces of gastrointestinlal
may be seen'
Symptoms like dysphagia' choking vomiting

93. Ans. is'd i.e., C,{TCH 22 lRefNclson l*le ck'*a}


CATCH 22 stands for -
atresia'
o Cardittc defect - double arch aorta' VSD' pulmonary
o Abnormal facies
o ThYmichYPoPlasia
o Cleft Palate
o HYPocalcemia
.H i.e., Frontat lob e |Re! lnitian Heart j 1989 Maylune; 4l (3): 19a.3}
94. Ans. is
of Fallot is M'C' congenitatheart disease causingbrain
abscess'
o Tetralogy
by parietal lobe'
o Frontal tru, lr';r**rnest siteior abscess localisation followed
p' 4Ll;Nclsor ch' 435\
95. Ans. is'a'i-e., Ebstein anomaly lRef: GhaiTele anomaly'
accessory pathway seen in individuals
with Ebsteins
o People with wPw may have more than one

HEART FAILURE
\
.d, o. P. Ghai 8th/e p, j96 dt 7e/e p. 373)
96. Ans. is i.e., Congenital heart diseas e |Ref :
-+ Congenital heart disease' '
o The most common cause of CHF in infants rheumatic heart disease'
childien -+ Rheumatic fever &
c The most common .""r" .f Cftp in the older
o'P' Ghai }th/e p' 396 & ?hle p' 3731
97. Ans. is [, i.e., Hlpoplastic left heart syndrome [Rel
left heart syndrome causes CHF in First week'
r Amongst the given options, hlpoplastic
98. Ans. is 'd'i.e., Hepatomegaly teel O.P, Ghai 8,h/e p. 39V & Th/e p. 375; Nelson tr,hle p. 1977]
o In infants, heart failure may be difficult to identify.
r Prominent menifestations include:
i) Symptoms
r Tachypnea, tachycardia r Feeding difficulties r poor weight gain
r Excessive perspiration r Irritability r weak, cry often hoarse
r Noisy, labored respption r Flaring of alae nasi r Intercostal and subcostal retractions
ii) Signs
rWheezing , rHepatomegaly
r Cardlomegaly i . cailop rfrytnm 1s:;
o Clinical assessment bf luglar venous pressure in infanJs may be difficult because of the
shortness of the neck and
the difficulty of observing a relaxed state.
o Edema of face occurs early, Edema on thefeet occurs late.

99. Ans. is 'a' i.e., Pulmonary edema [Ref: O. p. Ghai Bth/e p. 398 6 Vh/e p. iZS]
o Pulmonary edema is due to Lt sided heart failure that results in increased back pressure
in pulmonary circulation.
100. Ans. is 'a' i.e., ACE inhibitors [Rel aP Ghai Sth/e p. 398-399 6 Hanison tVh/e p. 15601
r Drugs for patients with hlpertension and cHF -+ 1. Diuretics 2. AcEinhibitors 3. B-blockers

RHEUMATIC FEVER
l0l. Ans. is t' i.e., communicable disease {Re! a.p Ghai Bth/e p. 434 6 vh/e p. 379; Nelson tSth/e p. 11401
r The disease is immune mediated, not a communicable disease.
l02. dns. is t' i.e., Fever [Rel Nelson tsth/e p. il4t; o. p. Ghai 8th/e p. 435 6 vh/e p. 3s0]
o Fever is a minor criteria
o Major criteria are: Carditis, arthritis, subcutaneous natule, chorea, and erythema
marginatum.
r Minor criteria : (A) Clinical: fever, arthralgia, previous RF or RHD; (B) taboratory: Increased ApR, CRp, ESR,
leukocltosis.
r Essential criteria: Increased ASLO titre, postivie throat culture, recent scarlet fever.
103. Ans" is'd'i.e., Elevated ESR [Rel: O.p. Ghai B,h/e p. 4j5 6 Th/e p. 3s0]
o Elevated ESR is a minor criterian.
o Carditis (Pancarditis), Arthritis, and chorea are major criteria.

104. Ans" is 'b' i.e., Arthralgia [Ref Nekon ttthte p" 152 (table js2-2)]
o In Rneumatic fever = major criteria is arthritis, arthalgia comes in minor criteria.
105' Ans, is 'd'i.e., Pancarditis [Rel o,p. Ghai Sth/e p, 434 6 v'le p. 380; Nelsan trth/e p. 1141]
r It is a pancarditis involving all the three layers -+ Myocardium, endocardium & pericardium.
r06. Ans. is t' i.e., Can be associated with AR fRef: o.p. Ghai 8th/e p. 436 6 vh/e p. j81l
Carey-Coombs Murmur
o Heard in patients with acute rheumdtic fever.
o Occurs due to inflammation of mitral valve cusps or excessive Ieft atrial blood flow
as a consequence of MR.
o Soft middiastolic murmur usually follows Sr.

rc7 " Ans. is 'H i.e., Acute rheumatic fever [Ref: o. p. Ghai 8*/e p. 4j6 6 Th/e p. 381)
108" Ans. is 'a' i.e., Arthritis l&ef a,p. Ghai Bth/e p. 434, 436 6 Th/e p, 3ga; Nelson IBil,le p" t 14tl
r Arthritis is the most frequent maj or manifestation (75% of cases).
109. Ans.is'a',i.e.,choreaisaggravatedduringpregnancy[Rel o.p.GhaiSth/ep.4i66Vh/ep.380-381;Harrisonl6th/e
p. 1978; Nelson 18th/e p. 11421 g
o The characteristic picture of chorea is th at it ofien occur in isolation,either unaccompanied
by other major manifestations
of RF or after a latent period of several months, at a time when all other manifestutlorr. of RF
have subsided.
oSexhormones(estrogen)orpregnancycancauSerecurrenceS.
About option'd'
Erlthema marginatum (not erlthema multiforme) is seen in RF.

110. Ans. is'a'i.e., Mitral regurgitation [Rel 0.P. Ghai 8'hle p. 436 & lh/e p. j84; Harrison 16h/e p. 1978, 1979;
Nelson 18th/e p.1142)
r In rheumatic fever vall'ular abnormalities usually are associated with chronic disease but mitral regurgitation is seen
in acute rheumatic fever.
t Mitral regurgitation is the commonest manifestation of acute as well as previous rheumatic carditis. - O.P. Ghai

111. Ans. is'd' i.e.,Pulmonaryvalve [Rel Ghai Th/e p. 381)


r Pulmonary valve involvement is never seen.

ll2. Ans. is 'a' i.e., Erythromycin [Re/ A.P. Ghai 8*/e p. 438 6 6h/e p. 377, 378)
r Drug of choice -+ Penicillin
o Drug of choice in penicillin allergic patients -+ Erlthromycin
I 13. Ans. is 'd i,e,, Carditis lRel A,p. Ghai 9e/e p' 437 lt Vh/e p' 3831

Suppressive therapy of RF
r If patient has carditis with CHF --) Steroids
r If patients has carditis without CHF -) Steroids or aspirin (steroids are preferred)
r If patient does not have carditis -) Aspirin
The total duration of suppressive therapy is 12 weeks.

114. Ans. is ? i.e., 12 weeks fRef: O.P. Ghai 8*/e p. 437 6 7h/e p. 383)

INFECTIVE ENDOCARDITIS
115. Ans. is 'c' i.e., Staphylococcus aureus fRef: A.P. Ghai 7h/e p. i91; Harrison 17/e p. 790; Nelson 18tb/e p. 1953)

Most common organism causing IE


t In native valve -) Staplrylococcus aureus
t In prosthetic valve -) Coagulase negative staphylococcus

116. Ans. is 'd' i.e., Small ASD [fief O.P, Ghai Vh/e p. 3901
o Infective endocarditis is rare in secundum ASD, unless associated with mitral valve prolapse.
t Endocarditis is unusual in sites with a small pressure gradient as ASD -Harrison
: Infective endocarditis is very rare in patients of ASD -Ghai

ll7. Ans. is t' i.e., Staphylococcal endocarditis [Rel: O.P Ghai thle p. 444 6 7h/e p. 390-i9il
r Any fever in a patient with known heart disease raises the question of endocarditis. It can occur over the mitral valve
or Aortic valve in patient with Rheumatic fever.
t The minimum uiteria fo. the diagnosis of endocarditis consists of unexplained fever of Z-lO days duration in a
patient with known heart disease. If this is associated with other clinical manifestations of endocarditis, the
diagnosis becomes more firm. (In this case besides fever, hematuria and palpitations are other 2 symptoms oJ
endocarditis).

HYPERTENSION

118. Ans. is'c' i.e., Renal Parenchymatous diseas e LRef: O.P. Ghai 8e/e p. 451 6 7e/e p. 432i
r Essential hypertension is relatively uncommon (107o) in children in contrast to aged persons.
o The four most common causes of hlpertension in children are : -
i)
Chronic glomerulonephritis (most common) 25 o/o -)
ii) Chronicpyelonephritis -+ l0 o/o

111) Renal artery stenosis -) l0 o/o

Essentialhypertension 0
iv) -+ rc6h
I19. Ans. is ? > d i.e., Renal Parenchymal disease > Chronic glomerulonephritis l&ef CPDT l$th/e p. 598\

r Hypertension is children is commonly of renal origin'


. it is anticipated as a complication of renal parenchymal disease CPDT
r - chronic
Among the renal parenchymal diseases, chronic glomerulonephritis is the most common cause followed by
pyr'loneph ritis.

X20" Ans" is'au i.e", CGN t-l{eJ See previous explanatiarus)


r Chronic glomerulonephritis (CGN) is the most common cause of hlpertension in children.
121. Aas. is t'i.e., Glorneruloaephritis {Ref O.p. Ghai Be,/e p. 451 & V,/ep_ 4j2l
r Tiris child is suffering from hypertensive crisis.
Hypertensive trisis
t ln kyltertensive crisis, BP is rapidly rising or a high BP level is associated with neurological manifestations,
henrt t'ailure or pulmonary edema.
r Ii-rpertensive crisis may be divided : -
i) Ilypet'tiensive ernergencies-+ Situations in which immediate reduction of Bp is needed (within minutes),
usually with parentral therapy.
ii) Elypertensive urgencies -+ Situations in which reduction of BP is needed within hours, usualiy with oral
agents.
iii ) ,4ccelerated malignant hypertension -+ Situations in which papilledema, hemorrhage, and exudate
are
associated with markediy elevated BP. The diastolic pressure is usually greater than 140 mm Hg.
;1J Hyfiertensive errcephalopathy -+ Situations in which markedly elevated BP is associated with severe headache
ir:ei varior:,; ilteraiion in consciousness.
r 5':, ihe chil<i irr q'Lrestion having hypertensive encephalopathy i.e., markediy elevated BP with impaired sensorium.
r iilpertensive enr;*phalopathy may develop in any cause of secondary hypertension if Bp is markedly elevated.
r li{*st commoil catse of secondarv hl,pertension in a child is chronic glomerulonephritis.

MISCELLANEOUS
122" ,4ns. is 'a' i,e", ,{cute &}lear*xlatie fever
I Rlieurnatie carditis is the most common acquired heart disease in chiidren and most common cause of CHF in
. ,-rl14fe 11.

123. Ans" is t'i.e", p*A i l?-eJ': I?"J" Mektea trStt,/e p. t5)


L24. Ans. is tf i.e., Al! qrf above
}8t'/e Chap. ttr22}
{ReJ: Netso*
o Coarctationofaorta(COA),interruptedaorticarchandpersistentpulmonaryhypertensionofnewborn(ppHN)can
cause differential cvanosis.
125. Ans. is 3'i.e., Lowe's S;rndrome {Ref: O.P Ghai#/e p- 423 {2 7e/ep. XI

#t
1" lnfection x Viral, Bacterial, Fungal, Rickettsial, parasitic.
2- Metabolic dnd sndairihe dlsorders x
..

3. Neurologicol and muscular disorders a Friedreich ataxia, Muscular dystrophies.


:4::'!gvi4:,., ': ': ' i:, t Ad{arytyein P.fren othiiizi ne; [ead; En:i-tine; €]i
5. llematolaEical e Sickle celi anemia, Thrombotic thrombocytopenic purpura.
6. *eicldiso#ers r Glomerulonephritis
7. Neoplostic r Rhabdomyoma, myoxoma, Leukemia, Lymphoma.
&: Miscellaieous r ldiopathic aortitis, Cystic fibrosis.

126. Ans. is ? i.e., Klippel - Trenaunay syndrome {Ref: Nelson 18"/e p. 26691

Klippel -Trenaunay syndrome {KT syndrome}


r f1-le triad of KT svndrone tp-
r C:.itaneous.,.ascular tnatrformation -+ Most commonly capiilary malformation.
r ,jilnv capillari, malfcrmaticrr and soI1 tissue hypertrophy.
I Venous abnormalities -+ Deep venous system may be absent or hypoplastic.
r The anomaly present at birth and usually involves lower limb'
e When there is associated Arterio - venous malformation, it is known as Klippel - Trenaunay - Weber syndrome'
e Thick walled yenolts varicocities tlpically become apparent ipsilateral to the vascular malformation -> May
be

pulsatile because of AV malfurmation.

127. Ans. is 'd i.e., I vein and 2 arteries [Re/; o.P Ghai */e p. 402 & 6elc p. 674]

ANSWERS OF VARIOUS OTHER EXAMINATIOI\IS

128. Ans. is '& i.e.,Cornplete heart block lRef Myutrgk park S'hle p' 4|

c Maternal lupus erythmatosus has been associated with a high incidence of congenitai heart block in offspring'

l2g. Ans. is'b- i.e., Ostiurn secondum fR.ef. Hasbeen explairced]

130. Ans. is 'c' i.e.,squatting lfuef: A-P" Ghoi 8*le p' 421 & Vhlc p' 409],
r The patients of ToF assume a sitting posture (squatting) as soon as they get dyspneic.
o TOF is the commonest congenital lesion in which squatting is noted.

131. Ans. is'b'i.e., Diversion of the septal defecllRef Nekan l*le p" 1922, 1915, 192CI1

r Double-outlet right ventricle with transposition of great arteries is known as Taussig-Bing anomaly.
It also has

associated VSD.
o Surgical correction is bY: -
A) Rastelli procedure which includes
: patch closure of the VSD, with left ventricular flow is directed to aorta.
an extra cardiac
r Connection of right ventricle to pulmonary artery by ligating the proximai artery and placing
homograft conduit between the right ventricle and distal pulmonary artery.
OR
B) Arterial switch (Jantene) procedure
r The operation involves dividing the aorta and pulmonary artery just above the sinuses and re-anastomosing
them in their correct anatomic positions'

L3Z. Ans. is "H i.e., Puknonary stenosis lRef : O. P. Ghai 8&/e p. 42A, 421 e /n/e p. A9|

133. Ans. is'd'i.e., Tetratcgy of fallot lRef: Hasbeen explai*ed]


o Features in this child suggestive of TOF :-
i) CYanosis at2Year (not at birth)
ii) No palIor, dysnea or edema + No CHF
iii) Normal sized heart
e These features suggest the diagnoiis ofTOF'
r These have been explained earlier.

134. Ans.is.b i.e.,pDAwithreversalofshunt[Ref o.P.Ghei*elcp.418,41g{rftlep.407;Cha.ndresamotnyWTd/ep-


342\

135. Ans. is'H i.e., Indomethacin therapy lRef: aP Gk*i 8*/e p" 419 {, */e p. aa7}

136. Ans. is 'c' i.e., Transposition of great vessels [-&ef O. P. Ghai 8* le p. 425 & */e p' 414]

137. Ans. is 'H i.e., Transposition of great vessels [Ref O. P. Ghai Sele p. 425 d7 */e p- ala]

l3g. Ans. is
.b,
i.e", Transposition of great arteries [Raf: Nelson 78e/e p. $a7-ag, ]919-2]; A-P" Ghai *le p- 425 &
7ele p. 412-131

139. Ans. is t'i.e., prostaglandin ErlRef : GuptqS" 8*le p. 2$; A"P Gboi 8*/e p. 419 er Ple p- V34;Davidsatls
20h/e P. 7441 !
140. Ans. is .d' i.e., Absence of flow rrurmurs over scapular region lRef Herrisan l*le p. 1462; l6e/e p' 1387;
O.P" Gh*i }tu/e P. 432 & Ple P" a20|
:.!:A:t ::, :i. ii:..:;.. :: : :i :.
:1+:q::'i::1:1!rr . lr'

l4r. Ans. is ? i.e., Rheumatic carditis [Rel 0.p. Ghai 8*/e p. 436 6 fl/e p. 380, 372)
o Cardiac failure causes cardiomegaly
o MC cause of cardiac failure in children (elder than infants) is rheumatic
fever and rheumatic heart disease.
142. Ans. is'c'i.e., 0.04 - 0.06 [Re, O.p. Ghai B*/e p. 3gg 6,fl/e p. 372]
o Premature and neonates -)
0.04 mg/kg
o 1 months to 1 year -)
0.08 mg/kg
r Between 1 to 3 years -)
0.06 mg/kg
r Above 3 years -+
0.04 mg/kg

143. Ans. is'b'i.e., Aortic stenosis;'c'i.e., Petent ductus artriosus {Re! iulyungKpark Sn/e p. 463, 6641
r ASD usually does not cause HF.
o AS & PDA increase the strain on left ventricle _) LVF
r PS increases the strain on right ventricular failure _+ RVF

l44. An* is'b'i.e., coxsackie B IR,J o.p. Ghai Bn/e p. 442 & fl/e p. a2sl
o The commonest cause of myocarditis is coxsackie B infegtion, which
occurs any where from the age of a few hours
to 7 weeks, with a peak around two weeks. ' '
145. Ans. is'c',i.e., Endocarditis tRef o. p, Ghai g*le p. 436 & */e p. 3s0-3s21
146. Ans. is 'b'i.e., Arthritis [Ref park 2r$/e p. t7s; o.p. Ghai g&/e p. 436, 43zl
I There is no residual damage in the joint affected by rheumatic arthritis and there is complete resolution.

t47. Ans. is'c'i.e., 95/50 [Rel O.p. Ghai Be/e p. 452, 453 & */e p. a31)
148. Ans. is h'i.e., Cardiac deformities fRef: Has been etplained)
o It causes Ebstein anomaly.

lIr

,
T3 Tf }-} Y T} ,q r}"-r ft m q r
ff..n*rtgqfi." t LjgE x
{-r

qYqTFh,,g
!--, -:. LJ A t_r"

AIRWAY OBSTRUCTION

c Children (especially infants) are more prone of upper airway obstruction because :-
i) Airway is small and narrow(Pclee) .
ii) Proportionately larger head and tongue(DNB rs), the tongue is often pushed against the palate.
iii) Lar,vnx is anterior and cephalad(DNBrs) with anterior inclination. Larynx extendsfrorn Crto Coveyteb'rsl
oo).
leyelecl
iv) Vocal cord is at Crlevel and Subglottic regian is the narrawest portion of laryfix@NB ls'PGt0)"

v) Epiglottis is large@Ns te ohd o*"go shayteflQct oo;.

vi) Trachea bifurcates at T, or T ulevel@Nn ts).

lmoportant signs of respiratory tract obstruction


A) Wheezing
o Wheezing referes to high pitch whistling sounds audible without auscultation by the stethoscope.
o It is produc edby lower airway (bronchi and bronchioles) partial obstruction -+ Sufficient air must flow throughthe
narrowed airway to produce the wheezing.
r Common causes of wheezing in children :-
I Lower respiratory tract infections
r Loeffler's syndrome
Inhaled foreign bodies(PGl 06' 05)

r Bronchiolitis & Bronchial asthma@G106 0s)

r Hypersensitivity pneumonitis
r Cystic fibrosis.
r External compression from enlarged mediastinal lymph nodes, cysts or tumor.
r Tropical eosinophilia
ea*rrnn'5 Respiratory System

B) Stridor
r Stridor refers to the physicai finding of excessively n'isy breatnirrg a,cl is ge*eral due to airway obstruction.

lnfecticns
Congehital
Croup (laryngotracheobronchitis)
Laf yn goma lac ia(Att
tt4s 8a, PG] /s)
Subglottic hemangiorna
Epiglottitis Vocal cord palsy
Laryngeal papillornas
Bacterial tracheitis
Subglottic stenosis
2-3 months
Retropharyn geai abscess
vascula r ring{ArMs 8a, PGt 7s)
6=7 mOnthsrP6/ee

Most camman cause of stridor in infant and


children is laryngomalacia{An3,pctast.
Foreign body aspiration
rlnfants and toddlers use their niouths to explore
their surroundings. so, they are very prone
to aspirate a l.oreign body.
The most common age group ts 6 months
ta 4 years"
I Azlost common foreign bo<ly aspkated is ruufs (pet#iuts)e*,",t a7).
s Most airway foreign bodies rodge in
s bronchus (right more trtaru refls.ru,s se..
I Typically' the child presents with acute onset of coughecls3) (most-comm0?{cupc,E ,e)),
chacking, gagging, rvheezing,
respiratory distress, aphonia, draoling and stridct"( pcr s3).

Treatment
1) Foreign body in upper respiratory tract
I Foreigr1 bodv can be removed by Abdorninal thurst (Heirnlich e{))
maneuver('l, or Magill forceps.
2i Foreign body in lower respiratory tract
I The treatment of choice is prompt removai
of foreign body by rigid bronchoseope (endoscope)(r\..trx Are7,

e It is the mast camtman congenital abnormality


of the larynx?'(;r08,Dp(;08. lhorkhallll 01,AF,2).Laryngomalacia
is the r,losf
ftequent cause of stridor or naisy breathing in infant!,.rot, eilNrs es,pG 02, es, up 01),
r It occurs as a result ofa floppyportion ofthe larynx (in supraglottic
larynx) that iras not yet developed the strength
rrovide rigid support to the airwa1,. to

* $ttring inspirstion ne|'*i ive presslire is created through lary* , l'hich rcs*lts i' a collapse of these structures into the
aii:lvay and a narrolvel. breatlr ii-lg passage.
c Partial obstruction is the source of ihe noise
r'vith breathing (stridor), and sometimes
clranosis.
o Tie halimark sigu lricludes interwiilee*
siridorn.icstly in inspiratiane.;r02,Apa2).It is
usually rnare praruinent when
the. infant is lying on kis/her back (supiwe
pasitiafl c.02' tt16rkhaillt0a)), crying,
feetling, excitetl or has a cold. stridor gets
relieved an pl^citug tke patient in prane positianeGl att' 02' ltiarkhattt 01),
This is usually first noticed in the
*-f lifs{te azl . first few weeks
I ii :nay
worsen over the first ferv months and becorne
loucler. Tnis is irecause as the baby
grolvs, inspiratorl, force is
grealrr' wlrich canses greaier coilapse of the larvngeai
st*lr:t*res into the airway'. vhis is nsually
worst at 3-6months
and then gradually improves as the rigidity
of the cartirage iniproves.
o Mast children are sy?nptsflt
free by I ta Z yeflrse,Gr aB ) .
o Sonreiinres, cyanosistApa2) ntav occur.
e i-)irect larvngoscopy sholl s :_

t amega sha'ped epiglatt;s@Gt 08 02' triiu* 06)


)i.e. ercngated a*ri curleer om !f sslfi oir,ar oo1.
l Floop,;, tall, toreshortenerj aird thin arvepiglottic
fcicis.
r .Llrominent arytenoids.

' iarvngomalacia is a s*lflirnitins eon{litio*(Jec'r.,8' {'}r).


Treatneenr oflaryngomalacia is
,11,::::::.:'icnts
':r:r1i ;r-1.1'ri[5,tnc
reassurance(up
t;rlr';r:lt,l.inr;L- rrl.r.al) r. r :r);)(,; rc5ri]Jtdi\ ir,,it
irrlecri,r,s.
Laryn giotracheobronchitis {Croupl
e Laryngotracheobronchitis is the most common infectious cause of obstruction of lawer respiratary tract\I ls)
in child.ren
usually occurring between the ages of 6 months and 3 years.
e Male children (boys) are characteristically more frequently involyed than females (girls){u t0t.
o CroupisaViralinfectionqlse)cansedbyparainfluenzaVirustypeT(mostcommon)@rlMs83'Pcr80),RSYinfluenzavirus,
83, PGI80)
adenovirus and rhinovirusAllMs
e There is fuistory of anupperreg)iratory tractinfectionfor 1-3 days{A'o') (e.g. rhinorrhea, mild cough, fever, pharyngitis)
followed which there are signs of obstruction like hoarsene ss, crortpy cough/barking cough(Ar ls) (Barking seal or Dog)
(AIIMS 08' AI 07' PGI oo), o0).
and inspiratory stridor(Ar 07' PGr
Intially stridor is seen on cryinglArrMs o8),
Drooling and dysphagid
ar e char act er i stic ally ab s ent.
o Symptoms are characteristically worse at night and are aggravated by agitation and crying.
r X-ray neck shows steeple sign(uP0s' sGPGr0'.

o Mild cases (i.e. no stridor at rest but seen on crying or activity) require only supportive therapy (hydratiorciP$I
00),humidified oxygen@Groo)) and minimal handling. Moderate to severe cases (stridor at rest) are treateci i:.1r
axygen@etoo), hydration withJluidsecloo), corticosteroids, i.e. high dase dexamethasone (drug of chaice@ilMsa8)), a*:.1

nebulized epinephrine. Antibiotics have no roleecloo),

Epigloftitis
o It is the inflammation of epiglottis with inflammatory edema of hyopharynx. It is a true medical emergency.
e H.influenzae is the most common causative organismallMs 0s, 04, e4, PGr 08, NEET).
Other organisms are -) Pneumococci,
Streptococcuspyogenes, N.meningitidis, Staphylococcus

Clinical manifestations
o Onset is sudden. Symptoms are -) fever, dysphagia(Pcrost, drooling(pcros), muffled voice, inspiratory retractions, cyanosis
and soft stridor. Patients often sit in snffing dog position. Respiratory arrest may occur.

'The risk of sudden death for persons is high due to sudden airway obstructiono(DPce6).

Plain lateral soft tissue radiograph of neck


e Thickening of the epiglottis - the thumb sign(Pcr0&04)
o Absence of a deep well defined vallecula - the vallecula sign

Treatrnent of epiglottitis
e Once the diagnosis of epiglottitis is made, endotracheal intubation must be performed immediately.
e After an airway is established, cultures of the blood and epiglotiis should be obtained and patient started on appropriate
intravenous antibiotics to cover H. influenzae (Ceftriaxone or equivalent cephalosporins). "Third generation
cephalosporins are prefeneil as first line agents because of incieasing resistance to ampicillin. Cefiriaxone is the
08) D.
treatment of.choice for epiglottitisecl
c Other antibotic options are :-
t Ampicillin plus sulbactam t Cefuroxime : Clindamycin
La ryn giotracheobronchitis (Croupl
o Laryngotracheobronchitis is the most common infectious cause of obstruction of lower respiratary tract@Ils) i?i child.ren

usually occurring between the ages of 6 months and 3 years.


o Male children (boys) are characteristically more frequently involved than females (girls)@l10).
r CroupisaViralinfection(A'8e)catsedbyparainfluenzaVirustypel(mostcontmon)(urus83'Pc18a),RSVinfluenzal'irus,
83' PGr 80).
adenovirus and rhinovirus(AttMs
o ThereishistoryofanWerregtiratorytractinfectionforl-3days@Io7) (e.g.rhinorrhea,mildcough,fever,pharyngitis)
ts) (Barking seal or Dog)
followed which there are signs of obstruction like hoarsene ss, croupy cough/barking cough@I
(AIIMS 08' At 07' PGt oo),
and inspiratory stridorql 07' PGr 00).,
Intially stridor is seen on crying!4llMs o8).
Drooling and dysph.agia
are characteri stically ab sent.
o Symptoms are characteristically worse at night and are aggravated by agitation and crying.
r X-ray neck shows steeple sign(uP0t sGPGr0I).
o Mild cases (i.e. no stridor at rest but seen on crying or activity) require only supportive therapy (hydratirm(PGi
00),humidified oxygenqeloo)) and minimal handling. Moderate to severe cases (stridor at rest) are treated i;1,,
Oxygen@eroo), hyilration withJluids{PGI00), corticosteroids, i.e. high ilose dexqmethqsane (drug of chaice@I;I4s$8)}, ar,i,

nebulized epinephrine. Antibiotics have no role@G100).

\ns. is d'i.e-, Croup


'', 'i.:',,'Thedii,enl,nec-k:X-ray,is,s}oinin$taperingql!:i$:glpt-tieaitwayiiiaehea-+,Steeple9l lri1{ine.bottlgsign
(characteristic of croup).

Epiglottitis
r It is the inflammation of epiglottis with inflammatory edema of hyopharynx. It is a true medical emergency.
c H.influenzae is the most common causative organismqrrus 0s' 04' e4' PGI08' NEE'),
Other organisms are -) Pneumococci,
Streptococcuspyogenes, N.meningitidis, Staphylococcus

Clinical manifestations
e Onset is suilden. Symptoms are -.> fever, dysphagia@Gr08l droolingrPcr0s), muffled voice, inspiratory retractions, cyanosis
and soft stridor. Patients often sit in snffing dog position. Respiratory arrest may occur.

'The risk of sudden deatlyfor persons is high due to sudden airway abstruction"tDre%) .

Plain lateral soft tissue radiograph of neck


o Thickening of the epiglottis - the thumb sign@Gro& 04)

r Absence of a deep well defined vallecula - the vallecula sign

Treatment of epiglottitis
r Once the diagnosis of epiglottitis is made, endotracheal intubation must be performed immediately.
o After an airway is established, cultures ofthe blood and epiglottis should be obtained and patient started on appropriate
intravenous antibiotics to cover H. influenzae (Ceftriaxone or equivalent cephalosporins). "Third generation
cephalosporins are preferred as first line agents because of increasing resistance to ampicillin. Cefiriaxone is the
u.
treatment of choice for epiglottitistPGl 0e)

r Other antibotic options are :-


t Ampicillin plus sulbactam t Cefuroxime - Clindamycin
Laryngiotracheobronchitis fCroup]
o Laryngotracheobronchitis is the most common infectious cause of obstruction of lawer respiratory trsctot ls)
i?1 children
usually occurring between the ages of 6 months and 3 years.
o Male children (boys) are characteristically more frequently involved than females (girls)6t t0t.
e CroupisaViralinfection1lse)cassedbyparainfluenzaVirustypeT(mostcommofl)(AilMs83'Pcr8o,RSV,influenzavirus,
83' PGI 80).
adenovirus and rhinovims4llMs
o Thereishistoryofanupperrespiratorytractinfectionforl-3daysalq7) (e.g.rhinorrhea,mildcough,fever,pharyngitis)
followed which there are signs of obstruction like hoarsene ss, croupy cough/barking cough(Al ls) (Barking seal ar I')og)
(AIIM908'A107'PGIoo),
andinspiratory strido/Ar07,PGro0), Intially stridor is seen on cryinglAllMsos). Drooling and dysph-agic
are ch ar act eri sti c ally ab s e nt.
o Symptoms are characteristically worse at night and are aggravated by agitation and crying.
r X-ray neck shows steeple sign(uP05 sGPGro'.

o Mild cases (i.e. no stridor at rest but seen on crying or activity) require only supportive therapy (hydrati{}?i(P<;'
00),
humidified oxygen(Petoo)) and minimal handling. Moderate to severe cases (stridor at rest) are treated tr;:;
oo),
OxygenQcr hydration with fluids{Pct 00), corticosteroids, i.e. high dose dexamethasone (drug of chcfce(rl"Ms 031
J, ani,
nebulized epinephrine. Antibiotics have no role(PGI00).

a) Epiglottitis
b) Foreign body aspiration
c) Pneumonia
4.,
$.re1ii11p],.:1,,,4;;' .,; ;,,;,.

s_rg4 oi rWine- bottle sign

Epiglottitis
e It is the inflammation of epiglottis with inflammatory edema of hyopharynx. It is a true medical emergency.
0s' 04' e4' PGt 08' NEET).
o H.influenzae is the most common causatiye organism4llt'rs Other organisms are -) Pneumococci,
Streptococcuspyogenes, N.meningitidis, Staphylococcus

Clinical manifestations
c Onsetissudden.Symptomsare-+fever,dysphagia(Pcr08),drooling,Petott,muffledvoice,inspiratoryretractions,cyanosis
and soft stridor. Patients often sit in snffing dog ltosition. Respiratory arrest may occur.

"The risk ofsudden deoth for personsis high dueto sudden airwoy obstruction'4DPcs5).

Plain lateral soft tissue radiograph of neck


e Thickening of the epiglottis - the thumb or)
sign@croa
e Absence of a deep well defined vallecula - the vallecula sign

Treatment of epiglottitis
o Once the diagnosis of epiglottitis is made, endotracheal intubation must be performed immediately.
e hfter an airway is established, cultures of the blood and epigiottis should be obtained and patient started on appropriate
intravenous antibiotics to cover H. influenzae (Ceftriaxone or equivalent cephalosporins). "Third generation
cephalosporins are preferred as first line agents because of increasing resistance to ampicillin. Ceftriaxone is the
0s)".
treatment of choice for epiglottitis{Pcl
o Other antibotic options are :-
t Ampicillin plus sulbactam s Cefuroxime t Clindamycin
,:;:5:,' Ri s iimt o ry Syste m

a Chloramphenicoluipnete3) a Cefotaxime
e Intravenous antibiotics should be continued for 2-3 days, followed by oral antibiotics to complete a 10 days course

A child with airway obetruction showing following X-ray"


Diagnosis is -

a) Epiglottitis
b) Foreign bodyaspiration
c) Pneumonia
d) Croup

,{ns. is ? i.e", Epiglottitis


o The X-ray neck is showing Vallecula sign (V sign) and thumb sign, characteristic of acute'epiglottitis.
o Vallecula is the pre-epiglottic space occupied by pocket ofair found at the level ofhyoid bone, just anterior
to epigloftis. The Vallecular sign is present when, instead ofdeep linear space, a shallow V shaped space is
seen. See foltowing normal X-rays & with epiglottitis.

Bronchiolitis
o Bronchiolitis is the most common serious acute lower respiratory tract infection in infants and young children.
o Most mlnerable group is between the ages of I and 6 months@Gr02'01'e8). Bvt the disease can affect children up to 2
years. More common in males. Bronchiolitis is predominarrtly a viral ilisease.
i) Rsy (most common)@NB 1s' AI o<'s+' AIIM9 00' PGI0s)

ii) Parainfluenza virus 3, 1, 2


iii) Adenovirus
iv) Influenza virus
v) l\lycoplasmapneumoniae
o There is history of upper respiratory tract infection (iever, rhinorrhea, cough) for l-2 days. Followed which there is
wheezing(Per 03' 01' AIIMS 00), cyanosis, tachlpnea, respiratory {lsf1ss5(rcr o:, A[Ms 00) and dysnea.
o On examination there are rales, crepitation and ronchi. Liver and spleen are pushed downwards due to hyperinflation
(emphysematous) of lung(PGI03'01). For the same reason there is hyperresonance on percussion and anteroposterior
diameter of chest is increased(Pcr03'0r).
(PGr03).
a Leukocytecount is normal or slightly elevated Chest X-ray shows hyperinJlation of lunglPcl03, 0r), downward
displacement of diaphragm and hypertranslucent lung fieids.
o Complication: Chronic airway hlperreactivity (asthma(pcrot)) may develop.
o Treatment is mainly symptomatic(pcl 02) which includes humid atmosphere, Or, bronchodilators (B-agonist,
ipratromium, epinephrine nebulized) and antipyretics. Though antibiotics have no rclq ribavarin, when indicated,
is the antiviral agent of choice@EE'L" DNB 12' AIIM9 01' PGI eB) ,
Stridor Wheezing

. lnfant & children . 1-5 years '. 1-3 years . . 1-6 months
. 6 months to 4 years
' lntermittent H/O upper respiratory Sudden onset . Violent paroxysm off ' H/O rhinorrhea (URTI)
inspiratory stridor tract infection . Severe respiratory . Wheezing
. lnspiratory stridor Coughing .
. No other complaint distress Respiratory distress
. Barking (Croupy) cough . Sore throat, Chocking . Chest crepitation
Gagging
dysphagia . Chest x-ray-)
I I
Wheeze
I
I I Aphonia Hyperinflation
I I I
I Stridor (incomplete V
I

+ + + obstruction)
Laryngomaiacia Croup Acute epiglottitis Bronchiolitis
I
Y
Foreign body aspiration

Bronchiolitis obliterance
o Bronchiolitis obliterans is a rare chronic lung disease of the bronchioles and smaller airways.
o Bronchiolitis obliterans most commonly occurs il ped.iatric population after respiratory infections (i.e. adenovirus,
mycoplasma, measles, influenza, pertussis).
r There is respiratory obstruction due to inflammation and fibrosis.
o Cough, fever, cyanosis and respiratory distress followed by initial improvemen{At 11' AItMs 08' oz) maY be the initial signs
of bronchiolitis obliterans. Progression of the disease may ensue with increasingdyspnoea, cough, sputum production
and wheezing.
o Chest radiographs may be relatively normal compared with the extent of physical findings but may demonstrate
hylterluscency@l11,AlIMs08,07) and,patchy infiltrates (occasionally, a swyer Iames syndrome, i.e. a unilateral hlperluscent
lung has developed).
11' AIIMS 08'
o Pulmonary function tests demonstrate varaible findings but qpically shows signs of airway obstructionqr
07).

o Ventilation perfusion scan shows a tlpical motheaten appearance of multiple matched defects in ventilation and
perfusion.

PNEUMONIA
o Pneumonia is an inflammation of the parenchyma of lungs, and mostly caused by bacterial or viral infection.
o Most common cause of paediatric pneumonia is respiratory syncytial virus (RSV)(DNB 12' Ar 04) . Other viruses causing
pneumonia are influenza yirtts\t1a) (2d most common virus), adenovirus, rhinovirus, and parainfluenza virus.
AIes).
o Most common bacterial cause of lteiliatric pneumonia is streptococus pneumoniae (pneumococcus)al Bacteria
1s'

causing atypical pneumonia commonly are mycol,lasma and chlamydia.


r pneumonia is often preceded by symptom of upper respiratory tract infection like cough and rhinorrhea. This is
followed by fever (high in bacterial pneumonia), tachlpnea, cyanosis and retraction of respiratory muscles. Tadrypnea
is the most consistent manifestation.
o Staphylococcal pneumonia is more commonly associated pneumatocele\I t3) (multiple thin walled air containing
cysts@IIMS03)), lung abscess, empyema@ttMs"'), pneumothorax(uP00) and purulent pericarditis. There may be positive

history of antecedent skin infection (pyoderma)u'nusos), (Note : Pneumatocele may also be caused by E,coli and
kleb s i ella pneumoni a).
o Measles causes primary giant cell (Hecht's pneumonia)(Arr3).
o Organism complicating pneumonia with lung abscess are staphylococcus aureusLrtr), H. influenzae,
er.),
S, pneum o ni arrar p seudomonas, and kl eb si ella@I e1 )
.

Tratment
o For mildly ill patient (not require hospitalization)
r Emperical treatment + Amoxicillin is the DOC.
t Atypical (mycoplasma) pneumonia -+ macrolides (Azithromycin, Erythromycin)Gress.
o For Hospitalized patients
r IV cefuroxime or cefotaxime or celtriaxone.
r If clinical features suggest staphylococcal pneumonia, vancomycin or clindamycin are also added.

'A cldld twtth.pneumohia has fullowlng chest,X-ralr'Mqit


llkelyeausatryeorganisrn'1t- : :.' : ,

a) Str. pneumoniae
b) Slaph.4ureus'
c) Str. pyogenesl
d) tisteria,,:

Ans.'islbtie,,St4phaureus ,:r, ' ' .,: .', '

r The given chest X-rarr is showing laige pn€umatoeele in right lung,:,+ characteristic rof giap! aureus pneu
monia.

BRONCHIAL ASTHMA
o Bronchial asthma is a disease characterized by arn increased resltonsiveness of airway.s to a yariety of stimulus

and causes recurrent attacks of wheezing, breathlessness, chest tightness and cough.
r Thesymptomsareduetobronchoconstrictionwhichisreversibleinalargemajorityofthecases,eitherspontaneously
or in response to treatment. Asthma is the corumonest chronic illness during childhood.
o Childhood asthma usually improves with agsailMsst).
o Asthma is type-l hyersensitivity and is mediated by IgEazus ea). Usually there is positive family history of asthma or
other atoltic iliseases (atopic dermatitis, allergic rhintis)Auus s<1.
Clinical presentation
e Most characteristic clinical feature of asthma is wheezinglAl 13' e7).
Other clnical features are cough, dysnealt sz),

tachycardia, apprehension, restlessness, cyanosis@te7), fatigue, hJper-resonant chest and pulsus paradoxus. In chronic
cases, chest becomes barrel shaped(tl e3). Clubbing of finger is not seen\r e7).

Treatment
o Treatment of asthma can be divided into :-
A) Treatment of Acute Asthma
1) Mild attack
s Inhaledshortacting$r-agonistlikesalbutamol(abuterol)isthedrugofchoice@||7).Ifthereisnosatisfactory
response, inhaled antichlolinergic (ipratropiura) is added.
rChiidren refractory to inhalation therapy are treated by intravenous aminophylline (theophylline@IlMs s7)).

2) Acute severe atack (status asthmaticus)


't First step is to give Orinhalation(ArrMs02). Inhaled short actingFr-agonist (salbutamol/albuterol) is the
drugs of choice(Al07), and are given along with systemic corticosteroidsallMs s7) (hydrocortisone).

ffi
r Other drug which intravenous aminophylline
(theophylline) and in

B) Treatment of chronic asthma (persistent asthma)


: Inhaled corticosteroids are the drugs of choice (budesonideeclee), beclomethasone, fluticasone). This is because
regular uses of Fr-agonist causestoleranceedee).Intermittent use of inhaled short acting Br-agonist is indicated,
as needed for symptoms.
r Other drugs used are sodium chromoglycate, nedocromil, long acting inhaled Fr-agonists, and leukotrine
antagonists
r For exercise induced asthma, treatment immediately prior to vigorous activity or exercise is usually effective. The
medications most widely usedfor exercise induced asthma are short-andlong-actingbeta agonistsNEEr). Alternative
drugs are sodium chromoglycate, nedocromil and leukotrine antagonists.

cYsTtc FrBRoslS (MUCOVISCIDOSIS)


o Cystic fibrosis is an autosomal recessive\ItMs 11' PGI06) disorder of ion transport in epthelial cells that affect Jluid
secretion in exocrine glands and the epithelial lining of the resl,iratory, gastrointestinal and reproductive tracts.
o Cystic fibrosis occurs due to mutation in the CFTR (Cystic fibrosis transmembrane regulator,)ailMs 11) gene on long
armecr B) of chromosome-7 ed 06), CFTR was initially recognized as chloride conductance channel but it is now
recognized that CFTR can regulate multiple ion chanruels and cellular processes. These include : Chloride channels,
Potassium channels, Gap junction channels, Bicarbonate ions.
o Most CF patients have 3 ilistinct abnormal charactetistics.
1) The ducts of the mucus-secreting glands are obstructed due to an inuease in viscosity of these secretions leading to
glandular dilatation and destruction.
2) CF patients are prone to chronic bacterial colonization and infections.
3) The sweat glands are not obstructed in CF patients because in serous glands such as sweat glands there are
1s).
abnormal concentrations of inorganic ions, rather than glandular obstruction with thick mucus(DNB

o Obstruction of airways leails to bronchiectasis and atelectasis; pancreatic duct obstruction leads to pancreatitis and
ts).
malabsorption; and plugging of bile ducts leads to obstructive jaundics@Nn
Clinical features
o Most patients with CF present in childhood. Approx l87o of patients present within the first 24 hr of life with
gastrointestinal obstruction, termed Meconium ile:us,MAHE 07).
o Other systems dysfunctions are not present at birth(Pcr0o, they may manifest within the first year or two of life
including respiratory tract system.
1) Lungs and sinus disease
r Sinusitis r Pulmonary hypertension -+ can cause heart failure
r Bronchiactasis r Allergic bronchopulmonary aspergillosis
r Nasal pol)?s r Lower respiratory tract infection
i Chronic lung disease
2) GIT, Liver and pancreas
t Meconium ileus@A$E 07) r Rectal prolapse
r Malabsorption r Pancreatitis
t C n stiP ati n(MAHE )
o o
07 r Intussusception
r Meconium peritonitis r Bile duct obstruction and biliary cirrhosis

3) Endocrine and growth


r Diabetes r Osteoporosis r Poor growth

4) Infertility
r Tn men due to absence of vas deference r In women due to thick cervical mucus.
Respiratory System

o Most common infecting organism is Pseudomonas aerugino.ur'4fi'usii),.epreciaiiy non-mucoid straip(NEEr).

Diagnosis
r The diagnosis is confirmed by demonstration of a high sweat chloride (>60 mEq/L) on repeated rneasurernents(Pcr
06 ),

c A normal sweat chloride dose not exclude the diagnosis.


e Genotyping and other measurement of nasal membrane potential differerrce, paflcreatic functian should
tests such as

be done if there is high clinical suspicion of cystic fibrosis\ilMs 11' 07) .

qONGENITAI. LE IQNS

Bronchoganic cysts
e Bronchogenic cyst is a congenital lung defect, which develops from abnormal lung budding of primitive foregut.
* Most of the bronchogenic cyst arise in the mediastinum, where they are usually located in the mid mediastinum near
carin.t6llMse6). About 15% of Bronchogenic cyst also occurs in pulmonary pdrenchyma(AttMseo). When located in the
lung they are more common in right lung and lower iobe.
I They are lined by cuboidal or ciliated columnar epithelial cells and are usually f,lled with mucoid material"
o Infection of the cystleadin{utttsoel b lung abscess (Mediastinal bronchogenic cysts do not communicate with the bronchi
but those situated within the lung may communicate with the airways leading to abscess formation).
r Compression can produce either hyperinJlation of the lung or atelactasis due to obstru ction. Tension pneumothorax
(due to rupture of cyst that communicates with bronchus) may occur.

Pulmonary sequestration
o In this congenital anomaly, there is a discrete mass of lung tis sue without any mormal connectian to the airway system.
As there is no connection to airway, the lesion does not fill on bronchography4t ou.
r It may be :-
i) Intrapulmonary (intralobar) -+ More common
ii) Extrapulmonary (extralobar)
o Sequestrated tissue receives its arterial supply from the systemic arteries (rnost cawmonly aorta)6t se).
. it returns blood to the right side of heart throuch IVC (extrapulmanary seqwestratian) ar tr'sulmonary veins
(intrapulmonar y s e questration)Qct ss t
.

c The sequestration functions as a space-occupyinglesian within chest, it dcles not function in gas exchange, and clces
nQt contribute to a ieft to right shunt or dead space.
e GaQc or pancreatic tissue may also be found in sequestrated segment.
e Associated anomalies -+ cystic adenomatoid malformation, congenital diaphragmatic rl.er.ia, esophageal cvsts.

Macleod syndrome
o It is also krrown as Swyer - lanes slndromseuqs).
o It is a cause of unilaterfll?Gl0s) hyperlucent lung. It may present as acute pneumonia, but most patients are discovered
when a chest X-ray is obtained for an unrelated reason.
o This is not a true etnphysema@Glqst.
o On examination there is hyperresonance and a small lung with rnediastinum shifted towards the more abnormal lung.
o There is no vascular a.brxormality(Pcl0s).
r ldo specific treaiment is required, it may become iess symptorlatic with

Congenital Gystic adenomatoid malformation of lung (CCAM)


. CCAM is a developmental defect of terminal bronchioles. CCAM is divided into 5 types :
1) CCAM-0 (acinar dysplasia): Incompatible with life.
2) CCAM-I t It is most cantntan type and has best ltrognosis, Cysts space are lined by pseudostratified ciiiated
coiumnar epithelium.
3) CCAM -2 : It is 2d ntast covwmofl type and causes respiratory distress it firsi month of Ii is associarted with
renal agenesis, cordiovascular defects, diaphragwatichernia, and syringamyelie"
Cn.rp r, * . **roooxto,Y sf iiin
-*iere is exPansion of a 'imal shiii ttr;,
+) CCAM-3: fi occitrs exclusivei,v i\,[ales.

comPressian of other lung'


u'aiied c1'sts are
5) CC,4M-4: It is very rare" l'{uitilocuiated'

: '-
: r;1. 1:..:.,:.,:r... , : .,.'ggtrligARyof CF;APTERl1l ONELIf,iERStQLIlfK'REVtSOfitf'anmfE'FATTEf$Il. ' : ': '
iarge epigiotiis' larynx extends from C' tc C ''
lfilpartantfeatures of" neonatal
:'"" airtuay., t*''i &* narrort//
^'"'^''" ' Smail
'
^""'' larqe iongue, ornega Snraped &
narrowest, more compliant chest wall'
";;;r;-ir', region
suLglottic
hypersensitrvity pneumonitis, external compression
by
,oru*, o,r *nun in chir6ren: F.sthnra. foreign body,, lnfection,
mediastinal lYmPhadenoPathY'
cord pa!sy, subglottic hemangioma
epigicttitis, laryngomalacia, vascular ring, vocal
lmportant causes of stridor in children: Croup,
or stenosis.
and yaung ch i/dren : LarYnomalacia'
Most common cause of stridor in infant
Laryngonralacia'
Mast common congenital abnormality of lerynx:
prominent in supine position and rerieved i, prone
position'
;;;;;;r rr;;;;;r**oto,r,n ,lnspiratory inrermittenr srridor more

l Onega shaped epigtottis is seen in: Laryngomalacia'


self rii]lliifl,J ;cndition'
thatIt it is a selt
Treatment a{ laryrtgomalacia:Only reasssurance "-:t"'.1
due
I
ttl:
::t::-
ior,eign lrrci'1. asrriration.
in a child is mostly
Acute anset of cough, dyspnea and stridor
;;;;;;;'';';;-:;;.;-;,,;'hiailroreisnbcdvinachird:cor'sh removal'
g lmmediate managenent of foreign loclt asoiraticn
in lov'rcr re:'pirctt:'r'v tract (e'g'lung/branchi): Rigid bronchoscopic
:, !l,4ost common cause *f stridor shortly a{ter
birfh : Laryngoma|aeia-

Sealbarkingtikecroupycoughischaracteristicof:Laryngotracheobronchitis(croup).
/pe 1.
Mostcommoncauseofcroup(laryngotracheobronchitis):Parainfluenzavirust,
adenovirus' rhinovirus
Other causes of croup:RSV influenza virus'
is aggravated by crying'
-3
presentation cfcroup: Hio URTI for 1 days
foliowed by barking cough, inspiratory stridor which
Typical
Teatment of choice for croup: Dexamethasone
(helium plus O')'
hyd ration' nebulized epinephrine' heliox
ather measures in treatrnent of croup :Oxygen'
Drugswhichhavenoroleincroup:Antibiotics,sedatives,opioids(morphine).
;e X-ray sign of crouP: SteePle sign
@ Most common cause of epiglottitis: !l' influenzae
w Rladiologica! signs af epiglattitis:Thumb sign' vallecula sign'
e Most common cause of bronchiolrtis : RSV
. on ches t X-raY is seer; i;': : Bronchiolitis'
* Dyspnea, wheezing, bilateral crepitation with bilateral hyperinflatian -o)
(drui* of aheieei'
anr;i-:l ret!rs, aerosoiixed ribavarin
,!, .rreatment af bronchioriiis incrudes: Hurnid atmosphere, or, bronchodirators,
I':used bv ' ''
a Aerosollzed ribavarin is drug of choice fcr: Acute irronchioiit:; ''
lviorphine'
Dr-g wiffi no role in treatment of bronchiotitis:
(mc cause) foilowed by influenza virus (as 2nd mc cause)'
; ;;:r rr;*ion cause of pneumonia in chirdren: RSV
pneumoniae (Pneumococcus)'
* commonest cause of bacterial pneumonia in children: Streptococcus
,,isr rc ,^onnrrirr rq parainfluenza virus, rhinovirus'
^-- virus, adeno.lirus, narainfluenza rhinovirus
g Virus causing pneumonia in children: RSV influenza
e Notrd caue of viral pneuittaitia: Mumps virus'
(mcst co r.nrnon), less commonry by E. coii and klebsieila.
* or"u*loirorrle is seen in pneur' oniawit'r :Staphylococcus aureus
* Prirnary giant cell pneumonia (llecht's pneumonia) is causeC by Measles'
"'

krebsie[a, E. cori, H.influenzae, pseudomonas'


* ,*o*rroT|r rourrr of r,og obscess :5taphyrococcus, pneuffioco€cus,
pn€urfiatoc'ele as Carnp!ications af pneumanio : Staphylccoccus.
e |,,4ost rcmmon causeai pneurnatharax, emphyema, lung abscess,
.caplcsrnapneumonia:Maerolides{erlAhrcrrrycin,azithr<rrnycin).
a ')ruq of choice lor mY
sf right apical iobe'
a Labecf lung ftlcst.arfrn1cni1t involved in aspiration: Postericl s*E!-fient :-
.featutes a{ chitdhaad asthma: flais*d igE' fanriiy'If:eisonill hisjtory cf aiopil cerrnat.itis, impioven"e"it !
r"' *z;r'fi ivr"i:
* lffipr{tct}t
.ri1, i aXgr;i54.

- riol i, fgolt)tgsrlh't!-thtctlaslrtntc;: Ret::; l!- i'j; '- 'r:liu"


14rh'"::ig
, .;r' :,rr,.,*, isli.: ci::,;r,:' ."GtL,re of 3sthft14
CHrp r en s ResPiratorYSYstem
'lhere is expansion of a -whoit: t ta): ai,t r;-llstirual shiit au
4) CCAht-3: It occurs exclusiveiy in iitales"
comPression oJ otfuer lwng'
walied cysts are P1'esent
5) CCAtrl-4: lt is very rare' '\'lullrlo'-rrllird'

CH4PTERIE{ 6NE,IINERS{QUICKRE1{I5iO$IfCIRS{EEFAT:TEftNii'' '':' -'


',.,t.."., :.: '.: SUMMARYOF r. .
.r.:
larynx extends from C. to C
narrow, large toncue, orrrega shaped & iarge epigiottis,
,

tfitportant features of neonatal airway:5mall &


subglottic region narrowest, more compliant chest
wa!l' ..
";o"r;rr:r; compression by
l

,niu,*, nuo*u, infection, hypersensitivitv pneumonitis, exterrral


foreign body,
riur*u or*nur)r' iin

#;;;.l-tmphadenoPathY'
ring, vocal cord palsy, subglottic hemangioma
lmportant cLuses of stridor in children: Croup, epiglottitis, iaryngornalacia, vascu|ar.
or stenosls.
Most common cause of stridor in infent and young children: Larynomalacia"
Laryngomalacia'
Most common congenital abnormality of laryn'x: posltlon'
in supine positicn ancr r'erieved in prone
;;;;;;,r r,r;;;;;r***)r,-,l , roiratory int"rmitt*nt ,t,i,*n, more prominent

Ontega shaped epiglottis is seen in: Laryngomalacia'


Treatrnentaflaryr,qomalacra:Onlyreasssurancethaiitisaself!ilr:!:ir,;ccrdition.
a chilrl is mosily due io : iore icr^, l.lrdy aspiraticn.
. Acute anset af ccugh, dyspnea and stridar irt

, Commonest sign af irrtrabronchia!fareign


baCy in ct child'Couqh'
'
louter resy2!rst*ry tract (e,g.lung/bronchi): Rigicl
bronchoscopic removal.
.' lmmediate nlan{tgo-!.ner1t a{ fareigc badlt as,iratian
in
,*:;r*;;;*-;,.j-r)"',-rl,u"oi"i"r',r'*'r'birth.Larvnsomalacia'
..sealbarkinglikecroupycoughischaracteristicof:Laryngotracheobronchitis(croup).
Parainfluenza virus tYPe 1
,.. Most common cause of croup laryngatracheobronchitis):
'

adenovirus' rhinovirus
Other causes of croup: RSV, influenza virus'
. by crying'
by barking cot)gh, inspiratory stridcr which is aggravated
, Typicol presentation of croup: H/o URTI for i -3 days foilowed

Teatment of choice for croup: Dexamethasone


rhrine' heliox (helium Plus O')'
:.Othermeasuresintreatrnentofcroup:Oxygen,hydration'nebulizedepinep
:':Drugswhichhavenoroleincraup:'{ntibiotics'seciatives'opioids(ri'rorphine)'
X-raY sign of crouP: Steeole sign

G Most common cause of epigtattitis: H' influenzae


' Radiological signs of epiglofriris :Thumb sign' vallecula slgn'
. Mast common cause of bronchiolitis:RSY
chest x-rrvt is see;: i;: : Bronchioritis'
; ;;;r;;,;,;r,,rrring, aiiutrrotr,rpitation with bilaterat hyperinfiation an choiee)'
antii:]'!ieiics, aerosolized rib*varin idrus of
* Trcatmentof bronchiolitisinc!udes: Hunnid atmosphere, or, bronchodilarors,
a -. .r^ L--^^-t.inr!t:: I-ar :<ori hrt I
,lAerasolizedribavarinisdrugofchoic.:fcr:Acutebrcnchioiiti:{lcusedt}v=11]'].
;; frrt:E with no role in treatmenl of bronchiolitis: Ivlorphine'
virus 2nd mc (as cause)'
RSV (mc cause) followed by influer,za
, Mcst common cause of pneumonlrs in children:
Streptococcus pneumoniae (Pneumococcus)'
*, tommonestcause of bacterial pneumonia in children:
rhinovirus'
- t/irus@usingpneum"rir,, "ri,i^rrr: RSV, influenza virus. adenorrirus, parainfluenza virus,

l\ot a iaue of vira! pneumoitia : Mumps virus'


and ktebsiella'
aureus (mo:t cornmon)' less commonly by E' col!
* oi.,rr*rrorrb ir *un, in pn"ur:,oniawith :staphylococcus
aPrimctrygiantcellpneumoniu{ilecht,spneumonia)iscausedby..Measles.
. tmportant
-.
of lLt:tg obscess: staphvlococc
us, pneumococcus. klebsiella,
t,'i,l E. coii, H influenzae, pseudornonas'
causes ..sraphyrococcus.
pneurnatoceie)'"',",'),,fr', ,'r')rZrrlrirr"manta
as compiications of pr
* lvlast cafllmon cause of pnettmatharax, emphyena' lunE Abscess'

pneumoni*l Macrolides (erf hroi-liycin' azithromycin)'


Drug of thoice far mycoplasrna

Lcbeaflungmost.arnfi.lanlyinvo|vedinaspiration:PosterisrSegmentofrightacicalloLle'

. rrl,i gleriise,

@ ilct c feature of chit'Jhoo'J asthnid : iiaisis :3G iiEE


i: ;'::rs':cl'
.if3r:,-:rr i:''tetisti:ci:r';i'Lt !eetureo{ asthna V hc.'.l 9'
,lCr,x,q:te l:5,
.:
Rcsp i rat o r y sy stem

a Not a feature of asthma: Clubbing


,* Drugof choicefor asthma in children:Shi"t acting inhaled p,_agonist. : :: ,

Drug o{ choice for exercise induced asthma: lnhaled pr-agonist.


a Drugs not uied intreatmentaf asthima : Se:datives, opioids.
* Typical presentation of bronchiolitis obliteran-s: Moderate fever,
mild dyspnea and nonproductive cough which improve initially,
later developing high fever, severe dyspnea, productive cough with liyperluceny but
on X-ray and obstructive pFT.
&

o Cystic fibrosis is :An autosomal recessive disorder due to mutation in cFTR gene on long arm of chromosome 7.
r* in,cyiticnuros*:Non-mucoid pseudomonas.
"Mottigmmbnlnfecting'organlsn
& Diagnostictest of choice in cysticfibrosis: Transepithelial nasal membrane potential
difference.
* Mucoviscidosisis most commonly associated wlth : Fibrocystic disease
of pancreas.
& True about bronchogenic cyst: Mostly in mediastinum near
carina, muliiloculated, often infected, only 15olo occur in lung.
.;Blqadsupply.of:{ungseg.qes|rytlon:Aorta..oritsbranches'..
?
& venous drainage of pulmonary sequestration: IVC (extrapulmonary
sequestration), pulmonary veins (intrapulmonary sequestration).
&'

* Components of Kartagener's syndrome : (i) Bronchiectasis, (ii) sinusitis, (iii) situs invertus, (iv) lnferttiility.
* KartagenerS''syndrAme:isasubgroipof,tprimaryiiliarydyskinesia., ..i' , ...... .,:..,r:
& Most common posterior mediastinar mass in chirdren : Neurogenic tumor.
*'. Mast cq m ma n eause of A:s ai M in c rt it d ren : srrep{o c o cc u s p n e u {n an i ae. :

& Most common cause of acute bronchitis (not bronchiolitis) in children : lnfluenza virus.
* Cyanisi5 and respiratory'syrnptoms improve on aying ; Choana! airesic.
* Most common pulmonary tumor in children: Carcinoid.

xan
QUE5TIONS

l. Pediatirc afuway difrer from adult _ @ET luly 15 pauern) c) Should be done
a) Large tongue d) None ofabove
b) Short epiglotitis
c) Narrowest part is glottis
d) Larynx in lower posittion LARYNGOMALACIA & CROUP
2. Leveloltrachea trifurcation in pediatic patient is-
11. A 3 month old child presents with intermittent
(CET luly 15 pattern)
stridor. Most likely cause is - (AI 01, AIIMS Dec 95)
a) T, b) r, a) Laryngotracheobronchitis
c) T, d) r, b) Laryngomalacia
3. True about upper airways of neonate_ @GI Dec 00) c) Respiratoryobstruction
a) Cricoid is narrowest part d) Foreign body aspiration
b) Larynx extend from Cn to Cu 12, Most common cause of stridor shortly after
c) Epiglottis is big & omega shiped
d) Atl
birth - (NEET Dec.t2 pattern)
a) Laryngeal papilloma
4. Child'srespiratoryphysiologydifiersfromadult b) Laryngeal web
because of - @GI June 99) c) Laryngomalacia
a) Smaller airways d) Vocal cord palsy
b) Increased O, demand 13. 2 week old neonate with history of stridor in supine
c) Decreased tidal volume
d) Decreased residual volume
position - what is treatment -
GET Jub) ts pattern)
a) Oral calcium
5. Most comnon cause of stridor in infant andyoung b) Nebulisation
children - @il India Dec.lj pattern) c) Wait &watch
a) Abductor palsy b) Croup d) Antibiotic
c) Laryngomalacia d) Epiglottitis t4. Steeple sign is seen in - (CET Nov 15 Pattern)
a) Acute epiglottis
FORf IGN BODY ASPIRATION b) Laryngotracheobronchitis (croup)
c) In both cases
6. Acute onset ofcouglr, stridor and dysponea in a d) None ofabove
child is mostly due to - ecr sj) 15. A child-with three days history of upper respiratory
a) Foreign body tract infection presents with stridor, which
b) Acute Asthma decreases on lying down postion. What is the most
c) Aspirationpneumonitis probable diagnosis - (Ar 07)
d) Primary complex a) Acute Epiglottitis
7. Immediate management of a child with foreign body b) Laryngotracheobronchitis
inhalation is - (All India Dec.14 pattern) c) Foreign body aspiration
a) IPPV d) Retropharyngeal abscess
b) Bronchoscopy
c) Tiacheostomy
16. False regarding croup is - (Al1 India Dec15 pauern)
a) Disease include epiglottis, laryngitis,
d) Exploratory Thoracotomy laryngotracheitis
8. In child, foreign trody in lung - (NEET Dec.12 pattern) b) Brassy cough is main presenting feature
a) Rigid bronchoscopy c) Causes upper airway obstruction
b) Chest x-ray d) AII of above
c) Flexible endoscopy 17. A2year old child is brought to emergency at 3 AM
d) Direct laryngoscopy with fever, barking cough and stridor only while
9. Commonest sign of intrabronchial foreign b6dy crying. The child was able to drink normally. On
in children is - examination respirator rate is 36lmin and
a) cough
/ (cuPGEE ee)
b) wheeze temprature is 39.6"C. What will be your next step -
c) Dyspnoea d) Srrfuor @IIMS Nov 08)
10. Swiping of oral cavity not to done in foreign body a) Racemic epinephrine neburization
ingestion in children because - b) High dose dexamethasone injection
(All India Dec15 pattern) c) Nasal wash for influenza or RSV
a) Oral cavity small, inability to clear d) Antibiotics and blood culture
b) Leads to in advertentlypushing offoreign body t8. A 4 yr old child has 'seal barking, like croupy cough.
deep in respiratory system Management includes A/E - @ci tune zooo)
a) O, inhalation c) Pneumococcus
b) Antibiotic d) Streptococcus
c) Hydration 27. In a child I years the commonest cause of resp.
d) Morphine Infection witl wheeze is - (PGt es)
a) RSV b) Influenza viurs
EPIGLOTTITIS & BRONCH IOtITIS c) Adenovirus d) para influenza
L9. Which of the following is the aetiological agent
PNEUMONIA
most often associated with Epiglottitis in
children -
28. The commonest cause of Bacterial pneumonia in
All India Dec.13 Pattern,AIlMS May
a) Streptococcuspneumoniae
05, Nov 04, May 94)
children is- All India Dec,14 pattern)
a) Streptococcuspyogenes
b) Haemophilus influenzae type b
b) Hemophilusinfluenzae
c) Neisseria sp c) Streptococcus pneumoniae
d) Moraxellacatarrhalis d) Staphylococcusaureus
20. The most common etiological agent for acute 29. Most common cause of lower respiratory tract infec-
bronchiolitis in infancy is -
tion in 3 year old child is (All India Dec15 Pattern)
(CET Aug.13 Pattern, At 06,94) a) Klebsella
a) Influenza virus b) H-influenza
b) Para influenza virus
c) Streptococcalpneumonia
c) Rhinovirus d) Staphe aureus
d) Respiratory syncytial virus
30. An infant develops cough and fever. The X-ray
21. A 6 months old baby coming with H/o increasing
examination is suggestive of broncho-pneumonia.
difficutty in breathing of
2 days duration and on All of the following viruses can be the causative
examination baby is afebrile & B/L wheeze & CXR
agent ex€ept - (Al 0,1)
shows B/L hyperinflation of the lungs with normal
a) Parainfluenzarriruses
I{BC count, the diagnosis is - (PGI Nov 14)
b) Influenza virus A
a) Bronchiolitis b) Asthma c) Respiratory syncltial virus
c) Ch. Bronchitis d) Pneumonia d) Mumps virus
e) F.B.
31. Most common cause of pneumonia in children is?
22- A l1 month old child pre$ents with complaints of
(CET Aug. 12 Pattern)
respiratory distress" On examination there is
a) RSV
bilateral crepitation and wheezing. Which of the
b) Streptococcus pneumonae
following is the nrost likely cause - (ArrMS Nov 2K) c) Staphylococcusaureus
a) Pneumonia d) Klebsiella
b) Adenovirus
32. A child with pyoderma becornes toxic and presents
c) Respiratory synchl'tial virus
d) Rhinovirus with respiratory distress. His chest radiograph
shows patc\ areas of consolidation and multiple
23. Treatrnent ofbronchiolitis includes all except -
bilateral thin walled air containing cysts. The most
(All India Dect5 Patternl
likely etiological agent in this case is-
a) Macrolides b) Humid oxygen
(AIIMS Nov 03)
c) Bronchodilator d) AII of above a) Mycobacterium tuberculosis
24. Treatment of choice in bronchiolitis in - b) Staphylococcus aureus
(NEEl'Dec.12, CET Nov. 12 Pattern) c) Mycobacterium avium intracellulare
a) Ribavirin b) Amantadine d) Pneumocystiscarinii
c) Vidarabine d) Zidorudine _ 33. Pneumatocele is caused by- (All tndia Dec.13 pattern)
25. A month old HlYpositive child following URTI a) Staphylococcus
developed sudden onset ofbreattlessne$$. The chest b) Streptococcus
x-ray shows hgrerinflation. The O2 saturation wa.s c) Streptococcus pneumoniae
greater than 90%. The treatment of choice A d) P. carnii
is- (AIIMS May 01) 34. Pneumothorax could be a complication of -
a) Cotrimoxazole b) Ribavarin (All India Dec.14 pattern)
c) IV Ganciclovir d) Nebulized Acyclovir a) Staphylococcalpneumonia
26, Aerosolized ribavirin is used in the treatment of b) Pneumococcal pneumonia
bronchiolitis with - (CE?'Nov.14 Pattern) c) Klebsiellapneumonia
a) RSV d) Viral pneumonia
b) H.influenza 35. The commonest organism causing Ernpyema in
::,::r::.ir;:ai:

a child under 2 years - (ArrMS 91) child with asthma is -


a) E.Coil a) Inhaled short acting beta-2 agonist
b) Staphylocccus b) Oral short acting theophylline
c) Pneumococcus c) Oral ketotifen
d) Klebsiella d) Leukotrieneantagonists
36. Giant cell (Hecht\) pneumonia is due to - 46. Child requiring repeated short acting
(All lndia Dec.13 Pattern) bronchodilators and what could be next line of
a) CMV b) Measles management -. ,
c) Malaria d) P.carinii a) Methylxanthines
a
37. The drug of choice in mycoplasma pneumonia in b) Short acting budesonide
children is - (AIes) c) Oral prednisolone
a) Tetracycline b) Streptomycin d) Montelukast
c) Cotrimoxazole d) Er1'thromycin 47, In a child with excercise induced asthma, which
is done -
38. The lung abscess in children can be causedbyall
(NEET Dec.12 Pattern)

except- 6I eL) a) Prophylaxis with steroids


a) Pneumococcus b) Prophylaxis with Beta, agonist
b) E. Histolltica c) Prophylaxis with theophyline
c) Staphylococcus d) Breathing excercise
d) Klebsiella 48. A child of less than one year with asthma treatment-
39. In an infant with aspiration pneumonitis, the most (All India Decl5 Pattern)

common lung segment tb be involved is - a) MDI with Spacer


(AIIMS May e3) b) MDI with Mask '
a) Left apical b) Right apical c) MDI with Spacer with Mask
c) Right middle d) Rightbasal d) MPI with mask

ASTHMA BRONCH IOLITIS OBLITERANS

40. NOT a feature of childhood asthma is - (AIMS Dec 94) 49. A 3 month old child has moderate fever and non
a) History of atopic dermatitis productive cough'and mild dyspnea. After course of
b) Raised lgG level mild antibiotlc the condition of the child improved
c) Improves with age transiently but he again develops high fever,
d) Absence ofwheezing after exercise productive cough and increased respiratory distress.
Chest X ray shows hlperluscency and PFT shows
41. Characteristic symptom of asthma -
obstructive pattern. Most probable diagnosis is -
(All India Dec.13 Pattern)
(AI 11, AIIMS May 07)
a) Clubbing b) Wheezing
a) Alveolarmicrolithiasis
c) Slridor d) Bradycardia
b) Post viral syndrome
42. All are seen in asthma except - (AI 97)
c) Follicularbronchitis
a) Cyanosis b) Wheezing d) Bronchiolitis obliterans
c) Clubbing d) Dysnea
50. A7 year old child presents with non productive
43. All are used in acute attack of asthma in a4 year old cough, mild stridor since 6 months. On oral
child except - @IIMS lune 97) antibiotics, patient is improving but suddenly
a) Theophylline develops wheezing, productive cough, mild fever. X-
b) Corticosteroids ray shows hyperlucency and PFT shows obstructive
c) Sedatives curve. Most probable diagnosis is -
d) Salbutamol (Al 11, ?9,AllMS Nov 08,07)

44. A 3- year old boy is brought to the casualty by his a) Post viral syndrome
niother with progressive shortness of breath for 1 b) Bronchiolitisobliterans
day. The child has history of bronchial asthma. On c) Follicularbronchitis
examination, the child is blue, gasping and d) Pulmonaryalveolar microlithlasis
unresponsive What will you like to do first -
(AIIMS Nov 02)
cYsTtc FtBRosrs
a) Intubate
b) Administer 100 % oxygen by mask 51. Which of the following exocrine glandular ducts are
c) Ventilate with bag and mask' not obstructed in cystic fibrosis - (CET luly 15 Pattern)
d) Administer nebulised salbutamol a) Pancreas
45. Most common mode of treatment of a I year old b) Lung
c) Sweat gland b) Pulmonarysequestration
d) All of above c) Cystic fibrosis
52. Infant with cystic fibrosis (cF) are likely to develop- d) Bronchopulmonarydysplasia
(All lndia Dec.14 Pattern) e) Bronchogenic cyst
- ''-a|-XQconium ileus
b) Loo\e motions MISCELLANEOUS
c) Vomiting
d) Constipation 50. A child present with recurrent sinusitis and
recurrent chest infections. Chest X-ray reveals
53. Which of the following statements about cystic
dextrocardia and situs invertus. The diagnosis is -
fibrosis (cF) is not true - @nMSMay 11)
(AIIMS Nov 11)
a) Autosomal recessive disorder a) Kartagener'ssyndrome
b) Abnormality in CFTR which leads to defective b) Good-pasture's syndrome
calcium transport c) Ehlers-Danlos syndrome
c) Mutation in cystic fibrosis transport regulator d) William Campbell syndrome
54. Most comrnon organism associated with cystic 01. Bronchitis in children is caused by - (PGI Nov 14)
fibrosis - (AIIMS May 11)
a) H. influenzae b) R.s.v.
a) Pseudomonas aeruginosa (non mucoid) c) Mycoplasma d) EBV
b) Burkholderia cepacia e) Influenza virus
c) Pleisomonas
62. In which disease, syrnptoms improve with
d) Aeromonas
crymg - (NEET Dec.12 Pattern)
55. 4 year old boy presented with recurrent chest a) Tetralogyoffallot b) Choanal atresia
infections. Sweat chloride test was done, showed c) Bronchial asthma d) All of above
values of 36 and 42. What is the next best
63. Most cornrnon cause of mass in posterior mediasti-
investigation to confirm the diagnosis?
(AIIMS May 11, Nov 07)
num in children - (PGl June 01)

a) 72hour fecal fat estimation a) Rhabdosarcoma

b) CT chest b) Dupiication cyst of oesphagus


c) Transepithelial nasal potential difference c) Lymphoma
d) DNA analysis of delta F 508 mutation d) Neuroblastoma
e) Thymoma
56. A child is brought to the paediatric OPD with fever
64. A girl child with fever, cough, dyspnoea with x-ray
of 24 hours duration. History reveals 3 episodes of
showing right lower lobe patchy consolidation, for
chest infection and passage of foul smellingXtools.
which treatment was given. After 8 weeks symptom
The most probable diagnosis is - (AI t2)
improved but x-ray showed more dense consoli-
a) Cystic Fibrosis
dation involving the whole of the right lower lobe.
b) Maple Syrup urine Disease
What is the next best line of investigation?
c) Bilirubin Congugation Defect
(AIIMS Nov 07)
d) Criggler Najar Syndrome
a) Bronchoscopy (AIIMSNov0T)
57. Mucoviscidosis is most commonly related to - (At e6)
b) Culture from nasopharynx
a) Fibrocystic disease ofpancreas c) Barium esophagogram
b) Duodenal atresia d) Allergic skin test
c) Diaphragmatic hemia ()5. A Primi mother having 3 weeks rnale child, which is
d) Annular pancreas
presenting with noisy breathing sound child is
afebrile, sleeping & feeding well & on clinical
CONGENITAL LUNG DEFECTS examination, He is normal. Management protocol
include - (PGl Dec 08)
58. Following are true about bronchial cyst except - a) Begin IV antibiotics
(AUMS e6)
b) Reassure & give saline nasal drop
a) Mostly mediastinal c) Give recemic epinephrine
b) 50-70o/o occur in lungs d) Order for a chest X-ray
c) Usuallymultiloculated e; Clve some decongestant
d) Are infected quite often
66' lVlost common pulmonary tumor in children is -
59. A 4 years child presents with a history of chronic a) Carcinoid (All India Dec 15 Pattern)
left lower lobe pneumonitis. On contrast b) Small cell carcinoma
bronchography, the area involved with the c) Adeno carcinoma
pneumonitis does not fill whereas the area around it d) Squamous cell carcinoma
does fill. The most likely diagnosis -
a) Asthma
III
ANSWERS
1. Ans. is 'a' i.e., Large tongue fRef, Miller's Anaesthesia 6h/e p. 2j8a)
o Infants have relatively larger tongue. Larynx is cephalad (higher) position with subglottic region being the narrowest
(not glottis). Epiglottis is large and omega shaped.
Ans. is 'H i.e., T ,{Ref: Textbook of pediatric physical examination susan sawyar p. 304)
o Bifurcation of trachea in children is at Trlevel while in adult at Tr- Toleuel.

3. Ans. is 'c' i.e., Epiglottis is big & omega shaped [Rel Morgan 3'd/e p. 849; Logan Turner tTth/e p. 382)
o Epiglottis is soft, large and patulous, Omega shaped and inclined at 450.
o The Subglottic region rather than the rima glottidis is the narrowest portion of the airway.
o Larynxlies opposite 2d, 3'd and 4th ceryical vertebra at birth.

4. Ans. is 'd i.e., Smaller airways lRef: Nelson 18tu/e p. 17191


o Younger infants are more I'ulnerable because : -
l) Smaller airways 11) More compliant chest wall 11i) Poor hypoxic drive
5. Ans. is t'i.e., LaryngomalacialRef: Nelson 18th/e p. tr767; O. P. Ghai Vh/e p. j40l
c Laryngomalacia is the most common congenitallaryngeal anomaly.
c It is the most common cause of stridor in infants and children.

FOREIGN BODY ASPIRATION

6. Ans. is 'a' i.e., Foreign body tRel O.P. Ghai 9th/e p. j91 6 Vh/e p.
p. 1769;CPDT l}th/e p. 505)
367-358; Nelson 18th/e

foreign body"
"Choking and coughing episodes accompnied by wheezing are highly suggestive of airway - Nelson
"Foreign body aspiration should always be considered as a potential cause of stridor and airway obstruction in
children". --- O.P. Ghai
7. Ans. is'b'i.e., Bronchoscopy {Ref: Nelson 18th/e p. 1770; CPDT 18th/e p. 5051

8. Ans. is'd i.e., Rigid bronchoscopy [Ref: O.P. Ghai 8th/e p. 391 (z 7h/e p. 367)
o Treatment of choice is removai of foreign body by rigid bronchoscope with appropriate antibiotics.

9. Ans. is 'd i.e., Cough fRef: Nelsan 18th/e p. 1770; CPDT 18th/e p. 5051
10. Ans. is 'b' i.e., Leads to in advertently pushing of foreign body deep in respiratory system [Ref: Clinical
peiliatrics 2d/e p. 7121

LARYNGOMALACIA & CROUP

11. Ans. is'b' i.e., Laryngomalacia[Rel O.P. Ghai 8'h/e p. 369 & Vh/e p. 340; Nelson 18tu/e p. 1762)
o Laryngomalacia is the most common congenital laryngeal anomaly.
o It is the most common cause of stridor in infants and children.
12. Ans. is t' i.e., Laryngomalacia [Rel O.P. Ghai 8th/e p. 369 6 7/e p. jA0l

13. Ans. is t' i.e., Wait & watchlftef: Nelson 18'h/e ch. j8j.1l
o Most common cause of stridor (prominent in supine position) in neonates is laryngomalacia.
o Treatment is only reassurance.
14. Ans. is'b' i.e., Laryngotracheobronchitis (croup) lRef: Nelson l8,hle ch. 3821

o Croup (Laryngotracheobronchitis) -+ Steeple sign.


o Acute epiglottitis + Thumb sign and Vallecula sign.

15. Ans. is 'H i.e., Laryngotracheobronchitis [Re/: O.F. Ghai yth/e p. j76 6 7h/e p. 351; Nelson lyth/e p. 17631
o This child has : -
i) 3 days history ofupper respiratory tract infection.
ii) Followed by stridor
o These features suggest the diagnosis ofcroup.
t6. Ans. is'c'i.e., Causes upper atrrwayobstruction
[Ref Ghai 7,t,/e p. 35]
: Croup
e 3lHi is[TJ,]3,"J:ffi],:ffTfffilIjj:l:::':li:r::I,r.,
variety o&condition *ti.t lrr.t.ra; ;;,'""""''u1v
traci oDsl
obstructroa (not upper respiratory
tract)
ute epiglottis, laryngitiis, trachiobronchitis.
c. Brassy corroh i"
Brassv cough the *^;-
is rh^ main presenting fealure.
77. *:T.:J;;.fl":ljxff:::ll::::,*::'l:"'^'r,*:':ontB'h'|ep
1765;aFGhaiB'h,ep3766vh,ep351)
and stridor on crying in a
;ffi*Tr";::*
Mariagement of eroup
2 yearotd child ,"r*.* ,;:;;;; ::3:::;
c The main treatment for children
with croup is airway management.
o Tieatment of respiratory distress
shouid ,"i.. p.r".rq1;;;; .",
o Following treatment is indicated testing.
-
t) Humidifed oq)gen
2) HydrationwithJluids
3) Corticosteroids
t Are benificial in, even in mild croup.
! They decrease the edema in the
laryngeal mucosa through their
e They.reduce anti-inflammatory action.
the need of subsequent epinephrine
use,
d"xi*"io,o,n"
4) '.:t:;:::;:;:;:;;;,ed
are --> o'ot
6p),"y,,red), im dexamethasone or nebutized
budesonide.
a Mechanism of action is believed
to be constriction of precapillary
fluid resorption from the interstitial arterioles through
space and a de.rease in laryngeal B-receptors, causing
r Indication for nebuiized stridor are _ edema.
i) Moderate to severe stridor at rest
iii) Respiratory distress
ii) The possible need for intubation
s) Hetiox (Bio/o hetium,",t;;;;;;;;" May vv #..T,HT,.hildren
--l be !'eLLrYE witl severe croult who may need
intubation. 'r Lrrl-tclren wfi}]
6) Antibiotics are not indicated
. 7) Sedatives should not be given

l8' Ans. is 'b'i'e., Antibiotic, t'i.e.,


Nlorphine [Ref: a.F, Ghai B,h/e p.
c Sear barking rike croupy cough 376 d_ vh/e p. 351_352)
is a feature oflaryngotracheobronchitis
(croup).

EPIGLOTTITIS & BRONCHIOLITIS

1()
lus influenzae rype b
"ii;f,f,,,1;i;,::-,Y,nn :s' it is almost always causecl
{Ref : o.p. Ghai 8,1'/e p. 375 6 7h/e p. i51;
cFDr 1,th/e p. 5a1l
by Haemophilus influenzae
2{r. A na i. (,{r : ^. n ^_. .
Ans'is'd'i'e.,Respiratorysyncytiarvirus[frel: type 8,,. - cpDT
o"F.Ghaia"np.sstiZ!l,'ro"uri,"iirrlan*u/ep.tzza)
'#;{r:i,;',::::f";:::if;i!,i,!f"t f,, Bronchiotitis
is Respiiatory synctiat virus.
RSV is responsibte
for more
21. Ans. is 'a'i.e., Bronchiolitis
tRel o.p Ghai yth/e p. 381 6
7h/e p. 357; Nelsan 18th/e p.
1775; CFDT 18th/e p. 5141
e Biiateral wheeze, hlperinflation
of lungs and normal WBC c ount
are characteristic oiacute U.orr.t with respiratorl, distress in a 6 months
ioti,i.. old infant
22' Ans. is t'i.e., Respiratory
synchytiar virus [Relr o.p. Ghai
8th/e p. 381 6 7h/e p. 356_357; Nerson lVh/e p. u16]
o The child presenting with respiratory
distress and bilatpral crepitations
23, Ans. is 'a'i.e., &{acrolides
and wheezing, we have to suspect
brochioritis.
[Ref; A.p, Ghai gth/e p. 3g2 d. Tt,/e p,. 3Sg;Nelson lgth/e p" 1776)
Treatmeni of Bronchiolitis
c Treatment is essentially symptomatic.
o The child should be ,urrld in
humid atmosphere.
a Oz_ remains the mainstay of treatment.
o Antibiotics - Though antibiotics
have no role* but Ribavarin can
o Iffever is present antipyretics sometime be used.
can be usecl.
o Continuous positive airway pressure.
' 0' adrenergic drugs and Ipratropium bromide are
also not recommended for
infants < 6 months.
^{ns. is 'a' i.e., Ribavirin lRef: O.P. Ghai th/e p.

o Ribavirin (Vibrazole), when indicated, is the antiviral agent of choice.

Arrs. is 'b- i.e., Ribavarin fRef: Nelson 18th/e p. 1776; o.P. Ghai 8th/e p. 382 lt 6th/e p. j53)
c The child with HIV is suferingfrom Bronchiolitis.
o Antibiotics are not usually used in the treatment of Bronchiolitls who are previously healthy but Ribavarin shortens the
duration of treatment in patient who have -+ Immunodeficiency, Congenital heart disease, Chronic lung disease
c The child in the question is having immunodeficiency due to, HIV infection. Therefore Ribavarin should be used in
the treatment of this Patient.

16. Ans. is'a' i.e., RSV IRel O.P. Ghai 9th/e p. j82 6 Vh/e p. 35/
3:. Ans. is'd i.e., RSV tRe/ O.P. Ghai 8n/e p. 381 dt Vh/e p. 3561
r Wheezing due to respiratory tract infection in a I year old child is due to bronchiotilis'

PHEUMONIA

lB. Ans. is t' i.e., Streptococcus pneumonia lnef: O.P. Ghai 9th/e p. j77 6 7h/e p. 352; Nelson 18th/e p. 17951

o Stre7ttococcuspneumoniae(pneumococci)isthemostcommonbacterialpathogensfollowedbychlamydiapneumoniae
ani mycoplasma pneumoniae. - Nelson 18th/e 1795

19. Ans. is t' i.e., Streptococcal pneumonhlRef: Ghai Vh/e p. 3521

o Most common bacterial cause of pediatric pneumonia is pneumococctts (streptococcus pneumoniae),

30. Ans. is t' i.e., Mumps virus [Rel o.P. Ghai 8th/e p. 377 6 vh/e p. 352; Nelson 18'h/e p. 1795)

o The peak attack for viral pneumonia is between the age of 2nd and 3'd years and decrease thereafter.
o Viruses causing pneumonia are -

IRSV ) Mostcommon rAdenovirus r Rhinovirus


r Tnfl,enza J r Parainfluenzavirus

3l . Ans. is 'a' i.e., RSV tRel Oski's paediatrics : Principles & practice p. 13951

"Respiratory syncytial virus (RSV) is the most common cause of pediatric pneumonia" - Oski\ Paediatrics
"Reipiratory viruses are the single most important cause of community-aquired pneumonia in pediatric'age group"
- Iawetz

I Most aomrnon cause of pediatric pneum6ni6 --+"RSV , :. , ; :


'pediatric,pneumOnia
a commOn'baCteriat cause'of -+ str€ptocoecus pneumoniae

.-o-
i.e., Staphylococcus aureus fRef: O.P. Ghai 8th/e p. 378 dt Vh/e p.
j53l
32. Ans. is
c pneumonia with pneumatoceles (multiple thin walled air containing cyst) is s/o staphylococcal pneumonia.
. o The diagnosis of slaphylococcal pneumonia is suspected in a newborn or an infant with respiratory infection who has
evidence of staphylococcal infection elsewhere in the body (here it is pyoderma).
JJ. Ans. is'a'i.e., Staphylococcus I Ref: O.P. Ghai 8tu/e D. 378 d, Vh/e p. i531
c Staphylococcus aureus is the most common organism causing pneumatoceles and pneumatoceles are considered
p atho gnomonic of st aphylo co ccal p neumo nia.
o E.coli and.klebsiellap.terr-o.,ia may have pneumatoceles. -- O.P. Ghai 6'h/e 350
j78 6 p. 354)
34. Ans. is 'd i.e., Staphylococcal pneumonia [Rel O.P. Ghai 8th/e p. Vhle

r Staphylococcal pneumonia is the most common cause of (amongst all pneumonia)


t Pneumatoceles t Pneumothorax t Lung abscess
t Purulent p ericarditis t Empyema
o Other organisms commonly causing empyma -+ Streptococcus pyogenes, Pneumococci

35. Ans. is 'b' i.e., Staphylococcus [Ref O.P. Ghai Sth/e p. 379 6 7/e p. 35a)
36. Ans. is 'b'i.e., Measles [Ref; Harcison lVhle p. l2]S]
t cell (hecht's pneumonia) is most often documented in
immunocompromized and malanourished patients
#t#JJ:J:nt
o It is a rare complication of measles. .t

37. Ans. is 'd'i.e., Erythrornycin [Ref O.F" Ghai 8,h/e p.


379 & Vh/e p. 355; Nelson tythle p. ]Z9B]
o Atlpical (mycoplasma) pneumonia is
treated by macrolides (Azithromycin, Erlthromycin).
38. Ans. is 'b' i.e., E. Histollti ca lRef C\DT p.
lg,t,/e 528; Robbins Thle p. 250; Nelson lythle p. tg02l
r Organisms commonly causing lung abscess are _

r Pseudomonas r
iY,iir"Kr:^l',...r, itifJ{r"#}""' Bacteroides, fusobacterium, peptostreprococci
o Others are -) Nocardia, Legionella, Fungi (candida
and aspergillus)
39. Ans. is'b'i.e., Right apical lRef:.Grant,s atlas af anatowy jgl
ltrh/e page
r
Lobes of lung most susceptible to aspiration
in supine position -+ po;t-segment
o superior segment of right rower robi, superior of Rt. (Jpper lobe(Most common),
segment of reft lower lobe,

ASTI"IMA

40. Ans. is 'b' i.e., Raised IgG level A.p Ghai ytk/e p.
[Ref: 353, 384 & p"
Zh/e 3SB_359;Nelsate l8nle p. 953, 954;
C?DT t&*le p. 1049, 10501
o It is tvpe I hlpersensitivity, which is mediated by IgE
a
-+ IgE is raised(not IgG)
n be family hklory of atopic disease
T"::.*o,
c Childhood
@iopc deriatitis, oi"rgi, rhinitis).
asthma usually improves with age.
Mild exercise can cause bronchodiratation and
may reduce wheezing.
,o
41. Ans. is'b'i.e., Wheezing lRef: Ghai Z',/e p. 3S9l
r Most characteristic symptom of Asthma is wheezing.
42. Ans. is t' i.e., Clubtring fRef: O.F. Ghai 8th/e p. 353 & Vh/e p. 3591
"Clubbing of
fingers is unusual in uncomplicated cases,,.
43. Ans. is t'i.e., Sedatives lRef: O.p. Ghai Bth/e p. - O.P. Ghai
386_jgt & Vh/e p. 360_j62; CpDT tythle p. N58l
o Pharmacological treatment of Asthma
i) Mild
r Inhalation of sh3rj acting p, receptor agonist, e.g.
r If there is nosatisfactory response, addlnhared salbutamot (albuteror).
(ipratropium).
r In patients who are refractory to inhaled therapies,
"ffirn"ti"rrgtc
,.". (theophylline) may be effective.
ii) Moderate and severe "*inZplrylline
t oxygen plus z systemic steroids plus t continuous administration of aerosolized salbutamol
44. Ans' is 'b'i'e" Administer loao/o arby masft
[Ref: o.p. Ghai 8*le p. 391 6 7/e p. j6a462; IAp Tdle p. 4A4]
Management of life threatening asthma
{status asthmaticus}
1) Immediate O.inhalation
2! Inj,ection of tirbutaline or adrenaline (s.c.)
3] Inhalation of salbutamol or terbutalinte and ipratropium
4) Injection of hydrocortisone llmg/kg.
45. Ans. is 'a'i.e.' Inhaled short acting beta-2 agonist
fRef: preuious expranations]
o Treatment of choice for acute asthma -+ i
Inhaled short acting Bz agonist.
e Treatment of choice for chronic asthma -+ -'
Inhared gr".o.o;ti.oiJ'r.
o Even in chronic asthma, stage I (miid intermittent),lnhaled
short acting Bz agonist is used, as needed for symptoms.
46. Ans" is'b'i.e., Short acting budesonide {Ref: CpDT tY*/e p. ta57}
o Tolerance is potential problem with Bz_agonists.
a

' not be used o,' unvi"g,riar schedule


::'j;1ilfitJ:::*'o -> due to tolerance, bronchial hlperactiviry not reduced

ll'

ru
o It is advised that patients requiring regular medication should be treated with inhaled steroids and use of p,
ed patients agonist inhaler should be restricted to symptomatic relief of an acute attack.
{ Ans. i"e., ?rophylaxis ryith beta agonist [Ref: O.P. Ghai 8*/e p. j90; Vh/e p. 365-366; CPDT 18th/e p. 1058]
is'lf
o The meilications most widely used by people with exercise induced asthma are short and long acting beta 2-agonist
t
bronchodilators.

48. Ans. is "C ie., MDI viith spacer with rnask fnef Ghai 7h/e p. 36j)

r MDI (Metered-dose inhaler) alone require better press and breath co-ordination so used above 12 years of age.
o MDI with spacer overcome breath co-ordination so used above 4 years of age.
rcocci r MDI with spacer with mask can be used successfully in children below 4 years of age.

BRONCHIOLITIS OBLITERANS

+9. Ans. is tl'i.e., Bronchiolitis obliteranslRef: Nelson 18tu/e p. 1781, 17821

o Bronchiolitis obliterans is a rare chronic lung disease of the bronchioles and smaller airways.
r Bronchiolitis obliterans most commonly occurs inpediatric population after respiratory infections (i.e. adenovirus,
mycoplasma, measles, influenza, pertussis).
o Other causes include connective tissue disease (i.e. juvenile rheumatoid arthritis, systemic lupus erythematosis, scle-
roderma)
o Bronchiolitis occurs in all age groups.
tasa] o Cough, fever, cyanosis and respiratory distress followed by initial improvement may be the initial signs of bronchiolitis
f
obliterans.
o Progression of the disease may ensue with increasitg dyspnoea, cough, sputum production and wheezing.
. . Cheit radiographs may be relatively normal compared with the extent of physical flndings but may demonstrate
hyperluscencyind patcfu infiltrates (occasionally, a swyer fames syndrome, i.e. a unilateral hlperluscent lung has
developed).
o pulmonary function tests demonstrate varaible findings but typically shows signs of airway obstruction.
o Ventilation perfusion scan shows a tlpical motheaten appearance of multiple matched defects in ventilation and
perfusion.
o CT scan demonstrates patchy areas of hlperluscency.
r Open lung biopsy or iransbronchial biopsy remains the best means of establishing the diagnosis of bronchiolitis
oblite.rans.

50. Ans. is 'b- i.e., Bronchiolitis obliterance {Ref : See above explanationl

cYsTrc FrBRoSls
51. Ans. is t'i.e., Sweat gland
o The sweat glands are not obstructed in CF patients because in serous glands such as sweat glands there are abnormal
concentrations of inorganic ions, rather than glandular obstruction with thick mucus.
o Obstruction of Lirwiys leads to bronchiectasis anil atelectasis; pancreatic duct obstruction leads to pancreatitis and
malabsorlttion; and plugging of bile ducts leails to obstructive jaund.ice,

52. Ans. is'd i.e., Meconium ileus [Ref Nelson 18*/e p. 1807; O.P. Ghai 8e/e p.
j9j & 7/e p. 369]
r Most patients with CF present in chfdhood. Approx 187o of patients present within the first 24 hr of life with
Gastrointestinal obstruction, termed Meconium ileus'
o Other systems dysfunctions are not present at birth, they may manifest within the first year or two of life including
respiratory tract sYstem.

53. Ans is'o- i.e., Abnormality in CFTR which leads to defective calcium transport [Rel Robbin's 8*/e p. 465-681
o Cystic fibrosis is an autosomal recessiye disorder of ion transltort in epithelial cells that affect fluid secretion in
exocrine glands and the epithelial lining of the respiratory, gastroiytestinal and reproductive tracts.
o Cystic fibrosis occurs due to mutation in the CFTR gene'
o CFTR was initially recognized as chloride conductance channel but it is now recognized that CFTR can regulate
multiple ion channek and cellular ptrocesses. These include :- Chloride channels, Potassium channels, Gap junction
channels, and Bicarbonate ions.
Ex,, ri :.,llesrJirAtai! Sy gt e m
:

ua'
'dns is i"e., Fseudomonas aerugenosa (non rnucoid)
lRef: Nelsarc pediatrics lgth/e p. l2ag, ulal
a The most commin and important pathogen in cystic
fibrosis is Pseudomonas aeruginosa,which causes infections
that, once established, are impossible to eradicate.
e Acquisition of Pseudomonas infection begins early in childhood.
The initial acquisition of p. aeruginosa in the CF
lung is with nonmucoid strains and occurs early in life. Over time there is transition
to mucoid strain, and with age
mucoid strain becomes predominant with age.
o Non-mucoid P. aeruginosa can be eradicated with aggressive anti-pseudomonal
antibiotic theraphy. Mucoid strain
cannot be eradicated by current antibiotics and therefore predicts shortened
survival.
Etr
.A.ns. is t' i.e., Transepithelial nasal potential difference [ll.ef. a.F" Ghai 8,h/e p. 393 & 7h/e p. 369; Nelsan n Sth/e p.
18A4,18A3)
o The sweat test is the standard approach to diagnosis.
o The diagnosis is made by elevated sodium and chloride level in the
sweat > 60 meq/1.
o Two test on different days are required for accurate diagnosis.
e A normal sweat chloride dose not exclude the diagnosis. Genotyping
and other tests such as measurement of nasal
membrane potential dlfference, pancreatic function should be done ifihere
is high clinical suspicion of cystic fibrosis.
s6. '{ns. is 'a' i"e., Cystic Fitrrosis fRef. Idarrison t7h/e p.1632, id3,l; R.obbins Th/e p. 494; I{elsow lgth/e p.1g0g;
R.udolph's Fediatrics 2 1
*/e p 1 9 7 2l
"

e Recurrent chest infection in a child with evidence of exocrine pancreatic insfficiency (bulky, foul smelling
stool) suggest a diagnosis of cystic fibrosis.

57" Ans. is oa' i.e., Fibrocystic disease of pancreas e fRef. Nelson rgth/e p. g07l

EOruGEN ITAL LIJ'\IG DEFECTS

58" ,A^ns. is 'b' i.e., 58-7$a/ar occurs in lungs {R.ef. Scbwartz gth/e p. 59Zl
c Only 15% of bronchogenic cysts occur in lungs (puimonary parenchyma).
&

59. Ans. is 'b' i.e', Fulmonary sequestration tfi€l Nersan nSth/e p" tr784)
o Sequestrated lung has no connection to airu'ay, so this part does not
fill on bronchography

MISCELLANEOUS

60" Ans is'a' i.e., Kartagener's syndrome lRef: Netsow p. 1g1Z-1glg1


c Kartagener's syndrome is a subgroup of primary ciliary dyskinesia.

Primary ciliary dyskinesia:


e It is an autosomal recessive syndrome.
* It is charactetized bypoorlyfunctioningtilia. There is absence or shortening of Dynein arms that are responsible
for the coordinated bending of cilia.
e Approximately half of the patients with primary ciliary dyskinesia
have kartagener's syrdrome.

Poor functioning of cllia contribute to retention of


secretions and recurrent infections
a Differential pulses/Blood Sinusitis Lack of ciliary activity interferes with bacterial
clearance, predisposing sinuses to infection rs a
m a n ifestati o n of Co a rctati on
& Situs inversus -) Due to ineffective cell motility during embryogenesis
& lnfertitlity -) Due to ineffective mobility of sperm
61. Ans" is 8 i.e., Influenza virus [Rel: Nelsan l*/e p" IzZTl
r Bronchitis refers to nonspecific bronchial inflammation.
c Acute bronchitis is a syndrome, usually viral (influenza) in origin, with cough as a prominant symptom.
62. Ans, is 'tf
i"e;, Choanal atresi* ER* *"p" G]wi *fe p. 3 6, e Fk p JSrJ
r Bilateral choanal atresia is a very serious life-threatening condition because the baby is unable to breath directly after
birth as neonates are obligate nasal breathers.
r In some cases, this may present
as cyanosis while the baby is feeding because the oral air passages are blocked by the
tongue.
e The cyanosis may improve when the baby cries, as the oral airway is used at this time.
r These babies may require airway resuscitation soon after birth.
63. Ans. is t'i.e., Neuroblastorna I Re/: www.merck.coml
"Most common posterior mediastinal masses in children are neurogenic tumors'l - www.merck.com.
Posterior rnediastinal masa in children
1) Neurogenic tumors (Most common) 2) Foregut cyst (Second most common)
3) Ly'mphoma (Third most common) 4) Germ cell tumors
5) Lymphangiomas and agiomas
Mediastinal neurogeni( tumors are:-
r Mediastinal neuroblastoma I Ganglioneuroma r Ganglioneuroblastoma
r Schwannoma r Mediastinal pheochromocytoma

&. Ans" is ? i.e., Bronchosccpy lR$ Indian leiarnal af prartieel pediatries 2w2 p 4l:32\
o A history of recurrent pneumonia in the same lobe or segment or slow resolution (> 3 months) of pneumonia on
successive radiographs3uggests the possibility ofbronchial obstruction and the needfor bronchoscopy.

65. Ans. is'b" i.e., Reassure &give saline nasal drop lRef park2*lep" 156|
r This 3 weeks male child with noisy breathing does not have chest indrawing or fast breathing (as clinical examination
is normal).
r So, this child is having simple cough or cold, no pneumonia.
o Management includes :-
(i) Reassurance to mother (iii) Keep the baby warm
(ii) Frequent breast feeding (iv) Clear the nose
6. Ans. is "a' i"e", Careinoid ER.6 Pedi&triq swrger.y jaucnatr zSIe|

III
G.&SYK,*XefYffiSY'KKAt
Y'EACT

ESOPHAGUS AND STOMACH


Tracheoesophageal fistula (TEF) and esophageal atresia
o Esophageal atresia is the most common congenital anamoly of the esophagus. More than 90% are associated with
tracheoesophageal fi stula.
o The most common variety is the one where uqryter Ttart of esophagus ends blindly and the lower part is connected to
trachea by afistula, TEF should be sispected in all cases of hydroamnion.
o There may be associated anomalies -+ VeCtgRf (vertebral, anorectal, cardiac, tracheal, esophageal, renal, radial,
limb) syndrome.

Classification of tracheoesophageal fistula (TEF)


o TEF is classified into five types based on presence of esophageal atresia and location of fistula:-
1) Tlpe A : There is esophageal atresia without TEF. There is no gas in abdomen. It is 2"d most common tFpe.
2) Type B : There is proximal TEF. There is no gas in abdomen.
3) Type C : There is proximal esophageal atresia with ilistal TEF. Gas in abdomen is present. ltis most common.
4) Type D : Both proximal and distal fistula are present. Gas in abdomen is present. lt is least common.
5) Tlpe E : Isolated TEF (H or N type) is there.

Gep#g
Clinicalfuatures
. Neaonates present withfrothing, excessive drooling coughing and cyanosis
withfeedinglArrMsl3,ol).This is followed
by respiratorY distress(en'us 0i 0r). Newbor n regurgitates of its first and subsequent
feeds.
Diagnosis
o Inability to pass a nasogastric tube('4rrMs 03)
confirms the diagnosis of TEF.
r Lateral chest X-rayarMso3) shows a lucent proximal pouch that may displace trachea anteriorly.
r X-ray abdomerr(A,Ms shows gas in stomach in commonest variety type-c rEF.
03)

Congenital hy$rcfrophic pyloric stenosis


o Hypertrophic pyloric stenosis is the commonest surgical disorder of the
stomach during infancy. pylorus is thickened
and elongated and its lumen is narrowed due to hlpertrophy of circular muscle fibers
of pylorus.
r It is More common in boys.It is More common in infants of blood group ,8, and ,o,.
r Associated congenital defects are -) Tracheoesophageal fistuta and hlpoplasia or agenesis
of inferior labial frenulum.
It may be associated with Turner syndrome aid Trisomy 1g,
t Symptoms are not present at birthallMsu). The vomiting usually starts afier 3 weeks
of age(Atmsu).
clinical manifestations of coilgonital hypertrophic pyloric atenoris
o Pyloric stenosis is usually not present at birth,
o Nonbillous vomiting is the initial symptoms of pyloric stenosis, The vomiting
usually starts after 3 weeks of age@rrns
M), but
symptoms may develo?t as early as the lst week of life and as late as the 5 months
of lifsreilusoas.
o Emesis may follow each feeding, or it may be intermittent. After vomiting,
the infant is hungry and wants to feed
again' Due to vomiting, there is progressive loss of -+ Fluid, Hydrogen ion, Chlorides.
This results in hypochloremic
metabolic alkalosisallMs 06' 02). Serum potassium levels qre usually maintained, but there may
be a total body potqssium
deficit.
r In early stages the kidneys are able to compensate by reabsorbing hydrogen and chloride ions in
exchange for sodium,
potassium and bicarbonate, however as the body stores ofsodium and potassium becomes
depleted, these ions are
selectively retained while hydrogen is excreted in urine, causing the
Ttaradoxical aciduria@rrMs0r.). This aggravates the
alkalotic stage.
Diagnosis of congenital hypertrophic pyloric stenosis
o History of vomiting without bilear la nmr) (nonbilious vomiting).
e Physical examination -
i) Palpable mass in the epigastriumall3,Narr)
ii) Afier feeding, visible gastric peristatic wave that progresses across the abdomen@cr ezt.
r USG confirm the diagnosis
t criteria -+ Pyloric thickness > 4 mm or an overall pyloric length > 14 mm.
o Contrast studies -
1) Shoulder sign + A bulge of the pyloric muscle into the antrum.
ii) Double tract sign -+ Parallel streaks of barium in the narrowed channel.
iii) Elongated pyloric channel.

Various signs in CHPS


Treatment of congenital hypertrophic pyloric stenosis
e The surgical procedure of choice is Pyloromyotomy.
e The traditiona! Ramstedt procedurePclT) is performed through a short transverse skin incision.
o Other methods are --> (i) Atropine therapy (ii) Endoscopic ballon dilatation

INTESTIIIIAL T}ISORDERS

MEGACOLON

e Megacolon is an abnormal dilatation of colon that is not caused by mechanical obstruction. It may be :-
1) Congenital megacolon: Hirschsprung disease.
2) Acquired megacolon : It may be due to poor bowel habi{scPcr 0} drugs (antipsychotics), and infection (chaga's
disease).

Congeni&il a$anglionic megacolon (lIirschsprung diseasel


e Congenital aganglonic megacolon is caused by defectirte migration of neural crest cells into the mesodermal layer
of gu{AIst).
r There is aganglionosis (absence of ganglions) in a portion of intestinal tract. Intestinal segment lacksboth Meissner
submucosal and Auerbach myenteric plexases. This results in functional obstruction wTth contraction of affected
gg) oe).
(aganglionic) segmen/Alrus and intestinal dilatation proximal to affected segmen{Pa Massive dilatation of
proximal colon may cause megacolon.
c Rectum is always affectedwlth involvement of more proximal colon to variable extent. Most cases involve the rectum
and sigmoiil only, i.e. rectosigmoid region.

Clinical features
o ThereischaracteristicallydelayedpassageofmesenisTn6nnsu'02).Thisisfollowedbybiliousvomitingandabdominal
distension. Enterocolitis is seen between 2-4 weeks with bloody stools and fever.
o Complications may occur, e.g. perforation, constipation and rectal bleeding.

Diagnosis
c Rectal manometryar13) is a useful screening procedurear'3). There is failure of the internal anal sphincter to relax
(pressure fails to drop in internal sphincter) after dilatation of rectum. An abnormal test is not absolutely diagnostic
but a normal test excludes the diagnosis(AllMsee), i.e. it is a sereening test.
o Rectal biopsy is the definitive diagnostic procedurs@I t3). It shows :
AUMS}4)
1) Absence of ganglion cells and ganglia in affected segmenf

ii) Thickening andhypertrophy of nerve trunk(AilMsee).


c Barium enema shows contra.cted aganglionic (affecteil) segment and dilatation of normal proximal
segmen{ArrMs rr). }ust distal to dilated segment there is trarusitional zone, which is partialiy dilated (just proximal to
contracted aganglionic segment). t

o Technique demonstrating ACHE (acetylcholinesterase) staining in aganglionic instestine are also of interest in
diagnosis(Ar rs).

Treatment
o Surgical resection of the affected segment and anastomosis of normai segments is the treatment of choice. Surgical
options are Swenson, Duhamel, Soave and Boley procedures. Diversion colostomy is done initially to gain time if
patient is not fit for surgery.
The diagnosis in t}te given barium
enema study is _

A) CHPS
b) Hirschsprung disease

-d) Tropical sprue

Ane" is'b' i.e., Hirschsprung&seas€


r The given barium study is showing
d,israr contracted agangrionic
proximal dilated segment, proximal segment with
to transitional zone.

N EONATAL INTESTINAL OBSTRUCTION

r Impgrtant causes of neonatal intestinal obstruction


areieiunoileal atresia (most
(2d mc : 4,yo/Nrnq, meconium common: 50o/o), duadenal atresia
prug syndrome, meconeum ileus,
and marrotation of gut.

o Meconium plug syndrome


-> when meconium prug causes obstruction of colon.
Hirschsprung oisease, maternat ow, It is seen in
,ut"rnut f r"J.;;;r;, or",raturity, sepsis,
c Meconium lleus -+ when meconium plug hypothyroicJism.
obstruo ir"ri. Ii, seen in cystic fibrosis.

a Most characteristic clinical


feature of neonatal intestinal obstruction is bilious vomitingal 03).
There is distension
which is caused bv accumuration of
gas and nuid. zo-eov, of this intestinat
::,::;;:;,r:;:#:rl;ffi,'*n gas

Oiagnosis
t The mainstay of diagnosis of intestinal
obstruction is plain abdominal
a X-ray whichshows :_
Complete intestinal obstruction: Fluid
: gas (air) levels.
x Meconeum ileus: Growd,glass appearance.
* Doudenal atresia: Double bubble sign(Nrar) (two
air_fluid levels).
Nlejunal atresia: Triple bubble sign. (three airfluid
levels).
a Ileal atresia: multipie bubble sign (multiple
air fluid levels), and christmas tree
r volvulus neonatorum is diagnosed by birium appearance.
contrast studies with
However' perforation is suspected plain fluoroscopy lborium mear follow through).
.if X-ray abdomen is the initial investigation
of choiss{u to).

Also know
httussusceptian is the most common cause
' of intestinal obstruction between
3 months
to 6 years of age (not in neonates).

INTlJSSUSCEPTION

I Intussusception occurs when a portion of the alimentary


tract is telescoped into an adjacent
segment.
common cause of intestinal obstruction
" ':';:;:"st between 3 months and 6 year of
age{Nmr).It is more corrmon

c Intusssception may be : _

i) Ileocolic (most common)(tr t4


ii) Cecocolic
iii) Ileoileal
r The upper portion of bowel that invaginates
into lower portion is called intussusceptum.
c Lower portion which recieves telescoped upper portion is
carled intussuscipiens.
Pathogenesis
o Between the age of 3 to b months, cause of acute intussusception is idiopathic.
o It is postulateil that gastrointestinal infection or the introduction of new food proteins result in swollen peyer's
patches in the terminal ileum(Ato2). The prominent mounds of tissue leads to mucosal prolapse of the ileum into the
colon thus causing an intussusception.
t In 2-8o/o of patient, recognizable lead points for the intussusception are found, such as a meckel's diverticulum, in-
testinal polyp, neurofibroma, intestinal duplication, hemangioma or malignant condition such as lymphoma.

Clinical features
o There is sudden onsel of severe paroxysmal colicky abdominal pain that recur at frequent intervals. Child cries loudly
(screaming@Gr03)) and there is straining efforts with legs and knee flexed.
r Blood and mucus produce a current jelly stool and there may be reflex vomiting.
o There may be abdominal distension@Gro3) and tenderness.
r Occasionally intestine prolapses through rectulrlr producing mass per ,"r1urn(ed oo)
.

Diagnosis
r On palpation there is sausage - shaped tender palpa$ls tnq5seetot) and emptiness in right iliac fossa (dance sign).
r Barium enema is both diagnostic and therapeutic. It shows claw sign and coiled spring sign.
r'X-ray abdomen shows :-
i) Crescent sign : Intussusception lead to point into gas filled lumen.
ii) Target sign: Mass in right upper quadrant forms a shape of target.

Treatment
r Correction of intussusception by barium enema is the initial management of choice.If it fails, surgical correction is
done.

1,..,,,,,:r,,,1,The,$i@,.bari@errc*n,'study,,ii sho*jng::iai,tdil tryingslgtt },characteristic of


intussusception.
The giveufigq6is silof, r .., ,

, a)l Elirschsprungdisease
, .

b) Tropical sprue
c) Coeliac sprue
d) Intussusception
', t''':

r : There is:'elaw signt, characteristic ofintussusception.

IiIFLAMMATORY BOWEL DISEASE

r Inflammatory bowel disease (IBD) is used to represent two distinct


disorders of idiopathic intestinal inflammation :-
l) Crohn disease or regional ileitiseclos)
ii) Acerative colitis(Pcros)

Crohn disease (Terminal ileitis or regionat enteritis)


r Any portion of intestine can be involved, but most commonly small intestine (especially distal ileum) is involved (in
40o/o cases)@mr'AIe8)' Both small intestine and colon are involved in 30o/o
of casssurnr.arst), end only colon is involved
in 309o.
r In the affected segment, mesentric fat wraps around the bowel
surface -+ creeping fatarr 13)
r The intestinal wali is rubbery and thick, as a consequence
of edema, inflammation, fibrosis, and hypertrophy
of the
muscularis propria -+ lumen is almost aiways narrowed -+
string sign(rvrrr) on barium meal.
o A classic feature of cD is the sharp demarcation of diseased
bowel segments from adjacent uninvolved bowel.
r When multiple bowel segments are involved, the intervening bowel is normal
-+ skip lseisnsarq).
r There are serpentinelinear ulcer along the axis ofbowel.
r As the intervening mucosa tends to be relativelyspared, the mucosa acquires
a coarse texture -+ cobblesto ne6rs4)
appearance.
r Narrow fissures develop between the folds of the mucosa.
r Fissures can penetrate deeply through the bowel wall and
leading to bowel adhesions and serositis.
r Further extension of fissures reads to fistura or sinus tract formation.
o There is transmural inflammation affecting all layers
of bowel wallarr^seT).
r Sarcoid like noncaseating granriiomas(NEEr) may be present
in all tissue layers.
o Neutrophilic infiltration into the crypts results in formation
of crypt abscess.
o Fibrosis of the submucosa' muscularis propria, and mucosa
.,r..rr*lly l"uds to stricture formation.
o There is an increased incidence of cancer of GIT in patients
with long-standing CD, but the risk of cancer in cD is
considerably less than in patients with chronic UC.
o On barium enema there is Hose pipe (lead pipe) appearnace(A rrMs 04)
andcobble _ stone appearnace4rrMs 04) .
r Screening test is presence of anti-saccharomyces cerevisae antibody (ASC4;rr"urr,0e).
Antibody formation is com_
mon against cell wall of yeast, Saccharomyces cerevisae in patients
of cD.
e Extraintestinal manifestations of crohn disease are
skin manifestations4rrMseT) (erythema nodosum), migratory
polyarthritis, anlcylosing spondylitis, sacroilitis, uveitis, clubbing,
Hepatic pericholangitis and primary biliary cirrhosis.

r cD is associated with HLA-DRI/DQw5 and NoD 2 genes and an


abnormal T-cell response, particularly cD4
+ T cells {T*1 cells). :
TUefAtowidgendoscopicpictureisdiqgn6gfi6qf;,r :i.,,:,,1,,,
Y'"" r .,,
. a)''T?opid *u*u '''"'ll{t'I:.r:::q:::tii: ' 'l'' r l' r t'.'"'
' "'t"'"'

d) Crohni disease

.'..

'Ans:,,istif!;e*CrOhtrsdiieage, r.,' : ': .,:1 : 1. '.1'.1: '.:' , 1,. ' 1i'' ' .

" c .' Tsiiis'ibbblestane appearanei, >charactefistic of crohnt.diseaie'

Ulcerative colitis
o Uicerative colitis involves the rectum and extends proximally.
o In severe cases entire colon may be involved + pancolitis.
r It is a disease ofcontinuity, and skip lesions are not found.
o In some patients"distal ileum may also develop muscosal inflammation
+ backwash ileitis.
r only mucosa and submucosal layers of colon are involvement, deep layer
are not involved usually.
o Isolated islands of regenerating mucosa bulge upward to
create ps"uJopolyp (Ar s8, e4) .
e There are superficial mucosal ulcer but they are not serpentine
as seen in cD.
o There is inflammation of crypts -) Crlptilis(er,usoar.
o Neutrophilic infiltration into crypts leads to formation
of crypt abscess.
c In sever cases of UC, toxic damage to the muscularis propria
and neural plexus lead to complete shutdown of neuro-
muscular function -+ colon progressively swells to create toxic
megacolon(.4re8, e4).
c There may be epitheliat dysplasia with progression to frank carcinoma.
c Barium enema shows : (i) Garden hose appearance; and (ii) pipe stem 8e).
appearance(Ar

o UC is associatecl w.ith HLA-DR2, potymorphism in lL-l0 gene andan abnormal r-cell response
particularly of
CD4 T-cells (T ce lls).
*2

Thefollowingendoscopic picture is diagnostic of -


' a) toprcal sprue
b) Coeliac sprue '

. cI Ulcerative colitis
d) Crohn's disease

Aus, i*.lcl,,i.e.i Ulcerative colitis


o colonoscopy is showin g pseudoporyp.s ) seen in urcerative coritis.

NECROTIZING ENTEROCOLITIS

o Neonatal necrotizing enterocolitis is the most common life


threatening emergencyof gastrointestinal tract in the new
born period.
o The disease is characterized by various degree of mucosal or
transmural necrosis of the intestine,
r It occurs iL pfematufe or small for gestational age (LBW) infonts. Artificial feeding
is a predisposing factor@croa).
r Clinical manifestations of necrotizing enterocolitis are divided into three stages (Bell,s stages).
1) Bell's stage-1 (Supected NEC) : Unstable temperature, apnea,
bradyc ardia, mild abdominal distensionurt3,es,pcr
e8),
vomiting, blood in stool(Pcr0s).
2) Bell'sstage-2(definiteNEC):Diminishedbowelsound(Al13,e8),metabolicacidosis6rrr),mild.thrombocl.topenia,
pneumatosis intestinalis with dilatation of bowel, abdominal
distension@r 13,sB).
3) Bell's stage-3 (advanced NE C) : Intestinal perforationwith frank
signs of peritoni tis;pneumopsyifsnsy/n{qt B,ss).
Tratment
o Stage 1 and 2 (without perforation) : Conservative treatment given : IV Jluid, nil oral, antibiotics\trMs 12)

o Stage 3 : Treatment depends upon :-


1) Definitive ascites but no perforation : Peritoneal drainage (paracentesis) is done\I 08).
ii) Perforation: Laprotomy with surgery (resection of necrotic bowel with ileostomy) is indicated.

MALABSORPTION
r Steotorrhea is defined as increased fecal fat excretion (> 7gmlday).
r Azotorrhea is defined as increased fecal nitrogen (> 3 gm/day), which is derived from protein.
o Both these conditions occur in malabsorption syndrome due to any cause as fat and proteins are not absorbed.
r Important causes in children are chronic pancreatitis (pancreatic insuficiency){ttss), coeliac disease (coeliac sprue)
(Aree),
ileal disease (Crohn's disease), intestinal lymphangiectasia, and bacterial overgrowth Slfldroms@tss).
r Work-up of a child suqpected to have chronic diarrhoea and malabsorptions includes the following investigations :

i) Repeated stool examination (especially for giardia)


ii) Fecalfat excretion studies
111) D-xylose test
w) Intestinal bioPsy@IIMSo4)
v) Specific tests e.g. sweat chloride for cystrc fibrosis
vI) Serology for celiac disease
v11) E xo cr in e p ancre ati c
fun ct i on

Celiac disease {celiac sprue/gluten-sensitive enteropathy}


r Celiac disease is a chronic malabsorption syndrome due to intestinal hypersensitivity to gliadin, component of gluten
as B) os) os).
in $rtrg41afl , oat, barely@uus and rye4ut"rs
o Strongest association of coeliac disease is seen with HLA-DQ26L03)followed by H2A-DQ8. Other HLA types (BBto.,
oo),
DR3 and DR7) have also been associated.
o Coeliac disease may be associated with dermatitisherpetifurmis.

Clinical features
r The child presents afier 6 months of age when weaning is started and gladin containing grains are introduced in the
diet. Child presents with diarrhea, anorexia, weight loss andfailure to thrive1re3).
r There is malabsorption with deficiency of iron, folic acid, andfat soluble vitamins (A, D, E, K).
o CNS manifestationsarse.) like ataxia and seizure can be seen.
o There is secondary lactase (disaccharidasq) deficiency@Gree). Individuals with celiac disease have a higher chances of
malignancy : enteropathy associated T-cell lymphomaecl ee) (most common) and small intestinal aderuocarcinoma.

Diagnosis
r The definitive diagnosis is based onhistopathologicalfindings which show :

1) Villous atrophy and Crypthyperplasiaql02) wlth decrease in villus : crypt ratio{Pcl00).


ii) microvilli brush border(Pclqq)
Loss of
111) Inflammatory cells are present in lamina propria(At 02)
: plasma cells, macrophages, lymphocytesecr 00),

eosinophils and mast cells.


iv) One of the characteristic feature is that overall mucosal thickness remains same('4r02) (as villous atrophy is
compensated by crypt hyperplasia).
v) Mainly proximal intestine is involved.
r Other characteristic diagnostic finding is symptomresolution on glutenfree dieteclee).
r The most sensitive non-invasive serological tests are the presen ce of IgA antibodies to tissue transglutaminase or IgA/
IgG antibodies to deaminated gliadin. Anti-endomysial antibodiesatls) are highly specific but less sensitive.
Treatment
o The treatment of choice is gluten free dietr.4rse.) life_long.

Lactase.deficien$J
r Lactase deficiency is the most common enzyme deficiency in human.
I Congenital lactase deficiency is due to mutation in lactase gene on chromosome-2.
c Acquired lactase deficiency is seen in infective enteritis (viral
or bacterial)arost, cystic fibrosis, pEM, cow,s milk
protein intolerance and use of neomycine.
Patient present with diarrhea, vomiting, bloating and abdominal
' ee)'Reducingsubstancesandorganicacidsarepresentinthestool.
painfollowing intake of milk or milk producls@rrus
Hydrogenbreathtestispositive@NB13).

DIARRHEA IN CHILDREN

r Diarrhea is defined as passage of liquid or watery stooles more than three times
a
I Persistent diarrhea is defined as acute diarrhea of infectious origin which lasts for day@cros1.
> 14 days (2-4 weeks).
c chronic diarrhea is difined as diarrhea rasting for more than
4 weeks.
o Intractablediarrheaisaformofchronicdiarrheainwhichno
infectiousetiologicalagentcanbeidentified.rtmaybe
seenin cysticfibrosis@Gro,),blleductatresia,cowtmilkproteinintolerance(milkallergyecr00)),abetalipoproteinemia,
secretory diarrhea dueto tumors@croo), carbohydrate intolerance,
lactase intolerance, congenital enterokinase and
trypsinogen defi ciencies, and pancreatic insuffi ciency.
c Dysentery is defined as presence of blqod mixed with pus and mucus in s1,,61seeros). shigella is the most common
cause of dYsentryoio-e' ea).

Most common viral cause and overall most common criuse


of diorrhea in children isrctaviflJl/,ctos,AttAso-,se,Atss).
Other viral causes are Norwalk virus, qy'snilyiyystanusss), ostrovirus
an6 grkirilurrno*rrq.
Most common bacterial cause of {iarrh ea is enterotoxigenic
E.coli/.t 12) followed v/ shigella,
'!e by J.r:'rrrq' salmonella,
rsrlrrvrltrr-'
EIEC and campylobacter.
Parasitic causes of diarrhea are giardiasis (especially in immunocompromizedo,,EnL
cryptosporidum
Lr /v.vrt.,Ur rqui
and E. histolytica. "

CONSTIPATION

r Constipation is extremely common in pediatric population.


r The vast majority of cases (90-95%o) are functional i.e. 90-95% have
functional constipationunMs e3)
with no organic
disease.
o In 1-5% cases, the organic disease can be d.iagnosed as cause
ofconstipation, i.e. only 1-5% have organic causes of
constipation.
o Organic causes ofconstipation are:
i) Intestinal; Hirschprung diseass0*rz), Anal/rectal stenosis,
Anal fissure, anteriorly displaced anal opening, stric-
tures(Nxnr).
ii) Dretgs: Narcotics, vincristine, Psychotropics.
ii) Metabolic/endocrine:Cysticfibrosisecln),hypothyroidismNEEr'PGI 10),panhlpopituitrism
iv) Neuromuscular: Cerebral paisy, Psychomotor retardation, spinal cord lesions, Myotonic dystophy, Neuropathy or
myopathy of GIT
v) Other causes: Low fibre dietecr rc),
Milk protein allergy.

e Chronic abdominal pain is defined as recurrent or persistent bouts of abdominal pain that occur over a minimum of
3 months.
o The commonest cause of chronic abdominal pain in older children is functional (non organic) that may be due to
emotional or behavioral problems(AllMs 83)
.

o Most common cause in an infant is colic (evening colic).

:
: r'::i:'r.:,'::::l:: &lniillign$!pat1hii*|,::;::,,,;.,,''.'..:i=,1:.,rLij!iil'pci:isonlng{rern
Constipation Pancreatitis
'..i:i:'i r .i.: f::'

:: :1:.:=.:.:.::.: :.a- .:..,,:.:.1.::.


'.

Colic (Evening colie)


c Colic is characterized by intermittent episodes of abdominal pain in young infants who are etherwise well.
a It usually begins at 1-2 weeks of age and can persist till 3-4 months of ags@rscas'1'Mc8r). Crylng occurs in the late
afternoon or evening. The diagnosis is confirmed when the infant cries more than 3 hours per day
for more than 3
days per weekfor more than 3 weeks.

Causes
o Exact cause is unknown. o Overactive child and overanxious parents
r Aerophagia c Cow's milk intolerance
e Hunger e Overfeeding with high carbohydrate containing food
e Immaturity of intestine

I{ EPATOBI LIARY DISORDERS

NEONATAL CI{OLESTASIS

c Neonatal cholestasis is defined as prolonged elevation ofserum levels ofconjugated bilirubin beyond the first 14 days oflife.

o Neonatal cholestasis can be divided into extrahepatic and intrahepatic disease and clinical features of any form of
cholestasis are more or less similar.
a Neonatal hepatitis is the prototlpe of intrahepatic cholestasis
e Biliary atresia is the prototlpe of extrahepatic biliary atresia
A) Extrahepaticcholestasis
r Extrahepatic cholestasis (obstructivejaundice) is due to obstruction of bile flow outside the liver.
r It is characterized by elevation of alkaline phosphatase\lrMs\6), 5-nucleotidase@IlMs06) and. gamma glutamyl
transp eptidaseqllMs 06' 02' At o1),

r There is predominantly conjugated hyperbilirubinemia.


r Important causes of extrahepatic neonatal choiestasis are extrahepatic biliary atresia\I 01, ArtMs or),
choledochal
cys{Al04'AIIMS02), and bile duct stenosis.
B) Intrahepaticcholestasis
r It is caused by disorders ofliver like :-

I) Metabolic: Tyrosinem ia,galactosemia(MPss), Gaucher's disease, Niemann-Pick disease, fructose-intolerance


e8)
and cystic fibrosis@P .

ii) lnfections: Hepatitis B and CaIIMse% rubella, CMV, slphilis, toxoplasmosis, Coxsachievirus
AI.MS 02)
rii) Idiopathic : Neonatal heltatitis{Al0a' '

r There is elevation of serum transamiruases (ALT, AST) and mixed (conjugated & unconjugated)
hlperbilirubinemia.

t Most common couse ol neanatal chitestasis end intruhepotic neonatal cholestasis:


Neonatal hepatitis (idi op athic neonatal hepatiti s)NEEtt
t 2nd most cbmman cause of neonotal cholestosis : Extrahepatic biliary atresia
t Most comman csuse af extrohepatic neonotal cholestasis; Extrahepatic biliary
atresia
) Srd most (omman csuse of neonAtal cholestasis;q , antittypsin deficlenci,/ilt

tdiopathic neonatal hePatitis


r giant cell hepatitis. This tlpe of cholestatic jaundice of unknown cause presents with features
It is also referred to as

of cholestasis and a typical liver biopsy appearance'


o It is characterized by:-
l) Obstructive jaundice (cholestasis)
ii) Giant cell transformation in liver@ilMs
01' AI01)

ilr) Patent bile ducts.


(i'e., with giant cells) -
r Other conditions which also presents with similar clinical and histological features
a) a, antitrypsin deficiencyartMs1l) d) Vital infections
b) Alagille syndrome ) Byler disease

c) Niemann - pick type C disease (NPC) f) Progressivefamilial intrahepatic cholestasis (PFIC)

, 1amma-glutomyl tronspeptidase is characteristlcally decreased in idiopothic neonatal hepatltis.

Extrahepatic biliarY atresia


r This is the most common catse of extrahepatic cholestasis in newborn. There is absence of patent bile duct draining
the liver.
r The child u sually present at 2-3 weekswith jaundiceatos'Pcto3'AilMsee). Occasionally jaundice canappear sincebirthqllMs
ee).
There is conjugated hyperbilirubinemia?Gt
03, Auus ss),
acholic stoolsecn B' AIIMi ee), durk (turmeric) colored utineql
o:), and hepatosplenomegalY.
and gamma - glutamyl
r Blood investigations show elevated direct bilirubin, alkaline phosphatase, S-nucleotidase
01' AI04)
transpegttidaseat
04, AttMS o2).
Similar to neonatal hepalltis giant cells are ltresent in liverqrrMs .

B). There is absence or inability to


r HIDA scan shows absence or decreased intestinal secretion of radionuclideed
visualize contracted gall bladder on USG'
t percutaneousliverbiopsyisthemostspecificanddiagnosticTtrocedul's\toa)lodifferentiatebetweenintrahepaticand
extrahepatic biliary involvement.
r Treatment of choice is Kasai's procedure (hepatopottoenterostomy).

r Biliary atresla is the most common indication for liver trarsplantation ln childrenffr''1.

PORTAL HYPERTENSION

r Normal portal venous pressure is 7 mmHg.


r portal hypertension is defined as an elevation of portal venous pressure > 10-12 mmHg or 5 mmHg more than IVC

pressure.
8s' Iipmet 87)
r It the most common cause of gastrointestinal bleed and esophageal variceal bleeding in children(wsc
is '

r Causes of portal hlpertension are divided into :-


CHaprr,r.
A) Presinusoidal (extrahepatic or prehepatic)
x itisthemostcommoncauseofportalhypertensioninchildrenQfi'r'N'EErnrrMS03). Itisduetoportalveinobstruction
from any cause.
x The portal hlpertension is usually non-cirrhotic, i.e. non-cirrhotic portal hypertension.
x Two most important causes of non-cirrhotic portal hypertension are :-
r) Extrahepatic portal vein obstruction (EHPVO)GI I I
' NEEI' lrIIMs 03)

* It is the most common cause of portalhypertension in children in India@EEl) Arrrl{s03).


Obstruction of
portal vein is caused by thrombosis (intraluminal obstruction) or by external compression by tumors/
pancreatic mass (extraluminal obstructior-r).
s The prcsentation is usually seen in childhood (7't and 2"d decade)(Ar i3). Clinical features are variceal

T;:*T:,[;:rilffi ;il*1"];,,,1#1ffi :::',1:'Jff 'iiL:.".'J,i:::r:ili:


j:".'LlT::
ii) Non- cinhotic portal Jibrosis
o Presentation and everlthing is almost same as EHPVO, except that lhe usual age of presentatian is
adultho o d (2 5 - 3 5y e ars).
B) Sinusoidal (hepatic or intrahepatic)
a It occurs due to intrahepatic obstruction. Most common cause is cirrhosis followed by veno-occlusive disease
and congenital hepatic fibrosis. lt is the most common cause of portal hypertension in developed conutries. LFT
are deranged, there is jaundice, ascites, hepatomegaly and massive splenomegly.

C) Post-sinusoidal (Post-hepatic or suprahepatic)


r It is due to obstruction of hepatic vein or IVC e.g. Budd-chiari syndrome. Presentation is similar to hepatic
t)?e'

Dribbling and regurgitation after feed, respiratory distress, frothing & bubbling at mouth in a neonate: Tracheo-esophageal fistula.
Congenita! hypertrophic pyloric stenosis (CHPS) usualty presents: After 3 weeks (but may be as early as 1 week).
Metabolic derrangement in CHPS: Hypochloremic alkalosis with paradoxical aciduria and hyponatremia.
Non-billous vomiting with palpable mass in epigastrium : CHP5.
Ramstedt operation is done in: CHPS.

Most common cause of acquired megacolon: Chagat disease.


Hirschsprung disease is due to : Defective migration of neural crest cells into mesodermal layer of gut.
Most commonty afftected region in Hirschsprung disease: Rectosigmoid region.
Typical presentation of Hirschsprung disease'. Delayed passage of meconium.

lnitial (screening) test of choice for Hirschsprung disease : Rectal manometry.


Definitive diagnosis of Hirschsprung disease is made by : Rectal biopsy.
Biopsy findings in Hirschsprung disease:Absence of ganglia & ganglionic cells with thickening & hypertrophy of nerve trunk.
Dilatated segment of bowel in Hirschsprung disease is: Proximal to affected (contracted) segment.
Most common cause of intestinal obstruction in a neonate:Jejunoileal atresia (mc) followed by duodenal atresia (2"d most common).
Most common cause of intestinal obstruction in children: lntussl,isception.
Most common symptom of smal! bowel obstruction in a neonate: Bilious vomiting.
Mostcommon causeof abdominal distensioninintestinal obstruction:Swallowedair.
lnvestigation of choice of volvulus neonatorum with suspected perforotion : Plain X-ray.

Likely cause of lntussusception in a child: Peyer's patch hypertrophy.


Bloody current jelly stool, abdominal distersion, sausage shaped palpable mass in child screaming with severe abdominal pain : lnlus-
susception.
lnflammatory bowel diseases (lBDs) in children: Crohn disease (regional ileitis), ulcerative colitis.
Commonest site of involvement in CD: Distal ileum, followed by distal ileum with colon.

Skip lesions are seen in: Crohn's disease.


Cn.q,p,:rt n

A) Presinusoidal (extrahepatic orprehepatic)


x Itisthemostcommoncauseofportalhypertensioninchildren@Irr'NrEErArrMso3). Itisduetoportalveinobstruction
from any cause,
x The portal hypertension is usually non-cirrhotic, i.e. non-cirrhotic portal hypertension.
x Two most important causes of non-cirrhctic portal hypertension are :-
t) Extrahepatic portal vein obstruction (EHPVO)(AIri' rYEEr AILYSor)

A[Ms t'3).
* It is the most common cause of portal hypertension in children in India$EE\ Obstruction of
portal vein is caused by thrombosis (intraluminal obstruction) or by external compression by tumors/
pancreatic mass (extraluminal obstruction).
* The presentation is usually seen in childhood (7" and 2d decade){Arr3). Clinical features are variceal
bleeding (hematemesis, melena), and mild splenomagaly. As the pathology is proximal to liver, LFTs are
normal, there is no hepatomegaly/cirrhosisiascites/jaundice and massive splenomegaly does not occur.

ii) Non- cirrhotic portal fibrosis


e Presentation and everlthing is almost same as EHPVO, except that lhe usual age of presentation is
adultho o d (2 5 - 3 5 y e ars).

B) Sinusoidal (hepatic or intrahepatic)


x It occurs due to intrahepatic obstruction. Most common cause is cirrhosis followed by veno-occlusive disease
and congenital hepatic fibrosis. It is the most common cause of portal hyperteision in developed conutries. LFT
are deranged, there is jaundice, ascites, hepatornegaly and massive splenomegly.

C) Post-sinusoidal (Post-hepatic or suprahepatic)


a It is due to obstruction of hepatic vein or IVC e.g. Budd-chiari syndrome. Presentation is similar to hepatic
tlpe.

SUMMARY OF CHAPTER IN ONE LINERS (QUICK REVISION FOR NBE PATTERN)


Dribbling and regurgitation ofter feed, respiratory distress, frothing & bubbling at mouth in a neonate:Tracheo-esophageal fistula.
Congenital hypertrophic pyloric stenosis (CHPS) usually presents: After 3 weeks (but may be as early as 1 week).
Metabolic derrangement in CHPS: Hypochloremic alkalosis with paradoxical aciduria and hyponatremia.
* Non-billous vomiting with palpable mass in epigastrium : CHPS.
Ramstedt operation is done in : CH PS.
Most common cause of acquired megacolon: Chaga s disease.
Hirschsprung disease is due fo : Defective migration of neural crest cells into mesodermal layer of gut.
Most commonly afftected region in Hirschsprung disease:Rectosigmoid region.
& Typical presentation of Hirschsprung disease: Delayed passage of meconium.
lnitial (screening) test of chioice for Hirschsprung disease ; Rectal manometry.
Definitive diagnosis of Hirschsprung disease is made by : Rectal biopsy.
Biopsy findings in Hirschsprung disease: Absence of ganglia & ganglionic cells with thickening & hypertrophy of nerve trunk.
Dilatated segment of bowel in Hirschsprung disease is: Proximal to affected (contracted) segment.
Most common cause of intestinal obstruction in a neonate: .lejunoileal atresia (mc) followed by duodenal atresia (2"d most common).

Most common cause of intestinal obstruction in children: Intussusception.


Most common symptom of small bowel obstruction in a neonate: Bilious vomiting.
Most common cause of abdominal distension in intestinal obstruction: Swallowed air.
lnvestigation of choice of volvulus neonatorum with suspected perforation:Plain X-ray.
Likely cause of lntussusception in a child: Peyer's patch hypertrophy.
Bloody current jelly stool, abdominal distersion, sausage shaped palpable mass in child screaming with severe abdominal pain : lntus-
susception.
lnflammatory bowel diseases (lBDs) in children: Crohn disease (regional ileitis), ulcerative colitis.
& Commonest site of involvementin CD: Distal ileum, followed by distal ileum with colon.
Skip lesions are seen in: Crohn's disease.
'ffi**=._
CHerrgR 6

: :zudopolyps and toxic megacolon are features of : Ulcerative colitis'

-:td pipe appearance an barium enema is seen ln :Crohnt disease'


,,.t a feature of neonatal n;jecrotizing enterocolitis: lncreased bowei sound (bowel sounds are decreased or absent).
* portant features of necrotizing enterocolitis: Abdominal distension, diminshed bowel sound, pneumatosis intestinalis, pneumop-
=' toneum, metabolic acidosis.
-,eatment of choice for perforated necrotizing enterocalitis : Paracentesis (peritoneal drainage) followed by definitive surgery.
)efinitive diagnostic test for cause of chronic malabsorption: lntestinal biopsy.
:zotures of coeliac disease: Villous atrophy, crypt hyperplasia, inflammatory cells in lamina propria, maintained mucosal thickness'
,:ot (mucosal thickness is normal).
a feature of coeliac disease: lncreased thickness of mucosa

,r,roi*ion rhould be avoided in coeliacdiseose : wheat. oa! barley, rice (but not maize).
leliac disease is associated with : HLA-DQ2 (mc), HLA-DQB (2'd mc), HLA-88, HLA-DR3, HLA-DR7'
;";r r;;;;, aunrirncy after infectious enteritis: Lactose'
"nry*o
a for lactose deficiency and lactose intolerance : Hydrogen (H, ) breath test.
-est
(2"d mc).
* l.4ost common cause of cholestaticjaundice in newborn: ldiopathic neonatal hepatitis (rnc), extrahepatic biliary atresia

lmportant features of extrahepatic cholestasis (e.g. bitiary atresla) : Raised alkaline phosphatase, 5-nucleotidase and
GGT.

disease.
Giant cells are seen in : Neonatal hepatitis,, EHBA, q1 antitrypsin deficiency, viral infections, Niemann-Pick
Diagnostic procedure for extrahepatic biliary atresia (EHBA): Percutaneous liver biopsy.
(EHPVO).
Most common cause of portol hypertension in children in lndia: Extrahepatic portal venous obstruction
Most common cause of massive hematemesis in a child: Portal hypertension.
t Most common cause af diarrhea in clrildren: Rotovirus.
(but not reovirus)'
& lmportant viral causes diarrhea in children: Rotovirus, calcivirus adenovirus, astrovirus, norwalk virus
I Most common bacterial cause of diarrhea in children in lndia:ETEC'
Parasites causing diarrhea in immunocompromized child: Giardia, cryptosporidum.
n commonest couse of ab:dominal pain in children: Behavioral (functional) pain.
Most common cause of abdominal pain in an infant: colic (evening colic).

Evening colic usually disappears at the age of : 3-4 months.


atresia, o-1 antitrypsin deficiency,
Common causes of cirrhosis in children:Viralhepatitis (HBV HCV), idiopathic neonatal hepatitis, biliary
Wilson,s disease, hemochromatosis, galactosemia, Cystic fibrosis, and parasites (opisthorchis sinensis, fasciola).
(GERD) in children : Regurgitation.
6 Most common and earliest symptom of gastroesophageal reflux disease
E To diagnose GERD in children ; Regurgitation plus one of the following : Respiratory symptoms/failure to thrive/poor weight gain.
j Double bubble sign in children: Duodenal atresia, Ladds band, annular pancreas, pancreatic head
tumor, pancreatic pseudocyst'

Childhood cholilithiasis is seen ln: Neimann Pick's disease'


e Most common cause pancreatic pseudocyst in children : Acute pancreatitis due to trauma.
a Most common Gl malignancy in children: Lymphoma'

& Causesof bloodyvomitusisneonates:Stressgastritis,hemorrhagicdiseaseof newborn,swallowedmaternal blood'

For anorectal malformation x-ray (invertogram) is done: At 12-24 hours after birth.
* o6r",r* oirr*,m,arcln UuOAte on anterotat-usG; Fetal esophageal atresia'
w Home made emiting agentfor children in suspected poisoning :syrup of ipecac.
Most common cause of vomiting in neonate; Aerophagy.
& Most common cause af bleeding per rectum in infants:Anal fissures'
Most common indication of liver transplantation in children: Biliary atresia.

xxx

d
QUESTIONS

TRACH EO.ESOPHAG EAL F ISTU LA MEGACOLON


l. A newborn has dribbling after feeds. He has respira- 8" Acquired megacolon in children most commonly
tory distress and froths at the mouth. Diagnosis is - due to - @ll lnrlia Dec.t4 pattern
(Ar 01) a) Psychological problems
a) Tracheoesophageal fistula b) Bad bowel habit
b) Tetralogyoffallot c) Chagas disease
c) Respiratory distress syndrome d) Hirschsprung's disease
d) None of the above 9. Aganglionic segment is encountered in which part
) A newborn batly had normal APGAR score at birth of colon in case of Hirschsprung's disease -
and developed excessive frothing and choking on (CET Nov.14 Pattern)
attempted feeds. The investigation of choice is - a) Distal to dilated segment
(AIIMS May 03) b) In whole colon
a) Esophagoscopy c) Proximal to dilated segment
b) Bronchoscopy d) In the dilated segment
c) MRI chest
10. A male infant presented with distension of atrdomen
d) X-ray chest and abdomen with the red rubber
shortly after birth with passing of,Iess meconium.
catheter passed per orally into esophagus
Sutrsequently a full-thickness biopsy of the rectum
was performed. The rectal triopsy is likely to show -
CONGENITAL HYPERTROPHIC PYLORIC STE- (AIIMS Nov 04)
NOSrS a) Fibrosis of submucosa
b) Lack ofganglion cells
3. Congenital hypertrophic pyloric stenosis usually c) Thickened muscularis propria
presents - @IIMS May 0a)
d) Hyalinisation of the muscular coat
a) Within 2 days after birth 11. True statement regarding Hirschsprung,s disease-
b) Around 1 week after birth (AIIMS lune 99)
c) Around 2 weeks after birth a) Giant ganglia are present
d) Around 2 months after birth b) Mucosa is involved and show foldings
4. The metabolic derrangement in congenital pyloric c) Manometry excludes the disease
stenosis is - (AIIMS Nov 02,06) d) Rectal biopsy is contraindicated in infants
a) Hypochloremic alkalosis 12. Failure to pass meconium within 48 hrs of birth in
b) Hyperchloremic alkalosis a newborn with no otrvious external atlnormality
c) Hlperchloremic acidosis should lead to the suspicion of - (AIIMS Nov 02)
d) Hlpochloremic acidosis a) Anal atresia b) Congenital pouch
A 40 year old male presents with recurrent bouts of colon
vomiting since 9 months because of pyloric obstruc- c) Congenital aganglinosis d) Meconium ileus
tion, The compensatory biochemical change is- 13. Hirschsprung disease is confirmed by -
(AIIMS May 01) (All India Dec.13 Pattern)
a) RespiratoryAlkalosis a) Rectalbiopsy
b) Respiratory acidosis b) Per/Rectalexamination
c) Paradoxical aciduria with hyponatremia and c) Rectal manometry
hlpochrorenia d) X-ray abdomen
d) Metabolic acidosis t4" True about Hirshprug's disease - (PGI Nov 14)
6. A. 2months old boy presenting with palpatrle a) Aganglionic segment is contracted not dilated
abdominal mass in the epigastrium. The clinical b) Descending colon is most common site
diagnosis is (There is no bile in vomitus)- c) Barium enema is diagnostic
(NEET Dec.12 Pattern) d) Barium enema shows calciflcation
a) Duodenal Atresia b) Choledochal cyst 15. Hirschsprung disease, marker - used for early diagrro-
c) Pyloric stenosis d) Oesophageal Atresia srs - (All India Decl 5 Pattern)
7. 3 month infants with abdominal palpable mass & a) Acetyl cholinesterase b) Adrenaline
non trilious vomiting - (All India Dec.l j pattern) c) VIP d) None of above
a) Hypertrophic biliary stenosis
b) Hypertrophic pyloric stenosis
c) Tracheoesophageal fistula
d) Duodenal atresia
HTESTINAL OBSTRUCTION diagnosis - (Ar 11)
a) Rectal Polys
,in. Most common cause of Acute Intestinal Obstruction b) Intussuception
in neonates is - (NEET Dec.12 Pattern) c) Meckel's Diverticulum
a) Rectal atresia b) Malrotation " .d) Necrotizing Enterocolitis
c) Duodenal atresia d) Acute Intussusception
Which one of the following is most suggestive of IN FLAMMATORY BOWEL DISEASE
neonatal small bowel obstruction - (4t 03)
a) Generalizedabdominaldistension 25. Common site of regional enteritis is -
b) Failure to pass meconeum in the first 24 hours (NEET Dec.12 Pattern)
c) Bilious vomiting a) Colon
d) Refusal offeeds b) Rectum
iF. Most important cause of abdominal distension in c) Distal ileum and colon
intestinal obstruction - (CET lune 14 Pattern) d) Caecum
a) Cases produced by bacterial activity 26. Skip granulomatous lesions are $een in -
b) Cases diffused from blood (All India Dec.13 Pattern)
c) Swallowed air a) Ulcerative colitis
d) Products ofdigestion b) Crohn's disease
x9. A neonate presented with fever, lethargy, abdominal c) Whipple's disease
distension, vomiting and constipaton. Clinically he d) Reitert disease
was fiagnosed as volvulus neonatarum wifh suspect- 27- ?seudopollps are features of -
ed perforation. Best investigation would be - @I 10) (All lndia Dec.14 Pattern)
a) Plainx-ray a) Crohn's disease
b) Barium enema b) Ulcerative colitis
c) Upper GI endoscopy c) Celiac sprue
d) Barium meal follow through d) Whipple's disease
28. Toxic megacolon is seen ia - (All lndia Dec.14 Pattern)
INTUSSUSCEPTION a) Chronic nonspecific ulcerative colititis
b) Crohn's disease
20. Commonest cause of intestinal obstruction in chil- c) Colonic diverticulosis
dren is - (NEET Dec.12 Pattern) d) Hamartomatous pollp
a) Intussusceptionb) Volvulus 29. In which of the following conditions the lead pipe
c) Hernia d) Adhesions appearaoce of the colon on a barium enema is seen -
2t. A l0 month old infants presents with acute intes- (AIIMSNov 04)
tinal obstruction. Contrast enema X-ray shows the a) Amoebiasis
intussusceptions, likely cause is - (Ar 02) b) Ulcerative colitis
a) Peyer's patch hypertrophy c) Tu.berculosis of the colon
b) Meckle's diverticulum d) Crohn's involvement of the colon
c) Mucosal polyp 30. An eight year old boy had abdominal pain, fever
d) Duplication cyst with bloody diarrhea for 18 months. His height is
.,,, 100 cms and weight is 14.5 kg. Stool culture was neg-
A 6 months oldbabywith H/O bloodydiarrhoea of
2 days duration with abdominal distension and on ative for known enteropathogens. The sigmoidosco-
examination the baby screams, diagnosis is - pywas normal. Duringthe same period, childhad
(PGI Dec 03) an episode of renal colic and passed urinary gravel.
a) Intussuscetion The mantoux test was 5 x 5 mm. The most probable
b) HUS fiagnosis is - (AI 03)
c) Appendicitis a) Ulcerative colitis
d) Ac. Enterocolitis b) Crohn's disease
23. Recurrent obstruction, mass per rectum and diar- c) Interstinaltuberculosis
rhoea in child - (PGI June 2000)
d) Strongyloidosis
a) Intussuception b) Rectal prolapse
c) Internal hernia d) Haemorrhoids NECROTIZI NG ENTEROCOLITIS
24. A previously healthy infant presents with recurrent
episode of abdominal pain. The mother says that the 31. True atrout nenatal necrotizing enterocolitis -
(All lndia Dec.l j Pattern)
child has been passing altered stool after episodes
of pain, but gives no history of vomiting or bleeding a) Abdominaldistension
per rectum. Which of the following is the most like$ b) Pneumoperitoneum
c) Decreased bowel sounds CELIAC DISEASE
d) All of the above
32. All are features ofneonatal necrotizing enterocolitis 38. The following cereals should be avoided in patients
except - (Ar e8) with celiac diseases, except - (AllMS Nov 03)
a) Abdominal distension b) Increased bowel sound a) Wheat b) Barely
c) Metabolic acidosis d) Pneumoperitoneum c) Maize d) Rye

A premature neonate on top feeding develop ab- 39. Antiendomysial antibody is used in screening of -
dominal destension and bleeding per rectum. He (All India Decl, Pattern)
recently recovered from acute upper respiratory a) Myasthenia gravis
tract infection. What is the probable diagnosis - b) Auto immune hepatitis
(PGI Dec 08) c) Coeliac disease
a) Necrotizingenterocolitis d) Graves disease
b) Vahulus 40. The histological features ofcoeliac disease include
c) Meckeltdiverticulum all of the following Except - (CET Nov.14 Pattern)
d) Intussusception a) Crlpt hlperplasia
A newborn sufiering from perforated necrotizing b) Increase in thickness of the mucosa
enterocolitis is having very poor general condition. c) Increase in intraepithelial ll.,rnphocytes
He is currently stabilized on ventilator. Which of the d) Increase in inflammatory cells in lamina propyria
following should be done in tlre management of this 41. Gluten sensitive enteropathy is most strongly associ-
patient - (Ar 08) ated with - (All India Dec.14 Pattern)
a) Conservativetreatment a) HLA-DQ2 b) HLA-DR4
b) Resection and anastomosis c) HLA-DQ3 d) Blood group'B'
c) Stabilization with membrane oxygenator and defer In celiac disease A/E - (PGI lune 2000)
surgery a) Gliadin is cause
d) Peritoneal drainage by putting drains in the flanks b) Associated with HLA-BS
35. A neonate is suspected to be suffering from necro- c) Decreased villi to crypt ratio
tizing enterocolitis Nrc). On further examination d) Increased brush border
and investigation, he is diagnosed to be Bell's stage I
NEC. The management of choice would be -
LACTASE DEFICIENCY
(AttMS May 12)
a) Laparotomy and proceed
43. A boy comes with complains of vomiting, bloated
b) Insertion of bilateral pelvic drains
abdomen and abdominal pain. He has history of
c) Conservative management with IV fluids and
attending ice-cream eating competition last night.
antibiotics
He also has past history of similar episodes follow-
d) Initial conservative management and laparotomy
ing ingestion of milk and milk products. The likely
after 24 hours
cause - (AllMS Nov 99)
a) Pancreatic amylase deficiency
MALABSORPTION b) Lactase deficiency
c) Salivary amylase deficiency
36. A 12yeadold girlhashistoryofrecurrentbullcy d) Foodpoisoning
stools and abdominal pain since 3 year of age" She One of the intestinal enzymes that is generally
has moderate pallor and her weight and height are deficient in children following an attack ofsevere
below the 3rd percentile. Which of the following is infectious enteritis is - (Ar 0s)
the most appropriate investigations to make a spe- a) Lactase b) Trypsin
cific diagnosis ? (AllMS Nov 04)
c) Lipase d) Amylase
a) Small intestinal biopsy
Lactose intolerance in a suspected patient is diagnosed
b) Barium studies
by giving an oral load and measurirrg concentrations
c) 24hrs fecal fat estimation
of? (CET Aug.1j Pattern)
d) Urinary d-xylose test
a) CO2
37. In children presence ofincreased fecal fat excretion b) H2
and increased fccal nitrogen levels is a feature ofAll c) One/two carbon compounds
Except - (At ee)
d) N2
a) Pancreatic-insufficency
b) Bacterior overgrowth syndrome
c) Coeliac sprue CHOLESTASIS
d) Ulcerative-colitis
In a child presenting with obstructive ]aundice all
afe seen except - (AIIMS Nov 06)
a) Gamma glutamYl transPePtidase turmeric colored urinc and pale stools since birth.
b) Alkalinephosphatase Examination reveals liver span of l0 cms. The most
c) Glutamatedehydrogenase specific investigation for establishing the diagnosis
(At 03)
d) 5'Nucleotidase wouldbe -
a) Liver function tests
{-. In neonatal cholestasis, if the serum gamaglutamyl-
b) Ultrasound abdomen
transpeptldas€(gamma GTP) is mote than 600 IU/L the
(AI 04, AllMSNov 02)
c) Peroperative cholangiogram
most likely diagnosis is -
d) Liver
a) Neonatal hepatitis b) Infection lirpty
c) Sclerosing cholangitis d) Biliary atresia
{& Neonatal cholestasis is seen in - (AIIMS Iune 98) PORTAL HYPERTENSION
a) Chronic hepatitis
b) HepatitisB&C 55. Portal hyfiertension in children in India is common*
ly due to - (AllMS Nov 0j)
c) Glycogen storage disorders
a) Indian childhood cirrhosis
d) Mucopolysacchroidosis
b) Extrahepatic portal venous obstruction
49. Most cornmnn caus€ of cholestatic janudice in new c) Idiopathic portal hlpertension
born is - (NEBT Dec.12 Patter-n)
d) Hepatic out flow tract obstruction
a) Hypoplasia ofbiliarYtract
b) Neonatal heptitis 56. A 12 year old boy presents with hemetemesis,
malena and mild splenomegaly. There is no obvious
c) Choledochal cYst jaundice or ascitis. The most likely diagnosis is -
d) Physiological (AI 11)
50. The most common genetic cause of liver disease in a) EHPVO b) NCPF
(AI 02)
children is - c) Cirrhosis d) Malaria with DIC
a) Haemochromatosis b) o1 antitrlPsin
57. Most common €ause of portal hlpertension in chil-
deficiency (NEET Dec.12 Pattern)
d) dren is -
c) Cystic fibrosis Glyocgen storage
a) ExtrahepaticcomPression
disease
b) Budd chiari syndrome
c) Veno-occlusive disease
NEONATAL HEPATITIS d) Post necrotic
58' A 7 year old girl from Bihar presented with three
51. A neonate presents with jaundice and clay coloured
episodes of massive hematemesis and melena' There
stools. Liver Biopsy shows giant cells diagnosis is -
(AIIMS Nov 01, AI 01)
is no history of jaundice. On examination, she had
a large spleen, non-palpable liver and mild ascites'
a) Neonatal hepatitis with physiological iaundice
Portal vein was not visualized on ultrasonography'
b) Neonatal hepatitis with extrahepatic biliary atresia
tiver function tests were normal and endoscopy re-
c) Physiological jaundice
vealed esophageal varices. The most likely diagnosis
d) Physiological jaundice with extrahepatic biliary
is - @t 03)
atresia
a) Kala azar with portal hypertension
52. A neonate isbeinginvestigatedforiaundice' Aliver b) Portal hlpertension of unknown etiology
biopsy shows features of a "Giant CelUNeonatal c) Chronic liver disease with portal hypertension
hepatitis". Which one of the following conditions d) Portal hypertension due to extrahepatic
(AIIMSNov 04)
usually results in this case - obstruction
a) Congenital hePatic flbrosis
Ramu, a 8 yr old boy presents with upper Gl bleed-
b) Hemochromatosis
ing. On examination, he is fouad to have splenomeg-
c) Alpha-1-antitrypsin deficiency
aly; there are no signs of ascites, or hepatomegaly;
d) Glycogen storage disease TYPe 1
esophageal varices are found on UGIE Most likd
(AI o1)
diagnosis is -
EXTRAH EPATIC BI LIARY ATRESIA a) Budd chiari syndrome
b) Non cirrhotic portal hlpertension
53. A 2 months old exclusively breast fed child develops c) Cirrhosis
jaundice since birth, acholic stool high coniugated d) Veno-occlusive disease
Lilirubin in blood and absent urobilinogen in urine' 60. A child is brought by mother with H0 massive
(AllMS Nov 99)
The likely cause is - hematemesis with HO drug intake previouslywith
a) Hlpothyroidism NSAIDS and on Rx. Associated $'ith moderate sple-
b) Congenital biliarY atresia nomegaly diagnosis is - (PGI Dec 06)
c) Neonatal hePatitis a) Oesophageal varices b) Duodenal ulcer
d) Breast milk jaundice c) Drug induced gastritis d) Peptic ulcer
54. A 2 month baby presents with history of jaundice'
61. Most common caltse of seyere hemateraesis in a ABPOMINAL PAIhI
child is- (All India Dec.14 Pattern)
a) Portal hypetension b) Peptic ulcer 70. Recurrent atrdornianl pain in children in most often
c) Mallory weiss syndrome d) None of the above due to - ArrMS s3)
a) Roundworms
LIVER CIRRI.IOSIS b) Emotional/hebaviourai problems
c) Amoebiasis
62. Cause of liyer cirrhosis in childhood include- d) Giardiasis
(PGr 81, 84) 71. Comrnonest cau$e of abdominal pain in children is -
a) Alpha-1 antitrypsin deficiency (PGt 87)
b) Coeliac disease a) Porphyria b) Worm colic
c) Phenylketonuria c) Lead poisoning d) Appendicitis
d) Cow's milk intolerance 72. Colicgenerallydisappearbyage- (L/psc83, AMCBi)
a) 1 year b) 2 years
DIARRHE,q c) 4 months d) 8 months

63. The most comrnon cause of diarrhea in children is -


MISCELI.AruEO[J5
(All India Dec.11 Pattern)
a) Vibrio cholerae 73. Most common indication for liver transplant in
b) E. coli
children is - (CET July 15 pattern)
c) Rota virus
a) Viral hepatitis with fulminant hepatic failure
d) Pneumococcus
b) Biliary atresia
04. All can cause diarrhoea excef)t - (AIIMS lune 98) c) Metabolic disease
a) Rota virusb) Calci virus d) Hepatic tumor
c) Reo virusd) Adenovirus 74. Most cornmon GI rnalignancy of childhood -
65. A 6-month-old infant present$ to the'diarrhoea (All India Dec15 Pattern)
clinic'unit with sorne dehydration, The most likely a) Adenocarcinoma b) Lymphoma
organi$m cau$irlg diarrhea is - (AIIMS Nov 03) c) Sarcoma d) Carcinoid
a) Entamoebahistoiytica 75. Chronic coastipation in children is seen in all AiE-
b) Rotavirus (NEET Dec.12 pattern)
c) Giardia lamblia a) Hirschspringdisease
d) Shigella b) |ejunal polyp
65" The most comrnon bacterial cause for diarrhea in c) Hlpothyroidism
children in India is - (CET Nov.lt pauern) d) Stricture
a) Enterotoxigenic E.coli (ETEC) 76. &lost commoo cau6e of per rectal trleeding in infant
b) Enteropathogenic E.coli (EPEC) 1S- (All India Dec15 Pattern)
c) EnterohaemorragicE.coli (EHEC) a) Anal fissure b) Rectal poil.p
d) Vibrio Cholerae c) Intussusception d) Hypertension
67" A child is diagnosed to haye acute gastroenteritis. 77. Childhood cholelithiasis is seen in-
'
The consultingpediatrician wants to send a stool
(AIIMS lune 98)
sample to a lab which is 16-18 hrs, away. Which of
a) Hurrer Syndrome
the following mediums should he use to send this b) Mucopolysaccharidosis
sample - (AIIMS Noy 2000)
c) Neimann Pick's disease
a) Charcol cotton bud d) Autoimmune hepatitis
b) Carry Blair medium
c) Sterilized jar
d) A medium with high CO2 content QI'ESTIONS OF VARIOUS OTHER EXAMINA-
68- Intractable diarrhoea in children is caused by all TIONS
€xcept - @Gl lune 2000)
78. Pseudopancreatic cyst in a child is commonly due
a) Cystic fibrosis b) Giardiasis
c) Secreting tumors to- t at qqt
d) Milk allergy
a) Annularpancreatitis
69. Profuse watery diarrhea ia an imrnuno compro-
b) Drug induced pancreatitis
rnised child is due to - (NEET Dec.12 pattern)
c) Traumatic pancreatitis
a) Crlptococcus b) Amoeba d) Choledochal cyst
c) Giardia d) Lactose intolerance
79. A neru trorn with recurrent vomiting, cyano$is aft€r
each feed is likelyto tre ouffering from - (AppGE0s)

ffi
i:.iu i:::r;r,rii. :ir'.i.1r.::ir:l
:r'if ..,. :.i, ri)t:ita|1.11::,:,titiiri:,

a) Tracho oesophageal fistula c) Rotavirus induced diarrhoea


b) Tetrologyoffallot d) Intestinaltuberculosis
c) Congenital hlpertrophic pyloric stenosis 85. To indrice vomiting at home in a child who has
d) ARDS ingested a poison, the recommended agent of choice
80. A young boy presents with failure to thrive. Bio- wouldbe - (Comed 05)

chemical analysis of a duodenal aspirate after ameal a) Oral rehydratibn solution


reveals a deficiency of enteropeptidase (enterokinase). b) Mustard in warm water
The Ievels of which one of the following digestive c) Apomorphine
enzymes would be afiected ? (UPSC 07) d) Syrup ofipecac
a) Amylase 87, The commonest cause of vomiting in a one month
b) Pepsin old infant is - (Karn 06)
c) Lactose a) Pyloric stenosis
d) Trlpsin b) Cardiac chalasia
E I . I4 year old girl with history of abdominal pain c) Aerophagy
(periumbilical), postprandial, passing blood in stools, d) Gastro-esophageal reflux
feveS weight loss since ten months. She also has 88. ln necrotising enterocolitis earliest change seen in
episodes of passing blood in stools. What maybe the X-ray abdomen is - (Kerala 96)
likely diagnosis? (St. lohns 02)
a) Non specific bowel dilatation
a) Chronic appendicitis b) Gas in the intestinal wall
b) Chronicpancreatitis c) Gas in splenic flexure
c) Crohn's disease d) Ground glass appearance
d) Bulimia e) None
82. X-ray detect congenital anorectal malformation at - 89. Which of the following is a charactersistic radiologi-
(uP 07)
cal finding in neonatal necrotizong enterocolitis -
a) Immediately after birth a) Gas in the portal system (lipmer 2K)
b) 24-48 hours b) Gas in the intestinal wall
c) 48-72 hours c) Pneumoperitonium
d) After T2hours d) Air fluid levels
83. Intrahepatic cholestasis is seen in - (MP e8) gO. Pneumatosis intestinalis is most often seen in -
a) Galactosemia a) Neonatal necrotising enterocolitis (Karn 95)
b) Hypercalcemia b) Midgut valvulus
c) Haemochromatosis c) Meconium peritonits
d) Cystic fibrosis d) Neonatal visual perforation
84. Absent stomach bubble on antenatal ultrasonogra- gl. Most common bacteria causing diarrhea in children
phy is an important finding for antenatal diagnosis in India is - (Karn 11)
of- (UPSC e6)
a) Enterotoxigenic E. coli
a) Congenital heart disease in the foetus b) Enteroinvasive E. boli
b) Oesophageal atresia in the foetus c) Enterohemorrhagie E. coil
c) Omphalocele in the foetus d) Enteropathogenic E. coil
d) Spina bifida in the foetus 92. Oesophageal atresia may occur as a part of VACTER
85. A two year old child presents with persistent diar- group of anomalies. What does'TE'stand for?
rhoea acidic stools and presence ofone per cent of a) TetralogyofFallot (uP sc II 10)
reducing substance in the fresh stools. What is the b) Thoracic empyema
most probable diagnosis ? (UPSC 07) c) Tracheo-oesophageal fistula
a) Cystic flbrosis d) Talipes equinovarus
b) Lactose intolerance

IIT
ANSWERS
TRACFI EOESOPHAGEAL FISTU LA

t. Ans. is 'a' i.e., Tracheoesophageal fistula iRef o,R Ghai }e/e p. rz6
& vhlc p.Jsr; Nelson rrn,le p. rs43, rs44l
e/F of tracheo esophageal fistula -
a) The newborn regurgitates all ofits first and subsequent feeds.
b) Saliva pours almost continuously from the mouth of the infant (This is'the sign of tracheoesophageal atresia.
It does not occur in any other condition)
c) Attacks ofcoughing and cyanosis occurs on feeding.

z. 'd' ite., X-ray chest and abdomen


tft'i:
0.P- Ghai p, 176 dt Vhle p.351; Nclsom
with the red rubber catheter passed per orally into esophagus
[Reli
8.th/e p. 1543, tS44J
t}'h/e
r This is a case oftracheo esophageal fistula.
r If a nasogastric tube is tried to pass into the infant it will come against an obstruction
within 10 cm. If this occurs
the diagnosis is virtually certain.
r Radiological confirmation of diagnosis -
a) A lateral chest X-ray -+ It shows a lucent proximal pouch that may displace
trachea anteriorly.
b) X-ray abdomen -+ Gas in the stomach is seen if there is communication between
the iower part of the esopha
gus and trachea which occurs in the commonest variety of tracheo-esophageal
fistula.

CONGENITAL HYPERTROPHIC PYLORIC STENOSIS

? Ans. is t' i.e., Around 2 weeks after birth tRd o"p. Gh6.i Sele p. 279 & Vele p.253; Nslso,l tSthlc p. 1555]
t The symptoms vomiting usually starts after 3 weeks of age but
symptams may develop as eaily as 1,, week of life
and as late as 5 months of life.
- Nelson
4. Ans. is 'a' i.e., Hypochloremic alkalosis [Rel a,p, Ghai 8lhle p. & I'lc p.253; Nelson
229 rSth/e p. tsss, .r5s6]
r Due to vomiting, there is progressive loss of -+ Fluid, Hydrogen ion,
chlorides
r This results in hypochloremic metabolic alkalosis.
5. Ans' is t' i.e., Paradoxieal aciduria with hyponatremia and hypochloremia
tRef o.p" Ghei Sthlep. zz9 & ple
p.253; Alelson tSth/e p, tSSZ)
r Persistent vomiting leads to loss of large amount of gastric fluid.
r Gastric fluid consists of large amount of HCt.
r Thus in pyloric stenosis, repeated vorniting causes hypochloremic metabolic alkalosis.
o In earlystages the kidneys are able to compensate by reabsorbing hydrogen
and chloride ions in exchange for
sodium, potassium and bicarbonate, however as the body stores of sodiirm
and potassium becomes depleted,
these ions are selectively retained while hydrogen is excreted in urine,
causing the paiadoxical aciduria.
r This aggravates the alkalotic stage.

6. Ans. is t' i.e., Pylorie stenosis B*iley (r Lave p. 899; O"p, Ghe.i g,hle p. 229
fRef: Z3,,Ue & Vhle p. 253; Nelson trpthle p.
r55sI
Diagnosis of congenital hypertrophic pyloric stenosis
r History of vomiting without bile (nonbilious vomiting).
r Physical examination -
1) Palpable mass in the epigastrium
ii)After feeding, visible gastric peristatic wave that progresses across the
abdomen.
7. Ans. is'b'i.e., Hypertrnphic pytr,ric stenosis [Rs Netrscn tvhre chapter 326]

r See above explanation.


MEGACOLON

n. Ans. is 'b'i.e., Bad bowel habit [Ref Bailey 6 Lave 24tu1e p^ ttl6)
o Acquired megacolon is caused by
r Poor bowel habit r Infections -+ chaga's disease
r Idiopathic r Medication Antipsychotic drugs
9' Ans. is'd i.e., Distat to dilated segment [Ref: Nelson l8thle p" 1565, 1566;
O,p. Ghai Sthlep. 2851
c An intestinal segment lacks both Meissner submucosal and Auerbach
myenteric plexuses. This loss of enteric neural
coordination leads to functional obstruction and intestinal dilation pro*imal
to the affected segment.
10. Ans. is 'b' i.e., Lack of ganglion cells lRe! Nelson r8n/e p. 1566, 1567; op Ghai gthlep. 2BSl
"Whenever neonate presents with delayed passage of stool,
Hirscfisprung's disease should be suspected,,.
o Rectal suction biopsies demonstrate _
i) Hypertrophied nerve bundles
ii) Absence of ganglion cells
ll. Ans. is t'i.e., Manometry excludes the diagnosis [Rel Robbin, s vh/e p. g0s; chandrasoma Taylor 3d/e p.59]
o Anorectal manometry is a useful screening procedure for
the diagnosis of Hirschsprung,s disease . A normal
response in course of manometric eualuation precludes diagnosis
of Hirsihsprung! disease.
o However the confirmation of diagnosis in case of Hirschspru.rg'.
dir.ur. is d'one by rectal biopsy.
12. Ans. is t' i.e., Congenital aganglinosi s [Re! Nelson IB,h/e p. tS66]
o Read whrtNelson says about Hirschsprung's disease (Congenital
aganglinosis).
o "Ninety nine percent of full term infants pass meconiui withii sairs
of Lirth. Hirschsprung disease should be
suspected in any full term infant with delayed passage of stool,,.

13. Ans. is 'a'i.e., Rectal biopsy fRef: Netson tBth/e Chapter 329,j1
o Rectal suction biopsyis procedure ofchoice.

14' Ans. is 'd i.e., Aganglionic segment is contracted not dilated


tRel Nelson 18th/e p. 1566, 1567; a.p, Ghai 8e/e p.
28sl

15. Ans. is'a'i.e., Acetyl cholinesterase [Re, Nelson lgth/e ch. 329.3; Histochem, 1990]

INTESTINAL OBSTRUCTION

16. Ans. is t'i.e., Duodenal atresia


[Ref: Internet]
o Most commolt cause of neonatal intestinal obstruction
--> lejunoileal atresia
o Second most common cause of neonatal intestinal obstruction
--> Duodenal atresia
17. Ans. is t' i.e., Billous vomiting
"Billous vomiting is the most common symptom,,.
Internet
t8. Ans. is t'i.e., Swallowed air [Rel Harrison tdhle p. tk03l -
r In intestinal obstruction distension of the intestine is caused by accumulation
the obstructed segmented.
t - --- of gas and fluid proximal to and within

c 70-80o/o of this intestinal gas consists of swallowed air.

19. Ans. is '* i.e,, Plain X-ray fRef: lones clinical pediatric surgery Sth/e p. 40]
o Barium contrast studies withtluoroscopy (Barium mealfollow
through) is the investigation of choiceas X-ray is not
reliable in early stages ofvoh,ulus.
o Horvever, if the perforation is suspected, barium contrast should
not be used because of risk of perotinitis . water
soluble contrast studies (iodine based) should be used. Plain x-ray of abdomen/chest
is the initial investigation to
demonstrate free air under the diaphragm in suspected. cases of pirforation.
C H.+prrn 6 Gastrointestinal Tract
INTUSSUSCEPTION

2t" Ans" is'a'i.e., trntussusception


[Ref, l{elsor' l&tkle p. }569}
c Intussusception is the most common
cause of intestinal obstruction between
3 months and 6 year of age,
2t. Ans. is 'f i.e., Feyer's patch hypertrophy
[Ref. Nelsan tSth/e p. 1242]
.llt:I"":*i:1*,Tg:.ry1l:slinitinfectionorrhe-introdu.tio.,ofnewfoodproteinsresulr
in swollen
::,
causing *: j:':,T:1"::::3:
an intussusception.
prominent mounds or tissue reads to ;";; ;;fi#: ;;lil: ileum intopeyer's patches
the colon thus
22. Ans. is 'l i.e.,Intussuscep tian
[Ref : Nelsan lgth/e p. 1569]
Clinical features of intussusception
o Sudden onset
c severe paroxysmar coricky pain that recurs
at frequent intervars.

: i:iilJ#t"mpanied
by straining efforts with legs'and t r...
n.*.a and toud cries (screaming).
o Blood and mucus produce a current jelly
stool.
o Reflex vomiting is an early sign.
c In later stages vomiting becomes bile stained.
c Abdominal distension and tenderness
develops as intestinar obstruction becomes
e Tender sausage - shaped palpable mass.inthe more acute.
right upp.. ubdolnen with its long axis
* Occasionally intestine prolapses through rectum] cephalocaudal.

23. .dns. is 'i i.e.,Intussuceptian [Ref : Nelsan lgth/e p. 15d9]


o Intussuception may cause
r Diarrhea with current jelly stool
a Recurrent obstruction
r Mass per rectum due to advancing intestinal prolapse.
About other options
o Rectar prolapse, internar hernia and
haemorrhoids do not cause diarrhea.
24' watker's Pediatrie Gastrointestinat Disease
*:;r:;y, W"r::;:i";;r[Ref: Sth/e p. 1ii4, t3ts, t3t5; Asherafi
r
The infant in question has:
i)Recurrent episodes of abdominal pain
ii)Passing ofaltered stools after episodes
ofpain
iii)No vomiting or bleeding per rectum.
o Most common presentation of rectal
pollps and Meckel's diverticulum is painless
ruled out) rectal bleeding (option a & c are
e Necrotizing enterocolitis is seen in premature
infants and is manifested in-neonatal period.
Symptoms progress
vomiting, Iethargv, p"";f.;;;;io
pur.ug. of grossly broody stoor *a r.ut,r"i oi.r,o.r.
i:;T#i:1fi'.,Htension'
' Most common presentation is recurrent
abdominal pain (paroxysmal clocky
il:X;:i::i:1.I-'intussusception'
o The absence ofhistory ofbreeding per
rectum is creating some confusion here.
o However' altered stooi should ralse the
suspician of intuisuception because intussuception
"current jeily stool" (altered tlpically present with
stool) representing a mixture of blood,
I So, amongst the given options, best
answer is intussusception.
mucus and stool.

INFLAMMATORY BOWEL DISEASE

25' Ans' is t' i'e" Distal ileum and colert


rRef Nerson rSrhre p. 158a, 15g1; Rabbins P,re p.
:
sa7]
o Any portion of intestine can be involv
ed,but most commotnly small intestine
is involved.
26. Ans. is 'b' i.e., Crohn2s Disease
{ReJ.: Robbirc,s Z*le yt. B4Zl
o A classic feature of cD is the sharp
demarcation of diseased bowel segments
o when multiple bowel segments are i-nvolved, from adjucent uninvolved bower.
the intervenr"gi"*.1 is normal + skip
lesions.
Anc, ia 'U i.e,,M*ratiye calltis {Ref : Netson t?tu/e p. 1575, tS77; Robblns Zh/e p. 549)
o Pseudopolyp is seen in UC.
]E- Aas is'i i.e.,Cbranicmaaspecifrculcen*ivecolitir f%ef: Nelson 18h/e p. 1578; Robbins 7tu/e p, 849)
o In sever cases of UC, toxic damage to the muscularis propria and neural plexus
lead to complete shutdown of
neuromuscular function -) colon progressively swells to create toxic megacolon.
t9. Ans. is ld'i.e., crohn's involvement of the colon [Rel www.springerlink.com)
On barium enema CD shows hose pipe (lead pipe) appearance.
j'CI. Ans. is'b'i.e., Crohn's disease [Ref Nelson lgth/e p. 15g1, 15g2)
See each option one by one.
Option a
r In this child sigmoidoscopy is normal.
r Ulcerative colitis almost always start from rectum.
r So, sigmoidoscopy is not normal in UC.
Option b
o All the given clinical features of this child can occur in CD.
o Renal colic is due to renal stone.
Option c
r Montoux is negative.
r In a child less than 2 years of age montoux is strongly positive if the child has tubercular infection.
Option d
r Stool culture is negative for enteropathogens.
o So, strongyloidosis is excluded.

H ECROTIZING ENTEROCOLITIS

31. Ans, is *ff i"e", AJlotth* abav* {Ref; Netson 1*/e p. 755; Ghai ft/e p. ti9)

rl'l'
j,!.
'{'

32. Ans. is 'H i.e., Increased bowel sound [Ref: O.p. Ghai 8th/e p. 165 6 Vh/e p. 139; Nelson l8n/e p. Z55l
c Bowel sounds are diminished or absent.

33. Ans. is 'd i.e., Necrotizing enterocolitis [Rel o.p. Ghai 8th/e p. 165 dr vh/e p. t3B, 1391
r This child is -
i) Premature
ii)
On artificial feeding (top feeding)
iii) Having abdominal distention and bleeding per rectum.
o All these are suggestive ofnecrotizing enterocolitis :-
i) The greatest riskfactorfor necrotizing enterocolitis is prematurity
-- Nelson lythle755
ii) Almost all patients of neonatal enterocolitis are artificially fed prior to the onset of illness.
iii) In stage I, there is abdominal distention and blood in stool.
r So, the child in question is suffering from necrotizing enterocolitis.
About other options
o The hallmark of vohrrlus in neonatal period is bilious vomiting.
7;. Aas" is'# i-e.,Tlksativecolitio {Ref : Nelson 1*/e p. 1576, tszT; Robbin's Th/e p. s49)
r Pseudopollp is seen in UC.
18. Ans is'd i,e."Chroaic noaspecificulceratwecolitis {Re! Nelson tye/e p, t|zs;
Robbink Th/e p. B49l
r In sever of UC, toxic damage to the muscularis propria and neural plexus lead to complete
cases
shutdown of
neuromuscular function -+ colon progressively swells to create toxic megacolon.
1O
Ans. is 1d'i.e., crohrt's involvement of the colon [Ref, www.springerlink.coml
On barium enema CD shows hose pipe (lead pipe) appearance.
30. Ans. is'b'i.e., Crohn's disease {Ref: Nelson lgth/e p. ligl, 15g2)
See each option one by one.
Option a
o In this child sigmoidoscopy is normal.
r Ulcerative colitis almost always start from rectum.
r So, sigmoidoscopy is not normal in UC.
Option b
r All the given clinical features of this child can occur in CD.
r Renal colic is due to renal stone.
Option c
r Montoux is negative.
r In a child less than 2 yeats of age montoux is strongly positive if the child has tubercular infection.
Option d
r Stool culture is negative for enteropathogens.
r So, strongyloidosis is excluded.

NECROTIZING ENTEROCOTITIS

31. Ans, is '* i"e., iJlatthe fuoy*{Re! Netaon 1*/e p.755; Ghai p/e p. 1391

unstabletempelaturel apnee,bradycard'ia, lethargy,Ahild abdominaldistension,vomiting blood in


stool.

t\Ll:qa1:.i!ti:::6.==+:tts:!1sffi;:
Stage tll : r Above signs + infant has low BP, bradycardia, apnea, acidosis, DlC, anuria
Advanced r Frank signs of peritonitis with abdominal tenderness, distension and erythema of abdominal
wall.
r X-Ray: shows Pneumoperitonium.

32. Ans. is'b" i.e., Increased bowel sound fRef: O.P. Ghai }th/e p. 165 dt Vh/e p. 139; Nelson lgth/e p. 7551
o Bowel sounds are diminished or absent,

,J. Ans. is 'a' i.e., Necrotizing enterocolitis fRef: o.p. Ghai gth/e p. 165 6 vh/e p. 138, 13g]
r This child is -
i) Premature
ii)
On artificial feeding (top feeding)
iii)
Having abdominal distention and bleeding per rectum.
o All these are suggestive of necrotizing enterocolitis :-
i) The greatest risk factor for necrotizing enterocolitis is prematurity. Nelson lgthI e 7 55
ii) Almost all patients of neonatal enterocolitis are artificially fed prior to the onset of illness.
iii) In stage I, there is abdominal distention and blood in stool.
o So, the child in question is suffering from necrotizing enterocolitis.
About other options
e The hallmark of voh'ulus in neonatal period is bilious vomiting.
. IntussuscePtion presents between the age of 3-9 months (not in neonatal period).
r Symptoms of meckel diverticulum usually arise in the lst 2years of life (average2.5year)
and most common
presentation i4s painless rectal bleeding.

34. Ans. is ? i.e., Peli-tgngal drainage by putting drains in the flanks {Ref: Gellis & Kagan's Current pediatric therapy
15e/e p. 220; Grosfeld's Pediatric surgery t&/e p. 14401
Treatment of NEC
A) Stage 1 6 2 (without petforation) : Conservative treatment in the form of iv fluid, systemic antibiotics, nil ora-
and gastric decompression.
B) Stage 3 (with perforation): Treatment depends :-
i) Difinitive ascites but no perforation: Peritoneal drainage (paracentesis) is indicated.
ii) Perforation: Laprotomy and surgical resection of necrotic bowel with ileostomy.
From this discussion, we choose the answer to be peritoneal drainage because :
1. Patients is on ventilator (too unstable).
!
2. Surgery for NEC is resection and creation of ileostomy, which is not given in the options.
3. Primary anastomosis is done only in stable patients with walled-offperforation.

35. Ans. is t' i.e., C^olgervalive management with IV fluids and antibiotics [Re//rrry://www.netsuic.org.anlnets/
handbook /index.efm?Doc*id=6j4,http://www.abhb.govt.nz/newborn/ teaching resources,radiilogy/nec"htm)

MALABSORPTION

36. Ans. is ? i.e., Small intestinalbiopsy [Ref ]Iarrisan t*/ep. fi7AiZVl; O.p. Ghai&/ep.2S0-251 & Vh/ep.271_ZV2)
r Clinical profile of the child given in question make the diagnosis of malabsorption syndrome associated with
chronic
diarrhoea.
r Intestinal biopsy is necessary to differentiate chronic diarrhoea due to different pathophysiologic reasons.
37. Ans. is id'i.e., Ulcerative colitis {Ref Reud belowl
o This child has steotorrha (increased fecal fat) and azotorrhea (increased fecal nitrogen). Important
causes in children

i) Chronicpancreatitis i|) Bacterial overgrowth syndrome iii) Ileal disease (crohn\ disease)
iv) Celiac sprue v) Intestinal lymphangiectasia

CELIAC DISIASE

38. Ans. is 'c' i.e., Maize [ReJ Nclson 18e/e p, 1624; Robbin"s */e p. 843; Chqndrusoma Taylor 3d/e p. Sgll
r The fundamental disorder in celiac sprue is sensitivity to gluten which is alcohol soluble, water insoluble
protein
component. It is found in wheat and closely related grains such as oat, barely and rye.
39. Ans. is t' i.e., Coeliac diseases fRef: Nelson ISth/e ch,590 (Table. 590-12)l
r Antibodies in coeliac disease are anti-endomysian, antigliadin and anti-transglutaminase.
40. Ans. is 'b' i.e., Increase in thickness of mucosa {Ref: Robbin's Vh/e p. g43; Nelsan lgthle p. 1624)
r In celiac disease proximal intestine is commonly involved while ln tropical sprue whole of the intestine is involved.
c Biopsy of intestine in celiac sprue demonstrates.
t_Villous atrophy, loss of microvilli brush border
t The crypts are hyperplastic and become elongated.
s Presence of following cells on lamina propria -+ Plasma celk, Lymphocytes, Macrophages,
Eosinophils, Mast cells.
r Remember that overall mucosal thickness remains same in celiac sprue
s The basic abnormality is thought to be an inteased rate of loss oj epithelial cells. Crypt cells show in*eased
activity. This causes hypertrophy of crypt, But even this increased activity cannot keep-pace with loss of cells,
resulting in progressive decrease in height of villi causing villous atrophy
t The epithelial cells show decreased cytoplasm and mucus
t The intestinal biopsy shows a decreases in the villus: crypt ratio.
41, Ans. is 'a' i.e., HtA - DQ2fRef: Robbin's Yh/e p, 843; Nelson l}th/e p. 1624; Harrison lVhle p. 1gg0)
. Almost all individuals with celiac disease share the major histocompatibility complex II HLA-DQ2 or HLA-DQ8.
o Gliadin is deamidated by the enzyme transglutaminase and deamidated gliadin peptides bind to DQ2 and DQ8.
o Recognition of these peptides by CD4+T cells leads to secretion of IFN-y, which damages the intestinal wall.
r Other HLA types associated with Coeliac disease are HLA-88, DR3 and DR7.
4i. Ans. is'd'i.e., Increased brush border {Ref: Robbin's ?h/ep. 84j;Netson l8e/ep. 1624)
o There is loss of microvilli brush border (not increased brush border).

II-ACTASE DEFICIENCY

{1. Ans. is 'b' i.e., Lactase deficiency [Rel O.P. Ghai &h/e p. 110, 281 & //e p. 27i-2v4; Nelsor t8e/e p. J598]
e History of ice cream ingestion and subsequent symptoms of vomiting, bloated abdomen, are characteristic findings
oflactase deficiency.
4{. Ans. is 'a' i.e., Lactase p. 281 6 fl/e p. 223)
[ReJ Nelson 18e/e p. "1598; Ghai 6h/e
'Because Lactase activity in the mucosa is at best marginal, this enzyme is particularly likely to be depleted
secondary to diffuse mucosal disease'- Nelson
{i. Ans. is 'b' i.e., HZ lRef: Nelson lSth/e p. 15981
r Hydrogen breath test is used for lactase deficiency and secondary lactose intolerance.

CHOLESTAS!5

{5. Ans. is t'i.e., Glutamate dehydrogenase [Rel See aboye explanation)


r Enzpnes raised in extrahepatic cholestasis (obstructive jaundice) : Alkaline phosphatase, gamma glutamyl transpeptidase
and 5'-nucleotidase.

47. Ans. is1d'i.e.,Biliaryatresia[ReJsuqgeryofliver,bileductandpancreasinchildrenby&dward&oesrdundSfinges


2d/e p. 112, Liser disorder in Childhoad - Mowst p. 831

"GGT levels are commonly elevated to values more than 10 times normal in biliary atresia while in hepatic
causes it is raised to about three times normal!'
Normal values for GGT is about 5-40 IU/L and thus a value of 600 IU/L in the patient in question reflects an
elevation of more than ten times the normal value. The patient is thus likely suferingfrom 'biliary atresia'.

48" Ans. is '-o" i.e., Hepatitis B & C [Ref 0.P. Gkai 8'h/e p. 327 & */e p. 29j; Nelson tr8:/e p" 1669-1674)
o Hepatitis B & C cause intrahepatic neonatal cholestasis.

49. Ans. is'b' i.e., Neonatal hepatitis lRel CPDT 18th/e p. 6441

Common Gauses of Neonatal cholestasis


"Idiopathic neonatal hepatitis accounts for 25-50o/o ofcases ofneonatal intrahepatic cholestasis'l - CPDT

1.'ldiopathic'Neonatal hepatitis : 25-50o/o


2. Extrahepatic biliary atrsia (EHBA) : 20o/o

3. o, antitrypsin deficiency: 15o/a

50. Ans. is'b'i.e., ar-antitripsin deficiency [Re/: See above explanation)


r Amongst the given options a, antitrypsin deficiency is the best answer because neonatal hepatitis and extrahepatic
biliary atresia are not there.

NEONATAL HTPATITIS

51. Ans. is'b'i.e., Neonatal hepatitis with extra hepatic biliary atresia t,Re/ CPDT 15e/e p. 524; Nelson 1*/e
r ]aundice wilh clay coloured stools suggests cholestasis. p. 13171
e::6

r Both neonatal hepatitis and extrahepatic biiiaryatresia


cause cholestasisjaundice.
r "Giant cells" are characteristics of both .r.orutul rr.putiti.
o Therefore the answer is Neonatar ura Exffahepatic Biliary Atresia.
hepatitis with extrahepatic Biiiary
Atresia.
52' Ans' is t'i'e', Arpha- I - antitrypsin deficienc
y fRef: C,DT r*th/e p. 644)
other conditions which also presents with
similar clinicai and -histological features
a) a, antitrypsin deficiency b) Alagille syndrome - (i.e., with giant cells)
c) Niemann _ pick type C disease (NpC).
d) viral infections e) Bylir dkiase
I orogr"rrir"Tr*iiiriiiorhepatic chorestasis (pFIC).

EXTRAHEPATIC BILIARY ATRESIA

53. Ans. is 'b'i.e., Congenital Biliary atresia


[Ref. CpDT
lSth/e p. 647; A.p. Ghai 6thle p.
o conjugated bilirubinemia, achoric stools 17 & Zele p. 293|
and absent urobirinogen in biood
(Obstructive jaundice). indicate complete and persistent
cholestasis
r Both biliary atresia and neonatal hepatitis presents
with cholestasis, but comprete and persistent
biliary atresia. cholestasis is seen in
r usual age of presentation of extrahepatic
biliary atresia is 2-3 weeks but it can present with jaundice since
birth.
r Hlpothyroidism and breast milk jaundice CPDT
presents as unconjugated bilirubinemia. -
54. Ans. is'd'i.e., Liver biopsy tRef Nelson 18tu/e p. 16721
The given crinicar picture is suggestive
of either of the foilowing diagnosis:
I
Neonatal hepatiits
r
Extra hepatic biliary atresia
"Percutaneous liver biopsy is the
most valuable procedure in the eualuation
provides the most reliable discriminatory of neonatal hepatobiliary disease and
euidence,,. _ Nelson 1672
POBI4T HYPERTENSTON

55.
l&ef a.F.GtcaiB,*/ep.3Ls&7etep.2B9;Netsonrse/e
;\r;;:;i'e"Extrahepaticportalycnousobstrucrion
c Most common cause of paediatric portal hypertension in India is EHpVO.
56. Ans is 'd i.e.,EHpVO fRef, C*rrent Diagnosis & Treatment
Surgery by Daherty p. 532|
r Information in question are:
i) Patient is 12 years male
ii) Hematemesis, melena and mild splenomegaly
-+ signs of portal hlpertension
iii) No |aundice or Ascitis
o So, the boyin question has portal
hypertension.
o Three most common causes of portai
hypertension are:
Extrahepatic portal vein obstruction (EHPVO)
ll.
ii) Non-cirrhotic portal fibrosis (NCPF)
iii) Cirrhosis.
o In cirrhosis, jaundice & ascitis
are prominent. so, this option
is ruled out.
' );ilJ: xcpr" Both these ca uu'.., non-,,,,iiic portat hypertension,i.e.
l;'i::il[t.tf,:rf.1na portar hypertension
c Both these have simirar crinical picture
except for the age of presentation.
male children (tst anJznd d..na.j *t it. NCpF
: ffiy.:"1lrT::iH':."#lifr: is a disease oraduits (2s-3s,years).
Signs of portal hypertension
o Esophageal varices
o Hematemesis and melena
o Splenomegaly

+
Signs of Liver dysfunction
o Jaundice & ascitis
o ftrt

Absent Present
+ +
Non-cirrhotic portal hypertension Cirrhosis

Child (1st-2nd decade) Adult (25-35 years)


+ +
EHPVON CPF

Differential diagnosis of three most common causes of portal hypertension in !ndia


Ans. is 'a' i.e., Extrahepatic compressi on lRef: Nelson 18th/e p. 17091
ln children extrahepatic portal hypertension (non cirrhotic) is more comllron than the intrahepatic or cirrhotic
portalhypertension!' - Ghai
post sinusuoidal causes like Budd Chiari syndrome and veno occlusive disease are also observed but are less
common.
Three most common causes of portal hlpertension in children in India -
i) Extrahepatic portal vein obstruction (most common)
ii) Cirrhosis
iil) Noncirrhotic portal fibrosis
58. Ans. is'd' i.e., Portal hypertension due to extrahepatic obstructionfRef: Refer textbelowl
o Splenomegaly and esophageal yarices point to the-.diagnosis of portal hypertension while absence of iaundice
and deranged. liver function test indicqte an extrahepatic etiology.
Differentiation between Extrahepatic and Hepatic / Post hepatic portal hypertension
r Post sinusoidal and hepatic causes of portal hlpertension will have
t Evidence of Liver parenchymal damage I deranged LFT (jaundice)
t A dilated portal vein on ultrasonograplry
r Extrahepatic presinusoidal obstruction will not have any effect on the Liver and hence there will be no evidence

of h ep at o c ellular dam age.


r
No jaundice / No raised deranged LFT
Also non visualization of portal vein is a characteristic of portal vein thrombosis, an Extrahepatic cause.
o Esophageal yarices and splenomegaly are a manifestation of portal hypertension and will be present in all the
above sites of obstruction.

59. Ans. is'b'i.e., Non cirrhotic patial hypertensionfRef: lnternetf


e Clues in this question are -
i) Upper GI bleeding from esophageal varices iii) No ascites or hepatomegaly
ii) Splenomegaly rv) No jaundice
o All these indicate Non-cirrhotic portal hypertension.
r Two most common causes of extrahepatic portal hlpertension are
i) NCPF-+ seeninadults
ii) EHPVO -+ seen in children
o So, this is a case of non-cirrhotic portalhypertension due to EHPVO.
Ahout other options -
t syndrome (Hepatic vein thrombosis) and venocclusive
disease, there will be hepatomegaly, jaundice
}r':l^*liari
ano ascrtes.
r In cirrhosis, jaundice and ascites will be seen.

60. Ans. is 'a' i.e., oesophageal varices lRe! o.p. Ghai 8h/e p. 3rg-i20 6 vh/e p. 28g_2g0)
r Clues in this question are -
1) H/O massive hematemasis 2) Splenomegaly
r Amongst the given options, splenomegaly will be seen only in
.rophugeul varices due to portal hlpertension.
61. Ans. is 'a' i.e., Portal hnrertensio n [Ref: See above explanation]
"Massive hematemesis in a chird is almost
always due to variceal bleeding,,.
Schwartz
Variceal bleeding is due to portal hlpertension. -

LIVER CIRRHOSIS

62. Ans. is 'd i.e., Alpha-r antitrypsin deficiency fRef: cpDT rSth/e p. 664)
r Causes ofcirrhosis in children are _

1) Post necrotic
r Viral hepatitis -+ HBV HCV I Idiopathic neonatal hepatitis r o,-antitrypsin deficiency
r Autoimmune or drug induced hepatitis r Wilson disease r Hemochromatosis
r Tyrosinemia r Galactosemia
2) Biliary
r Biliaryatresia r Choledochal cyst r Tumor of bile duct
r Caroli disease r Cystic fibrosis r Primary sclerosing cholangitis
r PFIC r Paucity ofintrahepatic bile ilucts
3) Hypersensitivity to drug -+ phenytoin

4) Parasites -+ Opisthorchis sinensis, Ascaris, Fasciola.

DIARRHEA

63. Ans. is t'i.e., Rotayirus {Ref: Ananthnarayan #/e p. 529; O.p. Ghai B&/e p. 291)
"Rotaviruses are recognized as the most common cause of diarrheal
disease in infants and children,
- Ananthnarayan
64. Ans. is t'i.e., Reo virus [Ref: Nelson lgth/e p. 1605, 1606; Op Ghai 8th/e p. 291]

65. Ans. is 'H i.e., Rotavirus [Ref: o,p. Ghai gth/e p, 29r 6 6h/e p. 261; Nelson rgth/e p. 160s, 1606)
o Rotauirus is the single most common cause of diarrhea
amongst children in both developed
o ETEC is the single most common cause of bacterial d-iarrhea"amongst developing world. and
children in the develop,iing world.
o ETEC is the most common bacterial cause Traveler\ dia,heaa.'-
for
66. Ans. is'a' i.e., Enterotoxigenic E.coli (ETEC) {Rd o.p. Ghai Ei/e p.
291 6 7tule p. 2611
o Enterotoxigenic E.coli (ETEC) is the most comrnon bacterial cause
of diarrhea in India.
67. Ans. is 'b' i.e., Carry Blair medium [Rd Nelsor lThle p' 1274; 16h/e p. 751
c In acute gastroenteritis stool culture should be obtained as eaily as possible.
o "Fecal specimens that cannot immediately be plated for culture can be transported to the laboratory in a non-
nutrient holding medium such as Carry-Blair to prevent drying or overgrowth of specific organisms.". - Nelson

68. Ans. is 'b' i.e., Giardiasis lRef : Internetl


o In intractable diarrhea, no infectious etiological agent is identified (thus, giardiasis cannot be a cause).

69. Ans. is t'i.e., Giardia fRef: Nelson 17h/e chap 27jl


r Diarrhea in immunocompromised child (AIDS) include bacterial (salmonella, compylobacter) ,protozoal(giardiasis,
crlptosporidium), and viral (CMV HSV) causes.

ABDOMINAL PAIN

70. Ans. is'b' i.e., Emotional/behavioralproblems


o The commonest cause of chronic abdominal pain in older children is functional (non organic) that may be due to
emotional or behavioral Problems.
71. Ans. is t' i.e., Lead poisoning [Rd See above table)

o Amongst the given options, only lead poisoning is a common cause'


72. Ans. is t'i.e.,4 months
o It usually begins at 1-2 weeles of age and can persist till 3-4 months of age.

MISCELLANEOUS

73. Ans" is 'b' i.e., Biliary atresia lRet Ghai Vh/e p. 2951
o Biliary atresia is commonest indication for liuer transplantation in pediatric age group.

74. Ans. is'-o- i.e., Lymphoma fRef: Nelson l8hte Ch' 491,492, 342)
o Lymphoma is the most common malignancy of the gastrointestinal tract in children. About 30o/o of children with non-
Hodgkin lymphoma present with abdominal tumors'

75. Ans. is 'b' i.e., fejunal polyp lRef: o.P. Ghai 8il/e p.
js4 & 7tu/e p' 2s5l
o ]ejunal pol1p is not an organic cause ofconstipation. Other three options are organic causes ofconstipation.
76, Ans. is 'd i.e., Anal fissure lRef: Nelson 18th/e Chap. 341)
o Anal fissures are the most common cause of rectal bleeding in infants and children.

77. Ans. is t' i.e., Neimann Picks diseas e lRe! O.P. Ghai dh/e p. 616 6 Vh/e p. 29i; Nelson lVh/e p. 4641

ANSWERS OF VARIOUS OTHER EXAMINATIONS

7g. Ans. is t, i.e., Traumatic pancreatitis lRef. O,P, Ghai Sth/e p. 289 d, 7tu/e p. 260)
o Blunt abdominal trauma is the most common cause of acute pancreatitis in childhood.
o pseudcyst occurs in up to 15% ofpatients as a complication ofacute pancreatitis.

79, Ans. is'a' i.e., Tracheo oesophageal fistula {Ref : o,P. Ghai 8'h/e p. 176 6 vh/e p. 151}

80. Ans. is t' i.e., Trypsin [Ref : Hartison lVh/e p. 1877]


o Enterokinase deficiency is due to an absence of the brush border enzyme that converts the proenzyme trypsinogen to
trypsin and is associated with diarrhea, growth retardation (failure to thrive) and hlpoproteinemia.

gl. Ans. is t, i.e., crohn's disease lRef : crohn\ disease has been explainedl
82. Ans. is'H i.e., 24-48 hours lRef : O.P. Ghai }th/e p. 177 d2 Vh/e p' 152!
o An X-ray film of the abdomen is obtained l}-24hours after birth, with baby being kept in an inverted position.

83. Ans. is 'd i,e.,Galactosemia &'d' i.e., Cystic fibrosis lRef : O,P, Ghai 8th/e p. 327 dt Vh/e p. 29jl
84, Ans. is 'H i.e., Oesophageal atresis in the foetus lRef: O.P. Ghai 6th/e p. 178 & 7h/e p. 151; Baity & Love 24th/e
p. 9941
85. Ans. is'H i.e., Lactose intolerance lRef: O. P. Ghai 6h/e p. 281 6 7h/e p. 273; Harrison 16h/e p. 2i0, tZ6Z)
86. Ans. is ? i.e., Syrup of ipecac lRef : KDT 5'h/e p. 6aAl
87. Ans. is t'i.e., Aerophagy fRef.: O.P. Ghai 6th/e p. 263 d, 7tu/e p. 2511

88. Ans. is'a'i.e., Non specific bowel dilatation fRef: O.P, Ghai 9th/e p. 165 & 7h/e p. 139)
o Earliest X-ray finding in neonatal necrotizing enterocolitis -+ Nonspecific dilatation of intestine.
o Characteristic X-ray finding of neonatal necrotizing enterocolitis -) Pneumatosis intestinalis (Gas in intestinal wall).

89. Ans. is'b'i.e., Gas in intestinal wall [Rel Nelsan 18th/e p. 755; a,P. Ghai *th/e p. 165 d, Vh/e p. 1j9l
90. Ans. is'a'i.e., Neonatal necrotizing enterocolitis lRef: Nelson 18th/e p. 755; O,P. Ghai 8th/e p. 165; Thle p. l jgl
91. Ans. is'd i.e., Enterotoxigenic E. coli [Ry' A.P. Ghai 8th/e p- 291 dr Th/e p. 261)
92. Ans. is t' i.e., Tracheo-oesophageal fistula lRef: Antenatal diagnosis of vtacterl anomaly, RD Dani, V Gandhi,
GN Thakkar PN

IIT
WKKM.&'KY Y'Kr&CY

CONG EN ITAL MALFORMATION

o Kidney develops from two sources :-

i) Metanephros (metanephric mesoderm or blastoma)


r It is derived from intermediate mesoderm and forms excretory unit of kidney, i.e. major part of nephron :
Glomerulus, PcT(lipnere8), loop of Henle and DCT.
ii) Ureteric bud
r It is derived from mesonephric duct and forms collecting part of kidney, i.e. pelvis, calyces and collecting
es. etJ
tUbUleSAttMs .

o At birth kidney has lower concentrating ability. A neonatal kidney achieves concentrating ability equivalent to adult
by the age of 7 yearat oa)
. A neonate can concentrate urine upto a maximum of 700-800 mOsm/kglK"-. o,), but c&n
dilute the urine to 50 mOsm/kg (as like adults).

Renal agenesis
o Renal agenesis is the absence of kidney due to failure of development.
o Unilateral renal agenesis is associated single umbilical artery@nnr), absent ureter, contralateral vesicoureteral reflex,
and absent ipsilateral vas deferens.
'o Bilateral
renal agenesis or Potter syn drome(MAHE\s) is associated with maternal oligohydraminosMAHEos),pulmonary
hlpoplasia andpotterfacies (widely separated eyes with epicanthic folds, low set ears,broad compressedflatnose@AHE
05),
receding chin). The condition is incompatible with life, death occurs shortly after birth due to pulmonary hypoplasia.

Multicystic renal dysplasia (multicystic dysplastic kidneyl


r Multicystic dysplastic kidney is a congenital condition in which the entire kidney is dysplastic with a preponderance
of cysts.
o Kidney multiple cysts of varying sizes that do not communicate and no identifiable parenchyma is presenr
is replaced by
o Contralateral vesicouretral reflux is present in l5o/o cases. Contralateral hydronephrosis is present in 5 - 10% cases.
o It is the most common cause of an abdominal mass and most common cystic disease in a neonates@r os, 03).

r Childhoodpolycystickidneydiseasehasautosomqlrecessive1tls)inheritance,thereforeitisalsoknown asautosomal
recessive p oly cystic kidney diseas e.
o Defective gene is the PKHDL (Polycystic Kidney and Hepatic Diseasel) which codes for a proteinf brocystin.
r It is associated with maternal oligohydramnion, Potter's syndrome, pulmonary hypoplasiaLloe), hepatic cysts and
orrMs 07)
fibrosisro'ot, , and bilateral ductal atresia.

Clinical presentation
o Majority of patients present in infancy. Presentation is variable :-
1) Renal : Enlarged cystic kidney at birthql 0e), hypertension, renal failure, proteinuria.
2) Respiratory: Respiratory distress dte to pulmonary hypoplasia\rle).
3) Hepatic: Hepatomegaly, portal hypertension (esophageal varices, hypersplenism).
o Both kidneys are markedly enlarged and show innumerable cysts radiatingfrom medulla fs flxs ssygsyutrbrsoz).
r MRI of kidney shows radially arrangedfusifurm dilated collecting ductsallMs0T).
r Prenatal USG shows a salt and pepper appearance ofkidney.
Nephronophthisis (Juvenile Nephronophthisis)
r It is the most common genetic cause of end stage renal disease (renal failure) in childhood and adolescence.
r It is an autosomal recessive disease.
o The patients present before the age of 20 years. The patient presents with polyureaLuMs03'01), polydipsia, anemiaallMs
03),
growth retardation (short statureallMs03'ot)). BP is norrnal\IlMs0r) and theie is no proteinuria or hematuria (no
red cells ifi urineatrMs oi))
.

o Radiological investigations show bilateral small kidneys{dttus 03, 0I)


with multiple cysts at corticomedullary junction.
o Nephronophithis is associated with many extra-renal manifestations like retinitis pigmentosa, cloboma of eye, hepatic
0r),
fibro5isro"*tol cerebellar vermis aplasia, and cone-shaped epiphysis.
o Blood investigations shows features of renal failure, i.e. raised blood urea nitrogen and creatinine@IlMs 03. ot) .

Ectopic ureter
o A ureter that drains outside the bladder is referred to as an ectopicureter.
r Ectopic ureter usally drains the upper pole of a duplex collecting system (two ureter -+ duplication of ureter).
r It is more common in girls.
o The most common site of opening of ectopic ureter is posterior (prostatic) urethra in malesLl ee) and urethra near
bladder neck in females. Other sites of opening in males are seminal vesicle, prostatic utricle, ejaculatory duct and vas
deferens (in decreasing order ofpercentage).
a ln females other sites for opening are urethrovaginal septum, vagina and cervix.
a Females usually present wlth urinary incontinance and UTI (dysuria)uMu se,Iipmera1). Treatment includes :-
1) If renal function is satisfactory -+ (Jreteral reimplantation into bladder neck or ureteroureterostomy.
2) Ifrenal function is poor -+ Partial or total nephrectotny.

GLOMERULAR DISEASES

NEPHROTIC SYNDROME

r Nephrotic syndrome is a group of disease characterized by :

i) Massive proteinuria (>3.5 gm/day){t t tst


ii) Hypoalbuminemiaat B' e7' AuMses'e6)
111) Edema
w) Lipiduriaqtl3) (lipid cast)
v) Hyperlipidemiaur13)
vi) Hlpercoagulability.
o Proteinuria is the most significant of these and leads to hypoalbuminemia, which is the cause of edema(/JlMs 12' 10)

(Note : in nephritic slmdrome, edema is due to sodium and water retention).


o The proteins decreased in nephrotic syndrome (due to proteinuria) are albumin (most commofl)4rrMseE es'Ale7) causirtg
edema, transferrinaumses'es'AIe7) (causing microcytic hlpochromic anemia), cholecalciferol binding protein (causing
hypocalcemia@EEr)), thlroxine binding globulin (causing decreased thyroxine level), and ceruloplasmirr1rr[se&ss,Ar
e7).
Fibrinogen level is e8, es, AteT)
increased(ATlMs due to increased hepatic slmthesis.
e There infection. Spontaneous bacterial peritonitis is the most common infection@Gr
are increased chances of 03, MAEE 08,

07).
Most common cause is streptococcus pneumoniae followed by E.coli (?d most common).

Primarlr nephrotic syndrome


c Most children (90%) with nephrotic syndrome have a form of ifiopathic (primary) nephrotic syndrome.
e Causes of idiopathic (primary) nephrotic syndrome are :-
1) Minimal change disease (85o7o1tdr n' wnnr' PGr 08' B)

ii) Mes angial proliferativ e glomerulonephritis


o")
111) Fo cal segmental glomerulosclerosis(Pcr
o Thus minimal change disease is the most cotntnon cause of nephrotic syndrome in children@ 13, NEEI; PGr 03)
.

ifiinimal change disease (lipoid nephrorisl


o Minimal Change Disease is the most common cause of Nephrotic syndrome in childrenal li, NEET) .
o Edema and Selective proteinuriaarrMsee'PGree) are features of nephrotic syndrome. Fever may be present on account
of increased susceptibility to infection.
o Minimal change disease prsents with insidious onset of nephrotic syndrome in children below 6 years of age.
. Hypertension is not a feature of neqthrotic syndrome and is rare in Minimal change disease. Hematuiia (a
finding of nephritic syndrome) is also rare.
o Pathological findings include
l. Light microscopy -+ No abnormality{arott
2. Electron microscopty -+ Obliteration (loss) andfusion offoot processes of epithelial cells.
3. ImmunoJluorescence -+ No ileposits of immune reactants(Pctoe)
4. Serum complements -+ Normal levels
o Urine shows lipid castat 0e), hyaline cas/o' ot), no RRC(.Ar 0e)
or WBC.
r Minimal change disease has very good prognosis with excellent response to steroidsar0l,ee'Pcree).

* ln case of development of adverse effects of corticosteroids or in cases of corticosteroid resistant nephrotic


syndrome (FSGS), cyclosporin(ArrMs rl)or ryclophosphamideuttHsu,03i}areused,

':,:*rE
:::::l:t
1,,;,.::4,,
;.a,a:a:|.'..

a:l::a.:aa,a

..;,:aa:,::
:

Secondary nephrotic syndrome


o when nephrotic syndrome occurs
as a part of systemic disease or is related
to drug or other toxin it is termed secondan,
nephrotic syndrome' Important causes are
systemic disorders (IgA nephropathy,
HBC, HCV), malignancies and drugs.
postinfe.,r"";;il, ;;;;;;,,
Congenital nephrotic syndrome

' nephrotic syndrome within the


first 3 months of life are considered to have congenitar
::ffi.].h,o,*t#
o The most common cause of this syndrome
is finnish-type congenital nephrotic syndrome, an autosomal recessive
disorder.
r Two most common genes involved are _

NPHS=1
LlPhrlfilAf.MsoI'lrde061. S;ipt diaphragm
!rylyolicsyndromeoffy[ 1pe

NEPHRITIC SYNDROME

'i:'#;:t*:il:,'n;:5::::"".*i:,i:'"":':::i,l'll affecting the kidnevs, more specincarv


1""'1'rshlpertension,
and is characterized'bvhaematuria, protenuria,
fr"HJfi:Ifiif edema and oliguria.

r The haematuria is slight giving the urine smok


appearance. Erythrocltes are detectable
by microscopy or by-
chemical testing for hemoglobin.
t Presence of RBC casts in urine is crassicar
feature of nephritic syndrslns,t tst.
r Presence of dysmorphic RBCs along with
normal RBCs differentiate hematuria from glomerular disease from
bleeding originating in rower urinary tract (where
alr RBCs have normar morphology).
2) Proteinuria
r Proteinuria is mild (< 3 gmlday) also called
nephritic rairge proteinuria or subnephrotic
Proteinuria is nonselective. range proteinuria.

3) EdemarN,Er)
r usually mild and results from sodium and water
retention.
4) HlPertensl6l(nrrrl
r Is variable depending upon the severity of glomerular disease.
s) Oliguria
r Reflects the severity of glomerular involvement.

c Cause of edema in nephroticsyndrome : Hynootlyminemia(Att$s tz to


(hypoproteinemio).
.','eauieaf{i.deff atrnneb c.iwi,'tiiitodi,$) andwctel retentlannEEr}-l

r Causes of acute nephritic syndrome in


children are :_
1) Primary glomerulonephritis: Acute glumerulonephritisrrcroo)
(post-streptococcal), RPGN, MPGN,
IgA nephropathy. focal GN and
2) systemic diseases: SLE, pAN wegner's granuromatosis,
HSB cryoglobulinemia.
Post streptococcal glomerulonephritis
o PSGN appears 7 to 4 weeks afier a streptococcal infection of pharynx or skiru (impetigo)ect 0I). It is caused by certain
nephritogenic serot)?es of group A B-hemoly'tic streptococcus (not by all serotlpes).
o The lesions are caused by immune complex deposition and activation of complement + complementleyel decreases@G'
01).

o It occurs most frequently in children 6 to 10 years of age.


o Clinically child presentswith acute nephritic syndrome -
i) Hematuria iv) Proteinuria
ii) Oliguric acute renalfailure v) Hypertension
iii) Azotemia vi) Edema
r PSGNcausesacuterenalfailure,butnotchronicrenalfailure(Pclor)--smorethang5o/oofaffectedchildreneventilally
recover totally with conservative therapy,

Treatment
o Management is directed at treating the acute effects of renal insufficiency and hypertension.
e Although a 10 day course of systemic antibiotic therapy with penicillin is recommended to limit the spread of
nephritogenic organisms, antibiotic therapy does'not affect the natural history of glomerulonephrifis{tet ott
.

IgA nephropathy (Berger's diseasel


o It is the most common tlpe of glomeru{onephritis world wide.
e ChildrenGrqs'e8'ArrMs01'e6) and young adults are commonly affected.
r It occurs after an upper respiratory tract infection or gastrointestinalinfection{At0s'e8'ArrMS0l'e6).

Clinical presentation
o Grosshematurit(Al\8'e8'AIIMsor'e6)within 7-2daysatos'et'ArrMsol'e6)afteronupperrespiratorytractinfection.Thisfeature
distinguish it from PSGN. In PSGN hematuria occurs 7-21 days after respiratory infection.
0s' e8' AilMs 01' e6).
c Can also present with nephritic or nephrotic syndromeat
e Mild to moderate hlpertension.

Pathotrogry
0s' e8' AIIMS 01' e6)
o Focal and segmental mesangialqt proliferation.
os'sa, drrMs 01' e6)
e IgAGt deposition in mesangium.
e Serum complement level is normalqr 0s'e8' ArrMs 01' e6)
. This feature also differentiates it from PSGN. In PSGN complement
(Cr) level is decreased.

r Whereas lgA nephropathy is an isolated renal disease, s imilar lgA deposits are alsofound in systemic disorders of children
like Henoch shoileiin,puipura,'which ilso,iave reial.vasculltis'qs o'co:mpan€it,'brvtew af tie high frequency of lgA
nephtopathy la chltdren, lgA can be used.as matkerfor renal vasculllls{At08;e8'Afi8sat.:e6t. ,':.
r There is no proven treatment. The drugs which are used : corticosteroidsallMs,B),
ACE inhibitors and AT-II antagonists
antibiotics, and fish oil.

*lenoch-Schonlein purpura
r Henoch - Schonlein purpura is vasculitis of small vessels (capillaries, venule
or arterioles) and characterized br
deposition of IgA in the wall of involved vessels(nzn4s 0s).
r H'S. purpura is characterizedby tetrad of purpura, arthritisd/EE'r, PGtos),
glomerulonephritis(NErr), and abdominal
paino'tnnr'ruo8). Gross hematuria is seen in2}-3}o/oof cases(Arrs).
o All changes result from the deposition of circulating immune complexes,
containing IgA and complement on the
endothelium of the blood vessels throughout the body and within the glomerular mesangial
regions.
c Diagnosis confirmed by presence of palpabte purpura with normal platelet count@EEr, pcr 0B) along
is
with one or more
of the following: abdominal painNmr eq08), arthralgia/arthritis@EEl: PGI0s) and
mesangial deposition ozt.
of lgA(enus
t Renal manifustations usually start at the onset or within 3 months of onset
of other symptoms@NB 13).

A clrild *it& hematurta aad abdominal


Bain ispresenting ae
in foll,owing fig*re, The diagnosis ix-
a) Minimal change disease
b) HUS
c) PSGN
d) HSP

Ans. is t'i,e., HSP


r The figure in question is showing Purpura, HSP has classical presentation
of hematuria, arthritis, ab-
dominal pain and purpura.

Pauci*immqne glornerulonephritis
I Pauci-immune glomerulonephritis is dehned as the iack of anti-GBM antibodies or immune
complex.
r It is a type of rapidiy progressive glomerulonephritis.
e Causes are :-
i) Idiopathic renal-limited crescentic glomenrlonephritis (AI{CA associatecl).
ii) Microscopic PAN
lli) Microscopic polyangitis
iv) Wegner\ granulomatosis
o It is treated aggressively with glucocorticoids with or without cyclophosphamide
or azathiopr inearrMs02)

HENOOLYTIC UREMIC SYNDROME

c HUS is characterized by the triad of -


1) Anemia (microangiopathic hemolytic anemia)
2) Renal failure (microangiopathy of kidney involving glomerular capillaries and arteriales).
3) Thrombocytopenia (due to platelet destruction)
o E'coli (EHEC/Shiga toxin producing) is the most common cause. Shigella (2
m.c. cause), salmoneila, Bartonella,
campylabacter and viral prodrome (echovirus, coxsackievirus,HlV) are other causes.
o HUS is common in children under 2 years of age, (Bfi may also occur in older
children).
o It usually follows an episode of acute gastroenteritisqrrMs ee' e8).
The prodrome is usually of abdominal pain, diarrhoea
and vomiting. Shortly thereafter, signs and symptom s of acute hemolytic anemia, thrombocytopenia and acute renal
Ii,nqet) ensue. Sometimes neurologicalfindings ako occur (But usually absent and ilifferentiates HUS
failuvs{tttusss'
from TTP)(A.ee).

Lab findings
a Thro mb o cy top eni s4uu s ss' st )

o ARF -+ I nUN and creatinine with oliguriaallMsee,es)


e Peripheral smear -+ Helmet cells, Uur, ,r11t1iPne'n'), fragmental RB6s{dnussss (Schistocytes)
o Hemoglobinuria with hemosiderinuria
o Leukocytosis
o Proteinuria & Microscopic hematuria
r Normal PT and APTT

URINARY OBSTRUCTION AND UTI

U RINARY TRACT OBSTRUCTION

Posterior urethral valves


o lt is the most common cause of obtstructive uropaihy in children(A|Ll).
o These are symmetrical folds of urothelium extending dist-ally from prostatic urethra to external urinary sphincter. It
most commonly lies just distal to the verumontanum or at the verumontanum.
c It occurs only in males(Pcl 10),

r It behaves as flap valves so, although urine does not flow norrr,ally a urethral catheter can be passed without dfficulty.
Sometimes, the valves are incomplete and the patient remains without symptoms until adolescence or adulthood.
o Approximately 30o/o of patients experience end stage renal disease(Pcrr0). Vesicoureteral reflux occurs in 50% of
patients.
o Complications include lrydronephrosis@EBr' pct 10) , distended bladder@r ls) with dilatation of prostatic urethra@cr 1o),
recurrent (JTINEET'PGr10) ar'd renal failure (posterior urethral valves is the most common cause of end-stage renal
disease in a male child).

c Voiding cystourethrogram (VCU) is the investigation of choice(ArrMs 03).


VCU findings are :-
i) Ditatation of prostat:ic urethra(Karn'ss)
ii) Transyerse linear filling defect corresponding to the valve
o Endoscopy can also be used. Prenatal diagnosis can be made by USG, which shows 'key hole' appearance.

Ureterocele
o It is characterized by narrowing of ureteral orifice, (pinpoint ureteral orifice) with cystic dilatation of terminal ureter.
o It is more common in females and is associated with duplication of ureter.
o Child may present with vesicoureter reflux, unilateral hydronephrosis(Aroo) and UTI.
o Vesico-cystourethrogram (VCU) which showsfillingdefectinbladder^Ioo), Cobra-head
is the investigation of choice
appearance ofureterocele and droopinglilly appearance ofkidney. USG show hy droureter andhydronephrosis(Al 0o).
o Treatment of choice is endoscopic removal of ureterocele
A Childwith urinarytrect obstruction is showing
fiollowing excretory urograph. Diagnosis is -
a) Posterior urethral valve
b) Meatal stenosis
c) Ureteric stone
d) Ureterocele

Ans. is ? i.e., Ureterocele


e The excretory urograph is showing co hra-head appearancewLthdistal
-- ' *
"""-^ slightly
-*: ureter ditated and surrounded
rruLu
by a thin lucent area.l 'r-'

Felvi-ureteral junction ohstr.rction


o It is characterizedby intrinsic stenosis of a segment ofureter in region ofpeluiureteric junction.
r It is more common in males and affects left side more.
I It is the most common cause of upper urinary tract obstruction in 6hildlsn(arrsl.
e There is palpable renal mass in newborn with abdominal,
flank or back pain.
*USG shows hydronephrosis with dilated renal pelvis but normal (non-dilated) ureter.
e 'Pyeloplasty' is the surgical procedure of choiss{dr tot when indicated.

VESICS[.IRTTERIC REFI.UX

* Retrograde flow of urine from the bladder to the ureter and renal pelvis is referred to as yesicoureteric reflux.
* It is the most common cause of recurrent UTI in children\IlMs Ar 0s). It is more common in
03' 0a)
femalssailMs .
e The reflux is present since birth, but it is usually detected at 2-3 years of age, however infemales the age of detection
is eaflierqilMs 04) .
* VUR presents as recutrent U71, causing pyelonephritis or cystitis. It is the most common cause
of pyelonephritis in
children. Infection is followed by renal scarring(aro:r and hypertension.
* Micturating cystouretrogram is the investigation of choigs{dr sst.
* Based on micturating cystourethrogram, VUR is divided into :-
Grade I : VUR does not reach the renal pelvis.
Grade II : VUR extending upto renal pelvis without dilatation of pelvis or calyceal fornices.
Grade III: VUR extending up to kidney with mild dilatation or tortuosity of ureter and renal pelvis;
and no or
minor blunting of calyceal fornices.
Grade IV : Moderate dilation or tortuosity of ureter, renal pelvis and calyceal fornices with normal appearance
of
papillary impression on calyces.
Grade V : Gross dilatation and tortuosityofureter, renal peivis and calyceal fornices with loss ofpapillaryimpression
on calyces.

II
I
Treatment Reeommendation Sor WUR
1) Grade I d 1I: Antibiotic prophylaxis only.
CnaprBn 7,
.Uiinctrtt,Trtrct
n
2) Grade III & IV t

1) Age 0 to 5 years : - Antibiotic prophylaxis and follow upGr oz, oo)


ii) For age > 5 years : - Antibiotic prophylaxis for unilateral VUR and surgery for bilateral VUR.
3) Grade V:
i) Age < 7 year : Antibiotic prophylatsis(Alrms oa' ost

ii) Age 1-5 years : antibiotics for unilateral and surgery for bilateral VUR
i11) >5years;surgery.

c lf scarring is present at any age after 1 year, surgery is indicated.

Note - Surgical management involves, modifying the abnormal uretrovesical attachement to create a 4:l
or 5:1 ratio of ntramural length to ureteral diameter, and it is done by reimplantation of ureter.

A child with VUR has followirxg micturating eystourethro-


gram. Grade of VUR is -
a)I
b) rr
c) Ill
d)v

Ans. is t'i.e., V
* There is gross dilatation and tortuosity of ureter and renal pelvis > Grade V VllR

U RI IU&&Y T&&CY !N FECT'&&,{

e UTI is more prevalent in males during infancy and in females afier infancy. Most common causative agent is E. coli
(DNB r2), AIIMi0i).
followed by klebsiella and proteus. Most common cause of recurrent UTI is VURaI05,

CNinieal featalres
o Clinical features depend on the type of UTI :-
1) Pyelonephritis : Abdominal or flank pain, feverallMs 00),
nausea & vomitinglAllMs 00),
abdominal distension\IlMs
oo),
and malaise.
2) Cystitis: Dysuria, urgency, frequency, incontinence, suprapubic paln and malodorous urine.
3) Asymptomatic bacteriuria : Positive urine culture without symptomsal ee).

Diagmosis
* A UTI may be suspected based on the symptoms or finding on urinalysis or both, but a urine culture is necessary
for confirmation and appropriate therapy. Thus the diagnosis of UTI depends on having the proper sample of urine.
There are several ways to obtain a urine sample, some are more accurate than others.
a In older children midstream urine sample is sufficient but in infants (< 1 year) suprapubic aspiration is best method
to collect urine specimen for cultureqilMs 0a) .
o Microscopic analysis show pyuria (> 5 WBC/hpf) (/r 0r)
but it is not confirmatory1l 03),
because infection can occur
without pyuria and pyuria can present without infection{etost.
* Deep stick test for nitrite and leucoclte esterase can add to diagnosis but are not specific. WBC casts are specific
for pyelonephritis.
. Urine culture is the mainstay of diagnosis. > 100,000 colonies/ml (with or without symptoms) or> 10,000 coloruies/
04).
ml in symptomatic patient confirm (JTIGtrMs
r Microscopic examination of urine showing 1 bacterium per oil-immersion field on gram stain also confirms UTI
(equivalent to > 100,000 colonies/ml).
o Vesicocystourethrogram is indicated in a male child with > 1 UTI episode and in female child with > 2 UTI epidoses
within 6 months.

Treatment
r On suspection of rJTl, urine sltecimen is sentfor culture and emperical antibiotic treatment is started6'rMso0).
Cotrimoxazole is used in uncomplicated UTI and 3'd generation cephalosporin is used in complicated UTI. After
culture report, specific antibiotic therapy is started.

" ,ana.atiytawit ab;n:oimattty, Recommended investigitions are UsGrar'"'*'r:;,radionucllde scan (DJi4$AaIaso;, and
: filisturatinggystourethrogrqm'(MCU), ,', ' " I :'': ,

Chronic pyelonephritis {CPN}


o It is characterised by renal inflammation & fibrosts induced by recurrent or persistent renal infection,
vesicoureteric reflux(NEnrA'03) or other causes of UTI.
r Progressive renal scarringwhich leads to end stage renal diseaes.
r Inrefluxnephropathy, Intrarenalrefla\@EEr'A103) ofinfectedurineinducesrenalinjurywhichhealsbyrenalscarring.
Ar03),
r Being - straight, short & wide urethra, recurrent UTI is more common in female(NEEr so CPN is also twice as

common is female than male.

Xanthogranulomatous pyelonephritis
r Xanthogranulomatous pyelonephritis is a form of chronic pyelonephritis characterised by destruction of renal
parenchyma and the presence of granulomas, abscesses and collection of lipid laden foamy macrophages (foam cells).
t os)
Age of presentation ranges from infancy to 76 yearsal .
c Focal form being more common in childrenal o:) --1 Appear healthy.
r Those who affected diffusely, present with non-specific symptoms of chronic infection.
i) Weight loss
ii) Fever
iii) Lethargy
iv) Failure to thrive

o Proteus is the most common causative organism.

RENAL FAILURE

Acute rena! failure


o In children, Acute renal injury is defined as (i) Doubling of baseline serum creatinine, and/or (11) Urine output < 0.5
ml/kg/hr for 12 hours.
r Acute renal failure is defined as (i) Tripling of baseline serum creatinine, and/or (ll) Urine oury,fi less than 0.3 ml/
kg/hr for 24 hourstDNl t3) .
r Acute renal failure may be i- '

i) Pre-renal ARF (pre-renal azotemia)


r Itisduetodecreasedrenalperfusionasaresultofdiminishedcirculatoryvolume.Thereisnorenalparenclrymal
damage.Important causes are dehydrationarl0), CHF, sepsis, hemorrhage etc.
ii) Renal (intrinsic renal) ARF
r It is due to renal parenchymal
damage. Important causes are glomerulonephritis IIUS(pclor,
ATN, renal vein
thrombosis, interstitial nephritis, rhabdomyolysis and tumor infiltration.
iii) Post-renal ARF
r It is due to obstruction ofurinary tract, e.g. posterior urethral valve, ureterocele
etc.

Spedficgravity orarine > 1.20

< 2A m Eq/Lttt rotattusest


> 40 mEq/t-

Blood urea lntrogen / ueatinine ' > 2OIN ro,aar,ttes)

Schwartzfotmulaqi.,#rr)iis used to calculate creatinine clearance in children


. Creatinine ilearance = K x heighJ l creatinine :

K is a constant aRd depends uPon'ag{eunsw,'muscle mass,rnnnsoat on6 *"thod of creatinine estimation{AilMsost.

MISCELLANEOUS

Bartter syndrome
o Bartter syndrome is an autosomal recessive disorderarrs) caused by mutation
in gene coding fo r basolateral chloride
channel (Clc-kb). There is loss of sodium, chloride, potassium and calcium in urine.
o The major clinical findings are hyponatremiaLltMs 04' 03,02), hypokalemia(AtrMs 04, or,or),
polyureaLrrMs o,t,03, o2),
polydipsiaqllMs o't' os' 02),'metabolic alkalosis@I 1s),
normal to low BP, hypomagnesemia (only in some patients),
lrypochloremia, hypercalciuria (causing nephrocalcinosis)Ltttus o<,03),
and growth retardation (failure to thrive)
(AIIMS 04' 03)
. Neonate may have sensory-neural hearing loss(,{r Is)

Gitleman syndrome
o It is an autosomal recessive disorder catsed by mutation of genes encoding
sod ium-chloride cotransporter (NCCT).
There is hypocalciuria, hypokalemia, hypomagnesemia and metabolic alkalosis.
Clinically there are recurrent muscle
cramps and growth retardation.

Dent's disease
o Dentt disease, a familial proximal tubular syndrome, is an x-linked recessive(AllMs rr) disorder of proximal tubules
characterized by: -
l) HypercalciuriaqllMs 11)
and nephroc^lginesis\,us 11)
and nephrolithiasis
ii) Low-molecular-weight proteinuria
iii) Metabolic bone disease/RickeSIGIIM; 11)

iv) Progressive renal failure


v) Marked male predominance
o In addition to above features, other renal proximal tubular defect can cause:
i) Fanconi syndrome (Aminoaciduria, proteinuriaurrMsrl),phosphaturia)
ii) Glycosuria, uricosuria and kalliuresis
iii) Impaired urinaryacidification
o Dent's disease occurs due to mutations that inactivate a voltage-gated chloride
transporter, CLC-S.In some cases, it
is associated with mutations in the UCRL-I gene thatis also mutated in the
oculocerebral syndrome of Lowe.
Cueprrn 7 lJrinoryTract
ldiopath ic hyperealeiuria
o It is autosomal dominanf disorder characterized by hypercalciuria (calcirm excretion > 4 mg/kg/day) without
hypercalcemia. It presents as recurrent gross hematuria, persistent microscopic hematuria and dysuria. Nephrocalcinosis
may occur.
e Treatment includes oral thiazide, low sodium and low protein (meat) die{Attus 04' 03)
, and adequate hydration (increased
0i). Dietary restriction of calcium is not recommendeilbecause of obligate requirement of calciunt
water intake)(AltMs04'
M\ o1' or)
for grow lh'
A I I
.

Most common abdominal mdss is a neonote: Multicystic renal dysplasia.


Most common abdominal mass in 1 months to 3 years:Wilms tumor or multicystic renal dysplasia.
Most common renal problem in j to 6years: Minimal change disease causing nephroiic syndrome.
Most common renal problem in 6 to 14 years: Acute post-streptococcal GN.
Most common renal cystic disease in infancy: Multicystic dysplastic kidney (Unilateral renal dysplasia).
Chitdhood polycystic kidney disease (CPKD) is acquired as: Autosomal dominant.
lmportant associations of CPKD:Maternal oligohydramnios, pulmonary hypoplasia, hepatic cysts & fibrosis, biliary atresia, Portter's
syndrome.
LJnilateral rena! agenesis is associated with: Single umbilical artery.
Most common site of opening of ectopic ureter in males: Posterior (prostatic) urethra.
tmportant feotures of nephrotic syndrome'.Proteinuria (>3.5 gm/d), hypoalbuminemia, edema, hyperlipidemia, lipiduria.
Proteins decreased in nephroticsyndrome:fransferrin, AT-lll, ceruloplasmin, cholecalciferol binding protein, thyroxine binding globulin.
Protein not decreased in nephrotic syndrome : Fibrinogen.
Cause of edema in nephrotic syndrome: Hypoalbuminemia.

Cause of edema in nephrotic syndrome'. Salt {sodium) and water retention.


Finnish type of congenital nephrotic syndrome is due to mutation ln : Nephrin protein coded by NPHSl gene.
Genes involved in congenital steroid resistant nephrotic syndrome: NPH52 which codes for protein podocin.
Most common cause of nephrotic syndrome in children: Minimal change disease (Lipoid nephrosis).
Light microscopy finding in minimal change disease:No finding.
Electron microscopy findings in minimal change disease: Loss and fusion of foot processes of epithelial cells.
lmmunofluorescence finding in minimal change disease'.No immune complex deposition.
Presentation of minimal change disease: Selective proteinuria & edema.
Best response to steroid is seen in : Minimal change disease (lipoid nephrosis).
Characteristic feoture of nephritic syndrome in children: RBC casts in urine.
Marker for renalvasculitis in children: lgA level
Hematuria with dysmorphic RBCs in urine 2 days after an attack of upper respiratory tract infection : lgA nephropathy.

HSP is characterized by deposition of '.lgA in vessel wall and in mesangium.


Classical tetrad of HSP: Palpable purpura, abdominal pain, arthralgia/arthritis, glomerulonephritis
Not a feature of HSP : Thrombocytopenia
Renal involvement is not seen in HSP : After 4 months of onset of other sym ptoms.
Treatment of pauci-immune GN in children: Corticosteroids (prednisolone) with or without cyclophosphemide or azathioprine.
Triad of hemolytic uremic syndrome: Anemia, renal failure ( t gUN, 'l creatinine, oliguria) and thrombocytopenia.
Most common cause of Ht)S iGastroenteritis by E. coli (EHEC).

Characteristic peripheral smear findings in HUS: Fragmented RBCs (shistocytes), helmet cells, burr cells.
Most common cause of urinary tract obstruction in a male infant: Posterior urethral valve.
lmportant features of posterior urethralvalve: Hydronephrosis, distended bladder, recurrent UTl, vesicoureteral reflux in 500/0.

lnvestigation of choice for poor urinary stream sinceblrth : Voiding cystourethrography.


Most common cause of acute retention of urine in a male child: Meatal ulceration with scabbing.
Most common cause of chronic retention of urine in a male child: Posterior urethral valve.
t
ithout
a
inosis
a

a
eased
a
lcium
a

I
a

a
r's
a

a
a

ta

III
Cn,tp'r:rr'7

'.' ethod of choice for a newborn not passing urine : USG


: child with non-functional kidney, best evaluation: DTPA scan
)ynamic scan for renal function: DTPA scan, renogram or MAG3
>iatic scan for evaluation of 'structure' of kidney : DMSA
jest method for treatment of renal stone > 0.6 cm in a child: Extracorporal shock-wave lithotripsy (ESWL)

',\ost common cause of rencil artery stenosis in children in tndia:fakayasu arteritis.


^,,1ost common cause of renal artery stenosis in children in developed countries: Fibromuscular dysplasia.
Vost common cause of renalvein thrombosis (RVT) in children: Nephrotic syndrome due to minimal chang e disease.
lammon causes of RVT in newborn and infants: Asphyxia, dehydration, shock, sepsis.
lausesof nephrocalcinosis: ldiopathic hypercalciuria, Bartter's syndrome, Distal RTA. hyperoxaluria, Dent's disease, loop diuretics
fu rosemide).
) full term neonate can concentrate urine to:700-800 mOsm/kg.
4n older child can concentrate urine upto:1 200-1400 mOsm/kg.
aoncentrating ability reaches equal to adult level at'.1 year of age.
A newborn can dilute urine upfo : 50 mOsm/kg (similar to older children).
^ilost common infection in nephrotic syndrome: spontaneous bacterial peritonitis
Most common cause sf recurrent lJTl in a child Nesicaureteric reflux (VUR).
VUR is more common in: Newborn females.
Most common cause of renol scarring in a j year old child: VUR induced pyelonephritis.

Most common cause of UTI in children:E.coli.


Most appropriate method for collection of urine specimen for culture in infant:Suprapr-rbic aspiration.
Chronic pyelonephritis is more common ln : Females.
lmportant indices of pre-renalARF: Specific aravity of urine >1.2, Urine osmolality >500, Urine sodium < 20mEq/1, FENa < 10,6, BUN/
creatinine >20.

rtx
QUESTIONS

l. Theneonatalkidneyachievesconcentratingability c) PulmonaryHlperplasia
equivalent to adult's kidney by - (Ar 04) d) Flat chain
a) One year ofage 9. Pulmonary hypoplasia with uropathy diagnosis is-
b) Eighteen months of age
(CET Nov 15 Pattern)
c) Three to six months of age a) Potter syndrome
d) Just before puberty
b) Patau syndrome
2. Whic one of the following is the most common cause c) Perthe disease
of.abfisminal ma;s in neonatec - d) All of the above
(All India Dec. 14 Pattern)
a) Neuroblastoma
b) Wilm's tumour ECOTOPIC URETER
c) Distended bladder
d) Multicystic dysplastic kidneys 10. A3year old male child diagnosed to have acute
UTI developed left flank pain on ulatrasonogram,
the left ureter w{rs found to be duplicated the
most probable site of opening of ectopic ureter
will be - (CET luly 15 Pattern)
3, Which of the following is the most common renal a) Prostatic urethra b) Vas deferens
cyetie disease in infants is ? @ET Nov. 15 pattern) c) Seminal vesicle d) Trigone ofbladder
a) Polycystic kidney
b) Simple renal cyst
c) Unilateral renal dysplasia
d) Calyceal cyst
4. All of the following are true about childhood
polycystickidneydisease, except - PATHOPHYSIOLOGY
(Arls)
a) Autosomal dominant
b) Pulmonaryhypoplasia
It. All are features of nephrotic syndrome in children
except -
c) Renal cyst present at birth (All India Dec.13 Pattern)

d) Hepatic fibrosis
a) Lipiduria
b) Hyperlipidemia
5. Baby born at 30 weeks for 18 year old primi gravida c) Hyperalbuminemia
of weight 2 kg which died after 48 hours. Apgar d) Proteinuria
scores were 5 and 8 at 1 and 5 minutes. On autopsy
,1 All of the following are decreased in nephrotic
bilateral enlarged kidney with multiple radially
syndrome, except - (AIIMS lune 98. Dec 95, AI 97)
arranged cysts. Which of the following finfing is
expected to be associated with ?
a) Serum transferrin
(AIIMS Nov 07)
a) Imperforate anus b) Serum fibrinogen
b) Hepatic cyst and fibrosis c) Serum ceruloplasmin
c) Absence ofureter d) Serum albumin
d) Holoprosencephaly 13. Which is seen in nephrotic sundrome -
(NEET Dec.12 Pattern)
6. Unilateral renal ageneois is associated with -
(NEET Dec.12 Pattern)
a) Low serum calcium
a) Polycystic disease ofpancreas
b) Raised AT-III
b) Hiatus Hernia
c) Low lipid
c) Single umbilical artery
d) Platelet activation
d) Hlpogonadism 14. Edema in nephrotic syndrome is due to -
(AllMS May 12, Nov t0)
7, "Potter'c oyndromd' ie assoclated with -
(CET Nov.14 Pattern)
a) Sodium & water retention
a) Renal anomalies
b) Increased venous pressure
b) Severe oligohydramniot
c) Hypoalbuminemia
c) Flattened nose
d) Hyperlipidemia
d) All the above
8. Not a finding in potter syndrome- CAUSES
(All India Dec15 Pattern)
a) Bilateral renal agenesis
15. Most common cause of Nephrotic Syndrome in
b) Polyhydromnios children- (All India Dec.1j, NEET Dec.12 Pattern)
a) Minimal change disease
CHApts&:i7

r r igA nephropathy cushingoid with blood pressure af L28l&6 rnmHg


;; Mesangial GN and small subcapsular cataracts. The rnost appropri-
.1) FSGN ate therapy ofchoice is - (AIIMS Nou 04, May 03)

Child with proteinuria, generalised edema. With


a) Longterm frusemide with enalapril
hvpoproteinernia & hyperlipidemia-M"C. eause is-
b) Cyclophosphamide
sis is- (CET Nov 15 Pattern) c) Intravenous immunoglobulin
Ptttern)
a) Minimal change nephrotic syndrome
d) Intravenous pulse corticosteroids
b) IgA nephropathy 24. A child comes with steroid resistant nephrotic
c) Mesangial glomerulonePhritis syndrome secondary to FSGS, neit responsive to
d) FSGN methylprednisolone. l{hat next should be given -
- The most €ommon gen€ defect in idiopathic steroid
tAllMS .\4,ty I l)

resi$tant nephrotic syndrcme - a) Oralcyclophosphamide


(Ailins Abr 1 1, May 07,Not 06) b) Oral cyclosporine
a) ACE b) NPHS 2 c) Oral mycophenolate
e
c) HOX 11 d) PAX d) IV cyclophosphamide
m,
. i. The finnish type of congenital nephritic syndrome
occur$ due to gene mutation affecting the following NEPHRITIC sYNPROME
;:tern)
protein - @t 06)

a) Podocin 25. Characteristic feature of nephritic syndrome in


b) Alpha-actinin children - (All lndia Dec.13 Pattern)
c) Nephrin a) Lipid casts in urine
d) CD, activated Protein b) WBC casts in urine
c) RBC casts in urine
M I NIMAL C},IAI{GE DISEASE d) Albumin in urine
26. Which of following is not seen in nephritic
19. A year otd child suffering from nephrotic
5 synelrome - (NEET Dec.l2 Pattetn)

syndrorne is responding well to steroid therapy. a) Edema b) Hematuria


't
I&&at would be the rnost likely finding on light mi- c) HTN d) Hypocholestremia
: tn) croscopy- (A101)
27. Child with B.P.1901tr trO, pedal edema ++ ; facial
a) No finding edema ascites-absent. Gross hematuria diagnosis
b) Basement membrabne thicking ls- (PGl lune 2000)
c) Hypercellular glomeruli a) Acute G.N. b) Nephrotic syndrome
d) Fusion offoot processes c) Renal thrombosis d) Renal amyloidosis
10. Eest response to steroids is observed with -
(AtttndiaDec.llPattem)
IGA NEpHROpATHy & HSp
a) Focal glomerulonephritis
b) Lipoid nephrosis 10
ao. Marker for the renal vasculitis in children is -
c) Membranous GN (AI 98, AilNIS SePt 96)
d) Membranoproliferative GN a) Increased IgA level
2t. A.ll are trtre about minimal change G.N. except - b) Low complement level
(PGI Dec 99)
c) t Antineutrophilic cltoplasmic antibody titre
a) Selectiveproteinuria d) Increase antinuclear antibody
b) IgG deposition in mesangium 29" 12 years old Shyarn presented with gross hematuria
c) Common in age grouP 2-9 Years with 80% dysmorphic RBC's 2 days after a attack
d ) Responds to steroid's of upper respiratory tract infection diagnosis is -
1) A7 year old girl is brought with cornplaints of (AIIMS Nov 01)
generalized swelling of the body' Urinary a) Microangiopathic thrombotic anaemia
exarnination reveals grade 3 proteinuria and ttrre b) IgA Nephropathy
presence ofhyaline and fatty casts" She has no c) PSGN
history of hematuria. Which of the following d) H.S. purpura
statements about her condition is true - (At 0e)
38. A child presented with cola coloured urine, protein-
a) No IgG deposits or C3 deposition on renal biopsy uria 2+ & h/o rash 2 week ago. Probable Dx is-
b) C3 level will be low (PGl Nov 14)
c) IgA nephropathy is the likely diagnosis a) IgA nephropathy
d) Alport's syndrome is the likely diagnosis b) HSP
23. d 9 year old boy has steroid dependent nephrotic c) HUS
syndrorne for the last 5 year. The patient is markedly d) Wegener Granulomatosis
31. 10 year old boy with gross hematuria, loin pain, c) Wegener'sgranulomatosis
diarrhoea urine examination is normal. Serum C3 d) Kawasaki disease
Ievels are normal diagnosis is - (CET luly 15 Pattern) 1(t A S-year old child presents with perivascular IgA
a) Berger's disease deposition and neutrophilic collection. There is er-
b) PSGN ythernatous rash on the lower limb and non-blanch-
c) Microangiopathichemolpic anemia ing purpura. Probable diagnosis is - (ArrMS Nov ul
d) None ofthe above a) Henoch-Schonlein purpura
32. A sixyear old male baby presents to a hospital with b) Wegener's granulomatosis
recurrent gross hematuria for 2 years. There is no c) Vasculitis
h/o burning micturition or pyuria. Urine routine d) Kawasaki's disease
examination demonstrated no pus cells and urine 40, In Henoch-Scholein purpura renal involvement is
culture was sterile. Serum C3 levels were normal. not seen generally if no involvement till ?
What is the most probable diagnosis ? (Ar 08)
(CET Nov.14 Pattern)
a) Wilmb tumor a) 1 month after onset
b) IgAnephropathy b) 2 month after onset
c) Post-streptococcal glomerulonephritis c) 3 month after onset
d) Urinary tract infection d) 4 month after onset
JJ. The renal biopsy of a 6-year-old boy with recurrent 4t. A child was diagnosed as a case of pauci-immune
gross hematuria shows IgA nephropathy. The
crescentic glomerulonephritis. The treatment to be
urinary protein excretion is 130 mg/day. Which of given in this child is - (ArrMS May 02)
the following is the most appropriate next step in a) Prednisolone + Cyclophosphamide
the management - (AIIMS May 04)
b) Methylprednisolone
a) Administercorticosteroids c) Cyclophosphamide
b) Give Azathioprine d) Immunoglobins
c) Start Cyclosporine
d) Urinary bag sample
34. Henoch-Schonlein purpura is characterizedby the HEMOLYTIC UREMIC SYNDROME
deposition of the following immunoglobulin around
the vessels - (AllMS Nov 05)
42. A 8 yr old boy presents with petechie, azotemic

a) IgM b) IgG oligurea altered sensorium in casualty. There is


c) IgA d) IgE
history ofdiarrhoea for the past 5 days. The clinical
diagnosis is - (AilMS Dec 98)
35" All are seen in Henoch schonlein purpura except -
a) Acute prophyria
(NEET Dec.12 Pattern)
b) Idiopathic thrombocltopenic purpura
a) Thrombocltopenia c) H.S. purpura
b) Glomerulonephritis d) H.u.s.
c) Arthralgia
d) Abdominal pain 43. Most common cause of hemolytic uremic syndrome
is - (All lndia Dec.14 Pattern)
36. In HSP gross hematurea is seen in what o/o
of chil- a) E.coli
dren- (All India Dec15 Pauern)
b) Shigella
a) 5-10% c) Salmonella
b) l0-20Vo d) Psedononas
c) 20 - 30o/o
d) 30 - 40o/o
44. In shigella dysentry associated hemolytic uremic
37. A year old male had non blanching rashes over the
8
syndrome, the false statement is - (NEET Dec.12 pauern)
a) Leucocl,tosis
shin and swelling of knee joint with haematuria +++
b) Neurological abnormalities
and protein +. Microscopic analysis of his renal
c) Hepatic failure
biopsy specimen is most likely to show - (AilMS Noy 07)
d) Thrombotic angiopathy
a) Tubular necrosis
b) Visceral podocyte fusion
c) Mesangial deposits of IgA URINARY OBSTRUCTION
d) Basement membrane thickening
38. A child develops non-blanching macules and pap-
ules on lower extremities, mild abdominal pain and POSTERIOR URETHRAL VALVE
skin biopsy showed IgA deposition. Most appropri-
ate diagnosis is - (AIIMS May 09) 45' Most common cause of urinary obstruction in a
a) Drug induced vasculitis male infant is - (All India Dec.14 Pattern)
b) HSP a) Anterior urethral valves
b) Posterior urethral valves VESICOURETERIC REFLUX
c) Stone
,A d) Stricture 52. The most common underlying anomaly in a child
Ier- d6. Which is not a fcature of posterior urethral valve - with recurrent urinary tract infections is-
lnch- (NEET Dec.12 Pattern) (AIIMS Nov 03, AI 05)
lov 11) a) Palpable bladder a) Posterior urethral valves
b) Painful stress incontinence b) Vesicouretericreflux
c) Recurrent UTI c) Neurogenic bladder
d) Hydronephrosis a) Renal calculi
4;. A 3-year-old boy presents with fever; dysuria and 53. Vesicoureteric reflux is more common ln-
gross hematuria. Physical examination shows a (AttMS May 0a)
prominent suprapubic area which is dull on a) Newborn females
tttern) pecussion. Urinalysis reveals red blood cells but no b) Older girls
proteinuria. Which of the following is the most c) Older boys
likely diagnosis - (AIIMS May 06) d) Only during pregnancy
a) Acute glomerulonephritis 54. The most common cuase of renal scarring in a 3
b) Urinary tract infection year old child is - (AI os)
c) Posterior urethral valves a) Trauma
be d) Teratoma b) Tuberculosis
v 02) {8. One year old male child presentedwith poor urinary c) Vesicoureteral reflux induced pyelonephritis
stream since birth. The investigation ofchoice for d) Interstitial nephritis
evaluation is - (AllMS May 03) 55. The treatment of choice for primary grade V vesico-
a) Voidingcystourethrography (VCUG) ureteric reflux involving both kidneys in a6 month
b) USG bladder old boy is - (AIIMS May 03, Nov 04)
c) Intravenousurography a) Antibioticprophylaxis
d) Uroflowmetry b) Ureteric reimplantation
c) Cystoscopy followed by subureteric injection of
OTHER CAUSES teflon
d) Bilateral ureterostomies.
al 49. Most common auomaly of upper urogenital tract 56. A 4 month old female child presents with grade IV
e8) causing obstruction in children - (All lndia Dec15 Pattern) Vesico- Ureteral Reflux ffUR); without dilation of
a) Uretero pelvic junction stenosis urinary bladder. The Treatment of choice is -
b) Ectopic uretheral opening (Ar 2000)
c) Ureterocele a) Septran + Followup
d) Ectopic ureter b) Re - implantation of ureter
ne 50. A child presented with intermittent episoiles of left c) Injection ofcollagen at ureteric orifices
"n)
sided flank pain. ultrasonography reveals large d) BilateralUreterostomy
hydronephrosis with dilated renal pelvis and cortical
thinning with a normal ureter. Kidney differential
function was observed to be 197o which of the
UTI
following is the best management - (At 10)
57. Most common cause for UTI in children less than 5
a) Nephrectomy
years is ? (CET Aug. 12 Pattern)
b) Pyeloplasty
a) E. coli
c) External drainage
b) Klebsiella
d) Endopylostomy
c) Staphylococcus saprophyticus
51. A 6 year old girl presents with Recurrent E.coli d) Pseudomonas
infection in urine. Ultrasound of abdomen shows
58. A five year old male child presents with cofiphints
Hydroureter and Hydronephrosis. Micturating
of fever and abdominal distension. He is having
cysto- urethrogam shows filling defect in urinary
vomiting for the last five days. On examination there
bladder. The likely diagnosis is - (AI 2000)
are 6-8 Pus Cellihpf in urine. WBC count shows 787o
a) SacrococcygealTeratoma
neutrophils. What is the best line of management-
b) Vesicoureteric Reflux - grade II
(AIIMS Nov 2000)
c) Duplication of Ureter
a) S.end urine for culture and sensitivity and wait for results
d) Ureterocele
b) Send urine for culture and sensitivity and start I.V
antibiotics immediately
c) Send urine for culture, do an USG and start
chloroquine
d) Radio nucleotide studies MISCELLANEOUS
59. Which of the following is the most approprlate
method for obtaining a urine specimen for culture
in an 8 month old girl - (AilMS May 0a) BARTTER SYNDROME
a) Suprapubicaspiration
b) Indwelling catheter sample 66. True about bartter's syndrome are all except -
c) Clean catch void a) Hlperkalemic alkalosis (All India Dec15 Pattern)
d) Urinary bag sample b) Presents in neonate with ototoxicity have bartin
60. An 8 year old boy during a routine check up is found gene mutation
to have E. coli 1,00,000 cclml on a urine culture. c) Decreased K* assorption from thick descending loop
The urine specimen was obtained by mid-stream d) Autosomal recessive
clean-catch void. The child is asymptomatic. Which 67, AZ month old girl has failure to thrive, polyuria and
is the most appropriate next step in the manage- medullary nephrocalcinosis affecting both kidneys.
ment- (AIIMS May 04) Investigations show blood pH 7.4&,bicarbonate 25
a) Treat as an acute episode ofurinarytract mEq/ 1, potassium 2mBqlt, sodium l26mBqlland
infection chloride 88 mEq/I, The most likely diagnosis is -
b) No therapy a) Distal renal tubular acidosis (AIIMS Nov 0a)
c) Prophylactic antibiotics for 6 months b) Primaryhyperaldosteronism
d) Administer long term urine alkalinizer c) Bartter syndrome
61. Which one of the following statements is false with d) Pseudohypoaldosteronism
regard to pyrria in children - (At 03)
a) Presence of more than 5 WBC/hpf (high power DIAGNOSTIC IMAGING
field) for girls and more than 3 WBC/hpf for boys
b) Infection can occur without pyuria 68. 2yrs female has been sucessfully treated for uri-
c) Pyuria may be present without Urinary tract narytract incfection. Next step in management is-
infection a) Do nothing (AilMS e8)
d) Isolated pyuria is neither confirmatory nor b) Ultrasound
diagnostic for Urinary tract infection c) Ultrasound + DMSA scan
62. Not true about chronic pyelonephritis in children - d) 6 monthly culture sensitivity

a) Associated with Ureteric reflux (NEET Dec.12 Pattern) 69. In a child, non - functioning kidney is best diag-
b) Associated with Intrarenal reflux nosed by - (At os)
c) Associated with renal scarring a) Ultrasonography
d) Males are more affected than female b) ryu
c) DTPA renogram
d) Creatinineclearance
RENAL FAILURE
70. Method of choice for a New born child not passing
63. In children renal failure in terms of urine output is urine for 36 hrs -
defined as ? (CET Nov. 13 Paxern) a) Ultrasound of kidney & bladder (AIIMS May es)

a) Less than 0.3 ml/kg/hr b) CT Scan


b) Less than 0.5 ml/kg/hr c) Cystoscopy
c) Less than 0.8 ml/kg/hr d) X-ray pelvis
d) Less than I ml/kg/hr
64. A child has diarrhea since 8 days. He is dehydrated OTHER MISCELLANEOUS
and urine output is reduced. Which of the following
is not correct regarding the renal failure in this 71. lntra uterine hy&onephrosis of 32-34 weeks-rnan-
patient - (Ar 10) agement- (All India Decl5 Pattern)
a) Urinary sodium > 40 m.eq/L a) Intrauterinedrainage
b) Urinary osmolality > 500 Mosmol/L b) Wait until 3 weeks
c) FENa< 1olo c) Tmmediate delivery
d) BUN/creatinin > 20 d) Require serial USG and other associated anomalies

65. In SCHWARTZ formula for calculation of creatinine 72, A male child with Fanconi syndrome with nephro-
clearance in a child, the constant depends on the calcinosis has a variant of dent disease. All are true
following except - (AllMS Nov 06) except - (AIIMS May 11)

a) Age a) Hypercalciuria
b) Method of estimation of creatinine b) Proteinuria
c) Mass c) Similar presentation in father
d) Severity of renal failure d) Rickets
d) Radio nucleotide studies MISCELLANEOUS
59. Which of the following is the most appropriate
method for obtaining a urine specimen for culture
in an 8 month old girl - (AIIMS May 04) BARTTER SYNDROME
a) Suprapubicaspiration
b) indwelling catheter sample 66. True about bartter's syndrome are all except -
c) Clean catchvoid a) Hlperkalemicalkaiosis (All lndia Decl5 Pattern)
d) Urinary bag sample b) Presents in neonate with ototoxicity have bartin
60. An 8 year old boy during a routine check up is found gene mutation
to have E. coli 1,00,000 cclml on a urine cultsre. c) Decreased K* assorption from thick descending loop
The urine specimen was obtained by mid-stream d) Autosomal recessive
clean-catch void. The child is asymptomatic. Which 67. L2 month old girl has failureto thrive, polyuria and
is the most appropriate next step in the manage- medullary nephrocalcinosis affecting both kidneys.
ment- (AIIMS May 0a) Investigations show blood p}{7.48, bicarbonate 25
a) Treat as an acute episode ofurinarytract mEq/ 1, potassium 2mEqll, sodium 126 mEqll and
infection chloride 88 mEq/I. The mnst likely diagnosis is -
b) No therapy a) Distal renal tubular acidosis (AllMS Nov 04)

c) Prophylactic antibiotics for 6 months b) Primary hlperaldosteronism


d) Administer long term urine alkalinizer c) Bartter syndrome
61. Which one of the following statements is false with
d) Pseudohlpoaldosteronism
regard to pyuria in children - (At 03)
a) Presence of more than 5 WBC/hpf (high power DIAGNOSTIC IMAGING
field) for girls and more than 3 WBC/hpf for boys
b) Infection can occur without pyuria 68. 2yrs female has been sucessfully treated for uri-
c) Pl.uria may be present without Urinary tract nary tract incfection. Next step in management is-
infection a) Do nothing (AilMS e8)
d) Isolated pluria is neither conflrmatory nor b) Ultrasound
diagnostic for Urinary tract infection c) Ultrasound + DMSA scan
62. Not true about chronic pyelonephritis in children - d) 6 monthly culture sensitivity
a) Associated with Ureteric reflux (NEET Dec.12 Pattern) 69" In a child, non - functioning kidney is best diag-
b) Associated with Intrarenal reflux nosed by - (Ar os)

c) Associated with renal scarring a) Ultrasonography


d) Males are more affected than female b) rvu
c) DTPA renogram
d) Creatinineclearance
RENAL FAILURE
70, .Method of choice for a New born child not passing
63. In children renal failure in terms of urine output is urine for 36 hrs -
defined as ? (CET Nov. 1j Pattern) a) Ultrasound of kidney & bladder @IIMS May es)

a) Less than 0.3 ml/kg/hr b) CT Scan


b) Less than 0'5 ml/kg/hr c) Cystoscopy
c) Less than 0.8 ml/kg/hr d) X-ray pelvis
d) Less than 1 ml/kg/hr
64, A child has diarrhea since 8 days. He is dehydrated OTHER MISCELLANEOUS
and urine output is reduced. Which of the following
is not correct regarding the renal failure in this 71. Intra uterine hydronephrosis of 32-34 weeks-man-
patient - (Ar 10) agement- (All lndia Dec15 Pattern)

a) Urinary sodium > 40 m.eq/L a) Intrauterinedrainage


b) Urinary osmolality > 500 Mosmol/L b) Wait until 3 weeks
c) FENa<1%o c) Immediate delivery
d) BUN/creatinin > 20 d) Require serial USG and other associated anomalies

65. In SCHWARTZ formula for calculation of creatinine 72. A male child with Fanconi syndrome with nephro-
clearance in a child, the constant depends on the calcinosis has a variant ofdent disease. All are true
(AIIMS Nov 06) except * (AIIMS May 1t)
following except -
a) Age a) Hypercalciuria
b) Method of estimation of creatinine b) Proteinuria
c) Mass c) Similar presentation in father
d) Severity ofrenal failure d) Rickets
with single umbilical artery - (SGPGI 0s)
/ -r. 8 yr old child with BP 180/100 mm Hg' urea 90'
cells' 1-2 a) Central nervous sYstem
creatinine 5.3, urinalysis shows 15-20 pus
past h/o of b) Cardiovascular
RBC, protein 1+ & has no significant
diagnosis is- c) GenitourinarY
similar complaint. Most likely d) Skeletal

a) Post infective glomerulonephritis (AttMSNov


10) 81' In nephrotic syndrome the essential feature is
-
(AIrMs
b) Accelerated hlpertension with ARF a) Pioteinuria 83)

b) Hypoalbuminemia
c) IdioPathic RPGN c) Hlperlipemia
d) Chronic interstitial nephritis with VUR d) Edema
renal
A 3 -year old boy is detected to have bilateral
Choose among the following the most important
74.
.rf.ir. Metabolic evaluation confirms the presence 82'
lab finding in nephrotic syndrome -
gN e5)
normal blood levels
oi*utf."a hypercalciuria with a) B-l protein
of calcium, m-agnesium, phosphate'
uric acidand
b) Hlperkalemia
creatinine. A diagnosis of idiopathic hypercalciuria
all' c) Hlpoalbuminemia
is made. The dietary management includes
- (AllMS MaY 0j' Nov 04) d) Hypertension
excePt syn-
a) Increased water intake 83' Most common infection in a child nephrotic
drome- MAHE 07' 08)
b) Low sodium diet
a) Spontaneous bacterial peritonitis
c) Reduced calcium intake
d) Avoid meat Proteins b) Pneumonia
and hematu- c) UTI
75. A child presents with abdominal colic
a stone 2'5 cm in diameter d) Cellulitis
ria. On oltroror*gtuphy is-
is seen in the renal pelvis' The next step in manage- 84. The commonest type of renal lesion in children
(rN es)
ment of this case is - (AllMS Nov 2000' Al 01) a) Lipoid nePhrosis
a) PyelolithotomY b) Nephroureterostomy bj frt.-U.u"o proliferative glomerulonephritis
c) Conservative d) ESWL c) Focal glomerulonePhritis
of renal artery stenosis in
d) Diffuse glomerulosclerosis
76. Most common cause of
children in India is - iAttMS Mav 07) 85' A year old child presents with four days history
3

a) TakaYasu Aortoarteritis p.rflit"r, of face, fever and tea coloured urine'


any of
b) Fibromedial hlPertroPhY ;;;i;g the course ofhis disease, he can have(UPSC
the complications except -
06)
c) FibrointimalhlPerPlasia
d) PolYarteritis Nodosa a) Hlpokalemia
found in - b) HlPertensive encePhaloPathy
77. Renal vein thrombosis is most commonly
c) Acute renal failure
a) Focal glomerulosclerosis @Gt lune 97)
d) Acidosis
b) Membranous infant
c) Minimal change GN 86' Straining and dribbling of urine in a male
with recurrent urinary infection should lead to
d) AcutePYelonePhritis (UPSC 07)
the suspicion of -
78. A 3-month old infant presents with bilateral
following can a) Vesico-ureteric reflux
medullary nephrocalcinosis' All of the
except - b) Posterior urethrai valve
cause medullary nephrocalcinosis
IALIMS MoY 09) c) Peivic ureteric junction obstruction
a) Hlperoxaluria d) Phimosis
b) Bartter's sYndrome micturat-
c) Prolonged use of furosemide 87. Posterior urethral valve is diagnosed on
d) ARPKD ing cystometrogram bY - (Karn 95)

a) Dilatation of posterior urethra


EXAMINA- b) Bladder neck contracture
QUESTIONS OF VARIOUS OTHER c) Vesico-Ureteric reflex
TIONS d) Bladder wall hlPertroPhY
to
capacity in 88. An infant with severe dehydration secondary
79. The maximum urinary concentration suddenly presents with proteins and blood
(Karnataka 02) diarrhea
full term neonates is -
in urine. The most probable diagnosis is-
a) > 1000 mOsm/litre a) Renal veln thrombosis
(Delhi PG Feb. a9)
b) 350 - 450 mOsm/litre b) PyelonePhritis
c) 900 - 1000 mOsm/litre c) AcuteglomerulonePhritis
d) 600 - 700 mOsm/litre
d) Lower nePhrosis
80.Malformationsofthefollowingorgansystemofthe
fetus are found to be most commonly associated

TII
:Sallt;:i;l :;;t,l :'2:i,'r::.,..i 1:. ir' ::.'

ANSWERS
1. Ans. is 'a'i.e., One year of age
[Ref: paediaiics Urology, e"or rr"
"Normal adult kidnev can produce
urine with of greater than
.an,osmolality 1000 water. The healthy neu.
-m lsm/kg
';;;':!i::,:;?::::;r;;,";:xi;i':j!,:,i;i:i'1,i:,i;:;,'m.,ati,v
increases graituatfy a"ri"g'ri, .r.nty s00.700 m.sm / kg
i few months a;;;;e;r";;;':;:!r:J:;!":;:r'::r#";;";:r;
?;;:r"" one year
on the other hand' the newborn can dilute his urine to a minimum
of 50 mosm/kg much rike an order
child.
2' Ans. is t'i.e., Murticystic dysprastic Kidney,s
[Ref, o.p. Ghai 8th/e p. 468 {r vh/e p, 470)
"Multicystic dysplastic kidney is the most
common cause of an abdominal
mass in the new born, _ Nelson

CONGENITAL ANOMALIES
Ans' is t' i.e., unilateral renal dysprasia
[Rel sutton Radiorogy *th/e p. 11051
o The multicystic dysplastic kidney
is the commonest form of congenitar
complete uretric obstruction infetaf cystic renar dysprasia, and is due to
lrfr. Th, condition xLru,o,tti uilateral; bilateral disease is lethal.
4- Ans. is 'a'i.e., Autosomal dominant
fRef: Netson lgth/e p. 17061
' 7:i::::;,i;:t',i;';hi;";,i:::.sehas autosomat recessive inheritance,thererore
it is arso known as autosomat
5. Ans. is 'b'i.e., Hepatic cyst and fibrosis
[Ref Nelson tSth/e p. 2185]
o The,crinicar findings suggest autosomar recessive
porycystic kidney disease.
r Both kidneys are ma*eilly enlargedand
grossly siow innumeroui ryrtrthroughout
the cortex and medulla.
' H::ffiiff[|'u'"'.,ffi;il[* micro"cvsts radiating *,iuiro from to the iorixi;; primarily within the
o Liver involvement is characterize
d'by bile duct proliferation andectasia
as well asby hepaticfibrosis.
6. Ans. is t'i.e., Single Umbilical Artery
[Ref Nelson lVh/e p. t7B3; O.p. Ghai 6th/e p. 467]
Malformations associated with unilateral
renal agenesis
o Single umbilical artery o Controlateral vesicoureteric reflux
o Absent ureter o Absent Ipsilateral vas defrens
7. Ans. is t' i.e., All of the above
[Ref O.p, Ghai 8th/e p. 505 6 Vh/e p. 170; Nelson tgth/e p. 2221]
o Potter syndrome is characterized
by _

i)Bilateral renal agenesis


ii)
Pulmonary hypoplasia
iit) Potter facies -> widely separated eyes with
epicanthic folds, Iow set ears, broad
receding chin. compressed flat nose,
o Maternal USG demonstrates _

i) Oligohydramnios ii) Nonvisulization of bladder


iii) Absent kidney
8. Ans. is'b'i.e., Polyhydromnios
[Rel Nelson l}th/e p. 537]
Potter syndrome
o Bilateral renal agenesis-is incompatible
with extrauterine life and is termed potter syndrome.
o Death occurs shortly afier
birth pulmonary
from hypoplasia,
o The newborn has a charactlristii
facial ippr)rinrr, termed potte,r
facie.s. The eyes are widery separated
folds' the ears are low ,it, rh, ,i* is broad an'd compressed
:;::#;;#."'c Jlat, the chin is receding, and there arelimb

' :ii;if;:;;':r;;;:r:l:f1,';;::*.'whenmaternatutrrasonographvdemonstratesotigohydramnios,nonvisuatization
9. Ans. is'a'i.e., Potter syndrome
[Ref Nelson lgth/e ch, Sjfl
CnxFin.r

ECTOPIE URETER

&ms, is ';t i."*.,Ilrfistattie mrellara ii({i }f,rrlsoru xStt'le 1;' 2238}


Opening of ectopic ureter in Male
a Posterior (prostatic) urethra above external sphinctor --> 47o/o
r Seminal vesicle ) 33o/o

x Prostatic utricle -) l0o/o


r Ejaculatory duct 1 5o/o

r Vas deferens -) 5o/o

NEPHROTIC SYNDROME

PATI.IOPHYSIOLOGY

l1 Afis. ig '{' n,*.,*4rypxr.albtu"mzyxt*wa'tal{li:-l': N*tls*w lN't'k: 1t. Jt92; {.}"1}" Gluti 7'}'le p. a50]
o Nephrotic syndrome is a group of disease having different pathogenesis and characterizedby -
1) Proteinuria (> 3.5 gm/day) 4) Hlperlipidemia
2) Hypoalbuminemia 5) Lipiduria -+ lipid casts
3) Edema 6) Hypercoagulabiiity
t2. Ans. is 'H i.e., Serum Fibrinogen lRe! Rabbin's Vh/e p. 978; tlanisaru trVh/e p. 1790; Nelsan 18th/e p. 21921
c Fibrinogen level is increased in nephrotic syndrome d/t increased hepatic synthesis of fibrinogen.
I i. t\tx*. isud i.*.,.{,qrw scr&fi:} *a\*izxwa f l{rl': .1lso it,c cl:'avL: t:xpluu*lion}
o In nephrotic syndrome there is hypocalcemia and secondary hyperparathyroidism as a consequence of loss of
cholecalciferol binding protein.
o Secondaryhyperparathyroidism causes hypophosphatemia by increasing renal phosphate excretion.

L4" Ans, is l.e.,Hypoalburninemia lRef : Diseases of kidney and utinary tract 4th/e p. 37761
'c'
e proteinuria in nephrotic syndrome leads to hlpoalbuminemia that results in decreased colloid osmotic pressure
which results in edema.
"The edema formation of nephrotic syndrome is secondary to increased sodium retention from intravascular
yolume depletion from low plasma oncotic pressure due to hypoalbuminemia."

cAusES

n'", is '&' r..{,,,W,lvzivuxa', *lzareg* dis*xse UtrtI: *.lt {ih*i &'t'/* t:.'}77 t::'?'t'i{' l}' 4571
i&arp.
o Minimai Change Disease is the most common cause of Nephrotic syndrome in children
*ty" k*s, ia"az' i.*., Millvlwz"*-tr *fueffiVp I{"S. llt$ t}{'ani 7't'le p. 45)
L7. Ans. is ob' i.e. NpHS-2 lRef : Robbinb vh/e p. 98i-984; Nelson tr
gth/e p. 2192, 21951

,t:,i;,t:idC1f[i

NPHSl 19q13 nephrin slit diaphragm Nephrotic syndrome of finnish tYPe

NPH52 1q25-3'l podocin slit diaphragm Steroid resi stant nephrotic syndrome

[8. Ans. is t' i.e", Nephrin lRef : Nelson 18'h/e p. 2192, 2195)

&,I I I{ I TMAL CI.IAN6 E DISEASE

lo Ans. is ,d i,e.,No finding lRef: o,P. Ghai 9th/e p. 477 6 vh/e p. 452; Nelsan 1Yth/e p. 2190, 2192)

e Minimal Change Disease is the most common cause of Nephrotic syndrome in children.
o Minimal change disease has very good prognosis with excellent response to steroids.
c Pathological findings include
7. Light microscopy
-) No abnormality
2. Electron microscopy -+ Obliteration (loss) andfusion offoot processes
ofepithelial cells.
3. Immunofluorescence -+ No deposits of immune reactants
4. Serum complements -+ Normal levels
20. Ans. is 'b'i.e., Lipoid nephrosis
fRef: O.p. Ghai 8,h/e p. 479 dz Th/e p. 452; Nelson
r Minimal change GN is also referred to as lipoid
lyth/e p. 2193]
nephrosis.
r Minimal change GN has an excellent response
to steroids.
21. Ans. is 'b'i.e', IgG deposition in mesangium
[Ref: o.p. Ghai 8th/e p. 427 6 vh/e p. 4s1_452)
There is no depositon of immune reactants.
.,,,
Ans' is'a'i'e" No IgG deposits or c3 deposition
on renal biopsy [Ref Netson
This girl is having
r8n/e p. 2rg0-2r92]
i) Generalized swelling (edema)
ii) Grade 3 proteinuria
11i) Fatty casts (lipiduria)
iv) No hematuria
o All these features suggest the diagnosis
of nephrotic syndrome.
o Minimal change disease is the
commonest cause of nephrotic syndrome
r Minimal in chirdren.
change disease is characterized by absence
oiigc a.porr* andlo. comprement on immunofluorescence.
23. Ans' is 'b' i'e.l cyclophosphamide
rRef: o.p. Ghai 8th/e p. 479 6 Vhre p.
Ifa steroid dependent patient develops severe
452;.Nerson 18th/e p. 2193)
corticosteroid toxicity (cushingoid
and or growth failure), then alternatives toxicity, hlryertension cataracts
available are
a) Cyclophosphamide c) Tacrolimus e) Levamisole
b) Cyclosporine d) Mycophenolate
Treatment of Nephrotic syndrome

Nephrotic syndrome
+
Daily steroid (prednisone)

Respond within 2 weeks


Fail to respond within g weeks
+
Steroid reponsive nphrotic syndrome +
Steroid resistant nephrotic syndrome
+
Daily prednisone for 6 weeks +
Adverse effects of steroids ----* Alternative drugs
+ -) ,-* tcustringoioj (cycl ospo ri n e or cycl ophosph
Alternate prednisone for 6 more weeks/ a mi de)

Remission Relapse

,,,
lnrrequent _ Frequent
Sternid d
+
Restart prednisone

Also know
Infrequent relapses -) If patient gets 3 or less relapses in a
a
year
Frequent relapses -) If a patient
has 4 or more relapses in a year
Steroid dependent -) When relapse occurs within 2 weeks
of discontinuation of drugs
Steroid resistant -) when patients do not respond to the initiar
t/t or respond to it transientrv
r. .'i::l':l:i
lelr 4BiI
Ans is'H i.e., Oral cyclosp otine lRef: Readbelowl
o The treatment options for steroid resistant nephrotic syndrome are:
(i) Calcineurin inhibitors (cyclosporine, Tacrolimus)
(n) IV or oral cYcloPhosPhamide
(lll) Levamisol
(iv) MycoPhenolate
(v)Pulse corticosteroid
r All above immunosuppressants are used along with corticosteriods (Prednisolone or methylprednisolone)'
t Cyclosporine and cyclophosphamide are most commonly
used'
therapy for steroid resistant nephrotic syn-
o Despite these options, there is lack ofconsensus on first line appropriate
drome.
o According to Indian fournal of pediatric (vot. 46, Ian 17 ,2009)
the efficacy of these durgs are (in decreasing order):
> IV cyclophosphamide + Prednisolone > Pulse Corticosteroids
Tacrolimus + prednisolone > cyclosporine + Prednisolone
+ Prednisolone'
(IV dexamethasone + orol ,yrloplrirphamide + Prednisolne) > oral cyclophosphamide
r So, amongst the given options, best answer is cyclosporine'

:5. Ans, is 'C i"e.,RBC casts in urine lRef: Nelson 18"/e p' 2169)
lPresenceofRBCcastsinurineisclassicalfeatureofnephriticsyndrome,
td' i.e., Hypocholestremia {Re! o.P. Ghai rth/e p. 471 & 7h/e p' 447; Nelson 18'h/e p' 21681
)6. Ans. is
affecting the kidneys, more specifically glomerular
r Nephritic sl,ndrome is a coliection of signs associated with disorders
hypertension, edema and oliguria'
disorders and is characterized by haematuria, protenuria,

27. Ans. is
,l
i.e.,Acute G.N. lRel O.P. Ghai yth/e p. 474 d, 7h/e p. 447; Nelson 18th/e
p. 21681
caused by acute
r Hematuria, hypertension and edema suggest the diagnosrs of nephritic syndrome
glomerulonePhritiq.

IGA NEPHROPATHY & HsP

28. Ans is 'd i.e.,Increased IgA IR'l Nelson 18'h/e p' 21711
adult and is perhaps the most frequent type
r IgA nephropathy or Berger's disease affects children and young
of glomerulonephritis in children'
29. Ans. is'b' i.e., IgA Nephropathy [Ref.: O.P. Ghai \e/e p' 474 6 Vh/e p' 446; Nelson 18'h/e p' 2171)
and dysmorphic RBCi) 2 days after upper respiratory
r The patient is having glomerulo nephritis @ross hematuria
tract infection.
-+ IgA nephropathy, Post streptococcal glomerulonephritis, H'S' Purpura'
r Three conditions can manifest like this

Glomerutonephritis after Upper Respiratory tract infection

After 2 days After 10-12 daYs

IgANephroPathY
o Isolated hematuria
(no symPtom other
than hematuria)

Post streptococcal ll.S. Purpura


slomerulonePhritis o Symptoms such as :
o Headache Coliclqt abdominal Pain
o Edema Polyarthrilis
o H1;pertension Purpuric rash
o Gen. Sym. such as anorexia
naus ea,Yomiting, malaise
46.

47. Ans. is t'i.e. Posterior Urethral Valve lRef: Nelson lgth/e p. 22411
It is a case of Posterior urethral valve presenting with urinary tract Infection.
PUV is the most common cause of obstructive uropathy in a male child.
The keyto diagnosis are -
Age and sex of the patient
Urinary tract infection is rare in male child , greater than one year old and less than 5 years old. presence ofurinary
tract
infection in this age group suggests , presence of some obstructive uropathy. ( posterior urethral valve in this
case)
Presence of Suprapubic dullness
Presence of suprapubic dullness suggests urinary retention, which is seen in this case due to obstruction
caused by
posterior urethral valves.

48. Ans. is 'd i.e., Voiding cystourethrography lRef. Nelson 18n/e p. 22411
o Poor urinary stream since birth suggests urinary tract obstruction (risually infravesical).
o Most common cause of urinary tract obstruction in a male child is --> gtosterior urethral yalve.
o And the best diagnostic method for posterior urethral valve is voiding cystourethrogram.

OTHER CAUSES

49. Ans. is 'a' i.e., uretero pelvic junction stenosis fRef: Radiologla Brasileiral
o Most common cause of urinary tract obstruction in children -+ Posterior urethral valves.
o Most common cause of lower urinary tract obstruction in children -+ Posterior urethral
valves.
o Most common cause of upper urinary tract obstruction in children -+ PUJ obstruction.

50. Ans. is'H i.e., Pyeloplasty


Pelvi-ureteral junction (PUJ)/u reteropelvic (U pJ)
o It is most commonly caused by intrinsic stenosis of a segment of ureter which is usually
localized to the region of
pelviureteric junction. Other causes include ureteric folds & extrinsic obstruction by crossing lower pole
veisels.
Presentation
tHydronephrois revealed by maternal IJSG (Dilateil renal pelvis but normal ureter).
r Palpable renal mass in new borns or infants.
o Abdominal flank or back pain typically exacerbated by drinking large volumes
of tiquid.
o UTl/Haematuria after minimal trauma.
Epidemiology
r Left side more commonly involved than right (Bilateral = 10%).
o Male more commonly involved than females (M : F = 2 : 1).
Initial Diagnosis by Ultrasonography
o Hydronephrosis with dilated renal pelvis.
o Ureter if visualized is of normal caliber (not dilated).
Renal Differential function
Nuclear Renography is used to determine Renal Differential function
o A differentialfunction of < 40o/o is considered significant and ofien an indicationfor
surgical management.
Management
o Management depends on the differential renal function and AP diameter of pUI -
:
l) Differential renalfunction ) 40o/o especially when AP diameter of PUI < 30 mm -> Conseryative treatment.
2) Dffirential tenal function < 40o/o especially when AP diameter of PUI > 30 MM -+ Surgical
treatment is
indicated. Pyeloplasty is the surgical procedure of choice. other surgical procedures used are external
drainage
(nephrostomy ), endopyelotomy and nephrectomy.
o Differential function of < 10o/o is often considered as an arbitary cut off for nephrectomy.
51. Ans. is'd'i.e., Ureterocele lRel Nelson 18th/e p. 22381
c IVPinureteroceleusuallyshowsarounilfillingdefect,sometimeslarge,inthebladdercorrespondingtotheureterocele,
and characteristic finding ofduplication ofthe collecting system (ureteroceles are nearly always associated with ureteral
duplication).
About other options
o An isolated VUR or Duplication of ureter do not explain the filing defect in the bladder.
o Sacrococcygeal Teratoma :

r The most common presentation here is with an abnormal, obvious protruding mass from the sacral area. Out of
the 4 varieties known, only Tlpe iV which contributes 9.8 ie entirely pre-sacral and not visible externally
o/o

(90.\Vo tumors are visible externally).


r Bladder and Rectum maybe displaced anteriorlybut a filting defect is not characteristic as in ureterocele'
r Ureters may be partially obstructed resulting in hydro-ureter and hydronephrosis.

vEslcou RETERIC . REFLUX

52. Ans. is 'H i.e., Vesicoureteric reflux fRef. O.P. Ghai 8th/e p. 506 6 Vh/e p. 456; Nelson 18th/e p. 2228-22331
"The most common abnormality seen in a child with UTI in a voiding cystourethrogram is vescioureteric reJlux.
It is identified in approximately 40% of patients"'

-5^r. Ans. is 'l i.e.,New born females lRe! Nelson 18th/e p' 22301

o Reflux is more commo nrnfemales and it is usually detected earlier in females so we can say that reflux is present from
birth but it is usually detected at 2-3 years of age but in females the age of detection is earlier.
54. Ans. is t' i.e., Vesicouretrial reflux induced pyelonephritis lR.ef: Pediatr Nephrol 1993 Aug; 7@) : 361' 4 The small
scarred kidney in childhood;. Risdon R.Al
'Reflux nephropathy is now a generally accepted term to describe small scarred kidneys discovered during childhood;
it recognises the close association between this renal lesion and vesicoureteric reflux (VURI.
Renal scarring is most commonly a result of chronic pyogenic infection of the kidney or chronic pyelonephritis.
Chronic pyelonephritis occurs only in patients with major anatomic abnormalities, such as obstructive uropathy,
struvite calculi or, most commonly, VUR (in 30 to 45% of young children with symptomatic UTI).

55. Ans. is 'a' i.e., Antibiotic prophylaxis lRef.: Nelson 18th/e p. 22331
r Since the age of the boy is less than 1 year we wili try medical management.
o See the below given tables from Nelson and the answer will be clear.

r-il Any Antibiotic prophylaxis


llL=,v 0-s Antibiotic prophylaxis Sl-rgqry
il-tv 6-10 U/L : Antibiotic prophylaxis Surgery
tl{- l[" ::.;1
...A'a10.' ,,'r. ',..1..,,11',',1.-:1',,." ...'.':gfi'iiangJf ,,,,,.,'":..':.
.,

<1 Antibiotic prophylaxis Surgery

':': V:,. :' : 1'--5 ". rNo


',, Ui't :,Antibiotjc,piaphylaxii ,;Surgery

1-5 No B/L : Surgery

,,',-V,,. ', 1 -,5.' Yes Surgery


6-C Surgery

o Grading of VUR : is based on the appearance of the urinary tract on Micturating Cystourethrogram (MCU).

Grade I Reflux into a non-dilated ureter


:i '
lf: . "::.Reftux into.tt e uppercoll*ctlo*g spem w,itlraut:alilataiioi"r.';': ':
lll
Grade Reflux into dilated ureter and /or blunting of calyceal fornices

M " " .Reflux into,a Erolsly dilaiqd' trreter,


6rade
GradeV Gross dilatation of the ureter, renal pelvis & calyces: calyces show loss of papiltary
impression.
Note - Surgical management involves, modi4ring the abnormal
uretrovesical attachement to create a4:r or5:1
intramural length to ureteral diameter, and it is done ratio of
by reimplantation of ureter.
56. Ans. is 'a'i.e., Septran + follow ap [Aef: See above explanation]
Treatmentof grade MUR
o Age 0-5 years
r Give antibiotic prophylaxis first
r Follow up the patient, if recurrences occur, surgicar management shourd
be done.
e Age6-l0years
1) Unilateral -+ Same as above
2) Bilqteral -+ Surgery

UTI
57. Ans. is '{ i,e,, E. coli lRef: Nelsan lSthle p. 2224-227; CpDTl6,h/e p.
715}
o 75 to 90o/o of all infections are caused by E. coli
followed by Klebsiella and proteus.
58. Ans' is 'H i'e" send urine for culture and sensitivity and
start I.v. antibiotics immediately
@/e p. 455 & 7"/e p. 456; CPDT L|th/e p. 63A; lRef a.p. Ghai
Nelson lgth/e p. 2225, 22261
' abdominal distension with 6-8 pus cells/hPf in urine
suggest the diagnosis of UTr, more specificaily
iiltJ;:p;1g'
o The investigation which almost confirms the.diagnosis
of u.T.I. is presenceo/6-g puscells / high power
e "The presence of pyuria (>5 white blood celk/ high power field.
field) is consktent with urinary tract infection.,,
e once U'T'I' is suspected, urine specimen - CPDT
Dorit wait for results of the culture.
is sent for culture and treatment started :
59. Ans. is ?' i.e., Suprapubic aspiration
lkef A.p" Ghai *le p. 484 & frle p. 456; N*an t*/e B. 2225; Cambell,s
Uralagy thle p. tS47]
o There are several ways to obtain a urine sample, some
are more accurate than others. These methods
l Midstream urine sample are -
r It is satisfactory in toilet trained children and in circumcised
boys. But in young infants and boys who cannot
'K::r{!.'- f"reskin, it is not reliable. Such samples will usuatly ieflect periuretilratand prepucial organisms
2' collection in an adhesive sealed sterile collection bag after
disinfection ofthe genitals
r Is a useful technique in infants, but potential contamination
from genitals and perianal area can occur.
3' urine specimen through catheterization or indwelling catheter
r Urine specimen through a catheter is less likely to ie
-
contaminated than the voided specimens as described
above' rf is also invasive than su?trapubic aspiration, but it is
less
not as sensitive as suprapubic aspiration.
r A catherized specimen is reliable if the first portion of the
urine that may contain urethral organisms is discarded
and specimen is taken from later flow through the
catheter, but it has the disadvantage of being
of potentially introducing urethral organisms into trqumatic and
sterile bladder.
r Suprapubic aspiration _
t suprapubic aspiration is best method to coilect urine specimen
t It is the most reliable method for curture.
r It can be performed safely in children and in premature infants by 2l
or 22 gaugeneedle.
r This procedure avoids any sort of contamination, however it is invasive.
So it is used only in infants and selected
patients.
o Note:
t The sensitivity of suprapubic aspiration is 99 %
t The sensitivity of calherization is 95o/o

ffi. Ans' is of Le'" Treat as an acute episode of urinary tract


infection tk.ef: Ne;son r*le B. 22zsl
o If the urine culture shows ' 100000 colony counts/ml,
the definitive diagnosis of u.T.I. is confirmed.
r It does not matter whether the patient is symptomatic or asymptomatic. If the diagnosis of U.T.I. is confirmed on
urine culture, the patient should be treated accordingly.
o Criteria for diagnosis of U.T.I. on midstream urine sample -
r If 3 100,000 colonies/ml + UTI confirmed (Does not matter whether it is symptomatic or asymptomatic).
r If 10000 colonies/ml and the patient is asymptomatic -+ UTI unlikely
r If 10000 colonies/ml with symptoms of U.T.I. + U.T.I is likely
61. Ans. is 'a' i.e., Presence of more than 5 WBC/hpf for girls and more than 3 WBC/hpf for boys [Ref: Nelson
18th/ep.2226)
o P1'uria is the presence of more than 5 wBCs / high power field, both for boys and girls
o Pprria suggests infection, but infection can occur in the absence ofpyuria. - CPDT
o Pyrria can present without UTI. - Nelson
r So, Isolated pyrrria is neither confirmatory nor diagnostic for UTI. - Nelson
62. Ans. is'd'i.e., Males are more affected than female lRef: Nelson lVh/e chap S3B; www. emedicinemedscape.com)
o Being - straight, short & wide urethra, recurrent UTI is more common in female, so CPN is also twice as common is
female than male.

RENAL FAILURE
63. Ans. is 'd i.e., Less than 0.3 ml/kg/hr fRef: Principles of pediatric and neonatal emergencies by panna Choudhury,
Arvind Bagga, Krishna Chugh, Siddharth Ramji 3dle p, 158)
o In children Acute renal failure is defined as either one or both of the following :-
i) Tripling of baseline serum creatinine.
ii) Urine output less than 0.3 ml/kg/hr for 24hrs.

64. Ans. is'a' i.e., Urinary sodium > 40 m.eq/L lRef Nelson l9th/e p. 22071
o Dehydration causes pre-renal azotemia.
r In pre-renal azotemia, urinary sodium is less than 20 m.eqlL.
65. Ans. is t' i.e., Severity of renal failure flnternet referencel
o Schwartz formula (for creatine clearance in child)
Creatinine clearance = K x height/creatinine
K = constant
r K depends upon -) Age, Muscle mass, Method of creatinine estimation

MISCELLANEOUS

BARTTER SYNDROME

66. Ans. is'a'i.e., Hyperkalemic alkalosis fRef: Nelsan l}th/e ch. S31l
r There is Hlpokalemia with metabolic alkalosis (not hlperkalemic metabolic alkalosis). Other options are correct.
67. Ans. is t' i.e., Bartter syndrome lRef: Nelson 18th/e p. 2201, 2202; O.P. Ghai 8*le p. 501 6 Zh/ep. 4691
Clinical features of the infant -+ Polyuria, Growth retardation, Medullary Nephrocalcinosis
Electrolyte abnormalities: -
t Decreased potassium (Normal j.5 - 5.0 meq/L) t Normal Bicarbonate (Normal 21-30 meq/L)
t Decreased sodium (I'{ormal 136 - 145 meq/L) t Increased pH (Normal 7.38 - 2.44 meq/L)
t Decreased Chloride (Normal 98 - 106 meq/L)
C/F and electrolyte abnormalities in Bartter's syndrome
t Hypokalemia t Polyurea and Nocturea (d/t hypokalemia)
t Metabolic alkalosis t Increased urinary chloride (cause Hypochloremia)
t Normal to low blood pressure t Hypomagnesemia (seen in minority of patient)
: Growth retardation . Hypercalciurea (causes nephrocalcinosis which is visible on ultrasound)
:r.rr"rr,,r.,l.i.::.:rli1,:.r:. r'r.l:r.tr....,. .1I......i'
iit:itr-.,_ltlt::r'i.',:i ::itlr:rr.t.r.rr:aa :'.:.::.:.: a :. . ..

;li1il.ttl.,.t:;:i..,1.::r:
1r*j-i::i:rrii.,:.i.rr1*:::ri:ii:j!1iiii;:t:ii:r;,:rj:rit?a::::,::....t',a:t|a:,.t:.:a:aa:.,::.:.,,1t!].r':f:,.ti.,-.:,....'
::. :.:.,::.:,..:
:' t:,
:..:.,: :.:..::

77. Ans. is'b'i.e., Membranous GN [Ry': Harison lVh/e p. 1g15)


o Nephrotic syndrome accompanying membranous glomerulopathy
and certain carcinomas are the most common cause
ofRVT.
Remember
o Most common cause of RVT in adult ) nephrotic syndrome due to
membranous glomerulonephritis.
o Most common cause of RVT in children --> nephrotic syndrome due to
minimal ciang" diseasi.
o Most common cause of RVT in newborns and infonts
-+ Asphyxia, dehydration, shoik and sepsis.
78. Ans. is 'd' i.e., ARPKD {Ref : Clinical pediatric Nephrology 2d/e p. 49)
Nephrocalcinosis
o Deposition of calcium in the substance of the kidney.

Causes of nephrocalcinosis
r Idiopathic hypercalciuria r Hlperoxaluria
o Distal renal tubular acidosis o Diuretics for preterm baby
o Bartter syndrome r Dent's disease
o Primary hlperparathl,roidism o Furosemide use

ANSWERS OF VARIOUS OTHER EXAMINATIONS

79. Ans. is 'd' i.e., 600-700 mosm/Litre lRef : o.p Ghai Sth/e p. 467 6 vh/e p. 4411
o A full term infant can concentrate his urine to a maximum of 700-g00
mosm/kg.
o An older child can concentrate 1200-1400 mOsm/kg.
o A newborn can dilute his urine to a minimum of 50 mOsm/kg much like
an older child.

80. Ans. is t'i.e., Genitourinary lRef : Has been explainedl


81. Ans. is'a'i.e., Proteinuria lRef : o.p. Ghai 8th/e p. 472 d, vh/e p. 450; Nelsan lgth/e p.
29al
o All the given options occur in nephrotic slmdrome, but proteinuria
is the most characteristic, which is the basic
defect.
o The underlying abnormality in nephrotic syndrome is an increase in permeability
of the glomerular capillary wall,
which leads to massive proteinuria and hypoalbuminemia.

82. Ans. is t' i.e., Hypoalbuminemia lRef : o.p. Ghai gth/e p. 477 d2 vh/e p. 4501
83. Ans. is'a'i.e., Spontaneous bacterial peritonitis [Ref : O.P. Ghai'yth/e p. 477-479
6 Vh/e p. 453;Nelsan lgth/e
p. 21esl
o Spontaneous bacterial peritonitis is the most frequent type of infection.

84. Ans. is 'a' i.e., Lipoid nephrosis lRef : O.P. Ghai p. 477 d2 Vh/e p.
Sthle 451; Nelson \gthle p. 21921
o Minimal change GN is also referred to as lipoid nephrosis.
85. Ans. is'a' i.e., Hlpokalemia
r It is a case of acute nephritic syndrome which can result in ARF.
o In ARF there is hyperkalemia (not hlpokalemia) and acidosis.
o Acute nephritic syndrome is also characterizedby hlpertension, So hlpertensive
encephalopathy may also occur.
86. Ans. is 'b'i.e., Posterior urethral valve [Ref : Nelsan lgthle p. 2241]
o The informations provided in this question are _

i) Straining -+ Sign ofobstruction


ii) Driblling -+ Sign of obstruction and incomprete bradder emptying.
iii) UTI -) May be due to urinary obstruction.
o All these suggest the diagnosis of obstructive uropathy, and posterior
urethral valve is most common cause of
obstructive uropathy.

87. Ans. is'a'i.e., Dilatation of posterior urethra lRef : Nelson lgth/e p. 22a2 fig (54a141
Micturating cystourethrogram findings
i) Dilation of prostatic urethra
ii) Transverse linear
flling defect corresponding to the valves.

--l'ns' is'a'i.e., Renal vein thrombosis [Rel Nerson tgth/e p. 21si-21g4)

rrt
ilENTRAt NHEAV#LI h
{nqr{1 r-H- g: & 6
&X}AM1VA

SEIZURE ANSEPILEESY

Seizures can be classified into :-


t) Generalizedseizures
r tonic-clonic seizures, aitd
These are absence seizures, myoclonic seizures, clonic seizures, tonic seizures, generalized
atonic seizures.
2)
:"fi::::::li\pt, porriot seizure and comptex partial seizure (temporattobe epiiepsy). carbamazepine is the d,rg
e')
of choice for partial seizures(At .

FEBRILE SEIZURES
seizures are defined as seizures
r Febrile conr,,,lsions are the commonest cause of seizures during early childhood. Febriie
which occur during fever. Mostly occurs between 6 months to 5 yearsAuttssz).
c Theconuulsionsare notrelatedtoilegreeoftempraturel;se(AttMSeT)bvtarefrequentiftempraturerisesabruptlvabove
39" c.
e Spontaneous remmissionLilMseT) occurs with ro pastictal neurological deficit and EEG changes few days after
tbe

seizure is normal.
e Recurrentfebrile seizutes occur in 30-50% of cases('{i13"Arr-Mse7)'

r More than 90% of febrile seizures are generalized'


o Acute respiratory illness are most commonly associated \'vith febrile seizures'

Clinical features
* There are following tlpes of febriie convulsions :-
1) Simplebenign
Central Nervous System

IIt is more common. It lasts lbr less than 70 minutes@Gl00) and.is generalized in nature.
There is usuaiiy singl;
seizure in same day. No post-ictal neurologicat deficit occurs@Gt 0u
.

2) Complex (atypical)
I It lasts for more than 15 minutes and is focal in nature. More than one seizure
occur in same dav.
r Risk factors for recurrence offebrile seizures are :-
i) Positive family history@NB ls, At la, PGt 00) iv) Early onset (< I vear)(;r tot

ii) Atypical (camplex) seizure(Al10) v) >1 seizure in same dav


iii) Focal features vi)
Neurodevelopment retardation (developmental delay)ter nt
c Children with febriie convulsions are at increased risk of developing epilepsyin future.
Itmay occur in 2-7% af caseslt
i3).

Treatment
r Prompt reduction of temperatureby hydrotherapy (sponging) and antipvretics (paracetamole,
ibuprofen) is the
most important measure. If seizures lasts for more than 5 minutes, diazepa:nr. {r.ectal
or IV) is the anticonvulsant oi
clnoice(At 8-4). Phenobarbitone is an alternative.
I For prophylaxis (prevention of recurrence) of febrile seizures, intermittent prophylaxis
is used and the drug ot
choice is oral (or rectal) diazepam{D'vr 13'AttMS01'e6'PGI00) or other benzodiazepenes.
in faiied iliierr;riltent prophviaxis.
continuous prophylaxis is indicated by sodium valproate or phenobarbitone. carbamazepine
and phenytoin are
ineffectivefor prophylarcis[arse]. x'leasures to lower temperature like sponging and antipyretics
{paracetamal) have
no rale in prophylaxis(DNB 13' nrMs 1' e6' PGI 0a) .
0

r$YSCLONIC EPITEPSY

* It is characterized by brief rnuscular contractions with loss oJ bocty tone.Itmay be : (i) Infantile spasm; and (ii) )uvenile
myoclonic epilepsy.

Iafantile spasm (Infantile myoclonus or 1{est syndrorne)


o The onset is usually between 3-B months of life
r it is characterizedby triad of-
i) Salaam spells (sudden droping of the head &
Jlexion of arms)
ii)
Mental retardation
iii) Hypsanhythmic pattern of EEG (diffuse high voltage slow spike and chaotic activity)
o The spasm occur in clusters usually on waking.
r Prognosis for normal mentai development is poor.
* Vigabatrin (drug of choiceul B) ), ACTH (2d choice)Gl ee)
and. cor.ticasteroids are used for treatment.

The given EEG i* secn in which type of epilepsy -

a) Febrile seizure
b) Myoclonic epilepsy
c) Absence seizure
d) Grand-mal epilepsy

Aap, is'bu i.e., Myoclonic epilep*y


r The EEG in picture is showing abnormal slow chaotic background \^/ith multifocal
spikes suggestive
hysparrhythmia ) characteristic of myoclonic epilepsy.

Juvenile nnyoclonic epilepsy


r It begins in early adolescence (12-16 years) and is characterized bybilateral rnyoclonrr(rcros); which occtr mostfreque*t!.

T
in morninglAltMs ou) after awakening. It can be provoked by sleep deprivation.Consciousness is usually preserved.
o It may be associated with generalized tonic clonic seizures(Pcl0s) and absence seizure (only in 7/3td cases(AilMs06)).
o It isbenign condition, although complete remission is uncommonqilMso6).
a
o EEG shows a 4-6 Hz irregular spike and wave discharges (fast spike).
e Family history of epilepsy is positive in most cases.
o Seizure respondswell('urusoc) 1o appropriate anticonvulsant.Valpraateis drugchoice(DNB13'A110'PGI07'05) aldis required
r0),
for lifelonglretoz'os). Other effective drugs are topiramate(Al lamotrigine, felbamate, clonazepam and zonisamide.

NEONATAL SEIZURES

a The most prominent feature of neurological dysfunction in neonatal period are seizures.
a Subtle seizures are the most commofl typs(uoo'ut*s07) and are characterizedby movement of upper limb ('boxing]
'hooking', 'swimrning') and lower limb ('bicycling'); sucking;lip smacking; and apnea. Other types of neonatal seizur*s
are focal, multifocal, myocionic and tonic.
a Focal seizures have better prognosis@Na t3) (afier subtle seizures which carries best prognosis).
a Causes ofneonatal seizures are :-
1) Hypoxic ischemic encephalopathy -) Most common cause (60oTolNnnzenusot:t .

ii) Intracranialhemorrhage
iii) Hlpoglycemia
iv) Hyponatremia(Pclog)
v) Pyridoxine deficiency{Attnsoe)
vi) Drug withdrawl
vii) Benign familial neonatal seizures
viii) Infection
ix) Hlpocalcemia, Hypomagnesemia
x) Disorder of amino and organic acid metabolism, hyperammonemia
xi) Developmental defects
xii) No cause found (idiopathic)

,.r Sel;zur*s dr.reto hypoxi€ iich:eniceneephqldpathy &irry worst progn.asii.


t ldiopathic seizure has bad ptrognosis.
t Seizure due to hypocalcemia has better prognosisiAt i3i
.
o Focal seizures have better prognosis.

r Drug of choice for neonatal seizures is phenobarbitonealrMseT). Phenytoin is an alternative.

STATUS EPILEPTICUS
e It is defined as a condition in which the duration of one seizure is> i0 minutes or multiple seizures occur in 30 minutes
without gaining consciousness between seizuie episodes.
o Important causes are meningitis/encephalitis, head trauma, idiopathic, developmental neurological abnormaiities,
hyponatremia@Gl08), hypoglycemia, pyridoxin deficiency and drug intoxication.
o Initialdrugofchoicefortreatmentisivlora.zeparn@EEr'Alee)followedbyfosphenytoin(pheq.toin) andphenoiralhrl-orie.

ABSENCE SEIZURES (PETITMAL SEIZURES(IIi'S98]} . -


e Absence seizures are characterized by sudilen brief lapse of consciousness without lass of postwrai eavatr,:t!. Tlt*
seizure typically lasts for only seconds, consciousness returns as suddenly as it was lost **d there 6s mo;rcsf iefad
oe).
confusionAt
o It begins in childhood (4-8 years) ar 0') or early adolescence and are the main seizure tipe in children with eprlepsy.
o There is sudden briefloss ofconsciousness, which lasts for a few seconds.
r No loss of posture
t No incontinance of urine or stool
t I'lo breathing dfficulty
r It may be accompanied by subtle bilateral motor signs
such as
t Rapid blinking of eyelids
t Chewing movements
t Small amplitude clonic movements of the hands
r Absence seizures may occur in murtiple every day (can
occur hundreds of times per day).
I The electrophysiological hallmark of typical absence
seizure is a generalized symmetric 3 Hz s p,ike@r0a,ArrMs87)
and
slow wave discharge that begins and ends suddenly on
a normar EEG background.
t Hyperventilation often precipitates the attacksLr 04).
t Drug of choice for absence seizures is Valproate(Al ss) (Previously
it was ethosuxamide)(Ar t3, e4).other drugs used are
lamotrigine and clonazepamLl e4) .

DQC far generalizedtanic clonic.seizure (6rand mdt), abr.n.. ,ai..ri"ffiffi11*,, 4,


Atanic
seizu re, myocl sn ic epilepsy -+ ValPro6lgtri ':t.
e DOC for partial seizure *+ CarbamaZepine / oxtarbazepinetDBl rs,A! sn
r DOCforstatusepllepticus. -+ lV lorazePam!^rErD '

o DOC for febrile seizures -+ RectolDiazepam


c DOC for seizure of eclampsia -+ Magnesium sulfate.
o DOC for infantile spasm -+ Vigabatrinrq';t.
I DO(formyqclonicepilepsy -+ ValProicacidulesl.

CNS INFECTIONS

r cNS infections are meningitis (infection of meninges), encepharitis (infection


of brain parenchyma),
meningoencephalitis (infection ofboth), andbrain abscess.
causative agents maybeviruses (most common), bacterial
(2"d most common), fungus and parasites.

4!gTE BACTERTAL MENINGtTts

o Acute bacterial meningitis (pyogenic meningitis)


is caused most commonly by three organis m : streptococcus,
pneumoniae' N' meningitidis, and H. influenzae. However,
the causative agents are different for different age
:-
t Neonates:GroupBstreptococcus(Str.agalactiae)ismostcommon@cro7,es,s8,Ar05,o4,ArrMs04);E.coli2ndmostcommonecr
07)
,

s 2 months to 3 years : Pneumococci\I04,01'es,ArIMsse) > Meningococciur04) > H. influenza@EEr,Ar04).


CH:aPrtR"8
* 3 year to 20 years : MeningococciiAl t3).

t > 20 years: Pneurnococci.

e Following risk factors predispose to infection by specific organisms :-


e Splenectomy and CSF leak: S. pneumoniee(NEEr).
r Occult bacteremia: N. meningitidis.
; Myelomeningocele and CSF shunt placement: Coagulase negative staphyiococci.

Clinica! features of meningitis in children


c The onset is acute and the symptoms are :-
r Fever r Shrill cry r Photophobia I Ptcsi s

r Irritability r Bulging fontanel r Neck rigitSiti, I i;:u i:,:


: Headache r Seizures r Generalized hl,pertonia
r Projectile vomiting r Altered sensorium r Dipiopia

Signs
t Kernig's sign(,"'nt) -) Extension ofknee is resiricted to less thae I350, when tiie i:,j'o is in 9*i' ilrxqi.il ;r,,-i;
c Brudzinski sign -) On flexing the Neck, there ls flexion of the hips ancl krees.
a Tachy cerebrale -) If skin of the abdomen is lightlv scratched, flushing ilay b* seen.

The followi*S test in ecute nneningitis is known as -


i:l
a) Kernig sign
b) Brudzinski sign %.:,\-b
frffi "-'
i
c) Oppenheim sign
d) None

Ans. is h'i.e., Kernig sign


r In acute meningitis, when hip is flexed to 90o, extension of knee is resisted beyond 120- 130' ) Kernigs -eiqr:

The following test in acute meningitis is known as -

a) Kernig sign
b) Brudzinski sign
c) Oppenheim sign
d) None

Ans. is'H i.e., Brudzinski sign


o In acute meningitis, passive flexion of neck causes flexion ol hip and knee + Brudzinski sign.

Cornplications of meningitis in children


* Seizure is the commonest complication of meningitis in childrenailMs e{).It occurs in about 30% of patients.
e Other common complications of meningitis.
1) Subdural empyema or effusions - It is commonly seen after meningitis with H. Influenza.
2) Brain Abscesses
3) Neurological impairment -+ Hemiplegia, Aphasia, Mentol retardation, blinciness.
4) Auditory impairment (sensorineural deafness(Pct t0)) -+ Especially afier harmJluenzae infactar(DNs t:' Ai se, Arnls ei)
5) Hydrocephalus
6) \rentriculitis, Arachnoidits@Gl rc)

7) Cranial nerve palsies


8) Thrambosis of dural yenous sinuses
9)'Iransyersemyelitis
la) systemic complicatian - shock, Myocarclitis, status epilepticus, syndrome
of inappropriate ADH secretion
o sensarineural hearingloss occurs in 5-10% of patient in H. influenza
meningitis. Thus, ABER (auditory brainstertt
evokedresponse), also calledbrainstem evokedfeslronse audiometry (BERA)
is advised@NB13,Atee,ArrMSs7).

CSF findings
o Study of CSF shows :-
i) Markedly increased leukoclttes (neutrophits/polymorphonuclear leukocytes predominate)alus se)
ii) Markedly increased protein(AllMsee)
lii) Markedly decreased sugar\IlMs ss)

iv) Decreased chlorideullt'ts ee)

v) Increased CSF C-reactive prateinal sl)

Bocterial Elevated PMNs Raised Decreased Low


Vircl NormatAlightly elevated Lymphoryte5 .,r.Baised..r',,' ..Nnfmal.,,.tj',.. ., Nor,mal
Tubercular Elevated Lymphocytes Normal/ Decreased Very low
slightly raised
Fungal Elevated Lymphocylqs. ,:,,' Ndrmdl l '-:,.r:,.
'
,"Orn rnn,,,:,".':::rNij*1,hdl
slightly mised' '',-

a Normal values of CSF finding in a children are :

i) Pressure -+ 50-80 mm HrOrs'ts) iii) Protein -+ 20-40 mg/dl


ii) Leucocytes -+ < 5 or > 750lo lymphocytes iv) Glucose -+ > 50 mg/dl (2/3of serum glucose)

Neonatal nneninEitis
a GroupBstreptococcus(streptococcusagalactiae)isthemostcommoncauseofneonatalmeningitisoetoz,st,es,Ar0s,Arrlts
o'),
followed by E. coli as the 2d most comnnon cause@Gr 07, Ar 0s).
Other causative agents are Listeria monocytogenes(rl
0s),
klebsiellaecr0s), staphylococcus aureus and coagulase negative staphylococcus.
I Bacterial meningitis in the newborn and the first 4 to 6 months of life has
many aqpical features -
i) Neck rigidity and Kernig's sign are seldom prominent(orissass).
ii) Anteriorfontanel may of may not be bulginglorissass).
o Following arouse the suspicion of bacterial meningitis in a newborn -
i) Vacant stare vi) Poor cry
ii) Refusal to suck (Poor feeding)(orissa eB) vii) Fever or hypothermia
iii) Alternate irritability and drowsiness viii) Shock or circuiatory collapse
iv) Persistent vomiting with fever ix) Tremor or convulsions
v) Poor tone x) Neurological deficits of varying tlpes
Treatment of bacterial rneningitis
o For empericai therapy 3'1 generation cephalosporin (ceftriaxone or
cefotaxime) and vancomycin are used. After getting
the culture and sensitivity report, specific antibiotic is started :_
i) N' Meningitidis:Thirdgeneration cephalosporin (Ceftriaxone@Gt07) ar Cefotaxime@Gr07))
is the DoC. penicillin
G can be used in penicillin serusitive strainsGIIMS02).
ii) H. inJluenza : Third generatian cephalosparins(Al04) (Ceftriaoxone or cefotaxine).
rii) Streptococcus agalactiae : penicillin G or ampicillin.
l, ^, ; ; ;1' * i ; i ;;i;;*i;it
i:,

iti) Streptococcus pneumoniae : Penicillin G in sensitive strains or 3'd generation cephalosporin plus
t') Staphylococcu.s : Nafcillin for methiciliin sensitive strains and vancomycin for MRSA.
"""."-f.tr.
vi) Listeria: Ampicillin plus gentamycin.
vii) Pseudomonas, E. coli and klebsiella: Ceftazidim plus aminoglycoside.

- J BERCULAR
'WEN'NGlTI5

o Tuberculous meningitis usually arises from the formation of metastatic caseous lesions in the cerebral cortex or
meningitis that develops during the lymphohaematogenous spread of primary infection. This initial lesion increases
ln size and discharges tubercle bacilli in the subarachnoid space.
o This results in gelatinous basal exudateGl 0s' es' DPG 0e) which interferes in the normal flow of CSF, in and out of the
ventricular system at the level of basal cisterns leading to a communicating hydrocephalusLl 0s' ss, DPG 0e)
.

Clinical manifestations of tuberculous rneningitis


o The clinical course of TB meningitis has three stages -
1. Prodromal stage or stage of invasion
x Insidious onset x Headache and vomiting
x Low grade fever, loss ofappetite x Photophobia
e Disturbed sleep x Constipation
2. Stage of meningitis
x Signs of meningeal irritation develops - Neck rigidity, Kernig's sign
3. Stage of coma
x Loss ofconsiousness x Bradycardia
x Pupils are dilated & fixed x Chyne-strokes or Biot type ofrespiration
x Episodic decerebration

CSF in TB menimgitis
a LOW SU*AfGI 07' e4' AilMS e8)

a High Proteinal oT' e1' AtIMs e8)


a Predominant cells -+ LymphocytesutqT'ea)
a VerY low chlorideailMs e8)

a Opening pressure is elevated

Complications of TB menlngitis
o 'lherr. rnavbe i&farcts(nre') in ihe br':iir: ririr,.to v:.:r,-iJar oacir.i:;iur
o 1,'irious neuroiogical *;rnifustaiions see n ro;rtir f il:vi iitcrr_rrie :
x Hemiplegia (20o/o), qtadriplegia (19%), monoplegia (3Zo)
* Cranial nerve palsies (14%1r",ut
x Hemiballismus (117o)
x Tremors (6.1%)
x Midline cerebellar syndromes (4%)
a Ptosis(Ale7) & ophthalmoplegia
x Decerebrate Rigidity (3%)
x Decorticate Rigidity (3%o)

x Cerebellar Hemispheric Lesions (1.0%)

v' RAL nnnnxtN&0EzueEph{ALt?'s


* Viruses are the most common cause of infection. There is some degree of involvement of brain parenchyma along with
meninges, i.e. meningoencephalitis.
* Enteroviruses are the most common cause(At' r' I0). Other viruses causing meningoencephalitis are HSlz I d, 2, LIHI/- 6,
VZV andmumps.
r CSF examination in viral meningoencephalitis shows
lymphocytes, normal/mild elevated csFpressure,
normal/slightly
raised protein and normal glucose.
r rrsvusually affects temporal lob ealss) andcauses focal seizures. protein
levels in csF may be very high (usually
in viral meningoencephalitis csF protein level is normal
or slightly increased). HSV-I affects children and
and causes focal involvement (unlike other viral infections). adults,
HSV-2 affects neonates(AnMs02) and causes difftrse
involvement.
t YZY causes cerebellar ataxia.
r Mumps causes very mild disease and hearing loss(NnxD can be seen due gth
to nerve damage. Unlike other viral
infections, CSF glucose is low.
r HHV-6 mainly affects immunocompromized patients.

BRAIN TUMORS

o Brain tumors are the second most common (A100)


malignancy of childhood next only to leukemia.
Brain tumors are the
most common solid cancer of childhood . over two-thirds
of brain tumors are infratentoterial@r [u).Most
tumor occur near the midline of these
-> commonly obstruct CSF circulation and cause hydrocephal^st.nootearly in disease.
These can also cause t tct and' papilledemaaloo) (In
infonts papilledema may be absent because of open sutures and
bulgingfontanelle).
r There is age related differences in primary location
of tumor :_
1) Infant (l't year of life) -+
Mostly supratentorial, most common are choroidal
plexus tumor and teratoma.
2) 1-10 years -+ Mostly infratentorial, most common qre medulloblastoma
and cerebellar
astrocytoma (juvenile pilocytic astrocytoma).
3) After 10 years -+ Mostly supratentorial, most common is difuse astrocytoma.
r Most common intracranial (brain) tumor in children
-) Cetebellar astrocytoma (gliomaltNnar,er ut,
c Second most common intraranial (brain) tumor in children
-) Medulloblastoma.
t Most common infratentorial tumor in children
-) Cerebellar astrocytoma (glioma).
c Second most common infratentorial tumor in children -+ Medulloblastoma.
t Most common supratentorial tumor in children
-) Craniopharyngioma.
t Most common posterior cranialfossa tumor in children
-) Cerebellar astrocytoma^len) (glioma).
: Second most common posterior cranialfossa tumor in children _) MedulloblastomaAree)
t Other common posterior craniattfossa tumor in children -) Ependynomaruss),brainstem glioma.

t' suprutentorral reg,ion of the brain is rocated above the tentorium


cereberi; and contains the cerebrum.
r The infrutentorial'region of the brain is located betow
the tentorium cerebelli and contains the derebellum and
brainstem. ":

* BrEin stem and ibrebeilum are contained in the posferior craniarfossa.


r'As-brainstemandcerebellumareinfratentorial inlocationandcontainedinposteriorcranialfossa-+Theirtumorsare
infratentorial posterior fossa turhors, e.g., cerebellar astrocytoma,
brainstem glioma, medulloblastoma (cerebellar).

Sign and symptoms of Brain tumors in childran


I children with brain tumors frequently have symptoms and signs that strongly
suggest the appropriate diagnosis. The
clinical ttiad of symptoms associated with increased ICT -+
headache, vomiting, blurreil vision / diplopia, is
the
hallmark of brain tumors in childhoodullMs 02). These symptoms
occur more frequently with mid-line posterior
(infratentorial) tumors owing to the propensity of lesions fossa
close to the ventricular csF flow system.
r Symptoms of the brain tumors vary depending upon which part of the
brain the tumor is found :-
a$:htsa'pollelior foisa tumors. but they do not'prese nt wiih increased I

gliorna has.vrpast Bro$osis'

Astrocytomas
o These are the most common brain tumors in children. They are usually located in posterior cranial fossa@rrs) and
most common site involved is cerebellum.
o Astrocytomas are divided into :-
l) Low grade astrocytomas (juvenile pilocytic astrocytoma, fibrillary infiltrating astrocytoma)
ii) High grade astrocytomas (glioblastoma multiforme, anaplastic astrocytomas).
o Mostofthechildhoodtumorsarelowgradeastrocytomas(aror)(especiallyjuvenilepilocyticastrocltomaldtts))and
occur infirst two decades6ros).
e Most common site is cerebellum, i.e. cerebellar astrocltoma.
o There is no sex predilectionc4ro8) and the prognosis is very good(Aroa).

Medultoblastoma
o Medulloblastoma is the most common PNET (primitive neuroectodermal tumoT)(etesl located in posterior cranial
fossa.
o It is the 2d most common infratentorial posterior fossa tumorates) (after cerebellar astrocytoma).
o It arises from midline cerebellar vermis thus presents as midline cerebellar swellingleuussat.It may extend into 4e
ventricle causing hydrocephalus.
o Males more commonly affected.
are
o Histopathology shows Homer Wright rosettes and positive reaction for synaptophysin.
o 17p deletion is associated with medulloblastoma.
o Chang staging is used for prognosis.

Craniopharyngeoma
c Craniophyngioma is the mos{Vommon supratentorial tumor in children^r B)
, lt is a suprasellar tumor\ItMs e5)
which
presents as cystic massarrMs es). AI eT)
CalcificationaltMses, is common.
o It arises from squarntrus epithelial cells crests of embryonic rathke\ pouch. ln children they present as -) visual loss,
growth failure, Bitemporal hemianopsiaailMs 04) .
o Craniopharyngeomq is the most common cause of suprasellar calcificationar eT, artMs ea) . Calcification is characteristic
midline and immediately above cells.
ENCEPHALOPATHY

r Encephalopathy are disorder of brain parenchyma, which may be static (cerebellar palsy) or progressive (e.9.
mitochondrial encephalopathy).tt can be caused by infections, toxins, hypoxia (perinatal) and metabolic disorders.
Microcephaly is commonly associated with encephalopathies.

CEREBRAL PALSY (CP}

o Cerebral palsy is defined as 'Static (non progressive)' 'Neuromotor disorder' of 'Cerebral origin'. CP is due to an
insult of developing brain which results in maldevelopment and disorderly anatomic organization of the brain. CP
may be classified into :-
A) Spastic CP
s It is the most commn type (65%) of Cpoer wt. There is spasticity(Pcr 0') and increased tendon reflexes. Mental
retardation and seizures may occur. It is divided into :-
1) Spastic hemiplegia
o It is due to large vessel (MCA ataxia) stroke. Spasticity is seen on one side ofbody arms>legs>face@NB
15)
.

r Seizures (in 1/3'd) and mental retardation (in 1/4'h) may occur.

2) Spastic diplegia
o It is due to premature birth. There is bilateral spasticity legs > arms@NB'5) resulting lagging of legs while
crawling (commando crawling).
r It is least commonly associated with seizures and mental retardation.
o Periventricular leukomalacia is characteristicallMs 0e).
3) Spastic quadriplegia
. It is due to birth asphyxia.There is spastic weakness of all 4 hmbs arms ) legs@Nn ts).
o It is most commonly associated with seizures (in 50%) and mental retardation@NB ").
r Scoliosis may occurutusez).
o It is the most severe type of CP@NB
1s).

B) Ataxic CP
r It is the secondmost commontype. There is ataxi"{rcrra), h14>otonia and nystagmus. Forebrain is more commonly
involved than cerebellum.
:
It is commonly associated with other form of CP, e.g. spastic hemiplegia (mixed CP(PGr05)).

C) Hypotonic (atonic) CP
r There is marked hypotonia(Pc' to (floppy child or Jlaccid paralysis(Pcr oa)), normal or brisk tendon reflexes and
extensor Babinski response.
r Severe mental retardation is there.
r Forster sign (flexion ofhip and leg on holding the baby verticle) is positive.
D) Extrapyramidal (or athetoid) CP
: There is dyskinesia such as athetosis(Pct0s), dystonia, choreiform movements, tremors and rigidity@Gr05).
r Mental retardation, seizures and hearing defect may be seen.
r Extrapyramidal CP due to peripartum total asphlaria{om r:r ir associated wi.th bilateral symmetrical lesions@NB
13)
in posterior putamen and ventrolateral thalamus, and appear to be carrelate of neuropathologic lesion called
status marmoratus(DNB'3) in basal ganglia. These lesions have marbled appearan6s@Nrr:).
E) Mixed type
r There is diffuse neurological involvement of mixed Qp e@Gt
0s)
.

M ITOCHONDRIAL ENCEPHALOPATHIES

o These are a group of disorders caused by mitochondrial DNA mutation. The various disorders are htlELAS (mitochondrial
encephalopatlry,lactic acidosis and stroke), MER&F (myoclonic epilepsy with ragged redfibre), Leigh subacute necrotizing
encephalopathy, LHON (Leber Hereditary Optic Neuropathy), kearns-sayre syndrome (KSS), and NA RP (Neurogenic
0e).
weakness with ataxia and retinitis Ttigmentosa)@IlMs Mitochondrial disorders are always maternally inherited.
One important acquired mitochondrial encephalopathy is Reye's syndrome.

Reye's syndrome
o Reye's syndrome is a metabolic mitochondrial disorder characterized by:-
r) Fatty infltration ofliver
ii) Encephalopathy(Pclos)
lii) Sometimesfatty infiltration of kidney.
o So, it so also known as Encephalopathy withfatty degeneration of viscera.
o It is an acute self limiting metabolic insult resulting in generalized mitochondrial dysfunction due to inhibition of
fatty acid oxidation. Precipitating factors -
t Viralinfection@Gro3,uPs7)) Influenza B * S{xeratass),Yaricella (Herpes tlpe 3(Kerotaes)),Echovirus-2, Adenovirus(("'"/'
rsl, EBV Coxsackievirus A
: S alicylates (Aspirin{Nnnr' ur sz ;1

Clinical manifestations of Reye's syndrome


r There is hepatic encephalopathy?et os1
and brain edema(Pcl 0s)
. There are five stages for clinical pictures -
1) Stage I -) Quiet, lethargic, sleepy, vomiting, anorexia, apathy.
2) Stage II -) Deep lethargy, delirium, restlessness, agitated, hyperventilation.
3) Stage III -) Light coma, decorticate rigidity.
4) Stage IV -) Seizures, deepening coma, decerebrate rigidity
5) Stage V -) Deep coma, flaccidity, loss of reflexes, respiratory arrest"

t laundice is infrequent in Reye's syndrome -) Bilitubin is not raised(Pct 03' Iipmerel).

Laboratory findings of Reye's syndrome


t Hypoglycemia@Gt 83' 7e)

. Hypoglycorrboea ) tr, CSf glucose@Gr83'7e)


a f pT(uPeT,lipnetel)

t Increased ALT and AST (> 3 times)(uosc86'pGI83'7e)


t Normal alkaline phosphatase
s Normal bilifubinqt ls, PGt 03,lipner el)
c Hyperammonemia
86)
c Mixeil respiratory alkalosis and metabolic dcidosis(upsc
r On liver biopsy -
s Microvesicular fatty changes@et oa' B' Iipnet el)
s Absence of inflammation(tipner e1)
t Depleted glycogen stores
r Kidney
t Swelling and fatty degenration of PCT.

HYDRAT{ENCEPHALY

r It is a rare encephalopathy that occurs in-utero.


r It is characterised by destruction of the cerebral hemispheres with transformation of the same into a membranous
sac containing cerebrospinal fluid and the remnants of cortex and white matter The midbrain and brainstem are
.

relatively intact.
o The cause ofhydranencephaly is unknown, but bilateral occlusion ofthe internal carotid arteries during early fetal
development would explain most of the pathologic abnormalities.
a Affected infants may have a normal or enlarged head circumference at birth that grows at an excessive rate
PostnatallY(errs).
a Transillumination shows an absence of the cerebral hemispheres(Arr5).
a The child is irritable, feeds poorly, develops seizures and spastic quadriparesis, and has little or no cognitive development.
a A ventriculoperitoneal shunt prevents massive enlargement of the cranium.
a Porencephaly is considered a less severe degree ofthe same pathology.

DEVELOPMENTAL DEFECTS

I{EURALTUBE DEFECT

I NTDs account for most congenital anomalies in CNS. It results from failure of neural tube to close spontaneously
between 3'd-4th week of in utero development.
o NTDs have multiple etiological factors -
r Maternal diabetes
r Maternal exposure to alcohol, radiation, valproate, carbamazepine
t Zinc and folate deficiency
r Trisomy 13 and 15
r Maternal malnutrition is an important risk factor for development of NTD. There is decreased maternal
folate
levels in NTD affected pregnancies -+ Periconceptional folic acid supplementation decreases the occunence of
es).
NTD(AIee,
r NTDs allow excretion of fetal substances like alpha-fetoprotein and acetylcholinesterasear ee)
into amniotic fluid,
as prenatal biochemical markers for diagnosisal ss) .
which are rsed
r Neural tube defect may be divided into :-
A) Spina bifida : Which includes :-
i) Spinabifida occulta
r It is the mildest and most common type. Commonest site is lumbosacral region, S, being the commonest.
There is bifid spinous processes of the vertebrae.
ii) Spinabifiila aperta
r Commonest site is dorso-lumbar region The defects are meningocele, meningomyelocele (myehmeningocele)
(Ar 01),
syringomy elo cele and my elocele.
B) Others:Theseareencephalocele{Aloa),anencephallqlry4),dermalsinus,tetheredcord,diastematomyeliaandlipoma
involving conus medullaris.

Meningocele
o There is protrusion of meninges through a defect in neural arch. This contains only CSF. There may be associated
genital tract abnormalities, e.g. vaginal septa and rectovaginal fistula.
'eneprrn 8 CentrclNervous Systefi

Myeloxx**titt gseete gcntyeloe*le}


{n:*er* iN
r There is protrusion of meninges along with some neural eiements, e,g. spinal cord or cauda eilliina. ii is ihe mosf
site of rlefect:-
severe form. The commonest site of lesion is lumbosacral region. Clinical presentation depends tipor]
esr
i) Sacral myelomeningocele : Bladder and bowel incantinenceirr \{lth perianal anaesthesiir.
ii) Mid-lumbar myelomeningocele: Flaccid paralysis of lower iitnbs(o'e'),urinary dribbling,loxed ancl strthindersint
and lower limb deiormities, e.g. ciub tbot, subluxationr'dislocation of hip etr-:.
e5)

* Hydrocephalus is seen commonly(Alee) and is associated wlth chiari rualformati*tt tytrte II.
Treatment includes surgical repair of clefect (ivithin 3 days) to prer.,ent infection. lliis is ibllo..t'eil b,v shuirt
* placr:;:tei'i
1i ) q0
for hydrocephalus. If there is cSF leak, csF and blood sample should be send .for culture a*.*! sensitivid,'i'1rr'u's
that perioperative antibiotic can be changed accordir.rg to sensitivity.

Eneepka!oeeIe
* Thereisprotrusionofbraintissue(cerebralcorter,cerebeilumorbrainstem) ttiroi.rgiracranii'rltieiect(craniun:bi:.,'l;i;:)'
Most common site is occipital region" Eiydrocepliaius cail !rccur due to aqueductsi stewosis, chiati tm*lf,eww*'tiait *r

D andy-Walker syndrorne.

&r'lencepl'la!y
* There is failure of develcpment of cerebral i"remispheie and cerebellum. It is associated r',ith cJefrpal*l:e art'l {lI-*Fs it
(r\rrEr L).
is the earliest anomaly which can be detect by antenatal {tsG (at 10-11 weeks} "r-I

&iagaosi.s ix t-[re givex figlrre is -

a) Tuberous sclerosis
b) Spina bifida occulta
c) Meningocele
d) Meningomyelocele

Ans. is'H i.e., Spina biliti* occulla


* Dimple ut 1o*.. back just above the anus is a cutaneous marking of spina bifida occulta

&iagntxie i* tlle givea &gcre is -


a) Tuberous sclerosis
b) Neurofibromatosis
c) Meningocele
d) Meningomyelocele

Ans" is ?'i.e., &{eningoraYel*ce}e


* There is protrusion of meninges and spinal cord, characteristic of meningomyelocel.

&iagnosis in tke give* &gzere is -


:l
lr
a) Tuberous sclerosis
!,
b) Neurofibromatosis
c) Meningocele
d) Meningomyelocele

,&ns. is ?'i.e., &{elaingoee}e


o There is protrusion of meninges containing only cSF ) Meningocele.
ARNOLD-CHIARI MATFORMATION

o Arnold-chiari malformation consists ofherniation of


the cerebellum and four\th ventricle into the foramen magnum,
flattening of the base of the skulr, and spina bifida with meningomyelocele.
r Chiari malformation is divided into :-
i) Tlpe I : Produce symptoms during adolescence or adulthood is usually
not associated with hydrocephalus. The
deformity consists of displacement of cerebellar tonsils into the cervical
canal.
ii) Tlpe II : It is charact erized by progressive hydrocephalus('4r.r5) with a myelomeningocele (Ar 15)
innewborns.
There is a failure of pontine flexure during embryogenensis,
which results in elongation of 4th ventr icte; kinking of
brainstem and breaking of quadrigeminal plate (tectum)(At 1r. along
with displacement of inferior vermis, pons
and medulla into cervical canal. This causes widening of cervical canal (syringomyeliaarrs)).
iii) Type III : usually associated with occipital encephalocele and causes
abundant neurological deficit.
iv) Type IV : characterised by lack of cerebeller development and usually
not compatable with life.

DAN DY.WALKER MALFORMATION

o It is characterized by hlpoplasia or absence of cerebellar vermis GI ls.Arrnsel)


with cystic dilatation of 4n ventricle in
posterior fassa(Ar rt A,Ms es)' There is hydrocephalus(Ar 15 All,s es), agenesis
of corpus callosum and increased head size
with prominent occiput.
o Dandy-walker malformation can be differentiated from congenital
aqueductal stenosis by normal size of posterior
fossa in the later(Ari5 AIIMS02) (posterior fossa volume is increased
in Dandy-walker malformation).

NEURAL MIGRATION DEFECTS

r These are caused by defect in migration of neurons to their proper


site due to defective radial glial fibres which are
required for proper neural migration. The important defects are
:_
l) Lissencephaly(Agyria)
r There is absence ofcerebral convorutions with four layer cerebral
cortex (normally it has six layers), enlarged
lateral ventricles, microcephaly and microophthalmia.
r It may be associated with Miller-Dieker syndrome (characteristic facies : prominent forehead, anteverted
nostrils, micrognathia and prominent upper lip).
2) Schizencephaly
r There is presence of unilateral or bilateral clefts within cerebral
hemisphere.
3) PorencePhaly(MP oo' AIse)

r It is characterized by presence of cysts in brain due to development defect


or acquired infarction or injury@p
oo,AI 99)
.

: Most common site is sylvian fissure. It may be associaed with Dandy-Walker


syndromeue on).

4) Holoprosencephaly{eroat
r It is caused by defective cleavage ofporencephaly.

cRANTOSYNOSTOSTS

o There is early fusion of cranial sutures :


i) Scaphocephaly(Dolichocephaly)
s commonest type and charactetizedby early closure of sagittal sutures with
anteroposterior elongated skull.
ii) Brachycephaly
r Early closure of coronal suture(ArMs e& e7) with anteroposterior flattened
skull.
ARNOLD-CHIARI MALFORMATION

o Arnold-chiari malformation consists of herniation of the


cerebellum and forlth ventricle into the foramen magnum,
flattening of the base of the skull, and spina bifida with meningomyelocele.
r Chiari malformation is divided into :_

'i) Type I : Produce symptoms during adolescence or adulthood is


usually not associated. with hydrocephalus. The
deformity consists of displacement of cerebellar tonsils into the cervical
canal.
ii) Type II : It is characterized by progressive hydrocephalus(Ar 15) with
a myelomeningocele @r rs) innewborns.
There is a failure of pontine flexure during embryogenensis, which
results in elongation of 4,h ven tricle; kinking of
brainstem and breaking of quadrigeminal plate (techta;ro"e, along
with displacement of inferior vermis, pons
and medulla into cervical canal. This causes widening of cervical canal (syringomyelia(Arrs)).
iii) Type III : Usually associated with occipital encephalocele and causes
abundant neurological deficit.
iv) Tlpe IV : characterised by lack of cerebeller development and usually not
compatable with life.

DANDY.WALKER MALFORMATION

r Itischaracterizedbyhypoplasiaorabsenceofcerebellarvermis(,{rrsdrMses)withcysticdilatationof4thventriclein
posterior fassa(Ar '5 ArMs e8). There is hydrocephalus(' r.I,, /,Ms es), agenesis
of corpus callosum and increased head size
with prominent occiput.
o Dandy-walker malformation can be differentiated from congenital
aqueductal stenosis by normal size of posterior
fossa in the later('{rr'AzMs02) (posterior fossa volume is increased
in Dandy-Walker malformation).

NEURAL MIGRATION DEFECTS

r These are caused by defect in migration of neurons to their proper


site due to defective radial glial fibres which are
required for proper neural migration. The important defects are :_
1) Lissencephaly(Agyria)
r There is absence ofcerebral convolutionswith four layer cerebral
cortex (normally it has six layers), enlarged
lateral ventricles, microcephaly and microophthalmia.
r It may be associated with Miller-Dieker syndrome (characteristic facies : prominent forehead, anteverted
nostrils, micrognathia and prominent upper lip).
2) Schizencephaly
r There is presence of unilateral or bilateral clefts within cerebral hemisphere.
3) PorencePhalY(MP oo' AI ss)

r It is characterized by presence of cysts in brain due to development defect


or acquired infarction or injury@p
00, 4t 99)
.

r Most common site is sylvian fissure. It may be associaed with Dandy-Walker syndromeGeo,t).
4) Holoprosencephaly{tt ut
r It is caused by defective cleavage ofporencephaly.

cRANTOSYNOSTOSTS

e There is early fusion ofcranial sutures :

i) Scaphocephaly(Dolichocephaly)
t Commonest type and chatacterizedby early closure of sagittal sutures
with anteroposterior elongated skull.
ii) Brachycephaly
r Early closure of coronal suture(,u,rse& e7)
withanteroposterior flattened skull.
CHAP?ER
iii) Trigonocephaly
r Early closure of metopic suture.
iv) Plagicephaly
r It may be associated with Chotzen syndrome. It is of two types :
a) Frontal plagicephaly is lhe 2d most common craniosynostosis (after scaphocephaiy) and involves early
fusion ofcoronal and sphenofrontal sutures.
b) occipital plagicephaly is characterized by early closure of lambdoid sutures.
v) Turricephaly
r There is cone shaped skull due to early closure of coronal, sphenofrontal and frontoethmoidal
sutures. It is
associated with Ffeffir syndrome.

I.IYDROCEPi.IALUS

c Hydrocephalus is charac terizedby an increased volume of CSF in association with progressive yentricular dilatation
(v entriculomegaly ){ ou n t : t .

o The types ofhydrocephalus are:-


1) Non-comrnunicating (Ar15)
(obstructive) hydrocephalus
r It is the mos t common type of hydrocephalus seen ir.r children and results from obstruction within the ventricular
system.
r It is most commonly caused by aqueductal stenosis(DNB 13, AI s8, e6)
.

r Most common acquired viral cause of aqueductal stenosis is mumps@Nn rc).


I As most common obstruction is at aqueduct level, there is dilatation of lateral ventricles
and third ventricle
with normal 4,h ventricle.
2) Communicating (non-obstructive) hydrocephalus
IIt is due to obliteration of subarachnoid basal cisterns or malfunction of arachnnifl yil1irer rcl,
thus CSF cannot
be absorbed.
I Two most common causes are subarachnoid hemorrhage (most comruon) and postinjlammatory(Ar ss, e6)
due
to meningitis.
r As obstruction is in subarachnoid space, there is enlargernent (dilatation) ofall the ventricular cavities as
well as subarachnoid space(rrts)"
o Investigation of choice for hydrocephalus is MRI(AI15' because it can demonstrate the
extent of hydrocephalus and
uderlying cause.

Signs of Increased intracranial tension in infants


r Separation ofcranial sutures -+ E(trliest sign^Ise).
o Wide bulging anterior fontanelleallMs s7).

t Increased head circumference(AllMs . ez)

t Papilledema does not occur in infants because separation of sutures and open fontanelle compensate for increase
in the intracranial pressure. However, if ICT rises very rapidly papiliedema may occur.
o Macewan or ctacked pot sign occurs after sutures and
fontanelle haye closed (not in neonate)$uMs s7).

o Projectile uomiting
a Headache -+ Particularly in early morning
c Diplopia 6 sixth nerve palsy
o Sun set sign{AttMseT) -+ Eyes deviate dorvnward because of impingement of the dilated suprapineal
recess.
e X-ray shows "beaten silver" appearance of skull due to increased convolutional markilg.
Pseudotumor cerebri
o-Pseudotumor cerebri is characterized'by increased
intracranial pressuretilMssz) inthe absence of an identifiable
intracranial mass or hydrocephalus. cardinal features of pseudotumour
cerebri.
t Headache, Clinically resembling that of brains tumor
t Normal cerebrospial fluid
t Normal or small ventricular sizeLilMssz)
t CT & MkINormalaIIMSez)
t No focal neurological deficisLttus szt
t Transient visual obscurations and papiiledema with enrarged blind spots.

MENTAL RETARDATION

o Mental retardation is cha ructerizedby an rQ < Tg^nus e3)


and, adaptive skills 2sD below the mean The
overall prevalence
of mental retardation is2.5o/o@G106). Most common type
of MR is ailfl p1p{Nnnr) @s%).MR is more colnmon
in males.
o Mental retardation is quantified in terms of Intelligence
quotient (Ie) which is calculated by following formula : -

Mental age
lQ @PG os, Kdata ss,
= x I 00
--------
Chronological age
CrurrEn I
o A child with IQ < 70 is called rnentally retardation [4re3)

(Pct02)
Normal intelligence > 90

Borderline intelligence 70 _ gg (KERALA13)

Mental retardation
(AltMsos,ss)
MiId MR 50 - 69
Moderate MR 35-49
Severe MR 20-34 6GPG.ol)

Profound MR 0-19
(DpG PGI 02)
o An average normal trQ is considered to be IOO . However, the range actually extends from 90- 109.

Causes
o Important causes of mental retardation are :-
A) Maternalfactors: Advanced age,poor socioeconomic status(Pcl06),consanguinity, lbxemia, placental insufficiency,
teratogenic drugs, radiation exposure.
B) Metabolic : Phenylketonurit(Pu ot), galactocemia, hemocystinuria@cl 08), mucopolysaccharidosis, histidinemia,
organic aciduria.
c) Chromosomal disorders : Dowm syndrome(Pct 08' Keatuto e4) (most common genetic cause), fragile X-syndrome@Gt0s)
(2d most comman lenetic cause, but most common inherited cause@EBr)), Klinefelter.
D) Endocrine : Hypothyroidism and cretinism(K""'to e') are Ttreventahle causes of Mp,Nntr' u ss)
.

E) Natalfactors: Birth injury, birth asphyxiflReratae4),intracerebral hemorrhage.


F) Others: Tuberous sclerosis(Pcl08),head injury, CNS infection, kernicterus, child abuse, hypoglycemia, malnutrition,
autism, intrauterine infections (TORCH), and cerebral palsy.

Scales used for mental retardation


o Children 0- j years: Barley Scales of Infant Development - II (BSID - II)
o Children > 3 years: Wechsler scale.
o Nonverbal patient : Leiter-R scale.

NEUROFIBROMATOSIS

Neurofibromatosis type-l (Von-Recklinghausen disease)


e Itisthe mostcommonneurocutaneousdisorder. NF-I isdiagnosediftwoof followingsevensarepresent:-
1) Six or more cafe-au-lait macules: > 5mm in prepubertal age and > 15 mm in postpubertal age.
2) Axillary or inguinal freckling.
3) Two or more Lisch nodules (hamartomas on iris).
4) Two or more neurofibroma or one plexiform neurofibroma.
5) A distinctive osseous lesion : Sphenoid dysplasia or cortical thinning oflong bones.
6) OPtic glioma\lrMs 03)
.

7) A first degree relatiye with NF-1.


e Otherfindingsarepseudoarthrosisoftibia,scoliosis(mostcommonorthopaedicproblem\Ite)),shortstature,mental
retardation, hypertension, epilepsy, meningiomas, ependymomas, astrocltomas, pheochromocltomas andleukemia
(m o st c o mm on j uv enil e my el o m ti c le ukemia(
o"' )
o n o cy ).
o NF-l is an autosomal dominant disorder due to mutation of NF-I gene ot chromosome 17, coding for protein
neurofibromin-1.
Cneprun 8 Central Nervous System

A eleild is le*ving fol&owing skin lesisns. Xlignosis is -

a) Neur*fibromatosis
b) Tuberc'.;"ssclerosis
c) VHL disease
d) Tr.rberous scierosis

na'i"e,,
Ans, is Neuro&hr$ma*osis
a This chiid is having cafe-au-lait spots (flai pigmented spots) + neurofibromatosis type-1.

A cldld is having follerwimg ski:l lesi{l&s" Sign*cis is -

a) Neurofibromatosis
b) Tuberous sclerosis
c) VE{L disease
d) Tuberous scierosis

Ans" is'd i.e., lnleuro&i:ralrxlarosis


a Axillary freckiing with multiple cafe-au-lait spots are seen in Neurr:fibromatosis.

ficN eu nofr brormatssf s rype-3


a NF-2 may be diagnosed when one of the tbllor,r,ing t\4,o f'eatures ;1e presen.i.

't" Eilateral *o;*usLic reur,*tma --i. Most r:listincti-re 1le:li r:ii:

Jieriruirbr,-;t'ra, ;iieningioma, giiorla. Slchwiirir..r;-*a or jr:venlle pr,:ii surrilriiriiLt' caraiact.

Elr mertrin,

?$ m & ffi &u s $e L ffi mss s { &e


E e", ffi &} q{ g
N- L&'5 m 5 e ee$ m}

s Tub:lous sc.li:ro:is is chara;te;12*,-i 1-,,v :-

oJ)
Periu.rrgrral fi hror*easi-t+'n"' (Koenerr.s trirror).

,li lrleru;-rlasrns : - Subepencivmal gi;'.ni cetr1 asti-cci'toma, eperidymona, rhai:etrofi:1y*ri1a$,r,rf fu,1666('it';o), angiq:rirr.:;:;:l:.
oi rr:,r iiirlnel,, iiver, irdrenal and paricreas.

?}ie clrild in picture may have foll*wialg tumrors exeept -

a) Ependymoma
b) Rhabdcmyoma of heart
c) Angir:mvoma of kidney
d) SCC of lung
CHeprsn 8

Ans.is'd'i.e', SCC of lung Small cel carcinoma


* The picture l. *t o*lrrg facial angiofibtoma (adenoma sebaceum) of tuberous sclerosis'
of lung is not a feature of tuberous sclerosis'

sru R6E-WEBER SYrq BRCIeeq


(port
disorder characterized by facial capillary malformation
a sturge weber synd.rome is a rare congenitar vascular
affecting the brain and eye'
wine stain@Pe
ea))
and associated capillary venous malfurmation
dis-
and skin of face typicary in the ophtharmic and maxillary
c It consists of angiomas invorving the reptomeninges
12)
tributions of trigeminal nerveuilMs '
:-
a Important features of sturge-weber's s1'ndrome are
a Cavernoushemangioma(DPcse) e Seizures(ALMS")
s Port-wine stctin@P?e6)
x Hemiparesis x Mental retardation
a Pheochromocltoma
x Visual defects x Congenital glaucoma
x Transient stroke oil
rail-track (Trana traek or rail road) calcifi cation(ArIMs
* X-ray skul1 shows characteristic

The dlagnosis of glven figure ie -

a) NF-1
b) NF-2
c) Tuberous sclerosis
d) Sturge-Weber sYndrome

Ans, is'd' i.e', Sturge-l4kber syndrome


(on left side), a characteristic of Sturge-Wetrer syndrome'
* The figure is showing port-wine stain

vCIru HlppEL-t'$upALJ plqqeqg

bv classical
of vHL gene on chrornasome3' It is characterized
a It is an atrtosomal dominant disorder due to mutation s')' other associated features
triadof cerebellar hemangioblastoma(Atea), retinal angioma arrd rew:$ eli c&rcinowta\l
arepheochromoc}tomaandcysticlesionsinvariousorgans(k!rlrrer,.iir,er,pancreas).

r$r56EI"!'&&!&QUS

Abnorrmalitias of anterior fontanel antetior


months of ags(ttts'1.rss). The presence of excessivelylarge
e The anterior fontanel normally closesbetween 12-1g
closure is a recognized feature of some diseases' Important ones are :-
fontane! and its delayed
xAchondroplasiaxHypothyroidism(Pclgg)
x Congenital syphlis
a Down syndromsedoa)
q8) x IUGR
n irlxI)erfectaFcl
Osteogenesis
x Cretinism
x Apert sYndrome
x Malnutrition
x Coqgenital rubelia
r Thalassemia Inajor

fontanel is flat or srightly depressed, relative to


frontal and parietal bones' Bulging anteriorfontau'el
. ilflT* ;llr,".
ICT during infancy.Important causes of bulging anterior fontanel are :-
is reliable sign of raised
r Crying infant r Raised ICT
r Hydrocephalus r Vitamin A intoxication
: Corticosteroid withdrawal r Galactosemia
a Rickets@cro8) r Pseudotumor cerebri
t TetracYcline(Pclo8) r Corticosteroid therapy
r Nitrofurantoin & Nalidixic acid r Hperparathyroidism
one important cause of bulging anterior font anel is vein of Galen
' malfurmatiot arrMs 11, 06,Ar 0?). child presents with
CHFanMs t1' 06' dr oz) (high output failure), and, audible brui{AIIMs 11, 06, Ar 07)
onanterior fontanel. Lateral ventricles are
11' 06, Atoz)
dilatedullMs due to hydrocephalus.

r Normally, anterior fontanel is pulsatile. Pulsatlons may disappear


in morked elevation of lc',Atss).

lnfant Tremor syndrome


o It is a self-limitinglPcr'e) disorder characterized,by acute
and gradual onset.
o It is found in infant and young children (5 months
to 3 years), especially exclusive breast
feed children.Boys are
affected more commonly.
o Most cases occur in summer months in children
belonging to low socioeconomic group
o Etiological possibilities are malnutrition, vitamin B,rdeficiency,
magnesium deficiency and viral infection.
o There are 3 clinical phases - prodromar, tremor & post
tremor phases.
o There is hyperpigmentation?clos), especialy over dorsum
of hands@erost,feet@etoe), knees ankles@cra8), wrisss@eroe),
terminal phalanges dt thigh@etoat, At times there may be fever,
upper respirat ory tractinfection, diarrhoea, edema,
hepatomegaly & a tremulous cry. Tremor are coarse@crrrl
fast, Z_V ,y4rrper second, of low amplitude, initially
intermittent but become continuous later on. There is mild to moderate
pailor. pnenumoencephaly reveals cortical
atroPhY@crott,
o CT scan shows no abnormalities or mild atrophy.
o Subnormal intelligence is only long term sequele.
o Treatment is largely empirical, symptomatic & supportive.

Decorticate Vs Decerebrate posture


r Decorticate posture (flexor posturing or Mummy baby) is characteriz
edbyflexion of arm and extension of legs{unnr;.
It is due to damage to area in cerebrar hemisphere, internal capsule,
and thalamus.
o Decerebrate posture (extensor posturing) is characterized
by extension of all four limbs. It is seen in brain stem
damage (below red nucleus). Decerebrate posture is more omnious
than decorticate posture{Nwr).

Most common cause of convulsion in a child with fever : Febrile


seizure.

e Percentoge of recurr,ence in febrile seizures :30-50o/o of cases.


,:-.'
:ia
1-ri:,
i:t:ii
a
:+;:_
:::Ii-:

t
:.l:

I Drug of choice for


:11:::-
6
Drugs for 'iuvenile myoclonic epilepsy: Valproate
lorug or iholce). iopiramute,;
zonisamide. ";"r";;;;ililTi;;;;;;;i,
a

?.

b
Cnaptr,n 8 Central Nervous Systr,m'':'

. [amln deficiency causing neonaial selzure: Pyridoxin.


--estprognosisinneonatil seizure:Focal seizures (2"d best prognosis after'subtle seizureswhich cari'ies best prognosis).
good prognosis: Seizure due to hypocalcemia'
')eonatal seizure with
',' ost common type of neonata! seizure: Subtle seizure'

Drug of choice for neanctal setzure : Phenobarbltone


hypoglycemia, pyridcxin
impartant ccuses of status epilepticus in children: ldiopathi<, drugs, heacl trauma. meningitis/encephalitis,
Jeficiency.
Absence seizure is also called: Petitmai epilepsy (minor epilepsy)'
3 Hz splke-wave sn EE6, no
impartant features af absence seizure:seen in childhood (4-8 years), precipitation by hyperventilation,
oostictal confusion.
for absence seizure (peiitrnal epilepsy):'dalpraate
(ethosuximide is an alternative).
DruE of choice

Drug of choice for simple parciai seizure: Carbamazepine'


ii'titia durg of choice for status epilepticus : Lc razepa m '

Kernig's sign is seen in: Acute bacterial meningitis'


lltlast contman presentation of neonatal meningitis: Refusal to feed (poor feeding).

Most common complication of menrngitis in children: Seizures'


Residua! auriitory difect is seen in meningitis by: H' influenza'

,l,r-"rr,nro,* requirea in H. infiuenzoe meningitis: ABER/BERA'


raised protein, decreased glucose & chloride'
CSF findings in bacteriel meninglfls: Elevateci c5F pressure, elevated neutrophils {PMNs),
csF findings in TB meningitis: Elevated csF pressure, elevated lymphocytes,
normal/slightly raised protein, decreased glucose, very
low chloride.
streptococcus (str. agalactiae) follow:ed by E.coli as the 2"d n'lost common cause'
h/lost common causes oi neonatal rneningitis:Group B

streptococcus, E. coii, l-isteria, streptococcusfecalis, staphylocoecus, klebsiella'


lmpoftantbacteriacausing neo*atalmeningitis:Group B

causes of meningitis between 2 manths to


j years:Pneumococcus (mc), meningococcus (2"d mci, Fl. influenzae (3'd mc)'
!ilost cainmorl
Treatment of choice for H. i.nfluenzae meningitis:fhird generation cephalosporins'
Treatment of chaicefor meningocaccal meningitis :Third generation
cephalosporins (ceftriaxone / cefotaxinie); prenicillin can be Eiven
lor penicillin sensitive strains.
merringitis'
O rn,,O *,ir* *rn,ing,ti, shawing basal exudate and hydrocepholus on C7, iiagnasisls :Tubercular
nerve palsies, hemibailismr.;s, ptosis, decerebrate or
!{nfrartant camplieations cf rB rneningitls: Hemiplegia/quadriplegia, cranial
dc,corticate rigiditY.
Not c complication o{TB meningifis: Parkinsonism'
[\,l*st {amffiort cause of meningoencephalitis in chiidren: Enter*:viIus'
tcng herpes simplex vi; uses, t'teoit#[a! entepha!i;is ts cuLlsea most oflt"" r'y :
H sv'2 '
,.it!
Alast braln tLtmars in children cre: Irrfratentor!al'
astrocytoma (a type of giioma)'
illcst corlncn brain lumor,/j,ttr,Tcrania! tttmor in children: Cerebellar
Moste*ntntottpasteriorfassctu,,llorin children: Astrscytorna(:erebeilarastrocytoma)'
Brain turnor assaciateri v,/ith best prag,: osis in chiidren: Cerebellar astrocytoma

!{ilportafit pasteriorfossat*mars in children..cerebellar astrocytoma {rnc), medulloblastoma (2''d mc), brarnstern giioma, ependyn'loma'

lncreased signs of ICT in brain tumor'.Midline posterior fossa tumor'

tvlid! i ne svve! li ng ai:isi ng from cerebe!! u m : Medullohlastoma'

1,4*stcammansuprasellarcystlemsssinchildren:Craniopharyngicrrna
Mas! comrnan cause of suprasel!itr t*icificatian in chiidren: craniophatynEioma
ly'!ost com mon supratentorlal tu mar i n chi ld ren : Crariiopharyngioma'
tv4osl common tvpe cf cerebral prrirv : Spastic CP
Type of cerebral palsy commcnly csscciated with scoliasis: spastic quadrip!egic cP.

?!ue {}ba!.tt r3*tus mari.naratus: Seen iii athetoid CP, biiateral, <jue to asphyxia, in basal ganglia, marbled appearance'
r a s i'E :6 ;;rlvet;triru!ar !eukomalacia: Spastic dipleEla'
{leurai tube deleits are prevented by: Folic acid'
^+;. +: rlcetyicholinesierase
,ira*r,irhnlinc<ior:s, anci a!pha-fetoprctein in anrniotiC flUld
s dural
Markers for titbedeiecr
ilW*rE,R 8: cerrrrar Nervous system

e lmportant neural tube defecfs: spina bifida, meningocele,


meningomyelocele, myelocele, syringornyelocele,
cephaly. encephalocele, anen-
s
& Porencephaly is due to:Cerebral infarction (vascular
lesion) or head injury.
@ Most important diagnasffc pratcedure in leaky meiningomyelocele
:csF and brood sarnples for culture and sensitivity.
& Most common viral cause of aqueductal stenosrs :
Mumps.
a Most cammon cause of non-communicating (obstructive)
hydrocephalus:Aqueductal stenosis.
* Most common cause of communicating (non-obstructive)
hydrocephalus : subarachnoid hemorrhage (rnc),
meningitis (2"d mc). postinflammatory due to
& Mast cofilrnon eaise,of af fetar veitricuromegaly:
Aqueductat stenosis.
& lmportant features of pseudotumor cerebri :Raised lcll normal
ventricular
size, normal cr & MRl, no focal neurological
deficit.
& tmp;ovionr4rrns causlng pse_udotumor cerebri:.Tekacycline,
nalidixic acid, nitrofurantoin, steroid therapy
or. withdrawal.
& Grades of mental retardation: Mild MR (lQ 51-70),
moderate MR fle 36 5o), severe ,--i,o 21 -3s),profound MR
(rQ < 20).
* Most cOmmon genetic cause of mentol retardalion:
Down syndrome.
& Preventabl e ca use of m ental reta rd ation: Hypothyroid
ism,/cretinism.
@ Salicylates {aspirin} can cause Reye,s syndrome.
* Viruses causing Reye,s syndrome.: lnfluenza, varicella (Herpes
virus_3), echovirus, adenovirus, coxsackievirus,
EBV.
'* chortiateristicfeatureaif Reye'ss\ndrome:No jaundice{bilirubin
is not raised) despite,dysfunction of
liver.
I lmportant findings of syndrome: Hypoglycemia, Hypoglycorrhoea, t pr, ^
Reye's
phatase, hyperammonemia, mixed ,.rpirutory scor/sspt/LDH, normal bilirubin & alkaline phos-
alkalosis with metabolic acidosis.
c Brachycephaly is due to eaily fusion of: Coronal suture.
@ Most common tumor associated with NF-l: Optic nerve glioma.
*' Most tommon reukzm ia i nNF I' : Juvenite'myelomonocytic reukemia.
o Triad of seizure, mental retardation and adenoma sebaceum is seen
in: Tuberous sclerosis.
* '".,$idline incr'ocr.anial masswith bruit, cllF snd hydrocephalus saggefi
diag3a-rrs of : Vein of Galen malformation.
& lnterventricular hemorrhage can be prevented by: Antenatal
administration of Vitamin K.
.* &anana,5ig4ii'tfetelbraini5seen,lh:spina bifida,ehiari
lJ malformation, ,

* inheritance of neural tube defects : Multifactorial.


& Porencephaly is seen in: Dany-Walker syndrome.

&xe
eHAF,TE&.

QUESTIONs

EPILEPSY c) Response to anticonvulsants is poor


d) Associted absence seizures are present in majority
ofpatients
FEBRILE CONVULSION 10. Drug of choice for juvenile myoclonic epilepsy is ?
(CET Aug.13 Pattern)
1. True about febrile convulsions is (AllMS lune 97)
a) Phenytoin b) Lamotrigine
a) Recurrent in nature c) Valproate d) Zonisamide
b) Foliows high temperature
c) No spontaneous remissior-' 11. Following can be used in the treatment of myoclonic
seizures except - (Ar 10)
d) Occurs at 6 years onwards
a) Valporate b) Carbamazepine
Z. Fercentage ofchildren with recurrence in febrile
c) Topiramate d) Zonisamide
selzures * (All lndia Dec.13 Pattern)
a) 5-I0 o/o
b) 10-20 o/o
NEONATAL SEIZURES
c) 20-25 o/o
d) 25-50 o/o

3, % of children with simple febrile seizures developing t2. Most common cause of convulsion on the First day
epilepsy is - (All lndia Dec.14 Pattern) of life in a newborn is - (NEET Dec.12 Pattern)
a) l-2o/o b) 2-5Vo a) Anoxia b) Head Injury
c) 5-l0o/o d) I0-20o/o c) Hypoglycemia d) Hlpocalcemia
4. Which of the following is not associated with in- 13. Most common cause of seizure in newborn is -
crease in the risk of seizures in future in a child with (AIIMS MaY 08)
febrile seizures - (Ar to) a) Hlpoxia induced ischemic encephalopathy
a) Developmental delay b) Hypocalcemia
b) Late age ofonset c) Metabolic abnormality
c) Complex partial seizures d) Sepsis
d) Family history positive 14. Which vitamin deficiency is responsible for neonatal
5. Most important risk factor of recurrence of fetrrile seizure - @llMS i{ov 09)
selzure ls - (CET Nov 15 Pattern) a) Pyridoxine b) Vitamin C
a) Age ofonset <2years c) Thiamine d) Cobalamin
b) F'amily history 15. l.leonatal seizure carries best prognosis ifthe cause
c) Seizure at time of fever peak 1S- (All India Dec.13 Pattern)
d) Long prolonged fever prior to seizure a) Idiopathic b) Hlpocalcemia
6. A yr. old child with acute onset of fever of 1040 F
6 c) Infection d) Asphlxia
developed febrile seizures and was treated. To avoid 16. Commonest type of seizure in newborn -
future recurrence of seizure attacks what should be (At 08, AIIMS Nov 07)
grven - (CET Aug. li Pattern, AIIMS May 01)
a) Clonic b) Tonic
a) Paracetamol 400 mg + Phenobarbitone daily c) Subtle d) Myocionic
b) Oral Diazepan 6 hourly
t7. Drug of choice of Neonatal seizure is - (AIIMS Feb 97)
c) Paracetamol 400 mg 6 hourly a) Phenytoin
d) I.V. diazepam infusion over 12 hrs b) Phenobarbitone
c) Diazepam
MYOCLONIC EPILEPSY d) Sodium valproate

7. Hypsarrythmia in a child is due to -


18. Causes of status epilepticus in a child- (PGt Dec a8)

a) Hlpernatremia b) Hyponatremia
a) Grandmal epilepsy (CE1- Nov. 14 Pattern)

b) Petitmal epilepsy
c) Hlperkalemia d) Hlpokalemia
c) Myoclonic epilepsy 19. Best prognosis of neonatal seizures is in ?
(CET June 14 Pattern)
d) Reflex epilepsy
a) Myoclonic b) Tonic clonic
o
o. drug ofchoice for infantile spasm is-
(All India Dec.1i Pattern)
c) Focal d) Opsoclonus

a) Vigabatrin b) ACTH
c) Ethosuximide d) Carbamazepine ABSENCE SEIZURES
9. Which one of the following in the characteristic
feature of juvenile myoclonic epilepsy - (AIMS May 06)
20. All are features of absence seizures except - @l al)

a) Myoclonic seizures frequently occur in morning a) Usualiy seen in childhood


b) Complete remission is common
b) 3-Hz spike wave in EEG
c) Postictal confusion 30. The most common presentation of neonatal
d) Precipitation by hyperventilation
meningitis is - (Orissa 96
21, The feature of petit mal epilepsy in EEG is - a) Bulging fontanels b) Nuchal rigidity
(All India Dec.14 pattern) c) Poor feeding d) Conr,-ulsion
a) Spike and dome 31. Most common complication of meningitis in chil_
b) Continuous stunted spikes dren is (AI1MS May %
c) Waves at the rate of l0/sec. a) Hearing loss b) Seizures
d) Continuous tall spikes c) Hydrocephalus d) Mitralregurgitation
)) A school going boy was noted with vacant stare 32, Prior to discharge of a patient of H. influenza
several times a day. There was no history of fever, meningitis the essential investigation to be done is _
seizures and neurological deterioration. What is
(AIl India Dec'14 Pattern'
the diagnosis - (Ar 10) a) ABER
a) Atonic seizures b) EEG
b) Absence seizures c) Developmental screening test
c) Myoclonic seizures d) ECG
d) Schoolphobia
33. In a small child diagnosed with H.influenza men_
23, Absence-seizures are seen in - (AlI India Dec.14 pattern) ingitis, what investigation must be done before
a) Ground mal epilepsy discharging him from the hospital ? (CET Nor.ts pattern)
b) Myoclonic epilepsy a) BERA
c) Petitmal epilepsy b) MBr
c) CT scan d) X-ray skull
d) Hlperkinetic child
NEONATAL MENINGITTS
TREATMENT OF SEIZURES
34. A neonate develops signs of meningitis at seven
24. Drug of choice in absence seizure- (All India Dec.13 days of birth. The presence of which olthe fo[owing
Pattern)
infectious agent in the maternal genital tract can be
a) Carbamazepine b) Phenl,roin tlre causative agent of this disease - (ArrMS May 04)
C) ACTH d) Ethosuximide a) Neisseria gonorrhoeae
Drug for long term treatment of partial seizure is- b) Chlamydiatrachomatis
(CET Not 15 Pattern) c) Streptococcusagalactiae
a) Valproate b) Carbamazepine d) Haemophilous ducreyi
c) Eptoin d) J). A 25 year old woman had premature rupture of
Phenobarbitone
membranes and delivered a male child who became
26. Initial drug of choice in a child with status lethargic and apneic on the Ist day of birth and went
epilepticus - (NEET Dec.12 pattern) into shock. The mother had a previous history of
a) Lorazepam b) Phenobarbitone abortion I year back. On vaginal swab cultur--e
c) Valproate d) Phenltoin qrowlh of B-haemolytic colonies on blood agarwas
27. The drug of choice in petit seizures is - (Ar es) found. On staining these were found to be giam pos_
a) Clonazepam b) Ethosuximide itive cocci. Which of the following is the most likely
c) Diazepam d) Sodium Valproate etiological agent -
(AI04)
CNS INFECTIONS a) Streptococcus pyogenes
b) Streptococcus agalactiae
Normal intracranial pressure in a child is - c) Peptostreptococci
(CET luly 15 pattern)
d) Enterococcus faecum
a) 30-70 mm of HrO Most common cause of treonatal rneningitis _
b) 50-80 mm of HrO (CET Nov.14 pattern)
c) 100-150 mm of H,O a) Staphylococcus b) E. coli
d) 50-150 mm of HrO c) H. influenze d) Pneumococcus
37. The following bacteria are most often associated
BACTERIAT MENINGITIS with acute neonatal meningitis except _ @r 0s)
a) Escherichia coli
29. Kernig's sign is seen in - (PGr e8) b) Streptococcus agalactiae
a) Pneumonia c) Neisseriameningitidis
b) Acute bacterial meningitis d) Listeria monocytogenes
c) Mental retardation
d) Cerebral palsy
CAUSES OF NEONATAL MENIGITIS tory of fever, vomiting and alteration of sensorium
Cranial CT scan reveals basal exudates and hydro-
_i8. Most common cause of pyogenic meningitis in 6 cephalus, the most likely etiological agent is -
month to 2 years of age is- (AilMS lune 99, AI 0i,98) (il a4,9s, DPG 09)
a) Staphylococcus aureus a) Mycobacterium tuberculosis
b) Pneumococcus b) Crlptococcus neoformans
c) Streptococcus pneumonia d) H. influeuenzae c) Listeramonocltogenes
lo The most common causative agent of meningitis in d) Streptococcus pneumoniae
the age group of 6 months-3 year amongst the 47. All of the following are known complications of
following is - (CET June 14 Pattern) tuberculous meningitis except - (Ar 97)
a) Streptococcus a) Cranial nerve palsy
b) H. influenzae b) Cerebral infarction
c) Staphylococcus c) Ptosis
d) N. gonorrhoeae d) Parkinsonism
{0. The most common cause of meningitis in children 48. The CSF findings in TB meningitis include - (At e4, 07)
aged 5 yrs is- (All lndia Dec.13 Pattern) a) High sugar + low protein
a) H influnzae b) N. meningitides b) Low sugar + high protein and Lymphocytosis
c) Staphylococcus d) E. coli c) High sugar + high chloride
41. Most common organism causing meningitis in a 1
d) Low sugar + high protein and Lymphopenia
year old child - 6r oz) 49. A 10 year old girl presented with fever convulsions,
a) Streptococcuspneumoniae neck rigidity. CSF findings are protein 150 mg, sugar
b) H. influenza 40 mg, chloride 2500 mg with lymphadenopathy-
c) Listeria @IIMS Dec 98)
d) Neisseria meningitidis a) viral meningitis
42. A newborn of 7 days old presented with meningitis. b) Pyogenic meningitis
Most common cause - (pGI Nov. 15) c) Crlptococcal meningitis
a) E.coli d) Tirberculous meningitis
b) Streptococcal pneumonia
c) N. Meningitis ENCEPHALITIS
d) Streptococcal agalactiae
s0. Which of the following is the most common cause of
TREATMENT OF BACTERIAL MENINGITIS meningoencephalitis in children - (At il, 10)
a) Mumps b) Arbovirus
43. Haemophilus influenzae has been isolated from the c) HSV d) Enterovirus
CSF of two year old boy suffering from meningitis.
The strain is beta-lactamase producing and resistant BRAIN TUMORS
to chloramphenicol. The most appropriate antimi-
crobial in such a situation is - (AI 04) 51. True statement regarding Brain Tumor in children
a) Trimethoprim-sulphamethoxazole combination ts- (Ar 2000)
b) Ciprofloxacin a) Mostly is infra - tentorial
c) Third-generation cephalosporin b) Papilledema is rare
d) Vancomycin c) Is the most common tumour in children
M. A child presented with fever for 2 days, altered sen- d) Hydrocephalus is rare
sorium and purpuric rashes. His blood pressure is 5.1"" The most common Intracranial tumor in children is-
90/60 mmHg. Treatment of choice is - (AIIMS May 02) (NEET Dec.12 Pattern)
a) IV Quinine a) Glioma b) Ependymoma
b) IV Artesunate c) Meningioma d) Lymphangioma
c) IV Penicillin 53. Most common posterior fossa tumour in children is-
d) Chloroquine
(All India Dec.1j Pattern)
45, Treatment in days for meningitis for strept pneumo- a) Medulloblastoma
nia in children - (CET Nov 15 Pattern) b) Glioblastoma multiformae
a) 5 days b) 7 days c) Astroltoma
c) 10 days d) 21 days d) Meningioma
54. Tumor associated with best prognosis in children
TUBERCULAR MENIGITIS is- (At 97,94)
a) Medulloblastoma
46. A 10 month old child presents with two weeks his- b) Ependymoma
CAUSES OF NEONATAL MENIGITIS tory of fever, vomiting and alteration of sensorium
Cranial CT scan reveals basal exudates and hydro-
38. Most common cause of pyogenic meningitis in 6 cephalus, the most likely etiological agent is -
month to 2 years of age is- (AtrMS lune ss, At 01,98) (Ar 04,9s, DPG 09)
a) Staphylococcus aureus a) Mycobacteriumtuberculosis
b) Pneumococcus b) Crlptococcus neoformans
c) Streptococcus pneumonia d) H. influeuenzae c) Listeramonocy.togenes
19. The most common causative agent of meningitis in ci) Streptococcus pneumoniae
the age group of 6 months-3 year amongst the 47, All of the following are known complications of
foltrowing is - (CET June 11 Pattern) tuberculous meningitis except - (Ar e7)
a) Streptococcus a) Cranial nerve palsy
b) H. influenzae b) Cerebral infarction
c) Staphylococcus c) Ptosis
d) N. gonorrhoeae d) Parkinsonism
40. The most common cause of meningitis in children 48. The CSF findings in TB meningitis include -(Ar s4,02)
aged 5 yrs is- (All India Dec.13 Pattern) a) High sugar + low protein
a) H influnzae b) N. meningitides b) Low sugar + high protein and Lymphocfosis
c) Staphylococcus d) E. coli c) High sugar + high chioride
41. Most common organism causing meningitis in a I d) Low sugar + high protein and Lymphopenia
year old child - @t o7) 49. A 10 year old girl presented with fever con"ulsions,
a) Streptococcuspneumoniae neck rigidity. CSF findings are protein 150 mg, sugar
b) H. influenza 40 lrng, chloride 2500 mg with lymphadenopathy-
c) Listeria @IIMS Dec e8)
d) Neisseriameningitidis a) viral meningitis
42. Anewborn of7 daysoldpresentedwithmeningitis. b) Pyogenic meningitis
Most common cause - (PGI Nott. 15) c) Crlptococcalmeningitis
a) E.coli d) Tuberculous meningitis
b) Streptococcal pneumonia
c) N. Meningitis ENCEPHALITIS
d) Streptoco ccaT agalactiae
50. Which of the following is the most common cause of
TREAIMENT OF BACTERIAL MENINGITIS meningoencephalitis in children - (Ar 11, 10)
a) Mumps b) Arbovirus
13. Haemophilus influenzae has been isolated from the c) HSV d) Enterovirus
CSF of two year old boy suffering from meningitis.
The strain is beta-lactamase producing and resistant BRAIN TUMORS
to chloramphenicol. The rnost appropriate antimi-
crobial in such a situation is - (Ar 01) 51. True statement regarding Brain Tumor in children
a) Trimethoprim-sulphamethoxazole combination is - (Ar 2000)
b) Ciprofloxacin a) Mostly is infra - tentorial
c) Third-generation cephalosporin b) Papilledema is rare
d) Vancomycin c) Is the most common tumour in children
M. A child presented with fever for 2 days, altered sen- d) Hydrocephalus is rare
sorium and purpuric rashes. His blood pressure is 52. The most common Intracranial tumor in children is-
90/60 mmHg. Treatment of choice is - (AIIMS May 02) (NEET Dec.12 Pattern)
a) IV Quinine a) Glioma b) Ependymoma
b) IV Artesunate c) Meningioma d) Lymphangioma
c) IV Penicillin 53. Most common posterior fossa tumour in children is-
d) Chloroquine
(All India Dec.lj Pattern)
45. Treatment in days for meningitis for strept pneumo- a) Medulloblastoma
nia in children - (CET Nov 15 Pattern) b) Glioblastomamultiformae
a) 5 days b) 7 days c) Astroltoma
c) 10 days d) 21 days d) Meningioma
54. Turnor associated with best prognosis in children
TUBERCULAR MENIGITIS is- (Ar 97, e1)
a) Medulloblastoma
46. A 10 month old child presents with two weeks his- b) Ependymoma
c) Cerebellar astrocytoma
d) Glioblastoma-multiformis CEREBRAL PALSY
55. Young child with mass in posterior cranial fossa
63. Cerebral palsy in which upper limb is less affected
showing mural nodule. Diagnosis is-
than lower limb - (CET Nov 15 Patter-.
(All India Decl5 Pattern) a) Spastic diplegia
a) Hemangiocytic pericytoma b) Spastic paraplegia
b) Astrocltoma (pilocyric) c) SpasticQuadriplegia
c) Ependymoma d) Spastic hemiplegia
d) cru
64' Diplegia Vs quadriplegia-False is - (CETNov t5 patterr.
56. Common posterior cranial fossa tumours include all
a) Diplegia is weakness in upper arm more than leg
of following except - GET June 14 pattern) b) Quadriplegia is most severe form of cerebral palir-
a) Medulloblastoma c) Quadriplegia is usually association with MR &
b) Oligodendroglioma seizure
c) Ependymoma
d) All are corract
d) Cystic astrocytoma
65. Which type of cerebral palsy is commonly associated
57. A l0 year old child presented with headache, vomit_
with scoliosis and other orthopedic probi"** -
ing, gait instabilityand diplopia. On examination he
had papilloedema and gait ataxra. The most proba- (AIIMS lune ei
a) Spastic quadriplegia
ble diagnosis is - (ArrMS Nov 02) b) Anterior cerebral palsy
a) Hydrocephalus c) Spastic deplegia
b) Brain stem tumour d) Atonic cerebral palsy
c) Suprasellar tumour
d) Midline posterior fossa tumour 66. Not found in cerebral palsy - (pGr June 98
a) Hlpotoniciry b) Microcephaly
58. A l0 yr old boypresents with midline swelling aris_
c) Ataxia d) Flaccid paralysis
ing from cerebellum the diagnosis is - (ArrMi Dec s8)
67- True about status marmoratus is all except -
a) Astrocytoma
b) Glioblastoma multiforme (CET Nov.13 Pattern,

c) Ependymoma a) Present in basal ganglia

d) Medulloblastoma b) Associated with asphyxia


c) Have a marbled appearance
59. Which of the following statements about cerebellar d) Unilateral
astrocytomas in pediatric age group is false _
a) These are usually low grade tumors
@r 0s) 68. Most common sequelae due to periventricular leu_
komalacia -
b) These tumors have a good prognosis ,,IIMSNa,-os)
a) Spasric diplegia
c) These are more commonly seen in the 1st and 2"d
b) Spastic quadriplegia
decades
c) Mental retardation
d) These tumors are more common in females
d) Seizures
CRANIOPHARYNGIOMA
NEURAL TUBE DEFECT
60. Most common supratentorial tumour in children
69. Neural tube defects are prevented by -
is- (All India Dec.l j Pattern) 6r se, es)
a) Craniopharyngioma a) Pyridoxin b) Foiic acid
b) Astrocytoma c) Thiamine d) Iron
c) Glioma 70. 1 day old male baby delivered by LSCS
had swelling
d) Meningioma oyer back in midline- (All India Dec15 pattern)
61. Supra sellar cystic mass in children is-
(All India Dec'14 Pattern)
a) Medu[obrastoma
b) Craniopharyngioma
c) Meningioma
d) Secondaries
62. Most common cause of suprasellar enlargement
with calcification in children is - (ArrMi Dec es, Ar gt)
a) Iron b) Folic acid
a) Craniopharyngioma c) Thiamine d) VitA
b) Astrocl'toma 7t. Increase acetylcholinestrase in amniotic fluid indi-
c) Meningioma cates - (All India Dec.14 pattern)
d) Suprasellartuberculoma a) Open neural tube defects
Ctrerprna.,S

b) Oesophageal atresia a) Also k/a communicating hydrocephalus


c) Down syndrome b) Due to obliteration of subarachnoid cisternae or
d) Edward syndrome malfunction of arachnoid villi
72. All of the following are neural tube defects except - c) Diiatation of all 4 ventricle
(At 04) d) All of above
a) Myelomeingocoele 81. All are signs ofhydrocephalus in a neonate
b) Anencephaly except - (AIIMS June 97)
c) Encephalocele a) Enlarged head b) Sunset sign
d) Holoprosencephaly c) Crack post sign d) Depressed fontanelle
73. Porencephalyrefers to - (At ee) 82. Most common cause of fetal ventriculomegaly is?
a) Foetal alcohal syndrome (CET Aug.l3 Pattern)
b) Dandy-walker-syndrome a) Arnold Chiari maiformation -I
c) Vascular lesion due to degenerative vessel disease b) Arnold Chiari malformation -II
and head injury c) Aqueductal stenosis
d) Neural tube defects d) Dandy Walker malformation
74. Porencephalyis due to - (MP 2K) 83" Investigation ofchoice for hydrocephalus in infants?
a) Dandly walker syndrome (All India Decl5 Pattern)
b) Cerebral infarction a) Cranial USG b) CT Scan
c) Fetal alcohol syndrome c) MRI d) X-ray skull
d) Trisomy 13 84. Common deformity i*r chiari II malformation is -
75" \{hich of the following statements is false about (All India Decl5 Plttern)
Sacral Meningomyelocele - (Ar e5) a) Syringomyelia
a) Spasticity of the lower limbs is seen b) Meningo myelocele
b) Hydrocephalus is seen c) Hydrocephalus
c) Bladder incontinence may be seen d) AII of above
d) Lax anal sphincter is present 85. Dandy walker syndrome- True is -
76. A7-day old infant has a leaky meningomyelocele. (All India Dec15 Pattern)
', 'llhe rnost useful test for diagnosis and management a) Mostly had hydrocephalus
of the condition is - (AIIMS Nov 11) b) Cystic expansion of 4th ventricle
a) Blood-culture and sensitivity c) Mid cerebellar hlpoplasia
b) Urine-culture and sensitivity d) All are true
c) Rectal swab-culture and sensitivity 86. Followingfeaturedifferentiatehydranencephaly
d) Wound swab-culture and sensitivity from hydrocephalus - (cET luly 15 Pattern)
a) Hydranencephaly is static while hydrocephalus
HYDROCEPHALUS conl.inue Lo increase in size
b) Hydranencephaly does not cause increase in head
77. Commonest cause of non-cornmunicating hydro- size

cephalus in children is - AI e6) c) Cerebral cortex are deficiency/hlpoplastic in


a) Congenital anomaly hydranencephaly
b) Perinatal injury d) All of the above
c) Post-inflammatory obstruction
d) Brain tumors PSEUDOTUMOR CEREBR!
78. lrVhich one of the following is the cornmon cause of
congenital hydrocephalus is ? (At 05) 87. All are true regarding Pseudotumour cerebri except-
a) Craniosynostosis (AllMS Feb 97)

b) Intra uterine meningitis a) Ventricular system is of normal size or small


c) Aqueductal stenosis b) CT Scan shows punctate hypodense areas
d) Malformations of great vein of Galen c) No focal neurological deficit
v9. Most common cau$e of hydrocephalus in children
d) Intracranial pressure is raised
is- 6t e8) 88. }fhich one of the following drug$ {annot cause
a) Post inflammatory obstruction Pseudoturnour Cerebri? (UPSC o4)

b) Buddchiary syndrome a) Salicylic acid


c) Brain tumour b) Nalidixic acid
d) Perinatal injury c) Tetracycline
80. Non-obstructive hydrocephalus - true is -
d) Nitrofurantoin
(All India Decl5 Pattern)

/
MENTAL RETARDATION MISCELLANEOUS
89. A male child of 15 years with a mental age of 9 years 99. Yeung child with taughing spells. Diagnosis -
has an IQ of - (At oi) (All India Dec15 Panen
a) 50 b) 60 a) Hlpothalamic hamartoma
c) 70 d) 80 b) Tetralogy offallot
90. IQ between 50-70 indicates - (Ar 9s) c) Nitrous oxide poisoning
a) Mild mental retardation d) None ofthe above
b) Moderateretardation 100. Decortieatc child - Flase statement is -
c) Severe retardation (NEET Dec.12 Patten
d) Profoundretardation a) Acute Brain injury
91. \{hich is MC genetie cause of mental retardation - b) Susthalamic, CT & frontal iobe lesion
(NEET Dec.12 Pattern) c) More dangerous than decerebrate lesion
a) Tirberous sclerosis d) Flexion of arm & extension of lower limb
b) Cri-du-chat syndrome I01. Anterior funtanelle closes at - (CET July 15 Patten
c) Fragile-x-syndrome a) 2-3 months
d) Angel's syndrome b) 4-7 months
92. Which of the following is a preventable cause of c) 9-12months
mental retardation - (Ar es) d) l9-24months
a) Hlpothyroidism b) Down syndrome lW^ Closure of the anterior funtanelle is ddaned in aItr
c) Cerebral palsy d) All of the above except - (All India Dec.14 Pattem
93. Preventable cuse$ ofmental retardation are - a) Down's syndrome
(All India Dec.14 Pattern) b) Osteogenesis imperfecta
a) Downs b) Phenylketonuria c) Hlpogonadism
c) Cretinism d) Cerebral palsy d) Hlpothyroidism
94. Mental retardation can be proved if delayed mile- lO3. klost comrmon vir,atr ealrse of ac.qqired aquednctal
stones and slow or retarded growth seen upto which stenosis is ? (CET Nov.13 Paternl
age (tn year)- (All India Dec15 Pattern) a) Rubella
a) t2 b) 16 b) Mumps
c) 18 d) 20 c) Toxoplasma
d) Enterovirus
!
REYE'S SYNDROME 104. True about infantile tremor syndrome - (pGr Noy. i5)

" a) Hlperpigmentation of extremities


95. Reye's Syndrome is eause by aII except-
b) Fine tremor
(All India Dec.14 Pattern)
c) Cortical atrophy
a) Adeno virus b) RSV
d) Self limiting disorder
c) Herpes d) Influenza e) More common in girls
96. All of the following are tnre of R.eyels syadrome,
t05. Brachyeephaly is due to fosion af - (CET June 14 pauern)

except - (All India Dec. 15 Pattern)


a) Parietal suture
a) It fequently compiicates viral infections b) Sagittal suture
b) Prothrombin time is prolonged c) Lambdoid suture
c) Disease may be precipitated by salictylates d) Coronal suture
d) Deep jaundice is present f 06. Which one of the following is the most common
97. Aspirin is associatedwith- (NEET Dec.12 Pattern)
tumor associated with type I neurofibromatosis -
(ArrMS o3)
a) Reye's Syndrome
b) Sjogren Syndrome a) Optic nerve glioma
c) Reiter Syndrome b) Meningioma
d) None ofabove c) Acoustic schwannoma
d) Low grade astrocytoma
98. AII are seen in Reye's syndrome exc€pt-
(CET Nott. 15 Pattern)
rc7. Which among the following is the most common
a) Aminoaciduria tumour associated with neurofibromatosis in a
b) Metabolic acidosis child ? (At u)

c) Increased serum transaminases a) luvenile myelomonocltic leukemia


d) Respiratoryalkalosis b) Acute lymphoblastic leukemia
c) Acute monocltic leukemia
d) Acute myeloid leukemia
Cnepr:sn 8 Centrdl

!. A triad ofseizure, mental retardation and sebaceous c) Intermittentdiazepam


adenoma is seen in - TAIIMS lune 97) d) Prophylacticphenobarbitone
a) Congenital slPhilis 115. A child in status epilepticus should not be given -
b) Tuberous sclerosis ]ipmer 04)
c) Toxoplasmosis a) Clonazepam b) Phosphenytoin
d) Hlpothyroidism c) Lamotrigine d) Diazepam

-,9. A}year old child is trrought by parents lYith history 116. Which of the following has the worst prognosis -
of seizures and developmental delay. He has multi- $PMERaL)
ple hypopigmented macule$ over the baek. What is a) Rolandic epilepsy
the rnost probable diagnosis - (At 1o) b) Versive epilepsy
a) Neurofibromatosis tlPe 1 c) Absence epilepsy
b) Tuberous sclerosis d) Infantile spasm
c) Sturge weber's sYndrome ll7 " Most severe papilloedema is caused by- (Kerola 95)
d) Linear Sebaceous nevus syndrome a) Pituitary tumor
I 10. A 10-yr old male child was presented to the b) Frontal lobe tumor
pediatrician for evaluation ofa seizure disorder' On c) Posterior cranial fossa tumor
exarnination a vascutrar plaque was found along the d) N{edulloblastoma
ophthalmic and rnaxillary divisions of the trigem- 118. The common type of ceretrral palsy seen in hospitals
inal nerve. The mother informed the pediatrician is- (CUPGEE 95)
that the lesion was Present since birth and there was a) Spastic b) lv{onoplegia
no change in rrorphol.ogy.The most likely possibil- c) Quadriplegia d) Diplegia
rty is -
(AIIMS May 12)
119. Scapocephaly is due to premature closure of the -
(Orissa 99)
a) Sturge Weber sYndrome a) Coronal sutrue b) Metopic suture
b) Inlantile hemangioma c) Sagittal suture d) Lambdoid suture
c) Congenital hemangioma of
d) Proteus sYndrome
120. The following are recognised signs any symptoms
raised intracranial tension in a 9 month old infant,
111. A newborn presents with congestive heart failure, (sGPGt o5)
except -
on examination has bulging anterior fontanellae a) Bulging fontanel b) Diplopia
with a trruit on ausculation. Transfontanella usg c) Papiiloedema d) increase in head size
shows a hypoechoic midline rnass with dilated later-
121. A 2 year old child without fever develops bone pain,
al ventricles. Most lillely diagnosis is -(AIIMS Nov .l.1, 06'
vohiting and features of increased intracranial pres-
AI A7)
sure following excessive medication. The drug most
a) Medulloblastoma
likely to be responsible for this is - (UPSC e8)
b) Encephalocele b) Phenothiazine
a) Vit A
c) Vein of Galen malformation d) vftD
d) Arachnoid cYst c) Phenl'toin
ll2. Known to prevent intreventricular haemorrhage
122. The normal concentration of protein in CSF at 4
(PGr 96) weeks may be as high as- (PGI 8a' Delhi sa)
when given antenatallY -
a) 500mg % b) 25mgo/o
a) Phenobarbitine
b) Vitamin K
c) 50mg% d) Z5mgo/o

c) Both 123. In children with cerebral oedema which one of the


d) None following corticosteriods will be effective- (UPSC e6)

a) Hydrocortisone b) Prednisolone
c) Dexamethasone d) Betamethasone
QUE5TIONS OF VARIOUS OTHER EXAMINA-
L24. Banana sign seen in the foetal brain suggests -
TIONS (Comed 05)

a) Renai agenesis b) Encephalocele


I 13" Cornmonest cause of convulsions in a child with
(AIIMS 84, UPSC 08) c) Spina bifida d) Porencephaly
fever is-
a) Febrile convulsions 125. A2Syear old man with congenital cyanotic heart
b) Meningitis disease reports to emergency with history of head-
c) Epilepsy ache. On examination there is systolic murmur best
d) Hlpothyroidism heard on the teft side ofthe sternum in the fourth
intercostal space; the second sound is not split" Tem-
114. Managernent of typical febrile seizures include
Karnat 05)
perature is 37.50C. What is most likely diagnosis -
except - @PG 10.
(ICS 2K)
a) Sponging
a) Cortical vein thrombosis
b) ParacetamoiibuProfen
r.: :lrrlrl.t
...,i. .r ar,:i 'l]..'.]::l:tr:.:i
-,,ir;,i,..
r:'. rj:::,l.,r:r

CHepren I Central Nenrcus Svst

- j8. A triad ofseizure, mental retardation and sebaceous c) Intermittentdiazepam


-
adenoma is seen in @IIMS lune 9Z) d) Prophylacticphenobarbitone
a) Congenital syphilis Il5. A child in status epilepticus should not be given -
b) Tuberous sclerosis
(lipmer 04)
c) Toxoplasmosis a) Clonazepam b) Phosphenytoin
d) Hypothyroidism c) Lamotrigine d) Diazepam
'-
09 . A 2 year old child is brought by parents with history I I 6. Which of the following has the worst prognosis -
of seizures and developmental delay. He has multi- jIPMER 01)
ple hypopigmented macules over the back. What is a) Rolandic epilepsy
the rnost protrable diagnosis - (AI 10) b) Versive epilepsy
a) Neurofibromatosis type I c) Absence epilepsy
b) Tuberous sclerosis d) Infantile spasm
c) Sturge weber's syndrome
Lt7. Most severe papilloedema is caused by- (Keralo 95)
d) Linear Sebaceous ner,rrs syndrome
a) Pituitary tumor
1 10. A f 0-yr old male child was presented to the b) Frontal lobe tumor
pediatrician for evaluation ofa seizure disorder. On c) Posterior cranial fossa tumor
examination a vascular plaque was found along the d) N{edullobiastoma
ophthahnic and maxillary divisions of the trigem-
I 1g. The common type of cerebral palsy seen in hospitals
inal nerve. The mother informed the pediatrician
ls- (CUPGEE 95)
that the lesion was present since birth and there was
no change in morphology. The rnost likely possibil-
a) Spastic b) Monoplegia
ity is -
c) Quadriplegia d) Diplegia
(A]IMS May t2) f 19' Scapocephaly is due to premature closure of the -
a) Sturge Weber syndrome (Orissa 99)

b) Infantilehemangioma a) Coronal sutrue b) Metopic suture


c) Congenitalhemangioma c) Sagittal suture d) Lambdoid suture
d) Proteus syndrome r20. The following are recognised signs any symptoms of
i 11. A newborn presents with congestive heart failure, raised intracranial tension in a 9 month old infant,
except - (sGPGr os)
on examination has bulging anterior fontanellae
with a bruit on ausculation. Transfontanella usg
a) Bulgirig fontanel b) Diplopia
shows a hypoechoic midline mass with dilated later-
c) Papilloedema d) Increase in head size
al ventricles. Most likely diagnosis is -(AIIMS Nov tt, 06, L?l. A 2 year old child without fever develops bone pain,
Ar 07) vomiting and features of increased intracranial pres-
a) Medulloblastoma sure following excessive medication. The drug most
b) Encephalocele likely to be responsible for this is - (UPSC e8)

c) Vein of Galen malformation a) Vit A b) Phenothiazine


d) Arachnoid cyst c) Phenytoin d) vit D
Ll2. Known to prevent intreventricular haemorrhage 122. The normal concentration of protein in CSF at 4
when given antenatally - ed e6) weeks may be as high as- (pd Ba, Delhi 8a)
a) Phenobarbitine a) 500mg % b) 25mgo/o
b) Vitamin K c) 50mg%o d) 25mgok
c) Both 123. In children with cerebral oedema which one of the
d) None following corticosteriods will be effective- (UISC e6)
a) Hydrocortisone b) PreCnisolone
QUESTIONS OF VARIOUS OTHER EXAM!NA. c) Dexamethasone d) Betamethasone

TIONS 124. Banana sign seen in the foetal brain suggests -


(Comed 05)

113. Comrnonest caus€ of com.ulsions in a child with a) Renal agenesis b) Encephalocele


f,ever is- (AIIMS 84, UPSC 08) c) Spina bifida d) Porencephaly
a) Febrile comrrlsions 125. 425 year old man with congenital cyanotic heart
b) Meningitis disease reports to emergency with history of head-
c) Epiiepsy ache. On examination there is systolic murrnur best
d) Hlpothyroidism heard on the left side ofthe sternum in the fourth
114. Management of tFpical febrile seizures include intercostal space; the second sound is not split. Tem-
except - (DPG 10, Karnat 05) perature is 37.50C. What is most likely diagnosis -
a) Sponging fics 2K)
b) Paracetamoi ibuprofen a) Cortical vein thrombosis
b) Pyogenic septicemia
d) Muitifactorial
c) Cerebral abscess
d) Encephalitis
127. Acquired extra - cranial infection that causes Aque_
ductal Stenosis is - (upsc}4)
126. Neural tube defects havewhich one of the following
a) Bacterial endocarditis
inheritance pafterns ? - @psc_il og) b) Mumps
a) Autosomal dominant
c) Measles
b) Autosomal recessive
d) Staphylococcal septicemia
c) X-linked recessive

III
b) Pyogenic septicemia d) Multifactorial
c) Cerebral abscess 127. Acquired extra - cranial infection that causes Aque-
d) Encephalitis ductal Stenosis is - (UqSC 04.,
126. Neural tube defects have which one of the following a) Bacterialendocarditis
inheritance patterns ? - (UPSC-il 09) b) Mumps
a) Autosomal dominant c) Measlbs
b) Autosomal recessive d) Staphylococcalsepticemia
c) X-linked recessive

lrl
CHAPTE.R iB

ANSUfERS

EPILEFSY

TEBRII.E CONVUI.SION

Ans. is 'd i.e., R.ecurrent in nature fRef: A.P. Ghai I'h/e p. 556 6 Vh/e p. 528; Nelson l?th/e p. 245fl
o Febrile convulsions are the commonest cause of seizures during early childhood.
o Mostly occurs between 6 months to 5 years.
o The convulsions are not related to degree oftemprature rise but are frequent iftemprature rises abruptly.
e Spontaneous remmission occurs with no postictal neurological deficit and EEG changes few days after the
seizure is normal.
a Recurrentfebrile seizrxes occur in 30-50% ofcases.
Ans. is 'd' i.e., 25-50o/o lRel llekon 18th/e p. 2457; O.P. Ghai Vh/e p. 52Bl
o Recurrent febrile seizures occur in 30-50% ofcases.

Ans. is 't i.e., l-2o/o


o Children with febrile seizures have an increased risk of developing epilepsy.
o Between 2o/o to 7o/o of all children with febrile seizures develop epilepsy if foilowed upto the age of 25 years. Risk
depends on t)?e of febrile seizure :-
i) Simple febrile seizures --> 2% of all children with simple febrile seizures.
ii) Complexfebrile seizures -+ 6-8% of all children with complexfebrile seizures.
o Three major risk factors increase the risk of epilepsy :-
i) Complex febrile seizures.
ii) Pre- existing neuro dev elopmental abnormality.
iii) Epilepsy infirst degree relatives.

Ans" is 'b' i.e., Late age of onset fRef: CPDT tr}'h/e p" 720; Ctinical Paeddatric 3d/e p. j90; Op Gbai }th/e p. 5561

Risk factor fon seizures in future (Risk faetor for recurrence)


l. Positive family history 4. Early onset offebrile seizure (before 1 year)
2.Atypical compluc febrile convulsion 5. Focalfeatures
3.Neurodevelopment retardation (MR, CR development delay). 6. More than one seizures in the same day.

Ans. is 'b'i.e., Family history lRef: See above explanation)


o Among the given options, only positive famiiy history is a risk factor for recurrence.

b. Ans. is 'b' i.e., Oral Diazepan 6 hrly fRef: CPDT 1|th/e p. 726; O.P. Ghai 8'h/e p. 556-557 6 Vh/e p. 528; Nelson 18th/e
p. 24s81
Prophylaxis in febrile seizures
Prophylactic anticonvulsants are not given routinely in febrile seizures. They are required when -
i) Febrile seizure is prolonged or complicated
ii)Medical reassurance fails to relieve family anxiety.
Prophylaxis may be continuous or intermittent
lntermittent prophylaxis
o It is currently the desirableform oftherapy
a It is used during episodes of fever
o Indicated during first three days offever.
o Drugs that are used are - Diazepam and other benzodiazepenes (these drugs are used because they attain desired
lewls quickly) --> Diazepam is given oral or rectal.
Continuous prophylaxis -
o It is used when
e Intermittent therapy has failed s Recurrent atypical seizures
t Central neryous system disease a Family history of epilepsy
o Drugs used are -+ Sodiumvalproate or phenobarbitone.
Note - Carbamazepine and phenytoin are ineffective for prevention of recurrence.

;l,xi,l6&r.
MYOCLONIC TPILEPSY

.A.ns. is 'c' i-e., Mlrcdonic epilepsy lRef: O.P. Gh*i */e p. 559 & fr/e p. 531)
r Hysparrhl'thmic pattern of EEG is seen in infantile spasm (a type of myoclonic epilepsy).
8. Aa:s is'd Le,, Yigabatrin :lRef:,O.P. Ghai */e p-.559 & Ple p. sjll
t Vagabatrin is the DOC Now.

9" Aas" is'a" i"e., Myoclolric seruillr,es frequentlyocqrr ia morning [Re! Hatison lVh/e p. 2500)
o Juvenile myoclonic epilepsy is seen mostly in morning. Complete remission is uncommon, it respond well to
anticom.ulsant and is associated with absence seizure only in 1/3.d cases.
1,0- Ans. is t'i.e., Va$roate {Ref, Nelson 1,*/e p. zaffil
r Valproate is the drug of choice for myoclonic epilepsy and is required life long.

ll" A:rs" is '|t' i.e", Carbamazepine {Ref: Hawison 1*/e p 2507, 2508; trnternet)
Drugs used in myoclonic epilepsy
c Valproate (DOC) t Topiramate r Clonazepam
r Lamotrigine . Felbamate r Zonisamide

I{TONATAL SEIZURTS

lL Ans. is ',a" i.e-, Aaoxia :l&ef D"P, Ghai#/e p- S5S &'*/e p. 531; Nelson t*/e p. 212)
r The flve major causes ofneonatal seizures are -
1) Hypoxic ischemic encephalopathy (hypoxia, anoxia) --> most common 4) Meningitis
2) Hypocalcemia 5) Polycythemia
3) Hypoglycemia

13" Aas" is A'i.e., Hypoxia induced ischemic encephalopathy [eef O.2 Ghsi */e p. S5g-55g d" 7/e p. 531; C.p.D.T.
1*/ep" 621

14. Ans. is H i.e., Pyridoxin [Ref Nelsor 1*/ep.55s)


t Pyridoxine (Vitamin 86) dependency with seizures is an autosomal recessive condition.
r It is due to GABA deficiency in the brain, which is presumbly caused by decreased activity of GAD (glutamic acid
decarboxylase).
r Administration of large doses of pyridoxin usually results in a dramatic improvement of both seizures and EEG
abnormalities, as pyridoxin acts as a cofactor for GAD and improves its activity.

15. Ans. is lb' i"e", ffirmalcernia Leel A:P. ,Ghai 6"/e p. t77l
r Hypocalcemic seizures carry good prognosis.
o Idiopathic seizures have bad prognosis.

16" Ans. is t'i.e., Subtle {Ref; Internet referencQ


t Most common type of seizures is newborn are subtle siezures and they constitute 50% of all seizures.
r The are called subtle seizures because the clinical manifestation s are often mild andfrequently missed.
r Subtle seizures are usually mild paroxysmal alterations in motor behaviour or autonomic function that are not clearly
clonic, tonic or myoclonic.
tv- Ans" is "It' ie", Phenobarbitone lesf o.P Ghai */e p. 559 & */e p. 5j2; Neboa 1*/e p. 2afil
o Drug of choice for Neonatal Seizure is -+ Phenobarbitone
18. Ans" is'u- i.e." trIyponatrenia {Ref O,LGhai*lep" 559 &@/ep.50 )
Causes of status epilepticus in children
r Idiopathic r Drug intoxication
r Congenital/Developmental neurological abnormalities r Hypoglycemia
r Meningitis, Encephalitis t Hyponatremia
r Head trauma r Pyridoxin deficiency
Cseprun a Cenial Nervitui
-----------.-
Ans. is t'i"e", Focal [Ref Nelson lSthle Chapter 59i.7]
o Myoclonic seizures carry the worst prognosis in terms of neuro-developmental outcome and seizure recttrrence
e Focal clonic seizures have the best prognosis.

BSENCE SEIZURES

). Ans. is t' i.e., Fost ictal confiIsion lRej: Ila*ison l7*le p" 2499, 25A0; AP Ghai Sele p. 558j
-J welJ 'Absence seizures are characterized by sudden brief lapse of consciousness without loss of postural control. The
seizure typically lasts for only seconds, consciousness returns as suddenly as it was lost and there is no post ictal
confusion". --- Harrison
1. Ans. is 'a'i.e., Spike and dome [Rel Harrkon 17h/e p. 2499, 25AA]
c The EEG hallmark of typical absence selzure is a generalized symmetric 3 Hz spike and slow wave discharge.

)2. Ans. is'-r- i.e., Absence seizures lRef: A.P. Ghai 8e/ep. 558 & Tele p. 5301
r Absence seizure is characterized by a few seconds of impairment of conciousness without loss of postural control
There is associated eye blinking or staringwhich may occur as clustered events.

Ans. is t' i.e., Petitmal epilepsy tRef O.R Ghai *'h/e p. 558 6 Thle p. fiA\
---==--"' o Absence seizures are also called petitmal or minor epilepsy.
c Generalized tonic clonic seizures (GTCS) are also called grand mal or major epilepsy.
c Complex partial seizures are also called temporal lobe epilepsy or Psychomotor epilepsy.
* Simple partial seizures are also called cortical focal epilepsy.

TREATMENT OF SEIZURES
C.PD.Z
24. Ans. is ? i.e., Ethosuximide lRel O.P; Ghai Sthle p. 559,6 7h/e p. 5j2l

,.;;
c Absence seizure c Valproate * Lamotrigine
acid r o
Ethosuximide Clonazepam

: :EG
25. Ans. is'b'i.e., Carbamazepine [R{ GhaiTh/ep. 53)
* DOC for partial seizure -+ carbamazepine / oxcarbazepine
26. Ans. is 'a' i.e., Lorazeparn lRef O.P. Ghai #h/e p. 559 & Vhle p. 5251
c Drug used are )Lorazepam, Clonazepam, Diazepam, Phenltoin, Phenobarbitone, Thiopentol, Propofol, Midazolam.
e Lorazepam i.v. is now the preferred initial choice.

77 Ans" is t' i.e., Sodium valproate fRef: Harrison lVhle p. 2507, 25A8; Leurence */e p. 416\
Petit mal seizures are synonymous with Absence seizure
Now DOC for absence seizure is valproate (previously it was Ethosuxamide).

CNS INFECTIONS

LO. Ans" is 'b' i.e., 50-80 rnrn of HrO fRef; Nelsan t9'hle p. ch. 6a2 ftable 1))
e Normal CSF pressure in children is 50-80 mm HrO (5-8 cm HrO).

BACTERIAL MENINGITIS

29. Ans. is 'b' i.e., Acute bacterial meningitis [Ref O.g Ghei Sele p. 564 & Vhle p. 5i6]
c Three important signs of acute bacterial meningitis are Kernig's sign, Brudzinski sign and Tache cerebrale.
30. Ans. is t' i.e., Poor feedingfRef O.P. Ghai 9th/e p. 56j 6 Vh/e p. 517, 537; Nelson 18th/e p. 25151
r Among the given options, poor feeding is a common symptom.
o Neck rigidity and Kernig\ sign are seldom prominent
o Anterior fontanel may or may not be bulging
31. Ans. is'b'i.e., Seizure lRef: A.P, Ghai 9th/e p. 561 6 7h/e p. 536-5i7; Nelson 18'h/e p. 25161
o Seizure is the commonest complication of meningitis in children.
32. Ans. is'd i.e., ABER lRef: O.P. Ghai 8th/e p. 564 6 7h/e p. 5j7)
o AfterH.influenzameningitissensorineuralhearinglossoccursin5-10%of patientsduringlongtermfollowup.
o Therefore Auditory Brain Stem Evoked Response is done to check the peripheral auditory system.

JJ. Ans. is'd i.e., BERA fRef: See above explanation)


o Auditory Brainstem evoked response (ABER), also called Brainstem evoked response audiometry (BERA) should be
done.

NEONATAL MENIN6ITIS

34. Ans. is t' i.e., Streptococcus Agalactiae fRef Nelson 18th/e p. 2513, 79n
r Group B streptococcus (strep agalactiae) is the most common cause of neonatal meningitis.
J:t. Ans. is 'b" i.e", Streptococcus Agalactiae fRef: Nelson 18th/e p. 25131
'Group B streptococci are the commonest gram positive agents responsible for eaily onset neonatal pyogenic
infections and are usually clinically apparent within the first 24 hours of life'. Nelson
t Presence of hemolytic colonies on blood agar
-
(p hemolysis) and gram positive cocci in smezrs is almost diagnostic
of Group B hemolytic streptococcii infection (streptococcus agalactiae).

36. Ans. is 'H i.e., E. coli lRef : O.P. Ghai 8th/e p. 56j 6 7h/e p. 536; Nelson 18h/e p. 25131
"Group B streptococcus followed by E.coli are the two most common causes of neonatal meningitis".
Nelson lSthle 2513
So,
-
Most common cquse of neonatal meningitis -+ Group B streptococcus (Str. agalactiae)
Second most common cause of neonatal meningitis -+ E.coli

37. Ans. is t' i.e., N. Meningitides [Rel O.P. Ghai 8th/e p. 563 b Vh/e p. 5j6l
Causes of neonatal meningitis
r Group B-streptococcus (str, agalactiae) r Listeria monocfogens r Streptococcus fecalis
t E.coli r Staphylococcus aureus
r Klebsiella r Coagulase negative staphylococci

CAUSES OF NEONATAL MENINGITIS

38. Ans. is t'i.e., Streptococcus pneumoniae lRef: CPDT 18th/e p. 1089, 26231

Commonest causes of meningitis


e Neonatal -) Group B streptococcus most common, E.coli second most common.
o 2 months to 3 years -) Pneumococci > Meningococci > H. inJluenzae
o 3 years to 20 years -) Meningococcus
o > 20 years -+ Pneumococci

39. Ans is'b'i.e., H. influenzalRef: See above explanationl


c Amongst the given options, H. influenzae is the best answer because pneumococcus and meningococci are not there
in options.

40. Ans. is'U i.e., N. meningitides [Rel Previous explanationsl


4t. Ans. is 'd i.e., Streptococcus pneumoniae lRel Previous explanations)
Csapren B Central Nervous System
na' nd'
Ans, is i,e., E. eoli; i.e", StreptococeaX aganactiae [R.ef: I{etsore }"&,hle p. 2Sl3l
* Group B streptococcus followed by E.coli are the two most common causes of neonatal meningitis._ Nelscn
Note -
* For exams like AIIMS & AI, where only one options is correct, streptococcus agalactiae (group B streptococcus)
is best answer.

.
R EATTMEN{T OF BACTEM'AL MEr{'&!6IT'5

:-a Ans. is ne'i"e", Third gemeration eephalospeirins fRef: Nelson LEh/e p. 2519; Nwrrison l7th/e p. 26261
* DOC for H. influenzae meningitis Cs -> Third generation cephalosporin.
4, Ans" is 'e' i.e", Fenicillire {Ref: {}"P" Ghai I'h/e p. 565 d* vt'/e p. s3g; tr{s.rcisow lrhle p. 2626}
a The clinical presentation of fever, altered sensorium and purpuric rashes, is highly suggestive of meningiococcal
meningltis.
o Purpuric rash in quite characteristic of meningococcemia.
a I.V. Penicillin is the treatment of choice for meningioccal inf. among the given options.
* The DOC for meningiococcal infection is
x IIIrd generation cephalosporin e.g. Cefotoxime, Ceftriaxone.
+f, Ans. is t' i"e., l0 days fRet': Ghai Vh/e p. 539; Nelson t9,h/e ch. 6A2]
* Child with bacterial meningitis shows improvement in 10 days and rarely necessary beyond l4 days.

TU BEMEU !-AM METS I &'GIT'5

16. Ans, is na'i.e., Mycobaeteriurar Tuhereulosis flTef: Nelsan igthle p. 124g, l24g)
* Insiduous / gradual onset offever over a duration of two weeks, with features of raised intracranial tension in the
form of vomiting and altered sensorium with characteristic basal exudates and hydrocephalus on cranial CT
scan are almost characteristic of tuberculous meningitis.
od'
47. Ans" is i.e., Farkinsonism IRe,f O"F. Ghsi Vth/e p. 567 & Vh/e p. S4A)
* Parkinsonism is not a complication of tubercular meningitis.

18" Ans. is'tr'i"e", tr ow saagar + High protein, and Lynephoeytosis {Ref Has been explainecll

CSF im TB memingitis
e Low sugar * Predominant cel1s -+ Lymphocltes * Opening pressure is elevated
* High protein e Very low chloride

49. Ans. is 'd' i.e", Tuhererdous meeningiti s fRef: Nelson 17t'/e p. 2a39, See previous cluestions also)
o Typical signs of meningitis (Fever, convulsion and neck rigidity) with increased CSF protein and decreased sugar
level and decreased chloride level associated with lymphadenopathy suggest the diagnosis of tubercular meningitis.

E'Ve €pHA!-t?"t85

50. A*rs. is 'd' i,e.o Enterovirus {Ref: Nelson 18th/e p. 2521; E"F. Gkai Eh/e p. 56g}
"Enteroviruses are the most common cause of meningoencephalitis,, ------ Irlelson.

@&ANt\! YW&1&m5
<1 A.ns" is 'a'i.e., &dostly is infra-tentorial [Re/ Nelsom x.gtt'/e p. 2129]
o Brain tumors are the second most common malignancy of childhood next only to leukemia.
* Otter two-thirds of brain tumors are infratentoterial.
o Most of these tumor occur near the midline -) commonly obstruct CSF circulation and cause hydrocephalus early
in disease. These can also cause t ICt and papilledema (In infants papilledema may be absent because of open
s uture s and b ulging fo ntanelle ).
ffi{ffi ii: i:i:i;i:;, ;,, :::;*:.:' :' :n
:

52. Ans. is b' i.e., Glioma {Ref Nelson t#/e p. 2129-21331


t The most common Intra-cranial tumors in children are Astrocytomas (Cerebellar),
t As astrocytomas are 'Gliomas', the answer of choice here is ,Glioma,
53. Ans. is'€ i.e.,Astrocytorna {Ref: Netson l*/e p.2129-21331
r Cerebellar astrocl'toma is the most common posterior fossa tumor and has the best prognosis - Nelson
54. Ans. is t'i.e., Cerebellar Astrocytoma{Ref: Nelson 16h/e p. 1B6t)
Cerebellar astrocytoma is the most common posterior fossa tumor and has the best prognosis - Nelson
Worst prognosis is seen with -+ Brain-stem glioma.

55. Ans, is'U i.e., Astrocytoma (pilocytic)


r Astrocl'toma (pilocl'tic) are most common posterior cranial fossa CNS tumors in children.

55. Ans. is'U i.e., Oligodendrogliorna {Ref: Netson l?*/e, p. 2129,2133)


o Oligodendrogliomas are uncommon tumours of childhood, occur predominently in the cerebral cortex and originate
in white matter (Supratentorial).
Common childhood infratentorial post fossa tumors
1) Cerebellar astroc)toma (cystic astrocytoma) 3) Brainstem glioma
2) Medulloblastoma (Primitive neuroectodermal tumor) 4) Ependymoma
57. Ans. is {'Midline posterior f,ossa tumour tR{ O.p. Ghai 8e/e p. 571 & Vh/e p. 545; Nelson 18*/e p. 2134, 2129,
21301

"The classical triad of headache, nausea and vomiting, and papilledema is associated with midline or infratentorial
tumors", Nelson 18th/e 2129
-
58. Ans. is t' i.e., Medullablastoma lRef O.P. Ghai 8e/e p. 571 & Vh/e p. 545; Nelson 18th/e, p. 21j41
rMidline swelling arising from cerebellum in a child favour the diagnosis of medulloblastoma.
tRobbin\ states
r "In children medulloblastomas are located in midline but in adults they are found in lateral locations'l
Note -
. Astrocltoma is also a posterior fossa tumor, but it does not commonly present as midline mass.
59. Ans. is ? i.e., These tumors are more common in females [Ref. O.P. Ghai Bth/e p. 571 6 Zh/e p. 545; Nelson lgth/e
p.2130,213s1
o Cerebellar astrocltomas do not show any clear gender predilection and are equally common in both males & females.
r Lowgrade astrocltomas are more common in children than adults. The predominant group of astrocytomas is childhood
are low grade astrocytoma.
o cerebellar astrocltomas develop most commonly during first two decades of life.
o Most common type of astrocl'toma in children is fuvenile pilocytic astrocltoma, which has very good prognosis.

CRANIOPHARYNGIOMA

50. Ans. is 'd i,e,, Craniopharyngioma [Ref: O.p. Ghai B*/e p. 572 6 7/e p. 546; Nelson 18e/e p. 2135]
r Most common supratentorial tumours in children is - craniopharyngioma.

51, Ans. is'H i.e., Craniopharyngioma {Ref: O.p. Ghai B*/e p. 522 6 Vh/e p. 546; Nelson tyn/e p. 2135)
o craniophyngioma is the most common supratentorial tumor in children,
r It is a suprasellar tumor which presents as cystic mass.
62. Ans. is 'a' i.e., Craniopharyngioma {Ref: Sutton */e p. 1609; O.P. Ghai 8h/e p. 572 6 Vh/e p. 546; Nelson 18th/e p.
21 3s1
"Presence of Suprasellar
midline calcification immediately suggests the diagnosis of craniopharyngioma".
Craniopharyngioma is the most common cause of suprasellar calcification.
Crrn rr,rBa I
CEREBRAL PATSY

63. Ans. is'a' i.e., Spastic diplegia lRef Nelson l$hle ch. 598)
Involvement of limbs
o In spastic diplegia -+ Legs > Arms.
c In spastic hemiplegia and spastic quadriplegia -+ Arms > legs.
64. Ans. is 'd i.e., Diplegia is weakness in upper arm more than leg. lRef: Netson l&hle ch. 59a|

o In spastic diplegia, legs are involved more than arms. Other options are correct.

65. Ans. is 'a' i.e., Spastic quadriplegialRef: Tachdiian Pediatric orthopaedics p. 397\
"scoliosis is common, occuring in 15 to 25 percent of the total body CP (spastic quadriplegia CP)" Tachdjian
-
66, Ans. is 'b' i"e., Microcephaly lRef :O.P. Ghai S'hle p. 582 6 7h/e p. 559-56A1
o Hypotonia and flaccid paraiysis is seen in hlpotonic (atonic) CP. Ataxia is seen in ataxic CP.

67. Ans. is'd' i.e., Unilateral lRef: Nelson l$hle Chapter 598,99.5\
r Extrapyramidal cerebral palsy secondary to acute intrapartum near-total asphlxia is associated with bilaterally
symmetric lesions in the posterior putamen and ventrolateral thalamus.
c These lesions appear to be the correlate of the neuropathologic lesion called status marmoratus in the basal ganglia.
o These are present in fuli-term infants of basal nucleus lesions resulting from acute total asphpria.
o The lesions have a marbled appearance caused by neuronal loss and an overgrowth of myelin in the putamen, caudate,
and thalamus.

68. Ans. is 'a' i.e., Spastic diplegia lRef: O.P. Ghai 9th/e p. 582 dt Vh/e p. 5591

o Spastic diplegia is commoner in preterm babies and is associated with periventricular leukomalacia.

NEURAL TUBE DEFECT

69. Ans. is 'b' i.e., Folic Acid lAeT: O"e. Ghai 8th/e p. 576 6 Vh/e p. 5511
o Maternal malnutrition is an important risk factor for development of NTD. There is d.ecreased maternal folate levels
in NTD affected pregnancies -+ Periconceptionalfolic acid supplementation decreases the occurtence of NTD.
70. Ans. is'b' i.e., Folic acid lRef: Nelson 18'h/e p. 96.81
o The figure in question is showing neural tube defect. Which is prevented by folic acid.
71. Ans. is 'a'i.e., Open Negral Tube Defect lRef O.P. Ghai Sthle p. 576 & 7h/e p. 551; Nelson l8tule p. 244j]
o In intra embryonic life, neural tube is open at both end and freely communicate with amniotic cavity. Failure of closure
of neural tube results in persistant of this communication. This allows excretion of following fetal substances into
amniotic cavity -
l) Alpha - fetoProtein
2) Acetylcholinesterase
o These serve as biochemical marmkers for NTDs for prenatal diagnosis.

72. Ans. is 'd' i.e., Holoprosencephaly lRef: a.P. Ghai 8th/e p. 576 (t Vh/e p. 550; Nelson 18'h/e p. 244j1
Holoprosencephaty is a developmental disorder of the brain that results from defective cleavage of the
prosencel)halon. lt is thus a malfurmation of cortical development and not a neural tube defect.
73" Ans is'c'i.e., Vascular Lesions due to degenerative vessel disease and Head Injury fRef: Nelsan 18'h/e p. 2449)
porencephaly Refers to the presence of cysts or cavities within the brain, that may results from
1. Developmental defects (True porencephalic cysts).
2. Acquired iesions of infarction of tissues. (i' e. Vascular lesions) AV malformation or infarcts or (Pseudo-
porencephalic cysts).

74. Ans. is'b' i.e., Cerebral infarction fRef: See above explanationl
75. Ans. is 'a' i.e., Spasticity of the Lower Limbs is seen lRef a.p. Ghai I'h/e p.
576 dz vh/e p. 5511
o The neural tissue may be the spinal cord or the Cauda equina.
t In a sacrql meningomyelocele, howeuer, the neural tissie involved would be the cauda ecluina,
the inuolvement oi
which would results in a lower neuron picture and not an upper motor
neuron picture (ie. as in the inyolvement
of spinal cord)' spasticity would thus not be a
feature of-iacral Meningomyelocele. spasticity is a feature oJ
u'M'N' lesion and would thus be a feature of meningomyelocele occurriig higher
up, ii the lumbar as dorsal
region, where neural tissue involvement is that of spinil cord. (l,{ote:
Cauda equina are nothing but spinal
nerves which haue left the spinal cord' but yet have to leaie the vertebral
column, ie. the) are lower motor neurons)
a Bladder & Bowel incontinence may be seen.
a Hydrocephalzs is an important and frequently
associated finding with meningomyelocele.

/6. Ans is 'a' i.e., Blood-culture and sensitivity [Ref: Pediatric nursing care plans 4th/e p. 44g]
o Most cases of meningocele are recommended for treatment as
soon after birth as possible.
o In the case of a sac which is leaking fluid, the treatment is most
urgent.
o Generally, treatment includes a surgical procedure to close and .ei-ro,re
the soft tissue covering of back.
o As there are chances of bacterial meningitis and sepsis due to
csF leak, csF and blood saipte should be send
culture and sensitivity andperi-operative antibiotics (broad spectrum) can for
be changea to trr. ,p..lfic antibiotics after
the blood and CCF culture and sensitivity report.

77. Ans. is 'd i.e., Congenital anomaly lRef Nelson lSth/e p. 2452; Op Ghai Bth/e p. 5741

Non-communicating Hydrocephalus Communicating Hydrocephalus


Obstruction occurs within Ventricular svstem Obliteration of Subarachnoid cisterns or malfuncion
{ of arachnoid villi
Most common cause in children is Abnormality o Most common cause in children rs Subarachnoid
of aqueduct - Acqeductal stenosis (Congenital Hemorrhage which is usually a result of intraventricular
anomaly) hemorrhage in a premature infant. Blood in subarach_
noid space causes obliteration ofcisterns or arach-
noid vi1li.
o Pneumococcal and Tuberculous meningitis are other
causes and block the basal cisterns by thick tenacious
exudates.

78. Ans. is t' i.e., Aqueductal stenosis lRef Nekon 18th/e p. 24521
congenital hydrocephalus is most commonly of obstructive or non communicating
type. Abnormalities of the Aqueduct
(Aqueduct stenosis) are the most common cause of obstructive hydrocephalus.
Aclueductal stenosis therefore can be termed as the most ,o**i, cous"
of congenital hydrocephalus.
As obstructive hydrocephalus are more common cause for conginitat hydroiephalus than non - obstructiye
hydrocephalus, aqueductal stenosis is the single best answer.

'7() Ans. is 'a'i.e., Post inflammatory obstruction lRef: Read text berow)
r MC cause of obstructive (non-communicating) hydrocephalus --> aqueductal stenosis.
r MC cause of non-obstructive (communicating) hydrocephalus -) subarachanoid hemorrhage.
r But, amongst the given options best answer is option .a1

'A common cause of acquired hydrocepahalus, is post inflammatory obstruction.


Bacterial and tuberculous
meningitis haw a propensity to produce thick tenacious exudate that obstructs the basal
cisterns',.

80. Ans. is 'd' i.e., All of above lRef: Nelson l8,h/e ch, 592.1)
o Non-obstructive(communicating)hydrocephalusisduetoobliterationofsubarachnoidbasalcisternsormalfunction
of arachnoid villi. There is enlargement of all ventricular cavities aswell
as subarachnoid space.
75. Ans. is'a' i.e., Spasticity of the Lower Limbs is seen [Ref: o.p. Ghai B,h/e p. 57d 6 Th/e p. 551)
o The neural tissue may be the spinal cord or the Cauda equina.
e In a sacral meningomyelocele, however, the neural tissue inuolved
would be the cauda equina, the involvement of
which would results in a lower neuron picture anrJ not an upper motor neuron
picture (ie. as in the inyolvement
of spinal cord). spasticity would thus not be a
feature of-iaual Meningomyelocele. spasticity is a feature of
u'M'N' lesion and would thus be a feature of meningomyelocele occurriig higher up, ii the
lumbar as dorsal
region, where neural tissue involvement is thot of spinal cord. (l{ote: C"aud"a
,quiro are nothing but spinal
nerves which have left the spinal cord, but yet have to leave the vertebral
column, ie. the) are lower motor neurons)
o Bladder dz Bowel incontinence may be seen.
t Hydrocephalus is an important and frequently associated finding with meningomyelocele.

76. Ans is 'a' i.e., tslood-culture and sensitivity lRef Pediatric nursing care plans 4th/e p. 44Bl
e Most cases of meningocele are recommended for treatment as soon
after birth as possible.
r In the case of a sac which is leaking fluid, the treatment is most urgent.
o Generally, treatment includes a surgical procedure to close and rerirove
the soft tissue covering of back.
c As there are chances of bacterial meningitis and sepsis due to CSF leak,
CSF and blood ,rnipte shoulil be send
culture and sensitirtity andperi-operative antibiotics (broad spectrum) can be changed for
to the specific antibiotics after
the blood and CCF culture and sensitivity report.

HYDROCEPI.IALUS

77. Ans. is 'a' i.e., Congenital anomaly lRef: Nelson lgth/e p. 2452; Op Ghai gth/e p. 574]

Non-communicating Hydrocephalus Communicating Hydrocephalus


Obstruction occurs within Ventricular svstem Obliteration of Subarachnoid cisterns or malfuncion
* ofarachnoid villi
Most common cause in children is Abnormality o Most common cause in children is Subarachnoid
of aqueduct - Acqeductal stenosis (Congenital Hemorrhage which is usually a result of intraventricular
anomaly) hemorrhage in a premature infant. Blood in subarach-
noid space causes obliteration ofcisterns or arach-
noid vi1li.
c Pneumococcal and Tuberculous meningitis are other
causes and block the basal cisterns by thick tenacious
exudates.

/o. Ans. is t' i.e., Aqueductal stenosis [Ref Nelson 18th/e p. 2452]
congenital hydrocephalus is most commonly of obstructive or non communicating type.
Abnormalities of the Aqueduct
(Aqueduct stenosis) are the most common cause of obstructive hydrocephalus.
Aqueductal stenosis therefore can be termed as the most ,o**i, crus, of ,ongenital
hydrocephalus.
As obstructive hydrocephalus are more common cause for congenitat hyaroiephalus than non - obstructive
hydrocephalus, aqueductal stenosis is the single best answer.

79. Ans. is'a'i.e., Post inflammatory obstruction [Ref Read text berow)
o MC cause of obstructive (non-communicating) hydrocephalus -+
aqueductal stenosis.
o MC cause of non-obstructive (communicating) hydrocephalus -)
subarachanoid hemorrhage.
e But, amongst the given options best answer is option hi
'A common cause of acquired hydrocepahalus, is post inflammatory obstruction. Bacterial
and tuberculous
meningitis have a propensity to produce thick tenacious exudate that obstructs the basal cisterns,,.

80. Ans. is'd'i.e., All of above lRef: Nelson lgth/e ch. 592.1)
r Non-obstructive(communicating) hydrocephalusisduetoob/iterationofsubarachnoidbasalcisternsormalfunction
ofarachnoidvilli,Thereis enlargement ofallventricular cavitiesas well as subarachnoid
space.
Cnep rsn 8, , Cenfral ilbrV:ousrffs{stti

Ans. is t' i.e., Crack pot sign; ?' i.e,, Depressed fontanell efRef:O.P. Ghai 8'h/e p. 574 6 7h/e p. 549;Nekon 18't'/e p. 2453]
e "Cracked Ttot or macewan sign indicates raised intracranial pressure afier sutures and fontanels have closed"
o Fontanelle is wide and bulging (not depressed)
Ans" is t' i.e., Aqueductal stenosis {Ref: t{illiaw's 22n/e Ckapter 16; Nelsan 18th/e Chapter 592.11)
* Hydrocephalus is characterized by an increased CSF volume with progressive ventricular dilatation (ventriculomegaly).
* Obstructive or noncommunicating hydrocephalus/ventriculomegaly develops most commonly in children because
of an abnormality of the aqueduct or a lesion in the 4th ventricle.
* Obstruction of the aqueduct is the single most common cause offetal and neonatal hydrocephalus/ventriculomegaly,
* Nonobstructiye or communicating hydrocephalus most commonly follows a subarachnoid hemorrhage, which is
usually a result of intraventricular hemorrhage in a premature infant.
83. Ans. is'c' i"e", MRI [eef {liruicsl wewrolagy 3'dle p. 318}
* Investigation of choice for hydrocephalus is MRI because is can demonstrate the extent of hydrocephalus and
uderlying cause.
84. ,{ns. is'd'i.e,, .A.ll of above [Rel Nelsow tr9't'le Chap. 2$fl
c Hydrocephalus, meningomyelocele and syringomyelia are found in chiary malformation type II.
85. Ans. is 'd'i.e., ,4,I1 are true {Ref: Nelson 18th/e ch, 5921
a It is charact erizedby hypoplasia or absence of cerebellar vermis with cystic dilatation of 4th ventricle in posterior
fossa. There is hydrocephalus, agenesis of corpus callosum and increased head size with prominent occiput.
s6. "Ans. is t' i.e., Cerebral eortex are deficieney/hypoplastic in hyrlranencephaly {Ref: Nelsan 18'h/e ch. 592.11)

HydramancephaNy is & rars en(ephalopathy that oecurs im-uteno"


* Affected infants may have a normal or enlarged head circumference at birth that grows at an excessive rate postnatally.
o Transillumination shows an absence of the cerebral hemispheres.

PS E["'B8T[-N&4OR CHMEMR'

37, Ans, is'b'i.e., CT scan sleows punctete hypodense areas ["&rf CPWT X.8t1'/e p. f4A; 0.P"Gkwi 9'h/e p" 575 6
Vhle p" 55A; NelsoYt l*tkle P. 25251
* There is raisedlCT,normai ventr icular size, normalCT scan and no focal neurological deficit in pseu dotumor cerebri.

:3, Ans" is'a' i"e., Salicylic eci&l*ef: CPWT tr8'h/e p" 741; Nelsan tr&il'le p. 25251

* Salicylates do not cause pseudotumor cerebri. Tetracycline, nitrofurantoin and nalidixic acid can cause pseudotumor cerebri.

ryIKNYAL mFY&mmpr'0rx
89. Ans. is'b' i.e., 60 tRe"f 0.P. Ghai I'hle p. 584 & 7t'le p. 5621

The lntelligence Quotient (f.Q.) is calculated according to the formula :

Mental age
IQ= x 100
Chronological age
q
lQofthischild=' -x 100=60
15

90, Ans. is 'a' i.e., l\{i}d rnenta} retardation [l]'ef: A.P. Gkai 8'h/e p" 584 b Vhle p' 562)
Classifi eation of nnental netardation
o A child with lQ< 70 is called as mentally retarded.

51-70
36-50
21-35

r IQ level between 71-90 is designated as bordqrline intelligence.


o Down's syndrome is the most common genetic cause of mental retardation, and fragile-X is second to Down's.
Also know
o Most common inherited cause of mental retardation is fragile-X-syndrome (because down's syndrome is congenital
cause but not inherited).

92. Ans. is'a'i.e., Hypothyroidism [Rel O.P. Ghai 9th/e p. 585 6 Vh/e p. 562]
o Adequate care for prevention of hypothyoid state can prevent mental retardation in hypothyroidism.
r Down's syndrome and cerebral palsy are congenital causes of mental retardation and prevention of mental retardation
would be possible only by prvention of the syndrome itself.

93, Ans. is t' i.e., Cretinism IR.ef: O.P. Ghai 8'h/e p. 585 6 Vh/e p. 5621
o Cretinism is due to deflciency of Iodine, which can be preventable.
94. Ans. is t' i.e., 18 year fRef: Nelsan 18'h/e p. 38.2)

Diagnostic Criteria for Mental retardationtal Retardation


a) Significantly sub-average intellectual functioningan IQ score of 70 or below on an individually administered IQ test
(t'or infants, a clinical judgment of significantly sub-average intellectual functioning).
b) Concurrent defcits or impairments in present adaptive functioning (i.e., the meeting the standards expected for l

his or her age by his or her cultural group) in at least two of the following areas: communication, self-care, home
living, social/interpersonal skills, use of community resources, self-direction, functional academic skills, work,leisure,
health, and safety.
c) The onset is before age 18 years.

REYE',S SYNPROME

95. Ans. is 'b' i.e., RSV IRel Nelson 18th/e p, 1i94, 1695; CPDT 18th/e p. 663)
o Precipitating factors for Reye's syndrome : Salicylates (aspirin) and viral infection (influenza, echovirus-2, Aenovirus,
EBV vericella (HSV-3), coxsackievirus-A)

96. Ans. is 'd' i.e., Deep jaundice is present lRe! A.P. Ghai 9'hle p. 570 6 Vh/e p. 54i; Nelson 18th/e p. 1697; CPDT 18th/e
p.663]
o faundice is infrequent in Reye's syndrome -) Bilirubin is not raised.
Q7 Ans. is'a'i.e., Reye's Syndrome lRel Nelson l9thle Chap. j58l
98. Ans. is 'a'i.e., Aminoaciduria [Ref CPDT 18th/e p. 663]
Laboratory findings of Reye's syndrome
t Ilypoglycemia o Mixed respiratory alkalosis and metabolic acidosis
o llypoglycorrhoea-+ t CSF glrrot, o On liver biopsy -
. tpt t Microvesicular fatty changes
o t scor,
sGPT, LDH t Absence of inJlammation
t Normal alkaline phosphatase t Depleted glycogen stores
o Normal bilirubin o Kidney
. Hyperammonemia t Swelling and fatty degenration of PCT.

MISCELLANEOUS
99. Ans. is 'a' i.e., Hypothalamic hamartoma lRef lournal of neuropsychiatric dis,l
Laughing spells (also know as Gelastic seizure)
o Gelastic seizures are epileptic events characterizedby bouts oflaughter. Laughter-like vocalization is usually combined
It

with facial contraction in the form of a smile. Autonomic features such as flushing, tachycardia, and altered respiration
are widely recognized.
r Gelastic seizures have been associated classically with hypothalamic hamartomas.
lital o Hlpothalamic hamartomas are rare congenital lesions presenting with the classic triad ofgelastic epilepsy, precocious
puberty and developmental delay.
e Electrophysiological, radiological, and pathophysiological studies have confirmed the intrinsic epileptogenicity of
the hypothalamic hamartoma.
c Currently the most effective surgical approach is the trancallosal anterior interforniceal approach.
1 00. Ans. is'e' i.e", ffiqys &a* gero;ws *L*fr. dec*rebrate lesion
Decortieal Posture
o Also known as flexor posturing or Mummybaby
o Arms flexed/bent over chest, hand fisted, leg extended & rotated inward
o Damage to area in cerebral hemisphere, internal capsule, thalamus & upper part of brain.
r Decorticate posture is ominous sign of severe brain damage.
Decerebrate posture
r Also known as extensor posturing
o Extension of upper limb & lower limb (ELBOW EXTENDED)
for o Indicates brain stem damage (Below level of red nucleus)
o Decerebrate posture is more ominous than decortical posture
e, I 01. Ans" is'&' i"e., 18*24 m**ths lR$ Crey Anet&rry p- 1967
e Closure of anteriorfontanel -+ 1.5 years (between 1 to 2 years).
c Closure of posterior fontanel -+ 2-4 months.

l*2. Ans. is'el i.c",3lyp*gon:adi*m lR$ Ncl*aw l*hle p.677j


o Down's syndrome, osteogenesis imperfecta and hlpothyroidism cause delayed closure of anterior fontanel (not
hypogonadism).
103, Ans. is 'b' i.e., Murps lR$ Nelsow l&hle Ch*ptcr 245, 28Vj
o Out of the given options, only mumps and toxoplasma cause aqueductal stenosis. Toxoplasma is not a virus. Hence,
the answer is mumps.
104. Ans. is h'i.e., Hyperpigmentation of extremities; 'c'i.e., Cortical atrophp t'i.e., SeIf linaiting disorder [Ref
A"P. Ghai th le p. 581 & P le p. ssSl
"Infantile tremor syndrome is a self-limiting disorder characterised by acute 6 gradual onset with neural dt
psychomotor changes, pigmentary disturbances of hair 6 skin, pallor (z tremors". - O.P Ghai 7th/e p. 558
r Pnenumoencephaly reveals cortical atroplry.CT scan shows no abnormalities or mild atrophy. - O.P. Ghai.
f 05. Ans" is'd'i.e., Coranalswtrrre lRef: A"P.Ghai.dblep.6fr2j

Dolichocepllaly br' Scaphocephaly * Premature elosure of sagittal suture.


o Skull grows perpendicular to the open coronal suture
and expands in an AP diameter.

Trigonocephaly * Premature clasure of metopic suture.


r Theforehead appears pointed and ridged.
oxvi"prr+, eir"."pfrJv, t niowement ol a& the suture*
+y@a;-r;;,1.*lil,,,r,rr*r,,,.:.::.;.ir....*j:,:.:..::*;rii;.

f06. Ans. is ? i.e., Optic nerve glioma [Ref Nelson tfle p. 2a$l
o Optic glioma is one of the diagnostic criteria for neurofibromatosis tFpe-I.
lA7 . Ans. is ? i.e., )uvenile myelomonocytic leukernia facars in oncolagt p. j3l
lRef : Hemetopaietie growth
'Among leukemias, Neurofibromatosis - 1 is associated with nonlymphocytic leukemia particularly IMML and
my elo dy splastic sy n dro m e".
108' Ans' is 'b'i'e., Tuberous screrosis rRef: o.p. Ghai Yth/e p.
586 6 vh/e p. 564-565; Nerson lvh/e p. 2017, 20ls]
o Seizures' mental retardation and adenoma
sebaceum are seen together in Tirberous sclerosis.
109' Ans' is 'b'i'e', Ttrberous sclerosis
rRef a.p. Ghai 8'h/e p. 586 6 Vh/e p. 564; Crinicar paediatric 3dre p. 5391
' ff:::L"r:tT[:t and multiple hlpopigmented macules over
the back (ash leaf patch) suggest the diagnosis
of
I10. Ans. is 'a' i.e., Sturge weber syndrome
lRef: Roolc vh/e p. 39.20]
o This child has :
i) Seizure disorder
ii) vascular plaque along the ophthalmic and
maxillary divisions of trigeminal nerve
iii) Lesion presenting since birth and not chonging
i" ;";h;i"gy
o The diagnosis is sturge weber syndrome.
II l. Ans. is t, i.e., Vein of Galen malformation
veinofgalen.co.ukl
[Rel Ne/s on lgth/e p. 19gg; www.emedicine.cam/neuro/topic53g.htm;
wwu,.
I Midline intracranial mass with bruit on ausculation
and features of hydrocephalus and congestive
All point towards Vein of Galen Malformation. heart failure -
Vein of Galen Matforrnations(VGM)
r The vein of Galen is a large deep vein at the base of
the brain. It is located under the cerebral
drains the anterior and central regions ofthe hemispheres and
brain into the sinuses ofthe posterior cerebral
r vGM results from an aneurysmil malfurmation with fossa .
an arteriovenous shuntingof blood.vGM
heart failure in the newto tn resulting usually causes
ffi|"tO"t flom the decreased resistaic) and high blood flow in the
t include cerebral ischemic changes such as sfrokes or steal phenomena that resurt in progressi'e
f.tffiT:Srfndings
r The malformation may result in mass
effects, causing
malformation may cause obstruction of the rroqlelfve neurological impairment. Alternatively, the
cerebrospr,ii nr'ia (CSF) outflow arJ *.rr,
c Loud intracranial bruit maybeheard in hydrocephalus.
becaus" of tfr. utooJ turbulence in vGM.
a
o The vein of Galen abnormality is the
most freqr"nt oiii*nous malformation inneonates.
ll2. Ans. is'b'i.e., Vitamin K[Ref: Nelson l}th/e p.
712]
"Intravenous vitamin K should
be given before deliuery to all women receiving phenobarbitone or phenytoin
during pregnancy, to prevent intraventiicular
hemorrhage,,. '
Nelson
--
ANSWERS OF VARIOU5 OTHER EXAMINATIONS

113. Ans. is'a'i.e., Febrile convursions


[Ref a.p Ghai gth/e p. 5s6 6 7/e p. s2s]
ll4, Ans. is 'd' i.e., prophylactic phenobarbitone
{Ref : o.p, Ghai g,h/e p. 5s6 6 7/e p. s2gl
I15. Ans. is t' i.e., Lamotrigin e {Ref : O,p, Ghai gth/e
p. SS5 d? Zhle p. 525; Nelsan lVh/e p. 20ABl
I 16' Ans. is t' i.e., Infantile spasm lRef : o.p, Ghai Br,/e p. 55g-s6r 6 vh/e p. 531)
o Prognosis for normal mental development
is poor.
ll7 ' Ans' is t' i'e" Posterior cranial fossa tumor
&'d' i.e., Medulloblastoma [Ref : Nelson lgtu/e p,
o Most severe papilledema 212gJ
due to increased ICT is seen in infratentorial
o Medulloblastoma is posterior fossa midline tumors.
a posterior fossa midline tumor.

I18. Ans. is 'a'i.e., Spastic [Ref : O.p, Ghai gth/e p" Sg2 & Vh/e
p. 559)
I19. Ans. is t' i.e., Sagittal suture fRef: O.p. Ghai 6th/e p. 602]
120. Ans. is 'c' i.e., papilloedem a
[Ref : O,p. Ghai gth/e p, SZ0 &. 7,/e p. 544)
l2l, Ans. is 't i.e,,Vit A [Rel Has been explained]
o It is a case of pseudotumor cerebri.
CHrt?,H,iar,8.,

122. Ans. is 'd' i.e., 25 mg% lRef: Meharban Singh 3'd/e p. 3161
r It is 15-40 mg%.

Premature

sit:.:r:
47 mga/o DayT

:i.si*+,,._-_-& =..=,.....jil1i,.;ti.i:il..i

123. Ans. is t' i.e., Dexamethasone [Rel O.P. Ghai 6th/e p. 520 6 7h/e p. 541]
r Dexamethasone is Preferred.

124. Ans. is t' i.e., Spina bifida lRef : William's Obstetrics 21"/e p. 982)

I25. Ans. is t'i.e., Cerebral abscess fRef: O.P. Ghai 8'h/e p. 573 6 Thle p. 410)
r Brain abscess is frequent complication ofcyanotic congenital heart disease, TOF.

t26. Ans. is t' i.e., MultifactoriallRef : Dutta 6'h/e p, 492; Hollend and Brew's 16th/e p. 216; Ghai 6'h/e p. 531)

127. Ans. is'b' i.e., Mumps [Ref : I{arrisan 16th/e p. 734]


Aqueductal stenosis -
Causes of acquired aqueductal stenosis
r Neonatal meningitis r Intrauterine viral infections
o Subarachnoid hemorrhage r Mumps meningoencephalitis

ITI
xvx&y,&eotrc
mxsffiKm&K

G LYCOGEN STOR4GE DTSEASE (GLYCOGENOSES)

Glycogen storage diseases result from a hereditary deficiency of one of


' the enzymes involved in the synthesis or
sequential degradation of glycogen. clinical features depend on tlpe
of enzymedeficiency and organ involvement.
o Glycogen storage diseases can be divided into:-
1) Hepatic form (Liver glycogenoses)
r Liver is the key player in glycogen metabolism. It contains enzymes that synthesize glycogen for
storage and
ultimately break it down into free glucose, which is then released into blood.
So, An inherited deficiencv of
hepatic enzymes that are invorved in grycogen metaborism reads to -
i) Storage of glycogen in liver -+ Hepatomegaly.
ii) Reduction in blood glucose -+ Hypoglycemia.
r Examples of liver glycogenoses are _

1) Type I glycogenosis -+Glucose - 6 - phosphatase deficiency (Von Gierke disease)at ts, Nnnr).
ii) Type III
glycogenosis -+ Debranching enzyme d.efetrc'Ntnrt
lCori's disease/Forbe disease/limit dextrinosis)
iii) Type IV glycogenesis -+ Branching enzyme def (Anderson's
disease).
iv) Type VI glycogenosis -+ Liver phosphorylase def (Her,s disease(NEEr)).

2) Myopathic form (Muscular glycogenoses)


r In skeletal muscle, as apposed to the liver, glycogen is used predominantly
as a source of energy. This is derived
by glycolysis, which leads ultimately to the formation of lactate. So,
enzyme deficiency leads to -
i) Glycogen deposition in muscle
ii) Muscle cramp afier exercise
iil) Exercise - induced rise in blood lactase levels owing to block in glycolysis
iv) There may be myoglobinemia
r Examples -
i) Type II glycogenosis --> Acid maltase deficiency (Pompe's d1sease)(et n,Nnmt.
ii) Type V glycogenosis -+ phosphorylase deficiency (McArdle disease)ar r;,
iii) Typ e V II gly c o gen o s i s -+ pho sphofr u ct okin ase def (Tarut s disease).

Liver olycogenoses
Type-l Glycogen storage disease (Von Gierke,s disease)
o It is caused by deficiency of glucose-6-phosphatase in liver, kidney and intestinal mucosa.
o It is an autosomal recessiye disorder.
o Type la GSD is due to deficiency ofglucose-6-phosphatase and Type lb GSD is due to defect in translocase,which
transport glucose-6-phosphatase across microsomal membrane.
o It is charact etizedby hepatomegaly with recurrent hypoglycemia which is unresponsive to glucagon or epinephrine(Al
13' e3).
Other features are seizures, renomegaly@Gl01, Arse),short stature, hyperlipidemia(Ar 8s),
hyperuricemia(Ar Be),
and
e3).
lactic acidosis@ct

Other Iiver glycogenoses


t Type III (cori's disease/furbe's disease/limit dextrinosis), Type VI (Her's disease) and Type IX (phosphorylase
kinase) alsopresentwith hepatomegaly andhypoglycemia.Buttheir hypoglycemiarespondto glucagon or epinephrine
(differentiating from Von Gierke disease). Other differentiating feature from Von Gierke's
disease is that renal
involvement is not there in these, where as renomegaly is afeature of Von Gierke's diseas/:cr01).
. Type IV (Andersons disease) is usually present with liver cirrhosis.
t One important feature in type III and type IV ilisease is that deposited glycogen has abnormal structure.

Liver glycogenoses
+

Hepatomegaly Cirrhosis
I
Y
Renomegaly No renomegaly Type IV (Anderson)
Hlpoglycemia unresponsive Hypoglycemia corrected (Abnormal glycogen
to glucagon/epinephrine by glucagon/epinephrine structure)

i
I

+
Vm Gierke's disease (\pe 1) Abnormal Normal
(Normal glycogen structure) glycogen structure glycogen strucfure
+ +
Tpe III (Cori/Forbe/limit dextrinosis) Tpe W (Hers)
Muscle glycogenoses

Pompe's Disease (Type-ll) -


o It is caused by deficiency of lysosomal a-glucosidase (acid maltase).
o There is glycogen deposition in lysosomes in many organs causing heqtatosplenomegaly(eaus or);
cardiomegaly
(hypertrophic cardiomyopathy) causing h igh voltage (talll qpgroxus 01) and short pR interval;
muscle weakness and
hypotonia, floppy infant; CHF; macroglossia, and course ot)
facissanus .
o It is the only glycogen storage disease which is a lysosomal storage disorder(NEED.

McArdleb (Type-V) and Tarui (Type-V[) disease


o McArdle's disease is due to deficiency of myophosporylase and Tarui disease is due
to deficiencv of
phosphofructokinase.
..
.: ;,. .,....1,..:,...1re,I{.Xre.it*111.

r Both present with exercise intolerance with muscle cramp(Pct "), myoglobinemia, myoglobinuria
PGt e8)
rhab domY olY sis( .

GALACTOSEMIA AND FRUCTOSE INTOLERANCE

Galactosemia
o Galactosemia is a group of metabolic disorders which occurs due to defect in metabolism of galactose.
o There is inability to metabolise galactose which may be caused by deficiency of any of the following enzymes:
r) Galactose-l-phosphate uridyl transferase (most common)@Gt 01' AilMS eB).

ii) UDP-galactose-4-epimerase(PG 01)

01 )
111) Galactokinase(Pcl

o The defect in galactose metabolism results in accumulation of galactose-L-phosphate and galactical that are thought
to have direct toxic effect on the liver and other organs.

Clinical features
r Infant appears normal at birth, however, becomes symptomatic a few days to weeks after initiating galactose containing
feeds. Due to accumulation of galacticol, damage to nerve tissue and liver occurs which causes:-
t Hepatic parenchymal disease- jaundice, hepatomegaly and ascites@IlMse&). t Acidosis
t Mental retardation(PGl 01 ) t Seizures
: Vitreous hemorrhage : Splenomegaly
s Cataract : Hypoglycemia
o The accompanying GI symptoms are poor feedings, vomiting and diarrhoea.
o Fanconi syndrome (Aminoacidiuria, proteinuria, phosphaturia) may occur.
t There may be sepsisuttMseg),
c Glucagon has no effect on hypoglycemia of galactosemiaailMs 04)
.

Diagnosis
o Urine shows reducing sugarwhich can be detected by?uttstol
i) Benedicts reagentqllMs 10)

ii) Glucose oxidase method


r The confirmatory diagnosis is made by the enzyme assay.

Hereditary fructose intolerance


o It is due to deficiency of enzyme aldolase-B (fructose-1-phosphate alilolase){utus 00).
This results in accumulation of
fr u cto s e- I - ph o sphat e.

o After intake of fructose (e.g. sugar cance juicearlMs0t)) the child presents withhypoglycerniaGltMs 'r), jaundice,
hepatomegaly^ilMs0l), hemorrhage, vomiting andfructose (reducing sugar) in urine(urtrsott.
9lsoRDERS OF METABOLTSM OF pHENYLALAN|NE

Phenylalanine Phenylpyrovic acid phenyllactic acid

Alhinism 1
I phenylalanine (_) I
-t
I hydrotylase <- Phenvleketonuria
l,-r t J
I

DOPA
l
Tyrosinase ' ,
Phenylacetic acid
I
+ I
Transaminase
Melanine I J
p-hydroxy phenylpyruvic acid Phenylacetyl glutamine

I p-hydroxyphenyl (-)
Tlrosinosis
It
I
pyruvrc ondase

Homogentisic acid

It Honogentisic (-)
.. .. <l- Alkaptonuria
iI
aud oxttl$e
Maleyl - acetoacetic acid

t
I

Fumaric acid ] dsslongetic acid

Phenylketonuria
r It is an autosomal recessive disorder due to deficiency of phenylalanine hydroxylassrarrr;.
o As a result phenylalanine is not metabolizedby hydroxylase, and metabolism
is shifted to alternative pathlway and
there is increased concentration of phenylalanine, phenylpyruyateal\o), phenylacetate
and phenyl-lactate.
r Because phenylalanine is not converted into tyrosine, tyrosine becomes an essential amino acidat 10).
o Classial phenylketonuria is due to deficiency of phenytalanine hydroxylasefaiio,. Milder form
may be caused by def -
ciencyofdihydrobiopterln reductase that produces tetrahydrobiopterin,a cofactorfor phenylalaninehydroxylase.

Clinical presentation
o The babies arc normal at birth but may present with vomiting. Gradually
mental retrardation6r07) andgrowth
retardation develop. Baby ha slight complexion withblue iris1llT).Other feature s arc
microcephaly(Arr7),rash, hlpertonia,
seizures, exaggerated tendon reflex(uesc o<) , wide spaced teeth, enamel hypoplasia(upsc oq)
andhlperactivity.
o There is musty or mousy sf,6svNnr7 'uee) of urine and other body secretions
due to presence of phenylketones.
r Pregnant females with increased pheynlalanine (maternal phenylketonuria) may cause mental retardation,
mi c r o c eph aly,' gro w th r et ar d ati o n an d CHD s in b ab iesLilM s e s)
.

Diagnosis
r Elevated phenylalanine levels
o Elevated blood tyrosine level
o Presence of urinary metabolites of phenylalanine
o Guthrie's Test -+ It detects the presence of phenylalanine in serum
o FeCl. -> lt detects the presence of phenylalanine in urine. FeCI, is added to
patients urine. If it contains phenylalanine,
it will turn green&Enr).
c 2-4Dinitrophenol hydrazine -+ gives yellow precipitate with old urine.

Treatment
t Administration of low phenylalanine is the mainstay of treatment (but phenylalanine should not be completely
restricted
as it is necessary for growthat00)). Tetrahydrobiopterin can be used in milder form.
Alkaptonuria
o It is an autosomal recessive disorder. It is due to deficiency of enzyme homogentisic oxiilase. Large amount of
homogentisic acid is accumulated in body and excreted in urine.

Clinical manifestations
r Clinical features consist of -
i) Ochronosis -+ Dark spot on the sclera or ear cartilage.
ii) Arthritis -+ Spine, hip, knee.
r But both these occur in adulthood.
t The only sign of disease in children is a blackening of urine on standingluoz). This is caused by oxidation and
polymerization of homogentisic acid.
r High incidences of heart disease (mitral and aortic valr,rrlitis, calcification of heart valves, MI) have been noted.

ORGANIC ACIDURIA (ORGANIC ACEDIMIA)

o Inthedegradationo fessentialbranchedchaina*)ooriar(Valine,leucine,isoleucine),theintermediatemetabolites
are all organic acids.
r Thus deficiency of any enzyme (except transaminase) in the metabolism of these amino acids cause accumulation of
organic acidbefore the enzlrmatic block.
r Therefore, these disorders are called organic acedimia/organic aciduria.
r Important types of organic acidemia are :-
i) Methylmalonicacidemia
ii) Propionic acidemia
i11) Isoavleric acidemia
iv) Maple syrup urine disease
v) Qombined (multiple) carboxylase deficiency
vi) Glutaric acidemia
vii) Ketothiolas e deficiency

o Most of the clinical features of these disorder are same and include : refusal to feed, v otnitinglArtMs0o), neurological damage
(mental retardation, seizuresarlMs00), lethargy), metabolic acidosis (lactic acidosisarlMs00)),hyperammonemiaurrMs0o)
developmental delay, neutropenia and hypoglycemia.
o However, some manifestations are specific and help in the diagnosis.

Features of organic aciduria (see above)

No skin manifestations Skin rash (exfoliation) Normal Amonia level [I]peramfionemia


o Propionic acidemia Multiple curboxylase + +
/ilMS 99)
o Methyl melonic acidemia clelrctency Acyl-CoA dehydrogenas e Urea qtcel defea
o Ketothiolas deficiency
3 -hydro4t 3 -methyl glutaric

aciduria (glutaric acidemia)

Note :- (Irea qtcel m4tme defects also present with features of organic aciduria, but there is llyperammonemia wilhout
. kaoacidosis and there is no lactic acidosis.
Alkaplenuna
r It is an autosomal recessive disorder. It is due to deficiency of enzyme homogentisic oxidase, Large amount of
homogentisic acid is accwulated in body and excreted in urine.

Clinlcal manifestations
r Clinical features consist of -

i) Ochronosis -) Dark spot on the sclera or ear cartilage.


ii) Arthritis -+ Spine, hip, knee.
r But both these occur in adulthood.
t'Ihe only sign of disease in children is a blackening of urine on standing!A107). This is caused by oxidation and
polymerization of homogentisic acid.
r High incidences of heart disease (mitral and aortic vah,'ulitis, calcification of heart valves, MI) have been noted.

ORGANIC ACIDURIA (ORGAN!C ACEDIMIA)

r In the degradation of essentialbranched chain aminoacids (Valine,leucine, isoleucine),the intermeciiate metabolites


are all organic acids.
r Thus deficiency of any enzyme (except transaminase) in the metabolism of these amino acids cause accumulation of
organic acidbeiore the enzymaiic block.
r Therefore, these disorders are called organic acedimia/organic aciiluria.
r Important types of organic acidemia are :-
n Methylmalonic acidemia
ii) Propionicacirlemia
111) lsoavleric acidemia
iv) Maple syrup urine disease
v) Combined (muhiple) carboxylase deficiency
vi) Glutaric acidemia
v11) Ketothiolase ileficiency
r Mostoftheclinicalfeaturesofthesedisorderaresameandinclude:refusaltofeed,vomitinglArtMs"),neurologicaldamage
(mental retardation, seizures6"*so0), lethargy), metabolic acidosis (lactic acidosisartMs00)),hyperammonemiaailMs00)
developmental cielay, neutropenia and hypo giycemia.
r However, some manifestations are specific and help in the diagnosis.

Features of organic aciduria (see above)

K eto a ci do s is an d hyperamm on emm No Ketoacidosis

No skin manifestations Skin rash (exfoliation) Normal Amonia level Hyperammonemia


o Propionic acidemia Multiple carboxylase Y
I
J
/aI^ts99)
o Methyl melonic acidemia clelrcrcncy Acyl-CoA dehydrogenas e Urea cycel deJbct
o Ketothiolas
deJiciency
3 -hyd rory 3-me r hyl glu ruric
aciduria (glutaric acidemia)

Note :- Urea qtcel enqtme defects ulso present with features of organic aciduriu, but there is Hyperammonemia without
ketoacidosis und there is no lactic acidosis.
Crlrprrii
Alkaptonuria
r It is an autosomal recessive disorder. It is due to deflciency of enzyme homogentisic oxidase. Large amount of
homogentisic etcid is accumulated in body and excreted in urine.

Clinicai manifestations
r Clinical features consist of -
i) Ochronosis -) Dark spot on the sclera or ear cartilage.
ii) Arthritis ) Spine, hip, knee.
r But both these occur in adulthood.
o The only sign af disease in children is a blackening of urine on standing!'ar 07). This is caused by oxidation and
polymerization of homoqentisic acid.
r High incidences of heart disease (mitral and aortic valvulitis, calcification of heart valves, MI) have been noted.

ORGANIC ACIDURIA {ORGANIC ACEDIMIA)

r In the degradatiorr of essential branched chain amino acids (Valine,leucine, isoleucine),the intermeiiiate metabolites
are all organic acids.
r Thus deficiency of any enzyme (except transaminase) in the metabolism of these amino acids caLtse accumulation of
crganic acicibelore the enzymaiic block.
r Therefore, these disorders are called organic acedimia/organic aciduria.
o Important tlpes of organic acidemia are :-
r) Methyhnalonic acidemia
ii) Propionicacirlemia
lli) Iso avleric acidemia
iv) Maple syrup urine disease
v) Combined (multiple) carboxylase deficiency
vI) Glutaric acidemia
vii) Ketothiolase ilefciency
r Most of the clinical features of these disorder are same and include : refusal to feed , vomitinglAilMs 00),
neurological damage
(mental retardation, lethargy), metabolic acidosis (lactic acidosisGIIMS00)),hyperammonemia(AIIM|00)
seizures(AIIM9o0l,
developmental cielay, neutropenia and hlpoglycemia.
r However, some manifestations are specific and help in the diagnosis.

Features of organic aciduria (see abot,e)

K eto ucitlo s is and hyperamm on em ta No Ketoucidosis

No skin manifestations Skin rash (exfoliation) Normal Amonia leve1 Hyperarnmonemia


t Propionic acidemia Multiple curboxylase Y
I

J
tlInt,t99t
t Methyl melonic acidemia deltcrcncy
Acy I- C oA de hy- dr ogen as e Urea cycel defect
c Ketothiolas
deficiency
3-hydron 3-merhyl glutaric
ac idu ria (glfi ar i c aci d emi a)

Note :- Ureu cycel enq)me defects also present with features of organic aciduria, but there is Hyperammonemia without
ketoucidosis and there is no lactic acidosis.
Multiple rarboxylase defi ciency
o It is an autosomal recessive disease due to deficiency
of holocarboxylase synthase (required for synthesis of a-
carboxylases) or biotinidase (required for biotin metabolism,
a coenzyme for all carboxylases).
r Thus there is functional deficienc y of multiple carboxylases (acetyl coA carboxylase,
puruvate carboxylase, propionrl
CoA carborylase, methyl crotonyl CoA carborylase).
r child present with refusal to feed, vomiting, lactic acidosis(ArlMsee),hlpoglycemi a, seizuresarruseer,
failure to thrir-e.
hypotonia, generalized rash with exfoliatipn (exfoliative dermatitis)(ArrMsee)
arrddevelopmental delay.
o The urine has peculiar smell of tomcat urine.
o Biotinidase deficiency is also associated with features
of biotin deficiency (i.e. dermatitis, ataxia, and SNHL), in
addition to above features.
e Diagnosis is confirmed by enzyme assay in lymphocytes.
o The mainstay of treatment is biotin administration(Ar 07)
.

o It is an inborn error of m6tabolism of branched chain amino


acids valine, leucine and isoleucine. It is due to deficiencr.
of enzyme that catalyzes the second reaction in these amino
acids metabolism i.e. branched.h";-";;ffi#
'drogenase
which catalyses decarborylatio n@EEr'IK'el) ofbranched chain
amino acids (Arrr. As a result, the branched
chain amino acids, leucin eeGI s7' es' Ar ee' AP e6), isoleucin eeGI ez, ss, Ar es,Ap e6)
and valin e ecr s7, es,Ar ss,At st), andtheir q,_keto
acids accumulate in blood, urine and CSF. There is characteristic
maple syrup odor to the urine.

':!il;,, suspected because of peculiar odour of maple syrup found


in the body fluids - urine, sweat, cerumen etc.
o Diagnosis is confirmed by amino acid analysis showing
marked elevation in plasma level of leucine, isoleucine,
valine and alloisoleucine (a steroisomer of isoleucine not normally
found in blood) ar,d.depression of alanine.Leucine
levels are usually higher than those of other three aminoacids.
r Urine contain high levek of leucine, isoleucine and valine and their respective ketoacids,
I Theketo acids may be detected by adding a few drops 2-4 dinitrophenylhydrazine (DNpH) reagent
which produces
a yellow precipitate in positive test.

o Ferric chloride gives navy blue colour@Gr s7) withthe patients


urine.

Characteristic odour in urine

Glutaric acidemia (type II)

M4ple syrup urine disease Maple slrup

r Lysosomes are key components of the intracellular digestive


tract. They contain a battery of hydrolytic enzymes. These
hydrolltic enz)rynes catalyze the breakdown of a variety of complex
macromolecules. with an inherited deficiency of a
functional lysosomal enzYme,catabolism of its substrate remains
incomplete, leading to accumulation of the partially
degraded insoluble metabolite within the lysosomes. Stuffed
with incompletely digested macromolecules, these
l:llijilri :lrilltiiliiil:
..:i:f,:.:.: r:Lr':i..::i::i::,:,:

C'I{n$.T,E,lii

organelles become large and numerous enough to interfere with normal cell functions, giving rise to the lysosomal
storage disorders.
(Ar 13)
o All lysosomal disorders are hutosomal recessive' except for Hunter's syndrome and Fabry's disease, which are
X-linked recessive. Thus Hunter's syndrome and Fabry's disease affect only malesqttMs 08) .

Sphingolipidoses
e GM1, gangliosidosis p-gaiactosidase GMl ganglioside
r GM2 gangtiosidosis
l) Tay-sachs disease
ii) Sandoffdisease

Gtycogenosis Type II
o Pompe disease

Mucopolysaccharidoses
o MPSIH(Hurlefl a-L-Iduronidase Dermatan sulfate
c MPS ll (Hunter)
Iduronate sulfatase Heparan sulfate

r Wolman disease Acid lipase Cholesterol ester triglyceride

Facts about important individual lysosomal storage disease


r Gaucher's disease presents with hepatosplenomegaly, bone pain, bone marrow infiltration (anemia, thrombocltopenia)
e3),
pathological fr actures, Erlenmeyerflask deformity of ilistalfemur@et osteolltic lesions and osteosclerosis. Gaucher's
cells are present in reticulo-endothelial system, with PAS positive diastase resistance materialaflMs0a) accumulated in
them. On electron microscopy shows, elongated distended lysosomes containing the stored lipiil in stacks of bilayer
(in contrast to Niemann-Pick disease, in whichlipid is arranged as concentric or paralledlamellae@IlMs0a)). Treatment
o6' 03' AI04)
of choice is enzyme replacement therapyqnus .

r Niemann-Pickdisease presents with hepatosplenomegaly, and delayed milestones. Type A involves CNS and Tlpe B
involves lungs. There may be childhood cholelithiosis(A[Ms e8). Histiocltes shows PAS positive diastase resistan ceAilMs
inclusion which on microscopy shows concentric or parrallel lamellar arrangement@ItMs 04) .
0a)

r Fabry Disease presents with angiokeratomas and neuropathic pain. Lipid inclusions with "Maltese crosses" are seen
in urinary sediment.
r Metachromatic leukodystrophy presents with feature s of white matter degeneration. There is mental retardation(rN
03),
muscle wasting, decreasedtendonreflexes(rNost, optic atrophy(rN03) and genu recurvatum. Inclusion bodies are
positive for PAS and alacian blue.
r Krabbe disease presents with optic atrophy, opisthotonus, seizures, with radiological increased densilv in thalamusarMs
07)andlow density in areas in white matter.

r Farber's disease typically presents with joint swelling and nodules.


r Wolman's disease presents with hepatosplenolfiegaly@uns tt' to'oo) jaundice(AllMs11'10'0e),lymphadenopathy, poor weight
gain(failuretothrive), abdominaldistensionGfiMsll'10'oe),hypotoniaandanemiaqlrMsll'10'0e).Themostcharacteristic
11' 10'0e),
finding is adrenal calcificationallMs along with deposition of lipid.
Wilsqnt diseaSe fHepatolenticular degenerationl
c Itisan autosomalrecessiveale3) disorder of coppermetabolismarD causedbymutationofATpTBgen/avnrrtonlong
arm of chromosome 13.
c ATPT B gene encodes 'p type'ATP responsible for copper excretion and incorporation into ceruloplasmin in liver. The
defect in ATPT B gene results in copper accumulation in liver (primarily) and other organs like eye and CNS.
r Clinical presentation includes :-
i) Hepatic t Most common organ involved is liver artd can present withfatty change, acute and chronichepatitis{Allus
e') and cirrhosis.

ii) ganglia is involved primarily. There is dystonia, dysarthria, tremor(Al


CNS z Basal or),
and chorea. Motor deficit
and sensory involvement are characteritically absentallMs et).
11i) Psychiatric : Emotional liabilityre, ort, depresion hyperactivitiy.
iv) Ocular involvement ; Kayser-Fleischer ting@trr1 (brown/gree n deposits in Descemet's membrane of
cornea(At oa)), Sunflower cataractLr 13' NEEr)
.

v) Others: Coombs negative hemoftic anemiaullMser), fanconi syndrome (amino aciduria) and cardiomyopathy.

Diagnosis
c The gold standard for diagnosis is liver biopsy with quantitative copper assay -) concentration of copper in a
liver biopsy sample > 200 mglgdry weight.
o Other tests qre -

1) Serum ceruloplasmin level > low@NB


ls' AI 10)

ii) KF rings@Nar'urct
ll1) urine colp)er excretion -+ 13, Ar lo)
increased@NB
iv) DNA Haplotlpe analysis.

Leukodvstroohv
e Leukodystrophy refers to progressive degeneration of the white matter of the brain due to imperfect growth or
development of the myelin sheath, the fatty covering that acts as an insulator around nerve fibre.
I Myelin which lends its color to the white matter of the brain, is a complex substance made up of at least ten different
chemicals.
e The leucodystrophy are a group of disorders that are caused by genetic defects in how myelin produces or
metabolizes these chemicals.
o Each of the leucodystrophies in the result of a defect in the gene that controls one (and only one) of the chemicals.

Specif < Ieucodystrophies include -


-' Metachromctic leucodystrophy t Adrenoleucodystrophy s Conevansdisease
t Krabbe's disease t Pel iza eu s- M e rzhta ch er di sease s A{exonderdisease'
CnapTen
a Macrocephaly is most characteristic of Canavan's slfldrotns@Ittrs oa) .
a CT scan in Krabbe's disease disease shows increased density in thalamus(AttMs07) and low density areas in white
matter.
r CT scan in Metachromatic leukodystrophy shows diffuse symmetrical attenuation oJ cerebellar and cerebral white
matter.
ong
r CT scan in Alexander's disease shows degeneration of white matter, most prominant in frontal lobes.
r CT scan in Canavan's disease shows difuse white matter degeneration mainly in cerebral hemisphere.
^ie
Treatment of innportant inborn error of metabqlisnl
1. Alkaptonuria - Vitamin C, Nitisinone
2. Homocystinuria - Pyridoxine(uPsc07'PGl87,81'7e) + Folic acid
3. Cystinuria - Alkalization of urine + d-Penicillamine, captopril,
icit 4. Hartnup disease - Nicotinamide
5. Multiple carboxylase deficiency - Biotin
6. Metlrylmalonic acidemia - Vit B
t2(UPSC
07)

o,f 7. Hyperoxaluria - Pyridoxine


8. Tyrosinemia - NTBC, Liver transplantation

Cxunurla
r Metabolic disorder of autosomal recessive inheritance(irrs).
r Defect in transport of cystine and some other aminoacid across renal tubular cell & interstitial cellar
15)

e Selective increase renal clearance of cystine and other basic aminoacid in urine.
r High urinary cystine leads to radio - opaque hexagonal crystal(Ar Is).
c Crystal is soluble in alkali.
o Treatment -+ formation of stone can be reduced by irlkalization of urine.

.-
.. , ...
:l:i
- ,,,. -,...,r' '..-. , -l

a Recurrent hypoglycemia attackwith hepatomegaly:Von-Gierke's disease (type-1 glycogenosis).


a Hypoglycemia not responding to epinephrine/glucogon: Von-Gierke's disease.
a Not a liver glycogenosis: Pompe's disease (type-ll glycogenosis).
a Glycogaen staroge disesseswith predominant musle invalvemenr IType ll,VVll.
a Glycogen storage diseases with predominant liver involvement :Iype l, Ill, lV Vl, lX, Xl.

a Muscle'iranps with myoglobinura : Mc Ardle's disease itypeV glycoqenosis)'


a Coarse facies, hepatosplenomegaly and tall QRS complex: Pompe's diseae (type ll glycogenosis).
a VamikinQ, distended abdomen, diqrrhea andreducing sugar in urine in a neanaf€ : Galactosemia,
a Most common enzyme deficiency in galactosemla : Galactose-1-phosphate uridyl transferase.
I Coinjugaied hyperbiliiabinenlia, seizares,and cataract in a child: Galactosemia
a Hypoglycemia, hepatomegaly and reducing sugar in urine after ingestion of fructose (sugarcane iuice) : Hereditary fructose intolerance
(aldolase-B defi ciency).
a Multiple carboxylase deficiency is due to : Biotin deficiency-
o lnfant with refusal to feed, vomiting, seizures, ketoacidosis with skin manifestation: Multiple carboxylase deficiency (a type of organic
aciduria).
t tntroctable seizures, skin rash, lacticacidasis with hyperammonemia '. Atganic acidemia.
a Enzyme defi ciency i n phenyl ketonu ria : Phenylalanine hydroxylase.
a Treatmentofchoiceoifphenylketonuria.:Lowphenylalaninediet{phenylalanineisnotstoppedcompletely).
, tn phenylketonuria :Tyrosine becomes an essential amino acid.

s Fec\brt ifi urine of phenylkelonuria gives: Green coiour,


gives: Blue colour.
3 FeCl ,test in urine of Maple syrup urine disease
l, Deficieicy o,f a-keto acid decarhoxylase with blocked metabolism of branched chain ami,ino arids : Maple syrup urine disease-
a Darkening of urine on standing: Alkaptonuria.
* Mousy adoi ofurine : Phenylketonuria.l
Cttep!En
oa)
o Macrocephaly is most characteristic of Canavan's syndrotnsqilus .

o CT scan in Krabbe's disease disease shows increased density in thalamus(/I.MS07) and low density areas in white

matter.
r CT scan in Metachromatic leukodystrophy shows diffuse symmetrical attenuation of cerebellar and cerebral white
ic
matter.
r CT scan in Alexander's disease shows degeneration of white matter, most prominant in frontal lobes.
r CT scan in Canavan's disease shows diffuse white matter degeneration mainly in cerebral hemisphere'

Treatment of innportant inborn error of metabolism


1. Alkaptonuria - Vitamin C, Nitisinone
2. Homocystinuria - PyridoXine(uPscoT'PGt87'81'7e) + Folic acid
3. Cystinuria - Alkalization of urine + d-Penicillamine, captopril,
4. Hartnup disease - Nicotinamide
5. Multiple carboxylase deficiency - Biotin
6. Met@l malonic acidemia - Vit B ,r(utsc
oz)

7. Hyperoxaluria - Pyridoxine
8. Tyrosinemia - NTBC, Liver transplantation

Slunu{ta rs''
o Metabolic disorder of autosomal recessive inheritance(Ar
Is)
o f)efect in transport of cystine and some other aminoacid across renal tubular cell & interstitial cellar
c Selective increase renal clearance of cystine and other basic aminoacid in urine'
rs)'
r High urinary cystine leads to radio - opaque hexagonal crystal(Ai
c Crystai is soluble in alkalfl
c Treatment -) formation of stone can be reduced by blkalization of urine.

(type-'l
a Recurrent hypoglycemia attack with hepatomegaly tvon-Gierke's disease
a Hypoglycemianotrespantdingtoepinepi'hrine/glucogon:von-Gierke'sdisease'
a Not a liver glycogenosis: Pompe's disease (type-ll glycogenosis)'
a Giycigen staragediseases,with predomlnant musle involvement: Type ll, V Vll.
a Glycogen storage diseases with predominant liver involvement :lype l, lll, lV Vl, lX, Xl.
'
a Muscle,cr*nps with'myagilobinuna : Mc Ar.dlet'dlsease {!ypeV glycogenosis).
(type ll glycogenosis)'
o Coarse facies, hepatosplenomegaty and tatl QRS complex : Pompe's diseae

a Vo.mitifig, distended abdamen, diarrhea and reducing sugar in urine in a neonate : Galactosemia' :

3 Most common enzyme deficiency in galactosemia : Galactose-1-phosphate uridyl transferase.


* Conjugai.ed hyperbilirubinemie,'seizures'and cotaract in a child: Galactosemia.
a Hypoglycemia, hepatomegaly and reducing sugar in urine after ingestion af fructose(sugarcane iuice): Hereditary fructose intolerance
(aldolase-B defi ciencY).
* Multiplecarboxylase deficiency is due to', Biotin deficiency'
deficiency (a type of organic
& tnfant with refusal to feed, vomiting, seizures, ketoacidosis with skin manifestation: Multiple carboxylase
aciduria).
& tntiactable seizures, skin tash, lactic acidasis with hyperammonemia t Arganic acidemia'
& Enzyme defi ciency in phenyl ketonu ria : Phenylalanine hydroxylase'
& Treatmentaf choiceof pfi{nylketonuria:'Lowphenylalanine diet {phenylalanine is not siopped completety)'
* ln phenylketonuria:Tyrosine becomes an essential amino acid'
* FeClrtest in urine of phenylketanuria givels: Green Colour'

* FeClrtest in urine of Maple syrup urine disease gives: Blue colour'

* Deficiency of s-keto a,cid decarboxylase with btocked metabolism af branched


chain amino acids: Maple syrup urine disease'

* Darkening of urine on standinq : Alkaptonuria.


x Mousy odor of urine : Phenylketonuria.
Accumulated in Gaucher's disease : Cerebroside (glucocerbroside).

t Deficiency in Tay-Sach disease : Hexosaminidase -A (q-subunit).

t Bilateral deep brightthalamic appearnce: Krabbe's disease.

ln Wilson's diseose : Reduced ceruloplasmin, increased serum copper, increased urinary excetion of coppel increased liver copper.

Amongtheteucodystraphies, Macracephaly is most characteristicof : Canavan's disease.

t Cystinuria is characterized by: Recurrent urinary stones.

Hartnup diseose is characterized by: Massive aminoaciduria of multiple neutral amino acid without increase in neutral amino acid
in blood.

III
QUESTIONS

3 LYCOGEN STORAGE DISEASE itive forbenedicts test for reducing substance. The
substance in urine is - (AILMS Nov 10)
All are liver glycogenosis except- (NEET Dec.12 Pattern) a) Sucrose b) Glucose
a) Von Girke disease c) Galactose d) Fructose
b) Hers disease 9. An 8 days old child presents with yellow selera,
c) Tlpe III glycogenosis whitish stool and turmeric colour urine on 3rd day
d) Pompes disease of septicemia on broad spectrum antiboitics, the
Glycogen storage disease with predominant muscle likely diagnosis is - (AIIMS lune 98)

involyement - (All India Dec.1j Pattern) a) Galactosidasedeficiency


a) Tlpe I b) Ammonia toxicity
b) Type TTT c) Galactose 1-4 phosphatase uridyl transferase
c) Tlpe IV deficiency
d) Typev d) Glucose 6 phosphatase deficiency
A child presents with hepatomegaly and hlryoglyce- . 10. Child of Vasanthi was weaned from breast milk on
mia. There is no improvement in blood sugar even the 5th day and was given sugarcane juice the child
after adrninistration of epinephrine. W'hat is the developed hlpoglycemia and hepatomegaly Bio-
likely diagnosis - (At 1o) chemical examination showed hlpophosphaternia
a) Von girket disease and reducing substances in urine. The child is prob-
b.; Anderson's disease ably suffering frorn which of the following enzyme
c) Pompe's disease deficiencies - (AIIMS Nov 2000)
d) Mc Ardle's disease a) Fructokinase
A child presents with rnassive hepatomegaly and b) Aldolase B
hypoglycemia. There is no improvernent in blood c) Glucose 6 Phosphatase
glucose on administration of glucagon. The proba- d) Beta galactosidase

ble diagnosis is - (At os)


a) Von-Gierke disease . DEFECTIVE AMINO-ACIDS METABOLISM
b) McArdle disease
c) Cor'sdisease 11. An infant presents with history of seizures & skin
d) Forbet disease rashes. Investigations show metabolic acidosis
5. In a patient, muscle crarnps on exercise, +ve increased trlood ketone levels and NlIr. This child is
myoglobulinemia, the disorder is - (PGI lune 98)
likely to be suffering from - (Ar 02)

a) Pompe's disease a) Proplonicaciduraria


b) Myotonia congenita b) Urea cyclic disorder
c) Myotonic dystrophy c) Phenylketonuria
d) Mc ardle's disease d) Multiple carboxylase deficiency
6. An infant has hepatorenomegaly, hypoglycemia, 12. A 5 day old child presents with intractable seizures.
hyperlipidemia, acidosis and normal structured He had rashes all over the body. Blood examination
glycogen deposition in liver. What is the diagnosis - showed hlperarnmonemia and lactic acidosis. The
a) Her's disease (All lndia Dec. 15 Patern) probable diagnosis is - (AilMS 2K)

b) Von Gierke's disease a) Organic aciduria


c) Cori's disease b) Mitochondrial encephalopathy with lactic aciduria
d) Anderson's disease c) Phenylketonuria
d) Urea cycle enzyrne deficiency
:. Coarse facies, hepatosplenomegaly and tall QRS on
ECG are characteristic feature of - (Al.rMS Nov 01) 13. 3 days old newborn with unknown inborn error of
a) Glycogen storage disease tlpe II rnetabolism, hyperammonemia in blood-
b) Hurler's disease a) Maple syrup urine disease (All lndia Decl5 Pattern)

c) Hunters disease b) Urea cycle enzyme deficiency


d) Hemochromatosis c) Organic aciduria
d) Phenylketonuria
GLACTOSEMIA AND FRUCTOSE INTOLER-
t4. Treatment of multiple carborylase deficiency -
a) Biotin (At 07)
ANCE b) Pyridoxine
c) Thiamine
8. A 3 day child vomits everlthing he feeds, has a
d) Folic acid
distended abdomen & diarrhoea. The urine is pos-
15. Enzyme deficient in phenylketonuria - b) Cystinuria
a) Tp'osinase (AlllndiaDec.l4.pattern) c) Fabry's diseas
b) Phenylalanine hydroxylase d) Tyrosinemia
c) Tyrosine transaminase
d) Homogentisic oxidase
LYSOSOMAL STORAGE DISEASE
16. True statement regarding a 3 week old child with
Phenyl-ketonuria is all, except - (Ar 2000) 25. In Gaucher's disease there is accumulation of inside
a) Provocative protein meal tests helps in the the cells - (CET Nolu. 14 Pattem)
diagnosis - a) Galactosidases
b) Tyrosine becomes an Essential amino - acid in diet b) Sphingomyeline
c) Serum Phenylalanine is increased and urinary c) Glucosidases
Phenyl Pyruvate level is elevated d) Cerebrosides
d) Phenylalanine should be completely stopped in
26. Deficiency of enzyme in Tay-Sach disease -
diet
(All India Dec.13 Pattern)
17. In phenylketonuria, the treatment of choice is? a) Galactosidase b) Hexosaminidase
(AttMS Nov t0) c) Acid lipase d) Glucosidase
a) Limit intake of substrate for the enzyme 27. Only male are affected in - (AllMS May 08)
b) Provide the deficient amino- acid
a) Scheie's syndrome
c) Correct the enzyme defect b) Hunter's syndrome
d) Symptomaticmanagement c) Hurler's syndrome
18. A child has microcephaly, Blue eyes, Fair skin, and d) Gaucher's disease
Mental retardation, Ferric chloride test is positive.
28. For which of the following diseases is enzyme re-
placement therapy available -
a) Phenylketonuria (PKU)
(Al 04, AIIMS May 06, Nov 03)
b) Homocystinuria a) Albinism
c) Tyrosinosis b) Neimann-Pick disease
d) Alkaptonuria c) Metachromatic leukodystrophy
19. Mousy odour urine is seen in- (NEET pec.12 Pattern)
. d) Gaucher's disease
a) Maple syrup urine 29. A child presents with hepatosplenomegaly, abdom-
b) Phenylketonuria inal distension, jaundice, anemia and adrenal calcifi-
c) Isovalericaciduria cation. Which of the following is the diagnosis?
d) Cystinuria a) Adrenal hemorrhage (AIIMS May 11, May 10,09)
20. A child of phenyl ketonuric mother may develop - b) Wolman's disease
(AIIMS lune 99) c) Pheochromocltoma
a) Microcephaly, mental retardation, congenital heart d) Addisons disease
disease
b) Mental retardation, cataract, congenital heart
disease
LEUCODYSTROPHY
c) Hydrocephalus,cataract.
30. Macrocephaly is seen in which of the following syn-
d) Microcephaly, cataract, renal dysplasia.
dromes ? (LIIMS May o8)
21. In phenylketonuria FeCl, test with urine gives a) Metachromatic leucodystrophy
......,..cO1Or - NEETDec.l2Pattern) b) Adrenoleukodystrophy
a) Green b) BIue c) Canavant disease
c) Red d) Purple d) Krabbe's disease
22. Deficiency of enzyme a-keto acid decarboxylase 31. Deep white matter lesion with bilateral deep bright
leading to a block in the metabolism of branch chain thalamic appearance is suggestive of -
amino acids is observed in - (AIIMS May 0s)
a) Alexander disease (AIIMS Nov 07)
a) Maple syrup urine disease b) Canavan's diseases
b) Hartnups disease c) Krabbe's disease
c) Alkaptonuria d) Metachromatic leukodystrophy
d) Phenylketonuria
23. In maple syrup urine disease FeCl, test with urine
WILSON'S DISEASE
gives........ color - (All lndia Dec.14 Pattern)
a) Green b) Blue 32. Gene of wilsons disease is - (NEET Dec.12 Pattern)
c) Black d) Red
a) ATP 7A b) ATPTB
24. Darkening of urine on standing is associated with - c) ADPTA d) ADPTB
a) Alkaptonuria (Ar 07)
Cuepren 9 Metabotic
Gene for Wilson's disease is located on chromon- b) Homocystinuria
!ome- (All lndia Dec.13 Pattern) c) Nonketotichyperglycemia
b) 10 d) Ketotic hlperglycemia
, 13 d) t7 42. Which one of the following is not a pyridoxine de-
,* -{ old girl with the rnood and emotional
12 year pendent disorder-
-----__ liability has a golden brown discoloration in a) Homocystinuria
(UPSC 07)

r.lation
of,insid, descement membrane. Most likely diagnosis is - b) Methyi-malonic-acidemia
_: f I
.Vor,. 14 pafter ; Fabry's disease (Al o1) c) Cystathioninuria
: ) Wilson's disease d) Xanthurenic aciduria
;) Glycogen storaSe disease 43. Cystinuria is characterised by - (ALLMS 83)
d) Acute rheumatic fever a) Generalised aminoaciduria
:
True about wilson's disease - -(AI 10) b) Systemic acidosis
a5e _
a) Increase in urinary copper and increased serum c) Deposition of cystine crystals in Renal tubular cells
::.., Dec.l3 pafteh:
ceruloplasmin and copper d) Recurrent urinary calculi
:lnidase
b) Increased serum ceruloplasmin levels with 44. Cloudycornea is a feature of- (MH u)
ise
increased urinary copper a) Hurlert disease
: ILTIS t4q,08t c) Elevated hepatic copper leVel and increased serum b) Morquiot disease
ceruloplasmin levels c) Maroteaux Lomy disease
d) Increased in urinary copper and decreased serum d) All of the above
ceruloplasmin
45. All are seen in metachromatic leukedystrophy
True about Wilsons disease is? (cET Aug 13 Pattern)
except-
l-rlre re- a) Increased serum ceruloplasmin a) Mental retardation (IN 03)
b) Decreased liver copper b) Optic atrophy
;. 06, Nov
03) c) Increased urinary copper excretion c) Decerebrate posture
d) Decreased urine copper excretion d) Exaggerated tendon reflexes
W"ilson disease all are true lxcept - 46. Which one of the following is not a feature of
a) AR (All India Dec15 Pattern) Phenylketonuria? (uPSCo4)
b) KF ring a) Severe mental retardation
rbdorn_ c) Raised copper level b) Reduced tendon reflexes
rj calcifr- d) Raised ceruloplasmin level c) Enamel hlpoplasia
rsi d) Vomiting in early infancy
,r.l 10, 09)
M!SCELLANEOUS 47. Guthrie test can be used for the diagnosis of -
a) Tyrosinemia (UPSC t la)
18. Features ofcystinuria are - r4ll lndia Decl5 Pattern) b) Galactosemia
a) Impaired proximal tubular reabsorption of cystin c) Alkaptonuria
b) Autosomal recessive d) Phenylketonuria
c) Recurrent renal stone 48. Which is nnt a feature of wilsons disease in a child-
d) All of above a) Fanconi syndrome (AtrMS e1)
sYn- 19. Features ofRefsum disease are all except - b) Sensory changes
't.v 0g)
a) Ataxia (All India Decl5 Pattern) c) Hemolltic anemia
b) CCF d) Chronic active hepatitis
c) Ichthiois 49" Which glycogen storage disease doesrt't a{fect
d) Retinitis pigmentosa muscles ? (APPG 08)
-10. 2-5 year old child with DM, target HbA, C is - a) type I
. (All lndia Decl5 Pattern) b) type 2
a) <8% b) <7% c) tlpe 3
t7) c) <9 d) <60/o
o/o
d) tlpe 4
50. Massive arninoaciduria without a corresponding
increase in plasma amino acid level is characteristic
QUESTIONS OF VARIOUS OTHER EXAMINA. of which one of,the following diseases ?
TIONS a) Homocystinuria (UPSC-| 08)
b) Hartnup disease
41. Large doses ofpyridoxine are ofvalue in some cases c) Tyrosinemia
of- (PGr 7e,81, 87) d) Maple syrup urine disease
a) Phenylketonuria

rrtr
ANSWERS
GLYCOGEN STORAGE DISEASES

1. Ans. is 'd' i.e., Pompes disease [Ref : Nelson tSth/e p. 60j, 604]

lmportant Glycogen Storage disease

Liver glycogenoses Muscles glycogenoses

Disorders with hepatomegaly Disorders with Disorders with Disorders with


and hypoglycemia Liver cirrhosis muscle energy progressive
impairment skeletal myopathy
I
and/or cardiomyopathy
I
* tI

Type Name Enzyme defect Tvpe Name Enzyme defect


Type I Von Girke Glucose 6 - Type iV Andersons Branching
Phosphatase enzyme Type Name Enzyme defect
Type lll Cori/Forbe Debranching Type ll Pompes Lysosomal acid
Limit enzyme (Liver cr - glucosidase
dexirinosis and / or muscle)
Type Vl Hers Liver Muscle phosphorylase
phosphorylase
Phosphofructokinase
Type lX Phosphoryl Phosphoryl
ase kinase kinase
deficiency

Type Xl Fancon/ Glucose


Bickel transporter-2

) Ans. is t' i.e., Tlpe V fRef: Previous expalanations]

J. Ans. is 'l i.e., Von Girke's disease lRel See above explanation)
r Amongst the given options only Von Girke's disease presents with hepatomegaly and hlpoglycemia.
'A diagnosis of type I glycogen storage disease (Von Girke's disease) should be suspected whenever there is
hepatomegaly withlrypoglycemia that is unresponsive to glucagon or epinephrine".

4. Ans. is 'd i.e., Von Gierke disease [Ref : Nelson 18th/e p. 603, 604, 605; Harison lVh/e p. 2458; O.P. Ghai 9th/e p. 657
dr Vh/e p. 6361
Liver glycogenoses (Liver glycogen storage disease)
Presence of hepatomegaly and hypoglycemia alongwith absence of an increase in blood glucose upon administration
of glucagon suggets the diagnosis ofVon Gierke disease.
In Von Gierke disease, administration of glucagon or epinephrin results in little or no rise in blood glucose.
Mc Ardle disease is a Muscle glycogenoses and does not characteristically present with hepatomegaly and
hypoglycemia. Mc Ardle\ disease can hence be excluded.
Cori and Forbes are liver glycogenoses grouped under glycogen storage disease type lll. Although they present
with hepatomegaly and hypoglycemia, the administration of glucagon in these conditions provokes a normal
increase in blood glucose (Nelson 18th/e 605). The child in question does not show an improvement in hypoglycemia
afier administration of glucagon and hence is unlikely to have Cori or Forbe\ type III GSD.

5. Ans. is ? i.e., Mc Ardle's disease lRef : Nelson 18th/ep. 604-606)

McArdle's disease
o This is due to deficiency of muscle phosphorylase,
o Lack of this enzgne limits muscle ATP generation by glycogenolysis, results in glycogen occumulation.
o Clinical manifestions -
r Develop in late childhood or adult
s Exercise intolerance with muscle cramp
e It is important to note that two types of activity tend to cause symptoms:
i) Briefexercise ofgreat intensity such as sprinting
ii) Less intense but sustained activity, such as climbing stairs or walking uphill.
r Moderate exercise, such as walking on level ground, can be performed by most patients for long periods.
r Many patients experience a characteristtc "second wind" phenomenon -+ If they slow down briefly at the lst
apperance of muscle pain, they can resume exercise with more ease.
o There may be exercise - induced myoglobinuria secondary to rhabdomyolysis.
o This can lead to acuterenalfailure.
o Serum creatine kinase is elevated.

Ans. is'H i.e., Von Gierke's disease lRef : Nelson 18th/e p. 603, 604)
o A11 the given options are liver glycogenoses which present with hepatomegaly and hypoglycemia.
o Among the given options renomegaly is a feature of Von Gierke's disease.
e One important fact require specific mention. In type-III (Cori's) and tlpe-IV (Anderson's) glycogenoses, deposited
glycogen has abnormal structure. Where as in other glycogenoses, glycogen has normal structure. Thus option c
and d can be exciuded easily as glycogen deposited in question has normal structure (further these two do not have
renomegaly).
c Hert disease (t1pe-Vl) has deposition of glycogen with normal structure, but there is no renomegaly.

Ans. is 'b' i.e., Glycogen storage disease type I[ lRef: Nelsoru 18th/e p. 6A7)
a Hepatosplenomegaly, characteristics E.C.G. findings and coarse facies are seen in Glycogen storage Type II or
pompet ds.
o Hurler's and Hunter syndrome are mucopolysaccharidosis that presents with hepatomegaly and coarse facies, bul
characteristic ECG pattern of Pompes disease clinches the diagnosis in favour of Pompe's disease.

cAI-,qeTOSEMlA & FRUCTOSE INTOLERANe E

8. A.ns. is 'c' i.e,, Galactose {Ref: I'{elsan 18thle p. 609-61A; O.F. Ghai B'h/e p. 656 6 Vh/e p. 634, 6i5l
a Characteristic symptom i.e., vomiting, distended abdomen and diarrhea along with the presence of reducing substance in
the urine suggests the diagnosis of galactosemia

9, Ans. is t'i.e., Galactose l-4 phosphatase uridyl transferase deficiency lRef: A.P. Ghai E'hle p. 655 6 7h/e p.
534-535; Nelsan 78th/e, p. 609, 6101
e Feature of conjugated bilirubinemia on the 3rd day accompanied by sepsis in a neonate suggests diagnosis of Ga-
lactosemia.
e Inabilitytometabolisegalactoseoccursingalactosemiawhichmaybecausedbydeficiencyofgalactose-1-phosphate
uridyl transferase, UDP-Galactose-4-Epimerase and galactokinase.
e Out of three, most common is Galactose = l-phosphate uridyl transferase deficiency.
ob'
10. Ans. is i.e., Aldolase E IRel' Lippincat's Biachewistry 2dle p. 128; Nelsan tr8'h/e p. 511)
c Symptoms of hypoglycemia, hepatomegaly develping after ingestion of excessive fructose intake (sugar cane juice
contains the major dietary source offructose-Sucrose suggest the diagnosis ofhereditary fructose intolerance.

DEFECTIVE AIVI!NOACI DS METABOTISMI

11. Ans. is 'd' i.e., Multiple Carboxylase Deficiency fRef: Nelson 18th/e p. 6141
o The patient in question has symptoms of organic aciduria. Among the given options Propionic aciduria and multiple
carboxylase deflciency are organic aciduria. But skin manifestations are found in multiple carboxylase deficiency (not
in propionic acidemia).
o Urea cycle enzyme defect also presents with clinical features similar to organic aciduria. But in urea cycle defect there
is no ketoacidosis (patient in question has increased ketone bodies, i.e. ketoacidosis).

12. Ans. is'a' i"e., Organic aciduria fRef: www.en.wikip edia.orgl


e The child in question has symptoms of organic acidemia (aciduria), which can also occur in urea cycle enzyme
deficiency. But, there is no lactic acidosis in urea cycle enzyme deficiency.
Neonatal Hyperammonemia

Acidosis No acidosis

I
I

+
O rg an i c ac i d e m i u,/ucida r i a (Irea cycle defectl
13. Ans' is'H i.e., urea cycle enzyme deficiency lRef; Ghai vhre p. 6jj, 6i2, 62s)
o There is hlperammonemia without any acidosis -+ urea
cycre defect.

t4- Ans. is 'a'i.e., Biotin fRef: Nelson lg,hle p. 6141


r For the treatment of multiple carboxyrase deficiency, biotin
shourd be given.
t5. Ans. is 'H i.e., Phenylalanine hydrorylase [Raf a.p. Ghei Bthle p.
652 6 zhle p. 62s]
o classical phenylketonuria is due to deficiency of pheny-lalanine
hydroxylase. Mild form can be caused by deficiency
of dihydrobiopterin reductase.

16. Ans' is'd'i'e', Phenylalanine should be completely stopped in the


diet [Rel o.p, Ghai g,h/e p. 6s2 & vh/e p.
628-6291
o Phenylamine should be restricted in diet, however
it should not be comytletely eleminated, as it is necessary for
normal growth.
17. Ans. is'a'i.e., Limit intake of substrate for the enzyme
[Rt': Nelson rSth/e p. 530_31]
' T:1""t:tj":^"^l1l:j]h:'"1,
o
"lso
controversial. Dislontinuation of therapy, even in adulthood,
may cause deteriorarion
"j"i}"::l::i1]:.,y,.j*fbrmance. rhe current recommendation is ti"r'
restricted diet for life.
,tijii";;;;;;;;:;;;;;;;i;;;;;:
18. Ans. is 'a' i.e., Phenylketonuria [Rel; Nelson tSth/e p. 529; O.p.
Ghai Shlep. 652 dr Vh/e p. 625_629]

Clinical manifestations of phenylketonuria


c Normal at birth
o Symptoms occur after 1 months
i) Mental retardation -+ Not present at birth, deverops graduaily.
ii) Neurological signs -+ Irritability, tremors, conr.'ulilons, hlperkinesis and muscular hypertonia & Exagerated

iii) Blond hair, blue iris, fair skin.


iv) Musty or mousy body odour.
v) Vomiting
vi) Microcephaly
vii) Enamel hlpoplasia & Growth retardation

19. Ans" is 'H i.e., Phenyl ketonuria [Re/: o..e Ghai 8,h/e p.
652 & vh/e p. 629; Nelson rrth/e p. 5291
20. Ans' is'd i'e', Microcephaly, mental retardation congenital heart
p. 609; Nelsan l8thle p.
disease tRef o.p. Ghai grh/e p. 652 6 @le
5i1)
children born to phenyl ketonuria mothers have following features (Note
: mother is phenyleketonuric not child)
o Mental retardation t Growth retardation
t Microcephall c CongenLtal heart disease

2t. Ans. is 'a' i.e., Green


$ef : Nelson l}e/e p.53.1; 0.p Gbai 6th/e p. 609]
o-FeCl,
--+ 1/ detects\hg2leknce ofphenylalanine in urine. Fecr.
it will turn green.
add to patients urine. If it contains phenyraranine,
FeCl_ is added

't') Ans. is 'a'i.e., Maple Syrup urine disease [Re/; Harlter 26h/e p. 259; Nekon lg,h/e p. S4A, 54ll
r Maple syrup urine disease (MSUD) is d/t defect in enzyme - a-keto
acid dehydrogenase.
r It is a mitochondrial, enzyme complex consisting of
t a-ketoacid decarboxylase . Dihydrolipoyr rlehydrogenase and s Transacyrase
23. Ans. is'H i.e., Blue [Ref: Nelson l#le p. 54Il
o Ferric chloride gives navy blue colourwith the patients urine.
r
I
Ans. is 'a'i"e., Alkaptonuria [Re/; Nelson trth/e p. 534; O.p. Ghai B,h/e p. 653 & 7h/e p.
Cnerrrn 9 .Metubolk*iioi r
630; Lippincott 3d/e p. 222\
o Theonlysignofdiseaseinchildrenisablackeningofurineonstanding.Thisiscausedbyoxidationandpoll.n-rerization
of homogentisic acid.

. IISOSOMA!. STORAGE DI5EAsE5

Ans. is U'i.e., cerebroside [Ref : a.p. Gkai Bth/e p, 661 6 vh/e p. 6jg; Nelsaru la,t/e p, Sgs, sg6]
c In Gaucher's diseases, there is accumulation of cerebroside (glucocerebroside, i.e.
glucosylceramide).
-n. Ans. is 'b'i.e., Hexosaminidase [fiel l,lelsan l9th/e p, p. 638l
595, 596; A.F. Ghai Vt,/e
o The enzyme deficient in Tay-sach disease is Hexosaminidase-A (i.e., cx-subunit
of Hexosaminidase)

Ans. is 'b'i.e., Hunters syndrome fRqf : CTDT lgth/e p. j006, j007)


o Ali these are lysosomai storage diseases.
c Remember that -
'All lysosomal diseases are autosomal recessive except for Hunter's syndrome and disease which are X-
fabry's
linked recessive ".
o As Huntert syndrome is X-linked recessive, it will affect only males.

18. Ans. is 'd' i.e., Gaucher's disease [Ref: Netsan tgth/t p. 5g5, 5961
'Enzyme Reltlacement therapy is the treatment of choice
for significantly afltected patients - Harrison
'cerezyme' a recambinantly produced acid p grucosidase is being currently used.,

29. Ans. is 'b'i.e., tr{oXman disease IRef : Netson llthle p. 23s6, z3sn
e Hepatosplenomegaly, jaundice, abdominai distension and anemia, along with adrenal
calcification is the characteristic
presentation of Wolman's disease.

LEUCOEYSTROPHY

30. Ans. is t'i.e., Canavan's disease lRef: Internet reference]

Canavan's disease
l Autosomal recessive disorder
c Caused due to deliciency of the enzyrne N-Aspertoacylase.
e This leads to accumulation of N-Acetyl aspartic acid in brain and urine.
o It is characterized. by the clinical traid of -> Hypotonia, Head lag, Macrocephaly.
31. Ans. is t' i.e., Krabbe's disease fRef: I,{etson lgtt,/e p. 25021
Krabbek disease

W!LsON'S DISEASE

32. Ans. is'b' i.e., ATp 78 {Re.fi Robbins 9th/e p. 863-864)


o Wilson disease is an autosomal recessive disorder caused by mutation of
the ATpTB gene, resulting in impaired
copper excretion into bile & failure to incorporate copper into ceruloplasmin.
-)-). Ans. is t'i.e., 13 IRef: R"abbins Bth/e p. 864)
o The ATP 7 B gene is located on chromosome 13, which is responsible for
Wilson's disease.
34. /.ns. is'H i.e., Wilson's disease IRef: Nelson l7h/e p, l32l- j3221
o Emotional liability, tremors (Neurological symptoms) with brown discoloration
of descemet's membrane of cornea
suggest the diagnosis of Wilsont disease.
35. Ans. is 1d'i.e., Increased in urinary copper and decreased serum ceruloplasmin
tRef O.p. Ghai gthle p. 320-
32t (2 7h/e p. 292; Harrison IZh/e p.245tl
Eiagnosis of Wilson disease
c The gold standard fot diagnosis is liver biopsy with quantitative copper assay -) concentration of copper in a
liver biopsy sample > 200 mglg dry weight.
Other tests are -
o Serum ceruloplasmin level -+ Iow o Urine copper excretion -) increased
o KF rings r DNA Haplot)?e analysis.

36. Ans. is t'i.e., Increased urinary copper excretion lRef: See above explanationl

-7,/ . Ans. is'd'i.e., Raised ceruloplasmin level lRef: Nelson l\th/e ch. 354.21

MISCELLANEOUS

38. Ans. is'd'i.e., All of above fRef: Ghai 7h/e p. 469)


o Cystinuria is an autosomal recessive disorder, characterized by defect in transport ofcystine in renal tubule which
causes recurrent stones.
39. Ans. is'b'i.e., CCF tRel Nelson 18th/e ch. 86.21

Refsum disease
r Peroxisomal disorder.
r Defective enzyme - phltonoyl CoA oxidase.
r Clinical feature includes :- Impaired vision (retinitis pigmentosa), Icthyosis, Peripheral neuropathy & Ataxia.
40. Ans. is t' i.e., < 9o/o fRef: Nelson 18'h/e ch. 590)
o In children below 5 year of age, target HBA,C is 7 .5 - 9o/o.

180-250 7,s-9,0

ANSWERS OF VARIOUS OTHER EXAMINATIONS

41. Ans. is 'b' i.e., Homocystinuria fRef: Read belowl


o Pyridoxal phosphate dependent conditions are homocystinuria, oxaluria, cystathioninuria and xanthurenic
aciduria.

42. Ans. is 'b' i.e., Methyl-malonic acidem ia lRef: Nelson lfth/e p. 2202]
o For methyl malonic acidemia, Vit B12 is given.
43. Ans. is t' i.e., Recurrent urinary caliculi lRef: Nelson 18th/e p. 22021

Cystinuria
o It is an autosomal recessive disorder.
o The disease is caused by a defective high afinity transporter for L-cystine and diabasic amioacids present in the
proximal tubules.
o It is characterizeby recurrent kidney stone formation.
44.Ansist'i.e.,AlloftheabovelRef:A'P'Ghai9'hlep'661("7h/ep'63fl

+
Hurler/lH
Scheie's/l5
+
Hunter/ll
5anfliPo/itl
+
Morquio/lV
+
Maroteaux-LamY/Vl
+
Sity//ll
rvhich
.d, i.e., Exaggerated tendon reflexes [Rel; o.P. Ghai 8'h/e p. 662 & Thle p' 639, 561; Nelson 77'h/e p'
;15. Ans. is
2Ai2l
Ghai rth/e p' 652 d? Thle p' 629; Nelson 17h/e P. 399,40t)7
46. Ans. is.H i.e., Reduced tendon reflexes [Rel:"o"P'
e Reflexes are hlPeractive'

47. Ans.is1d'i.e.,Phenylketonuria[RefA.P'Ghai8'h/ep'652drVh/ep'6291
Sthle p' 320-321 & 7hle p' 6411
48. Ans. is'H i.e., Sensory changes tRef O'P' Ghai
Clinical features of wilson disease in children o KF ring
rr# o Acute or chronic liverdisease
i.#
::.:*** o Psychiatric disturbancesparkinsonism, 'e Hemolytic anemia

o Neurologicat -+ nigiJity, tremer, cerebellar ataxia. Fanconi syndrome

49. Ans. is'd i.e., Type I lRef : Harrison l7hle


p' 2458; AP Ghai &'h/e p' 6571
c Liver glycogenosis -) Tlpe I' try-tll'f IiIb' IV Vi' IX'
. glycogenosis -'>
trluscie Tlpe II' V' VII'
o In IIIa d iV ,rot.t. weakness and hlpotonia are also seen'

50. Ans.is.Hi.e.,Hartnupdisease[Ref Nelsom 78th/ep.539,54A;Ghai6hlep'611&7hlep'631]


amino aci<ls'
is aminclaciduria, which is restricted to neutral
o The main laboratory finding in Hartnup disease
r Plasma concentration of ne-utral amino acids remains normal'

rta
lrc
cF{rlffie€ffiffire
t:rlY T& ,f laT.} f
AU rVtugq.*

r Common childhood tumors are leukemias (AML, ALLlrea 08), lymphoma, rhabdomyosarcom&@'tr), brain
tumors@Gtot), neuroblastoma(Pct08), bone tumors, Wilm's tumors@G[08) and retinoblastomaNEEr), Most of these tumors
occtr in first 4rrr6rr*iu't, except for lymphoma which is uncommofl before 14 years@NB 12) .
t Most common childhood tumor is leukemia^'07' ee), especially acute lymphoblastic ieukemia.
c Second most common childhood tumors and most common solid childhood tumors are brain tumors.
t Third most common tumor of children is lymphoma.
o Fourth most common tumor of children and most common tumot in infancy is neuroblastomaqllMse3).
Fifth most common tumor of chiidren is Wilm\ tumor.
"
o Most common tumor infetus and neonate is sacrococcygeal teratoma6ll2),
: Most common abdominal tumor in children is neuroblastoma.
ts).
c Most common GI malignancy in children is lymphoma{et

ACUTE LYMPHOBLASTIC LEUKEMIA / LYMPHOMA (ALL)

c Acute lymphoblastic leukemia (ALL) encompasses a group of neoplasms composed of immature, precursor B (Pre-B)
or T (Pre-T) lymphocytes referred to as lymphoblast.
r Majority (85%) of all ALLs are precursor B-cell tumsls@crra) 1[ru1 tlpically manifest as childhood "leukemia'' with
extensive bone marrow and variable peripheral blood involvement. The less common precursor T-cell ALLs tend
to present in adolescent males as "lymphomas" often with thymic involvement@Ies'e4).
r Peak incidence of Pre B-cell ALL is evident between 3-5 years and ALL is the most common childhood cancer. Pre
T-cell ALL occurs in adolescent male.
c Tumor cells (lymphoblasts) contain cltoplasmic aggregates of periodic acid schiff (PAS) positive meterialare3).
Clinical rnanifestations of ALL
r Symptoms related to depression of normal marrow function.
r An accumulation of neoplastic blast cells in the bone marrow suppresses normal hematopoisis by physical crowding
and competition for growth factors. This results in : -
c Anemiaqlli) -+ Fatigue, Pallorat oEt
o Neutropenia(At") -+ Infection, intermittentfeyeratq4)
o Thrombocytopenia -+ Bleeding, p etechiaeql 0o),
ecchgnoses, epistaxis.
t These are the most common sign dr symptoms,

. Bone pain and tenderness


r Resulting from marrow expansion and infiltration of the subperiosteum.
r Generalized lymphadenopathy, splenomegaly and hepatom egalyat ts' o't1

r Caused by neoplastic infiltration ofthese tissues.


t More common in ALL than AML.
r Symptoms related to compression of large mediastinal vessels or airway
t In Pre-T ALLates,el) of thvmus.
. CNS manifestation
r Due to meningeal involvement.
r Headache, vomiting, nerve palsies.
t More common in ALL than in AML.
r Testicular involvement is also more common in ALL than in AML.

A {ftitd is diagnesed to taviag tentcninrtllii X-rny i* shown in the figure.


Type of leukemia ir -
A) AML , b) cML
c) T-cell ALL d) B-cell ALL

Ans. is'C i.e., T-cell ALL


r The given chest X-ray is showing anterior mediastinal mass. Mediastinal
mass is seen in T-cell ALL.

Cellular <lassifcation
r French-American-British (FAB) system divides ALL into three morphological subtypes :-
1) L,lymphoblasts
t It is most cammon type and has better prognosis. Cells have scanty cltoplasm and inconspicuous nuclei.
2) Lr lymphoblasts
I Cells arelarge and more pleomorphic in size with abundant cl,toplasm and prominant nucleus.
3) Lr lymphoblasts
r It is least common type. It is identical to Burkitt's lymphoma, i.e. Mature B-cellsat0T).
Prognostic factors in ALI-

Race White Black


Am l. l -: :.: 't,::::,i=i.*p-.,ii*,1i,1
:.4:
Sex FemaletAl t1. 0-. DNB 12. AtrMSae) MaletAtaT AtLv\02)

fyp.. Ll l111S.,lmNr:rsa..
Cytogenetics Hlperploidy{Dr: ts) Hypoploid/DNB 12' AI a7' AtlMS 02)

t (9: 12) Pseudodipioidy


t (72 : 21Sra"t u' "i az1 t (4 : 1 l)tAt tt)
Trisomy : - 4,10, L7 t (9 : 22){N ttt * BCR- ABL
t (8 : ll){dr ttt
t (1 : 19){dt ttt
CHePTen r,. a ChiI dfioaf farn6ls:,

> 50 X l0e/L(A,
tt D\B 12 \tt4sae'02)
T{,C 'a I 26.x1{Ot/lir}r3
15}

(-) (+)
LprphadenoPathY,
HepatosplenomegalY
{..}. . " - .'.. (+)
ldaniilgeal intolv€meet',, ,.

(-) (r)
Mediastinal mass
Early,ple'' $...ce
p7s -'8,:TslfAt!71'. . : :: l

.lmnrunoPhenolYPg . -r, ',


CD l0{t) Ye, ;
Mature B cell, T cell ALL
ot)
Absent Presefi{'o'IMs
Testicular involvement
' '. :, .i.$days,., ,, ', >.l8days: .:
Tiiirilof,remi.ssio-n:
< 10-4 < 10-3
Minimal residual disease

Treatntent ofALL
betwe en 2 and 2% years'
c Treatment of ALL is divided 4 stages. The total duration of treatment ranges

1. Induction of remission
is used to attain remision, i.e., to eradicate the
leukemic cells fiom bone marrorv'
r Induction therapy s4)
Anthracycline. Duration of therap,v
r Drugs used are ) Vincristine\t ts), preilnisolone, L - Asparginase(Ales. ,
is 4-6 weeks. .

1 r-hl€ *1"r.,-ny
i.iiJ +!r!4."r/

CNS involvement at the time of diagnosis' Moreover'


cNS acts as
r Most children with ieukemia have subclinical
site where leukernic cells are protected
from systemic chemotherapy because of blood brain barrier'
a sanctuary
so, treatment to keep leukemia from spreading to the cNS is often started with induction'
ce11s
eT)
rst
plus ctanial radiatianql
Treatment include -> Intrathecal methotrexatntt't

3. Intensification
into remission, the next step is to intensify the therapy
fcr a short period"
x Ifthe Patient goes
. Epipodophyllotoxin' Cyclophosphamide\I,o), Cytarabirct"
r Drugs used are -) Methotrexgte, L Asparginase,

4. Maintenance theraPY
atrd
on maintenance threrapy to maintaine the remission state
After consolidation, the patient is generally put
a
r
prevent re[aPse'
Predruisolane, Vincrist;ine,Duration of maintenace therapy
x Drugs used ar e - > 6-mer captopurine' Methotrexate'
is 2-2.5 vears.

(2 qnd testis'
lete remission ore t Bone marrow tmc), CNS mclNEEt)
* The common sites of t

cn bone m:xrrolr a$pi*


A ehitd with pareytnpcnia is having fuIlowiog slide
rate. Diagnosis is -
W@r,
&
a) ALL b) AML
c) CML d) None

Ans. io "d i,a, AI-tr,


q The slide is showing that marrow has been replaced by $mphoblasts
ofvarioussizes.NomyeloidorerythroidPrecursorsareseen'
MegakarYocYtes are absent'
AML is a cancer of myeloid line of wBCs,
' characterized by the rapid prolifer:ation
of abnormal cells which accumulate
in the bone marrow and interfere with the production
of normal blood cells. AML is the most
common acute leukemia
affecting adults.
o The diagnosis of AML is based on finding
that myeloidblasts make up more than
2Yo/oof the cells in the marrow.
r Myeloblasts are myeloperoxidase (peroxidase)
positive(pcr sr. Auer rods(,{r ei) (Represent abnormal azurophilic
granule) are also present in myeloblasts and
their presence is taken to be definite evidence
of myeloid differentiation.
Etiologyof AML
r AML has bee associated with Down ),
syndrome(Aro7,Kerala, Klinefelter synflyslnsaroz), patau
syndrome,roz),Fanconi
anemia, Bloom syndroms@u'"h04), Ataxia telangiectasia(KerataL4),and
Kostman syndrome. Other etiological agents
radiation' chemicals (benzene, ethylene oxide), are
and drugs (alkylating agentsGr t3),topoisomerase
o Neonates with down syndrome II inhibitors).
may develop 'transient myeloprolif)rative
disorder,6t0a) whichmay mimic AML.
e|assification of AML
c FAB classification divides AML into eight
types Mo to Mr.
* This scheme takes into account : _
i) The degree of maturation (M0 to M3)
ii) The lineage of leukemic blast (M4 to M7).

Mo minimally diferentiated AML

Clinical features of AML


r The clinical findings in AML are similar
to those in ALL i.e., suppression of normal
hematopoiesis in the marrow that
results in -
i) Anemia _+ Fatigue, pallor
ii) Neutropenia ,-+ Infection,
Fever
iil) Thrombocytopenia -+ Spontaneous mucosal and cutaneous bleeding.
This is the most striking clinicalfeature.
r Tumor cells in acute promyelocltic leukemia (Mr)
release procoagulant and fibrinolytic
factors that cause Disseminated
intravascular coagulation (DIC).
r Monocytic differentiation type (Mu and Mu) may present
with infittration of skin (leukemia cutis)
and gingival
hYPertroPlry{et sst.
. AML is a cancer of myeloid line of WBCs, characterized by the rapid
proliferation of abnormal cells which accumulate
in the bone marrow and interfere with the production of normal blood cells. AML is the most common acute leukemia
affecting adults.
o The diagnosis of AML is based on finding that myeloidblasts make up more than 20o/o of the cells in the marrow.
r Myeloblasts are myeloperoxidase (peroxidase) positive(Pcr e7). Auer rods(Ar e3) (Represent abnormal azurophilic
granule) are also present in myeloblasts and their presence is taken to be definite evidence of myeloid differentiation.

Etiology of AML
o AML has bee associated with Down Synd.romeql0T'Ke/ata14), Klinefelter syndrome\ror), Patau syndrome\I0T), Fanconi
anemia, Bloom syndrome(Kerata04), Ataxia telangiectasia(Kerata04), and Kostman syndrome. Other etiological agents are
radiation, chemicals (benzene, ethylene oxide), and drugs (alkylating agentsat 1s), topoisomerase II inhibitors).
s Neonates with down syndrome may develop'transient myeloprolifurative disorder'6r04) which may mimic AML.

elassification of AML
* FAB classification divides AML into eight types Mo to Mr.
* This scheme takes into account : -
i) The degree of maturation (M0 to M3)
i, The lineage of leukemic blast (M4 to M7).

Mn minimally diferentiated AML * Mfebperoxidasenegative


o Auer rods negative
I Express myeloid lineage antigen

MrAMLwith maturation (Most eommon) r Full range of myeloid maturation


r Myelo.peroxidasepositive
. Auer rods positive

M4 acute myelomonocytic leukemia I Both myelocytic & Monocytic differentiation


r Myeloperoxidase (+) ve )
l Auer rods (+) ve Mleloblastic

;
]
Nonspecific esterase (+) ve{il07) l Monoblastic

M" Acute erlthroleukemia c Dysplastic erythroid precursors


r !::rtlL
Myeloblasts seen in advanced age
:r: I : ::::ri.-:::-r::'..:

Clinical features of AML


o The clinical findings in AML are similar to those in ALL i.e., suppression of normal hematopoiesis in the marrow that
results in -
i) Anemia -+ Fatigue, pallor
ii) Neutropenia '-+ Infection, Fever
lii) Thrombocytopenia -) Spontaneous mucosal and cutaneous bleeding. This is the most striking clinicalfeature.
r Thmor cells in acute promyelocltic leukemia (Mr) release procoagulant and flbrinolytic factors that cause Disseminated
intravascular coagulation (DIC).
o Monocytic differentiation tJpe (M, and Mu) may present with infiltration of skin (leukemia cutis) and gingival
es).
hypertrophy(Al
CnAp.r,E.n

o Chloromas(Pclee) (granulocytic sarcomas or myeloblastomas) are localize masses


absence of marrow or peripheral blood involvement. These are seen in M4 & Ms.
o Lymphodenopathy, Hepatosplenomegaly, CNS involvement and testicular involvement are less common than ALL.

A child suffering from leukemia,is h*virg following figrre. The diagNosis ie


a) B-cell ALL
b) T-cellALL
c) MI AML
d) M4 AML

Ans.isU'i,e"; M4AML
o Gingival hyperplasia is more common in M4 & M5 AML.

Treatrnent of AML
c The most commonly used chemotherapy regimen is combinatron of cytarabine (cytosine arabinoside) and anthro-
cycline (Doxorubian or daunorubicin).3-4 cycles are given.
o Patients who fails to attain complete remission after two induction courses should immediately procee dto an allogenic
stem cell transplant (SCT).
o DOC far Promyelocytic AML is Tretinon (retinoic acid).

Treatment of relapse
r Patients etigible for allogenic SCTshould receive transplant expeditously at the first sign ofrelapse.
r If SCT is not possible :

r) Decitabine is the DOC(uPsce')


ii) Gemtuzumab azogamicin is an alternative

Prognostic factors of AML


o There are many prognostic factors for AML of particular note are : -
1 ) Age : - Age greater than 55 years or less than 2 years has been associated wlth poor prognosis independent of
cytogenetics.
2) AML evolving from prior myelodysplastic syndrome (MDS) is associated with poor prognosis.
3) Leukocl.tosis and/or an elevated peripheral blast count are associated with poor prognosis.

4) Cltogenetics : - AML prognosis is currently separated into three broad categories based on cltogenic results:-
l) Favourable: inv (761{dnmsozt, t (15:17), t (8:21).
ii) lntermediate: Normal(ArMsoz), 1 8, + 2L, +22, del (7q), del (9q), Abnormal 11q23.
il) Unfavorable: - Monosomy 5 or 76ItMs07), complexes with > 5 chromosomes involved, del (5q), abnormal 3q,
abnormal 17p.
5) Type : Mo, M6, & M7 types have poor prognosis (Mr, Mr(o"t), M. are associated with good prognosis).

BADPROGNOSIS

* Age < 2 years or > 55 years

*:,.M2.,4/i3, M4.fuirns af AML:;.;:,,,,. :,


',,,, ,.:M0;M$,M7forms.ofAMl ,.,' '. , .r" 1,,,,;,

* Blast cell with Auer rods . Complex karyotypes

r.*.:,Tle,<rs'X.:lOBlL.::,,:,,. ... .:,::. :.'::,. ..'.:, Q TLC>.100x.109/1


I t]s;t z), (821,),'r1u,.1.6w Ms.,on q Deletions 5q,7q (monosomies@iins07))
! ] :, ,.
,. aielrkimia:with'6ut'.p-recq.gdhgMD5.,,',
''{ :,"' :AMt:\4{ith.,preceding MDS or anticancer dru g expo5 ure

Hyperleuke(ytosis and Leukostasis


r Hyperleukocytosis refers to condition i.n which patients 'with acute leukemia present with an extraordinarv high
leukocyte count usually greater than 100,000/pl.
o Leukostasis (Also known as symptomatic hlperleukocytosis) is a medical emergency most commonly seen in
patients with AML or CML in blast crisis. There is sludging of leukemia blasts in the microcirculation (due to extremely
elevated blast cells) leading to obstruction of small vessels especially in the cerebral and pulmonary circulation.
o Treatment includes immediate cltoreduction by leukopharesis or by hydroqmrea. This is followed by induction
chemotherapy(" Supportive therapy in chdes IV hydrationq'on) , allopurinolGr 0e)
and alkalinization of urinsqt os;
". .

JEJEf ENITE MYELOMONOCYTIC LEU KEMIA

c luvenile CML is mostiy seen in children below 2 years of age.


c Itis philadelphia chromosome negativefiEEr) (in contrast to adult CML which is positive for pheladelphia chromosome).
o luveniie CML is associated with NF-1, monosomy 7, and deletion of long arm of chromosome 7.
o The child presents with thrombocytopeniaNEEr), bleeding, rash, lympharlenopathy(NEEr'), splenomegaly, xanthomas,
and cale au lait spots.
a The fetal hemoglobin (HbF) is raised@EEr' AItMs e7)
.

o The treatment of choice is hematopoietic stem cell transplantation (HSCT).

hI&EJROBLASTOMA

r Neuroblastoma is a malignant neuroendocrine tumor arising from any neural crest element of s1mpathetic autonomic
nervous systern(ruto).
r It is the most common malignancy of infancyqtqs), the most common abdominal cancer in childrenAltMs e7' AI e6' e4),

ls' 0e).
and most comrnon extracranial solid tumor of childhb/odur
10'oB))' ott1,
e Sites of involvement are adrenal medulla (mast commorr(Pcl pelvis, cervical area (neck(o"' posterior
o8),
mediastinumlea and paravertebral retroperitoneum.

e*inical features
o Clinical features are :-

l) Due to mass: Abdominal (flank) mass@EEr'A1e4) (most common presentation(Pcre3)), often crosses midline and
displaces the kidney downwords without comptressing collecting system@G108); cervical tumor may cause Horner
syndrome.
ii) Due to metastasis: Orbital proptosis@Gles), Raccoan eyes (wllateral or bilateral periorbital or eyelid ecchymosis),
fever, bone painar e4)
in skull with sutural diastasisat es' rcr rs), spinal cord compr'ession.
, lytic lesions
1i1) Due to catecholamine secretion by tumor: Tachycardia, headache, sweatindNEEr), episodic diarrhea@EEr'Ate4),

JlushinglAlea).
iv) (Incommonfeatures: Ogtsoclonus /opsotnyoclonusQcres), myastenia gravis.

r Metastasis is present in 6U-70o/oar0e) atthe time of diagnosis. Commonest site of metastasis is skeletal system@Gtqs) and
06' 02).
neuroblastoma is the most common childhood malignancy metastasizes to bonear Other sites of lnvolvement
are liver, lymph nodes and skin, Lung metastasis is uncommonqr 0e)
. Encasement of abdominal aorta and IVC may
i
occllr6los,Pclos).
c Bilateral periorbital ecchymosis ip metastatic neuroblastoma -+'Raccoan eye$'.

?ragnosis and treatment


r Good prognostic factors are age < 1 year, stage 1 and 2, Hyperdiploidy{'u oat l Triploidy, and Trk A erepression(Al 04) .
o Poorprognosticfactors areage> lyear; stage3 and4,N-mycamplification?r04'Pcto0)' deletionofchromosome l and
14; allelic loss of 11 q and 14 q; and gain of 17 q.
o Surgeryisthetreatmentofchoiceforstageland2.stages3(t4aretreatedbysurgerycombinedwithchemoradiation@Gl
87).

WI LM's TUMOR (N EPFIROBLASTOMA}

e It is the most common malignant tumor of kidney in children Most cases are sporadic. l-2o/o cases may have family
history.
r Familial predisposition is inherited in autosomal dominant manner. Mutation occurs inWilms tumor (WT-1) gene
on chromosome-7761"), CTNNB-1 gene encoding for protooncogene beta-catenin, and p53 gene.
r Wilms tumor is associated with three distinct syndromes :-
1) WAGRsyndrom e:Wilmt twor,Aniridia@Gl08'AilMse7'AIea), Genitourinary abnormalities@clq8) (includingllorse-
shoe kiilney(uPsc I0), Mental Retardation.
2) Beckwith Wiedemann syndromeqllMseT'AI e') : Enlargment of body organs (e,g. macroglossia(ot"'))
hemihypertrophy(rd08'Auussz'uea), renal medullary cyst, adrenal cltomegaly, and hypoglycemia.
3) Denys-Drash syndrome: Gonadal dysgenesis (Male pseudohermaphrodite), nephropathy with renal failure.
e Other associations with wilms tumor are Frasier syndrome, Sotos syndrome, Perlman syndrome, Simpson-Golabi-
Behmel syndrome, and Von-Willebrand diseasb.

!listology of Wilms tumor


e Wilms tumor comprises three elements : (i) Blastema; (rl) Mesenchyma; and (r11) Epithelium. lt is divided into :-
A) Favourable histology
r It is the usual form and carries good prognosis. It compries blastoma, epithelial and stromal (mesenchyme)
elements. Metastasis to lung may occur butbrain and bone metastasis is rareGIlMS04).
B) Unfavorable histology
r There is anaplasia. Clear cell sarcoma subtype metastasizes to bone(AilMs 0a).

Presentation of Wilm's tumor


At 0s)
o Asymptomatic ab dominal mass (mo st common )@EEr'

o Abdominal pain (30%)


o Hypertension (25o/o)

c Heamaturia (10-25o/o)
r Fever (200/o)
r Anorexia and vomiting
Prognosis
e Tumor stage at the time of diagnosis is the best prognostic indicator(4r06).
e Good prognostic indicators are stage 1 & 2, favourable histology, small tumor (< 500 gm) and age < 2yearc.
e Poorprognostic indicators are stage 3 &4, unfavourable histology, large tumor (> 500 gm), age > 2 years, hlperdiploidy
and renal vessel or capsule invasion.
Cnep:rrn.

e Bilateral periorbital ecchymosis in metastatic neuroblastoma -+ 'Raccoan eyes',

Frognosis and treatraent


o Good prognostic factors are age < 1 year, stage 1 and 2, Hyperdiploidy{u 1 Triploidy, and Trk A expressionqt .
oat on)

e Poor prognostic factors are age > 1 year; stage 3 and 4, N-myc amplificational14'
PGI00)
' deletion of chromosome I and

14; allelic loss of l1 q and 14 q; and gain of 17 q'


a Surgeryisthetreatmentofchoiceforstageland2.Stages3dt4aretreatedbysurgerycombinedwithchemoradiation@Gl
87).

WI *.M'5 TL.'MOR {ru EPHROBLESTOME}

* It is the most common malignant tumor of kidney in children lr'Iost cases are sporadic. 1-2% cases may have family
history.
* Familial predisposition is inherited in autosomal dominant manner. Mutation occurs in Wilms tumor (WT-1) gene
on chromosome-17(o,tn), CTNNB-1 gene encoding for protooncogene beta-catenin, and p53 gene.
c Wilms tumor is associated with three distinct syndromes :-
1) WAGR syndrorre : Wilnis ttmor, Aniridia?Gl0s' AilMSe7' Atsa), Genitortrinary abnormalities(Pc'0t) (including I{orse-
shoe kidney(uosc r0), Nlental Retardation.
08))
2) Beckwith Wiedemann syndrotneallMs e7' At ea) : Enlargment of body organs (e,g. macroglossia(Pcl
meduilary cyst, adrenal cytomegaly, and hypoglycemia.
hemihypertroqthy@Gt08,AttMss7'Atsa)n renal
3) Denys-Drash syndrome: Gonadal dysgenesis (Male pseudohermaphrodite), nephropathy with renal failure'
c Other associations with wilms tumor are Frasier syndrome, Sotos syndrome, Perlman syndrome, Simpson-Golabi-
Behmel syndrome, and Von-Willebrand disease.

Hister!ogy of Wilrms tumerr


e Wilms tumor comprises three elements : (1) Blastema; (ii) Mesenchyma; and (l:Lr) Epithelium.lt is divided into :-

A) Favourable histology
r It is the usual form and carries good prognosis. It compries blastoma, epithelial and stromal (mesenchyme)
0a)
elements. Metastasis to lung may occur btfi brain and bone metastasis is rareAilMs .

B) [Jnfavorable histologY
r There is anaplasia. Clear cell sarcoma subtype metastasizes to bone\rtMs 04)
.

Fresentation of Wilrn's turylor


AI0s)
e Asymptomatic abdominal mass (most commolt)(NBBr'
e Abdominal pain (30%)
e Hypertension (25o/o)
o Heamaturia (10 -25oto)
e Fever (20%)
o Anorexia and vomiting

Prognosis
* Tumor stage at the time of diagnosis is the best prognostic indicatorGl06).
* Good prognostic indicators are stage 1 & 2, favourable histology, small tumor (< 500 gm) and age < 2 years.

c Poor prognostic indicators are stage 3 & 4, unfavourable histology, large tumor (> 500 gm), age > 2 years, hyperdiploidy
and renal vessel or capsuie invasion.
Treatment of Wilm's tumor
o Treatment of wilm's tumor consists surgical resectionfollowedby chemo and ratiotherapy,

Wilms tumor
+
Surgical resection

Favorable histology Unfavorable histology


I
I

I
Stage III
Stage I & II I
Stage IV I
I
V
iI

Vincristin + +
Yiqcristin +
Vincristin Vincristin
+ + +
Ductinmycin
Dactinomycin Dactinomycin Dactinomycin
+
(actinomycin-D) + +
Doxorubicin
+ Doxorubicin Doxorubicin
+ +
Ratiotherapy to
Ratiotherapy to Cyclophosphamide
tumor bed
all sites -+ tumor bed + known +
sites for metastasis Radiotherapy to all sites

r Surgery involves transperitoneal radical n€phrectamy (radical nephroureterectotnyql


12)).

NEUROBLASTOMA VS WILM'S TUMOR

.l:.,,,.,3ii315;"i.]..'":,.,.:,.. J.
Age,:,... 1.:',:,,'".'::::,:.iilil:. :r, t.i rr', ;.. rr.r..::'.:.:.,.:,,

Clinical features Unilateral fl ank mass (asymptomatic abdominal Abdominal mass, GI or genitourinary obstruction, prop-
mass), abdominal pain, hematuria, hyperten- tosis, fever, bone pain, spinal cord compression, diarrhea,
sion, fever, anorexia, vomiting. flushfnS, olro:lor'.1 *I::1or'*: h"rdu.h...
..
Less common (15%) ;,lEEstc'r fs$j& ,i.,:;*1,{t1:.,, .

Midline Not crossed (confined to one side) Crossed

Airtii,,f: lW,itw *Ei.glti:,:

More common (40%)


, ...-
,Ai;*;4i7o7,,,;,:;:,':., ::, No (always intrarenal) l11:' .e.iiwE i-.*1i.,@.i$...$.!s€{${e:::::.:-=1;: . :,,:::

Retroperitoneal Less common (137o) More common (737o)


1rmpnad""!p@,.
Reiiaijiii1,at..,:,.. :,. :. -,.. .. ::. |r{g' l*oA-)
';:.,
*ipb "
Liver metastasis No (07o) Present (187o)
Cll.c.pren

An infant with large abdominal maes is pre*enting with fo$owtng X-r*y,


Diagxosi* iu -
a) Wilm's tumor b) Neuroblastoma
c) ALL d) Chest infection

Ans, is'H Le., Neuroblastoma


c Large abdominal mass with speckled calcification (as seen on X-ray)
suggestive of diagnosis of neuroblastoma.

&Syearx aMl&witk large abdomfutal nra** i* having following X- rry.*w


proba&lediagao*ts i* -
a) Wilrns tumor b) Neuroblastoma
c) ALL d) Chest infection

Ans. i$'d i.e,, Wilnde tumor


o X-ray is showing large soft tissue opacity in left upper quadrant, displacing
adjacent loops of bowel. Together with clinical findings, X-ray is suggestive
of Wilmt tumor.

[.ANGEftHAHS CELI HIST;OCYTOSIS (HISTTOCYTOSIS X}

* HistocylosisXischaracterizedbyproliferationofLangerhanshistioi:r.ies(tissu-ernacr'ophagesJ, Iii. ";,1


..-i;,t:i:.,Lir:ri,
th"ee caterg.crir:s :-
t) Lettrrer - -\iwe syndrafiTe(PGt e8)
ii] I'Iand -schuller-Christian diseflse(PcleB)
ui I Easinophilic granulama(Pct eB)

c'11.:rm*r::e11s in
each are d,erived from dendritic cells and e.*press S-100. CDla(zr''!'rtrr/ an,3 i"II-r!-1-rF,,'iir,. lrre:.trr,,-
of Birbeckgranules(ArMs03'PGI01) in the cltoplasm is characieristic. r-.rnder 1[s 6]r,;1;11r-:,1 yiir:lr:sr:,-'Ra, !rrL.r:!:- f!t:t:,..:1:::i
have a penta-laminar, rodlike, tubular apperance and sornetimes a dila"ied termirrai ei"d it*npis . ra.r:l:.-r 3:",ii.ir:1i!rir::i

Clinieal manifestataens at Sangerhans eeil histioeytosis


o It has diverse clinical features. These are :

1.) Skeletal involvernent :- It is seen in 80% of casss (75a/a cases lu'e -eent u'ith s*lii.rtr:, !.t,tic l.e siu.i. Skl,!r ,s tl: r*irqr
common site of involvement (Presents with solitary lytic l€.9i:3s\itiE'Er':r1{'s 'rt). l:r..:ol:rriirl,:iir. cl'spi:re l; ,1-,; 1.1 r;1-e!!:;;:5.;

of vertebral body and compression of spinai cord. In fiat long brories the lesicn is r:r!l<;4;11-{!11r ' :',''

'\rrMS0a),Irrvolement of mastoid bones leads to chroniea.ii,v infected ears.


2) Skin involement: It is seen in50o/o of patients. It usually manifests as seb,rrrhie dslrna.tr111r|"r''1"f itt'ii"t1"\i\)t) ,.i
scalp or diaper region.
3) llepatosplenomegaly(^Ttr\ AIIMS0l'A104): Seen in 2S9l+ cf patients. h4r.nifcsts ar i*ur<lice an,,! a.;rrii:p.
,1) Loealized or disseminated lymphadenopath,'r
5) Exophthalmos ; It i.c often B/L and is ca.useci. bv retrorbital acer-rrni.rlaticn ci grar,r'1r:rr:rrtoril 1!ssrrr

'7i Endocrine --> Hyptpituitaris'rn, Lll,ptt$,rt,itlist::. Lli*!;etes igsi1idgs {l'o!t;tyi$!''t'!ri'r: '::lii),!.

tability.
Treatment
o Localized (single system) disease is treated by intralesional corticosteroid, curettage or local radiotherapy.
o For Multisystem disease etoposide or vinblastin is given along with prednisolone.
o Refractory (unresponsive) cases are treated by cladaribine\ltMs0s), imatinib, cyclosporine, IFN-cr or stem cell
transplantation.

::: .:..,::
,',:a,' ,..1

.,:,t::,, ,.

: ,1::.: '

edges ',is,.e1assicil,, of Langerhans celi

OTHER PEDIATRICTUMORS

Germ cell tumors


c Germ cell tumors, as the name suggests, arise from Primordial germ cells (Precursor germ cells or gametoc),tes), Germ
cells develops early in life. In growing fetus, germ cells migrate from their point of origin to the gondal area. Germ cells
that still have to reach the gonads are called primordial germ cells. After reaching to gonads these become mature germ
cells and the end products of germ cell cycle are the egg or sperm. Germ cell tumors arise from primordial germ cells.
o Therefore germ cells tumor may be :-
1) Gonadal (In testis or ovary)
ii) Extragonadal : - When primordial germ cells fail to reach the right location and divide at the site of arrest. Extra-
gonadal germ cell tumors may occur at mediastinum (most common site), retroperitonium (2nd most common
site), brain, pineal gland, sacrococcygeal region.
r Germ cell tumor are : -
t Seminoma/ dysgerminoma
r Choriocarcino/na(Pcl oe)

t Emb r y on al ar cinom aQGI )


c
oe

t Teratoma
a Yolk/sac (endodermal sinus) tumor(PGt0e)

Rhabdomyosarcoma
: The most common sofi tissue sarcoma in children is rhabdomyosurcoma@Glee). The common site of involvement are: -
t Head.6 Neck (25o/o) -+ Most commorx(Pclqe)
t Genitourinary (22%) -+ 2nd most common
r Extremities
r Retroperitoneum
r Rhabdomyosarcoma is the most common nonocular orbital tumor is young children and is most common urinary
e7).
bladder malignancy in children(AllMs
o There are four histological rhabdomyosarcoma :-
i) Embryonal tyPe (60o7olrtctoat l
ii) Botryoid type is a type of embryonal form
J
Corn,ro., in childhood
iii) Alveolar tlpe (15%)
iv) Pleomorphic type (adult form) is rare in childhood and accounts for only 7%o of cases@Gt
os)
.
CHaptrn

I Stem c:

:s), Gern:
ierm cells
Lri'e gerru
:.n cells.

.i. Extra-
cofttlttotl

! l't !1f,!-tl )
x.t:* ChildhoadTamors

{ Most common urinary bladder malignancy in children: Rhabdomyosarcoma.


,rr Most common site rh*bdamyossrcama in children: Head & neck (mc), genitouJinary {2nd mc),
oif

a Type of rhabdomyosarcomo not seen in children: Pleomorphic type (adult form).

* Most <ommon inherited childhoo:d tumor i Eetinablastoma.


* iliattest cure rat.e in solid tumor in children: Retinoblastoma.
* ;fftportairlt'turn:ors causing proptosisin children: Retinoblastoma, rhabdornyosarcoma, lymphoma & fymBhosareoma, secondrles
ineuiablastcma, nephrobla:toma, Ewingt sarcoma,leukaemic infiltraticn). , i,

tauses of abdominal mass calcification in children: Cysts of adrenal gland, neuroblastoma, Wilm's tumor, pheochromocytoma ,-:f
adrenel, adrenal cortical carc!noma, ganglioneuroma.
& Mast cammon inherited malign*ncy in childhsod: Retinoblartoma.
& Differentia! diagnasis of lytic lesions of bone in children: Brown tumor, metastasis (neuroblastoma, leukernia), histiocytosis.
Highest are ra{e of solid tumor in children : Retinoblastoma, ::

@ l4ost runtmon benign tumor of infoncy:Hemangioma.


& *enefarretinablastama:FBlgeheon'chromosome13, ' . .. ,' -

6 t:-,er,e f*r neurofibromatosis-|: NF-1 gene on chromosome 17.

,& Gene fsr neurofibromatosis-2: NF-2 gene on chromcsorne 22.' ,

& t;1jilftis tumor gene :Wf-1 gene on chromosome-1 1.

& Most eernmon Gl malignancy in children: lymphoma.


6 ;'tr:!l "ottnd blue ce!l tumors in chiidren: Neuroblastoma, Wilm's tumor, rhabdomyocarcoma,
retinoblastoma, Ewing s sarfo.na- pri*-
itive neuroectodermal iumor lymphoma.

I]I

I
:jl6-i &,,,e*,,1-a Childhaod rumors

& Most common urinary bladder malignancy in children: Rhabdomyosarcoma.


, Mast camman site af rhabdomyosarcoma in chifdren: Head & neck{mc}. genitourjnary {2nd rnc}. :

s Type of rhabdomyosarcoma not seen in children : Pleomorphic type (adult form).


t Masteammoninheritedchildhoodfumor:Retincblastoma. r .

a llighest cure rate in solid tumor in children: Retinobiastoma.


* impart*nt tumars uusing proptosis in children':'Retinoblastorna, rhabcomyosarcoma,.lymphqma & lymph*sareoma. secoRdries
{rieureblastoma, nephroblastor,na, Ewing! sarcoma,leukaemic infiltration}.
eauses of abdominal mass calcification in children: Cysts of adrenal gland, neuroblastoma, Wilmt tumor, pheochromocytoma r:f
ar:rena!, adrenal cortical carcinorna, ganglioneuroma.
a M ast cc in ffi on i nh e ri ted m al i g na n cy i n chi l dhood ; Reti noLrtastoma.'
I Di{ferentia! diagnosis of lytic lesions of bone in children: Brown tumor, metastasis (neuroblastoma, leukemia), histiocytosis.
* Highest cure rate of solid tumar in children :.Retinoblastoma. :

& IrLo];t ran-rmon benign tumor of infancy:Hemangioma.


* Geiefcrretinoblastoma:RBl geneonchrsmosomel3. , . , ': ,1 .:l . ,.'l
,,
& Gerie frsr neurofibromatosis-\: NF-i gene on chromosome 17.
* #ene'fir neurofibrarnctasis.Z: NF-2 gene on chrornosorne 22;
Wiitr{s tumor gene :WT-i gene on chromosome-1 1.

& Most cammon Gl malignancy in children: Lymphoma.


& Stnr;il round blue cell tumors in children: Neuroblastoma, Wilmt tumor, rhabdomyocarcoma, retinoblastoma, Ewing's sarconr4 prim-
itlve n*uroectodermal tumor lymphoma.

rtl
Cnlpre B"ro
QUESTIONS

TRUE statement regarding childhood malignancy c) Immature T cell


ls- (AIIMS Dec 91) d) MlredBcell&Tcell
a) Wilm's tumour is the commonest t1" Poor prognostic indicator of ALL is - (AIIMS May 02)
b) AML is the most common malignancy a) Female sex
c) Neuroblastoma arises in the first 4 years b) Leukocyte count < 50,000
d) None of the above c) Age greater than 1 year
lLlntti 111"f.i:4!-q" r"
.\1O.1 rOno.nr0,t d) Hlpodipioidy
a) Leukemia (All India Dec.14 Pattern) t2. All of the following are good prognostic factors for
b) Lymphoma childhood. AII except - (AI o?)
c) Wilm's tumor a) Hlperdipioidy
d) Neurobiastoma b) Fernale sex
Which of the following is the most common tumor c) Pre B cell All
of the fetus and newborn - 6t12) d) t (12:21) translocation
a) Neuroblastoma t;
*i){}Apr{}gn.}:''rz ix;ltttt r.;i AL1- :tl'i; 21,1. t::\ii?\. '
b) Wilm's tumor a) Hlperdipoidy (t-ET-lj.r,. i-. r:',:r, " ;

c) Leukemia b) WBC cound < -5000


ti) Sacrococcygeal Teratoma c) Age > 10 years
N'Dt. ***t. itz *r*t di:*a*le - (NEET Det 12 Pattern) d) T (12:2i) translocation
a) Retino blastoma L4, The poor prognostic factor associated with ALL in
b) Rhabdomyosarcoma children is - (AIIMS Nov 09)
c) Neuroblastoma a) Total leucocyte count 4000-100000
d) Ameloblastoma b) Age less than 2 years
17' xepfurwwa i* c*|ld' r crz V*e&ts n:ix*ntll't ** rt a?-v,h.ai c) Testicularinvolvement
as* ? (CET'Aug 12 Pattern) d) Blast in peripheral smear
a) < 4 years b) < 14 years 15. Which of the following is poor prognostic factor in a
c) > 14 years d) < 1 year child with ALL? (AI i i)
a) Age 2-8 yrs
ACUTE LYMPHOBLASTIC LEUKEMIA b) Total leucoclte count < 50,000
c) t (9; 22);t (4;rt); t (1;19)
Most of the ALLs have (PGt e8) d) Absent blasts in peripheral smear
a) B-cell origin b) T-cell origin 16. L-asparaginase is particularly used in which type of
c) NK cell origin d) None -
leukemia @I e4'
':'i
A year old child presents with history of fever off-
5 a) AML b) CML
and-on for past 2 weeks and petechial spots all over c) ALL d) CLL
the body and increasing pallor for past 1 month. 1; {-rttz:tvr,rxt*v1- ,"1!.* o.:{ *v-rrar*td*i1a"rt rtl'*i:tr: tii ,l i -1.

(NEET Ili*.12 Fatlern)


Examination reveals splenomegaly of 2 cms below
costal margin. The most likely diagnosis is - a) CNS b) Lung
a) Acute leukemia @I o1) c) Liver d) Testis
b) Idiopathic thrombocytopenic purpura 18. Which of the following tnres of leukemia is
c) Hlpersplenism administered prophylactic methotrexate for CNS
d) Aplastic anemia prophylaxis - rlIq. i

Vrzt* a?:qs*|" &L1" - (NEET Dec.12 Pattern) a) ALL b) AML


a) Anemia b) Leukopenia c) CLL d) CML
c) Hepatomegaly d) All of the above Tlr,;gt, t:,*.* t;t hi.|. in qk id, ;*";: ';-i1 *:t-r;*i;t-
9. Which of the following presents as mediastinal a) methotrexate (All Indis Dei:15 Pattern)

enlargement - (Al es, 91) b) Vincristine


a) PromyeloclticLeukemia c) Vinblastine
b) CML d) Cyclophosphamids
c) ALL
d) Diffuse histiocytic lymphoma ACUTE MYELOGENOUS LEUKEMIA
10. ALL L3 morphology is a malignancy arising from
(Ar 07) 2{:. yt' tigt:* : :t', I
which cell lineage - A 1,.51. !: *:,?,. t1 E1'1',,u: :!-ti: -
"
a) Mature B cell (All hdia Decl5 Pattern)
b) Precursor B cell a) Acute megakaryocy'tic leukemia
b) Acute monocltic leukemia b) Fetal Hb is increased
c) Acute promyeloblastic lukemia (M.3) c) Philadelphia chromosome is positive
d) Er1'thro leukemia d) Lymphadenopathy
21. Good prognostic factor for AML - (cET luly 15 Pattern) 31. Increased fetal Hb is seen in - (CET lune 14 Pattern)

a) Age less than 2years a) |uvenile CML


b) Acute promyelocltic leukemia b) Congenital red ceil aplasia
c) Acute mega karyoblastic leukemia c) Hereditary spherocytosis
d) Associated with down syndrome d) AML
22. Non specific est€ra$e in prisent in- (PGt Dec 97)

a) Megakaryoclticleukaemia NEUROBLASTOMA
b) Lymphocyticleukaemia
c) ErFthroleukaemia 32. The most comrnon malignant neoplasm of infancy
d) AML is-
23. Non specific esterase is positive in all the categoriee a) Malignant teratoma (AI0s)

of AML except - (AI 07) b) Neuroblastoma


a) M3 b) M4 c) Wilms'tumor
c) M5 d) M6 d) Hepatoblastoma
24. Chloroma is due to- (All lndia Dec.14 Pattern) 33' A l-year-old child presented with a swelling in the
a) AML left flankwith episodes of flushing, diarrhea,
b) CLL sweating and bone pain. The diagnosis is - (At e4)

c) ALL a) Neuroblastoma
d) Non Hodgkin's lymphoma b) Wilm's tumor
c) Medulloblastoma
25. A child with acute myeloid leukemia presents rryith
d) Pheochromocltoma
hyperleukocytosis. Treatment includes all of the
following except - (Ar oe) 34. Most common presentation of neuroblastoma is -
a) IV fluid a) Lltic lesion in skull with suture diasthesis
b) Allopurinol b) Lung metastasis (At e8)

c) Alkalinization c) Renal invasion


d) Immediately start induction chemotherapy d) Secondaries in brain

26. All of the following syndromes are associated with 35. 4 years old child having palpable abdominal mass &
AMlexcept - @to7) hlryertension wit}t sweating & diarrhea is due to -
a) Down's syndrome a) Neuroblastoma (NEET Dec.12 Pattern)

b) Klinefeltert syndrome b) Nephroblastoma


c) Patau syndrome c) PCKD (Polycystic kidney disease)
d) Turner's syndrome 36. A mal.ignant tumor of childhood, that metastasizes
27. Genetic risk factors for leukemia are all except - to bones most often, is - (Ar 06,02)

a) Downb syndrome (Kerala o4) a) Wilm's tumor


b) Bloom syndrome b) Neuroblastoma
c) Ataxiatelangiectasia c) Adrenal gland tumors
d) Tirrner's syndrome d) Granulosa cell tumor of ovary
37. tvYhich of the following statements about
28. Transient myeloproliferative disorder of the aew-
born neuroblastoma is not true - (Ar oe)

is seea ia association wirh - @I 03) a) Most common extracranial solid tumor


a) Turner symdrome inchildhood
b) Down syndrome b) > 50 % present with metastasis at the time
c) Neurofibromatosis ofdiagnosis
d) Ataxiatelangiectasia c) Lung metastasis are common
d) Often encase aorta and its branches at the timeof
29. Predisposing factor for leukemia - (NEET Dec.12 Pattern)
diagnosis
a) Down syndrome b) Lowet syndrome
c) Fragile-x-syndrome d) None 38. Opsomyoclonus is encountered as C/F of -
a) Meningioma (NlIndiaDec.l4Pattern)
b) Neuroblastoma
JUVENITE CML c) Neurofibromatosis
d) Excision
30. All of the following are features of juvenile CML
To which of the following events is'good outcome
39.
except - (NEET Dec.12 Pattern)

a) Thrombocytopenia
in neuroblastoma associated - 6t 04)
a) Diploidy
.. :r'i:r.r..-...rrr:'..::.r..,,.i:ir.I
. ...: j:r.1,r.;::4.::r:t,.,tt:j:r
..
. -r.......,.1:1......,::;..:.::t.-

CH:$tiEa;,ililg

b) N-myc ampiification c) Chemotherapy


c) Chromosome 1p depletion d) Observation
d) Trk A expression
Neuroblastoma-good prognostic factor is - LANGERHANS CEI.L HISTIOCYTOSIS
a) N-myc amplification (PGl lune 2k)
b) RAS oncogene 49. Histiocytosis X is seen in excePt
c) Hlperdiploidy (All India Dec.14 Pattern)
d) Translocation a) Hand schuller Christian disease
Which is the cornmonest abdominal mass in neo- b) Eosinophilic granuloma
nate - c) Letter-siwe syndrome
a) Wilm's tumor (AIIMS Feb 97, AI 96,94) d) Torres syndrome
b) Polycystic kidney 50. A 2-year-oldchild comes with discharge, seborrheic
c) Neuroblastoma dermatitis, polytrria and hepatosplenomegaly.
d) Rhabdomyosarcoma IrYhich of the following is the most likely diagnosis -
t:. Child coming with proptosis, multiple skeletal limtt a) Leukemia (AIIMS May 01, Al 01' 94)

secondaries,suturalseparationishaving - (PGt95) b) Lymphoma


a) Neuroblastoma c) Langerhan's cell histiocltosis
b) Medullobiastoma d) Germ cell tumour
c) Retinoblastoma 51. Baby with re(uf,rent infection of ear & discharge &
d) None ofthe above eeborrheic dermatitis wlth hepatosplenomegaly with
+ 1- On USG a firass was found in atrdornen which was cystic skull lesions. Diagnosis is - (NEET Dec.12 Pattern)

displacing the kiduey laterally in I year old child- a) Hemophagocy'tic lymphohistiocltosis


(All India Dec15 Pattern) b) Langerhans cell histiocytosis
neuroblastoma b) nephroblastoma c) ALL
RCC d) All of the above d) Multiple myeloma
q? A child presented in the OPD with multiple
WILM'S TUMOR permeating lesions irwolYing all the trones of the
body which of the following is the most protrable
14. l{ilrds tumor is aesociated with all the following diagnosis - i.4ltilM'Jd 20aa)

except - 6llMS lune 97' AI 91) a) Neuroblastoma


a) Aniridia b) Metastasis from osteosarcofila
b) Beckwith syndrome c) Histiocfosis X
c) Polycystic kidney d) Metastasis from Wiim's tumour
d) Hemi-hypertrohy 53. A one year old boy presented with
45. A 7-year old troy with left renal mass had bone pain hepatosplenomegaly and delayed milestones. The
and was detected to have bone metastatic deposits. liver biopsy and bone marrow triopsy revealed
The most likely renal tumor is - (AIIMS Not 01) presence of histocltes with PAS positive. Electron
a) Favorable histology Wiims tumor rnicroscopic examination of these histiocytes is
,l.ttt'ts os)
b) Renal cell carcinoma most likely to reveal the presence af -
c) Clear cell sarcoma a) Birbeck granules in the cytoplasm
d) Rhabdoid tumor b) Myelin figures in the c1'toPlasm
)e) 46. The rnost cointnon presentation of a child with c) Parallel rays of tubular structures in lysosomes
Wilms' tsfirof is - (NEET Dec.12 Pattern, AI 05, AIIMS Dec 95) d) Electron dense deposit in the rnitocleondria
a) As asymptomatic abdominal mass 54- A2year old child presents with scattered lesions in
b) Haematuria the skull. Biopsy revealed Langerhans giant cells'
c) Hypertension The most cornmonly associated is rnarker with this
d) Hemoptysis due to pulmonary secondary condition will be - (AIIMS Nov 07)

47" The most important determinant of prognosis in a) CD la


wilrns tumor @t 06) b) cD s7
a) Stage ofdisease c) CD3
.n) b) Loss of heterozygosity of chromsome I p d) cD 68
c) Histology 55. True about Langerhan's histocytosis')(- (PGI Nov 11)
d) Age less than one year at presentation a) Can be associated with diabetes insipidus
48. r,{hich of the following is the treatment of choice for b) X-ray shows pathognomonic osteosclerotic lesions
Stage-I Wilm's Tumor - (Ar 12) c) Birbeck's granules in langhan's cell
)1) a) LaparoscopicNePhrectomY d) Proliferation ofantigen presenting cells
b) Open Nephroureterectomy e) Associated with sPecific HLA DR
56. Which of the following drugs is used for the c) Metastasis d) Histiocl,tosis
treatment of refractory histiocytosis - (AllMS Nov 08) 66. Highest cure rate is of - (NEET Dec.12 Pauern)
a) High dose methotrexate a) Wilm's Tumor b) Retinoblastoma
b) High dose cltarabine c) Rhabdomyosarcoma d) AIl
c) Cladribine
d) Fludrabine
QUESTIONS OF VARIOUS OTHER EXAMINA.
TIONS
MISCELLANEOUS
67. Most common malignant tumour in childhood -
57. fulost common Gt malignancy of childhood
a) Rhabdomyosarcoma (UP 07)
(All lndia Dec15 Pattern)
b) Leukemia
a) Adenocarcinoma b) Lymphoma
c) Lymphangioma
c) Sarcoma d) Carcinoid
d) Neuroblastoma
58. Cancer management in which of following
68. Second most common neoplasm seen in children is-
malignancies has dramatically increased the
a) Lymphoma b) Brain tumour (Karn )1)
survival? (AIIMS May 12)
c) Wilm's tumour d) Neuroblastoma
a) Esophagus carcinoma
b) Gliobiastoma multiforme 69. Which is the commonest childhood tumor -
c) ALL in children a) ALL b) CLL ]ipmer 0i)
d) Cholangiocarcinoma c) AML d) CML
59. Germ cell tumor(s) of paediatric includes all except- 70. A family of a child just diagnosed with acute
a) Pure yolk sac tumor (PGI Nov 09) lymphoblastic leukemia asks about the child's
prognosis. Which of the following is a poor
b) Leydig cell tumor
c) Choriocarcinoma prognostic indicator ? (UPSC-t oe)

d) Embryonal cell carcinoma a) Presence of mediastinal mass

e) Endodermal sinus tumor b) Age between I and 10 years


c) Hyperploidy with more than 50 chromosomes
60. Commonest tumor of face in chidren is - (PGI Dec 99)
d) WBC count less than 50,000/mm3 at diagnosis
a) Rhabdomyosarcoma
b) Sq. cell carcinoma 71. CML in children is associated with - (NLMHol)

c) Basal cell carcinoma a) Downt slmdrome


d) Mixed parotid tumor
'b) Klinefelter syndrome
c) Marran's syndrome
61. True about childhood tumor are all except -
d) Tirrner's syndrome
a) Wilmt tumor (PGtNov 14)

b) Neuroblastoma 72. A I year old child presenting with abdominal mass


c) Retinoblastoma and calcification on X-rays is suggestive of -
d) Embryonalrhabdomyoma a) Teratoma (PAL 93, PGI 86,93)

e) Pleomorphicrhabdomyosarcoma b) Neuroblastoma
c) Wilms tumour
62. Most common urinarybladder tumour in chifdhood
d) Rhabdomyosarcoma
1S- (AIIMS lune 97, AI 92)
a) I{aemangioma 73. A 2 day old newborn baby presented with micro-
cephaly, macroglossia, visceromegaly and a blood
b) Rhabdomyosarcoma
glucose level of 20 mg/dl. What is the most likely
c) Transitionai cell carcinoma
d) Squamous cell cdrcinoma diagnosis
a) Prader-Willi syrrdrome (orissa 98)
63. Commonest sarcoma in children is -
b) Beckwith Wiedman syndrome
a) Rhabdomyosarcoma (All lndia Dec.14 Pattern)
c) Werner syndrome
b) Lipoma
d) Cockayne syndrome
c) Angiosarcoma
d) Fibrosarcoma 74. Wilms'tumor is associated with the following except
64. Which of the following is the most common inherr"t-
a) Aniridia (UPSC-r 1o)
ed malignancy - (AIIMS May 0s)
b) Horse-shoe kidney
a) Infant leukemia b) Retinoblastoma
c) Hemihypertrophy
c) Wilmt tumour d) Neuroblastoma
d) Opsoclonus
65. A 10 years old child has lytic lesions in upper fe.mur,
75. Most common benign tumours during infancy is-
the differential diagnosis can be all except-
a) Lymphangioma b) Hemangioma (UP 07)
(AIIMS Dec 97)
c) Cystic hygroma d) Lipoma
a) Plasmacltoma b) Browns tumour

ITI
ANSWERS
1. Ans. is 'c'i.e.,Neuroblastoma arises in the first 4 years [Rel Nelson 18th/e p. 2097; Ghai 8th/e p. 616 & Vh/e p.
seal
Childhood malignancies
i)Most common tumor in childhood is leukemias especially acute lymphobiastic leukemia.
Second most common tumor -+ Brain tumors
ii)
iii) Third most common tumor -) Lymphomas
iv)Fourth most common tumor *) Neuroblastoma
v) Fifth most common tumors ) Wilms tumor
o Most of these tumors occtr below the age of 5 years except for lymphomas, rvhich is uncommon before 14 years of
age.

,,
Ans" is'l i"e.,I-eukemias lReJ Nelson 18th/e p. 2097; O. P. Ghsi 8'h/e p. 599 d* 7h/e p. 58Al
3. Ans. is'il i.e.,SacrococcygealTeratoma
c Sacrococcygeal Teratoma (SCT) ,s the most common neoplasm in the fetus and neu'barru.
'sacroccocygeal Teratoma (SCT) ,s the most common neoplasnl in the .fetus ond newlrarn with an estimated
prevalece of 1:30,000- 1:40,000 and with a 3:1 female preponderance.
- Handbook of Fetal Medicine by Shailesh Kumar (Cambridge [Jniv, Press) 2t]10/87

, Most common tumor in fetus and neonate -) sacrococcygealteratoma


. Most common tumor in infancy -+ neuroblastoma

4" Ans. is'd' i.e., Ameloblastoma [fiel O.P" Gh&i 9th/e p. 599 & Vh/e p. 59a]

o Ameloblastoma is not a childhood tumor.


r Other three are childhood malignacies.
5" Ans. is t'
i.e., > 14 years [Rel Nelson 19'h/e p. 1739]
r Lymphoma is the third most common cancer in pediatric age group.
I It is the most common cancer in adolescence accounting for > 25o/a of neu.'ly diagnosed cancers in persons 15 to 19

years old.
r Hodgkint lymphoma is the most common malignancy in adolescence.
r It is uncommon before 14 years of age.

ACUTE LYMPHOBLASTIC LEU KEMIA

6. Ans. is '*
i.e., B-cell origin lRef: O. P. Ghai 8th/e p. 600 & 7/e p. 5801
r Majoritp(85o/o) of all ALLs are precursor B-cell tumors that typically manifest as childhood "leukemia" n'ith
extensive bone marrow and variable peripheral blood involvement.
7. Ans. is'a' i.e., Acute leukemia lRef: O.P. Ghai 8th/e p. 601 6 Vh/e p. 5811
r Pallor (anemia), petechial spot (thrombocltopenia), fever (infection due to neutropenia) with splenomegaly suggest
the diagnosis of acute leukemia.
r Further, presence of splenomegaly eleminates aplastic anemia and ITP as the possibilities
"splenomegaly and lymphadenopathy are extremely uncommon in ITP and their Presence should lead one to
consider other possible diagnosis". - Robbini
"splenomegaly is characteristically absent in aplastic anemia and if present the diagnosis of a plastic
anemia should be seriously questioned".
r Hypersplenism' is a condition characterised by splenomgally and cytopenias with a normal hlperplastic marrow
and response to splenectomy..
Although cl,topenias resulting from hypersplenism may give rise to most manifestations mentioned in the question
these are more likely with massively enlarged spleens and not with minimally enlarged spleen as in the question
above (2cm below costal rnargin in a 2 year old child).

8. Ans. is'd' i.e., All of the above fRef: O"F. Ghai Vh/e p. 58A; Nelson L8'h/e p. 2115-21251

9. Ans. is t' i.e., ALL lRef: O.P. Ghai 8th/e p. 601 (t Vh/e p. 581)
"T cell vaiant of ALL presents with a wide mediastinal mass" - Ghai
10. Ans. is 'i i.e., Mature B cells tRef: O.P, Ghai Sth/e p. 600 b Vh/e p. 581; Robbins 7"/e p. 6771
r Lrmorphology is identical to Burkitt's lymphoma cells i.e., mature ts cells with surf'ace immunoglobuiin positive.

11. Ans. is ? i.e., Hypodiploidy lRef: o.p. Ghai 8th/e p. 600-601 6 Th/e p. 581; Nelson 18th/e p. 2120; Wintrob's
1lth/e p. 2145, 242)
. Poor prognostic cytogenetic abnormalities of ALL: Hlpodiploidy; pseudohlpoploidy; t(4:11); t(9:22) i.e., BCR-ABL;
t(8:14); t(1:19).
t2. Ans. is 'c' i.e., Pre B-Cell ALL {Ref: O.P. Ghai Bth/e p. 600-601 6 Vh/e p. 581; Wintrobe's llth/e p. 2145, 2421
o Pre B cell ALL has poor prognosis.

13. Ans. is t' i.e., Age > l0 years [Ref Devita's 6rt,/e p. 224A; Nelson 17h/e p. 1695; Wintrobe's clinical her,mtalagy
lLn/e Table 80.2)
r Age more than 10 years has bad prognosis.

14. Ans. is 'c' i.e., Testicular involvement fRef: Nelson 18e/e p. 2120; Wintrobe\ llth/e p. 2145; O.p. Ghai
8th/e p. 600-6011
o Testicular involvement is a poor prognostic factor.

15. Ans. is t'i.e., t(9;22);t (a;f l); t (1;f9) fRef: O.p. GhaiB,h/ep.600-6011
16. Ans. is t' i.e., ALL lRef: O.P. Ghai Bth/e p. 60i & fl/e p. 582; Nelson IBh/e p. 21191
o Induction therapy is used to attain remision, i.e., to eradicate the leukemic cells from bone marrow.
r Drugs used are -+ vincristine, Prednisolone, L - Asparginase, Anthracycline
17. Ans. is'a'i.e., CNS
o The common site of relapse of ALL --> (1) Bone marrow (most common - 200/o), (1i) CNS (5o/o) (iil) Testis (3ot6)
18. Ans. is'a' i.e., ALL fRef: O.P Ghai 8th/e p. 60j & Vhle p. 583)
o Most children with leukemia have subclinical CNS involvement at the time of diagnosis.
r Treatment include -+ rntrathecal methotrexate plus cranial radiation.
19. Ans. is t' i.e., Vinblastine {Ref: Ghai Ze/e p. 582)
o Vincristine, cyclophosphamide and firethotrexate are used (see text of the chapter)"

ACUTE MYELOGENOUS LEUKEMIA

20. Ans. is t' i.e., Acute promyeloblastic leukernia (M.3) tRel Ghai fl/e p. 584; Nelsan 1re/e rh. 49Sl
o {)apd ltrognosis : M, (AML with maturation), N{. (acute prornyelocl.tic ieukernia), Mn (acute myeloplonocytic
leukcmia).
c Faor prognosls : Mo (ririnilnaiiy diferentiated AML), Iv{u (acute er1.thro1eu.l:,mia), M, (acute megakaryocytic).

21. Ans. is'b'i.e., Acute promyelocytic leukernia {Ref: Readbetow)


o Acute promyelocytic leukemia (Mr) has good prognosis"
o Acute megakaryocl.tic leukemia (Mr) has poor prognosis.
o Age less than 2 years and > 55 years have poor prognosis.
'r1 Ans. is'd' i.e., AMt tRe/ Robbin's 7,/e p.692]

23. Ans. is 'd' i.e., Mu[Ref : Hematology Basic Principles and Practice by Hoffman, Benz et al4th/e p. 10g0)
r Non-specific esterase is seen in AMLs which have monocltic Lineage (M, & Ms ).
r In M. AML, usually non-specific esterase is usually not seen. But some cases miy show non-specific esterase
" Acute promyelocytic leukemia shows immature cells that are strongly myeloperoxidase positive, but some cases may also
show not-specific esterase positivity". --- Textbook ai lllustrated pathology
l Aees. is'a' i.e", Ae{L tRd.' A.F. Ghai }tt'/e p. 506 {, 7t'/* p' 584; trlabbin's Thle p. 694}

r Chloromas (granulocytic sarcomas or myeloblastomas) are localize masses composed of myeloblasts in the absence
of marrow or peripheral blood involvement in Mn & Mu AML.
25. Ans. is'd' i.e., Irnmediately start induction chemoth erarpy
o Treatment of hlperleukocltosis includes :-
1. Immediate leukoreduction (cltoreduction)
r Immediate leukoreduction is required to lower the blast cells'
r This can be achieved by leukopharesis or by administtation ofhydroxyurea.
2. Induction chemotheraPy
r Induction chemotherapy is initiated early, but only after attempts at leukoreduction (not immediatly) as immediate
administration of induction chemotherapy carries a high risk of acute tumor lysis syndrome.
r So, early induction chemotherapy is advised but not immediately.

3. Supportive/Prevention of complications
r Supportive therapy is given to prevent/reduce the metabolic complications of tumor lysis syndrome.
r It includes IV hydration, allopurinol and alkalinization of urine,

26. Ans. is 'd' i.e., Turner's syndrome lRef: O.P. Ghai 9th/e p. 6A5 6 Vh/e p. 584; Harrson 16h/e p. 6j1l

Etiology of AML
1. Hereditary -
Down syndrome, Klinefelter syndrome, Patau syndrome, Fanconi anemia, Bloom syndrome, Ataxia telangiectasia,
Kotsman syndrome.
2. Radiation
3. Chemical and other exposure
Benzene, smoking, ethylene oxide, paint, herbicide and pesticides.
4. Drugs
Alkalating agents, Topoisomerase II inhibitors, Chlorarnphenicol, phenylbutazone and less commonly chloroquine,
methoxlpsoralen can result in bone marrow failure that may result in AML.
Note: Turner syndrome also causes leukemia - Nelson 18th/e p. 2117 table 495.1
But I am not sure, whether it causes AML or not.

27. Ans. is'None' fRef: O.P Ghai 8n/e p. 605 6 7h/e p. 584; Nelson 18th/e p. 21171

Genetic conditions predisposing to leukemia


r Down syndrome r Diamond-Blackfan anemia r Turner syndrome
r Fanconi syndrome r Klinefelter syndrome r Neurofibromatosis tlpe 1
r Bloom syndrome r Schwachman syndrome r Paroxysmal nocturnal hernoglobinuria
r Li-Fraumani syndrome r Ataxia - telangiectasia r Severe comhined irnmune deficiency

28. Ans. is'b' i.e., Down's Syndrome lRef: Nelson 18'h/e p. 2122)
'Neonqtes and infants with Down's Syndrome may have a condition known as 'transient myelopraliferative
syndrome' which mimics congenital leukemia'

25" Ans" is'a' i.e", Down $yndrome [Ref Nelson 18't'/e p. 21221

JUVENILE CML

3S. Ans. is'c'i"e.,Fhiladetphiactlroffiosomei$positive[RdA.F"GhaiSth/ep.6A8eVhkp.586;Nclsonl$thlep.2l22l


iuventle tMI" is mostly seen in ckildren helow 2 years af age. Philadelphia chramasoane is rcegwtive awl
iewleocyte nunt is less than 100'A00/mne3 -Glni
. Age :'. l0-l2years ? Y€rr{.::-r:

r Bleedingmanifestation Absent Frequent


-i":'$tii6iii.r4,"ia -,',,,
Uncon?hin Fig{11en7:::":'
r Rash Absent Frequent
t Lymphadenopathy ::',:::: r:Qafe .,:. ,freqrqrf,,1,,
o Splenomegaly Marked Variable
e WBC count at diagnosii ,:.r,'>.'100r00O < 1001000,'.,:
. WBC type Granulocyte Monocyte
r.: Normeblastif -:,- ::.:.. Ufigstldl eommiSn .,

HbF Normal lncreased


a lmmunoglobulins r',.. Noriral ..,., 'lricreased'":'
a Muramidase Ievels Normal lncreased
i tlfieiponle.to 6us$l6fiq6r',: :' :'' :.::1:i,:,Geiod::...
P.oor ., ',, ',

Srlrrrival 2.5-3 years 9 months

31. Ans. is 'd i.e", |unenile Ce{L tftef A.p. Gbai S,hle p. 6AS & f,le p. 5S6l

NEUROBLASTOMA

32, Ans. is'b'i.e.,NeuroblastomafRef.Nelsonl?th/ep.2137;O,hGhaiBtulep.616&plep.S9A|


'Ileurablastoma is the mastfrequently diagnosed neoplaw in infants accaunfing;.fix"2y-3go/o aJ"rueanatalmaligtancies'
'lielsrsn'
o It has l.oliolving pecularities : -
s Mcst comrncn abdominal cancer cf chilclhood.
t Most common cancer of idfancy
t Most carnmort extracranial solid tumor of childhood (most common solid- trrrnor of childhoo,l is trrain t*mor).
33. Ans. is'ii.e.,Niuroblastoma[Ref O.P.GhaiSk/ep"616,617&P/ep.590;NelsonI*/ep.2137,213g]
r Sr'velling in flafik, episodic diarrhea, flushing, slveating ancl bone pain suggest the diagnosis of neuroblastoma"

34. Ans. is 'i i.e., Lytic lesions in skull, with sutural diastasis lRef O,p. Gh*i */e p. 616 & Vh/e p, s90; Nelson tffh/e
p. 2137, 21j81
e Most frequent site of metastasis from nueroblastoma is bone : where metastasis occurs very early.
o Lytic lesions in skull and suturai diastasis are manifestations of bony metastasis to skull, and hence the
earliest feature,
amongst the options in the questions.

Ans. is'a'i"e., Neur*blastnmalR.ef: A"p Ghsi gtt,/e p. 6]6 6 Vt'/e p" S9A]I
e Atrdominal mass and hypertension can occur both in neuroblastoma and wilm's tumor.
r But, sweating and diarrhea favour diagnosis of neuroblastoma.

36. Ans. is 'b' i.e., NeuroblastomalRel O.p. Ghai I'h/e p. 616 d, Vh/e p. 59a|
'Metastasis present in 60-700/o of children at time of diagnosis
are usually ta the skeleto*, characteristicaLly to the
fctcial bones and the skull and to the bane marrow and nodes'

37. Ans. is 'c' i,e,, Lung metastasis are comtnon [Ref Nelson lgele p. 21jg]
o Neuroblastoma is most common extracranial solid tumor of children.
o Lung metastasis is rare in neuroblasioma.
.liiiir 11: ,..r,... :irrl.. ;ni..:::iui
"

crrip irn r :o
"c*l*iiiild
metaslasis is skeletal
r Metastasis present in 60-700/o of chilclren at the time of diagnosis, and the most frequerut site af
system. Other common sites ate liver, lymph nodes and skin'
r Encasement of abdominal aorta and inferior venacava by turnor is common.
ntl' p' 2137, 21381
.&ffs. is i.e",Nemroblast*tt;:'al%ef Nelsaw 18th/e

o Neuroblastoma can present as paraneoplastic synrlrenne of autoimune origin mani&sling as ataxia or opr-rsti:voclr"'Lrs
(Dancing eyes and dancingfeat).
o In such cases primary tumor is in the chest or abdomen and the brain is negative for tum<lr'
r Some tumor produce catecholamines that can cause sweating and H'IN a'rd sotne relrase vasoactive
peptide (VIP) causing secretary diarrhea'

i9. Ans. is 'd' i.e., Trk A expression lRef: Robbin's Vh/e p' 7631

Prognostic factor for Neuroblastoma :

r Good Prognostic factor r Worse Prognostic factor


rAge<1year r Near diploid tumour
r StageI&II r Deletion of chromosome 1 and 14
r HlperdiploidY or TriPloidY r Amplification n-myc oncogene
r Expression ofTrk A gene

40. Ans. ist' i.e., Hyperdiploidy tnef Robbin's Vh/e p' 763)
{1. Ans. is t' i.e., Neuroblastomit[Ref: o.P. Ghai 8th/e p' 616 dt vh/e p' 590]
infitst two years of life l{euroblastoma
e The commonest intra-abdominal tumor
t l'fe Wilru's tumor
The commonest intra-abdominal tumor between 2d to 5'h yeat af
-

o The commonest intra-abdominal tumor in childten (no specification) age Neuroblastoma


in
(wilm's tumor is the second most common abdominal tutnor children)
Remember:
The cammonest cause of abdominal mass in Newborn is : Illultiple dysplastic
kidneys
(if *eoplasm or turnor has not been mentioned when asking for the commonest intra'-abdominal r11ass' the answer

will be multiple dyspiastic kidneys)

.a,
i.e., Neuroblastorna [Ref Nelsor 18th/e p. 21i7, 21i8; o.P Ghai 8th/e p. 616 dv 7h/e p. 590]
12. Ans. is
of neuroblastoma'
o proptosia, bony metastasis and sutural separation (due to skuil rnetastasis) suggest the diagrrosis

Axs. is'a' i"c,, lieurohlastslmea lfief {ik*i 7t'/e p 59{}}

WILM's TUMOR .

6 p'
Ans. is t' i.el, Polycystic kidney p. 591; Nelson 18th/e 214a1
14. lRef: o.P. Ghai }th/e p. 617 Vh/e

Wilmttumor
e Wilm,s tumor, also known as nephroblastoma, is the most comlnon malignant tumcr of kielney in cirildren'
Etiology
o Most of the tumor ate sPoradic.
o I - 2 Vo rnaY have familY historY.
o r-amilial predisposition to wilm's tumor is inherited in at autosomal dominant lnanner'
o Mutation of the foilowing genes are seen in wilms tumor'
r) Wiims tumor gene (WT 1) on chromosome 1l
ii) CTNNB.I gene encoding the protooncogene beta-catenin'
iii) ps3.
Witm's tumor is associated with three distinct syndromes :
45. Ans. is 'c' i,e., clear cell sarcoma fRef: a"p. Ghai B'h/e p. 617 6 vh/e p. 592; Nelson rgthle p. 214a, 2141)
o Clear cell carcinoma is a histological subtype of Wilm's tumour which frequently metastasize to bone (In
conventionai wilms's tumour bone metastasis is rare).
46. Ans" is'a'i.e., As asymptornatic abdominal mass lRef: O"P, Ghai B,t,/e p. 61g & Vhle p. 592; Nelson rgth/e p. 214tr1
Presentation of Wilm's tumor:
e Asymptomatic abdominal mass (most common) r Heamaturi a (10 -25o/o)
o Abdominal pain (30%) t Fever (20o/o)
c Hypertension (25o/o) e Anorexia and vomiting
47 . Ans. is 'd i.e", Stage of dis ease lRef: Rudolph 29th/e p. 164l
r Tumor staging at the time of diagnosis is the most important prognostic factor.

48. Ans. is 'b' i.e., Open Nephroureterectomy fRef. Sabiston |Bth/e p.20L2l
o The treatment of choice for Stage I Wilm's tumor is Transperitoneal Radical Nephrectomy (Radical
Nephroureterectomy) followed by chemotherapy with or without radiotherapy depending on tumor histology,

LANGERHANS CELL HISTIOCYTOSIS

49. Ans. is 'd'i.e., Torres syndrome fRef: O.P. Ghai yth/e p. 620 & 7h/e p. 595; Nelson lsthle p. 2159, 2$a; Robbin\ Tt,le p.
7o1l
o Histocltosis X is characterized by proliferation of Langerhans histiocytes (tissue macrophages).
o It is subdivided into three categories : (i) Letterer - Siwe syndrome, (ii) Hand -Schuller-Ciristian
disease,and (iii)
Eosinophilic granul oma.
50. Ans. is'c'i.e., Lagherhan's cell histnqtosis [Rel A.P. Ghai 9th/e p. 620 6 Vh/e p. s9S; I{elson tyth/e p. 21s9, 2rca}
r Seborrhic dermatitis, polyuria, hepatosplenomegaly and discharge suggests the diagnosis of Langerhans cell his-
tiocytosis.

51. Ans. is 'b' i.e., Langerhans cell histiocytosis [RS o.p. Ghai Btble p. 62a 6 Thle p. s9s]
52. Ans. is'c' i.e., Histiocytosis X [Rel a.p Ghai yth/e p. 62a d, vh/e p. 595; Robbins vh/e p. 7a1, 7021
o Osteollfrc lesions in a child involving all the bones of the body points towards the diagnosis of Histiocl,tosis X.
o The bony lesions in Histiocytosis x can present as a single lesion or as a wide spread iir.ur. involving virtually
any
bone ofthe body.
o The iesions are osteolytic and the common sites of involvement are -+ Skull, Vertebrae, Ribs 6 Clavicle.
About other choices
o Metastasis from Wilm's tumour It can be easily be excluded as the metastatic from Wilm\ tumour
--) does not involye bones.
e Neuroblastoma Though it can inyolye any bone of the bocly, but simultaneously
--) inyolvement of all the bones of the body without any common
manifestation is very unlikely to be neuroblastoma.
o Metastasis from Osteosarcorna Metastasis from osteosarcoma commonly goes to lung not bones.
--)
53. Ans. is 'a' i.e., Birbeck granules in the cytoplas m l&ef: a.P. Ghai &thle p. 62a-621 dr Vh/e p. 595-595; Nelsorc try*/e p
2159, 2160; Robbin's 7h/e p. 7011
r Hepatosplenomegaly, presence of histiocytes with PAS postive suggest the diagnosis of langerhans cell histiocltosis.
o Diagnosis of LCH:
Cn,r,prra

::.r:,grii:F
t The hallmark of LCH is clonal proliferation of cells of monocyte lineage.
r The presen ce of Birbeck granules in the c1'toplasm of tumor cells is characteristic of LCH.
dite) r Birbeck granules are tennis racket shaped bilameliar granules.
r As Birbeck granules are not seen in all tumor cells by election microscopy, the detection of S-100 and CDla
expression by immunohistochemical techniques makes the definitive diagnosis.

Ans. is'a' i.e., CDla fRef: Nekon 18th/e p. 2159, 2160; Robbin's Vh/e p. 701)
o The presence of Birtreck granules in the cltoplasm of tumor cells is characteristic of iangerhans cell histiocltosis.
r As Birbeck granules are not seen in all tumor cells by electron microscopy, the detection of 5-100 and CDla
expression by imrnunohistochemical technique aids in the diangosis.

re (In Ans. is 'a'i.e., Can be associated with diabetes insipidus; t' i.e.,Birbeck's granules in langhan's cell; 'd' i.e.,
Proliferation of antigen presenting cells; 'e' i.e., Associated with specific HLA DR fRef: Privious explanationsf
111 o Hand-Schuller-Christion disease (a tlpe of histiocltosis) is characterizedby triad of : -
i) Claviral bone defects n) Diabetes insipidus iii) Exophthalmos
r There is proliferation oflangerhans histiocltes (antigen presenting cells)
r LCH is associated with HLA-DR, S-100, CD1a.
o Birbeck granules are present in the cl,toplasm of tumor ceiis.

:6. Ans. is t' i.e., Cladaribine fRef: Nelson 18'h/e p. 21611


t For unresponsive disease following therapies are available :

i) Cyclosp;l iiie/;rriiitirpnr:ci,'te globulin ii) iiiratinib


lri) 2-chloro deoxyad.enosine (Cladribine) iv) Stem ceil transplantation.

lical
MISCELLANEOUS

Ans. is'-n* i.e-, Lpnphorma llltJ: Nelsorz n9't'/e C:. 491,,192, 3421

t Lymphoma is the most common malignancy of the gastrointestinal tract in children. About 30% of children with non-
ep. Ilodgkin lymphoma present with abdominal tumors.

58. Ans. is t'i.e., ALL in children


iii) c The surviyal rate has improued from zero,four decades ago to 20-75 percent currently, largely due to clinical
trials on new chemotherapeutic qgents and improvements in stem cell transplantation (SCT) technology.
o Other [hree tumors have poor prognosis.

lls- 59. Ans. is 'b' i.e., Leydig cell tumor [Ref: CPDT l*'h/e p. 965, 966, Nelson 18th/e p, 2153, 2154)
o Germ celi tumor are : -
t Seminoma/dysgerminoma t Embrybnal carcinomat Yolk/sac (endodermal sinus) tumor
s Choriocarcinoma t Teratoia
60. Ans. is'a' i.e., Rhabdomyosarcoma lRef: O.P. Ghai \th/e p. 618-619 & 7h/e p. 593; Nelsan 18th/e p. 21441
r The most cofirmon soft tissue sarcoma in children is rhabdomyosarcoma.
n), r Head & Neck (including face) are involved most commonly.

61. Ans. is t' i.e., Pleomorphic rhabdomyosarcoma fRef Nelson 18th/e p. 2144)
o Pleomorphic type (adult form) is rare in childhood and accounts for only 7o/o of cases.
62. Ans. is'b' i.e., Rhabdomyosarcoma fRef: O.P. Ghai 6th/e p. 575; Nelson 18th/e p. 2144)

63. Ans"is'a'i.e",kha'bdcxnyc$are{}maLRel:{}.P.Ghai8't'let}.6rs&Vh/ep.592;Nels{ry, lStt'/ep.21447


64. Ans. is'b'i.e., Retinoblastoma [Rel Robbin's 7/e p. 284-2851
: "R.etinoblastoma is the most strking example of inhirited cancer syndrome. Approximately 40o/o of retinoblastomas
arefamilial. Carrier of this gene have a 10000 fold increased risk of deueloping retinoblastoma usualy bilateral".
r Each child of parent with familial bilaterai retinoblastoam has a 50% risk of inheriting the retinoblastoma gene, of
these 90% will develop retinoblastoma.

65. Ans. is 'a' i.e., Plasmocytoma[Ref: Hsrrisan 17h/e p. 701]


o Plasmocytoma can be easily ruied out.
. It is a localized form of multiple myeloma which usually occurs in 5th to 7th decade.
o Metastasis, histiocltosis and brown tumor can cause l1tic bone lesions in childhood.

66. Ans. is'b'i.c., Retinoblastoma [Rd Nelson IVhle p.a9fl

Retinoblastoma 97o/o

ffi!,6s"
Rhabdomyosarcoma

ANSWERS OF VARIOUS OTHER EXAMTNATIONS

67. Ans. is'b' i.e., Leukaemia lRef: O.P. Ghai Bth/e p. 599 & Vh/e p. 550)
68. Ans. is'b'i.e., Brain tumour lRef: O.P. Ghai 8th/e p. 5gg d, Vh/e p. 545, Sg0)
69. Ans. is'a'i.e., ALL[Ref : O.P. Ghai 8,h/e p. 599 d2 7h/e p. 5811
70. Ans. is'a'i.e., Presence of mediastinal mass lRef : o.p. Ghai gth/e p. 6a0-6u & Thle p. 5gll
71. Ans. is'a'i.e., Down syndrome lRef O.P. Ghai Bth/e p. 638 6 Vh/e p. 5g6l
o Patients with Down syndrome are more prone to develop juvenile type of cML.

72. Ans. is 'b" i.e., NeuroblastomafRef: Has been explainedl


o Both neuroblastoma and wilms tumor can present with abdominal mass and calcification.
o But, at I year, it is most likely neuroblastoma.
r The mean age of presentation of wilms tumor is 3 years.
73. Ans. is'H i.e., Beckwith-Wiedman syndrome lRef Hasbeen explainedl
r Beckwith-Wiedman slmdrome has been explained -+ See wilms tumor.

74. Ans. is t'i.e., Opsoclonus lRef: O,P Ghai 8n/e p. 617 & fhrc p. iOZl
75. Ans. is'H i.e., HemangiomtfRef: Nelson l9th/e p. 21451
o Hemangiomas, the most common benign tumors of infancy, occur in 10% of term infants.
El,r.r .,...iiP

HEE{AT*I#GY
r Erlthropoiesis is the production of red blood cells (er1'throcl'tes). Er1'thropoiesis begins between 10-14 days of
gestation. The predominant site of er1'thropoiesis varies with age.

* 2nd-101h week of gestation (mosoblastic stage) a Yol.k sac


.' ::
:3,:. I Oh week24weekof gestation64"s' (hepatic stage) ! Lindr(rslrM-s.!!1..1'r . .

* 24 weeks - till birth (myeloid stage) e Bone marrow throughout the skeleton

:,1*.:., Birth - upto puberty * Bone marrow throughout the skeleton

* After puberty (after 1 6-1 8 years) Q Bone marrow of vertebrae, ribs, sternum, skull, pelvis,
and proximol ends of humerus & femur

Fetal RBCs in comparison to adult RBCs :

r) Contain less 2, 3 DPG(AIIM9es'e7)


ii) Contain less iron@IlMses'e7)
lii) Shart liYedallMs e7)
iv) Larger in size (volume)attMses'e7)
iv) Resistant to alkali denaturationqilMs e8' e7)

ANEMIA

r Anemia is defined as a decrease in the amount of red blood cells or hemoglobin concentration in the blood.

DISORDERS OF HEMOGLOBIN

r Hemoglobin molecule is made up of four subunits, each containing a protein part (globin) and a non-protein part
(heme).
o The heme part is same in ali qpes of hemoglobins.
r The protein part (globin) is different in different hemoglobin, which is made up of two pairs of poilpeptide chains.
Different pollpeptide chains are o, P, y, 6, e, (. Different hemoglobins (depending upon there pollpeptide chains)
are HbA or adult hemoglobin (c"r} r), Hbf or fetal hemoglobin (a;y ,), HbA, or minor adult hemoglobin (o-16 ), Gower-l
(e y
), Gower-2 (are) and Portland ((7 ).
ls' AIrMS 83) upto 8th week; these
o In early fetal lifu three hemoglobin Gower- 7, Gower-2 and portland are synthesized@I
are called embryonichemoglobins. After that predominant hemoglobin is fetal hemoglobin (HbF) upto 36 weeks@rlMs
0s'04)
after which synthesis of hemoglobin switches from HbF to HbA (adult hemoglobin)Gnusos'oa). After birth, adult
values of HbA are achieved by 6 months to 1 years.

o Disorders of hemoglobin may be of two types -


1) Qualitative defect of hemoglobin (hemoglobinopathy)
; Hemoglobinopathies are disorders of hemoglobin, characterized by a defect that results in abnormal structure
' of one of the globin chain of hemoglobin.
r Examples are - Sickle cell anemia, HbC, HbE, HbD-Punjab, Hb O-Arab, Hb Lepore.
2) Quantitative defect of hemoglobin
r These disorders are characterizedby a defect that results in decreased rate of synthesis of one of the globin
chain of hemoglobin.
I Example is Thalassern:a(PGi et;).

c Most common type of disorder sf hemoglobin is thalassemia in the world.

THALASSEMIA SYNDROME
o Adult hemoglobin is a tetramer, composed of two cr globin chains and two p globin chains. Thalassemia is an inherited
autosomal recessive disorder. In Thalassemia, the gentic defect results in reduced synthesis of one of the globin chains
of hemoglobin.
e p-Thalassemiaiscausedbydeficientsynthesisofp-chainwithnormala-chainsynthesis@IlMsol), cr-thalassemiais
caused by deficient synthesis of ct-chain with normal B-chain synthesis.
e Molecular defect in pathogenesis are :-

A) B-thalassemia
r It is mostly due to point mutation which changes an amino acid codon to a stop codon (non sense mutation).
Rarelyit may also occur due to deletion orinsertionwhich results iny'ame-shift- mutation(Pcrql).These mutation
0r)
can cause defect at different steps of p-chain synthesis : (i) Aberrent splicing (catled splicing mutation(Pcr :

most common); (ii) Premature termination of translation (chain terminator mutation); or (iii) transcription
defect Qtromotor region mutation).

B) o- thalassemia
r The mostcommon cause of reduced o-chain synthesis is deletidn of cr-globin genes. Rarelynonsense mutation
may also cause o-thalassemia.

p- thalassemia
o The abnormality common to all B-thalassemias is diminished synthesis of structurally normal p-globin chains,
coupled with unimpaired synthesis of cr,-chain4llus01). As B-chain is absent, there is compensatory increase in
production of o, y, and 6 chains, resultinginincreased amount of HbA2 (o-16r) anilHbF (o-ryr)(Nnnrmut06'PGt04).
r There are three forms of B-thalassemia :-
1) Thalassemia major (Cooley's Anemia)
r It is in individulas homozygous for p-thalassemia gene. It is severe transfusion dependent anemia.
seen
r Severe Anemia manifests at 6-9 months after birth. There are prominent frontal and cheek bones, jaundice,
hepatosplenomegaly(a['uso6),increased risk of pigment stone due to extravascular hemolysis, iron overload and
secondary hemochromatosis.
CHeptsx
r Perpheral blood smear shows anisocl,tosis, poikilocltosis, microcytic hypochromic anemiaqllMsa6), target
cellsqllMs 06) , bosophilling stippiing and fragmented RBC'
r HbF is markedly raised (60-90o/o)@Gt04). There is JUCV JtrriCU, JITACUC, t.errrm iron, tserum ferritin,
tpercentage saturation , and Jtotal iron binding capacity.
r Bone marrow examination shows marked erlthroid hlperplasia, and reversed myeloid to erlthroid ratio (normal
is 3:1).
r fragility of RBCGIIMS06).
There is deueased osmotic

2) Thalassemia minor (thalassemia trait)


r It is seen in individuals heterozygous with only one B-thalassemia gene. It is the mildest form with mild or no
anemia.
r Patients are asymptomatic and are identified during laboratory findings.
s Thalassernia trait Ttrovides ptotection against falciparum malaria$Itusee).
t HbAris characteristically elevatedqEEr) (> 3"so/o(PGIee)). HbF is mildly raised (5%).
r NASTROF test is used for screening of thalasemia tr@it 6rrMS 11)
. It not a diagnostic test(At as)
"

r Laboratory findings may be similar to thalassemia rnajor.


3) Thalassemiaintermedia
r The features are between thalassemia major and minor.
0s' 01' AilMs
o Diagnosis of thalassemis is established by Hb electrophoresisql
01)
.

r X-ray skull shows crew cut appearnaceGrs:), and hair on enil appearance(Pcle7'AIIMSe') (Hair end appearance is also
seen in sickle cell anemia(Pcls7'Air'trsrz), hereditary spherocl'tosis and G6PD deficiency).

(such
Blood picture of beta thalassemia includes microcytic hypochromic RBCs; anisocltosis and poikilocytosis
as ovaiocytes, schistocyte); talget celIs, basophilic stippling and nucleated RBCs (NRC).

lhelagsemic lacies {maxilla


hyperylasia, liat rasal
brldge, frontal bcsingi

Alpha - thalassemia
o people who do not produce enough alpha globin chainhave alpha - thalassemia@EED. Alpha globin chain is made by
four genes, each gene contributes to 25o/o of the cr-globin chains'
r The severity of cr,-thalassemia varies greatly depending on the number of cr-globin genes affected
-

i) Silent carrier state


r Single o-globin gene is deleted. These individuals are completely asyrnptomatic.
ii) cr-thalassemiatrait
r Two o-globin genes are deleted. These individuals are asymptomatic with some red cell abnormalities like
B-thalassemia minor.
iii) HbH disease
t Threegenesofa-globinchainaredeletedl(Ar11).Withonlyonegene,theslmthesisofcr-chainismarkedlyreduced
and tetramers of excess p-globin, called HbH, form.
r HbH has extremely high affinity for oxygen and therefore is not useful for oxygen exchange, leading to tissue
hlpoxia disproportionate to the level of hemoglobin.
r Patients have moderate to severe anemia that may require occasional blood transfusion.
iv) Hydrops fetalis
r There is deletion of all four cr-globin genes. In the fetus, excess y-globin chains forrn tetramers, known as
hemoglobin barts.
t Hemolgobin bart has such a high afinity for oxygen that it delivers almost no oxygen to tissuesal0s) .
r Most individuals die before or shortly after birth. In utero blood transfusion have allowed the birth of children
with hydrops fetalis who then require life long blood transfusions.

SICKLE CELL ANEMIA ANDTRAIT

r Sickle cell anemia results from mutation in B-globin gene.


e It is caused by a point mutationfirnr) at the sixth position of the p-globin chain leading to substution of a valine
residue for a glutamic acid residue(NEEr'Ar03'ArrMS 0') resulting in sickle hemoglobin (Hbs).
r Sickle ceIl anemia is an autosomal recessive disorder. If an individual is homozygous for the sickle cell mutation,
almost all the hemoglobin in the red ceII is HbS. In heterozyogotes, only about 40% of the hemoglobin is HbS, the
remainder being normal hemoglobins.
o Heterozygous state provides protection againstfalciparum malaria(Al13,11,AIIMS\3) (Thalassemia trait, G6pD deficiency,
and pyruate kinase deficiency also protect against falciparum malaria).
o Hbs molecule undergoes pollmerization and aggregation in deorygenated state. This produces distorted sickle or
hollyJeaf shaped RBCs. Inititially sickling is reversible, but with repeated oxygenation and deoxygenation RBCs
become permanently sickle shaped.
r Factors affecting sickling are :-
l) HbS concentration
r It is the most important factor. HbS concentration > 50o/o precipitates sicklingletus oat .
2) HbA and HbF concentration r
: HbA and HbF have inhibitory effect on polymerization of HbSUIIMS 04, PGI 13,05). Thus in heterzygous state (sickle
cell trait) when HbS is only 40%, remaining 60% HbA inhibit polymerization and patient remains asyrnptomatic
similarly, since HbF concentration is high till 6 months of life, babies are asymptomatic during this period.
3) Deoxygenation of HbS
r Polymerization occurs oz ly in deoxygenated state.A reduced pH(PGrI3, 08)
causes sickling by decreasing tendency
of binding of Hb to oxygen.
4) Duration of time RBCs exposed to deoxygenation
r Increase time of presence in microcirculation (capillaries) cause deoxygenation and sickling. Hence sickling
is confined to microvascular beds where blood flow is sluggish, e.g. spleen and bone marrow.

Clinical features
r The clinical manifestations of sickle cell anemia are -

1. Chronic hemolysis
t Ineversible sickle cells have rigid, nondeformable cell membranes that lead to dificultyin negotiating the splenic
sinusoids, sequestration, and rapid phagocytosis. This results in extra - vascular hemolysis. Some intravascular
hemolysis may also occur because of increased mechanical fragility of severely damaged ceils.
Cuepren tr HematologY

Vasoocclusive sYmPtorns
t molecules and are abnormally sticky that
Reversiblesickle cells express higher tiran normal levels of adhesion
lungs, spleen and penis'
is responsible for occlusion of microcirculation especially of bones,
qre fhe most common manifestations of sickle cell anemia. They commonly manifest as -
I
r) Pairuful bone crisis?
ii)Hand-footsynd'rome@Ni15)+Dactylitisofbonesofhands/feet.
III) Autosplenectomy@Gra0)-+predisposestoinfectionbyencapsulatedbacteriae'g.H.influenza&pneumococcus'
iv) Acute chest syrrdrome
This is the most dangerous crisis'
v) Sequestration crisis -+ ,\cute painfui enlargement of spleen'
vi) Aplastic crisis -+ due infection by parvovirus - B19
vii) CNS symptoms -> Seizure, stroke
8)
viil) P r i ap i sm6tt
MS e
and infarction of p enis

ix) Leg ulcers


x) Most comrnon cause of osteomyelitis is salmonellaNEEr)'
(Str. pneumoniae, H' influenzae)(Alrs) because of
xi) There may be infections due to capsulated organisms
autosPlenectomY.

and the only signifi-


t sickle cell troit {heterozygous state) is olmost always osymptomotic
defect presenting with isothenuria'
cont manifestation is the renal concentroting

line

oil. lnvestigation
I3r 1s)'
o peripheral smear shows sickle cellshl
13)
, target cells('{r and Howell - lolly bodiesql
:ie
in most of other anemias ESR is increased)'
e ESR is ilecreased(only important anemia in which ESR is decreased,
metabisulfite'
::.\'. o There is positive sickling test with a reducing substance like sodium
trait?Gl s8) (40% HbS & 50% HbA) and one
o Hb electrophoresis shows firyo bands in heterozygous state/sickle.cell
eor bands in homozygous state (only HbS). Hbs is slower
moving than ItrbA towards pasitive electtodeqttMson) '
lCs r Bone changes on X-raY show :-
1) Fish mouth vertebtae@Glnt)
sst
ii) Crew hair cut (hait on end) appearance of skull{eu '

,Hair on end appearance' is also seen in thalassemia, hereditary spherocytosis and G6PD deficiency'

;i.1e r There may be catdiomegaly(Pcres) and leukocytosis@Gle8) '


and calcium deposition in spleen) are found' Gamma Gandy
.tic r Gamma Gandybodiesa,Msr:) (nodules of hemosiderine
od. bodies are seen in all causes of congestive splenomegaly
: scA, cML and liver cirrhosis'

]a\'

:lltre peripheralblood smear shou'ing sickle cells, target cells, and Howelljallyo'odies'
I
: illC

-iar
Besides hemoglobin S, there are other hemoglobinopathies due to point
mutation in beta globin chain of hemoglobin.
There are HbC (in ltalians and Black); HbE (prevalent in BengalrPctro),
tusam & East lndia); and HbD {in Funjabisrdrs, & Gujratis).

GLUCOSE 6 PHOSPHATE DEHYDROGENASE (G6PD) DEFICIENCY

r G6PD deficiency is an X-linkeil recessive disorder@Gl 06)


, characterized by hemoll,tic anemia on exposure to oxidatiye
sfress.
o Normally RBCs are protected from oxidant injury by reduced glutathione. Reduced glutathione is generated from
oxidized glutathione and the reducing equivalent for this reaction is provided by NADPH. This NADPH is generated
in HMP shunt by enzyme Glucose-6-phosphate dehydrogenase(Pcre8), while oxidizing glucose-6-phosphate.
r If G6PD is deficient, NADPH production will be reduced that results in increased susceptibility of RBC to oxidative
damage because of unavailability of reduced glutathione. So, whenever there is oxidant stress, e.g. by drugs, infection
or foods, hemolysis occurs. G6PD deficiency causes episodic intravascular and extravascular hemolysis.
o There are three tlpes of G6PD deficiency:
i) Tlpe-1 (mildest form)
ii) Trlpe-2 (moderately sever)
111) Type-3 (most severe formal 13)
).
r Important peripheral smear findings -+ Heinz bodies, Bite cellsoEEr), Spherocyles@Gree).

s Enzyme deficiency causing hemolytic anemia: G6PQ pyruvate kinase, aldolase and hexokinase.

OTHERTYPES OF ANEMIA

lron deficiency anemia


o Newborn has iron store sufficient for erythropoiesis for 9 months after birth, thus iron deficiency anemia occurs after
this age.
c lron is absorbed from proximal small intestine mainly from duodenum@Gt0e).
o Cow milk contains more iron than breast milk(Pcr 0e), however bioavailability of iron of breast milk is much greater
than cow milk.

Clinical features
o Pallor(Aro') is the most important sign. Other features are fatigue, pagophagia (eating of inedible substances like clay),
irritability(Ar0'),anorexia, decreased attention span, decreased alertness(Pcr0e), litharryut04), tachycardia, cardiac
dilatation and systolic murmur.

lnvestigations t
o Bloodpictures:Microcytichypochromicanemia@Gt|s),ring(pessary)cellstueat,poikilocltosis,Targetcells(unusual).
o Bone marrow : Hypercellular marrow, erythroid hyperplasiaAr 0a), depleted bone marrow iron (bone marrow iron
decreases earlier than serum iron@Glos)).
o Parameters:Decreasedserumferritin(eailiestchange)@Gloe'Als6'ss),reducedserum iron,increasedtotalironbinding
capacity (TIBCyoe'ost, reduced percentage saturation, increased transferrin receptors, increased red cell porphyrin
and increased hepacidin level.
o Red-cell distribution (RDW) width is increased@Gton).
o There is decreased MCVt(Ar0a),MCH and MCHC.
,es+prnr. tr Hemat&gy

.t: .'):.'t".4' ... ' ,r,1.: t ,,.tl1.


..): :..;: ,,, 7:..,.,, *i1 1';. rrt q

. H'ypochromicmicrocyticanemiawithwideningofcentralpallor> 1/3oftotatRBcdiameter.
dr$f

-- rr
Treatment
I Oralironsaltsaregiven 6mglkglday.Treatmentisgiven tillSweeksafterachieymentofnormalHbleveltoreplenrs::
iron stores. The dose of iron can be calculated by a formula, i.e. 3x body weight (kg) x Hb deficit gm/dl.
c The earliest response is rapid subjective improvement, e.g. increased appetite, diminution of fatigue,
and. decreased
irritability(NEEr) (within 24 hours). Earliest hematological response is increase in reticulocyte countar 06) (within
i :
days).

Meoaloblastic arernia (macrocytic anemial


o Megaloblastic anemia is caused by deficiency of vitamin B or
,. folic acid.
o Important causes of vitamin B,, deficiency are malabsorption, nutritional deficiency, pernicious
anemia (intr6.-:
factor deficiency), gastrectomy, ileal resection and ilitis, fish tape-worm infestation, bacterial overgrowth syndro::.
(in blind loop syndrome and diverticula of bowel).
e Important causes of folic acid deficiency are dietary deliciency, malabsorption, intestinal diseases,
drugs (phenrro---
valproate, methotrexate etc.), and pregnancy.
o Other causes of megaloblastic anemia are thiamine deficiency{ttruse6), pyridoxin deficiency, hereditary
oroti;
aciduria@ttMs e6),
congenital dyserthropoietic anemia, Di-Gueglielmo syndrome and.hypothyroidism(Aunse6) .

Features of megaloblastic anemia


o Blood pictute Hypersegmented neutropftils (earliest findings), macrocltosis, anisopoikilocl'tosis,
macroovalocries
few tear drop cells, basophilic stippling, Cabott ring, Howell-jolly bodies, pancytopenia (anemia, Ieukopen:..
thrombocytopenia).
o Othen t UCV, t lfCFI, normal MCHC, increased serum LDH.

r Peripheral smear in megaloblastic anemia showing hypersegmented neutrophif w1th nine lobes.

COAGULATION DISORDERS

Tests used to evaluate different aspects of hemostasis are :-


i) Bleeding time
r Prolongation generally indicates the defect in platelet number or
function.
ii) Prothrombin time (PT)
I It fesfs the extrinsic and common coagulation pathways\ttusro) . So, a prolonged pT can results from deficien;
of factor V VII, X, prothrombin or fibrinogen.

II
iii) Partial thromboplastin time (PTT)
I0). prolonged PTT can results from deficiencl
r It tests the intrinsic and common coagulationpathwaysqnms So, a
os)), lX, X, XI, XII, prothrombin or fibrinogen.
of factor V VIII (factor Vlllc, Von willebrand faessvqr

t Thus in common coagulation pathway defect both PT and PTT are elevatedailMslo).

iv) Activated clotting time (clotting time)


r It also tests the intrinsic and common coagulation system. So it is prolonged in deflciency of same factors as

for prolonged PTT.


v) Thrombin time
r It tests the conversion of fibrinogen to fibrin and is elevated in fibrinogen deficiency.

Hemophilia-A
r Hemophilia A is due to deficiency of factor VIIIGGL ' eT)

r Hemophilia A is inherited as an X-linked recessive trait'


r Factor VIII is an intrinsic pathway component required for activation of factor X. Clinical manifestations are due to

defect in coagulation system :-


i) Large post traumatic ecchymoses or hematoma
ii) Prolonged bleeding after a laceration or any form of surgical procedure.
iii) Bleeding into weight bearing joints.
t petechiae are characteristically absent (in contrast to platelet dysfunction where bleeding occur from small
vessels of skin and mucous membrane, e.g. patechiae)'
o Severity of disease depends on the activity of factor VIII :

i) Mild disease : 60%-50% of normal activity


ii) Moderate disease z 2-5o/o of notmal activity
111) Sewre disease : < lo/o activity?Gl,6) . t

r Laboratory findings include :


Al
i) Elevated PTT@G|06'
sT)

At
BT and platelrl ,ounltod
oa'
ii) Normal PT,
0z)
.

r Factor VIII- specific assay is diagnostic'


r Treatment includes recombinantfactor VIII, desmopressin and cryoprecipitate.

A ceild with rccurrent knee $Ifielling haf following picture.


ffiatmaybethecnuqe-

b) ITP

.
:,. dl Glanzrr.rana's thrornb-osthenia

Ans. is h'i.e., Hemopbilia


' ,r Bleeding into weight bearing joints is characteristic of coagulation disorder (like hemophilia). Other three
' optionslre defectln plateletl r,r&ich usually present with pelechiae & ecctrl'rnosis,

Yon-Willebrand disease
r Factor VIII has two comPonents :

\) Factor VIIIc (procoagulant protein) synthesized in liver (main source) and kidney
ii) Von-Willebrand factor (VWF) synthesized in vascular endothelium (main source)Gte8) ar,d megakaryocytes.
r VWF factor helps in stabilization of factor VIII and also facilitates plateiet adhesion. Thus deficiency of VWF results
in:-
1) Clinical features of factor Vlll deficiency
r Bleeding into weight bearing joints; post traumatic ecchymosislhematoma; prolonged bleeding after surgery or
laceration; and prolonged PTT with normal PTaIls).
ii) Clinical features of platelet dysfunctlsn(etnasrzl
r Bleeding from small vessels of skin and mucous membrane (e.g. epistaxis); menorrhagia; GI bleeding; and
prolonged bleeding time(al os).
o Von-Willebrand disease is an autosomal disorder with autosomal dominant inheritance in more than goo/oLrMs aa)
and autosomal recessive inheritance in remaining. It is divided into :-
i) Type- I (most comm on t 70o/o) : Autosomal dominant; there is reduced quantity (partial deficiency) of VWF.
ii) Tlpe-2 (25o/o) : Autosomal dominant, with qualitative defect in VWF.
iii) Type-3 : Autosomal recessive; there is complete deficiency of VWF. It is the most sever form.
o Laboratoryfindings include: -
i) Prolonged PTTo{ros
ii) Normal PT
iii) ProlongedBTailMs e7)

iv) Defective ristocetin induced aggregation (ristocetin agglutination test)Gnus ezt

v) Positive tourniquet test (Hess test).


e Treatment includes cryoprecipitate and desmopressin.

Other important defects of coagulation


o Hemophilia B (Christmas disease) is due to deficiency of factor lyNrer).It is inherited as x-linked recessive trait.
Clinical features and laboratory findings are similar to Hemophili a-.\. Freshfrozen plasmfl ar recombinantJ'actor IX
are used for treatment.
e Hemophilia C is caused by deficiency of factor XI. It is associateh with less sever bleedin gat 04) butmore prolonged
PTT than after hernophilias. Clotting time is also prolongeci.
e Parahemophilia is caused by deficiency of factor V. There is prolongation of clotting time, pTT and pI..
e Factor XIII deficiency presents wlth bleeding from umbilical stumpuilus oz, 06), lltracraniai hemorhage , recwrcent
miscartiageuuus oe),
delayed wound healing, hemarthrosis, hematoma, hematuria and hernatochezia(AllMs 06). There
is normal PTT, PT and thrombin titne(AIIMS 06) . (Jrea clot lysis test is positive{AllMs oro) (disolution of clot in 5M urea).
Mainstay of treatment is FFP and factor XIII concentrate.
e Deficiency with no bleeding : Deficiency of factor XII, kailikerin/prekallikerin and HMW kinogen carse prolanged
PTT, but no bleeding.

Hemorrhaqic disease of newborn


o Normal newborns are moderately deficient invitarnin K. This deficiency increases on 3.d to 5,h day when, vitamin K
level falls further. The serum ievel of vitamin K starts increasing from 13'h to 15'h day of life ancl achieves adult ievei
at 3 months. This is normal, and does not produce any bleeding.
c In hemorrhagic disease of the newborn there is delay in achieving the normal vitamin K level.It is more common
in breast-fed infanss{et oet because breast milk is a poor source of vitamin-K.
o Vitamin K deficiency produces severe bleeding due to deficiency of vitamin K dependent clotting factors (II, VII,
IX, 1;rattsr. Bleeding occurs from umbilical cord stump on 2dl3'd day after birth. Intracranial bleding and large
intramuscular hemorrhages may occur. Increased prothrombin time and increased partial thrombaplastin time is
seen(ArrMSeT),

o The disease can be prevented by parentral vitamin K at birth$to6).

DISORDERS OF PLATELETS

Conqenital platelet dysfUnction


o Platelet adhesion refers to binding of platelets to subendothelial matrix anci requires Ib-IX a glycoprotein of plateiet
membrane. Platelet aggregation refers to adherence of plateiet to one another and requires IIb-IIIa glycos:rc,tr:rn orr
platelet membrane.
r Bleeding into weight bearing joints; post traumatic ecchymosis/hematoma; prolonged bleeding after surgery or
laceration; and prolonged PTT with normal PT1I0s).
ii) Clinical features of platelet dysfunctiolazusrzr
r Bleeding from small vessels of skin and mucous membrane (e.g. epistaxis); menorrhagia; GI bleeding; and
prolonged bleeding timeal os).
e Von-Willebrand disease is an autosomal disorder with autosomal dominant inheritance in more than gAo/oaIIMS 00)
and autosomal recessive inheritance in remaining. It is divided into :-
i) Tpe-l (most common:7oo/o): Autosomal dominant; there is reduced quantity (partial deficiency) of VWF.
ii) Type-2 (25o/o): Autosomal dominant, with qualitative defect in VWF.
iii) Type-3 : Autosomal recessive; there is complete deficiency of VWF. It is the most sever form.
o Laboratory findings include :-
i) Prolonged PTTaros
ii) Normal PT
iii) ProlongedBT1ilMseT)
iv) Defective ristocetin induced aggregation (ristocetin agglutination test)Gurws ez)

v) Positive tourniquet test (Hess test).


o Treatment includes cryoprecipitate afld desmopressin.

Other important defects of coagulation


r Hemophilia B (Christmas disease) is due to deficiency of factor lX@EEr).It is inherited as x-linked recessive trait.
Clinical features and laboratory findings are similar to Hemophiii a- A. Fresh frozen plasffia or recombinant factor IX
are used for treatment.
r Hemophilia C is caused by deficiency of factor XI. It is associated rv.ith less sever bleedin g(AI 04) butmore prolonged
PTT than after hemophilias. Clotting time is also prolonged.
c Parahemophilia is caused by deficiency of factor V. There is prolongation of clotting time, pTT and pI-.
e Factor XIII deficiency presents wrth bleeding from umbilical stumpqtttrs 07, a6), intracraniai hemoruh age, recwrrerct
miscarriagealtMs 06),
delayed wound healing, hemarthrosis, hematoma, hematuria and hematocheziaultMs 06)
. There
is normal PTT, PT and thrombin tirne(AIIMS 06)
. IJrea clot lysis test is positiveattus oo6) (disolution of clot in 5M urea).
Mainstay of treatment is FFP andfactor XIII concentrate.
r Defieiency with no bleeding : Deficiency of factor XII, kallikerin/prekaliikerin and HMW kinogen catse prolonged
PTT, but no bleeding.

Hemorrhagic disease of newborn


o Normal newborns are moderately deficient in vitamin K. This deficiency increases on 3'd to 5,h day when, vitamin K
level falls further" The serum level ofvitamin K starts increasing from l3'h to 15th day oflife and achieves adult level
at 3 months. This is normal, and does not produce any bleeding.
c In hemorrhagic disease of the newborn there is ilelay in achieving the normal vitamin K level. It is more common
in breast-fed infonssLetoat because breast milk is a poor source of vitamin-K.
o Vitamin K deficiency produces severe bleeding due to deficiency of vitamin K dependent clotting factors (II, VII,
IX, X;rerts). Bleeding occurs from umbilical cord stump on 2ndl3td day after birth. lntracranial hleding and large
intramuscular hemorrhages may occur. Increased prothrombin time and increased partial thromboplastin time is
e7).
SeenGIIMS

e The disease can be prevented by parentral vitamin K at bir-thAl06).

DISORDERS OF PLATELETS

Congenital platelet dysfunction


e Platelet adhesion refers to binding of platelets to sutrerrdcthelial matrlx anci requires Ib-IX a giycoproteir of i.i;tei,,,i
membrane. Platelet aggregation refers to adherence of platelet to one another and requires IIb-IIIa gl",rcoDrctr:ie :r
platelet membrane.
r Congenital platelet defect may be of foliowing tlpe :-
1) Bernard-soulier syndrome
r It is an autosomal recessive disorder with defect in platelet adhesion due to deficiency of Ib-IX glycoproteir.r
13' PGr e6)
2) Glanzmann's thrombosthe tria1t
r It is an autosomal recessive disorder with defect in platelet aggregation(Ar '3) due to deficiency of IIb-IIIa
glycoprotein.
3) Wiskot-Aldrich syndrome (WAS)
r It is an X-linked recessive disorder caused by mutation in WAS protein (WASP) gere on chromosome I 1.

r It is characterized by classical triad of thrombocytopeniaNEEr), EcTemaNEEr), ar'd recurrent infections@Eur) .


r TlrereisimmunodeficiencywilhT-celldepletion@ilMse7)andilecreasedlgM\trttssz).IgGandIgA arenarmalqlltts
e7). er)
W is increASedquMs .

r T-cells are progressively depleted with progressive atrophy of T-celi regions in lymphnodes and spleen. B-cells
are notmal,
r Small sized platelets are characteristic(1IMs03).

ldiopathic thrombocytopenia purpura


o Idiopathic thrombocytopenia purpura (ITP) as an autoimmune disorder characterized by platelet, destruction b1-

autoantibodies of class lgcat 13' ArIMs e7)


.

o Destruction of platelets takes place in spleen, stili the spleen size remains normal(AilMS07'a4'ee), a characteristic finding.
r Autoantibodies are of IgG class(ar i3) and are targeted against platelet membrane glycoproteins IIb-IIIa or Ib-IX.
r Due to peripheral platelet destruction there is compensator y increase in megakaryoctyes in bone marrowAlrMs
04' ee)
.

Destruction of platelets in ITP istype lt hypersensitivity reaction.

Clinical and laboratory features


o There is sudden onset of bleeding into skin and mucosal surfoces in the form of petechiae or ecchymosis(AilMs07'04'ee),

pinpoint hemorrhage, melana, hematuria and menorrhagia.


10).
o Subarachnoid hemorrhage and intracerebral hemorrhage are serious complications, but fortunately are rare(Pcr
o Bleeding into joints is unusual(A[Ms ee) (it is more characteristic of coagulation/clotting factors defect).
o Spleen size is normalAilMs07'01'ee).
r There is decrease in blood plateletsaltMs ee)
and peripheral smear shows larger platelets. Bleeding time is ptolonged.
0s)
Specific antiplatelet antibodies can be detected(ocl .

t Bone marrow shows increased megakaryocylrtattMs


0s' ee)
.

Types
o ITP is dMded into acute and chronic forms :-
1) Acute ITP
r It is seen in age group 2-6 years@et
os' AIIMS s4)
with no sex predilections(Pcr 0s).
50% cases are preceded by viral
infection{eer os). It is self limiting(rcr ror with spontaneous remission is seen in 80% of cases(Pct
o<' Ar'IMs ee). 'Ihe

duration of disease is 4-6 weeks.


2) Chronic ITP
t It is more common in
female{Pcl
10, AIIMS 04)
and seen in adulthood, i.e. adult women less than 40 years. There is
no antecedent respiratory infection and spontaneous remission is uncommon. Duration of disease is > 6 months
to I Year(Pcrro)'
c Congenital platelet defect may be of following tlpe :-

1) Bernard-souliersyndrome
r It is an autosomal recessive disorder with defect in platelet adhesion due to deficiency of Ib-IX glycoprotein.
2) Glanzmann's thrombostheniatt t3' pGI e6)

r It is an autosomal recessive disorder with defect in platelet aggregationar I3) due to deficiency of IIb-IiIa
glycoprotein.
3) Wiskot-Aldrich syndrome (WAS)
r It is an X-linked recessive disorder caused by mutation it WAS protein (WASP) gene on chromosome 11.

r It is charact erizedby classical lriad of thrombocytopenia@EEr), EczemadErr), and recurrent infections(Nlgr) .

r There is immunodeficiency with T-cell deptetion(AtlMseT) and decreasedlgMallvlssT).IgG and IgA are narmalG"ns
tr). IgE is increasedGtrMs e7) .

r T-cells are progressively depleted with progressive atrophy of T-cell regions in lymphnodes and spleen. B-cells
afe normal,
r Small sized platelets are characteristic(lrrMs 03).

ldiopathic thrombocytopenia purpura


o Idiopathic thrombocy,topenia purpura (ITP) as an autoimmune disorder characterizeci by platelet, destruction by
atrtoantibodies of class IgG(AI ts' 1Jtns sz)
.

c Destructionofplateletstakesplaceinspleen,stillthespleensizeremainsnormalGlMS0T'04'ee),acharacteristicfinding.
o Autoantibodies are of IgG class(Ar
i3) and are targeted against platelet membrane glycoproteins llb-llltt or Ib-IX.
o Due to peripheral platelet destruction there is compensator y increase in megakaryactyes in bone marrowGttMs
04' ee)
.

Destruction of plotelets in ITP is type lt hypersensitivity reaction.

Clinical and laboratory features


r There is sudden onset of bleeding into skin and mucosal surfaces in the form of petechiae or ecchymosistAllvs
a7' u1' eet
'
pinpoint hemorrhage, melana, hematuria and menorrhagia.
a Subarachnoid hemorrhage and intracerebral hemorrhage are serious complications, but fortunately are rare(oclo).
a Bleeding into joints is unusual(r"'s ee) (it is more characteristlc of coagulation/clotting factors defect).
07' 01' ee).
a Spleen size is normal(AilMs
a There is decrease in blood platelrltLt'tttvs ee)
and peripheral smear shows larger platelets. Bleeding time is prolonged.
0s)
Specific antiplatelet antibodies can be detected@ct .
0s' ee).
a Bone marrow shows increased megakaryocytesatrMs

Types
o ITP is divided into acute and chronic forms :-

1) Acute ITP
r It is seen in age group 2-6 yearc@Gl 0s' AIIMI e4)
with no sex predilections(Pcl 0s).
50% cases are preceded by viral
ArrMsee). Tte
infection@clor). It is self limiting(rcrror with spontaneous re'rflission is seen in 80o/o of cases@ctot'
duration of disease is 4-6 weeks.
2) Chronic ITP
10'AIrMS04) and seen in adulthood, i.e. adult women less than 40 years. There is
a Itis more common inf,emales@Gl
no antecedent respiratory infection and spontaneous remission is uncommon. Duration of disease is > 6 months
ro)'
to 1 Year(Pci
Cn-rpre,n

Treatment
a IV immunaglobulin is the treatment of choiceailM's
er)
for neonatal as well as childhood ITP. Glucocorticoids are often

r:sed in combination. Splenectomy may be helpful as it is the major site of


platelet destruction.

Causes of neonatal thrombocytopenia


o lsoimmunethrombocytopeniotAttMsss)
o Mothers with tTP (Autoimmune thrombocytopenio(Attuseil1'
o Motherswith antiphosphotipid antibody (Slg|anuses1'

a lntruuterine infections(Att[s
82' PGt^il with -> RLlbella, Syphilis, Toxoplasmosis, CMV, HSV
o Neonatal bacterial sePsis

MISCELLANEOUS

Kasabach-Merritt syndrome
* Kaposiform hemangioendothelioma in a neonate can cause thrombocytopenia due to platelet sequestration,
consumptive coagulopathy and microangiopathic hemolltic anemia.
o Heart failure may be caused by AV rnalformations'
radiation'
o The treatment used includes corticosteroids, IFN-cr, vinristine, Iaser photocoagulation, and
e Surgery is not indicated because of risk o;f bleeding'

Neonate polYcYthemia
(PCV) or erythrocyte volttme
o polycythemia is defined as central uenous heruatocrit [also ca)led packed cell volume
fraction (EVF)I level of greatet than 650/o'
trisomy 13, 18
c Important causes of neonatal polycl,themia are twin-twin transfusionGrsr), maternal-fetal transfusion,
h1'poxia, and infant of a diebetic mother"
or 21; adrenogenital syndrome, neonatal Grave's disease, chronic intrauterine
a In asyrnptomatic neonates with pcv > 75o/o anil in symptomatic neonates, treatment
of choice is partia! excharuge

transfusion@PE'r) '

o Asymptomatic neonates with PCV 65-75Vo are treated conservatively with closed monitoring.

€onstitutional PancYtoPenia
major hematopoietic lineages on an
e Constitutional pancltopenia is defined as decreased marrow production of the 3

inherited basis, resultilig in anemia, neutropenia and thrombocltopenia.


r Important causes are : -
r Fanccni Gfiefiriaor* ,n) : Amegakaryocl'tic thrombocl4openia
* Shw achman-Diamond syndrorme(
MH'o) r Down syndrome
e Dyskeratasis congenita@H
10) r Noonan syndrome
is inherited in autosomal recessil'e pattern(Ar
o Fanconi anemia is the most important constitutional "rr"-iu("rrrs), which
is). It is characterized by:-

1) Pancytopenia 2) Hyper pigmentation of tru"nk, neck, and intertriginous area'


3) Growth failure 4) Fanconi facies (small head, small eyes)
5) R.enal abormalitY 6) Praximal RTA (type II RTAST-L tst

* Fetal erythropoiesis starts at:10-14 days of gestation'


* Erythropoiesis itt Yd ttitn€ster oceuis ii ; LiVer, ',.:-"' .

* After birth, praginater stem cell originate from : Bone marrow'


& fetot Hb (llbi) is iomptetely reptaced'by ad*ti flb.bv:6'12'months of age;
* Switchover from HbF to HbA starts: 36 weeks of gestation.
&' ,:6te;$iiiil:i aile1,,.<.loorortotatub. .,,'','..,,:..'. :1il,.:i:,:i,:...,1 ,
.,,:, .:1;.:ti..r;..1t..a:. .;.:j]
* Molecular defect in sickle cell anemia : Replacement of glutamate by valine at position 6 of B-globin.
8, Sickle cell anemia is due to iPoint mutation
,, Sickle cell anemia is not due to: Deletion.
* Concentration of HbS required for sickling:> 5Wo.
, Sickling is affected by:Concentration of HbS (most important), deoxygenation and pH, duration of deoxygenation in microcirculation.
d):

c
q
s On Hb electrophoresis: HbS moves slower than HbA towards positive eleetrode.

!'
t ln sickle cell anemia true is I Autosplenectomy due to thrombosis and infarction.

!,
t X-rayin sicklecell anemia: Crew haircut appearance of skull and fish mouth vertebrae.
a
& Gamma Gandy bodies are seen ln : Sickel cell anemia, CML, Cinhosis.
*l
t Mutation in thalassemia: Mainly point mutation (missense mutation) causing aberrant splicing.
,*,

s ln a-thalassemia chain absent: cr-globin chain.


t:

a HbArconcenlratlliol.in Fjhalaslemio trait: >.3


l%,
!- $bF:ii hish{yeis:ed:in I I thala5sqmia major (tool*'s'gn€tiiidl:.'
o F-thalassemia is diagnosed by: Hb electrophoresis.
*''',^E-5Ifi6Ff lest,E' usAdfor :Screening o-f-p-thalassemia trail' ....r=

*
. Osm.oticfygility inthalassemia : De5rease!.
,*,':,.ltbftd{re-aiefu.,iiue,tsrThr,eealpfiagenes"defeitiirn': r:::r::' :", ,

, Thalassemia trait provides protection against: Falciparum malaria.


..*.: A dead born with hepatosplenomegaly and generalized edema : c'-thalassemia (hydropr Au" to Hb B;t).
& Death in Hb-Bort's is due to :Very high affinity of Hb for O, (cannot release Or).

*:
s Earliest hemoglobin to appeor tGower-1, Gower-2, Portland (embryonic hemoglobins).
t fetaln*trtrxi,,coiiwed,toeduit:tr8€]r: Alkafi.aenaturatioaresirtAnl;ldsi &r5ip,Pg,les.s iron,
q Biiite cells seenin: G-6PD deficiency.
:

t
* Most severe form.of G6PD deficiency:Type-3.,.
.
* .lran istabsorbedfrorn;Duodenurnr,'.""',. :' ",r'.:,,..
:"1

,
& Anemia in iron deficiency: Microcytic hypochronic.
a Earliest response to therapy in iron deficiency Diminution of fatigue anO i11t9t1!!!V a1d
,o|r*!o.., lncre3yrnplil":_,.... ..,.....
s
a Laboratory finding in Von-Willebrands disease: Prolonged aPTT and BT with notr.f pf.

*
o Hemophilia A is due to deficiency of : FactorVlll.
; ehiistnias dlsetiSe {Hemap:6i1;ia ,8} ii due,ta defitiincU'ol': Eactor- lX .. ' ':.. :: :;:.:,
.:.,,11.:,, ,

$ Hemophitia A inheritance :X-linked recessive.


a
a

a
a Bleeding from umbilicol stump in newborn: Factor Xltl deficiency.
'i.:!l:::::ii'.';3:'--'..:,]i]l:....::::::'':...il]t.1,]:]:]j::j]:]]].:1i,l...:]':];::]::'ii].|..'.
a Ng;€l!,$t9!.b{e6$n F$Xll;lFJMwkln6geni-prekaltikiein:.:=jj:;:
a Hemorrhagic disease of new born is dueto:Vitamin K deficiency.
a
a Hemorrhagic disease of newborn can be prevented by: Parenteral vitamin K at birth.
a
a Not seen rn /IP: Splenomegaly.
:

a ,$fJ,d?i$|&.i?:l$r$elrjfe uction,by,autoa*tibodies.
a Class of autoantibody in ITP:lgG.
a erdtti,ifP: i,r'Ndi3eil
a Chranic ITP: More common in females, spontaneous remission is uncommon.
r:::.j#:;,::t:i:,r,j.,:r .a :.:1;r:rittl:t: r' . :
a il?ledfi**Bt,qftha&€J
a Treatment of symptomotic neanatal polycythemra : Partial exchange transfusion.
a
C Blood volume in a preterm neonate ;90-11 0 ml/kg (average 1 00 mllkg).
t t,tt

3 ,t. 'tti.Slpii6€.i:ea.$.nlia,txotdlxiavbora.,:,'l87oto5.:.8%.
t Pediatric potycythemia is caused by: Cerebellar hemangioma.
*::,a:

I Constitutiona! pqncytopenia is seen in: Fanconi's anemia, dyskeratosis congenita, Shwachman diamond syndrome (but not in Dia-
mond-Blackfan syndrome).

c Epsilon aminocaproic acid and tranexamic acid are indicated in hemophilia when there is; Oral bleeding or epistaxis.

e Quinine causes thrombocytopenia: Antibody mediated.

III

d
QUESTIONS

HEMATOPOIESIS 11, Which of the following haemoglobin (I{b)estima-


tion will be diagnostically helpful in a case of beta
1. Erlthropoiesis starts in fetal liver during - thalassemia trait - (NEET Dec.12 Pattern, AIIMS May 06, UP

a) 2-4 weeks (All lndia Decl5 Pattern) 08)


b) 4-6 weeks a) Hb-F
c) 6-8 weeks b) Hbl-C
d) 8-10 weeks c) Hb-A2
2, Which organ is the primary site of hematopoiesis in d) Hb-H
the fetus before midpregnancy - (AttMS May 06) 12. Diagnosis of beta Thalassernia is established by -
a) Bone b) Liver a) NESTROFT Test (CET Nov 14 Pattern)
c) Spleen d) Lung b) Hb Al c estimation
3. The hernoglobin tc appear first in the fetus is - c) Hb electrophoresis
(All India Dec.14 Pattern) d) Target cells in peripheral smear
a) HbA b) Hb A2 13. All of the following are true of p thalassemia major,
c) HbF d) Hb Gowers except - (AllMS May 06)

4. Enrlrryordc hemoglobin is - (All India Dec.13 Pattern) a) Splenomegaly


a) Adult hemoglobin b) Fetal hemoglobin b) Target cells on peripheral smear
c) Gower d) None c) Microcytic hypochromic anemia
d) Increased osmotic fragility
5. When does switchover from fetal to adult hemoglo-
bin synthesis begin - (AllMS Nov 04, Nov A5, May 05) 14" NBSTROFT test is a screeing test for-
a) 14 weeks gestation a) p-thalassemia (All India Dec.14 Pattern,

b) 30 weeks gestation b) Hereditary spherocltosis AITMS Nov 11)


c) 36 weeks gestation c) Autoimmune hemolytic anemia
d) 7-10 days postnatal d) Megaloblasticanemia
6. Feature of fetal RBC in adult - (AIIMS lune 98) 15' Hair on end appearance is seen in X-ray skull in -
a) Alkali denaturation resistant a) Thalassemia (AIIMS Iune 97, PGI lune 97)

b) Small in size b) Hydrocephalus


c) Has nore2,3 DPG level c) Chronic malaria
d) More iron than adult RBC d) Sickle cell anemia
16. [n cr-thalassemia - (NEET Dec.12 Pattern)

ANEMIA a) Excess o-chain


b) No o-chain
7. Macrocytic anaemia in children is produced by all
c) Excess p-chain

except - (AtrMS e6)


d) No p-chain
a) Thiamine deficiency 17. Bart's hydrops fetalis is lethal because - (At os)
b) Orotic aciduria a) Hb Bart's cannot bind oxygen
c) Copper deficiency b) The excess o - globin form insoluble precipitates
d) Hlpothyroidism c) Hb Bart's cannot release oxygen to fetal tissues
d) Microcytic red cells become trapped in the
placental
THALASSEMIA
18. A child died soon after birth. On examination there
E" Defect leading to thalassernia lies in - was hepatosplenomegaly and edema all over body.
a) Haemoglobin (CET lune 14 Pattern)
Most probable diagnosis in - (AIIMS May 02)

b) Osmotic fragility a) B-thalassemia


c) RBC membrane b) o-thalassemia
d) Platelets c) Hereditaryspherocytosis
d) ABO incompatibility/sickle cell anemia
9. In beta thalassemia, there is - (AttMS May 01)
a) Increase in beta chain, decrease in alpha chain 19. 34 week primigravida punjabi khatri comes with
b) Decrease in beta chain, increase in alpha chain history of consanguineous marriage, with history
c) Decrease in beta chain, decrease in alpha chain ofrepeated blood transfusion to her sitrling since 8
d) Increase in beta chain, increase in alpha chain rnonths of age. The first diagnostic test is -
a) HPLC (All India Decl5 Pattern)
10. Hb A 2 concentration in thalassemia trait is -
b) Blood smear
a)1 b) 1-2.s (PGI lune 99)
c) Bone marrow
c) 2'5-3.5 d) > 3.s
QUESTIONS

HEMATOPOIESIS 11. Which of the following haemoglobin (Iltr)estima-


tion will be diagnostically helpful in a case of beta
l Erythropoiesis starts in fetal liver during - thalassemia trait - (NEET Dec.12 Pattern, AIIMS May 06,IJP

a) 2-4 weeks (All India Dec15 Pattern) 08)

b) 4-6 weeks a) Hb-F


c) 6-8 weeks b) Hbl-c
d) 8-l0weeks c) Hb-A2
2. Which organ is the primary site of hematopoie$is in d) Hb-H
the fetus before midpregnancy - (AltMS May 06) 12, Diagnosis of beta Thalassemia is estatrlished by -
a) Bone b) Liver a) NESTROFT Test (CET Nov 14 Pattern)

c) Spleen d) Lung b) Hb A1 c estimation


3. The hemoglobin to appear first in the fetus is - c) Hb electrophoresis
(All India Dec.14 Pattern) d) Target cells in peripheral smear
a) HbA b) Hb A2 13. All of the following are true of p thalassemia major,
c) HbF d) Hb Gowers except - (AllMS May 06)

4" Emtrryonic hemoglobin is - (All lndia Dec.lj Pattern) a) Splenomegaly


a) Adult hemoglobin b) Fetal hemoglobin b) Target cells on peripheral smear
c) Gower d) None c) Microcytic hypochromic anemia
d) Increased osmotic fragility
5. When does switchover from fetal to adult hemoglo-
bin synthesis begin - (AIIMS Nott 04, Nott 05, May 05) t4. NESTROFT test is a screeing test for-
a) 14 weeks gestation a) B-thalassemia (All India Dec.14 Pattern,

b) 30 weeks gestation b) Hereditary spherocytosis AIIMS Nov 11)


c) 36 weeks gestation c) Autoimmune hemolytic anemia
d) 7-l0dayspostnatal d) Megaloblasticanemia
6. Feature of fetal RBC in adult - (AIIMS lune 98) 15' Hair on end appearance is $een in X-ray skull in -

a) Alkali denaturation resistant a) Thalassemia (AIIMS lune 97, PGI lune 97)

b) Small in size b) Hydrocephalus


c) Has more2,3 DPG level c) Chronic malaria
d) More iron than adult RBC d) Sickle cell anemia
16. ln a-thalassemia - (NEET Dec.12 Pattern)

ANEMIA a) Excess o-chain


b) No o-chain
7. Macrocytic anaemia in children is produced by all
c) Excess B-chain

except - (AilMS e6)


d) No p-chain
a) Thiamine deficiency L7. Bart's hydrops fetalis is lethal because - (Ar os)

b) Orotic aciduria a) Hb Bartb cannot bind oxygen


c) Copper deficiency b) lhe excess o, - globin form insoluble precipitates
d) Hypothyroidism c) Hb Bart's cannot release oxygen to fetal tissues
d) Microcl,tic red cells become trapped in the
placental
THALASSEMIA
18. A child died soon after birth. On examination there
8. Defect leading to thalassemia lies in - was hepatosplenomegaly and edema all over body.
a) Haemoglobin (CET lune 14 Pattern)
Most probable diagnosis in - (AIIMS May 02)

b) Osmotic fragility a) p-thalassemia


c) RBC membrane b) o-thalassemia
d) Platelets c) Hereditaryspherocytosis
d) ABO incompatibility/sickle cell anemia
9. In beta thalassemia, there is - (AtlMS May 01)
a) Increase in beta chain, decrease in alpha chain 19. 34 week primigravida punjabi khatri cornes with
b) Decrease in beta chain, increase in alpha chain history of consanguineous marriage, with history
c) Decrease in beta chain, decrease in alpha chain ofrepeated blood transfusion to her sitrling since 8
d) Increase beta
in chain, increase in alpha chain rnonths of age. The first diagnostic test is -
a) HPLC (All India Dec15 Pattern)
10. Hb A 2 concentration in thalassemia trait is -
b) Blood smear
a)1 b) t-2.5 (PGl lune 99)
c) Bone marrow
c) 2.5-3.5 d) > 3.s
.':: - t.t:: :iiiila,:

f
:-rttr : 1 ti: a ._ r.

Cxg.lyTF.t1,1,1Ii

d) Hb electrophoresis a) Type 1 b) Tlpe 2


iii c) \pe 3 d) None
.iri
SICKLE CEI.L ANERNIA
,lr:
IRON DEFICIENCY ANEMIA
20. The prirnary defet which leads to sickle cell anemia
is - 6I oi) 30. A nine month old boy of Sindhi parents presented to
a) An abnormality in porphy'in part of hemoglobin you with complaints of pregressive lethargy, irrita-
b) Replacement of glutamate by valine in p-chain of bility & pallor since 6 months of age. Examination
HbA revealed severe pallor. Investigation showed Hb-3'8
A nonsence mutation in the p-chain of HbA mgTo; MCV-58 ft; MCH-19.4pglcell. Blood film
c)
d) Substitution of valine by glutamate in the a- chain shows osmotic fragility is normal (target cells and
of HbA normoblasts)' X-ray skull shows expansion of erlth-
roid marrow. Which of the following is the most
11. Mutation leading to sickle cell anemia- (At 04)
(PGl Nov 1a) likely diagnosis -
a) Crossover mutation
a) Iron deficiency anemia
b) Frame shift b) Acute lymphoblastic anemia
c) Deletion c) Hemogiobin D disease
d) Nondysjunction
d) Hereditary spherocytosis
e) Pointmutation
22. Commonest presentation of sickle cell anemia -
31. First change of irnprovemcnt &slted after iron tkera-
(AIIMS Nov 98) py is initiaterl - @EET Dec.12 Pattern)
a) Decreased irritabilitY
a) Priapism b) Bone Pain
c) Fever d) SPlenomegalY
b) Reticulcytosis
c) Increase in serum iron levels
13. }lared-foot syndrome is seerr ln - (cET luly 15 Pattern)
d) Replenishment of iron stores
a) Frost bite
32. The earliest indicator ofresponse after starting iron
b) Sickle cell disease in a 6-year-old girl with iron deficiency is - (AI 06)
c) Raynaud'sphenomenon a) Increased reticuloclte count
d) Thalassemia
b) Increased hemoglobin
21. In sickle cell trait, nurntrer of trands found in Hb -
c) Increased ferrltin
a)2 b)1 (PGl lune 98)
d) Increased serum iron
c)4 d)s
25. Following are the findings in sickle sell Anemia, p_uso Rp cpAs
except - (PGI lune 98) E Rs oF u I,AT[ohl
a) Fish vertebra
b) Enlarged heart
c) Splenomegalyusually seen }IEMOPHILIA
d) LeukocYtosis
26. Salmonellosis is most comnton in-
33. Major source sf lkn Wiltrebrand factor ivWF)
a) Erl.throcltes (CET Nov.14 Pattern)
a) Sickle celi anemia (NEET Dec.12 Pattern)
b) Neutrophils
b) Thalassemia c) Endothelial cells
c) Hemophilia d) Monocltes
d) Cystic fibrosis
27. Patients ryith siekXe *eL]- ana;etrr;la c&n haYe infection 34" Hemophitric A have following diagnostic fcatures
except - (At e7)
with all except ' @ll India Dec15 Pattern)
a) J vrtt factorb) t prr
a) H. strePtococcus Pneumonia c) f pra) Normal BT
b) H. influenza-B
c) Myco tuberculosis
d) All of the above VON-WItI.EBRAND EISEASf;

35'. The commonest mode of inheritance of Von


G.5.PD DEFICIENCY " Willebrand's diasease - (AIIMS Nov 2000)

a) Codominant
28. l{}eieh erlzy \ae defi cieney causes leemolytic anemia-
(All India Dec.14 Pattern)
b) Autosomal recessive
c) Autosomal dominant
a) G-6-PD b) Aldolase
d) X linked recessive
c) Isomerase d) Enolase

29" Mo6t severe form of G-6-PD deficiency -


36, The coagulation profile in a l3'year old girl with
(All lndia Dec.
Menorrhagia having von Willebrandn disease is -
13 Pattern)
a) Isolated prolonged PTT with a normal PT (At 0s)
b) Isolated prolonged PT with a normal PTT c) Decreased ADP release
c) Prolongation ofboth PT and PTT d) Desordered platelet secretion
d) Proiongation of thrombin time 44. Glanzrnans disease is - (All lndia Dec.14 Pattern)
37. Von willebrands diseaee all are true except - a) Congenital defect ofplatelets
a) Factor VIII C deficiency (All India Dec.ll Pattern) b) Congential defect of RBCs
b) B.T. Prolonged c) Defect of neutrophils
c) Normal Ristocetin test d) Clotting factor deficiency
d) Defective aggregation
ITP
OTHER TACTORS DETKITNCY
45. Iilhich of the following is generally not s€ern in idio-
38. A newborn babyprc€nted!{ith prefusebleeding
pathic thrombocytopeaic purpurs (IrP) -
fum urabilical stumtrr aftet birth. Frobable diagno- a) More common in females (AIIMSNott 04)
$ls l$ -
b) Petechiae, ecchyrnosis and bleeding
a) Factor XIII deficiency (AIIMS May 07, Nov 06)
c) Palpable spienomegaly
b) VWF deficiency d) Increased megakaryocltes in bone marrow
c) Factor XII deficiency
416. A 5 year oldboycome$ with overnight pctechial
d) Glanzmann thrombosthenia
spot$ 2 weeks back he had history of abdonrinal pain
39. A lS-p*r-old femanc presentcd to the erncrgency
and no hepatosplenoraegaly. Diagnosis is -
departraent with history of recurent epistaxig
a) Aute lymphatic leukemia (AIIMS May 07)
hematuria and hematochsuia- There tas a history of
b) Aplastic anemia
profuse blecding ftom l:lre umbilicus stuurp at birth.
c) Idiopathic thrombocltopenis purpura
Previous investiptions rcvcaled nnrmal pmthrom-
d) Acute viral infection
bia time, acthated partial thromboplastin tfune,
thrombin time and &brinogen levels. Her platelet 47. Drug of choice inNeonatal trT? is - (AILMS eL)

counts aswell es pl&t€l€t funstion tests werc normal


a) Platelettransfusion
but urea clot lysis test was positive. trffhich one of the
b) Prednisolone
folloruing dotting factor is mnst likely to bc defi- c) Dexamethasone
cient- (AIIMS May 06) d) Gammaglobulin
a) Factor X b) Factor XI .18. ITP false is - (All lndia Dec15 Pattern)
c) Factor XII d) Factor XIII a) Platelet transfusion should be avoided
b) Anticedent history of febrile illness
c) Overactive immune system
HEMONRHAGIC DISEASE OF NEU'BORN d) Steroid should be avoided

40, Bleeding ia hemorrhagic diseaue of newborn is due


MISCELLANEOUS
to deficiencl'of factors - (All India Dec15 Pattern)
a) 2,7,9,r0 b) 3,7, e, 10 49. Child of 5 weeks with Hemoglobin of 10 gmTo pale
c) 2,8,9, 10 d) 2, s,9, 10
on examination, diagnosis is - (CET Nov 15 Pattern)
41. Feature of hemorrhaglc disease of new bora is - a) physiologicalanemia
a) Prolonged prothrombin time (AIIMS lune 97) b) pathological anemia
b) Defective platelet count c) thalassemia
c) Prolonged bieeding time d) Iron dificiency anemia
d) Prolonged thrombin time 50. Fanconi's anemia - false is - (All India Decl5 Pattern)
l
42. Late onset hemorrhagic disease nf newborn is char- a) Autosomal recessive
acterized by all ofthe followiag features txcept- b)
l

Pancytopenia
a) Usually occurs in cow-milk fed babies (At 06) c) Tlpe I RTA
b) Onset occurs at 4-72 week ofage d) All are true
c) Intracranial hemorrhage can occur. 51. Bloodvolume in preterm neonate is - (CET Aug. 13
d) Intramuscuiar r,'itamin K prophylaxis at birth has a Pattern)
protective role
a) 90 m1/kg b) 80 ml/kg
c) 70 ml/kg d) 60 ml/kg

PLATELAT* DYSFUNCTION 52" Knshaan's syndrome-treatment is' (AIIMS Nov 09)


a) Anti-thymoclte globulin + cyclosporin
43" In thromboasthenia there is a defect in - b) Anti-thymoclte globulin + cyclosporin + GM-CSF
a) Platelet aggregation (All India Dec.lj Pattern) c) G-CSF
b) Platelet adhesion d) GM-CSF
53. A *eonate delwercd at 38 weeks of gestation wi€h a QUESTIOHS OF VARIOUS OTHER EXATIfiIITA.
birth weight of. 2.2 kg &evelops intoleran cc to feedl TIOHS
zttern) abdominal diste*rioa on second dalr Physical ex&rfr-
ination is unremarkahle. Sqrsis screen is negative. 58. Fetal Hb is replaced by adult Hb cornpletfu x - (rN se)
PCY is observedtabeT?o/o.l{hich of the foltowing a) At birth b) 2 months
represents thebest management option - c) 4 months d) 6 months
a) Hydration with IV Fluids (AI 11)
ak
59. af l{bF in a 6 month old infant is - (KeraLa 97 )
b) Partial Exchange Transfusion a) 10 b) 30
c) Presumptive treatment for sepsis c) 5U d) 60
d) Medical Management for intestinal obstruction
'4. ffi. Iron requirement ie detcrmine&frtmlhe eqwatian -
Absent thumb, radial deviaticr of wri$t, bowing of
io. a) 3 x nt. (kg) x Hb deficit (gm/dl) @MU 05)
far e*rm withthrombocytopenia which investigation
b) 3.3 x vw.(kg) x Hb deficit (gm/dl)
.oq) need rtrt to be dane? (Aiims May 08)
c) 4 x nt.(kg) x Hb deficit (gmldl)
a) Echocardiography
d) 4.3 x wt. (kg) x Hb deficit (gm/dl)
b) Bone marrow examination
c) Platelet count 61. Quinineinducedthrawbtcytopeniais - (lipmer 04)

d) Karyotyping a) Antibody mediated


b) Dose related toxicity
55, bfCV (fi) in infart of I meinth of age is - (PGI Nov 0e)
rn
a) 76-80 b) 80-100 c) Idiosyncraticreaction
d) Inhibits production ofplatelets
c) 90-100 d) 101-12s
)7)
e) 125-t35 62" Fanco[i's anemiai*a. - (lipmer 0j)
a) Constitutional anemia
56. Reticulocl4elevel in netryborn is? (CET Nov. 12 Pattern)
b) Hemolytic anemia
a) 0.2-L.5o/o b) 1-r.6Vo
c) lron deficierrcy anemia
c) 2.5-60/o d) 6-10.20/o
d) Auto - immune anemia
37. In infrnt, bone'marrow biopsy is donc from -
(All lndia Dec15 Pattern) 63. Drug which wrtyletd to hemoly*is in a child with
a) Sternum b) Iliac crest G6PD deficienryis - (Karn 11)

c) Tibia d) A11 of above a) Penicillin


b) Primaquine
c) Ceftriaxone
d) Erythromycin

III
ANSWERS
HEMATOPOIESIS

t. Ans. is U'i.e., 8-10 weeks fRef, Nelsan t?thle ch. 4461

2nd-1Oth week of gestation {mosoblastic stage}

r 24 weeks - till birth {myeloid stage)

, i1i!r;ir- i:r$:rr:.;r.:::;::,+.::2,.:| t=ii:i?t :=:,.i.*r::r : aa'!:: at. t.

r After puberty (after 16-18 years)

) Ans. is'H i.e., Liver lRef: Robbin's Vh/e p. 620; Nelson lgth/e p. 19971
o During 2"d trimester, the major organ for hematopoiesis is liver.
3. Ans. is'd'i.e., Hb Gowers {Ref: Nelsan trB,h/e p. 20011
o During human development from early embryo to infancy different hemoglobins appear in the following
order -

Embryonic hymoglobin (Gower - 1, Gower - 2, portland)


(Earliest) J
Fetal hemoglobin

J
Adrtlt hemoglobin

, Ans. is t'i.e., Gower {Rel See above explanation)


5 Ans. is ?'i.e., 36 weeks gestation fRef: Internet; Nelson 18th/e p. 2001)
I During the initial fetal life, the major hemoglobin slnthesized in the body is fetal hemoglobin, because in fetus, the
chain synthesized predominantly is the y chain. p chain is present in only trace amount in early embryos The rate of
synthesis of y and B chains and the amount of HbA and HbF are inversely related during intrauterine stage. Asthe age of
the fetus increases, the synthesis of B chains and the amount of adult hemoglobin in the blood increases. At 36 weeks
of gestational age the rate of synthesis of p chain exceeds that ofy chain. This is called switch oyer.
6. Ans. is'i i.e., Alkali denaturation resistant IReJ: Nelson tsth/e p. 2002]
Features offetal RBCs.
c It contains less 2,3 DPG o It contains less iron than adult RBC
o It is alkali denaturation resistant o Life span isless.
o It is larger in size thqn adult KBC -) Larger in yolume

ANEMIA
7. Ans. is t' i.e., Copper deficiency fRef: O.P. Ghai ?th/e p. 333 6 Vh/e p. 302; De - Grouchy's S,h/e p. 11]
r Miscellaneous causes of megaloblastic anemia
t Thiamine deficiency s Hereditary orotic aciduria r Congenital dyserythropetic anemia
r Pyridoxine deficiency : Di-Gueglielrno syndrome : Hypothyroidism

THALASSEMIA

8. Ans. is ? i.e., Haemoglobin{Ref : O.P. Ghai S*/e p. 341 b Vh/e p. 307; Robbins Ze/e p. 632)
9- Ans. is 'b' i.e., Decrease in beta chain, increase in alpha chain {Ref : O.P. Ghai Btu/e p. 341 dl 7 /e p. 307; Robbin s
7h/e p. 632; Nelson 18tu/e p. 2033, 20j4)
p-globin chains,
oTheabnormalitycommontoallB.thalassemiasisdiminished synthesis of structurally normal
coupled with unimpaired synthesis of o-chain'
10. Ans. is 'd' i.e., > 3.5 lRef : Nelson 18th/e p. 20j5, 20i61

Genetics Homozygous Double heterozygotes Heterozygotes

4].r;i:l:1i|
.: --.j:::i...:r::...i.a.::.:a:,.!:jrj!::::::ri:ili;::i.aj:..l::iiiiiltt:,i:il:l::l::ii!:i!ri::'.,i::a:1,1t::1-:i):::.;::1,,.1.

HbF 30-907o
=d?a=j,s"
.

HbA, <3.5a/o < 3.5o/o 3.6 -8o/o

Nestrof test
o This test is being widely used in India and other developing countries in thalassemia
screening Programmes'
to HbA2 estimation for confirmation of thalassemia trait'
o The cases which are positive, subjected

i 1. Ans. is
,c,
i.e. HbA, lRef: o.P. Ghai 8'h/e p. j41 d, 7h/e p. 308; Nelson ISth/e p. 2035, 20361

Hemoglobin A2
p of the normal adult hemoglobin
o In hemoglobin A, the polypepties consists of two o and two 6 chains i.e. the chain
is replaced bY the 6 chain.
e In normal adults hemoglobin A, is present in trace amounts. It accounts for l.5o/o
lo 3.5o/o of the normal adult
hemoglobin.
Clinical imPlication of HbA2
r HbA2 ievels have special application in the diagnosis of B thalassemia trait'
r Elevation of UbA, is present in these cases even through the peripheral blood smear is normal' but
r Microcytosi, uri oth., morphological characteristic;f P-thalassemia trait are also seen in iron deficiency
unlike B thalassemia trait HbA, level is decreased'

t2. Ans.is'c'i.e.,HbelectrophoresislRef:NelsonlSth/ep'2035'2036)
'The diagnosis of thalassemia syndromes is best established by Hb electrophoresis"

Ans. is d, i.e. increased osmotic fragility


lRe!. a.P, Ghai 8th/e p, j41 & Vh/e p. i08; CMDT 2affi p' 479; Clinical
13.
diagnosis and management by laboratory method 12th/e p' 558)

Osmotic fragilitY test


o Red cells are suspended in a series of tubes containing hypotonic solutions
of NaCl varying from '9o/o to 0'0%
incubated at room temperature, for 30 minutes and centrifuged.
each NaCl concentration'
o The percent hemolysis in the supernatant soiution is measured and plotted for
o Cells that are more spherical, with a decreased surface/volume ratio have a limited capacity to expand in
hypotonic solutons oni lyrc at a higher concentration of Nacl than do normal biconcave cells'

a
o Conversely, cells that are hypochronic and Jlatter have a greater capacity
to expand in hypotonic solutions,
lyse at lower concentration, are said to haue ilecreased osmotic fragility.
cells with increased surface/volume ratio are osmotically resistant and
are seen in iron deficiencY,
. fh.r.
thalassemia, liver disease and reticulocytosis'

t4. Ans. is 'd i.e., Thalassemi Hemoglobinopathies by Anupam Sachdeva lth /e p' 100)
t fRef:

o NESTROFT ( Naked Eye Single Tube Red Cell Osmotic Fragility Test)
is a screening test for detection of beta
thalassemia trait.
p. & 6h/e p, 310; Chapman 3'd/e p' 392J
15. Ans. is d i.e., Thalassemia &'d'i.e., Sickle cell anemia [Rel O,P, Ghai }tu/e i43
Bone Changes in Thalassemia
1. Bones become thin and pathological fracture rnay occ.or'
2. Massive expansion of ttre marrow of face and skull
produce s characteristics facies.
3. Severe maxillary hlperplasia and malocclusion'
4. prominant wldening oi diploi. spaces with
"hair on end appearance" caused by vertical trabeculae'
cortical bone
Crew-cut appearance-+
5. striking expansion of hematopoieiically active marrow that erodes existing
and induces new bone formation giving rise to crew cut appearance.
.,Hair on end appearance" -+ Sickle cell anemia, Hereditary spherocltosis, G-6-PD deficiency
Causes of
15. Ans. is'H i.e,, No cr-chain {Ref. Robbiub ftle p. 635)
I People who do not produce enough alpha globin chain have alpha - thalassemia.
N7. Am" i$ '€ i,e., ffb Bart's cannot release oxygen to fetal tissues {R$: Robbins p/e p. 636)
'In the foetus excess gamma globin chains form tetramers (Hb Barts) with such high afrnity oxygen that they
for
deliver almost no oxygen to tissues' - Robbins
18, An$, is T Le.,afuha-thalassemia lRef: N*sn t*/e p. 2037; O.p. Ghai */e p. 3a2 d, #/e p. 311, 3127
r Out of the given 4 tlpes of hemolytic anemias only alpha thalassemia is able to give rise to such severe presentation
birth. In all other conditions the newborn is either normal or has mild jaundice.
at

19, Ans, is lf i,e., Hb electrophoresis {Re! Readbetowl


r Repeated blood transfusion to her sibling since 8 months of age suggests that the sibling is suffering from some type
of Hereditary anemia.
r In this case, the most probable is hemoglobinopathy with HbD, as HbD is more common in punjabis (and Gujratis).
r Tlpe of hemoglobin is detected by Hb electrophoresis.
r In this case, Hb electrophoresis of the woman should be done. If she comes positive for abnormal hemoglobin, she
should be counselled about termination of pregnancy.

SICKI.E C-LL AilEMIA


20" Ans' is "H i,e", Replacement of glutamate by valine ia p chafu of HBA {Ref Nelson 1ye/e p. 2026;
r Sickle cell anemia results from mutation in B-globin gene.
o It is caused by a point mutation at the sixth position of the
B-globin chain leading to substution of a valine residue
for a glutamic acid residue. resulting in sickle hemoglobin (HbS).
2t. Ans, i6'€ i.e.,Foiat mutation{Ref"O.p. GhaiBtulep. 3,M & frle p. 3101
)) Ans. is rY i.e,, pain
Bone {Ref: O.?. Ghai */e p. 344 {r fl/e p. 311; N*on t8h/e p. 2026, 2027; Robbins Vh/e p. 6j1,
5321

r Among the given options, bone pain is most common.


23. Ans. is'b'i.e., Sickle cell disease {Ref: Nelson l9/e ch. 4021
r Dactylitis, often referred to ashand-foot syndrome,is frequentlythe 1st manifestation of pain in children withsickle
cell anemia, occurring in 50% of children by 2 yr of age.
24" Ans. ie ** i,e.,2 {R€f, O.P. Ghai S*/e ?. 3451
o In hemoglobin electrophoresis, various t)?es of hemoglobin move at varying speed. In sickle cell trait (heterozygous
state), there are two types of hemoglobin - HbS (40%) and HbA (60%). So, on electrophoresis two different bands are
produced. In Homozygous state, almost all hemoglobin is HbS. So, single band is produced.
25. Ans, is 'l i.e., Spleaornegaly usually seeo lRef: Robbits 7*/e p. 630, 629; Chapman 3d/e p. 3921
r Splenomegaly is found in early stage of sickle cell anemia. By continued infarction and hypoxia - leads to progressive
scaring and shrinkage of spleen, so that by adolescent or the adulthood, only a small nubbin of fibrous tissue may
be left; this is'called autosplenctomy.
26. Ans. is 'l i.e., Sickle cell anemia {Ret O.p. Ghai Vh/e p. jl;
Haruison t7*/e p. 80a)
3

"Salmonella spp. and S. aureus are the major causes of long bone osteomyelitis complicating sickle cell anemia".

Harrison
)'7
--
Ans. is 'c' i.e.,Myco tuberculosis {Ref Nelson lg,u/e Chap,ter 462.1)
r There maybe infections due to capsulated organisms (Str. pneumoniae, H. influenzae) because of autosplenectomy.

G.6-PD DEFICIENCY

28. Ans. is 'a' i.e., G-6-PD lRef: O.P. Ghai B,h/e p. 339 b Vh/e p. 306; Nelson lyth/e p. 2041)_
r G6PD deficiency causes episodic intravascular and extravascular hemolysis.
19. Ans. is 'd i.e., Type 3 fRef: ltelsan l#hfe p. 2{}4t}
e It is of 3 types :
: Type I is the mildest form.
r Type II is a moderately severe form.
r Type III patients suffer from a non-spherocltic hemolltic anemia even without exposure to drugs.

IRON DEFICIENCY ANEMIA

30. Ans" is 'a' i"e., Iron deficiency anemia lRef: Reed tex below\

Hb 13 - 18 gmolo 3.8 gmTo Anemia


:j[rlav:::.'::.:.:.: .1. -:.. .:,,,, :',1,.. 'lllitrotyti!
SG98-.i1. ,,,..,;,,,..i:',.,.,,:,,-58..$.,, ,.:'.,',,;:,,,:...,,,,;;,;;,,,, :1

MCH pglcell
28-33 pglcell 19.4 pglcell
Antunia

o Microcltic hlpochromic anemia with normoblast and target cells suggests the diagnosis of Iron deficiency anemia.
r Though target cells are uncommon, they may be seen some times.
o Normoblasts : The bone marrow reveals a mild to moderate increase in erlthropoetic activity
manifested by increased number of normoblasts. - Robbins

31. Ans is'{ i"e., Decrcased irritability f&ef Wi*trobekhematotaglt l*fc p. 8291
"When specific iron therapy is given, patients often show rapid subjective improvment, with disappearance or marked
diminution of fatigue, lassitude, and other nonspecificc symptoms (e.g. irritability). This response may occur before any
improvement in anemia observed" Wintrobe's
o The eariiest hematological evidence of response of treatment is an increase in the percentage of reticulocltes and
their hemoglobin content.
a So,
T Overall earliest indicator of improvement --> decreased fatigue, lassitude and other nonspecific symptoms
I Earliest hematological evidence of improvement -+ Increased reticulocl.te count.
32. Ans. is'* i-e.,Increased Reticulocyte eount lRef : 0.P, Ghai #kle p,- 334 & Thfe p. j;all:
Reticulocyte count is the first to rise. This is followed by elevation of haemoglobin levels. Body iron stores are
restored alter correction of haemoglobin levels.
'Rise in Reticulocyte count occurs by the second or third day. This is followed by elevation of haemoglobin levels.
Body iron stores are repleted after correction ofhaemoglobin levels. ' - Ghai

OISORDERS OF COAGULATION

HEMOPHILIA

JJ. Ans" is'C i,e., Endothelial cells fRt' Rohbir;s 7h/ep. 6551
r Von-Willibrand factor is synthesized in vascular endothelium (major source) and megakaryoq,tes.
34. Ans. is t' i.e., t pf tnrf a.P. Ghai re/e p. 351 & 7/e p. 321]
c Hemophilia A is inherited as an X-linked rccessive trait.
o Laboratory findings ,f pff , Normal PT, Normal BT, and Normal platelet counts

VON-T'III LLEBRAN D DISEASE

35. Ans. is t' i.e., Autosomal dominant lRef Rabbids Tele p. 6551
r Most common pattern of inheritance is Autosomal dominant (> 90o/o)
36. Ans' is 'a'i'e', Isolated prolonged PTT with a normal PT
p. 207j1
IRel o.p, Ghai gtt/e p. jsl 6 6th/e p. 323; Nelson tYth/e

t von willebrand's factor acts as a plasma carrier of


factor VIII and circulates in the blood as factor vln-vwF
complex' rts deficiency thetefore impairs the intrinsic pathway
of coagulation and prolongs the prr as the intrinsic
pathway of coagulation remains unimpaired, pf is nit
alteriil.
)/. Ans' is '* i.e., Factor VIII C deficiency & t'i.e., Normal Ristocetin
test fRef: Robbinb 7/e p, 6s5)
o von willebrand disease is characterized by the deficiency
of vwF, while the synthesis of factor vIIIc is normal.
r vwF can be assayed by ristocetin agglutination test which measures the
ability of ristocetin to promote the
interaction between VWF and platelet membrane glycoprotein
Ib_IX.

OTHER FACTORS DEFICIENCY

38' Ans' is 'd i'e', Factor XrrI deficiency [Ref : l.B. Henry clinical diagnosis and management by laboratory
20th/e p, 650l
methods
e Bleeding from umbilical stump suggests Factor XIII deficiency.
39' Ans' is t' i'e' Factor XIII tRel !,8, Henry, clinical Diagnasis and Management by Laboratory Methads 20th/e p,
6so1
o Individuals with a positive bleeding history, particularly with features
such as delayed bleeding, umbilical stump
bleeding or miscarriages and in whom the initial panel of screening
test is negative , should be tested for Factor
XIII deficiency.

HEIIORRHAGIC D|SEASE OF NEWBORN


44. A.ns. is 'a' i.e,, 2,7r9,10l&ef: Nelson lgthle p. 103.4)
r Haemorrhagic disease of newborn is due to deficiency ofvitamin 'K', resulting in deficiency
of vitamin K dependent
factors (II, VII, IX, X).

41" Ans. is 'a' i.e., Prolonged prothrombine time lRef : Nelson lSthle p. 2029, zz31
o In hemorrhagic disease of the newborn there is delay in achieving
the normal vitamin K level.
t There is increased prothrombin time and increased partial thromboplastin time.
42. Ans. is'a'i.e., usually occurs in cow-milk fed babies
[Ref : Nelson rrth/e p. 773)
"Haemorrhagic disease of new born is more common in breastfed
infants because breast milk is a poor source of
vitamin K"
------ Nerson

PLATELET DYSFUNCTION

43. Ans. is ua' i.e., Platelet aggregation lRef : Robbin\ Vh/e p. 653)
Glanzmann's thrombasthenia
c Defect in platelet aggregation due to inherited deficiency ofglycoprote
in IIb-IIIa.
44. Ans. is'a'i.e., congenital defect of platelets lRef Rabbins vh/e p. 65i)

ITP

45" Ans. is t' i.e., Palpable splenomegaly tR# a.p. Ghai Bth/c p, 3sr 6 vh/e p. 3201

46, Ans. is t'i.e., Idiopathic thrombocytopenic purpure


lRcf : Nelson tSth/c p, za12, 20831
e Petechial spots without hepatosplenomegaly in a 5 year old boy suggest
the diagnosis of ITp.
47. Ans. is'd'i"e., Gammaglobulin [Rel Nelsan t}th/e p, 2ABZ]
-- t Ans. is 'd' i"e., Stererid should be avoideel fRef: Gh*i 7t'/e p" 32al
o Corticosteroids are used along with IV immunoglobulin. Other options are correct.

MISCELLANEOUS
Ans. is 'd i.e., Fhysiological anernia ffl'ef: A:newin irz inf*ncy, pedistric itc review *neric*n academy of pediatrics
2012)
r Physiologic Anemia of Infancy
1) Hemoglobin drops to low point at age 6 to 8 weeks
2) Erythropoietin nadir drops Hemaglobin
a) Term Infants: Hemoglobin drops to 9-11 g/dl
b) Preterm Infants: Hemoglobin drops to 7-9 g/dl
oc'i"e., RTA
Ans. is Type-I Nelson 78't'/e ch. 52.,4681
tflel
o There is type II RTA in Fanconi anemia (not type I RTA). Other options are correct.
Ans. is'd i.e,n 90 mllkg fkef: Maternal,fetal andneanatal physiologyby Susan Tucker Blarklsumt 4'h/e p. 235; Advsnces
iw pediatrics by Dutta p. 1541

mp Preterm 9o-1 10 ml/kg (average ml/kg)


'tor llo
-tlerm .,,80.90 m|/kg {average85. n'*lkg}' .
1-3 months 7s ml/kg
3 months *'adults 70 mVkg

52. Ans. is t' i.e., G-CSF fRef: Nelsan lSth/e p. 9131

Kostmann's syndrome (severe congenital neutropenia)


o Kostmann's syndrome,an autosomal recessive disorder, is an inherited disorder of the bone marrow in which there
is arrest of maturation of neutrophils at promyelocyte stage.
e There is congenital neutropenia and neutrophils are often completely absent in the blood at time of diagnosis.
r Because of neutropenia, these patients suffer from severe infections e.g., omphalitis (infection of navel), Pneumonia,
Skin abscesses, otitis media.
o Kostmann's syndrome is believed to be caused due to defect in receptor of granulocyte colony stimulating
factor (GCSF) on neutrophils (granulocytes). The purpose of this receptor is binding of the granulocyte to the
cy,tokine (GCSF) in order to give signal to the cell to mature and multiply.
o Patients with Kostamannt syndrome produce GCSF but because of the defet in GCSF receptor the response of
neutrophils to normal amounts of GCSF in the blood is reduced. However, they can respond if the amount of GCSF
is increased -+ These patients will respond to higher dose of GCSF,

53. Ans is 'b' i.e., Partial Exchange Transfusion fRef Nelson 18th/c p. 77jl
r PCV 72o/o in a neonate suggests the diagnosis of neonatal polycl'themia.
c Polycythemia is defined as central venous hematocrit [also calleil packed cell volume (PCV) or erythrocyte volume
fraction (EVF)I level of greater than 650/o.
o The neonate in question is also having intolerance to feed and abdominal distension.
r So, this neonate is having symptomatic neonatal polycythemia.
Treatment of neonatal polycythemla
o Therapy in newborns with polycy,themia is based on both the measured centrai venous hematocrit level and the
presence or absence of symptoms.
(1) Symptomatic polycythemia
o Partial exchange transfusion is the treatment of choice.
(2) Asymptomatic polycythemia
r In asymptomatic polycyhthemia, treatment depend upon hematocrit level (PCV).
(i) PCV 65-750/o
r Perform cardiorespiratory monitoring and monitoring of hematocrit and glucose levels every 6- l2 hours
and observe the patients for symptoms
r If hematocrit (PCV) becomes more than 757o, consider partial exchange transfusion (pET).
(ii) PCV > 75o/o
r Partial exchange transfusion (pET) is the treatment of choice.

54. Ans. is'd i.e., Echocardiography lRef; Internet reference|


o Characteristic features such as absent thumb andthrombocytopenia suggests Fanconis anemia.
Features of Fanconis anemia
r Skin pigment change + Darkened areas of the skin, cafe-au-lait spots, vitiligo
o Upperlimb --) Absent or hypoplastic thumb, absent or hypoplastic radii, dysplastic ulnae,
clinodactyly
o Gonads -+ Abnormal or atrophic testis
o Head and face -+ Microcephaly, sprengels deformity, spina bifida
r Kidneys -+ Ectopic or pelvic, horseshoe, hlpoplastic or dysplastic
r Abnormal eyes/eyelid
r Hematological abnormalities
Pancytopenia + thrombocytopenia and neutropeniathese are the most common and most serious symptoms of
Fanconis anemia.
o Bonemarrowfailure -+ ItsamajorcomplicationofFanconisanemia
o Acute myeloid leukemia -+ Fanconis anemia have elevated risks of developing acute myeloid leukemia.
o Myelodysplastic syndromes -+ Increased risk of myelodysplastic syndrome m f'anconis anemia.
Common tests for Fanconis anemia include:
o Complete blood count (CBC) o Bone mar?ow biopsy o Chromosomal breakage analysis
o HLA tissue typing o Hearing test o Hand X-ray and other imaging studies
o Developmental test r Ultrasound of the kidneys e Amniocentesis or chorionii rillou, campling

55. Ans. is 'H i.e., 80-f 00 lRef: A,P. Ghai Bthle p. 332 & */e p. 298, Nelson IBn/e p. 2}r.f]
r Mean MCV at the age of 1 months is i04 fL.

,.Birth
bloodf
(c-or! 16.s 5j 108 18.1
1-3 days 18.5 56 10g t B.9
, d.rr... s s! 107
.)l .. .. 12.2

1 month 14.0 43 104 10.8

'
6months 11.5. 91 j1.g

2-6 years 12.5 37 81 9.1

12-18 years 14.0 42 90 8.1

56. Ans. is 'e' i.e.r 2,5-6o/a fRef: Wintrobe's elinic*lhem*tolagt llthle p. S|


o Reticulocyte count is the percentage ofreticulocy.tes in total red blood cells.

: of.reticulocytes
Reticulocyte count Ym9er x 100
Total number oIRBCs
o The reticulocyte count serves as an important tool to indicate shortened RBC survival and the subsequent appropriate
response by the bone marrow to increase RBC production.
o Reticulocl'tes are young RBCs that have just left the marrow but still contain residual RNA.
o Normally, they remain for only 1 to 1.5 days in the blood.
o Normal reticulocytes count in an adult is 0.5% to 2%.
o In normal newborn range is 1.8o/o tg 5.8%.
i -. Ans. is t' i.e., Tibia [Rel lournal of Pediatric 1953; Pediatric (r child health 20081
Sternum: ->
o Hematopoietically active
o Site in cooperative patient like adult
Ilium: -+
o Apprehensive patient
o Useful in older children & adult
Tibia: -+
o Useful in newborn & infant and children below 2 year of age.

ANSWERS OF VARIOUS OTHER EXAMINATIONS

58. Ans. is'd'i.e.,6 months fRef: Nelson 18th/e p. 20011


o Synthesis of HbF decreases rapidiy postnatally, and by 6-12 months of age only a trace is present.

59. Ans. is'a'i.e., l0Yo lRef: See above explanation]


o By 6-12 months, only a trace is present.
o Amongs the given options, l0% is the minimum and therefore the best answer.

60. Ans. is 'c' i.e.,4 x wt (kg) x Hb deficit (gm/dl) fRef: O.P. Ghai 8th/e p. 335 dl Vh/e p. 301)
o Blood volume ml/kg. Each gram of hemoglobin contains 3.4 mg of iron. 50% of more iron should be
averages 75 to 80
given to replenish iron stores. Therefore iron requirements are determined from the equation.
Iron (mg) = r,r,t (kg) x Hb deficit (g/dl) x 80/100 x 3.4 x 1.5
= m (kg) x Hb deficit (g/dl)'x a
61. Ans. is'd i.e., Antibody mediated lRef: Gooilman & Gilman 10th/e p. 10891

o Quinine cause immune mediated thrombocltopenia.

62. Ans. is'a'i.e., Constitutional anemia [Ref: O.P. Ghai dt Th/e p. 313; Nelson 18'hle p. 20471

Constitutional pancytoPenia
r Constitutional pancltopenia is defined as decreased marrow production of the 3 major hematopoietic lineages on
an inherited basis, resulting in anemia, neutropenia and thrombocltopenia.
o Important causes are : -
s Fanconi anemia r Amegakaryocytic thrombocltopenia
r Shwachman-Diamond syndrome r Down syndrome
r Dyskeratosis congenita r Noonan syndrome

63. Ans. is'b'i.e., Primaquine lRef: Harrison 17h/e p. 657)

TII
KX\$MffiEKXETffiLffiGY

GROWTH HORMONE

Growth hormone deficiency (Hypopituitarism)


r Growth hormone defi.ciency may be congenital or acquired (infection, radiation, tumor infiltration). The child is
83r.
usually of normal size and weight at birthqrtMs This is followed by prolonged physiological iaundice.
r Clinical leatures are :-
t Short chitd.ren with normalbody proportions( r Truncal obesity.
r Markely increased subcutaneou 5 fal4nr'as
on) t High pitched voice6ttMso4)
t Proltortionate growth retardationAilMs
04' AI 8e) r Depressed nasal bridge
r Delayed skeletal age than chronologicalage4urusoa) r Prominent philtrum
r The height age is less than skeletal age and chronological age. r Frontalbossing
r Hypoplastic penis (micropenis(Af'usoa)) and scroturn t Delayed PubertYartus tss
r May present with severe hlpoglycemic,AilMs,4'ee) con'ulsions. t Delayed tooth eruptiottattMss3)
r Crowding of midfacial features t D elayed epiphyseal fusionallMs 83 )

r Genitals are small (sexual infantilism){efirusa:r t No mental retardation@Gt te)


: [.Jsually there is no family history.
r Dwarflsm from GH deficiency should be differentiated from other causes (differential diagnosis) :

r Family history +ve +ve usually -ve _ve

r Birth weight and height Normal Low Normal Normal

. Pattern of growth Slowfrom infancy Slowfrom Slow from a few Slow frorn birth
bwlh after birth months

c Epiphysealdevelopment Moderate progressive Normal Progressive Severe retardation


retardation retardation

r Puberty Delayed Normal Delayed Delayed

r5l
r End organ resistance to GH (GH receptor resistance) causes 'Laron syndrome' or 'Laron dwarlislnr(DNB
Hyperpituitarism
o MostcommoncauseofhyperpituitarismishormonesecretingpituitaryadenomaarlMs0l).
r The GH excess in children produces gigantism (in adults it causes acromegaly). There is accelerated growth with
enlarged body size (giant). Fatigue and weakness are earliest symptoms due to miid proximal weakness with muscle
atropy, i.e. giant child with weakness (weak giant(ArtMs04)). There may be features of compression due to tumor, e.g.
headach and visual field defects. Macroglossia, broad nose, raised ICI carpal tunnel syndrome and kyphosis may be
seen.
o Oral glucose challange test confirms the diagnosis (there is no decrease in GH leveis after glucose administration. In
normal person, glucose decreases GH secretion).

ADRENAL CORTEX AND AMBIGUOUS GENITALIA


o Adrenal cortex has three layers zona glomerulosa, zonafasciculata, and zona reticularis. The zona glomerulosa is
the outermost layer and is the site of mineralocorticoid synthesis@EEr). The middle layer (zona fasciculata) and the
innermost layer {zona reticularis) synthesize and secrete glucocorticoids and androgens. Though both layers (zona
fasciculata & reticularis) secrete both h orri'ones, zonafasciculata mainly secretes glucocorticoids and zona reticularis
m ainly se cr etes andro ge n s.

Familial glucocorticoid defi ciency


o It is a rare autosomal recessive disorder characterizedby low cortisol (glucocorticoids) with normal mineralcorticoid
secretion.
r It is characterizedby hypoglycemiaal ll' AItMs 1r' r0' 0e),
severe infections, hyperpigmentation, feeding problems, failure
to thrive, regurgitation and seizures (due to hypoglycemia).
r Zona fasciculata and reticularis are atrophied (thus there is deficiency of glucocorticoids), but zona glomerulosa is
preserved (thus mineralocorticoid is unaffected).

AMB!GUOUS GENITALIA

r Ambiguous genitalia refers to discrepancy between external genitals and internal gonads. These can be divided
into :-

A) Femalepseudohermaphroditism
, : Genotype is XX@rrs). Internal gonad is ovaryarrs) but external genitalia are virilizedar ls) (male differentiation).
t Congenital adrenal hyperplasia (CAH) due to 2l-hydroxylase deficiency is the commonest causeat NEEI, 13, 02,

pcr 07)
,

r Other causes are maternal virilizing tumor (arrhenoblastoma),maternal androgen administration, CAH due to
1l-beta hydroxylase deficiency, fetal placental aromatase deficiency{at tt), and WNT-4 gene mutationqt tt).

B) Male pseudohermaphroditism
: Genot-7pe is XY(ArIs). Internal genitalia are testis but external genitalia are of femalearrs).
r Important causes :-
l) Defect in testicular dffirentiation: Deletion of short arm of Y chromos ome,XY pure gonadal dysgenesis@Gl
pcl
oo
, Mixei gonadal dysgenesisal 13, 08)
.

2) Drtctintesticularhormonesynthesis:Leydigcellaplasia, 17-ahydroxylasedeficiencyqr13,PGros),3-BHSD
deficiency, 17-20lyase deficiency, 20-22 desmolase deficiency, l7-ketosteroid reductase deficiency.
3 ) Defect in androgen action : S-ct reductase deficiencyQct 0t)
, testicular feminization syndromea'ut) , Reifenstein
syndrome.
4) Defect in mullerian inhibiting hormone.
C) True hermaphroditism
r Both ovarian and testicular tissues are present either in the same (ovotestis) or opposite gonads.

CONGETTIITAL ADRENAL HYPERPTASIA (CAH)

o CAH, also called adrenogenital syndrome@NB rz), is an autosomal recessiye disorder caused by deficiency of e nzqes
involved in synthesis of steroid hormones in adrenal cortex.
r The deflcient enzyme maybe 2r-hydrorylase (most common(err3,DNBr2,NEED),l1-beta hydroxylase, 17 cr-hyd.roxylase,
3B- HSD, and cholesterol desmolase.

Pathophysiology
e All steroid hormones are symthesized from precursor pregnenolone which inturn is derived from<holesterol.

ACTH (adrenal);
LIIIT'SH (gonadal)

Cholesterol I
DHEA.S
I Cholesterol side chain cleavage enzyme (CyP 1t A)
+
Pregnenolone lTc[OH t IToH pregnenolon. J?9-lYu,Se'
I 178 HSD
------a------)
DHEA

J ,or',
Androsrenediol

Progesterone lTaoH
J,ors, lrBru
l7F nto
I

tI3BHSD
, t7 oH Progest..or. JZ?9-lY,ut1 Androstlnediorr" , Testosterone
I

t|
| 2l hydroxylase 2t hyd.o*ytu." I Aro-utur. Aro*utu."
t +
I
+
Deoxycorticosterone Deoxycortisol 178 HSD
I
,I Estradiol

iI
I I I hydroxylase llBhydroxylase
i
Corticosterone Cortisol
I

I I8 hydroxylase
t
18 OH CorJicosterone

tI
I
18 HSD -+ Renin
Aldosterone

li :.i:li i:i:l;ii::i:iS,&lr:S'ffi d$l.t::tt

o Now see effects of different enzyme deficiency


1) U-alpha hydrorylase deficiency
rAll pregnonolone is converted to mineralocorticoid pathway. So, there will be : -
i) Excess of mineralocorticoids -+ Sqlt retention & hypertensional 07)
ii) ' No androgens -) Fflminization of external male genitalia (male pseudohermaphroditism){Nntr truI). Female
development will be normal.

2) 1l-B hydroxylasedeficiency
I Cycle will go upto deorycorticosterone and deoxycortisoleurrMsll,Aroe,DNB13) which are raised. So, there will be

i) Excess deoxycorticosterone -) It has mineralocorticoid activity which results in hypertension and salt
1, 0e).
f etentio nqIIMs 1

,, ii) Excess androgen -+Yirllization of female (female pseudohermaphroditism).Inmales, there will precocious
puberty due to excess ofandrogens.
3) 21 hydrorylase deficiency
r Cycle will go upto progesterone. So, there will be :-
i) Deficiency of mineralocorticoid -+ Saltlosing - hypotension,hyponatremia,low chloride, hyperkalemiq,(NBr
AilMS 03)
.
ii) Deficiency of cortisol (glucocorticoid): HypoglycemiaallMs 03' PGI00),
hyperpigmentation (due to excess ACTH)
iil) Excessandrogens-+Yirtlizationoffemale(femalepsueohermaphroditism).Inmales,therewillbeprecocious
puberty due to excess of androgens@EEr).

4) 3p hydrolase (3b-HSD -+ 3b hydroxy steroid) deficiency


: Allmanifestationswillbesimilarto2lhydroxylasedeficiencybecauseofdeficientmineralocorticoidsandexcess
of androgens('ar ts) (especially DHEAaT Is)).

r ln all these enzymes deficiency, there will be loss of feed back inhibiton on ACTH because of deficient
cortisol. This results in excessive ACTH secretion and hyperplasia of adrenal --+ Congental adrenal
hyperplasia.
r ln all types of CAH, t here is hypoglycemia9claildueto deficiency of glucocorticoids.

2 1 -Hydroxylase defi ciency


r 21 - hydroxylase deficiency is an autosomal recessive(ArIs) disorder, which is divided into three t)?es:-
A) Classic salt wasting form
r This is a severe form ofdeficiency characterizedby features ofboth glucocorticoid (cortisol)/mineralocorticoid
deficiency (salt wasting) and androgen excess. (Ambiguous genltalia in females).
r Males may be normal at birth. This form typically manifests early, between 7-21 days of life and is a potentially life
thre at ening condition.
o Symptoms of classical CAH due to 21 hydroxylase deficiency are :
l) Clinical: Lethargy, vomiting, Anorexia, weight loss, dehydration, hypotension, shock(Nttr'A111'08'03).

ii) Mineralocorticoid deJiciency: Hyponatremia, hyperkalemia, hypotension(NEEr' AI 11' 08' 03) .


iii) Cortisol deficiency: Hypoglycemiaar11'08' 03), Hyperpgimention of areolar and scortal areas in males and
genital creases in females (dueio t eCfU secretion).
iv) Excess Androgen
t Males: Phenotypically normal for some years, precocious puberty.
t Females: Virilization (female pseudohermophrodite) .
ii) Deficiency of cortisol (glucocorticoids): HypoglycemiaallMs 03' pGI00),
hyperpigmentation (due to excess ACTH)
1rI) Excessandrogens)Yinlizati.onoffemale(femalepsueohermaphroditism).Inzales,therewillbeprecocious
puberty due to excess of androgens@E'r).
4) 3p hydrolase (3b-HSD --> 3b hydroxy steroid) deficiency
r Allmanifestationswillbesimilarto2lhydroxylasedeficiencybecauseofdeficientmineralocorticoidsandexcess
of androgens(ArIs) (especially DHEAarr5)).

ln all these enzymes deficiency, there will be loss of feed back inhibiton on ACTH because of deficient
cortisol. This results in excessive ACTH secretion and hyperplasia of adrenal + Conoental adrenal
hyperplasia.
ln all types of CAH, there is hypoglycemio(Pctoo)due to deficiency of glucocorticoids.

21 -Hydroxylase defi ciency


e 21 - hydroxylase deficiency is an autosomal recessivearrs) disorder, which is divided into three t)?es:-
A) Classic salt wasting form
o This is a severe form ofdeficiency characterizedby features ofboth glucocorticoid (cortisol)/mineralocorticoid
deficiency (salt wasting) and androgen excess. (Ambiguous genitalia in females).
o Males may be normal at birth. This form typically maniftsts early, between 7-21 days of lift and is a potentialb) life
threatening condition.
e Syrnptoms of classical CAH due to 21 hydroxylase deficiency are :
r) Clinical: Lethargy, vomiting, Anorexia, weight loss, dehydration, hypotension, shock@EEr' AI
11' 08' 03)
.

ii) Mineralocorticoid deficiency: Hyponatremia, hyperkalemia, lrypotension(NEEr' AI 11' 08' 03).

111) Cortisol defciency: Hypoglycemia(Al 11'08' 0'), Hlperpgimention of areolar and scortal areas in males and
genital creases in females (due to 'l eCfU secretion).
iv) Excess Androgen
t Males: Phenotypically normal for some years, precocious puberty.
t Females: Virilization (female pseudohermophrodite) .

B) Classic simple virilizing form


o Patients with classic simple virilizing form have impaired cortisol biosynthesis but do not develop salt loss. Clinical
features are caused solely by overprod.uction of androgens.
r The only manifestation is ambiguous genitalia in a female child with varying degrees of clitoral enlargement and
labial fusion.
tzt.
o BP is normal and there is no salt wastingldr
r Dependingonseverityofclitoralenlargementandlabialfusion,the diagnosismaybeestablishedatbirth,However
such patients may present in early childhood.

C) Non-clasJic(aquired/late) from
r Patients with non-classical form or late onset form produce normal amount of cortisol and/or aldosterone, but
at the expense ofproducing excess androgens.
o These usually present in childhood or early adulthood with premature pubarche & symptoms/signs of PCOD.
o Hirsuitism, Acne and Oligomenorrhea are the most common presentingfeatures.

. Recommendations for pregnancies ar risk of CAH consists of administration of dexamern"ron"aosr (20


mg/kg). lt started by 6 weeks of gestation(Aroor, this ameliorates the virilization of the external genitilia
in affected females chrionic villus sampling is then performed to determine the sex and genotype of the
fetus; therapy is continued only if the fetus is an affected female.
CHa.prpR

The most probable diagnoois in a child with following picture -

a) Cushing syndrome
b) 2l-hydroxylase deficiency
c) I 7-o hydroxylase deficiency

d) None

Ans. is'b' i.e., 2f -hydroxylase deficieney


r The figure is showing female pseudohermophroditism (clitoromegaly with fusion of labia majora and
rninora, and Ambiguous genitalia). The most common cause {and the only cause in given options) of female
pseudohermophroditism is 21 -a hydroxyiase deficiency.

GO.lrApAL prsoRpERS

o Gonadogenesis refers formation of gonads, i.e. testes in rnales and ovaries in females. Genital ridge (urogenital
ridge) is the site where gonads develop. 'Primordial germ cells' are developed in the 4th week by proliferation of
ee)).
endodermal cells(NnEril07) of the dorsal wall of hindgut (part of yolk sacai0s''{P The primordium germ cells
migrate into genital ridge, where proliferation of both germinal and nongerminal cells leads to formation of gcnads.
a WT-1 and SF-l geaes stimulate genital ridge to'form primitive (bipotential) gonad which then differentiate either into
ovary or testis. DAX-I , WNT-4 and RSPaI genes(xt:t:t) are involved in developmett of female gonads (ovary); and
SRY gensdrn; is involved in development o{ male gonds (testes)(NEE?) from primitive (bipotential gonads).
e In male embryo at 6-7 weeks sertoli cells secrete anti-mullerianhormone (AMH), also called mullerian inhibiting
substance which causes regression of mulierian (pararnesonephric duct), the major ducts in fernales for external
genitals. Then the leydig cells start secreting testosterone under the influence of placental lTCG(^rEEr) il 8-12 weeks.
Testosterone causes development of Wolffjan duct (mesonephric duct), the main genital duct in males for external
genitals.
e In female embryo, AMH is not secreted by c;vary, and mulierian duct development occurs.

Mutation in genes involved in gonadaldevelopment


o WT-l gene mutation leads to Deny-Drash syndrome, Frasier syndrome and WAGAR syndrome, all of which are
associated with ambiguous genitalia and dysgenic testis.
r SF-l gene mutation can lead to gonadal dysgenesis and adrenal insuficiency.
c SRY gene translocation on chromosome-X results in masculinization and ambiguous genitalia in females.
a WNT-4 gene mutation results is female pseudohermaphroditism (ambiguous genitalia in female){u ttt.

Androgen receptor insensitivity syndrome


o Androgen insensitivity syndrome, formerly called testicular feminization(D'\Br3), are XY individuals(''Yu"'atos) (male
karyotlpe) with afemale phenotype, i.e. male pseudohermaphroditism (ambiguous genitalia in males).
a Il is X-linked recessive disorder due to mutation in androgen receptors causing resistance to androgen action.
o Because androgen actjon is required for development of Walllian duct, thes'e patients have ftyp oplasitc/aplastic Woffian
duct, female external genitalia (lower 1/3 of vagina); Scant pubic hair and scant axillary hair.
o Testes is present it inguinal canal or abdomen (undescended fesfs), which produces testosterone and antimullerian
hormone (AMH). As testosterone receptors are insensitirre, it cannot act and testosterone is converted to estrogen.
Estrogen causes feminization of male.
e AMH leads to inhibition of mullerian duct thus uterus, fallopian tube and upper 2/3 of vagina do not develop, and
these patients fail to mensturate at puberty (primary amenorrhea).
c As testis is undescended, testicular malignancy can develop in adulthood. Thus orchidectomy is done in late
adolescence or eaily ad.ulthood.
illl

tz, Endocrinology

Precocions puberty
o Puberty before the age of 8 years in girls
or 9 years in boys is considered precocious
age of l0 years in giils is also considered as precocious. Ttuberty. Menarche before th,

c Precocious puberty is of two types :_


1. Central or true precocious puberty
I Results from excessive GnR'H, gonadotropins
and target sex hormone elab.rated by premature
hypothlamic pituitary-gonadal activation o:
(HpG) axis.
2. Peripheral orpseudo_precocious puberty
r Due to increased sex steroid secretion from either the adrenal
gland or the gonad5roxr r.st.
r It is independent of HpG axis activation

f-.;'i#;F:Ei?e:li
A. Central precocious puberty
1. ldiopathic : Sporadic or familial.
2. Central nervous system abnormalities
i) congenital anomalles of c NS: Hypothalamic hamortomaailMsss,s3),hydrocephalus,
porencephaly, arachnoid
" cysts.
ii) Acquired lesions of CNS : lnflammation, granuloma,
trauma, surgery, radiation@ctos),chemotherapy.
iii) Tumors of cNS : pinear tumors, optic arioma, ependymoma, carniopharyngioma.
iv) Hypothyroidism
B. Peripheral precocious puberty : lsosexual

1' avarian cquses: Mccune-atbright syndromE@Gtazn*ss'sl,


benign follicular cysts, gxrn ulosa-thteca celltumor-
-'-'.'"'i''\Jt5'7"e'vrq-r"r
s{Dt't,15,Aily5e5l;Gonadoblastoma..
2.' Adrenal causes : Feminizing adrenal neoplasia
3. Exogenous estrogen administratian

Boys
1. Testis: Leydig cell tumor, adrenal rest tumor,
testotoxicosis.
2,Adrenal:CAH{21orlt.fihydroxytasedefitiengy)tNgs6atMses,s3),yirilizingtumors{o|/',5].
3' hcc secreting tumors : Hepatomb, hepatobtastoma,
choriocarcinoma, chorionepithelioma, teratoma,
dysgerminoma.
4. Exogenoustestosterone
C. Heterosexualprecocity
1'Girls: virilization in girls due to virilizing CAH,
ovarian or adrenal neoplasia, polycystic
ovarian disease.
2'Eoys: Feminization due to estogen producing adrenal
tumors, exogenous estrogen, marijuana
smoking.
Note: Hypothvroidism usually causes delayed puberty,
but juvenite hypothyroidism ,";".;;.;;;
puberty. .r;;" pr"i'o.,o*

,lg-l
l.rl
Me@
r Central precocious puberty secondary to hlpothalamic
hamartoma.
Delayed pubertY
of 14 years in
signs of sexual development by the age
c Delayed puberty is ilefineil as failure of development of
budding by 13 years or absence of menarche by 16
boy. krgirls, delayed puberty is defined as failure of breast
17 years'
year(Alls) or lack of secondary sexual characters by

o Etiology of DelaYed PubertY:


l.chronic systemic iliseases?Gl06) ' Gluten enteropathy, anorexia nervosa' ot)
2. Constitutional delay in growth and pubertal development
-+ most common ca useqr '
oat'
3. Hypogonadotropic hypogonadism (Low FSH' LH){cet
or pituitarl level and
r It is also called as secondary or central hypogonadism. The defect is at hypothalamus
gonads have no stimulation by FSH and LH -+ No/reduced
there is no/reduced secretion of FSH and LH. Thus,
secretion of sex hormones'
traumatic iesions of CNS :
a) CNS causes : Congenital anomalies, intracranial tumors, inflammatory
diseases,

b) Primary gonadotropin deficiency : Hlpopituitarism' l'' Laurence-Moon-Biedl-Bardet'


rz),
c) Syndromes with gonadotropin deficie,ncy, Kallmanntu Prader-williLl
Froehlich.
0o' hyperprolactinemia'
d) Miscellane ous : Hypothyroidisffi@Gr
rst disordets) t
4. Hypergonadotropic hypogonadism (high FSH' Lgret ' Gonadal
Defect is at gonads level' There is no secretion of sex
r It is also caleJas primary or peripheral hypogonadism.
is not there -> increased secretion of FSH and LH(Arrs)'
hormones, as a result feedback inhibition on FSH and LH
(47 x){ysto,,,), Swyer,s
a) Turner syndrome (45 xqtlto",oul' Noonan syndrome, Klinefelter syndrome
slndroms(Alrs)
b) Anticancer therapy for neoplasms of gonads'
12)
c) orchitisql
d) Vanishing testes sYndrome.
e) Testosterone biosynthetic defects'
f) Inf,ltration and autoimmune disease of gonads

Cryptorchidism
of sexual differentiation in boys' At birth approxi-
r An undescended testis (cr-ptorchid) is the most common disorder
mately 4-5o/o of boys have an undescended testis'
first 3 months of life, and by 6 month
the
o The majority of undescended testes descend spontaneously during the
incidence decreases to 0'8%'
o If the testis has not descended at 4 months' it will remain undescended'
o Cryptorchidism is bilateral in 20o/o of cases'
o Consequences of cryptorchidism are :-
i) InfertilitYeqot)
ii) Testicular maligancy -+ Most common seminoma@Glo')
iii) Psychological effects of empty scrotum'
iv) Torsion
v) Associated hernia
done between 6-12 months (6 months is most
o Surgical orchidopexy is the treatment of choice and should be
06' At 04' ee)
aPProPirate)L|IMs '

THYROID DISORDERS

HYPOTHYROIDISM
hormone or a defect in thyroid hormone receptor activity'
o Hypothyroidism results from deficient production of thyroid

Congenital hYPothyroidism
(most common cause);abnormalities in development
o congenital hypothyroidism may be causedby iodine deficiency
l::il::::;i:|j4:;,::i.:.1:: :l::,i:t::::, :::,:.:).:,:,:
;i.:;:t!r;tla::::,a.::l:r.:,,..::f.:::
. ):,/t:..; :.. .

, ...a. -
z Eidouinology l

of thyroid (thyroid dysgenesisarrr)) or thyroxine synthesis; and presence of maternal anti thyrotropin antibod.ies.

o Mutation in Pendrin gene (SL26A4) located in chromosome T leads to pendered syndrome(Pcr04) which is characterized
by bilateral sensorineural hearing loss@Gl
04),
iliffuse colloidal {oitys@et o<; which later becomes nodular@c, or),

hyp othyro dism and mental retardation(Pcl 0a)


.

Clinical features
r Neonate is normal at birth due to presence of maternal thyroid hormones.
o First clinical sign is prolongation of physiological jaundice due to delayed maturation of glucuronide conjugation.
r Other features are :-
r Open posterior fontanel 6rts)ur4 *14" sutures r Umbilical hernia(Pclo
t Large tongusqtas) -+ Can cause respiratory dfficulties r Edematous, characteristic facies
r Rough dry skin(Dvr rs) which is cool & mottled : Constipation
r Hlpotonia, large abdomen, flaccid muscles r Pallor, hypothermia
r Sleepy, little cry, poor feed & chocking spells during feeding. r DelaYed PubertY(Pct 06' AnMs e!)
r Short neck with thick supraclavicular pad of fat may be present. r Refractory anaemia common
: Marked physical & mental retardation(DNB rs, ArMs oi),delayed social r Siow pulse, cardiomegaly & Heart murmur.
smile. r Delayed dentition.
r Short stature(oc'06), normal head size with short extremities. r Open mouth with protuberant tongue
t Short stature is disproportionate (upper segment > lower t Delayedbone growth (Bone age <
e4),
segment)(AIIMS chronolo gic al age)Gaus eo.
r Delayed relaxation ofdeep tendon reflexes.

o Note : ln congenital hypothyroidism there is hypotonia. Rarely the muscle may appear hypertrophied
(pseudohypertrophy) with and athletic appearance --+ Kocher-Debre-semeloignl synaromi'gives infont
hercules appearance.

Common differential diagnosis of congenital hypothyroidism(ai 1I)


o Congenital hlpothyroidism due to all causes has same clinical features. Except for central (hypothalamic) hlpothyroiclism,
T rlevel is low and TSII is raisedAl t3) .
o Presence or absence of goitre can differentiate some common causes of congenital hypothyroidism :-
i) Iodine deficiency
r It is the most common cause of congenital hypothyroidism.
t Goitre is present in iodine defeiciency.
ii) Thyroid dysgenesis
I It is due to developmental defect in thyroid gland and is the second most common cause of hlpothyroidism.
t Goitre is absent1l ll).
iil) Thyroid dyshormogenesis
: There is defect in synthesis of thyroid hormones.
t Goitre is presentqt tr)
.

iv) Centralhypothyroidism
r It is due to defective TRH secretion by hlpothalamus.
r There is no goitre and TSII is also reduced along with To6t tt).

v) TSH receptor blocking antibodies


r There is no goitre.

Diagnosis and Treatment


o Since the neonates are asymptomatic birth all newborns are screened for neonatal hypothyroidism. TSH is estimated
at
eitherin cordbloodatthetimeofbirth(preferuedal0s'AIMS03))orbloodobtainedfromheelprickaller2daysofbirth.
r Livothyroxine (10-l5ptg/kg) is the drug of choice for treatment.
Acouired juvenila hypothyroldism
o Acquired hypothyroidism in children can be caused by iodine deficiency (most commoz), thyroiditis, radiation,
histiocltosis, and drugs.
r Girls are affected more commonly than boys.
r Clinical features are :-
a Growth retardation\trMs ol) r Pufly face
r Disroportionately large head and trunk as compared to r Lethargy
t Increased upper and lower body segment ratioarlMse4) r Cold intolerance
t o"" no'
r Mental retardation Delayed puberty{
r Delayed skeletal maturation r Myopathy
r Deiayed dental development r Pseudotumor cerebri
t Multiple epiphyseat break' ctt\1\at) r TSI{ is raised d, T 3 k decreasedqttMs ")

& Mental retardation in acquired hypothyroidism is not as severe as in congential hypothyroidism'


a Severe epiphygeal dysgenesis is characteristic af hypothyroidism.

X-ray findings
o Delayed appearance of ePiPhYsis
o Multiple foci of ossifcation -+ Epiphyseal dysgenesis
o Epiphyseal breakinglAttMs01) of 12th thorasic and l't or 2d lumber vertebrae'
r Skull -s (i) Large fontanelle, (11) Wide sutures, (Iir) Wormian bonesqttMs 01)
Enlarged 6 rounded sella turcica
e Delayed dentition

Treatment
o Levothroxine is the durg choice to treat hypothyroidism in children(Mo"ipd0e) '

Cretinism
o Cretinism is nothing else but the most severe form of hypothyroidism.
o Cretinism is a condition of severely stuntedphysical and mental growth dlue to untreated congenital hlpothyroidism
or from prolonged nutritional deficiency of iodine (acquired hlpothyroidism).
r When cretinism is due to nutritional deficiency of ioidne@Gl0i) it is referred to as endemic cretinism and when due
to congenital hypotlryroidism it is called sporadic cretinism.
c Thus clinical features of cretinism includes b oth features of congenital hypothyroiilism (in sporadic cretinism) and
acquired hypothyroidism (in endemic cretinism).

The given child is sufiering from -

a) Hypopituitarism
b) Cushing syndrome
c) Addison's disease
d) Hypothyroidism

Ans. is 1l' i.e., Hypothyroidism


1; The child is having edematous faces with open mouth and protruded tongue. There is protruded abdomen and
short neck. The ch-ild is having congenital hypothyroidism.

HYPERTHYROIDISM

o Most common cause of hyperthyroidism in children is Grave's disease(AP08)


. It is more common in girls and usually presents in adolescence.
o The earliest feature is emotional disturbances.
o Other features are poor school performance, tremor, increased appetite, exophthalmos, moist skin, tachycardia and
heat intolerance.
o Neonatal hyperthyroidism (congenital hyperthyroidism) is mostly due to transplacental passage of thyroid
stimulating antibodies in mother suffering from Grave's disease. Clinical features are goitre (always present)@rcos;,
hyperkinesis, diarrhea, vomiting, poor weight gain, CHF(DPG 05), arrhlthmias, ophthalmopathy, accelerated skeletal
0e),
maturation(DPc hlT)ertension, hepatosplenomegaly and jaundice.

DIs-O_RDERS OF CALCIUM METABOLISM

Hypocalcernia
r Normal serum calcium level is between 8.9 - L0.2 mgldl. Symptoms of hlpocalcemia develop when calcium level falls
below 7.5 mgldl.
o Hypocalcemia presents with symptoms of tetany i.e., twitching, numbness, muscle cramp, carpopedal spasm and
laryngospasm.
r The characteristic signs are Chvostek sign (facial muscle spasm after tapping of face) and Trousseau's sign (flexion
of wrist and hlperextension of fingers after.inflation of sphygmomanometer cough).
o Treatment of choice for acute symptomatic hypocalcemia (tetany) is intrayenous calcium gluconateuuMs 00, uP 07) or
calcium chloride.

Hypercalcemia
o Hypercalcemia is defined as serum calcium > 11 mg/dl.
I Presenting features are hypertension, pol1uria, constipation, anorexia, depression, hlperreflexia, short QT interval,
band keratopathy, pancreatitls and peptic ulcer. Treatment of symptomatic hlpercalcemia includes furosamide,
glucocorticoids, calcitonin and bisphosphonates.
o Three important hlpercalcemic states are hlperparathyroidism, idiopathic hypercalcemia of infancy and familial
hypocalciuric hypercalcemia.
A) Hyperparathyroidism
r Most common cause in children is benigr parathyroid adenoma.
I There is increased serum calciumat 04), decreased serum phosphateal 04)
, increased alkaline phosphatase\, on)
,
increased serum parathormone, normal serum bicarbonate, and increased urinary calcium.
B) Idiopathic hypercalcem ia of infancy (I{illiams syndrome(Ar e3))
r It is due to abnormal metabolism of vitamin D.
r It is charact erizedby mental retardqtion, *upravalvular aortic stenosisQcl 8e),
etfin facies and motor deficit.
: Vitamin D and calcium intake should be restricted.
t It resalves spontaneously by age of 4 years,
C) Familial (hereditary) hypercalcemic hypocalciuria
r It is an autosomal dominant disorder due to mutati on ir calcium sensing receptorsAilMs oil in renal tubule and
parathyroid gland.
t There ishypercalcemia, hypophosphatemia, increased PTH, decreased urinary calcium (hypocalciuria)(etus
os)
snfl normal alkaline phosphatase.
r There is nephrocalcinosisqllMs 0s).

Hypoparathyroidism
r It can be caused by aplasia of parathyroid gland (Digeorge syndrome), HDR syndrome (hlpothyroidism, deafness,
renomegaly), autoimmune destruction or accidental surgical removal.
r Serum calcium is low with elevated phosphate.
o Musculoskeletal pain and cramps are the eailiest symptoms and progress to numbness, stiffness and tingling of the
hands and ferl(MAaE os).
ldiopathic hYPercalciuria
iru presence of normal blood calcium'
e It is an autosomal dominant disorder charucterizedby hypercalciuria
hematuria, dysuria and abdominal pain'3
r There is nephrolithiasis (renal stones)(Nmsos),recurrent

Diagnosis and treatment


ratio > 0'2'
o (Jrinary calcium excretion > 4 mg/kg/day and urinary calcium to creatinine
(meat) intake\IlMs os) a;11 increqseil hydtation
rTreatment includes oral thiazide , low sodium and ltrotein
of obligate requirmentfot
(water intake)4rMs02), Dietary calcium restriction is not recommendedbecause
osl
$fowfll^nms '

DIsORDERS RFLATEp rO lNsuLlN

Diabetes Mellitus
r DM can be classified into :-
1) Insulin dePend.ent DM (IDDM) of of pancreas'
r It is the most common type of DM in children.It is caused by autoimmune destruction B-cells
like Addison's ilisease'
r There is strong assoicat rarrwlthilLA DR3 ardDR4, and with other autoimmune disorder
Hashimoto's thyroidis and celiac disease'
2) Non-insulin dependent DM (NIDDM)
the prevalence of NIDDM
r It is usually seen in older age; however during last 2 decades there has been a rise in
in children due to increasing obesiry
r It is a polygenic disorder which is caused by multiple factors, genetic and environmental'
3) Maturity oruset DM ofyoung (MODY)
25 years'
r It is an autosamsl dominant disorder seen in at least 3 generations before age of
r It is due to defective insulin secretion'
r It is caused by mutation in glukokinase gene'
are increased fasting blood sugar > 126 mgldl,random blood sugar > 200 mgldi and blood
I criteria for diagnosis of DM
> mgtdl2 hours after glucose toaa of 1'75 g/kgiPctlil to a maximum of 75 gm?Gt0T) ift glucase tolerance
glucose 200

test.

Treatment
r IDDM is treated with insulin whereas NIDDM is treated by diet modification' exercise and oral hlpoglycemic'

afier the onset of disease@Glos) '


o ln IDDM,initial retinal examinalion should be done 5
.years
rlnNIDDM,fundusexaminationshouldbedoneimmediatelyatthetimeofdiagnosis'
,

Nesidioblastosis
(PHHr) or congenitalhyperinsulinism
r Nesidioblastosis, also call edpersistenthyperinsuliruemichypoglycemiaofinfancy
(CI),isthemostcommoncauseofhyperinsulinemichlpoglycemiainneonates(Arl,)'
r It is due to$ - cellshYPertroPhY'
tt)'
o It usually affects infants and childrenbt
t Adults are affected less commonlyul') ' lt)
r There recurrentlrypoglycemic atta*s6t '
are
1r),
octeride and nifedipine'
o Treatment includes iv glucose, glucagon, diazoxideql
@

&

(B

&

&

&

&

&

&

&
QUESTIONS

AMBIGUOUS GENITALIA ambiguous genitalia in a female child- (AI


a) Fetal placental steroid sulfatase deficiency
1" Fernale pseudohernnapfurodite is- (All India Decl 5 Pattern) b) Fetal placental aromatase deficiency
a) Female sexual characterstic with testes c) Congenital adrenal hlperplasia
b) Male sexual charaterstic with ovary . d) WNT-4 gene mutation

c) XY ll, A f0dayoldmalepseudohermaphroditechildwith
di Female sexual characteristic with both testes & 46K{.karyotype presentswith BP of f 10/80 mmfu,
ovary Most likely e nzyme deficiency is - (AI::
Male pseurlohermaphrodism - (All India Dec15 Pauern) a) 21 hydroxylase
a) XX genorype, male external genitalia b) 17 hydroxylase
b) XY genotype, female external genitalia c) ll hydrorylase
c) Testis and ovaryboth present d) 3- beta hydroxylase
d) Male external genetalia and ovary present t2. A fiveyear old boy preeents with precocious pubertr
Cauees *f fernale pseudoherrnaphrodisrr - and a blood pre$sur€ of 130/80 mmHg. Estimation
(All lndia Dec.13 Pattern) of which of the following will help in diagnoois -
a) i7-alpha hydroxylase deficiency @r n,
b) 21-alpha hydroxylase deficiency il t tz hydroxy-progesterone
c) Mlxed gonadal dysgenesis b) t Cortisol
d) All of the above c) t Aldosterone
The most common cause of ambiguous genitalia in a d) t fi deoxycortisol
newborn is - (Ar 02) 13. A S-yr old boy presents with pubic hair develop-
a) 21 hydroxylase deficinece ment. He ie tall and has increased pigmentation
b) 11 p - hydroxylase deficiency of his genitalia atd phallic enlargement. Blood
c) 17 a - hydroxyalse deficiency pressure is 130/90 mm Hg. Measurement of which
d) 3 p - hydroxysteroid deficiency of the following hormones would be mosi likely to
5. Fseudohermaphrodifism in a fernale child io rnost be diagnostic? (AIIMS May t-
cornrnomiy due to - (NEET Dec.12 Pattern) a) Increase L7 beta hydroxyl progesterone
a) 2l-hydroxylase deficiency b) Increase cortisol
b) I 7-hydroxylase deficiency c) Increasealdosterone
c) I l-hydroxylasedeficiency d) Incrbase 11 deoxycortisol
d) 3-hydroxylase deficiency 14. Congenital adrenal hyperplasia is associated with-
.&drenogenifal syndrorne is most eommonly caused a) Hypoglycemia (PGI Dec 2ii

by? (CET June 14 Pattern) b) Hyponatremia


a) 2l ahydrorylase deficiency c) Hypokalemia
b) 17 o hydroxylase deficiency d) Hyperkalemia
c) 3p hydroxTlase deficiency 15. A 8 day old breaet fed baby presents with vomiting,
d) Steroid sulfatase deficiency poor feeding and loose stoold. On examination the
2l-Hydroxytase defrctelacy - false ie 4All India Decl 5 Pattern) heartrate is l90/minute, blood pressure 50/30 mm
a) Most common cause of congenital adrenal Hg respirato ry rate 7 2 breaths I minute and capillarv
hyperlasia refill time of 4 seconds. Investigationc shorry hemo-
b) Autosomal recessive globin level of lsglDl,. Na-l2mEq/L, K-6.8 mEQ/I,
c) Femal pseudo hermaphroditism C1-8f mEq/L, bicanbon*e t5 ntBqlL, Urea 3A mgl
d) Male pseudo hermephroditism dL, cteaf;inine 0.6 mgldl. The most likely diagnosis
6. I 7 tqvdroxylaee deficiency causee-increase produc- ls- (AIIMS Nov 03

ticn rlf - (All India Dec15 Pattern) a) Congenital adrenal hyperplasia


a) Aldosterone b) Acute tubular necrosis
b) Cortisol c) Congenital hlpertrophic pyloric stenosis
c) Androstene , d) Galactosemia
d) A1l of above 15, 2 ryeeko old balryhaving ecro{al pigrncntation along
s. 3 beta h,v<lrcxysteroid dehydrogeaase deficiency with hyponatremia, gyp*.rglyeemia and hyperkalemia
eau$€6 inerease proeluetion of-
(AllIndia Dec15 Pattern) eazym*d*&*wnt- (CET Nov 15 Patterr.
a) DHEA b) Progesterone a) 11 betahydroxylase
c) Deoxycortisol d) Estradiol b) 21 o- hydroxylase
10. Which of the follorrying is the least common cause of c) 3- beta hydroxysteroid dehydrogenase
d) 17- hydroxylase deficiencY measurement blood levels of whicl of the following
willbe helpfirl ? (AIIMS May 0e)
l:. A S-year old girt presents with hypertension and vi- a) 17-hydroxyprogesterone
rilization. There is also finding of hypokalemiawhat b) Renin
is the diagnosis- (AllMS May 07)
c) Cortisol
a) 2l-hydroxylase deficiencY d) Aldosterone
b) 3-p hydroxy steroid deficeicnY
24. 2year otd child presented with diarrhoea a*d fbilure
c) 1i-P hydroxylase deficeincY to thrive; Btrocd cxamination show's - Na = IZ?-K - 6.
d) Conn's disease
EIe is rmost pr&ably *r$ering fro,m -
1 8. Most commo,n enzyme de&eiency leadiag to ehi[d- a) Bartter syndrome (NEET Dec'12 Pattern)
hocd hypertension is - (CET Aug.lj Pattern)
b) 21-o Hydroxylase deficiency
a) 17-Alphahydroxylase c) 11-p Hydroxylase deficiency
b) 2i-Betahydroxylase d) -aHydrorylase deficiency
17
c) 11-Betahydroxylase
25. A 4 week old gs6als childwiti normal genitalia
d) 3-Beta hydroxy steroid dehydrogenase
presents to the emergency department with seYere
19. A l-month old baby present with frequent vomiting dehydration, hyperkalemia and hyponatremia. The
and failure to thrive. There are features of moderate measurement blood levels of which of the following
dehydration. Blood sodium in f 22 mEqll and potas- willbehelptul? (AtlMS May 11)
sium is 6.1 mEqll. The most likely diagnosis is- a) 17 hydroxy progesterone
a) Gitelman syndrome (AI 03)
b) Rennin
b) Bartter syndrome c) Cortisol
c) 21-hydroxylase deficiency d) Aldosterone
d) 11-p hydroxylase deficiencY
Which one of the following drugs is used for fetal
10. Which of the following statements about 2l alpha therapyofcongenitaladrenalhyperplasia? (AI0s)
hydroxylase deficiency is false - @I 08)
a) Fiydrocortisone
a) Most common cause of congenital adrenal
b) Prednisolone
hlperplasia in children c) Fludrocortisone
b) Affected females present with ambiguous genitalia d) Dexamethasone
c) Afected males present with precocious puberty
27. A female has previous childwith congenital a&enal
d) Hypokalemic alkalosis is seen
hlperplasia. In the present Pregnarcy' steroid thera-
21. An I infant is brought to the emergen-
days old male py should be started - (Ar 2K)
cy department with vomiting lethargy, dehydration a) After karyotlping and determination of sex of the
and features of shock Clinical examination reveals
baby
hyperpigrnentation the genital skin and normal
of
b) At the time of delivery
external genitalia. Abdominal examination is unre- c) Before conception
markable. Blood tests revealed sofium of 124 rneqll' d) As soon as pregnancy is diagnosed
potassium of 7 meq/l and hypoglycernia. Which of
W -AEl progesterone level in congenital adrenal
the following is the most likely diagnosis -
hyperplasia in 1 year old child (in ng/dl)-
a) Congenital Adrenal Hlperplasia (CAH) (At 11)
(CET luly 15 Pattern)
b) Adrenal Haemorrhage
c) Acute Gastorenteritis with dehydration a) > 600 b) 1s0-300
d) Hyperaldosteronism c) 300-600 d) < 150
A 7-year-old child presents to the paediatric clinic w. thekaryaty.pe of a patient with androgen Ins€nsitif-
ity Sy.ndrome is - (All India Dec.14 Pattenx)
with amiguous genitalia increasingwith age. On
examfuiation her height, weight and blood pressure a) 46XX b) 46XY
were recorded within normal limits" Labia appeared c) 47 XXY d) 4s x0
bifrd $rith 2 separate perineal openings, phallic The karyoqyge in testicular feminising syndrome is?
length was 2.5 cm and no palpable gonads were a) XX b) XY (CET Nov. 13 Pattern)
noted in the inguinal region. USG shows presence of c) XXY d) xxxY
mullerian structures. The most probable diagnosis 31. Gonads to testes difierentiation - (NEET Dec.12 Pattern)
is- 6t12) a) SRY gene b) WNT-4 gene
a) Classic Salt-Wasting 21 Hydroxylase deficiency c) DAX1 gene d) None
b) Simple virilizing congenital adrenal hyperplasia
32- trNhat stimulates thegonads in male at &weekto
c) Complete Androgen Insensitivity Syndrome
sccr€t te$tosteroae - (NEET Dec.12 Pattern)
d) 5-Alpha Reductase DeficiencY
a) Inhibin from corpus luteum
23. A 3-month old male child with normel genitalia b) GnRH from hypothalamus of baby
presents to the emergency department with severe c) Placental HCG
dehydration, hyperkalemia and hlponttremia. Thc d) All of above
PRECOCIOUS & DELAY PUBERTY a) Short metacarpals in hands
b) Delayed puberty
33. Which of the following is not example of c) Delayed bone age
Pseudopuberty- (CET Nov 15 Pattern) d) Altered upper and lower segment ratio
a) tumor of hlpothalamus 43. 8 yrs. old child presents with lethargy multiple
b) ovarian tumor epiphyseal breaks, wormian bones with growth
c) tumor of adrenal gland retardation and mental retardation Diagnosis is -
d) AII of the above a) Rickets (AIIMS Nov 01)
34. All of the following can cause precocious puberty in b) Hlpothyroidism
males except - (NEET Dec.12 Pattern) c) Scurvy
a) 17 alpha hydroxylase deficiency d) Hlpoparathyroidism
b) 21 alpha hydroxylase deficiency 44. A child is sufiering from dry skin and is mentally
c) 11 p hydroxylase deficiency retarded-diagnosis - (CET Nov 15 Pauern)
d) None of the above a) Vit A deficiency
35. True precocious puberty can occur in all, except - b) Crebral palsy
a) McCune Albright syndrome (AIIMS Nov 93) c) Hlpothyroidism
b) Craniopharyngioma d) All above
c) Congenital adrenal hyperplasia 45. Which of the following is true regarding cretinism-
d) Hlpothalamichamartoma a) Short limbs compared to trunk (Ar 01)
36. All of following may tre causes of precocious puberty b) Proportionate shortening
in girls except - (ArrMS es) c) Short limbs and short stature
a) Hlpothalamichamartoma d) Short limbs and long stature
b) Mc Cuone Albright syndrome 46. A oneyear old child preentswith short stature,
c) Granulosa cell tumor of human ovary lethargy, & constipation. Clinical examinaton shows
d) Congential 21-o hydroxylase deficiency a palpable goiter. Laboratory investigations revealed
37. Hypergonadotropic hlpogonadism - a low T4 and elevated TSH which of the following is
a) Decrease FSH and LH @ET luly 15 Pattern) the most likely diagnosis. (At 11)
b) Decrease FSH and increase LH a) ThyroidDyshormonogenesis
c) Increase FSH increase LH b) Thyroid Dysgenesis
d) Increase FSH decrease LH c) Central Hypothyroidism
38. Delayed puberty in female- (All tndia Dec15 Pattern) d) TSH Receptor Blocking Antibody
a) No breast budding in l0 years 47. Short child with low T4 and raised TSH and swelling
b) Menarche > 16 year of pituitary, what is the diangosis? (AIIMS Nov 11)
c) Menarche > 1 year of brean budding a) Primary hlpothyrodism
d) FSH <20inl6year b) Pituitary tumor
39. Most common cause of delayed puberty in males is - c) TSH Secreting pituitary adenoma
a) Kallaman slmdrome b) Klienfelter syndrome d) TSH resistance
c) Constitutional d) Prader-willi syndrome 48. In neonatal screening programme for detection of
40. Hypergonadotropic hypogonadism is seen in all congenital hypothyroidism, the ideal place and time
except- (All India Decl5 Pattern) to collect the blood sample for TSH estimation is -
a) Turner sl.ndrome a) Cord blood at time of birth @I 0s, AIIMS May 03)

b) Down syndrome b) Heal pad blood at the time of birth


c) klinefeltersyndrome c) Heal pad blood on 4 day of birth
d) Swyert syndrome d) Peripheral venous blood on 28 day

41. A child with decreased levels of LH, FSH and Testos- 49. The features ofneonatal hyperthyroidism include all
terone presents with delayed puberty. Which of the except - (CET Nolr. 11 Pattern)

following is the most likely diagnosis - (Ati2) a) Triangular facies with craniosynostosis
a) Klinfelter'ssyndrome b) Congestive cardiac failure
b) Kallmant syndrome c) Advanced osseous maturation
c) Androgen Insensitive syndrome d) Goiter is rare
d) Testicularlnfection
HYPOPITUITARISM & HYPERPITUITARISM
HYPOTHYROI DI SM & HYPERTHYROIDISM
50. A 9 year old boy presents with growth retardation
42. NOT a feature of hypothyroidism is (CET lune 14 and propensity to hlpoglycemia. Physical examina-
Pattern)
tion reveals short stature, micropenis, increased fat
and high-pitched voice. The skeletal survey reveals MISCELLANEOUS
bone age of 5 years. Which of the following is most
appropriate diagnosis - 6ltMS Nov 01) 60. A l0-month old baby previously norrnal, suddenly
becomes distressed in his crib' The external appear-
a) MalabsorPtion
ance of genitalia was normal, except hyperpigrnenta-
b) Growth hormone deficiencY
c) Adrenal tumour tion. Blood glucose showed a level of 30 mgTo' What
is the most Probable diagnosis?
d) Thyroxin def,ciencY
a) 21 hydfoxylase deficiency (At 11, AttMS May 11' 10' 09)
51. Characteristic features of growth hormone deficien- b) Hyperinsulinism
cy include all of the following except -
c) Familial glucocorticoid deficiency
ai Short stature since birth (AlllndiaDec'l4Pattern)
d) Cushingt sYndrome
b) SymPtomatic hlPoglYcemia
c) Delayed tooth eruPtion 61. Which one of the following is the earliest manifesta-
d) Sexual infantilism
tion of Cushing's sYndrome-
(AltMS MaY 0s)
- @llMS Malt
a) Loss ofdiurnalvariation
'Weak giants'are produced by
01)
52, b) Increased ACTH
a) Thyroid adenomas c) Increased Plasma cortisol
b) Thyroid carcinomas d) Increased urinary metabolites of cortisol
c) ParathYroidadenomas
d) PituitarY adenomas 62. ACTH secretion is highest during -
a) Noon (A.ll lrulia Dec.14 Pattern)
t3. Laron dwarfism is caused bY -
b) Evening
a) deficiency of GH (cET Nov 15 Pattern)
c) Morning
b) GH recePtor defect
d) Night
c) deficiencYofthYroxin
d) ThYroxin recePtor defect 63. A old boy has a fracture of fernur" Biochem-
10 year
ical evaluation revealed Hb f 1'5 gmldl and ESR 18
54. Which is false in Congenital Hlpopituitarism?
a) Growth hormone level < 7 ng/ml mm Ist hr. Serum calcium l2'8rngidl, serurn phos-
phorus 2.3 mgldl,alkaline phosphatase 28 K'd units
b) HlPoglYcemia
c) BabY small at birth andblood utea32mg/dl.' Which of the following is
the most probable diagnosis in his case- ,A104)
d) DelaYed PubertY
a) Nutritional rickets
55. Short stature, secondary to growth hormone defi-
b) Renal rickets
ciency is associated with - @ILMS Dec 95)
c) HlPerParathroidism
a) Normal bodY ProPortion d) Skeletal dYsPlasia
b) Low birth weight
c) Normal ePiPhYseal develoPment 54. A baby presents with tetany' First thing to be done is
d) Height age equal to skeletal age administration of - (AltMS lune 2000)

for cranio- a) DiazePam b) Vitamin D


56. A. 7 yr old boy underwent neurosurgery
pharyngioma following which pituitary functions c) Calcium gluconate d) Calcitonin3
*"t" tot't. Which of the following hormone should 65. The immediate treatment of 10 kg weight infants
be replaced first?
tAt t.1) presented with tetany - UP 07)

a) Hydrocortisone b) Thyroxine a) IV DiazePam


c) Growth hormone d) Prolactin b) IV calcium gluconate with cardiac monitoring
c) IV slowPhenobarbitone
d) Wait and watch
UNDESCENDED TESTES
66. True about nephrogenic diabetes insipidus in children
A boy with undescended testis, your concern to
ask
57. secretion
for oPeration is due to ' ADH (PGI Dec 2000)
@GI oi) a) Decreased
a) Cosmetic reasonsb) Infertility b) Receptor insensitivity in kidney tubules
c) Risk of mallgnancYd) ImPotence c) Decreased ADH
d) Commonly associated with WiLM's tumor
58. Unilateral undescended testes is ideally operated
around -
(AI04) 67. Oral glucose tolerance test in children is done with-
a) 2 months of ageb) 6 months of age a) 1.5 gm/kg glucose @Gt dec 07)

c) 12 months of aged) 24 months of age b) 1.75 gm/kg glucose


at c) 2 gmlkgglucose
59. Orchidopex is done in cases ofundescended testes
d) 2.5 gm/kg glucose
the age o1- 6ttMS NoY 06)

1-2yrs e) 75 gmas an adult


a) Neonateb)
c) 5 yrsd) Puberty
68. In childrren with type I DM when is ophthalmologic a) Radio Iodine (ArrMS 92)
evaluatioa irdicated - @Gt May u) b) Lugols Iodine
a) At the time of diagnosis b) After 1 year c) Carbimazole
c) After 2years d) After 5 years d) Surgery
e) After 10 years 26. Cornmonest feature of hypothyroidism in children
69. ?ercentage ofdose givea as basal insulia in bolus is-
basal regi:nen in children is? (CET Nov. 1j pattern) a) Cataract (Ar 8e)

a) 0-25o/o b) 25-50o/o b) Recurrent seizures


c) 50-75o/o d) None of these c) Cold extremities
d) Laryngospasms
70. A mother is exposed to Diethyl stilbesterol during
pregnancy" All the following features maybe seen in
77. Infantwiti no social smile, no eyebrows, protruded
tongue. Diagnosis - (rN ee)
the child after birth except - (AIIMS May 02)
a) Clear cell carcinoma a) Cretinism
b) Microglandular hyperplasia b) Down's syndrome

c) Malformation of the vagina and uterus c) Mucopolysaccharidosis

d) Vaginal adenosis d) Rickets


78. A 6 month old infant is brought with a history of
71. A 6-month old boyweighing 3.2 kgpresents wiilr re-
constipation and excessive sleepiness. On exmina-
current vomiting and polyuria. Investigations show
tion, he is lethargic, has periorbital puffines, large
blood area 61) mg/dl creatiaine 0.7 mg/dl" calcium
to{gue and umbilical hernia. The investigation
f2.8 mg/dl. phosphate 3 mg/dl, pH 7.45, bicarbon-
which will help to diagnose this condition is-
ate 25 mFq/L and PTH 140 pg ml (normal <60 pg/ml).
(UPSC 2oo2)
Daily urinary calcium excretion is reduced. Illtra-
sound abdomen show bilateral nephrocalcinosis.
a) T4TSHassay b) Karyoptyping
The most likely diagnosis is - (AIIMS Nov 0s)
c) Rectal mucosal biopsy d) Knee X ray
a) Bartter slmdrome 79. Features of hnrothyroidism in infancy include the
b) Mutation of the calcium sensing receptor following except - (UPSC 06)

c) Pseudo-pseudohlpoparathpoidism a) Premature closure of posterior fontanelle


d) Parathyroid adenoma b) Coarse facies
72" The followiag is trae about Nesidioblastosis excqrt?
c) Umbilical hernia
a) Presents with hypoglycemic attacks (Ar 11)
d) Constipation
b) More common in adults than in children Treatment for childhood hypothyroidism is with -
c) Histopathology shows hyperplasia of islet cells a) T4 b) T3 (Manipal0S)

d) Diazoxide is used for treatment c) Levothyroxine d) TSH


81. 'Most common cause of thyrotoxicosis in childhood?
a) toxic nodular goitre (APPG o8)
QUESTIONS OF VARIOUS OTHER
b) toxic adenoma
EXAM!NATIONS c) graves disease
d) thryotoxicosis factitia
73. Adrenal hlperplasia due to 21 hydroxylase deficien-
cy is treated with low dose - (Kerala 04) 82. The most common presentation of hypopara-thy-
a) Androgen b) Estrogen roidism beyond the neonatal period is- (MAHA0s)
c) Cortisone d) Anti-androgen a) Syncope secondary to prolonged QT intervals
b) Tingling of extremities
74- A newborn baby p.resents with shock, hyperkalernia
and hypoglycemia. What is the most likely diagno-
c) Seizure
d) Bronchospasm
srs-
a) Septicemia (uPSC-r oe) 83' Association of sexual precocity, multiple cystic bone
b) Inborn error of metabolism lesions and endocrinopathies are seen in - (Comed 08)
c) Diabetes mellitus a) McCune-Albright's syndrome
d) Congenital adrenal hyperplasia b) Granulosa cell tumor
c) Androblastoma
Treatment of chnice in childhood thvrotoxicosis -
d) Hepatoblastoma

ltt
ANSWERS
AMBIGUOUS GENITALIA

ts"rt*. i^s "W &.*., b6*1* s*wxa| eh*raeter;st* with *v*ry { It e f: N*ts*w t *tk l e p- 2 3 6*}
o Genotype is XX. Internal gonad is ovary but external genitalia are virilized (male differentiation).
&xe. i& '-m* i.e., W g*ztts€yp*.,{*rxx*,X* eNterwal genit*lia
c Genotype is XY. Internal genitalia are testis but external genitalia are of female.

-1. &ws" is'W i,*.,22-xlptzalay&resxy|a*e &e&c|*ytct, {llef Nel,s*x l&,hfe V. t36$}


o 2 1 -hydroxylase deficiency causes female pseudohermaphroditism. Other two options cause male pseudohermaphroditism.
il. Ans. is '* i.e.,2lhydrorylase deficiency lRef,Nelson t9n/e p. 23601
o Congenital adrenal hlperplasia due to classic 21-Hydroxylase deficiency is the most common cause of ambignr;Lrs
genitalia in the newborn. Virilization is apparent at birth in the female and within the {irst 2-3 vearcof life in ti.re nririe.

:). A&s. is '&' i"e",2$-bydv*xyEase defis;iency fRef: Nelson lgth/e p, 236i))


e Classic 2l-hydroxylase deficiency is the most common cause of Ambiguous genitalia in a new born.
o Virilization is usually apparent at birth in females i.e Pseudohermaphroditism of afemale child.
6. &rss" is '# i"e", 7,1 a 3:ry&r*xyLas* &*{at;i*ncy {Ref: Neis*w l{ltt'lt e*d*critz*{*gy, Xss*w{i*{s *f
/}. 23d2; Wi$iam's
P*edi*tri* 4'hle p. 24e,24t;
r Adrenogenital syndrome is nothing else but congenital adrenal hlperplasia.
o Most common cause of congenital adrenal hlperplasia (adrenogenital syndrome) is deficiency of 2l-hydroxylase.
&ns, is '& i.*-,Wa** ps*r*&* Werr*x*ptxr*ditisna i\e{: N*ls** tr&,hi* clc. SSS}
o There is female pseudohermaphroditism (not male pseudohermaphroditism) in 2r-hydroxylase deflciency. other
options are correct.

8. ,4.ms" is '* i"e., &t&q*ster*rye. $tef: R.e*d betaw]


o ln 17 cr-hydroxylase (77 u OH) deficiency, pregnolone cannot be used to synthesize glucocorticoids and sex steroids.
r All pregnenolone is used for synthesis ofmineralocorticoids/aldosterone (see flow chart in the text ofchapter).

{}" &sxs" is'* i"*,",\}Wf,,A We{: {}fu*i Thle p. 49}}


o In 3 B-HSD deficiency, all pregnenolone is diverted towards DHEA (see flow chart in text of the chapter).

to. Ans is'l i,e.,Fetalplacental steroid sulfatase deficiency lRef williams 2vd/e p. BII
t Placental steroid sufatase deficiency is an X-linked disorder (affects only males) that does not lead to ambiguous
genitalia in afemale child.
c 'Placental steroid suJlatase deficiency is a rare X-linked disorder and all
affectedfetuses are males' -William\ 22,d/gl
o Congenital adrenal hlperplasia is the most common cause of ambiguous genitalia in female child.
r Fetal placental aromatase deficiency and WNT-4 gene mutation cause ambiguous genitalia in female child.
tl. Ans. is 'V i.e,,17 ahydroxylase lRef, Nelson t&h/e p. 2j66|
r This neonate has : -
i)Hypertension (normal BP in a neonate = 7Sl5O)
ii)
Male pseudohermaphroditism.
o This is seen in 17-alpha hydroxylase deficiency.

Adrenal enzyme deficiency

17-a hydroxylase 11-B hydroxylase 21 hydroxylase and j-B-HSD


. Salt retention & hypertension . Salt retention & hypertension . Salt losing
. Feminization of male (Male . Virilization of female (Female . Virilization of female (Female
Pseudohemaphroditism) pseudohermaphroditism) pseudohermaphroditi sm)
. Precocious puberty in male . Precocious puberty in male
12. Ans. is 'd' i.e., t tt deoxycortisol lRef: Nelson 18th/e p. 2j62]
r At the age of 5-6 years, BP in excess of 115175 mm Hg is considered as hlpertesion (This boy has BP 130/80).
r So, the boy in this question has : -
i) Precocious puberty
ii) Hlpertension
o Precocious puberty in male, with salt retention and hlpertension, occurs due to deficiency of 11-B Hydroxylase which
catalyzes the conversion of 11 deoxycortisol to cortisol.

13. Ans is ld' i.e., Increase 1l deoxycortisol [Rel Nelson 18th/e p. 2362)
c Presence of Precocious puberty in a male child and lrypertension (BP of 130/80 in a 5 year old child is considered
hypertension) suggests a iliagnosis of congenital adrenal Hyperplasia due to 11-$ Hydroxylase deficiency.
r levels and excretion of 11 deoxycortisol is increased.

t4- Ans. is'a'i.e., Hlpoglycemia,'b" i.e., Hyponatremia, t'i.e., Hypokalamia, t'i.e., Hyperkalemb[Ref. Nelson
18'h/e p. 2j621
r Also see previous explanations : -
r Depending into type of deficiency, congenital adrenal hlperplasia (CAH) may be divided into : -
1. Salt retaining (17 -a hydrorylase or 11-B hydroxylase deficiency)
r Due to excessive mineralocorticoids there is -+ 1) Hypernatremia r11) In cr e as e d chlor i de

l1) Hypokalemia iv) Hypertension


2. Salt losing (2f hydroxylase or 3-B HSD) deficiency
r Due to mineralocorticoid deficiency threre is -+
1) Ilyponatremia iii) Hyperkalemia
11) Low chloride iv) Low BP dehydration
r In all tlpes of CAH, there is deficiency of glucocorticoid that results nhypoglycemia.

15. Ans. is 'a' i.e., Congenital adrenal hyperplasia {Ref: Nelson 18th/e p. 2j621
r Clinicalprofileofbabygiveninquestionmatchonlywithcongenitaladrealhlperplasia(mostprobably2lhydroxylase
deficiency).
Clinical manifestations of 2lhydroxylase deficiency : Sign and symptoms of cortisol and aldosterone deficiency are
produced.
c Progressive weight loss o Dehydration o Hypoglycemia
r Anorexia r Weakness r Hlponatremia
r Vomiting r Hypotension r Hyperkalemia

Lab findings : Tlpical laboratory finding associated with cortisol and aldosterone deficiency including hlponatremia,
hlperkalemia, acidosis and hlpoglycemia.
-+ Point to the remember is that the other form of CAH like iTcr hydrorylase and 1-B hydroxylase produce hypertension.

1S. Asas" is'"n* i.e., ?1 u- hydroxylase > t'i.e., 3- treta hydrorysteroitl dehydrogenase lftef Nelson 18d'/€ ch. 577;
{}kr! 7"/e p. 4921
r Hlponatremia, hlperkalemia and hlpoglycemia is seen in 21 o-hydroxylase and 3 p-hydroxysteroid dehydrogenase
(3 B-HSD) -+ Both have similar manifestations.
r But best answer is 2l-hldroxylase deficiency, as it is the most common cause of CAH.

17. Ans. is t'i.e., 1l-B hydnrxylase deficiency [Re/ Seeprevious exl,lanfltionsT


r Hypertension, hlpokalemia and virilization suggest the diagnosis of t t-B hydroxylase deficiency.
i.{t, Ans. is'c' i.e., tr 1-3eta hyd,roxylase [Xr/ hklsnn 18il' / e Chapter 445)
r Salt retention and hlpertension is seen in 17-o hydroxylase deficiency and 1 1-p hydroxylase deficiency.
o 11-p hydroxylase deficiency is more common.
c 17 -a hydroxylase deficiency is an uncommon form of congenital adrenal hlperplasia.

19. Ans. is t' i.e., 2f hydroxylase deficien cy fRef: Nelsoz 1 8* /e p. 2362)


t BloodparametersindicateHyponatremiaandHyperkalemiainthispatient,Allconditionsotherthan2lhyd.roxylase
deficiency present with Hypokalemia.'
there is hlperkalemia (not hypokalemia)'
. In congenital adrenal hlperplasia due to 21 alpha hydroxylase deficiency,
o All other options have been explained'

21. Ans is
.a, i.e., congenital Adrenal Hyperplasia (cAH) fRef: Nelson 18th/e p. 23621
hypoglycemia and shock in this eight year old
o presence of wmiting, lethargy, dehydration, hyponatremia, hype*alemia,
male infant suggest a diagnosis of neonatal adrenal insuficiency/crisis'
t congenitalAd.renalHyperplasiaisthemostprevalen{rourriTodr"nalinsufi.ciencyinneonates(typicallyT-14days
joriaaiu"ryl and thi ilngle best answer amongst the options provided.
22. Ans. is
.b- adrenal hyperplasia fRef: Harrison 18th/e p'i053, i054; william's
i.e., simple virilizing congenital
p'240' 2411
Endocrinology, Essenlials ofPaediatrics 4th (Elsevier)
o Among the given options only 2lhydroxylase is the common cause of ambiguous genitalia' hypotension'
r Now the confusion'urir.s, be.au.e it-nyaroxytase deficiency tlpically presents with salt wasting and
whereas the child in question is having normal BP'
loss'
r Actually, some forms (simple virilizing form) of 2l-hydroxylase deficiency have normal BP and no salt
23. Ans. is'd i.e., 17 hydroxyprogesterone lRef : Harrison lVh/e p' 2345-23461
2L - hydroxylase deficiency'
o This is a case of congenital adrenal hlperplasia due to deficie rlcy of
which is then shunted into
o Because of the enzyme block ,there is increasedformation of iz hyilroxyprogesterone,
-
the production of testosterone.
o Levels ofcortisol and aldosterone are reduced'
{r 7h/e p, a59|
24. Ano, is 'W 1,e.,2|-u*lydraxyluile d*fisi*n*y t\e!: {},P, Gh*i 8,h/* p. 526-527
deficiency, causing hyponatremia and hyperkalemia'
r Na* is lZ2 (hlponatremia). That means, there is salt losing enzyme
hyperkaiemia. 17cr-hydroxylase and 1 1-B hydroxylase deficiency
o 21 hydroxylase deflciency causes hyponatremia and

. ;T[]H:,,ffiTffi.:::*:ilillir*",, K* and cr causing severe hypokaremia and hvponatremia

25. Ans is'd' i.e., AldosteronelRef: Horrison 77th/e p' 2345-23461


suggests aldosterone deficiency'
o The serum electrolyte abnormalities along with normal genitalia
Rule out 21 hYdroxYlase deficiencY
r Dont get .o.rfu.ed *ith previous explanation. In previous explanation the child was male'
o ln2l hydroxylase deficiency: .-,--
-^-,+^1i^ r^*
will have normal-1 genitalia for some years o{tcr
^^*6 -,oo," after
i) There will be precocious puberty in male child. So male child
birth (as in previous question)' . -.--r-r -,,..:r:-^+:^- ^F
after birth (due to prenatal virilization o1
ii) There will be virilization of female child, which occurs immediately
geneticallY female infants)
,,o.*al (not virilized). So, 21 hydroxylase deficiency is ruled out'
o In this child, female genitalia a..

Salt Losing (Salt Wasting)


[Dehydration, hlponatremia, hlperkalemia]

o Virilization of female child r Normal genitalia in female child


or or
o Normal genitalia of Male child o Normal genitalia of male child
(upto some year)
o Precocious pubertY in Male child +
i) Aldosterone deficiencY due to
aldosterone synthase deficiency.
i) zt hyoroxyltse deficiency (most common)
ii) 3 beta-HSD deficiencY

as there is salt wasting with normal gen-


o So, in previous explantion also, we cannot rule out aldosterone deficiency
italia in a male child'
as this is the most common type of salt wasting
r But, In that question we considered it as 21 hydroxylase deficiency
congenital adrenal sYndrome.
means primary aldosterone deficiency (due to al-
r Here, I would like to mention that, here, aldosterone deficiency
dosterone slnthase deficiencY)
Ans. is.d, i.e., Hypokalemic alkatosis is seen |Ref: Previous explanations]
there is hlperkalemia (not hypokalemia)'
r In congenital adrenal hyperplasia due to 21 alpha hydroxylase deficiency,
o AII other options have been explained'

i.e., Congenital Adrenal Hyperplasia (CAH) fRef: Nelson 18th/e p' 23621
Ans is'a'
hypoglycemia and shock in this eight year old
o Presence of vomiting, lethargy, dehydration, hyponatremia, hype*ale1nia'
maleinfantsuggestadiagnosisofneonataladrenalinsuficiency/crisis.
o congenitalAd.renalHyperplasiaisthemostpreuol"rirourriTailrenalinsuficiencyinneonates(typicallyT-14days
poridrliurryl ana *i in[te best q.nswer amongst the options provided.
Harrison 18th/e p'3053, i054; william's
Ans. is
.b- i.e., simple virilizing congenital adrenal hyperplasia fRef:
p'240' 2411
Endocrinology, Esseniials ofPaediatrics 4th (Elsevier)
o Among the given options only 2lhydroxylase is the common cause of ambiguous genitalia'
tlpicalty presents with salt wasting and hlpotension'
o Now the confusion arises, becaus. it-hydro*ylase deficiency
whereas the child in question is having normal BP'
deficiency have normal BP and no salt loss'
o Actually, some forms (simple virilizing form) of 2l-hydroxylase

Ans. is'd i.e., f 7 hydroxyprogesterone lRef : Harrison 17h/e p' 2345-23461


o This is a case ofcongenital adrenal hyperplasia due to deficiency of2l
- hydroxylase deficiency'
- which is lhen shunted into
o Because of the enzyme block ,there is increased formation of 17 hydroxyprogesterone,
the production of testosterone.
r Levels ofcortisol and aldosterone are reduced'
Ano, is
rv i.*.,21-aTsy&raxyaa.s* &&&r:Let:r*y tfleJ: {3"ff Gfua'i &',h/* trt' 526-527 {' 7h/e p" 469l
causing hlponatremia and hlperkalemia'
o Na* is 122 (hlponatremia). That means, there is salt losing en4nne deficiency,
deficiency
o 21 hydrorylase deficiency causes hyponatremia and hlperkaiemia. 17cr-hydroxylase and 11-B hydroxylase

causing severe hlpokaremia and hvponatremia.


. ;::[iH[::Tffi,T:H:i3iI]Xr*u., K* and cr
Ans is i.e., Aldosteron e lRef: Harrison 17th/e p' 2345'2346)
'd'
suggests aldosterone deficiency'
o The serum electrolyte abnormalities alongwith normal genitalia
Rule out 21 hYdroxYlase deficiencY
o Dont g.t.o.rfrs.d *ith previous explanation. In previous explanation the child was male'
o ln2l hydroxylase deficiency: -,r -^-:r^r:^ r^- ^^*- -,on.o
for some years o{+o,
after
i) There will be precocious puberty in male child. So male child will have normal genitalia
birth (as in Previous question)'
(due to prenatal virilization of
ii) There will be virilization of female child, which occurs immediately after birth
geneticallY female infants)
nor-ul (not virilized). So, 21 hydroxylase deficiency is ruled out'
o In this child, femJ genitalia u..

Salt Losing (Salt Wasting)


fDehydration, hyponatremia, hyperkalemia]

o Virilization of female child o Normal genitalia in female child


or or
e Normal genitalia of Male child o Normal genitalia of male child
(upto some Year)
o Precocious PubertY in Male child +
i) Aldosterone deficiencY due to
+ aldosterone synthase deficiency.
D 21 hydroxylase deficiency (most common)
ii) 3 beta-HSD deficiencY

as there is salt wasting with normal gen-


o So, in previous explantion also, we cannot rule out aldosterone deficiency
italia in a male child.
is the most common tlpe of salt wasting
o But, In that question we considered it as 21 hydroxylase deflciency as this
congenital adrenal sYndrome,
primary aldosterone deficiency (due to al-
o Here, I would like to mention that, here, aldosterone deficiency means
dosterone synthase defi ciencY)
r Aldosterone deficiency (along with cortisol deficiency) also occurs in 21-hydroxylase deficiency (see previous ex-
pianations)
?.6. An$. is tf i-e", Dexamefhasaae{Ref Nelson l*/e p.2j5a}
'Recommendations for pregnancy at risk consists of adinistration of dexamethasome a steroid that readity crosses
the plecenta'

27. Ans. is .d i-e", As snon as pregnancy is diagnooed {Ref Netso* l#/e p. %Gal
. Recommendations for pregnancies ar risk of CAH consists of administration of dexamethasone (20 mg/kg). It started
by 6 weeks of gestation, this ameliorates the virilization of the external genitilia in affected females chrionic villus
sampling is then performed to determine the sex and genotype of the fetus; therapy is continued only if the fetus is
an all'ected female.
r As ;rregnancy is diagnosed (by urine test) at around 6-8 weeks, so it should be the answer.
28- Ans. is 1a'i.e., > 6O0 (best probable answer) {Rel Medilineptus)
r Normal value of l7-hydroxlprogesterone in an infant/children > 1 months old is less than 100 ng/dl.
r In infant with cAH, 17-hydroxyprogesterone levels range from 2000-40000 ng/dl.

29_ Ans. is'-n- i.e., 46){I{ {Ref Harrison |flle p. 2344j


Individual with hndrogen insensitivity syndrome' are XY individuals (male karyotype) with a female phenotype.
Mutation in the androgen receptor causes resistance to androgen action and the androgen insensitivity syndrome
results.

30. .A.ns. is'H i.e", XY {Ref: tr:{a*ison' 17e/e p. 2341)


o Androgen insensitivity sprdrome (AIS), formerly known as testicular feminization, is an X-linked recessive condition
resulting in a failure of normal masculinization of the extern'al genitalia in chromosomally male individuals.
c Individual with 'androgen insensitivity syndrome' are XY individuals (male karyotype) with a
female phenotype.
31." Ans" ie 'd i"e., SRY gene {R,eJ: Nelson Chapter 5891
r XX chromosome with genetic factor such as DAXL and signalling molecule WNT-4 are necessary for development
46
of ovary.
o Y chromosome contains SRY gene which differentiates gonads to testes.
32. Ans. is t'i.e", Flacental HCG [Rel Nelson 1*/eChap.589)
I During embryogenesis, In male embryo, at 6-7 week of gestatin, secretion of first anti-mullerian hormone (also k/o
mullerian inhibiting substance) causes regression of mullerian duct.
r Then at 8-12 week, under influence of Placental HCG, testes started producing testosterone.

PRICOCIOUS & DELAYED PUBERTY

33" Ans. is'a'i.e., Tirrnor nf hypothalamus [ReJ: Nelsan 1*/e ch. 56j]
r Peripheral or pseud-precocious puberty is caused by increased secretion of sex steroids from adrenal or gonads
(testis or ovary).
r Central lesions (hlpothalamic) cause central or true-precocious puberty.

34. Ans" is'a'i.e., 17 a hydrorylue{ReJ Ne/soa l$h/e p. 2356)


rMaleprecociouspubertyiscausedbyll-Bhydroxylase, and2l hydroxylasedeficiency(notbylT-crhydroxylase
deficiency).

35. An$. is 'f


i.e.,McCune Albrigbt syndrome; t'i.e., Congenital adrenal hyperplasia {Ref Ghai */e p. Sj2
table {17"32) & fllep.4e9-s001
t McCune Albright syndrome and congenital adrenal hyperplasia cause peripheral or pseudo precocious puberty.

36" Ans, is ?' i-e., Congenital 2f-hydroxylase lRef: O.P. Ghai */e p" 532 tabl& (17.32) & 7/e p. 49* Nelson l*te p"
2j621
r Congenital 2 t hydroxylase deficiency causes precocious puberty in male due to excess of androgens.
r In female, it results in virilization
37- Ans" is t" i.a, [acrease F$Il increasc LH
o Hypergonadotropic hypogonadism is characterized by high FSH and LH.
e It is also called as primary hypogonadism or peripheral hypogonadism.
r The defect is at gonads level, so that sex hormones are not secreted and there is no feetl_back control on FSH ancl
t,H
-+ Increased secretion of FSH anil LH.

3S. Ans. is'H i,e, Mcn*rch e > 16 yexrr lR$; Gkai Z* le p- Wtl
o In girls, delayed puberty is defined as failure ofbreast budding by 13 years or absence ofmenarche by l1year ar
lack ofsecondary sexual characters by 17 years.

39. furs" i.c ti i.e-, C nstitution*llRf; *"P. Gh*i *fcp 535 tabtc (IZ.w) & Ple p. So3; AsWs &fc p. zg&j|
. Most common cause of delayed puberty is constitutional.
&. Ans, is'H i.e., Wwrln syndronre
r Turner syndrome, Klinefelter slmdrome and Swyert syndrome cause hypergonadotropic hypogonadism.

4I. Ans. is'o" i.e., I(alhnan's syndrome ln$. Wwr*,an I N le p. 30 l,fi


r Among the given options, only kallman's syndrome is cause of hlpogonadotropic
1JrSH, JtH; delayed puberty.
Klinfelter's syndrome and testicular infection (orchitis) cause hlpergonadotropic
ltfsfi, tt-Ht a.try.a puU.rry.
Androgen insensitivity syndrome is not a cause of delayed puberry
KallJnan's syndrgrne is an X-Linked disorder characterized by deficiency of GnRH with a resultant
decrease in t'SH
and LH levels producing an isolated Hypogonadotrophic Hypogonadkrr. xallmans syndrome may be seen in
both
men and women but is more common in men. It is typically also associated with agenesis or hypoptasia
of the
olfuctory bulb producing anosmia or hyposmiaa

HYPOTHYROI DISM & HYPERTHYROIDISM

&'" Ans- is ud i-e,fuwrtwreft*earparllwhaw&s1wef: A,P. Gkai */e p StGsIf & frlep. 482-48.j; Nelsar I*/e p 2jZ2\
Hypothyroidisrn causes -
i) Delayedbone growth (Bone age markedly delayed)
ii) Delayedpuberty &
iii) Alteration in upper and lower segment ratio (upper segment > lower segment)

43- Ans" is 'H i.e., Hypothyroidism [Ref a.P. Ghoi */e p. 51s & 7/e p. 481; Nekon I*le p" 2322, z3zs]
e Multiple epiphyseal breaks, wormian bones with lethargy, growth retardation and mental retardation
suggest the
diagnosis of hypothyroidism.
44" &"ns- isY i.c.,@g@.rw1d;i*w
a Hypothyroidism causes mental reterdetion along with skin dryness and olher features.

45. Ans. is'd i.e., Short limbs conpared to trunk; t'i.e", Short timbs and short stafirre
lR$ Netson l*/e p.
23i11
"The child's growth is stunted and the extremeties are short. The hands are broad and the fingers
are short,,.
"The child appears short and stocky'l

M- Ans is ? i.e., Thyroid Dyshonnonogenesis [Ref E*sentials af pediatric gle p. tzg]


r This child has :-
i) Thyroid swelling
ii) Low Tu & High TSH
iii) Abnormal weight gain and poor activities
r All these suggests the diagnosis of congenital hypothyroidism due to thyroid disorder (In hypothalamic diseases,
TSH levels will be low).
r The most common cause of non - endemic congenital hypothpoidism is thyroid dysgenesis.
1) In endemic regions:- Most common cause of congenital hypothyroidism is iodine deficiency in intrauterine alil
neonatal period (world wide).
ii) In non-endemic regions:- Most common cause of congenital hlpothyroidism is thyroid dysgenesis.
o However, in thyroid dysgenesis, thyroid gland does not enlarge to produce palpable goitre. So, this optiori is e-r:r.iui,li:d.
r Dyshormogensis is an uncommon cause of congenitai goitrous hypothyroiilism and account only about 10-15% of
cases of congenital hypothyroidism. It results from a deficiency of one or more enzymes (most commonly, thyroid
peroxidase) involved in thyroid hormone synthesis or secretion.

Congenital Hypothyroidism

Goitrous (Palpable Goitre Present) Non Goitrous (No palpable Goitre)


Thyroid Dyshormonogenesis , Thyroid Dysgenesis
Iodine Deficiency . Thyroid Receptor blocking Antibodies
(Thyrotropin Receptor Antib odies )
Placental passage of Antithyroid Drugs . Central hypothyroidism

Thyroid Dysgenesis No Low Elevated

,,'T$ :''Gti:': .rrf."L3]|l!1"11: 'n..:ii :ir,:j:., jjr].gt€V;i€dii:.,


-'
11,,:;,:;,il!!rr::{G.iiti,J
TSH Receptor Blocking Antibody Low Elevated
,. ..::::::::+trr:rl1::ii . :.. t!::::.:r r, r.!:rr:::i:r:r .ri::.;'.,.1

l{bntialHti$lhi,@'.j,r:l.i:r:r: .:: ::,,.r;;j,;.ld$if: .,.,=.,,11;,1;'Nl4rtilll0w..,

o So, amongst the given options, best answer is Dyshormogenesis.


o You keep in mind that the most common cause of congenital hypothyroidism with similar presentation (palpable
goitre) is iodine deficiency.
47. Ans is'a' i.e., Primary hypothyrodism [Re/ Nelson 18th/e p. 2319-23241
o Normally, T4 and T3 exert negative feedback on TSH secretion by pituitary in two ways:
t)Block the secretion of TSH by pituitary directly --> major action
ii)Block the secretion of TRH by hypothalamr.is (TRH stimulates the secretion of TSH by pituitary)
o In primary hlpothyrodism, T4 and T3 are not produced or produced in low concentration by thyroid. This results in
aboiition of negative feedback on TSH secretion. Thus, there is elevated TSH and pituitary can become hlperplastic
to produce more TSH.
r In pituitary cause of hlpothyroidism (secondary hlpothyroidism) both TSH and Thyroid hormone (T4, T3) are low
as pituitary does not secrete TSH and TSH is the major stimulation for production of T4 and T3.

About option d
o TSH resistance can also produce same picture i.e. raised TSH and low T4 with pituitary swelling.
o There is thyroid insensitivity to TSH which results in hypothyroidism (JT+ and T3). Because of reduced thyroid
hormone feedback, TSH is markedly elevated.
o However, Amongst the given options best option is primary hlpothyrodism because TSH resistance is a very rare
condition and further T4 levels are normal in Mild and moderate (Partial) TSH resistance.

Complete
.111...'' flofoyld hvn-9nl3sia .....
R"-::oiu"
. . r*:
:lr:,:+t ,t,':,.;, .l l!0Jmi!,,,:, hypoplasia
Mild Recessive Two
Partial (Mild) t Normal Normal Dominant One

48. Ans. is 'a' i.e., Cord blood at time of birth fRef: Neonatal Thyroid Sceening by Anju Virmani : Guidelines; Indian
fi
Academy of p ediatrics : http: / / w ww. iapin dia. org/ guidelines/ index. cfnl
o Neonatal hypothyroidism screening is done either in cord blood at the time of birth or blood obtained from heal prick
after 2 days of birth. Cord blood analysis at birth is preferreil approach as many patients do not turn up after 2 days.

49. Ans. is'd' i.e., Goiter is rare [Ref: Nelson 18"/e p. 2j30)
o Goiter is almost always present in congenitally hyperthyroid infants" -- Nelson
HYPOPITUITARISM & HYPERPITUITARISM

50. Ans. is 'b' i.e., Growth hormone deficiency lRe! O,P. Ghai 9th/e p, 511 6 Vhle p" 474; Nelson lSthle p. 22961
o Growth retardation along with hypoglycemia, micropenis, increased subcutaneous fat and high pitched voice is
characteristic ofGH deficiency. There is delayed puberity and skeletal age is less than chronological age.
51. Ans. is'a' i.e., Short stature since birth [ftel.' Nelson 18th/e p. 2296]
o The child with h)?opituitarism is usually of normal size and weight at birth.
52. Ans. is 'd' i.e., Pituitary adenoma lRef: Hanison 16h/e p. 2090; Nelsan l8.thle p. 2303, Xa4l
I Hlperpituitarism causes giants which have weak muscles and the most common cause of hyperpituitarism is
pituitary adenoma.
)-r- Ans. is'tr' i.e., GH receptor defect lRef: Nelson lVh/" ch' 5581

r Laron dwarfism is due to end organ resistance to GH (growth hormone recePtor resistance).

54. Ans. is t'i.e., birth [Rel O.P. Ghai 9e/e p. 511 6lh/e p, 474; Nslsan 18*/e p. 2296)
Baby small at

55. Ans. is 'd i.e., Normal body proportion [Rel O.P, Ghai 8n/e p. 511 6 Vhle p. 474; Nelson 18th/e p. 22961
56. Ans. is'a' i.e., Hydrocortisone
o Hydrocortisone, followed by GH are vital to be given in a child with lost pituitary function.

UNDESCENDED TESTIS

57. Ans. is'b" i.e., Infertility; t' Risk of malignancy lRel Nelson 18e/e p. 22601

Consequences of cryptorchidism
o Infertility o Torsion
r Testicular maligancy -+ Most common seminoma r Associated hernia
o Psychological effects of empty scrotum,

58. Ans. is'b' i.e., 6 months of age lRef: Nelson 18th/e p. 22611
"The congenital undescended testis should be treated surgically no later than 9-15 months. Surgical correction at
6 months is approPriate". - Nelson
lmportance of age at orchidoPexY
N Why after 4 months
testes descent spontaneously during first 3 months.
c The majority of the undescended
o So, orchidopexy should be done at 6 months, because spontaneous descend of the testis will not occur after 4
months of age.

B) Why before 2 years


l.To prevent infertility -+ Undescended testis is normal at birth histologically, but pathological changes develop
by 6-l2months that results in infertility. So, best time of orchidopexy is around 6 months. before the development
ofpathologic changes.
2.To prevent malignancy -+ It is uncommon for testis tumors to occur after orchidopexy performed before the age
of 2 years.

59. Ans. is 'b' i.e., l-2 years fRef: Nelson 18th/e p. 22611
"Congental undescended testis should be treated surgically no later than 9-15 months. Surgical correction at 6
months is appropriate". - Nelson

MISCELLANEOUS

50. Ans. is t' i.e., Familial glucocorticoid deficiency lRef : Nelsan 18th/e p. 2j561

Familial glucocorticoid defi ciency


o Familial glucocorticoid deficiency is a rare autosomal recessive condition.
r It is characterized by adrenal insufficiency.
r The pathological examination of the adrenal gland reveals that :-
1::,:: t: : .

r Zona glomerulosa -+ well presefired


. Zofiefasciculata -) atrophied
r Zonareticularis -t atrophied
r This causes low cortisol concentration because the zonafasciculata is primarily responsible for glucocorticoid
production.
r Because the zona glomerulosa is well preserved mineralocorticoid action is usually unaffected.
a Lou' cirutlating serum cortisol results in lack of
feedback inhibition to the hypothalamus which results
in increased ACTLI secretion from pituitary.

CNinital features
r Patients with familial glucocorticoid deficiency generally presents with signs and symptoms of adrenal
insufficiency with the important distinction thal mineralocorticoid production is ilroyt nor*ol.
o'Ihe most common initial presenting sign is deep hyperpigmentation of the skin, mucous membrane
or b<lth as a result of the action of adrenocorticotrophic hormone (ACTH) on cutaneous melanocyte stimulating
hormone (MSH) receptors.
r The syrnptoms are compatible with glucocorticoid deficiency. Many patients presents with recurrent
lrypoglycemia ar severe infections.
I In the neonatal period, frequent presenting signs include - feeding problems,
failure to thrive, regurgitation
and hypoglycemia manifesting as seizures.

61- Ans" is 1f Le", Loss of diurnal variation [R*J Nelsar 1*/e p. 2369; O.p. Ghai */e p. 524 & ?/e p. a90)
Nelsem writes -
"Laboratory
findings (for Cushing Syndrome)
s Cortisol levels in blood are normally elevated at 8 AM and decrease to less than 50o/o by midnight except in
infants and vttung children in whom a diurnal rhythm is not always established. In patients ,itt ,rihing
syndrome this circadium rhythm is lost, and cortisol levels at midnight and I AM aie usually comparable.
Abtuining diurnal blood samples present logistical dfficulties as part of an outpatient evaluation, bui cortisol
can be' measured in saliva samples, which can be obtained at home at the appripriate times of day. Night time
salivary cortisol levels are elevated and may be a screening test in obese chilitien.

62, Ano. is 't' i,e,, Morning {Ref: Netson 15"/e p- 2369)


I Cortisol levels in blood are nprmally elevated at 8.00 AM and decrease to less than 50% by midnight.

63, Ans" is t'i"e", Hlperparathyroidism tRef Nelsoa t*/e p. 2346)


r The clues given in this question are : -
l. I{ypercalcemia 2. Hypophosphatemia 3. Raised alkaline phosphatase
o AII are seen in hlperparathyroidism
t A low serum phasphorus level with hypercalcemia is characteristic of primary hyperparathyroidism,
About other options
r In Rickets (nutritional or renal) serum calcium will be decreased or normal (not increased).
o Skeletal dysplasia has normal calcium and phosphorus levels.

64" Ans. is t'i.e., Calcium gluconate JRef O.p. Ghai */e p. 5ZZ & fr/e p" 4BZ; Nelson 18tu/e p. 23401
r The baby presenting with tetany is hypocalcemic, so must be treated immediately with calcium gluconate I.V
65' Ans" is'b'i.e., IV calcium gluconate with cardiac monitory {Ref see abovel
66. Ans. is 'b'i.e., Receptor insensitivity in kidney tubules lRef: O.P. Ghai 8th/e p. 514 6 Th/e p. 4771
r In nephrogenic diabetes insipidus, level of the ADH is normal.
t The defects lies in the distal tubule where the cells fail to respond to ADH.

$7. Ans is 'H i.e., r.75 gm/kg glucose; b' i.e., 25 gm as an adult [Rc-f, Nelsor t8e/e p. 2946)
Oral glucose toleranre test
r Doses are
i) Adult -) 75 mg
ii) Chilil -) 1.75 gm/kg of ideal weight, up to a maximum of 75 gm.
Cneprpn

..r .;L,-,1,:,.,'', Otfiatl{tl$,$11:'::'"U*"*{,mg/40


Fasting 70-105 > 126

60 min 120-170 > 200

90 min 100-140 > 200

120 min 70 - 120 > 200

6&. Ans. is ?' i.e., -A"fter 5 yearc lR$: AFI Text b*ok *f Medicixe *fe p. l li6j
o In IDDM, initial retinal examination should be done 5 yrs after the onset of disease as the sight tlireateniirg t'tIil,lopatili
usually does not appear in the 6rst 5 ps & thereafter', these should be screetted annualy by rel,ita cxLre ris.
o For NIDDM, initial examination should be done right lionr the tirne of diagnosis, i:recause upio 21qi. paxit:tlts lr.t'ic
retinopathy at the time of diagnosis & the reafter annualil'.
r Diabetic women rvho become pregnant, shoulcl have dilated e1'e exanrination irt the tti'st li:iurt'str't- r* t:l*sr:1nllo",';,.'
throughout the pregnancy.
,4"*s, is
nb'i,*,, 25-3{ }itis*t', JS'ri i-ll:,:tll:'.::1ll
$*" ,qat-!l.t'.!:

r Several factors influence the initial daiiy insr.rlin close per kilograrn olbocl,v i,r'eigirt.
c The dose is usually higher in pubertal children.
r It is higher in those who have to restore greater deficits of bodir glycogen, protr:'in, arr,l lll siol.r:s alld u,ho, titt:it'ir': '

have higher initial caloric capacity.


c On the other hand, most children with new-onset diabetes have some residual o.-cell function (the "hottcvmoo;n'
period), which reduces exogenous insulin needs'
r Residual B-cell tirnction usually fades within a fell'months and is rel}ected as a steady increase in insulin requilelnenls
and rvider glucose excut'siotts.
r The initial insulin schedule should be directed torvard the oprtirnal clegrt:e of glucose contrttl in ln attetnpt 1o dt4:llcalr'
the activity of the P-ce11.
o Acceptable glucose control can be obtained n ith ner,r,insulin analogs used in a basal-trolus regimen, that is. rvrth slon'
onset, long-duration background insulin for between-rneal glucose control and rapid-onset insulin at each rneial"

:.Blilusinsrilin: ,':r::: : r

Age Target glucose Totaf daily insulin Basal insulin, o/o of Units added per Units added per 15
(yr] ' {Mg/dl} l ' tulkgtdl total daily dose lOO mgldt above . rgatmaal
target
0-5 1 00*200 0.6*0.7 25-30 0.50 0.50

5-1 2 80*'150 0.7-1.0 40-50 0.7s 0.75

12-18 80*1 50 1.0-1.2 40-50 1-2 1-2

e Newlydiagnosedchildreninthe"honeymoon"mayonlyneed60.T}oioofatuilreplacernentciose.'lbtaidailydoscper
kg increases with puberty.

78. Ans. is oH i.e., Microgla*dular *ryperplasia lfief Nelson 17'le p. 1s39]


r "W<lmen who were exposed to diet\lstilbestrol in utero have a high incidence of adenosis of the vagina anrl cri:vis.
These patient also have potential reproductive abnclrmalities, including infertility', habitual abortions iirrd lubal r.rrr,-l

uterine cavity abnormality.


Clear cell adenocarcinoma of the vagina and cervix is a rare sequaele of DES exposure in utero'l

'71 ,A"ns. is '-o- i.e., &trutatio* of ealciuxn sensing receptor {Ref : Nelsan 17h/e p. 215; Harris*w 16'hle p. 225*}

';$borxoryab.normalifies irtli€,patient i' Patient Normal


* lncreased calcium 12.8 mg/dl 9-10.5 mg/dl
* Normal phosphate (ln lowrange) 3.0 mg/dl 3-4.5 mg/dl

s lncreasedparathormone 140 pglml < 60 pglml


a Normal Bicarbonate 25 mEq/L 21-30 meq/L
*s lncreased Blood urea 60 m9 / dl 10-20 mg / dl
q Normal creatinine .7 mg/dl < 1-5 mg/dl
. rr,'',1i1 i'::i iir tir.ii
:t\:i
'i:;;tr:i:

It is a close diagnosis between hlperparathyroidism and familial hypercalcemic hlpocalciuria.


It is a case of mutation of the calcium sensing receptor
The clincher here is
o Age of the patient
o Decrease in urinary calcium excretion

Familial hypercalcemic hypocalciuria


o It is an autosomal dominant disorder caused by mutation in the calcium sensing receptor,
o Pathophysiology of familial hypercalcemic hlpocalciuria.
t The primary defect k the abnormal sensing of the blood calcium by the parathyroid gland and renal tubule.
t The calcium sensor responds to ECF calcium concentration by suppressing PTH secretion through
negative feed back.
t The mutations lower the capacity of sensors to bind calcium and the mutant receptors functions as though
blood calcium levels are low.
t As a result of the false perception that calcium level are low in the body, the parathyroid gland starts
secreting greater amount of parathyroid hormone and renal tubule starts reabsorbing greater amount of
calcium in the tubule.

r Comparison of hlper parathyroidism and familial hypercalcemic hypocalciuria


These two disorders have almost similar features, they differ only in the following aspect
I ) Urinarv renal calcium reabsorntion
r FHH ) > 99o/o of renal calcium reabsorption in the kidney (This causes decrease in urine calcium)
o Hlperparathyroidism -+ < 99o/o of renal calcium reabsorption in the kidney (This causes increase in urine calcium)
2) Age of patient
r FHH + Hypercalcemia is detected in affected members in first decade of life
r Hlper parathyroidism -+ Hypercalcemia rarely detected in patient with primary hyperparathyrcidism or
MEN syndrome who are less than 20 year of age.
3) Level of parathvroid hormone
o PTH can be elevated in FHH, but the values are usually normal or lower for the same degree of calcium elevation
than in patients with primary hlperparathyroidism.

Primary Hyperparathy , lncreased decreased lncreased lncreased lncreased


roidism

72. Ans is'b- i.e., More common in adults than in children


Nesidioblastosis
o In 1938, Laidlow coined the term nesidioblastosis to describe the neodifferentiation of islets of langerhans from
pancreatic ductal epithelium.
o Severe recurrent hypoglycemia associated with an inappropriate eleyation of serum insulin, C-peptide and
proinsulin defines this disorden
o The disorderlaterwas calledpersistenthyperinsulinemichypoglycemiaof infancy (PHHI); currentlymanyauthors
prefer the term congenital hlperinsulinism (CI).
o PHHI is the most common cause of hyperinsulinemic hypoglycemia in neonates.
o PHHI usually afiects the infants and children. Adults can also be affected but at a much lower incidence.

Histopathology
o Microscopically, pheochromocytomas are composed of polygonal to spindle-shaped chromaffin cells and their
supporting cells, compartmentalized into small nests or "Zellballeni'by a rich vascular network.
o The cltoplasm of the neoplastic cells often has a finely granular appearance, highlighted by a variety of silver
stains, because of the presence of granules containing catecholamines. Electron microscopy reveals variable
numbers of membrane-bound, electron-dense granules, representing catecholamines and sometimes other peptides.
The nuclei of the neoplastic cells are often quite pleomorphic.
Presentation
o Most patients with PHHI present shortly after birth with symptoms of hypoglycemia (e.g. hunger, jitteriness,
Iethargy, apnea, seizures). Older children, in addition to above symptoms, may also show diaphoresis, confusion,
or unusual mood or behaviour changes.
o The symptoms may be exacerbated by fasting and may improve after eating.

Treatment
e.
o Immediate treatment of hlpoglycemia is essential. Patients may require continuous IV glucose infusion. Glucagon
through
may also be administered.
is though r Other drugs used are diazoxide, octeride and nifedipine.
o Surgical treatment is indicated if medical therapy does not maintain normoglycemia.
nd starts
nount of
ANSWERS OF VARIOUS OTHER EXAMINATIONS

73. Ans. is t'i.e., Cortisone lRef : Has been explained]


r Cortisol deficiency is treated with glucocorticoids.
o Hydrocortisorre is giyen.
Note - For prenatal treatment dexamethasone is given.
e calcium)
74. Ans. is'd'i.e., Congenital adrenal hyperplasia lRef : Gupte S. 8th/e p. 511; Ghai 8th/e p. 522 6 6th/e p. 147, 492;
Nelson 18th/e p. 518, 2j61; Harrison 17h/e p. 226fl
:cidism or 75. Ans. is t'
i.e., Carbimazole fRef: Nelson 18th/e p. 23351
r Most pediatric endocrinologists recommend initial medical therapy using antithyroid drugs rather than radioiodine
or subtotal thyroidectomy.
: elevation o The 2 antithyroid drugs in widest use are methimazole and propylthiouracil.

76. Ans. is t'i.e., Cold extremities [Rel Nelson 18th/e p. 2322)


o Amongst the given options cold extremities is the best answer.
r Prolongation ofphysiologicaljaundice is the earliest sign. Nelson
-
77. Ans. is 'a' i.e., Cretinism lRef: Nelson 18th/e p. 2322)

)ecreased 78. Ans. is 'd i.e., T4 TSH assary [Ref Has been explained)
r It is typical presentation of hlpothiroidism

79. Ans. is'a'i.e., Premature closure of posterior fontanelle {Ref: Nelson 18th/e p. 2322)
r There is delayed closure ofposterior fontanelle.

80. Ans. is'c'i.e., Levothyroxine fRef: O.P. Ghai 8'h/e p. 5186 Vh/e p. 483)
::rans from
81. Ans. is t'i.e., Graves disease [Ref : O. P. Ghai 8'h/e p. 520 6 7h/e p. 485)

eptide and 82. Ans. is 'b'i.e., Tingling of extremities [Rel Nelson 18th/e p. 2i43, 2j44; CPDT 18th/e p. 955)
o Musculoskeletal pain and cramps are the earliest symptoms and progress to numbness, stiffness and tingling of the
:.nr- authors hands and feet.
o These are symptoms of tetany due to hlpocalcemia.

dence. 83. Ans. is'd i.e., McCune - Albright's syndrome fRef: O.P. Ghai |th/e p. 5i2 d2 7h/e p. 499)
McCune - Albright syndrome is characterized by -
i) Cafe - au - lait spots iii) Precocious puberty
-s and their ii) Polycystic fibrous dysplasia ofskeleton iv) Endocrinopathy + Hlpothyroidism

:r',- of silver
:1s r-ariable I:I
C TI P T

E{USCUtffiSKM,IMY&L
MX$ffiKMKKS

DISORDERS OF MUSCLE FIBRES

01).
Most of the diseases of muscle fi.bres are associated wllh elevation of creatinine kinase (MM isoenzyme)(AllMs The

diseases of muscle fibres are called myopathies (which also include muscular dystrophies).

MYOPATHY

r Myopathy refers to any skeletalmuscle disease that causes structurai changes or functional impairment of muscle'
Important causes of mYoPathY are :-
l) Congenital myopathies: Central core disease, nemaline myopathy, myotubular (centronuclear) myopathy.
Z) Endocrine and metabollc : Hypothyroidism, hyperthyroidism, hypoparathyroidism, hyperparathyroidsm,
cushing,s slndrome(AllMsor), acromegaly, DM, Vitamin-D deficiency (rickets, osteomalasia@nMs07))' oncogenic
(Myopatfu is not seen in X-linkedhypophosphatemic rickets(ArrMs07)),
osteomalacia(AlrMs07).
3) Channelopathles : Hypokalemic periodic paralysis, hyperkalemic periodic paralysis, normokalemic periodic
paralysis.
4) Inflammatory myopathies: Dermatorpyositis, polymyositisA"'so'), inclusion body myositis.
5) Toxic myopathT: Ethanol, chloroquine, corticosteroids, hypolipidemic drugs.
6) Inborn error of metabolism :Lipidmyopathy, mitochondrial myopathy.
o:), Becker's dystrophy, myotonic dystrophy'
7) Muscular dystrophies : Duchenne muscular dystroqthy{Attus
Limb girdle dystrophy, fascio-scapulo-humeral dystrophy, Oculopharyngeal dystrophy, spinomuscular
02)
atroPhYqIIMs '
o Asageneralrule,myopathieshaveproximalmuscleweakness,exceptformyotonicdystrophywhichmainlyinvolves
distal musclesallMso2) (Neuropathies usually have distal muscle weakness),
o A muscular dystrophy is a type of myopathy in which there is abnormal growth of muscle. It is characterized by :-
i) lt k a primary myopathy
ii) It has genetic basis
iii) Course is progressive
iv) Degeneration 6 Death of muscle fibers occur at some stage of the disease

Duchenne Muscular Dystrophy (DMD)


r DMD, also called peudohypertrophic mucular dystrophy, is the ruosf common hereditary neuromuscular dystrophy.
It is an XJinked recessive(DNB 15, Nrrr) di561dsL
o It is caused by a mutation in gene responsible for producing dystrophin (a sarcolemmal protein{Aro4'Mannipat0e)).
o The usual age of onset is 3-5 years(DN8 [s) (does not Present at birth or infancy), i.e., disease begins to manifest when
child starts walking.
r There is progressive muscle weakness@rl0) affecting proximal muscles of limbs. Child walk clumsily, has difficulty
in climbing stairsarlo) and the gait is waddling4tto) (Trendelenburg).
r Because of weakness of proximal (thigh) muscles, there is compensatory lrypertrophy of dktal (calf) musclss{At 10).

This is called pseudohypertrophy because it is not true hlpertrophy of muscles, instead the muscle is replaced by fat
and connective tissue.
o Deep tendon reflexes remains normal initially which may later on decrease6r's).
o Extraocular muscles are spared. Other features are scoliosis, epilepsy and mild mental retardation
r Patient die in the second decade of life because ofrespiratory failure and cardiomyopathy.
MUSCULAR DYSTROPHY
o A muscular dystrophy is a type of myopathy in which there is abnormal growth of muscle. It is characterized by :-

l) It is a primary myopatLry
ii) It has genetic basis
lll) Course is progressive
iv) Degeneration 6 Death of muscle fibers occur at some stage of the disease

Duchenne Muscular Dystrophy (DMD)


r DMD, also called peudohypertrophic mucular dystrophy, is the most common hereditary neuromuscular dystrophl.
It is an X-linked recessive@NB Is' NxEr) disorder.
o It is caused by a mutation in gene responsible for producing dystrophin (a sarcolemrnal protein@r0a'Mannipotoe)),
o The usual age of onset is 3-5 years(DNB 1s) (does not present at birth or infancy), i^e., disease begins to manifest when
child starts walking.
r There is progressive muscle weakness(Ari0) affecting proximal muscles of limbs. Child walk clumsily, has difficultr
in climbing stairsario) and the gait is waddling(uto) (Trendelenburg).
. Because of weakness of proximal (thigh) muscles, there is compensatory hypertrophy of distal (calf) musclesalo).
This is called pseudohlpertrophy because it is not true hypertrophy of muscles, instead the muscle is replaced by fat
and connective tissue.
o Deep tendon reflexes remains normal initially which may later on decrease(Ar").
e Extraocular muscles are spared, Other features are scoliosis, epilepsy and mild mental retardation
r Patient die in the second decade of life because of respiratory failure and cardiomyopathy.
r Gower sign(xo" 0a) is positive, i.e. the child uses hand support to stand from sitting position because of proximal
weakenss.
o Valleysignmaybepositive,i.e.hypertrophyofdeltoidandinfraspinatuswithwastingofposterioraxillaryfoldmuscles.
o Serum creatininephosphokinase (CPK)levels are elevated(DNBls'A110) many folds (< 160 IU/L).
r Histopathology demonstrates iliffuse degeneration and musclef bres of varying size,
o Becker's muscular dystrophy is a milder version of Duchenne muscular dystrophy in which dystrophin is not completely
absent. It is also X-linked recessive disorder and has less severity than DMD.

r There is hypertrophy of deltoid and infraspinatus with wasting of posterior axillary fuld muscles.
A child is suffering from some neuro-mu$cular disorder.
His calf picture is as follows. The diagnosis is -

a) Myasthenia qravis
b) DMD
c) Peripheralneuropathy
d) Myotonic dystrophy

ph):
Ans. is'b'i.e., DMD
r Pseudohypertrophy of calf muscles (as shown in figure) is characteristic of DMD (Duchenne muscular
dystrophy).
:len

Other Muscular Dystrophies


,1ty

Limb - Girdle muscular dystrophy


10),
o Limb - Girdle muscular dystrophy can be inherited as autosomal recessive (more common) or dominant pattern.
lat . It is due to mutation in genes for several sarcolemmal proteins, e.g. adhalen, dysferlin, coyeoline -3 or calpain -3,
sarcoglycan (cr., F, y and 6 ), fukutin, titin and telethronin,
r The age of presentation is late childhood (end of first decade). There is primarily weakness of hip and shoulder
girdle muscles with involvem ent of neck muscles. Cardiac involyement and mental retardation are not seen.

Fascioscapulo humeral muscular dystrophy


o Fascioscapulo humeral muscular dystrophy is an autosomal dominant disorder.
o Desease starts with asymptomatic facial weakness followed by (in sequence) scapular fixator, humeral, truncal and
Iower extremity weakness. Biceps and triceps are typically inyolyed with sparing of deltoid and
forearm muscles in
'popeye' arm appearance. Lower abdominal muscles are weaker than upper abdominal muscles resulting in'Beevor
sign'. Extraocular and bulbar muscles are spared.
o Extramuscular findings are Coat's disease (exudative retinal detachment), hearing loss, arrhlthmias and restrictive
lung disease.
r It can be associated with Mobius svndrome.

r Fascioscapulohumeral dystrophy showing facial weakness (inability to close eyes) and asymmetric scapular
winging.

Emery - Dreifuss dystrophy


o Emery - Dreifuss dystrophy is an X-linked recessive disorder due to mutation of gene encoding for nuclear
membrane protein'Emerin'.
r Child present with progressive weakness of scapulohumeroperoneal region.
o There may be cardiomyopthy with heart block and arrhythmias.
e CPK levels are normal.
\
Myotonic dystrophy
o Myotonic dystrophy is th e most common muscular dystrophy in adulfs. It is transmitted as autosomal dominant(Ar
8e)
pattern and is caused by triple nucleotide repeat of 'CTG', in DM gene on chromosome 19.
:

i,R L3 Musculoskeletal Disorders

r lzlyotonic dystrophy t)?ically starts in distal muscles('4r.Ilrs 02) (in contrast to other muscular dystrophies whici:
involve proximal muscles) . The classical form presents with myotonia, facial weakness (gives V-shaped upper lip , .

cataract, frontal baldness, endocrinopathies (hypothyroidism > hlpethyroidism; adrenal insufficiency; DM), cardiac
arrhythmia, GI motility disturbance and mild mental retardation.
o CPK levels are usually normal.

Congenital muscular dystopy


o Congenital muscular dystopy is due to defect in sarcolemmal protein'Merosin'which is also present in CNS.
r The disease presents at birth or infirst year oflife.
r There is hypotonia, weakness, arthrogryposis (multiple joint contractures), bulbar dysfuction of respiraton'
ir.rsufficiency.
o CNS involvement (as merosin is present in CNS) causes holoprosencephaly, Iissencephaly, cerebellar hlpoplasia
and agenesis of corpns callosum.

o Important motor neuron (anterior horn ceil) disorders in children are spinal muscular atrophy and poliomyelitis.

Spinal muscular atrophy (SMA)


o It is a degenerative disease of motor neurons that begins in fetal life and continue to progress in infancy and childhood.
o It is caused by m utation of SMNI gene resuling in progessive apoptosis of neurons form fetal period till death.
e The disease is classified into 4 tlpes : SNIAL (Werdnig - Hoffman disease); SMA2 ; SMA 3 (Kugelberg-Welander disease);
and SMA 4.
o Infant presents typically with hypotoniaro"Ms 02' 00),
generalized weakness, Hyporeflexia or areJlexia{AllMs 02, oo),
tongue
fasciculations(AllMso0) andfroglike pasture(AttMs00). Extraocular muscles and sphir.tcters are spared. Intelligence is
normfll\ililsoo).

r Generalized hypotonia with frog-like posture and tongue fasciculations in SMA:I (Werdnig-Hoffrnan disease).

Myasthenia gravis
r Myasthenia gravis (MG) is an autoimmune disorder, characterized by autoantibodies against acetylcholinereceptors
at myoneural junction.
o Girls are affected more commonly than boys.
o There may be association of thymoma (but less common than adult myasthenia gravis).
r Clir-rical f"eatures of MG in children are :-
1) Neonatal MG
r It is by caused by transpiacer.rtal transfer of maternal anti - Ach antibodies to the fetus.
2) Congenital MG
I It is caused by genetic defect in Ach recepto rs (there are na anti - Ach antibodies) .In both neonatal and congenital
MG, baby is hypotonic, i.e. 'floopy infant'. Reflexes are preserved.
M
iilt

,l;

rl:

3) |uvenile MG
r It is caused by production of anti - Ach antibodies in children. It is associated with other
autoimmune clisorcler
like Hashimoto thyroiditis, SLE and |RA.
r It presents with fatigue of muscles of masticaticln, swallowing and respiration. But in children
specific features
are ptosis and diplopia with normal pupillary respones.
r Usually distal muscles are involved, but in some cases proximal muscles may also be involved.
r Thereistlpical diurnalvariationinweakness(moreintheevening(Ar1s))and.fluctuatingpatternofweakness
(patient suddeniy becomes asymptomatic). The weakness inteases with activity.

Diagnosis
r Following tests are used :
i) Ocular ice pack fesf (appiication of ice pack to eye relieves ptosis)
ii) cholinergic tests (edrophonium/tensilone test and neostigmine test).

Treatrnent
o Anticholinesterases (physostigmine) are the drug of choice. Other treatment options are
corticosteroids, mycophe,olate
mofetil, azathioprine and plasmapheresis. Thymectomy is useful for juvenile MC but not in congenital/infantile
MG.

A {hildis hoyins following picture ill the evcntng and nor-


mal appearance in mnrning; The probahle diAgnosie is -

a) Myotonic dystrophy
b) Mysthenia gravis
c) DMD
d) Becker's dystrophy

Ans. is'H i.e., Mysthenia gravis


o Diurnal variation with ptosis in evening is characteristic of |uvenile MG.

FLOPPY INFANT SYNDROME

o Floppy infant syndrome refers to infants which have hypotonia (floppy).


c Infant cannot hold is head up and there is marked head lag as the baby is pullect up from supine ta sitting posture.
r A normal infant, on ventral suspension (i.e. supporting the infant with a hand under the chest) keeps his hack straight
and head slightly up, arms abductecl and knees partly flexed. A
Jloppy infant tlroops, oyer the hantl like 6n irwerted Lt.
r There is decreased muscular activity, frog like posture and delayecl milestones.
. Important causes of FIS are :-
1) CNS: Perinatal asphyxia, kernicterus, cerebral palsy, intracraniai bleecl, Down syndrofite(AttMs 0o.
2) Spinal cord lesions : werdnig Hoffman spinal muscular atrophy (sMA-1l,r,tut.s oo), poliornyelitis (both
are
motor neuron diseases).
3) Peripheral nerves I Acute polyneuropathy, tamilial dysautonomia, congenitai sensory neurgpathy
4) Myoneural junction : Infantile botulism@ltMs 00), neonatal mysthenia gravis.
5) Muscles : Muscle dystrophies, congenital myopathies, polymyositis, pornpe's disease.
6) Miscellaneous: Rickets, PEA.4, cretinism, Ehler - Danlos syndrome, Prader - Willi syndrclme.
ARTHRITIS IN CHILDREN

o Arthritis in children is diagnosed if there is :

A) Swelling or efusion in a joint


B) Any two of following: Decrease range of motion, tenderness, pain or increased local temperature.
o Differential diagnosis of arthritis in children includes :-
.L) Acute arthritis (< 2 weeles) t Rheumaticfever@Gt0q), transient synovitis, kawasaki disease, HSP, septic arthritis@Gl
oo),
trauma,
B) Subacute arthritis (2-6 weeles): Reactiye arthritis?Gl00), SLE, PAN, dermatomyositis, Ieukemia, neuroblastoma,
lyme's disease, brucellosis, sickle cell diseases, hemarthrosis.
C) Chronic arthritis (> 6 weelcs):: JRA?GI00), ankylosing arthritis, psoriasis, tubercular arthritis, IBD, Perthe's disease.

JUVENILE IDIOPATHIC ARTHRITIS

r fuvenile idiopathic arthritis is broad term that describes a clinically heterogenous group ofarthritides ofunknown
cause, which begin before 16 years of age.
o This term encompasses several disease categories :-
1) Iuvenile rheumatoid arthritis
2) luenile psoriatic arthritis
3) Iuvenile enthesitis related arthritis.

JUVENILE RHEUMATOID ARTHRITIS (JRA)


o JRA is the most common t1,pe of fuvenile idiopathic arthritis, thus, most often JRA is used as a synonym for juvenile
idiopathic arthritis.
o |RA is a group of chronic inflammatory joint disorder characterized by :

i) Arthritis of one or tnore joints(PGt06)


ii) Onset before 76 yearsat 13' PGI0e' 06)

iii) PersistingfoT ) $ 1rys*fu@Gr os' oc)

iv) Exclusion ofother causes ofarthritis.


o |RA can be classifled into following types :-
A) Pauciarticular (oligoarticular) JRA
r It is charact erizedby involvement offour or fewer joinfs. It is the most common type of JRA, Joint involvement
is assymetrical,

r There are two subtlpes :-


i) Type 1

r It is characterized by eye involvment with iridocyclitis, glaucoma, cataract.


o It is associated with HLA - DRs and DRW8,
t Antinuclear antibody (ANA) is positive but rheumatoid factor (RF) is negative.
ii) Type II
o It is characterized by self limiting iritis (not iridocyclitis) and is associated with }lLA-827,
o Both ANA and RF are negative.

B) Polyarticular |RA
r It is characterized by involvement of 5 or more joints.
r There are two subtypes :-
i) Polyarticular RA positive
.ItischaracterizedbysymmetricaljointinvolvementalongwithUyeitis(^IIM'lt'At,e),andrheumatoidnodules(
AilMS 11, AI 09).
\
o RA factor and ANA are positive.
ii) Polyarticular RA negative
o RA factor and rheumatoid nodules are not seen.
o Arthritis in children is diagnosed if there is :

A) Swelling or efusion in a joint


tenderness, pain or increased local temperature'
B) Any two of following : Decrease range of motion,
includes :-
r Differential diagnosis of arthritis in children
o0) synovitis' kawasaki disease' HSP' septic arthritisqcl
A) Acute arthritis (< 2 weetcs) t Rheurnatic feverecl ,transient
oo), trauma.
sLE, PAN, dermatomyositis' leukemia' neuroblastoma'
B) subacute arthritis (2-6 weeles)t Reactiye arthritis@Gtoo),
hemarthrosis'
lyme's disease, brucellosis, sickle cell diseases'
IBD' Perthe's disease'
ankylosing arthritis, psoriasis, tubercular arthritis,
c) chronic arthritis (> 6 weeks)r: IRA(pcro'),

JUVENILE lDloPATHlc ARTtlRlTls


group of arthritides of unknown
r broad term that describes a crinicany heterogenous
|uvenile idiopathic arthritis is
cause, which beginbefore 16 yeats of age'
:-
o This term encompasses several disease categories
1) IuYenile rheumatoid arthritis
2) luenilePsoriatic orthritis
3) luvenile enthesitis related arthritis'
(JRA)
JUVENILE RHEUMATOID ARTHRITIS for juvenile
arthritis,thus, most often ]RA is used as a synonym
o JRA is the most common type of ruvenite idiopathic
idioPathic arthritis'
o |RAis a grouP of chronic inflammatory
joint disorder characterized by :

i) Arthtitis of one or more iointe(eeroet


ii) Onset befote 76 years^tl3'Pct0e'06)
0e' 05)
iii) Persisting for >- 6 weaLs@Gr
iv) Exclusion ofother causes ofarthritis'

o |RA can be classified into following types :-


A) Pauciarticular (oligoarticular) JRA type of JRA' Ioint involvement
r It is charact eizedby involvement of four or fewer ioinfs. It is the rnost common
is assYmetrical'

r There are two subtYPes :-


i) TYPe 1

rltischaracterizedbyeyeinvolvmentwithiridocyclitis,glaucoma,cataract'
o It is associated with HLA - DRs and DRWS' I

rheumatoid factor (RF) is negative'


I

o Antinuclear antibody (ANA) is positite but


ii) TYPe II
(not iridocyclitis) and is associated with IJLA-827 '
o It is characterized by self limiting iritis
o Both ANA and RF are negative'

B) PolYarticular )RA
r It is characterized by involvement of 5 or mote ioints'
r There are two subtYPes :-
i) PolYarticular RA Positive
r Itischaracterize dbysymmetricaljointinvolvementalongwithlJveitis(Alr*sll'Al0s)'andrheumatoidnodules(
A[,"II1S 11' At09),
\
o RAfactor and ANA are Positive'
ii) PolYarticular RA negatite
o RA factor and rheumatoid nodules are not seen'
C) Systemic |RA
a It is characterized by systemic features hke fever with rash(NEEr'AIIMS oI), lymphadenopathy, and
tuus ot ),
hepatosplenotnegaly(Nnnr'
r There is neutrophilic leukocltosis with raised ESR.
r ANA may e p ositiv e, but RA factor is negative@EEr).
b

z Uv eiti s ( ir i d o cy cliti s ) i s ch ar act er i st i c ally as en{ AI IM s o t )


.

o Initial drug of choice for fRA is one of the NSAIDs. NSAIDs commonly used in children are Naptoxen, lbuprofen,
8e).
Aspirin@ es) and Indomethacinql

.,UVENII-E PSCIRIATIC ARTHRITIS


o It account for less than 10% ofcases ofjuvenile idiopatic arthritis.
o It is characterized by DIP joint involvement, nail pitting, psoriatic rash and positive family history of dactylitis.

JUYENILE ENTHESITIS RELATED ARTHRITIS


e It account for less than 10% ofcases ofjuvenile idiopathic arthritis.
o It is characterized by sacroilitis, involvement of lower extremity, joints, enthesitis. sausage digits, abnormal schober
test, oral ulcer and HLA-827 positive.

M'5EELLAT{ EOt,'5

lnfantile {childhood) osteopetrosis


o Osteopetrosis is characterized by the faiiure of osteoclasts to resorp bone.
e Osteopetrosis may be infantile osteopetrosis (malignant osteopetrosis or osteopetrosis congenita) or adult osteopetrosis
(b enign o steop etra sis).
e Features ofinfantile osteopetrosis are :-
r Increased bone density masking the distinction between cortex & medulla, bone within bone appearance (Marble bone)
(PGr 01)
.

r Brittle bone & easily fracturability{Pcr oa)

r Fracture healing satisfactory resulting in skeietal deforrnities'


r Myelophthisic anemia
r Hepatosplenomegaly(ocro,) & presence of immature leukocytes & normoblasts in the peripheral blood.
r Growth retardation, Macrocephaly(Pct oa).
r Thickening of cranial foramina & thus causing pressure on optic nerve & optic atrophy.
x Deafness & blindness
r Dental problems & osteomyelitis of the mandible.
x Learning disorder but may have normal intelligence despite hearing & visual loss(Pcr04).

eauses of knee pain in adolescence


o Time of the knee pain helps in the diagnosis of the disorders : -
1) Night pain indicates
1) lnflammatory disorder ) Iuvenile rheumatoid arthritis(Pcl02)'
ii) Bursitis,synovitis
ni) Tumor-s Osteosarcomaqcl 02)

2) Pain at the end of the day and after activity indicates


i) Mechanical Pain due to injurY
3) Continuous pain present day and night indicates
i) Infection -+ Osteomyelitised02), Septic arthritis
ii) ldiopathic adolescent anterior knee pain slytdrome (Chondromalacia patellalee'o't.
Causes of limplng in ehildren

O-4yearc 1. Developmental -+ CDH lDDgYeetoat


2. Trauma *) Fracture, sprain, contusion
3. lnfectious -+ Osteomyelitis, synovitis, septic orthritisf.Gtos)
e 4-lOyedrs l. Trauma -) Fracture, sprain, contusion
2. lnfectious -+ Osteomyelitis, synovitis, septic arthritis
3. Osteochondroses -+ Per th e's d i se oggPct 08)
4. Neoplasia -) Leukemia
5. lnflammatory -+ JRA
* l0-l8years 1. Trauma _> Stipped capital femorat epiphysisact 08),fracture, sprain, contusion
2. lnfectious -) Osteomyelitis, synovitis, septic arthritis
3. Neoplasia
4. Tarsal coalition -+ Osteosarcoma, Ewig's sarcoma
5. Osteochondroses -+ Osgood-Schlatter disease

* lncreased CPK suggests injury fo : Muscule fibres.


* Myopathy is not a feature of: X-linked hypophosphatemic rickets.
s Weakness in myopathies: Proximal muscles weakness (except myotonic dystrophy which has distal muscle weakness).
o Weokness in neuropathies: Distal muscles weakness.

* Defect in Duchenne muscular dystrophy: Sarcolemmal proteins.


a Duchenne's musculor dystrophy ls : X-linked recessive.
* Progressive weakness, difficulty in walking upstairs, waddling gait, hypertrophy ofcalf Duchennet muscular dystrophy.
o lnfection causing floppy infant:Clostridium botulinum.
* Most common chranic arthritis in children : JRA.
* Age criteria for JRA:< 16years.
* Criteria forJRA: One or more joints, < 16 years, > 6 weeks.
* Not seen in systemic JRA: Uveitis, RA factor.

o NotafeatureofJRA:Raynaudt phenomenon.
s Absence of long bones: Phacomalia.
a Drug causing Phacomalia:Thalidomide.
a lnfantile coxa yero: Blount's disease.
* Multiple recurrent fractures with norma! heolrng : Osteogenesis imperfecta.
* Multiple fractures with various stages of heoling : Battered baby syndrome.
* Most common cause of acute osteomyelitis in children: Staphylococus aureus.
e Most common cause of septic arthritis in childern : Staphylococcus aureus.
& Acute osteomyelitis differentioted from soft tissue infe'cton by : MRl.
* Septic arthritis of hip joint in infancy :Tom smith arthritis.
* Most common fracture in children:Forearm.
* 2nd bone most common fracture in children: Clavicle.

III
QUESTIONS

l. l0 years old Ramu has increasing muscle weakness c) May be normal initially lateron increase
and raised CPK levels. The most likely defect is in d) May be normal initially lateron decrease
plasma membrane of - (AIIMS Nov 01)
10. An infant presents with hypotonia and hyporeflexia.
a) Nerves During his intrauterine period there was polyhy-
b) Muscle fibres dramnios and decreased fetal movements. Most
c) Basement membrane probable diagnosis is - (AIIMS May 02)
d) All body cells a) Spinal muscular atrophy
2, Drooping eyelids in evening in boy -(CET luly 15 pattern) b) Congenital myasthenia
a) Myasthenia gravis c) Congenital myotonia
b) Oculomotor nerve palsy d) Muscular dystrophy
c) Botulism toxin l l. Give the most probable diagnosis of a I yr. old child
d) All of above of normal intelligence with features of hypotonia.
On examination there are tongue fasciculations and
MYOPATHIES he keeps his body in a frog like position -
a) Guillian Barre Syndrome (AIItulS Nov 2000)
3. Myopathy is seen in all except - (AIIMS May 07) b) Limb girdle atrophy
a) X-linked hypophosphatemic rickets c) Downs syndrome
b) Oncogenicosteomalacia d) Spinal muscular atrophy
c) Nutritionalosteomalacia L2, In a 6 months old baby, floppy infant syndrome is
d) Cushing syndrome seen commonly due to infection with -
+. All of the following are a$sociated with proximal a) Clostridium welchii (AIIMS lune 2K)
muscle weakness fxcfpt - (All India Dec.14 Pattern) b) Clostridium tetani
a) Spinomuscularatrophy c) Clostridium Botulinum
b) Duchenis muscular dystrophy d) Clostridium septicum
c) Polymyositis
d) Myotonic dystrophy
ARTHRITIS
5. Duchenne Muscular Dystrophy is a disease of -
a) Neuromuscularjunction (A104) 13. Most cornmon chronie arthritis seerr in ehildren -
b) Sarcolemmalproteins a) IRA (CET lune 14 pattern)
c) Muscle contractile proteins b) Reactive arthritis
d) Disuse atrophy due to muscle weakness c) Rheumatic fever
6. Duchenne muscular dystrophy true - d) Sepsis
a) autosomal dominant (CET Nov 15 Pdttern) 14, A ten-year old girl presents with swelling of one
b) onset at second decade oflife knee joint. All of the following conditions can be
c) universal increase in creatine kinase considered in the differential diagnosis except -
d) normal cardiac muscle fibre. a) Tuberculosis (Ar 0i)
A 6 year old boy presents with progressive w€akness b) |uvenile rheumatoid arthritis
in muscles, and difficulty in walklng upstairs. He has c) Haemophilia
dtfficulty in walking on his toes and has a waddling d) Villonodularsynovitis
gait. Hypertro,phy of calf muscles in noted. His CPK
levels are 10,000 IU. Which of thc following is the JRA
most appropriate diagnosis - (AI 10)
a) Duchenne muscular dystrophy. 15. Age criteria for iI{A (All India Dec.1j Pattern)
b) Polymyositis a) < 10 yrs.
c) Congenitai myopathy b) < 12 yrs.
d) Myotonia congenita c) < 14 yrs.
8. Duclrnne's rnuscular dystrophy - (NEET Dec.12 Pattern) d) < 16 yrs.
a) X-linked dominant 16, All of the following are f€aturcs of systemic |uvenlle
b) X-linkedrecessive Rheumatoid Arthritis cxecpt - (AIIMS May 01)
c) Autosomal dominant a) Uveitis
d) Autosomal recessive b) Rash
9. Irr duchenne the knee jerk - (All India Dec15 Pauern) c) Fever
a) Exaggerrated d) Hepatosplenomegaly
b) Decrease
l. l0 years old Ramu has increasing muscle wealuress c) May be normal initially lateron increase
and raised CPK levels. The most likely defect is in d) May be normal initially lateron decrease
plasma membrane of - (AIIMS Nov 01) 10.
An infant presents wlth hypotonia and hlporeflexla.
a) Nerves During his intrauterine period thcre was polyhy-
b) Muscle fibres dramnios and decreased fetal movements. Most
c) Basement membrane probable diagnosis is - (AllMS May 02)
d) All body cells a) Spinal muscular atrophy
2. Dronping eyelids in evening in troy -(crT fuly 15 Pattern) b) Congenital myasthenia
a) Myasthenia gravis c) Congenital myotonia
b) Oculomotor nerve palsy d) Muscular dystrophy
c) Botulism toxin ll. Give the most probable diagnosls of a 1yr. old child
d) All of above of normal intelligence with features of hlryotonia.
On examination there are tongue fasciculations and
MYOPATHIES he keeps his body in a frog like position -
a) Guillian Barre Syndrome (AIIMS Nov 2000)
3. Myopathy is seen in all except - (AllMS May 07) b) Limb girdle atrophy
a) X-linked hlpophosphatemic rickets c) Downt syndrome
b) Oncogenic osteomalacia d) Spinal muscular atrophy
c) Nutritional osteomalacia 12, In a 6 months old baby, floppy infant syndrome is
d) Cushing syndrome seen commonly due to infection with -
n All of the following are associated with proximal a) Clostridium welchii (AIIMS lune 2K)
muscle weakness except - (All India Dec.14 Pattern) b) Clostridium tetani
a) Spinomuscularatrophy c) Clostridium Botulinum
b) Duchenis muscular dystrophy d) Clostridiumsepticum
c) Polymyositis
d) Myotonic dystrophy ARTHRITIS
5. Duchenne Muscular Dystrophy is a disease of -
a) Neuromuscularjunctior (Alo4) 13. Most eommon chronic arthritis seen in children -
b) Sarcolemmalproteins a) IRA (CET June 14 Pattern)
c) Muscle contractile proteins b) Reactive arthritis
d) Disuse atrophy due to muscle weakness c) Rheumatic fever
6. Duchenne muscular dystrophy true - d) Sepsis
a) autosomal dominant (CET Nov 15 Pattern) 14, A ten-year old girl present$ with swelling of one
b) onset at second decade oflife knee joint. All of the following conditions can be
c) universal increase in creatine kinase consldered in the differential diagnosls except -
d) normal cardiac muscle fibre. a) Tuberculosis (Ar 03)

7, A 6year oldboypresents with progresslve weakness b) ]uvenile rheumatoid arthritis


in muscles, and difficulty ln walklng upstairs. He has c) Haemophilia
difficulty in walklng on his toes and has a waddling d) Villonodular synovitis
galt, Hypertrophy of calf muscles in noted. His CPK
levels are 10,000 IU. Whtch of the following ts the JRA
most approprlate diagnosis - (AI 10)

a) Duchenne muscular dystrophy 15. Age eriteria for JRA * (All India Dec.13 Pattern)
b) Polymyositis a) < 10 yrs.
c) Congenital myopathy b) < 12 yrs.
d) Myotonia congenita c) < 14 yrs.
Duchnne's muscular dystrophy - (NEET Dec.12 Pattern) d) < 16yrs.
a) XJinked dominant 16. All of thc followlng are features of syetemic |uvenlle
b) X-linkedrecessive Rheumatoid Arthritie cxecpt - (AIIMS May 01)
c) Autosomal dominant a) Uveitis
d) Autosomal recessive b) Rash
In duchenne the knee jerk - (AllIndiaDecl5Pattern) c) Fever
a) Exaggerrated- d) Hepatosplenomegaly
b) Decrease
17. All of the fbllowing are seen in Systernic ]uvenile a) Hemophilousinfluenzae (AllMS May 94)
Arthritis, except - (NEET Dec.12 Pattern) b) Staphylococcus aureus
a) Rheumatoid Factor + ve c) Gonococi
b) Hepatosplenomegaly d) Pneumococci
c) High fever with rash 22. Acute osteomyelitis can best be distinguished from
d) Elevated E.S.R. soft tissue infection by - (AMC 2K)
18. Which of the following is not a feature of Juvenile a) Clinicalexamination
Idiopathic Arthritis - (AttMS May 11, Al 09) b) X-Ray
a) Rheumatoid nodule c) CT scan
b) Spikes ofhigh fever d) MRI
c) Uveitis 23. Gower sign is classical of one of the following condi-
d) Raynaud'sphenomenon tion- (Karn 04)
a) Congenital myopathy
MISCELLANEOUS b) Werdig-Hoffman disease
c) Duchenne muscular dystrophy
19. Fhocomelia is - (NEET Dec.12 Pattern) d) Guillain-Barre syndrome
a) Absence oflongbones 24. Duchenne's muscular dystrophy is a disease of -
b.) Absence of brain a) Neuromuscularjunction (Manipallg)
c) Reduplication ofbones b) Sarcolemmalproteins
d) Absence ofheart c) Muscle contractile proteins
20. A one year old child presented with multiple frac- d) Disuse atrophy due to muscle weakness
tures seen in various stages of healing. The most 25. Drug of choice initially in |uvenile chronic arthritis is
probable diagnosis in this case is - (AIIMS Nov 05)
a) Scurry a) Salicylates (At 8e)
b) Rickets b) Indomethacin
c) Battered baby syndrome c) Prednisone
d) Sickle cell disease d) Phenylbutazone

QUESTIONS OF VARIOU5 OTHER


EXAMINATIONS
11 Septic arthritis in a 2 year old child is often caused
by-

!II

l
CuAprsnirr3
A[\JSWERS
1. Ans. is 'b' i.e., Muscle fibres [Rel Chandrasawta Taylor 3thle p. 9531
o Increase in CPK level with muscle weakness suggests muscle cell damage.
n Ans. is'a' i.c", h4y*stlaeraia gravis $re! Gkl;*i 7"le 7t. 5731
c Ptosis/drooping of eyelids with diurnal variation (more in the evening) is seen in |uvenile myasthenia gravis.

MYOPAT'H!ES

3. Ans. is'a: X-linked hypophosphatemic rickets fRef: Nelson 18th/e p. 2551; Ilarrison 17h/e p. 26941
o Myopathy is not seen in X-linked hlpophosphatemic rickets. Other three options are causes of myopathy.
*d'
4. Ans" is i.e., J!{yertonic dystrophy lR(: A.P. Ghai 6tt'le p. 551 d- 7t'le p. 557-57A; &ktrsom 18'hle p. 25,}4}

o The general rule in that :


e Myopathies have proximal muscle weakness.
a Neuropathies have distal muscle weakness.
e Myotonic dystrophy is an exception to this general rr.ile and mainly involves the distal muscles.
5" Ans" is 'tr' i.e", Sarcolemmal protein lRef: l'{elson 18th/e p. 2541, 25421
o Duchenne muscular dystrophy is caused by a mutation in the gene responsiblefor producing dystrophin.
o Dystrophin is 'subsarcolemmal protein' localized to the inner surface of the sarcolemma of the muscle fibre.
6" Ans. is k' i"e., Universal increase in creatine kinase fP.ef: Gkai 7h/e p.671

e DMD is an X-linked recessive disorder with usual age of onset at 3-5 years. CPK levels are elevated.
e Histoiogy of cardiac muscle shows absent of dystrophin on sarcolemma.
1 Ans" is 'a'i"e., Duchenne rnuscular dystrophy p.
lReJ: Nelsan LE'h/e 2541, 25421

o Progressive weakness, dilirculty in walking upstair, hlpertrophy of calf muscle with raised CPK suggest the diagnosis
of DMD.
nb'
8" .{ns" is i.e., X-linkecl rece$$ive tRef 0.p. {ileoi Sthle p. 595 6 7h/e p. 5661

r Duchnne & Becker's muscular dystrophy are X-linked recessive.


9. Ans. is'rl' i"e., &{ay he nc}rmal initially }ateron deerease Lllef: Nelson ISthle p. 6a8]
o Deep tendon reJlexes remain normal or are decreased in patients with DMD.

t0" Ans. is oa' i.e., Spinal muscular atrophy [Ref Nelson l9thle p. 2557; CI.P. Ghai 9nh/e p. 569, 591, 594 6 Vh/e p. 574, 573,
r aol

e Out of the given options only spinal muscular atrophy is able to cause hlpotonia and hyporeflexia during infancy.
Rest of them either present late in childhood or do not give the symptoms.

11. Ans. is'd' i.e., i"e., Spinal rnuscular atrophy fRef: CPDT 15'h/e p. 694; A.F. Ghai 8'h/e p. 559, 591, 594 & 7h/e p. 559,
574, 573, 613; Nelsaru 17hle p. 2A75, 2A761
e Features of hypotonia andfrog like position suggest the diagnosis of Floppy infant syndrome.
o There are many causes of floppy infant syndrome.
e Out of the four option given in the question three of these can cause FIS -
s Downs syndrome
v G.B. syndrome
e Spinal muscular atrophy
o Spinal muscular atrophy
x Features of normal intelligence and tongue fasciculations strongly suggest spinai muscular atrophy.
a Down's syndrome
x Can be easily ruled o:ut as intelligence is impaired in Down's syndrome.
o G.B. syndrome
x Cap be ruled out as it presents as an acute case with a past history offever or viral infections.
x Majority of patients recover within few days to 2 weeks.
' . :::' ..:.,.. .:., .:.:.:. ....... . :

*, -M* *. ulpd<qlet al. D- I sord er s

t2. Ans. is'c' i.e., Clostridium Botulinum lRel Nelsan 18tt'/e p. 1225)
r Botulism causes acute flaccid paralysis -+ floppy infant.

ARTHRITIS

Ams. is 'a' i.e., |Rd [?{ef; {}.F. Gkai B,},la p.624 $ Zhle 1>. 595 tabte (20.1}}
r
Amongst the given option only )RA causes chronic arthritis in children.
t4. Ans" is t' i"e., Haemophilia lRef Maheshwari 3d/e p.185; O.p. Ghai Bele p. 624, 625 {z p/e p. 5991
o Althoughallconditionsmentionedintheoptionsmayproduceswellingofonekneejoint,theclueliesinthefactthat
haemophilia is an X Linked recessive disease and is not manifested in giils who act only as carciers.

JRA

t5. ,Ans. is kf i"e", < 16,vrs" [ii{r. 0.p. Ghai Thle p" 6{}A}
r General criteria for diagnosis of |RA includes : -
l) Arthritis of one or more joints
ii) Onset below the age of 16 years
iii) Persistingfor at least 6 weelcs
iv) Exclusion ofother causes ofarthritis
t6. Ans. is'd i.e., Uveitis [RefO.P. Ghai 8th/e p. 626 b Th/e p. 6001
e The eye manifestation are seen in Pauciarticular and Polyarticular lF.Abut not in systemic
IRA.
r@ can be divided in 3 major clinical tlpes

JRA

Pauciarticular JRA Poly Articular JR{ SystemicJRA


. Mosl./requent JRA . 5 or more large / small . Fever

JA
. 4 or lessjoints involved joints affected . Rash
. 2 subtypes . 2 subtypes . Lymphadenopathy
. Splenomegaly
. ANA can be (+ve)

Type I Type II Rheumatoid Rheumatoid


Iridocyclitis present . Selflimiting factor (+ys) factor (-ve)
Glaucomas and cataract Iritis occurs but . . Occurs at any
Age of onset late
may occur not chronic age
(Adolescents)
ANA (+ve) Iridocyclitis . Uveitis present
RF (ve) . . ANA (-ve) . Rheumatoid
HLADR5 . (RF (ve)
. HLAB27 factor (+ve)
and DRW8 . Rheumutoid Nodule (+ ve)
associated
' ANA (+ys)

1.7. A*s" is na' i.c., Kke&naatoisl Fae tarr +ve [I{(r: {}.{1 Gh*i 8,hl* p.626 $ Thle p. 6AA}
Rernember
1. Eye manifestation (Irdocyclitis) can occur in both pauciarticular and polyarticular
|RA and absent
in systemic fRA
2. ANA-can be present in all forms of IRA
3' R.heumatoid factor & Rheumatoid Nodule are (+ve) only in some cases of polyarticular
form.
4. Rashes Splenomegaly and lymphadenopathy are present in system lRA.
Cn.csrER,t,3

18. Ans, is'{ i,e,,RaYnaud'o Phenomenon


(80-85%) of juvenile idiopathic arthritis. For symptoms of
o fuvenile rheumatoid arthritis is the most common form
JRA, See Prevlous exPlanations'
(> S0% cases); Some people use luvenile
Note : As IRA is the most common form of luvenile idiopathic arthritis
idiopathic arthritis as synonyms for IRA'

MISCELLANEOUS

tg. &x*. i*'*l i'*", A t**xn kwy,\z*rt'*t' l ll'':i: l}' {ikrti'''' ir: ;;'i't'St'
{ ;'

'Phacomelia' t ls a term apptied to a marked reduction in the size of limbs


(absent long bones)
fhe distal iirt of the limbs are placed very near the trunk '

recieved thalidomide during pregnancy'


o Seen in foetuses of mothers who

20.Ans'ie'Ci.e.,BatteredbabysyndromelRef:?arikhdh/ep'18fl
baby syndrome'
. Multiple fractures at different sites and of different ages suggest the diagnosis of Battered injury is at variance with the
whom d.gr.. and tlpe of
r Battered baby syndrome - should be considered in any child in

historY given.
rWheninjuriesofdifferentagesandindifferentstagesofhealingarefound.
serious injuries'
o when there is a purposeful delay in seeking medical attention despite
of ury"bo.r., subdural hematoma, failure to thrive, soft tissue swelling or skin
r who exhibits evidence of fracture
bruising.

QUESTIOI{5 Or VARIOUS OTI{ER EXAMINATIoNS

2l.Ans.is'b-i.e,,Staphylococcu$aureuslfuef:NelsonlTh/ep'2297J
o Bacteria are the most common pathogens in acute skeletal infections'
staphylococcus aureus infection is most comlnon'
o The microbial spectrum is deverse in suppurative arthritis, but
r Salmonella is the most common cause of osteomyelitis in children with sickle cell anemia'
22. Ans. is 't i.e.,MB.ll&e! Nelson 18th/e p' 28431
and for differentiation between bone
r MRI radiographic imaging technique lbr the identification of abscess
is the best
and soft tissue infection'
p' 595 6 Vh/e yth/e p' 568)
23. Ans. is 'C l.e.,Duchenne muscular dystrophy lRef : O'P' Ghai
24, Ans. is 'c' i.e.,Muscle contractile proteins [Ref : CMDT p' fi251
25. Ans. is ,a>v i.e., salicylatee > Indometh*cin{Ref: Nelson 18th/e p. 10051
o Initial drug of choice is one of the NSAIDS'
r NSAIDs commony used in children are Naptoxen, lbuprofen' Aspirin and indomethacin'
IAP 705
-
o For one option go ahead with salicylates -
IAP 745
.In USA, the pil, has approved aspirin, tolmetin, Naproxen and lbuprofen for usage in children" ' -

rlr
FL&JXM &
KX,ECTKffi&YYK

. The major component of body mass is water.


r The contribution of water to body weight varies with age.
r Total body water (TBW) body weight declines form as high as 90% in early fetal life to 75-80o/o at
as a percentage of

birth, Thereafer it delines progessively to 600/o by the end of one year. After that it remains constant.
o Distribution of water in diferent compartment of body is :-

Total bodyWater
t 600/o ol body weight (Pct07,04)

lntracellular fluid Extracellular fluid


o 4Oo/o ol body wei ght r/vEFr) o 2Oo/o of body weight PGt 04,ate7)

. 670/0 of total body water . 33olo of total body watgl rntrzt

lnterstitial fluid PIasma


o 150/o of body weight(Pcl07) o 5o/o ol body weight PGt07.04)

.25% oftotal body water r 870 of total body water


t 75o/o of ECF c25o/o of ECF

e Most oUuiaont ion (cation) in ECF is Na*, and normal serum sodium concentration is between 135-145 mEq/L,
Other predominant ions in ECF are Cl , HCO, and Ca*2.
o Most abundant ion (cation) in ICF is K* and normal serum concentration of K. is only 3.5-5.0 mEq/L. Other pre-
dominant ions in ICF are Mg2t, and organic anions.
tilater requirrnent in a norrnal child
r Water requirment of a child depends upon the weight of the child -
: Weight 3 10 kg : 100 ml/kg/day. For example a 10 kg chilil requires 1000 ml water per d.s,yliP""' tt; (100x10).
t Weight 10-20 kg: 100 ml/kg/day for Jirst 10 kg (i.e. 1000 ml) plus 50 ml/kg/day for every kg above 10 kg. For example
l5 kg child requires 1250 ml of water per day (1000 ml for first 10 kg and 250 for next 5 kg).
t Weight > 20kg: 100 ml/kg/day for lirst 10 kg (i.e. 1000 ml), plus 50 ml/kg/day for next 10 kg (i.e. 500 ml) plus 20
ml/kg/day for every kg above 20 kg. For example, a 30 kg child requires 1700 ml of water per day@re ott [1000 ml
for first 10 kg plus 500 ml for 10-20 kg plus 200 ml (20x10) for 20-30 kgl.

DITIYDBATIOil
r Dehydration is a conditon that occurs when the loss of body fluid, mostly water, exceeds the amount that is taken in.
o Most common cause of dehydration in children is diarrhea. Large amount of water and water soluble nutritive sub-
stances such as electrolytes, metabolites and vitamins are lost from the body during diarrhea episodes. Loss of water
from the body causes a reduction or shrinkage in the volume of ECF.
o Depending on the concentration of sodium, this ECF reduction can be classified into -

1) Isonatremicdehydration
r Sodium loss is parallel to water loss so that sodium concentration in ECF or plasma remains normal ( - i40
meq/L).
'. It occurs in about 50% of diarrheal cases ECF decreases and ICF is normal.
2) Hyponatremic dehydration
r Sodium loss is more than ECF loss It occurs rn 40-45o/o of diarrheal cases. Because of low osmolality (due to
reduced sodium) water moves from extracellular to intracellular compartment *) signs of dehydration (wrinkled
skin due to loss of elasticity).
t Thus ICF increases and ECF decreases(AllMs0o).
t This type of dehydration occurs when water intake is inad.equate1umsoo)
3) Hlryernatremicdehydration
r In about 5% of cases, especially in chiid who has been given fluids with more salts, serum level may be elevated
(> 150 mEq/Lteet o:t',,. Water moves from inside the cells to the extracellular compartment(Pct
03).

t Thus, ICF decreases and ECF remains normalQcros). This may mask the physicalfindings@ctos) of dehydration
and the patients may be undertaken as mild dehydration even in the presence of severe dehydration -+ com-
plications of severe dehydration may occur due to delay in management.
t So, hypernatremic dehydration is more dangerous(Perx).

Assessment and treatment of dehydration

Ey€ti.r.-i -':'r:: : ,Nonnal. . '... .,. ':, :.Sunken,,:.. IsJafrd.,.drlr.1.i..

Tears Present Absent


Mogth,$,;-1q.uglr* :MOiSt.,:",':r.: :::::'. ..'rBrY.",':i.: .'

Thirst Drinks normally, Thirsty, drinks eagerly Drinks poorly, or not able to
not thirsty drinkrA"5'
I ::i:ii:':::r"
Ski6.?inch "'i:,,:, :::':':
Girei'backtiiitkllel , Goatbatk5lol.ii$!,+*x*soot ir::
cgeslhag,.k,,.*€firtlow.$r!a?.mlt{et-,.
02 AllNE$ol . :.. .... | ..,.,.:

Decide The patient has lf the patient has two or more lf the patient has two or
No signs of signs, including at least one sign, more signs, including at least
Dehydratisnfiroz there is Some DehydrationII'02, one signl there is Severe
ailMs 00) ailM' 0o) 02' Att[s oo).
Dehydrationrfl
Ttiat ,:',:tr:t'i UieTreatment Pltn :
Weight the patient, Treatment Plan B(' Weigh the patient and use Treatment
g(iln4Aflil4s.tb): ,. 'i,, ., o2,Attfrisoo) plancwoz^ttfts@, Ufgently
Water requirrrent in a normal rhild
r Water requirment of a child depends upon the weight of the child -
t : 100 ml/kg/day. For example a 10 kg chilil requires 1000 ml water per day11ip"u tt) (lQ|x70).
Weight < 10 kg
:Weight10-20kg:100 ml/kg/dayforfirstl0kg(i.e.1000mI)plus50ml/kg/dayforeverykgabovel0kg.Forexample
15 kgchild requires 1250 ml of water per day (1000 ml for first 10 kg and 250 for next 5 kg).
: Weight > 20kg: 100 ml/kg/day for first 10 kg (i.e. 1000 m1), plus 50 ml/kg/day for next l0 kg (i.e. 500 ml) plus 20
ml/kg/day for every kg above 20 kg. For example, a 30 kg child requires 1700 ml of water per day(orc 0t1 [ 1000 ml
for first 10 kg plus 500 mi for 10-20 kg plus 200 ml (20x10) for 20-30 kgl.

DE}IYDRNTION
r Dehydration is a conditon that occurs when the loss of body fluid, mostly water, exceeds the amount that is taken in.
a Most common cause of dehydration in children is diarrhea. Large amount of water and water soluble nutritive sub-
stances such as electroly'tes, metabolites and vitamins are lost from the body during diarrhea episodes. Loss of water
frorn the body causes a reduction or shrinkage in the volume of ECF.
r Depending on the concentration of sodium, this ECF reduction can be classified into -

1) Isonatremicdehydration
r Sodium loss is parallel to water loss so that sodium concentration in ECF or plasma remains normal ( - 140
meq/L).
r It occurs in about 50% of diarrheal cases ECF decreases and ICF is normal.
2) Hlponatremic dehydration
r Sodium loss is more than ECF loss It occurs in 40-450/: of diarrheal cases. Because of low osmolality (due to
reduced sodium) water moves from extracellular to intracellular compartment -) signs of dehydration (wrinkled
skin due to loss ofelasticity).
t Thus ICF increases and ECF decreases@IlMs00).
t This type of dehydration occurs when water intake is inadequate(drrusoo:t
3) Hypernatremicdehydration
r In about 5% of cases, especially in chiid who has been given fluids with more salts, serum level may be elevated
(> 150 mEq/L{tct ost1.
Water moves from inside the cells to the extracellular compartmen{Pct 0s).

t Thus, ICF decreases and ECF remains normal(Pcr 08). This may mask the physical findings@er ot)
of dehydration
and the patients may be undertaken as mild dehydration even in the presence of severe dehydration -) com-
plicatlons of severe dehydration may occur due to delay in management.
t So, hypernatremic dehydration is more dangerousqclet).

ArsessnNent arNd treatrnent of dehydration

NOfmAl 'r::.::::r: ir:: r: u.:: .


EY€S' :".::.'.':,' ..'1
"::' '5unken,,
Tears Present Absent
'Mout| &':tougrc' Moist:' ,:' r :: ,,.-Dfy,,,,' ',

Thirst Drinks normally, Thirsty, drinks eagerly Drinks poorly, or not able to
not thirsty drinkrA,15,

SkinPinch' :',: :' Goej bac*'qljikly4 Goeshi'ktlittflfylete:-;rosoot :: ; r::::' '' caa; lia+,viry
'r: 5to-,wlyrl11?r{,3i@,,.- r

6ZAtl{llOaJ:'.' :r. :r'i'

Declde The patient has lf the patient has two or more lf the patient has two or
No signs of signs, including at least one sign, more signs, including at least
Dehydrationrfloz there is Some DehydrationrA'oz one signs, there is Severe
AilMS 00) AilMS 00) 02' attMs oo).
DehydrationrAt
Treat: ..:..'rr''':): ::. Use lreatm.ent P.lan' Weight the patient, Treatment Plan BGt Weigh the patient and use Treatment
hw3zAttMsoa)
: ,,. 02, AUMS$o) plan C(At
oz AnMs&)
U rgently
Note: Skln pinch is less useful in infonts or children with marasmus (severe wosting),
Kwashiorker (severe malnutrion with edema) and obesity'Ato2,Nano6)

Mothers educated Rehydralion_Ihercpy: Start lV fluids immediatelyrr rsr

to use increased Correction of existing water and electrolyte defecit Best IV fluid sotution is Ringer tactatekrahsl)
amounts of home
75 ml/kg ORS in first 4 hrluwe7) Normol saline can be usedtatt$sot)
available fl uids(/'P'"'
9s) Maintenance Therapy: Dextrose is not effective

ORS Packslsrannerest Replacement of ongoing losses due to continuing lOO ml/kglPcts3t is to be given as shown below:

given for use at home diarrhoea. AGE FIRST THEN


Breast feeding should
es)
Begins when signs ofdehydration disappear usually <12 months 3A ml/kg in Toml/kg
be COntinUed(ripfrer
in first 4 hours t hOufSGuPGEEoll 5 nours

12 months to 5 yrs. 30 minutes 21/2hrs


10-20 ml/kg ORS for each liquid stool.

r Inseveredehydration,ifintravenouslineisnotpossibleorfailed,intraosseousaccessintheproximaltibiaispreferred
qlternative in children less than 6 years of age(AIIMS 03' 0r). In children > 6 years, percutaneous femoral line should be
attempted.

Oral rehydration sotrutiolr {ORS}


c Because of the improved effectiveness of reduced osmolarity ORS solution, WHO and UNICEF are recommending
that countries manufacture and use the following formulation in place of the previousiy recommencied LIR.S sollltioi].

2.6(PGl02,uPsc
1o) Sodium 75iAr
15, pcl es,8e)

.'.;6eu..*;
.i.::li3.5inlla,6tiiiti ::,ii.
;i#iffi ., .

I.s(PGr02,uPsc10) Chloride 65{pcre5,se)

c Oral fluid therapy is based on the observation that glucose given orally enhances the intestinal obsorption of salt
and water, and is capable of correctingthe electrolyte and water deficit.

ReSoMal (Rehvdnatlon Solution for Severely hltalnourished Child}


r This represents the oral rehydration fluid recommended by WHO for severely undernourished children. These chiidren
are deicient in potassium and have relatively high levels of sodium.
r ReSoMal solution is therefore specificall;, designated with a lower sodium concentration and higher potassium con-
centration to meet the electrollte needs of these children.

Glucose 125
l
-:.Srditi*r.' 45IAl{.6Jr

Potassium 40
Chloride,.' :r:,70.:..::::

Citrate 7
I a'

0.3
l..6:66.,.2
300arI3)

HYPOVOLEMIC SHOCK

o Hlpovolemic shock in children may have following stages : -


i) Eaily compensated
r Immediately after hlpovolemia, body tries to maintain the BP to maintain adequate perfusion to vital organs
through a compensatory mechanisms.
t An increase in heart rate (Tachycardia) is the eailiest and most sensitive indicator for intravascular volume
10, AI 0e)
redht1tion@ilMs

ii) Late uncompensated


r If shock state continues or the compensatory mechanisms are not enough to maintain the metabolic needs of
the tissue, the shock, goes into uncompensated phase.

* Bloodpressure Normol(AtMto)(Orthostatic Hypotension

r RespiratoryRate TachYP nea (mil dltarws r o1


Marked tachypnea (due to acidosis)

Cool extremities with

ELECTROLYTE IMBALANCE

r Dietary requirment of sodium in children is 2-3 mEq/kg body weight per day and dietary requirment of potassium
13),
is 1-2 mEq/kg.body weight per dayal

Hyponatremia
o Hyponatremia is defined as serum concentrationofNa + < 130 meq/L Hlponatremia causes a decrease in the osmolality
ofECF.
r Because the intracellular space then has a higher osmolality, water Inevitably moves from the extraceilular to intracellular
space.
r Increase in intracellular water causes cell to swell. i
!

o Although cell swelling is not problematic in most tissues, it is potentially dangerous in the brain because it is contained l

in the fixed shell the skull. As brain cells swell, there is an increase in ICT. So brain cells swelling is responsibe for
most of the symptoms of hyponatremia.
t Anorexia, nausea 6 vomiting t Coma

1 C onv ulsiont(Kerata
e7' PGI 8e) s Drowsiness
: Headache t Circulatory failure and hypotension(Pcl 8e)
Hyponatremia can also cause muscle cramp and weukness6"'d"e7PGI8e),
PGI I

c Central pontine myelinosis is classically associated with overlay rapid correction of hyponatremia.
r Treatment of hlponatremia is :-
1) In low ECF volume & asymptomatic-give ORS/Normal saline.
2) In complicated like seizure-give 3-5 ml/kg of 3% hypertonic saline to raise Na by 5 meq/Lit(DNB
cases I5).

3) Never do rapid correction (> 12 meq/Lit in 24 hour) can cause Central pontine myelinolysis (CPM).

Hypernatremia
I Hypernatremia is defined as Na+ Concentration > 150 meq/L Most children with hypernatremia are dehydrated and
show tlpical clinical signs and symptoms.
o Children with hypernatremic dehydration tend to have better preservation of intravascular volume because of the
shift of water from the intracellular to extracellular space. This maintain blood pressure and urine output(KerataeT),
and allows hypernatremic infant to be less symptomatic initially and potentially to become more dehydrated before
medical attention is sought.
r Probably because of intraceilular water loss, the pinched abdominal skin of a hypernatremic dehydrated infant has a
"doughy" feel(uPsc 8e),
o Hyperntremia causes CNS symptoms due to neuronal shrinkage (water shifts out of the cell) : -
I Lethargy(r"'"totz) t Restlessness
: High pitched cry and hyperpnea ; Fever
: lrtitabilitY(uPsc e8' Keruta e7)
T Weakness
a Nausea
o Hlpernatremia is associated with hypoglycemia and mild hypocalcemia.
o Brain hemoruhage is the most devastating consequence of hl.pernatremia -+ may be subarachnoid, subdural or
meningeal.
o There is strong correlation between cerebral plasy and history of neonatal hypernatremia.
o Seizures and coma may occur, seizures are more common during correction of hypernatremia.
o Although central pontine myelinosis is classically associated with rapid correction of hyponatremia, it can occur in
children w ith hy p er natr emi a.

Hypokalemia
r Hypokalemia is defined as serum potassium values of less than 3.5 meq/L,Important causes are :-
1) Excessive Loss in GIT or kidney : Diarrheaqlel),laxative :use, RTA (distal or proximal), Gitelman syndrome, Bartter
syndrome, cushing syndrome@IlMse3) (due to adrenal adenoma or hyperplasiaale4)), hypomagnesemia
2) Drugs : Diuretics (thiazide(Ale4)/furosamide), cisplatin, aminoglycosides, amphotericin, p-agonists, theophylline
3) Others: Alkalosis, spurious hypokalemia in leukemia, anorexia nervosa.
o Symptoms are non-specific and are related to muscle (muscle weakness) and cardiac function (cardiac arrhphrnias).
o ECGclranges areprominentUwave(eailiest),JlatteningandinversionofTwave,prolongedPRinterval,wideQRs
complex, QT prolongation and depressed ST segment.

Hyperkalemia
r Hyperkalemia is defined serum K* concentration > 5.5 meq/L.Important causes of hyperkalemia are :-
as
1) Decreased renal exceretion : Renal failureal e4' AIIMS e3), Addison disease, CAH (2l-hydroxylase and 3 B-I-{SD
deficiency), ACE inhibitors/AT II antagonists, NSAIDs, potassium sparing diuretics, heparin, trimethoprim,
pentamidine, cyclosporine.
2) ' Trasnscellular shifi: AcidosisqllMse3), rhabdomyolysis, tumor lysis syndrome, insulin deficiency{AttMs,{ digltals,
'succinylcholine, malignant hyperthermia, exercise.
3) Others: Blood transfusion, spurious high value (hemolysis, thrombocytosis, leukocytosis).
o Patients may present with nausea, vomiting, paresthesia, muscle weakness, fatigue and ileus. ECG changes show tall
tented (peaked and natow) T-wave, prolonged PR interval, wide QRS complex, flat/absent P waye and ST-segment
depression.
Fluld &Ele*rolyte

Daily water requirment of a child weighing t 1kg : IOOO ml.


a Diily,water tequirment ih a chili weighing 3Akg:.1 700 mt.
, Mild tachycardia, mild tachypnea, slightly delayed copillary refill with normal BP:Early compensated shock.
t Merked.tacbyiaidia,marked,tat{rypnea, a1d mqrkedly d€leyed capittary,refrftwith hypotensrbn; LaG unc.smpensated;haak ,. ,

a Most dangerous dehydration : Hypernatremic.


* Hypernatremic.dehydrqtianischarac%kedby:Maskingofsignsofdehydration. : : :' . l

a A child with diarrhea and vomiting with inadequate water intoke: Extracellular hyponatremic dehydration.
, Earliest and most seirsitive indicator for decrease:in intravascular,volume;lincreased heart:rate. : :

t Skin pinch goes back immediately :No dehydration


* Skitt piiih goes ba,ikslawly $ut withirt 2 secand,s): Some dehydration.
a Skin pinch goes back very slowly (> 2 seconds): Severe dehydration
*
t Trreatment for severe dehydration : Plan C, i.e. 100m|/kg iv fluid.
{ EdstfluidWseveredehydration:Ringer: lactaie(norrna! satine
Ringer,s,lacfate notavailable).
is 2nd chcjice,if
.a Compositonof ORsperlitre: Nacl:2.6Em; glucoseanhydrous: 13.5gm; kcl:1.5gm;Trisodiumdihydrate: 2.9gm; sodium:75 mmol;
potassium 20 mmol; chloride: 65 mmol; citrate: 10 mmol; glucose: 75 mmol; osmolarity: 245 mmol.
!0 Sodium contents of ReSoMol :45 n mol/1.
o Omolarity of ReSoMal :300 n mol/l.

'l}
t Daily recommended dietary intake of potassium in a child:1-2 meq/kg.
t Acute hyponaffemio does nafeause,lDianhea',:.':'r.
, Convulsion in a child with delrydration is due to..Hyponatremia.
* Signs,ofhypernatremicdehydrailoniinitabiJity, lethaigy,,highpitchedcry,restles5ness.Doughf.J*in.,r,,,
* Hypokalemia is not seenin : Acute renal failure.
i' Hyperkalemia is not seenin : Cushingt syndrome.
o Treatment of vomiting with metabolic alkalosis in a child : lV normal saline with potassium.
* LatemEtabaticacidasis'isseenln:.Pretermbabyoncow,milk. ' . ., .',:,,,,:,.1, ._,,

, Most commonty used lV fluid preparation for neonate..N/5, 1 0olo dextrose.


& Route af choice for lV fluid in a child'16'yeariif lV line ii unsuecessful: lntraossious route in proximat tibii;

III
SUESTIOHS

kge, 9. 3 yearold child come in ER with H/ovomi'tin5loose


Daily w&cr requirement in child weighing3O
wate;, motion firr 3 cfays- $n exarninatiotu ehild was
height l23mxndts$A eif I m'?is- (Delhi PG Mar.09)
b) 17oo drowsy, sunliicn eye. Hy@hermia and *kin pinch
a) 1300 ml m1
d) 2s0o ml take tirne to revert bach diagnosis-
c) 2000 ml (All India Dec15 Pattern)
2. &faintenance fluid for a child weighing f 0 hg - a) No dehydration
(All lndia Dec.14 Pattern)
b) Mild dehydration
a) 1500 ml/day b) 1000m1/daY c) Some dehydration
c) 750m1/daY d) 500m1/day
d) Severe dehYdration
3. Which of the following would be the plasma osmola- 10, with diarrrhoea, skin pinch
4 year old giri wt" 12kg
tity of child with plasma Na' 125 m&qlL'glucose of pesbaekiery slowty & she is not ableto drink- false
l0bmg/dl, and blood urea nitrogen (BUN) af 14$ mgl is - (CET Nor' 15 Patt'rn)
(AllMS MaY 05, Nov 03)
dl- a) Develop severe dehYdration
a) 360 mOsm/kg b) Require plan C treatment
b) 306 mOsm/kg c) Fluid of choice-ORS
c) 3l2mOsm/kg d) Require hosPitalition
d) 318 mOsm/kg
tl. A Syear old boy passed 18 loose stools in last 24
4. An alert 6 rnonths old child is brought with vomit- hours andvomited tlyice in last 4 hour' He is irrita-
ing &diarrhea. RR-45lmin, HR-130/min' SBP-85 ble but drinking fluids. The optimal therapy for this
mm of Hg. Capillary refilling time is 4 secs' Diagno- child is-
(AllMS MoY 10)
s$rs- a) Intravenous fluids 6I a3)
a) Early compensated hlpovolemic shock
b) Oral rehYdration theraPY
b) Early decompensated hypovolemic shock for 4 hours followed by
c) Intravenous fluicl initially
c) Late compensated hypovolemic shock oral fluids
d) Late decompensated shock due to SVT
d) Plain water
5. The most sensitive indicator of depletion of intra- Treatment in 6 month old child with acute watery
12'
vascular volume in infant is- diarrhoea without signs of dehydration is -
a) Cardiac outPut (AIIMS MaY 10' AI 09)
a) Mothers milk and household fluids ]IPNIER es)
b) Stroke volume b) ORS and antibiotics
c) Heart rate c) Mothers milk and antibiotics
d) Blood Pressure d) Mothers milk and ORS
L3. A- 3 month old female infant weight 4 kg and is
DEHYDRATION sufiering from loose motions' On examiantion she
is founJto be suffering from some dehydration'The
6. Most dangerous dehy'dratianis ^ (All lndia Dec'14 Pattern)
amount of ORS to be given to her in the first four
a) HYPonatremic hour will be - (UPSC e7)

b) HYPernatremic a) 100 ml b) 300 ml


c) Isonatremic c) 500 ml d) 600 ml
d) Non-diarrhoeal cause
t4, When severe dehydration in a neonate occurs,
7. In a patient who has diarrhoea and vomiting with amount of fluid replacement in l't hour- (CUPGEE01)
inadequate water intake is sufiering from- a) 20-40 ml/kg b) s-10 ml/kg
(AtlMSlune2K)
c) i0-15 ml/kg d) 15-20 mlikg
a) Intracellular dehydration with hlpernatremia
15. A 4 kg infant with severe dehydration(l07' dehvdration)
b) Intracellular dehydration with hlponatremia needJthe following amount of intravenous fluid in
c) Extracellular dehydration with hlponatremia ]IPMER 80, PGIS-3)
the first 24 hours -
d) Extraceliular dehydration with hlpernatremia b) 800 ml
a) 500 mi
8. Kallu 2yrs child weighing6'7 Kg presents in the c) 100 ml d) 1200 ml
casualty with history of vomiting & diarrohea for e) 1400 ml
last 2 days. On examination skin pinch over the ant"
A child having F{/0 profu*e watery diarrhoea not
abdominal wall go quickly to its original position'
taking *rally and not p'assed urine sim'ce 2 days' what
Interpretation oi tki,, - pinch test in this chitd will (All lndia Dec15 Pnttern)
to be given -
be-
a) Milk
a) No dehYdration 61 02)
b) oRS
b) Some dehYdration
c) I.V. fluid
c) Sever dehYdration d) I.V. antibiotic
d) Skin pinch can not be evaluated in'this child
A child suffering from acute diarrhoea is brought (Karn 95)
to the casualty and is diagnosed as having severe a) Mild dehydration b) Moderate dehydration
dehydration with pH of 7.23. Serum Na-125, Serum c) Some dehydration d) Any dehydration
K-3, HCO3- 16 The best I"V. fluid of choice is -
a) 3% Saline (AIIMS May 01)
POTASSIUM & SODIUM
b) N/3 Saline + 107o dextrose
c) Normal saline 29" The reqtrirernent of pntassium in a child is -
d) N/3 saline + 5% dextrose
(All lndia Dec.1j Pattern, AI 06)
18. Treatment of choice in severe dehydration is - a) r-2mEq/kg b) 4-7 mEq/kg
(Kerala 94) c) 10-12mEq/kg d) 13-14mEq/kg
a) Isolyte-P b) Normal saline 30. Acute hlryonatremia in 8 month old infant may
c) Plasma d) Ringer lactate
produce all except- 1tpMERZs, pct s0)
19. WHO ORS contains - (PGI lune 02) a) Diarrhoea
a) Sodium chloride 2.5 g b) Conr,.ulsions
b) Potassium chloride i.5 g c) Muscle cramps and weakness
c) Glucose 20g d) Vascular collapse
d) Shift of water from ECF to ICF
31. Convulsions in a child with dehydration and vomit-
?&. 5year old chils{ with watery diarchea fer 7 days, on ing can only be due to- (Kerala 97)
exa**natian, weight * X0 kS, hangi*g skin f<rlds a) Decreased serum sodium
wit& trorma} skin pimeh" Soqliurn in slrs should be - b) Decreased serum magnesium
(CET Nov 15 Pattern) c) Decreased serum potassium
a) meq/Lit
45 b) 60 meq/Lit d) Decreased serum potassium
c) 75 meqlLit d) 90 meq/Lit 32, A S-month old formula fed infant has been brought
t t" [,ors $sr&&larity {}fl.$ - fa]se statment is - with complaints of watery diarrhoea of 2 days
(All lndia Dec15 Pattern) duration and irritability of one day duration.He
a) Glucose concentration 75 mmol/liter had been receiving WHO ORS at home. Physical
b) Sodium concentation 75 mmol/liter examination reveals a markedly irritable child with
c) Osmolarityis 311 mosm/liter a rather doughy skin and rapid pulse.The most
d) Postassium is 20 mmol/liere likely diagnosis is- (UPSC e8)
Z?* Os*r*larityofR.e$o&{at * @ilrndiaDec.l3pattern) a) Meningitis
a) 200 b) 300 b) Encephalitis
c) 311 d) 3s0 c) Hyponatremicdehydration
23, The sodium content of ReSoMal (rehydration solution for d) Hypernatremic dehydration
malnourished children) is - (Ar 06) 33. Child with seizure plus hyponatrernia treatment of
a) 90 mmol/L b) 60 mmol/L choice - (CET Nov 15 Pattern)
c) 45 mmol/L d) 30 mmol/L a) 0.9% NaCl b) % 0.a5% NaCl
24. The function of glucose in ORS - (PGI lune 96)
c) Dextrose + NaCl d) 3%o Nacl
a) Increase Na+ absorption by co-transport 34. Hypokalemia in an infant maybe due to all of the
b) Gives sweet taste of ORS following except - (At e4)
c) Increase Na+-K+ pump activity a) Adrenal tumor b) Acute renal failure
25. Sodium content in mmol/L in WHO oral rehydra- c) Thiazide therapy d) Diarrhea
tion solution is - (Karn 11) 35. Hlperkalemia in children may occur in all except -
a) 20 b) 7s (AIIMS Nov 93)
c) 90 d) 111 a) Insulin deficiency b) Metabolic acidosis
26. n&&at ie t"he amount of trisodium citrate dihydrate in c) Acute renal failure d) Cushings syndrome
WHO OR.S? (All tndia Dec.14 pattern)
a) 1.5 gm b) 2.s gm MISCELTANEOUS
c) 2-9 gm d) 3.9 gm
)'7 In malnourished children the following parameters 36. A child presents with peripheral circulatory failure.
are reliable indicators ofdehydration except - (Karn The arterial pH is 7.0, pCO, of 15 mmHg, pO
176
06) mmHg. Which of the following will be the immedi-
a) Skin turgor b) Dry buccal mucosa ate therapy - (AIIMS Not 04,05)
c) Oliguria d) Thirst a) Sodium bicarbonate infusion
28. As per the latest guidelines which of the following
b) Bolus of Ringers lactate

dehydration status requires ORT prescription -


c) Bolus ofhydroxyethyl starch
d) Dopamine infusion c) <6yearsage d) Anyage
37. A child had repeated vomiting and developed meta- 42. A breast fed child presents with hypernatremia
bolic alkalosis. The treatment given is - (Serum sodium > 170m Eq/L). His urine sodium is
a) Ringer lactate (AttMS lune 99) 70 mEqll,. Which of the following is the most likely
b) I.V. normal saline and potassium (AIIMS Nov 2000)
c) ORS a) Diabetes insipidus
d) I.V. Normal saline b) Acute necrosis
38. Late metabolic acidosis is seen in- (AttMS May ea) c) Severe dehydration
a) Term infant given formula feed d) Excessive intake ofsodium
b) Preterm baby getting cow milk 43. Howwill a staffnurse prepare normal saline from
c) Long term breast feeding 10% Dextrose (per 100 ml) - (AllMS Nov t0)
d) None ofthe above a) 20 ml 10% D with 80 ml NS
39. A 10 days old neonate is posted for pyloric stenosis b) 60 ml 10% D with 40 ml NS
in surgery. The investigation report shows a serum c) 60 ml 10% D with 40 ml NS
calcium level of 6 mg/dl. What information whould d) 80 ml 10% D with 20 ml NS
you like to know before you supplement calcium to 44. Fluicl of choiee in ehilql with burm < ?4 laclwr is -
this neonate - (AIIMS Nov 04) a) Fresh frozen plasma (All lndia Dec15 Pattern)
a) Blood glucose b) Serum protein b) Isolye-P
c) Serum bilirubin d) Oxygen saturation c) Ringer lactate
40" A 6 months old child having severe dehydration d) Platelettransfusion
comes to the casualty with weak pulse and unrecord- 45. 0"97o Na*l aomtain$ True as- (All India Dect; Pattern)
able B.P. Repeated attempt in gaining I.V. aceess has a) 0.9 gm of Nacl in 1000 ml of fluid
failed- The next best step is - (AIIMS May 01) b) 77 megof sodium is 1000 ml of fluid
a) Try again c) l54meg of chloride in 1000 ml of fluid
b) ]ugular vein catheterization d) 30 meg of sodium in i000 ml of fluid.
c) Intra osseus I.V. fluids
d) Venesection
41- In Pediatric advanced life support, intraosseous ac-
cess for drug/fluid administration is recommended
for pediatric age of- (AIIMS Nov 02)
a) <lyearage b) <syearsage
ANSWERS
l. Ans. is 'H i.e., 1700 ml lRef. O.p. Ghai 8th/e p. 72 6 Vh/e p. 511
r The child in question has weight of 30 kg.
o So, fluid requirment will be :-
i) 1000 ml for first 10 kg Plus
ii) 500 ml for 10-20 kg Plus
iii) 200 ml (zo x 10) for 20-30 kg
o Total requirment will be 1700 mL

) Ans. is'b' i.e., 1000m1/day {Ref: O.P. Ghai */e p. 72 $ th/e p. 51)
o Up to 10 kg, the requirement is 100 ml/kg/day.
r So, Maintenance fluid for a 10 kg child is 1000 ml/kg/day.

3. Ans. is '-o" i.e., 306 mosm/kgfRef : Ganong 21't/e p. 6]


Osmolality: Z (Na-) + .055 [Glucose] + .36 [BUN]
(mOSM/L (meq/L) (mg/dl) (mg/dl).
t Replacing values given in the question.
Osmolaity =2 (125 meq/l) + .055Xl08mgldL+.36 X 140mg dL
= 306 mOsm/L.

+. Ans. is'd i.e., Early compensated hypovolemic shock


r BP of the child is normal (at the age of 6 months, normal systolic BP ranges from 80 to 90 mm Hg).
o Heart rate is increased (normal is 80-120 per minute) + Tachycardia.
r Respiratory rate is increased (normal is25-40 per minutes) *> Tachypnea.
o Capillary refill time is increased (normal is < 2 secs).
r Therefore, this child is having early compensated shock.
5. Ans. is t' i,e., Heart rute fRef: CPDT
18th/e p. i75; Practical pediatrics Sth/e p. 162]
o An increase in heart rate is the earliest and the most sensitive indicator to any reduction in intravascular volume.
r Cardiac output and blood pressure are preserved initially because of compensatory mechanisms such as increase in
heart rate and peripheral vasoconstriction.
r Stroke volume is usually preserved in infants and compensatory increased in cardiac output is primarily achieved by
increasing heart rate.

DEHYDRATION

6" Ans. is 'b" i,e., Ilyperna*ernic {Rqf: A.P. Ghai 8e/e p. 7 j-76 (r 7e/e p. 261)
t Hypernatremic dehydration is more dangerous.

Ans. is t'i.e., Extracellular dehydration with hyponatremia fRef O.P. Ghai 9th/e p. 73-76 d2 7h/e p. 261; Nelson
17h/e, p 199-201, 1200, 18567
o Extracellular fluid consists of
t Interstitial (intercellular fluid)
t Circulating blood
t IntestinalJluids and secretions
r lntracellular fluid consists of fluid present within the cells.
o Loss of fluid in diarrhoea and vomiting comes from extracellular fluid. Therefore there is shrinkage or reduction in
the volume of extracellular fluid.
o Diarrhoea and vomiting both cause loss of Na* rich Jluid therefore there is decline in the serum sodium level
(hyponatremia).
r This causes fall in osmolality of ECF leading to shifting of Jluid from ECF to lCF causing further decrease in ECF
volume.

8. Ans" is 1d'i.e., Skin Pinch can't be evaluated in this child tnf O.p. Ghai Bth/e p. 7j 6 Th/e p. 2641
Frist step in assessing the skin pinch requires status of malnutrition of child as skin pinch is less useful in infants or
children with + Marasmus (severe wasting). Kwashiorker (Severe malnutrition with edema). Obese children
Child in the given question is 2 yrs old with the expected weight being 10 kg. But patient weight 6.7Kg. and so he is
certainlymalnourished. Skin pinch in such a malnourished child will not be able to evaluate the status of the dehvdration
correctly.
9- A*s; is ? i.e-,- $ev'ere dehydration
o Information in question are : (i) drowsy child, (i1) snuken eye, and (i11) skin pinch take time to revert back. These are
signs of severe dehydration. Treatment plan is C.

1&- Ane. is. s* i-e., Fleid of ehoicc-O,RS lRff Gbai 7e/e ch. 244!
11. Ane. i$ 'lf ie", oral rr&ydration therapy IR"f oP" Gh*i *fc p" zi & ple B" 264-26s1
The chilil in question is irritable but drinking fluids, He thus falls in the category of 'Some dehydration' and needs
to be managed in accordance with PLAN B.
Treatment plan B is oral Rehyilration therapy and hence the answer here.

12" 3 i.e., *Iothers rnilk and household ftrids, 'd'i .e", A{o[&ers mit& ard OB$ tQ.ef: O"E GEwi *le
Ans" is p. VS
e Pfep 26al
fremnent flan to prwent detrydreliofi - ?bn &
The mother should be educated to use increased amount of culturally appropriate home available fluids.
In addition, they should be given ORS packets for use at home.
Breast feeding should not be discontinued.

< 24 months 5.100 ml

) l0yr As much as wanted 2000 ml/day

13, Axe" is'b' i.e., iXK ml t&"f A.p. Qlsoi *le p" ZJ & Ple B" 265|

Grddtrines {or rreating patfents witt} so{ne dclqdration {&trt not scverc defrfdratiom} Ueaeme*G plan E
c Basic is to give 7 5 ml/kg of ORS in the first 4 hours.

<5kg 200-400 ml
5-8 kg 400-600 ml
8-11 kE 600-800 rnl
1 1-16 kg 800 - 1200 ml
16-30 kg 1200-2200 ml
>30k9 > 2200 ml

14" An$" is 3 i.e,, 2$-41 mUkg lW A"p" Qtrwi *le p zZ & Pte p. 265|
Nntravenous fiuid 6erapy for severe de@rdt$on - &eaGment plam C
o Give i00 ml/kg of the chosen fluid as follows

< 12 months 30 ml/kg in t hour 70 mllkg in 5 hr


12mt - 5 yrs 30 ml/kg in 30 minutes 70 ml/kgin 2vzhr.

t5" An*. is'a'i-e., SX! mI tR* A.p" 6kai */e p" zL-r3 & *fe p" 26sl
c Initially 100 ml/kg of fluid is given.
c So, fluid required = 100 x 4 = 400 ml.
16. Ans. is t'i.e.,I.V fluid
r Child in question is not able to drink (not taking orally) -+ Plan'C'treatment (immediate IV fluids).
11 Ans" is t' i.e., Normal saline {Ref: O. P. Ghai Sthle p. 73 (r */e p" 265)

Fk*id used in severe dehydration


* Best IV fluid for severe dehydration is Ringer'slactate.
* An ideal preparation would be Ringert lactate with 5o/o added dextrose, However, it is not available.
o If Ringer's lactate is not available, normal saline solution (0.9% NaCl) can be used.

18.

19" Ans. is'b' i"e., Potassium chloride-I.5 g1n; {Rsf O"P Ghai 6th/e p" 272 & Vhle p" 263; Park 2*/e p" 19vj

2.6 Sodium
...,29,rt,,.,'
'1.5
65
:,:. Zg :i.'[Or:,:.
75

2A. Ans. is t'i.e., 75 meq/Lit lRef: Freviaus ex,planations]


o Examiner just wants to know sodium content in commonly used reduced osmolarity ORS, which is 75 mmol/lit (meq/
lit).

21. Ans. is 'c' i.e,, Osmolarity is 3l I rnosrnlliter


o Osmolarity of low osmolarity ORS k 245

22. Ans" io 'b' i.e., 300 [Sd A.P. Glrai 6tble p. 109]
o Osmolarity of Re So Mal is 300.
Ans. is t'i.e.,45 rnmoUllRef O. P. Ghai*/ep. 109)
The sodium content of ReSoMal is 45 mmol/l --- Ghai 6'h/109

24. ,Ans.is'a' i.e., Increase Na+ absorption by co transport [Ref: O. P. Ghai 6'h/e p. 109)
e Orai fluid therapy is based on the observation that glucose given orally enhances the intestinal obsorption of salt
and water, and is capable of correcting the electrollte and water deficit'

Ans. is'H i.e., 75 {R.ef: OP Ghai Vh/e p. 263, table (10.7)1

26. Ans is t' i.e., 2"9 gm lRef: A.F. Ghai Vh/e p. 263)

27. Ans. is 'd i.e., Skin turger lRel O.P" Ghai */e p. 26a)

* Skin turger (skin pinching) is not helpful in : -


i) Merasmus ii) Kwashirkor ] Protein energy malnutrition
iii) Obesity

28. Ans. is t' i.e., Any dehydration [Ref O. P. Ghai */e p. 272-27j 6 7*/e p. 263]

poTAsSrrrM & soDluM

29. Ans" is 'a' i.e., 1-2 Meq/kg fRef: Nelson 18'h/e p, 2791

The recommended daily dietary intake of potassium in a child is 1 - 2 meq/kg


'Dietary consumption varies considerable even though 1- 2 meq/kg is the recommended intake' - Nelson 18th/279

30. Ans. is'a' i.e., Diarrhoea lRef: Nelsou. x}&/e p. 2771


o Brain cells swelling is responsibe for most of the symptoms of hyponatremia.
r Anorexia, nausea (t vomiting t Headache t Drowsiness
s Convulsions t Coma t Circulatory failure and hypotension
e Hyponatremia can also cartse muscle cramp and weakness.
o Central pontine myelinosis is classically associated with oveilay rapid correction of hyponatremia,

31. Ans. is'a' i.e., Decreased serum sodium lRef; See abave explanation]

32- Ans. is 8'i.e", lIypernatremic dehydration [Ref See abote explanatianl


o This child has -+ i) Irritability ii) Doughy skin iii) History of ORS taken at home
o Irritability and doughy skin is seen in hlpernatremic dehydration.
o Hlpernatremic dehydration occurs when fluid is given with more salts.
o This child must have taken more ORS than fluid i.e., concentrated ORS.
c About hyponatremic dehydration
r There is drowsiness (not irritability)
r Skin is dry inelastic (not doughy).

33. Ans. is tn' i.e., 3Yo NaCtr fRef: Nelsan n8d'/e ch. 52, 53; Gbai 7'/e p. 5)l
o In complicated hlponatremia like seizure-give 3-5 ml/kg of 3% hypertonic saline to raise Na by 5 meq/Lit.

34. Ans. is nH i.e., Acute Renal failure fRef: Nelsan l8"le p. 2821
c In acute renal failure, there is hyperkalemia. Adrenal tumor (causing Cushing's syndrome), diarrhea and thiazide cause
hlpokalemia.

JJ. ,{ns. is'd'i.e., Cushing's syndrorne fRef: Nelsan lSthle p.280f


o ARF, acidosis and insulin deficiency cause hlperkalemia. Cushing's syndrome causes hypokalemia.

MISCEI.LANEOUS

36. Ans. is 'H i.e., Bolus of Ringers lactate lRef: Harrkon 1&/e p. 265, 266; Nelso* l&h/e p. jAfi
It is a case of metabolic acidosis (lactic acidosis) due to peripheral circulatory fall:ure (It is a common knowledge that
in cases of peripheral circulatory failure metabolic acidosis occurs due to production of lactic acid).
In moderate or mild cases of metabolic acidosis i.e., when the pH is > 7 .2 metabolic acidosis needs no speciJic t/t.
T/T of the underlying cause is suficient.(it will correct the acidosis)
Once the underlying cause (here tissue perfusion) is corrected the pH will return to normal
But when the pH is < 7 .2 specific bicarbonate therapy is warranted because this degree of acidosis may depress cardrac
contractility.
According to Harrison
"In general severe acidosis (pH < 7,20) warrant the intravenous administration of 50-100 meq of NaHCO, over
30-45 minutes during the initial , 1 to 2 hours of therapy".
So here, besides fluid administration to correct tissue perfusion, HCO, transfusion is also required because pH is 7.
The question is which complication should be first attended to, peripheral circulatory failure or severe metabolic
acidosis.
Harrison states:
"The undeilying condition that disrupts lactate metabolism must first be corrected, tissue perfusion must be
restored when it is inadequate".

J/. Ans. is'b'i.e., I.V. Normal saline and potassiamlkef: Nelson 18th/e p. 3051

Treatment of metabolic alkalosis due to vomiting or nasogastric suction


o Most children with metabolic alkalosis have one of the chloride responsive etiology (e.g., vomiting).
o In these situations, administration of sufficeint sodium chloride and potassium chloride to correct volume deficit
is necessary to correct metabolic alkalosis.
o Adequate replacement of gastric losses of sodium and potassium in a child with vomiting can minimize or prevent
the development of metabolic alkalosis.
o With adequate intravascular volume and a normal serum potassium concentration, the kidney is able to excrete the
excess bicarbonate within a couple ofdays.

38. Ans, ic 'H ic- Pret nm baby getting cfir 'ryf,lilr<


{?ef Readw$*'l

|.at€ lncfiabolk addoCs (1*lA)


r Late metabolic acidosis is a special type of acidosis in apparentlyhealthy premature infant,
r This hlperchloremic acidosis appeares during second qnd third week of life, and resolves spontaneously within
a couple of weeks.

Definifion
'An acidosis occuring after second day of life, in which base excess values are lower than -Smeq/L on two
consecutive estimations done at least 24 hours qpart".

Efmdog!
o Prematurity is the most important single predisposingfactor in the development of LMA.
o The incidence varies considerably depending upon the gestational maturity and protein content of feeding formula.
. In preterm neonates fed on a modified cow's milk the risk of developing LMA is very hrgh.
9. Ans. i0 'U Le.,$crum protein {Ref Nelsor, tflle p. 205-2101
r Normal calcium level is 8.9-10.1 mg/dl (total calcium).
o Slightlyless than half of the total serum calcium exists in free or ionized form. Remainder is bound to protein (mostly
albumin). Ionized calcium is relevant for cell fuction.
"There are few clinical situations in which the total calcium is not an adequate surrogate
for the ionized Ca++
concentration. The most common and severe problem is the presence of hypoalbuminemia".
o Each 1 gm/dl of albumin in the serum binds about 0.8 mg/dl of calcium.
r A low total calcium concentration may be normal in a patient with significant hlpoalbuminemia.
r Now it is clear from above that a low level of serum protein lowers the total plasma calcium but not the ionized
calcium, So, before treating hypocalcemia, measure the serum protein level.

/10. Aas. iE'C ic.o Intra Osceuo I.y" fluids lRef: Love e Bailey 2*/e p. 290; 23d/e p. 2761
"If LV. route is unsuccessful, introasseus access in the proximal tibia of an uninjured leg is the preferred alternative
for children younger than 6 yrs. In children older than 6 yrs a percutaneous femoral venous line should be
attempted.

41. Ans" isu ie., < 6yre of age{Ref: Abo see above expianatian)
{2, Ans" is 'd i.c., f,xcessive intake of sodium ffuef: Ghoi @/e p" 97; Nelson 17/e p. 197-199)
o The child is having hlpernatremia. Serum sodium >170 mEqll- (Normal level is 1i5-145 mEq/L).
o Urine sodium is also very high, >70mEq lL (Normal urine sodium level in <20 mEq/L)
s This combination can be seen with excessive intake of sodium.
o With excessive intake of sodium there will be increase in serum sodium and excessive excretion of sod.ium in urine
(Kidney tries to compensate for increase in serum sodium by excreting large amount of sodium).
o Diabetes Insipidus --) Serum sodium conc. will be high but urine sodium conc. will be very low (Lack of
ADH leads to defect in concentration of urine).
o Acute necrosis
r Severe dehydration
--+ Urine sodium conc. will be high but serum sodium conc. will be low.
Urine sodium concentration will be low.
--)
43. Ans is'd" i.e", 80 mI 10% dextrose in 20 ml NS [Ref O.P. Ghai 7/e p. 288]
o The most commonly used IV Jluid preparation in neonates is N/5, 10o/o dextrose.
o So, in ideal preparation:
(i) NS should make 1/5th part (20o/o) of solution (i.e. N/5)
(ii) Preparation should have 10 gm ofdextrose in 100 ml (i.e. 10% dextrose).
10% dextrose in 20 ml of NS as it will provide 8 gm of
glucose at
. Amongst the given options, closest one is 80 ml of
normal saline concentration of N/5'

M. Ans. is t' i,e., Ringer lactatelRef: Nelson l*'hle ch' 74]

Fluid resuscitation in burn injury


r Parkland formula
3 mllkgf/' burn for children' RL
a) Initial 24 hours: Ringerb lactated (RL) solution 4 mllkglo/oburn for adults and
solution is added for maintenance for children:
1) 4 ml/kg/hour for children weighing 0-10 kg
2) 40 ml/hour +2 ml/hour for children weighing 10-20 kg
3) 60 ml/hour + 1 ml/kg/hour for children weighing 20 kg or higher
Thisformula recommends no colloid in the initial 24 hours'
b) Next24hours:collo\dsgiven aszo-6oo/oofcalculatedplasmavolume.Nocrystalloids'Glucoseinrvaterisadded
inamountsrequiredtomaintainaurinaryoutputoro.s-rml/hourinadultsandlml/hourinchildren.
7tu/e p' 5a]
45" Ans- is
.c,
i.e., 154 meq of chloride in lo00 rnt of fluid. l&ef: Ghai

Nomal saline (lsotonic saline)


o Contains - 9 gms of Nacl in 1000 ml of fluid
154 meq/sodium in 1000 ml of fluid
154 meq Chloride in 1000 ml of fluid
r CommonlY used in dehYdration

a) 5olo Dextrose I lOo/oDx

blN/2siiline'':, :: :,. : :.'.

c) N/5 saline

III
GK}WKYXCS
& GK&TMY'XE
MXSffiKMKKS

GENETIC DISORDERS

r Genetic disorders may be classified into following categories :-

A) Mendelian disorders (single gene disorders)


r These result from mutations in single gene of large effect. These may be autosomal dominant, autosomal
recessive and X-linked.
B) Diseases with multifactorial inheritance (polygenic disorders)
o These are influenced by multiple factors, both genetic and environmental. Improtant multifactorial disorders
],j,'PGts0), coronary heart disease, congenital heart diseases, diabetes mellitus,
are cleft lip/cleft palats(oNn
hlpertension and gout.
C) Chromosomaldisorders
r These are associated with numerical ot structural change in chromosome,

o Beside genetic disorders one other group of disorders, which is unrelated to genes or choromosomes, is called
Teratogenic events(Ar06) (Sometimes called congenital malformation). These are due to exposure to teratogenic
agents during Pregnancy.

Pedigree analysis
o pedigree analysis is the analysis of human gene transmission(A/rs). Pedigree chart is a diagrammatic method of
illustrating the inheritance of genes within a family'
r The starting point is often the identification of an affected individual, called the'propositus'or'proband', i.e. propositus
or ltrobanil is the individual which is studied in a pedigree, such as the individual with
a certain ilisease or other

inherited interest.
r After identification of propositus/proband, there is construction of a genetichistory or pedigree of the individual's
are used to denote sex,
family. By convention, each generation is displayed on a different level, and standard symbols
zygosity and other information about each family member'
Disotders

II

III

I
Affected Affected
il
witd Wild type
male female male l'emale

SINGLE GENE DISORDERS (MENDELIAN DISORDERS}

Autosomal dominant disorders


r Normally a gene pair has two alleles. When one allele becomes abnormal due to mutation it is called heterozygous
state.When both the alleles become abnormal due to mutation it is calledhomozygous state.
t Autosomal domiruant disorders are manifested in heterozygous state, i.e. only if one allel is abnormal the disease will
be manifested.So, at least one of the parent should be affected to transmitt the disease to child. Homozygous state
does not exist as it is not compatible with life, i.e. homozygous fetus usually dies in utero. All the affected individuals
will be heterozygous.
o If an affected person marries an unaffected one, i.e. if one of the parents is affected than every child has 5}o/o of
chance ofhaving the disease and 50% chance ofnot having the disease(NrE?).

Aa (affected parent) aa (Normal parent)

aa aa aa aa
(affected) (Normal) (affected) (Normal)

Note- 'a'represent nomal allel and'A'represent abnormal a11e1.

: If both the parents are affected than the chance of having unaffected baby is 2so/our 05. AIIMI 04)

Aa

o In this situation it is tricky to calculate the percentage ofhaving affected baby.


r Most of you must be thinking 75o/o, (very easy).
I But, it is not the case, as already explained homozygous state is not compatible with life and fetus dies in utero.
r So, the chances of having affected baby wiil be 50% because 25yo (AA) will die in utero.

c AD is the most common lWendelian pottern of inheritorncewErfl.


CH*prsR

*Huntigton disease ePoly- rFamilial nHereditary cMarfan syndro6s$w) rFamilial hyper-cho-


nNeurofibromatosis cystic polypo- spherocytosis o EDS (some var'ant6to7)) lesteiolemia
*Myotonic dystrophy kidney sis coli rVon-Willibrand oO steo g e n e s is i m p e rf ectst arcz t lAcute intermittent
oTuberous sclerosis disease rAchondroplasia porphyria
*Retinoblastoma

: Note t Ehlers - Danlos syndrome (EDS) has all three mendelian pattern of inheritance(At 07)
.

Autosoma! recessive disorders


r Autosomal recessive disorders are manifested in homozygous state, i.e. when both alleles at a given gene locus are
mutant. There may be following five situations -

l. One parent is carrier and the other is nornral - > 50oh of child will be carrier.

Aa

2. Both the parents are caruier --> 25% children will be afficted, 50% will be carrier and 25ok will be
normA/'"''"'.

AA Aa Aa

3, One parent is affected and other is carrier -+ 50% of child will be affected, 50% wilt be carrier"""o''.

AA Aa

AA Aa Aa

4. Both parents are affect ) 100% chitdren will be affectedryu").

AA

AA AA AA

5. Oneparent is uffected and one is normal -->All will be carrier'""'o')

AA Aa

n*
Cn-artx.*'1,15,

sHuntigton disease rPoly- rFamilial rHereditary tMarfonsyndrolnstAtoT) lFamilial hyper-cho-


lNeurofibromatosis cystic polypo- spherocytosis IEDS (some varian{Aba) Iesterolemia
rMyotonic dystrophy t<iOney sis coli rVon-Willibran d tOsteogenesis imperfecto^toT) oAcute intermittent
lTuberous sclerosis disease rAchondroplasia porphyria
rRetinoblastoma

r Note : Ehlers - Danlos syndrome (EDS) has all three mendelian pattern of inheritance@10') .

Autosomal recessiv€ disorders


r Autosomal recessive disorders are manifested in homozygous state, i.e. when both alleles at a given gene locus are
mutant. There may be following five situations -

l. One parent is carrier and the other is normal - > 5Oo/o of child lvill be carrier.

Aa

2. Both the parents are csrrier --> 25% children will be afficted, 50% will he carrier and 25'% will be
normaln"''o).

AA Aa Aa

3. One parent is uffected and other is carrier + 50% of child wilt be affected, 50% will be carrier'''""'

AA Aa

AA Aa Aa

4. Both parents are affict -s 100% chitdren witt be affected'uu').

AA

AA AA AA

5. One parent is ulfected and one is normal -+All will be carrier @"'0"

AA Aa
Note - | have given the percentage of genotypically affected children. Affection of phenotype will be different be-
cause all carrier will be phenotypically normal (as autosomal recessive disease is manifested only in Homozy-
gous state). So in 1st situation all child (100o/o) will be phenotypically normal, in 2nd situation 750lo child will
be phenotypically normal and in 3rd situation 500/o will be normal in 4th situation no child will be normal and
in 5th situation all will benormal. But you remember the precentage of affected children for which there is no
confusion.

Autosomal recessive disorders


..;:""ulgai6;.,,'+l,*q'#"Egier-,"fucm.*:5He*cI'-'-=-,u*,*l"
t Cysticfibrosisedls'Atto) o Sickle cell r Congenital e EDS (some o Friedreich ataxia
* PhenYl ketonurio{At13) anemia(4r13) adrenal variants) o Spinal muscular
o Galactosemia * Thalassemias hyperplasia o Alkaptonuria atrophy
* Homocystinuria
'
Albinism(At13' r Neurogenic
* Lysosomal storage dis. PGI 1 3)
muscular
* o,-antitrypsindeficiency atrophies
o Wilson diseaseiDNB r3)
* Ataxio
r Hemochromatosis Telengiectosio{At rzt

* Glycogen storage
3)
disorderst'^t 1

[Iniparental disomy
o Autosomal recessive diseases manifest in homozygous state. But there is one exception to this rule, i.e. Uniparental
disomyro'o't. Uniparental disomy occurs when a person receives two copies of a chromosome, from one parent and
no copies from other. In this condition if the child receives two copies of the affected chromosome from the carrier
parent, he/she will manifest the disease.

e Most of the metabolic disorders are autosomal recessive except:-


a Hunter syndrome, Fabry's disease, ocular albinism, Lesch Nyhan syndrome + X-linked recessive.
t Fomilial hypercholesterolemio, acute intermittent porphyria + Autosomal dominant

X-linked disorders

X-linked re(essive
o Except for a few conditions, all X-linked disorders are X-linked recessive.
o AsmalehasonlyaneX-chromosome,themalewithaffectedgeneonX-chromosewillalwaysmanifestthedisease4l
10).

o On the other hand, female has 2 X-chromosomes, heterozyogous female will be carrier@Egr) because of expression
of normal allei on the other X-chromosome.
o Note - Female with Tirrner syndrome (only one 'X'chromosome) can manifest X-linked recessive disorders.
o 50o/o boys of the carrier mother will be affectedAt 13' NEET) .

o All daughters of affected father will become carrier, i.e. affected father will transmit the defective X-chromosome to
all the daughters.
o Father will not transmitt the disease to son as boys do not inherit X-chromosome from fathe/NEEr) .
r Three important sceneria may be there.
i) Father is afected mother is normal: All female children will be carrier and male childen will be normal
x"x xux

ii) Father is normal mother is carrier:50o/o of male children (son) willbe affected@113'NEar), and 50olo of female
children will be carrier.

rll) Fgher carrier: 50% of male children will be afected, 50% of female children will be
is affected and mother is
affected and 50% of female children will be carrier

x"x xuv

X-linked dominant
r In this, both males and females are affected. All the sons of affected father are normal and all daughters are affected.
The affected mother transmitt the disease to half (50%) sons and half daughters.

xo

xox

Mastubskeietal, "", ,Hema{alqsiaql. , , l It:l t:r':tg, :.: .'.. .,,,.. :.,' .M€lobalic,', ..: .,, - ',. :,.,11r';. N.erVous,,,,,',, ::,,, .,.,.' .

:poi;"1t'
LE.' ::... :. i.: ::::1 :.:

r'.: i '* ,.HemophlllaA,,& r Agammaglobulinemia r Diabetes insipidus *, -F,ragile-X-''r. '' "


" .muscalai', , ., B$tEEt) . , r Wiskott-Aldrich a syndrome "..syndrilmei,: .

"' ", I'


',

'"'tl1stia4hy''p,E{" .ir* 'Chioni€ ''.sllnaiome, I Lesch-Nyhansyndromee *,rj',Cotrour blind,ness

*.,,,Beckei|s' ,, ' , ,,'r .graflulornatous ,

' ,,dyttroihf'n) ' '*, :, 'diseaae '

.6-6.PD deficiencyw

.t viia mina n reaisiarit rickeis


* Familial hypophosPhatemia

Fragile-X syndrome
o Fragile - X syndrome is the prototype of diseases in which the mutation is characterized by a long repeating
11' 0e)
sequence of three nucleotideseGr .
'&;fienetlcDisarders
r In fragile X syndrome,trinucleotide repeat mutation involve CGG on non coding region.
r Clinical features of fragile - X syndrome are :-
t Mental retardation r Mitralvalve prolapse
t Large testis@NB AMU (macro-orchidism)
13, 0s)
r Hlperexntesible joint
t Long face with large mandible@Mu 05) r High arched palate
: LAtge eVerted eArS@NB 13' AMU os)

o Fragile X syndrome is the second most common genetic cause of mental retardation,
after Downs syndrome,

A child wtth mental rera.rdatlon has followingpictura


Probable cause of mental retardation - ]

b) Fragile-X svndrome
c) Turner syndrome
d) Hlpothyroidism

oo*
Ans. is i.e., Fragite-X
r Mental retardation in child with large elongated face and large everted'ears is
characteristic of FragilelX
syndrome.

SINGLE GENE DISORDERS WITH UNCLASSIF;ED INHERITENCE

r These disorders are of follorving t)?es :-

Mitochonrial disorders
c Mitochondrial DNA is the only non-chromosomal DNA in human cells.
I Mitochondrial DNA is always maternally inherited. Thus diseases caused by mutation in mitochondrial DNA are
always inherited from mother to next generatiorx@Itus 12, AI07),

o All children from affected mother will inherit the tlisease. But no children will
inherit the disease from affected father.
o Both male andfemales are affected6llMslt'o"". Aff"rtrd son tloes not transmitt
the disease to next generation@cl t0),
e Important mitochondrial disorders are Pearson syndrome, Leigh syndro me, MELAS@I ts),
Kearns-Sayre Syndrome
(r{S5)r"o' "1 Leber hereditary optic neuropatfu (LHoN)rNmD, Chronic
progressive external ophthalmelagia@Grl,) and
NARPGrrus0e).

Triple nucleotide repeat mutations


o These diseases are chracterized by long repeating sequence of a particular trinucleotide
codon.
r Important disorders in this group are -
i) Huntington disease (CAG repeat)
ii) Spinocerebellar ataxia (CAG repeat)
iii) Fragile X-syndrome?Gr to'os) (CGG repeat)
iv) Freidreich ataxia (GAA repeat)
v) Myotonic dystrophy (CTG repeat).

Genomic imprinting
r A person inherits two copies of their genes, one from mother and one from father.
Usually both allels of gene are active
in ceils' However, in some cases, one of the allel (either maternai or paternal) is silenced
and only the counterpart
remains active.
r when the aliel inherited from mother is silenced/inactivated (paternal aliel is active), it is
called maternal genomic
imprinting. When paternal allel is silenced (maternal allel is active) it is called paternal genomic
imprinting.
. Thus genomic imprinting leads to preferential expression of an allel depending on its parental originqr
oe' os)

r Important examples of genomic imprinting are :-


1) Prader Willi syndromelEBr'PGI01)
AI
r It is due to paternal genomic imprinting{NrEr'
13)
in which there is deletion on paternal chromosome l5@113'
NEET, AIIMS 10)
.
r3),
r They are character ized by hlpotonia, obesity{At
ts),
short stature, mental retardation(Ar hypogonadism,
hyperphagia, and short hand and feet.
r There is d.ecrease in GnKH, LH, FSH and GHGI 12).
Ghrelin levels are raisedql 13).

2) Anglemann slndrorns(Pcr or)

r It is due to maternal genomic imprinting(AlI3'NEEI) in which there is deletion on maternal chromosome 15.
r They are characterizedby hypotonia, mental retardation, seizures, ataxia, and inappropriate laughter (Happy
puppets).
o Besides genomic imprinting, the above two slmdromes may aiso be caused by a phenomenon called Uniparental disomy
(UDp;rrr,r.t in which a person receives fwo copies of a chromosome from the same parent and no copy from other
parent. Prader Willi Syndrome may be due to inheritance of both copy of chromosome 15 from mother (Maternal
UDp(NEEIArMs
ro)) and Anglemann syndrome may be due to inharitance of both copy of chromosome 15 from father
(Paternal UDP(Arr3)).

CI-I ROMOSOMAL DI SORDERS

r These disorders are due to chromosomal mutations which results in either abnormal number or structure of chromosome'

A) prsoRpERS puE TO ABNORMAL NUMBER OF CHROMOSOMES


r There are23 patrsof chromosomes (46, XX in females and46,XY in males). An exact rnultiple of the 23 (haploid
number) is called euploidy. When the number of chromosomes are not in exact multiple of 23, it is referred as
aneuploidy. e.g. Trisomy or monosomy.

Triso mles (involvi n g auto so mes) Down syndrome (Trisomy 2'l ), Edward syndrome (trisomy 18),
Pata u synd rom e (trisomy 1 Sbttws
0& oG))
.

Trisomy (involving sex chromosome) Klinefelter syndrome (47, XXYi.


Monosomy (involving sex chromosome) Turner syndrome (45, X).

Down's syndrome
r Down's syndrome is the most common chromosomal disorder^'IMsnn) and most common congenital cause of mental
retardation (2d most common genetic cause of mental retardation is Fragile -X syndrome),

Cytogenetics
o Basic molecular defect in Downs syndrome is extragenetic material of chromosomeZl.It may be due to :-
1) TtisomY 27@ItMstt'PGI11)
r This is lhe most common cause (95o/o), There is an extra chromosome 21 (i.e. three 21 chromosomes) which
is due to meiotic nondisjunctional
10)
in ol'um. The extra chromosome is of maternal origin{Ar ro). The most
important risk factor is advanced maternal age (> 35 years).
2) Roberston translocation
r This account s for of cases. Extra material of 21 chromosome comes from Robertson tanslocation of iong arm
3o/o

of chromosome 21 to another acrocentric chromosome (22, or 14 or 15), i.e. t (22: 21), t (14:21), or t (15:21)
(NEE[,AilMs1l,PGI lr).
It has no relation with maternal age.
3) Mosaicism (Mosaic 276ILMS
lt PGI
'
11)
)
t In2o/oof patients there is mosaicism, i.e. occurance of two or more diflerent tlpes of cell population (46 XX I
Geiriefiit,Dlsordeis

47 XX + 2 I ) in same individual, due to mitotic nondisjunction of chromosome


2l. There is no correlation with
maternal age.
4) Partial trisomy
t rt is very-very rare. The long arm of chromosome 21 triplicates.
t Advanced paternal age has no correlation with Down's syndroms@nrD.

Cliniral features
o Most striking feature in the neonate is hlpotonia@EnD and although the
diagnosis is usually evident at this time, it
may sometimes be missed if the baby is very or his facial features are concealed by ventilatory apparatus.
Premature
o Other clinical features include :,
1) General:- Mental retardation@EEr), short stature
2) Cranio-facial:- Brachycephaly(ua*os't, epicanthic fold, protruding tongue, small ears, upward sloping palpebral
fissures (Mongoloid slant eGI o0)) , strabismus, nystagmus, Brushfieil spots in
irisavEEr, Ar 06).
3) Limbs:- Fifthfinger clinodactyly\IlMse3), single palmar crease (simian crease@EEr,pcl,00,ArrMSe3)),wide
gap between
first and second toes (sandle gap eGI0s)) .
4) Congenital heart disease@G.,2) :- Common AV canal, ostium primum/endocardial
cushion defect type ASD (most
06, DNB t2)), ysD, pDA,
common1r fallot tetralogy.
5) GIT:- Anal atresia, Duodenal 4fyssiq@rss), Hirschsprung disease(Arnr), annul ar p
ancr e asal e 4),
6) Increased incidence of leukemia (1o/o). Leukemias common are ALL (most
common)@Gl 02), AML (M7-AML)
transient my'eloproliferative disorders(NnEr), and]uvenil CML.
7) Others : Early onset of Alzheimer\ disease?G'02), Decreased immunity wlth recunent
infections{pcrorl obesity,
DM, Hypotlryysiflisyn4t oe) .

r Cognitive impairement with developmental delay and mental retardation is universal(A,,5).

Antenatal Screening for Downs syndrome


o Following methods are used :-
1) Triple tes{Alls): It includes (i) Unconjugated estrogen(Att*s03,Alse) (estriol): decreased; (ll) Maternal serum alpha-
feto pfotein (MSAFpyo"*t 03' AI ee) . decreased; and (iii) hcc(ArrMs 03, Ar ee) : increasedurrMs 07) (Note : All these three
markers are decreased in Edward syndrome)
2) New markets: These are (i) Increased inhibin A in maternal blood; and (ii) D
ecreased pApA (pregnancy associated
s oz
Plasm a Pt of ein )@uu I .
3) USG@I ls) : It shows : (i) Increased nuchal translucency (increased nuchal Ar se))
1s);
fold lllisftnsssulbrs ee,
in first
trimester(DNB Ductus venous flow reversed\IlMs oz); afid (ir) Nasal bone hypoplasiaurrMs 07).
(i1)
4) KaryotypingiArrs) : It can be done by chorionic villus samplin g at lo-l2weeks
or aminocentesis at 16,1g weeks.

A childwith mental retardation has following picture.


Probable causeof ment l retardation -

a) Down syndrome'
b) Fr,agile.X syndrome
',
c) Turner syndrome
d) Hypothyroidism

, r These are typical facial features of Down syrrdrome, i.e. flat faciei, upward
slant of eye, and open
mouth with protruding tongue. ,!'

Other important trisomies


r These arc Patau syndrome (Trisomy 73,113)) and Edward syndrome (Trisomy
7g.,EEr)).
Cnlptnn

& pqlatewEEr'attvsos'06)
Ceft lip and cleft & Low birth weight

& Flexed-fingerswith polydoctyly'AttMsos'M) 6 Closed fists with index finger overlapping the 3'd

w Occular hypotelorism{NEEr) * digit and the 5th digit overlaping the 4'h.

& Bulbous nose a Narrow hips with limited abduction

& Low set malformed ears & Short sternum

@ Small abnormal skull (Microcepholy'atusoe'oat1 & Ro cke r b ott o m f e etwEEr)

* Cerebral malformation especially holo prosencephaly'NEn) a Microcephaly

& Microophthalmio@ttqsos'06'03) and cloboma of iris. & Prominent occiput

& Cardiac malformations & Micrognathia

a ScalP defects(AttMsos'06) & Cardiac and renal malformations

Hypoplastic or absent ribs a Mental retardation

& Visceral and genital anomalies & 15olo cases are lethal in 1't year

@ Ro cker b ottom f e ettN EEr)


& Rocker-bottom feet may be seen both in Edward syndrome and Patau syndrome. But it is more characteristic of Edwards
sYndrome(uPscod.

Cat eye syndrome {Tnisomy 22)


o There is partial trisomy of chromosome 22(A107). The p (short) arm and a small part of q (long) arm of chromosomes are

threeinnumbers(insteadofnormaltwo). Verticalclobomasineyegivesappearanceofcateye. Importantfeaturesare:-


i) Ocular cloboma of iris, choroid or bptic nerve vii) Cystic dysplasia

ii) Preauricular skin tags or Pits viii) Mild mental retardation


iii) Congenital heart disease ix) Microphthalmia
iv) Anal atresia with fistula x) Microtia with atresia of the external meatus
v) Renal malformation -+UlL renal agenesis xi) Intra-or extra hepatic biliary atresia
vi) Antimongoloid slant of eYe. xii) Malrotation of gut.
Turner syndrome
of sex chromosome, i'e'
o It the most common cause of sex chromosomal abnormality infemales.Itrs catsedby monsomy
is

there is complete or partialloss of onex chromosome infemales (45 xo or 45 x6'o'))


and only 45 chromosomes(N'Er)

are there.
(cystichydromaal
r Clinical features in childr enare edema of dorsum of hands andfeet(Nanrt, sweiling of nape of neck
ea)),bilateralwebbingofneckal04),andcongenitalheartdiseases lmostcommonbicuspidaorticvalve@NB13)foliowed
os)), AS, MVP]'
by coarctation of aorta (2nd most commonqttMs
ts shott 0a' Ar ot),
webbed neck6' o')
low posterior hair lineqt
0a)
widely
o Clinical features in adolescen are stature@Gt
' '
cubitus
spacednipples(Ar03) with broad chest, hypertelorism, epicanthus, slanted palpebral fissure, ptosis, micrognathia,
hlpothyroidism, streak
valgus (increared carrying angle6l04)),sensorineural hearing loss, shortfourth metacarltal\ron),
ovariesa/ ri), and sexual infantilismecr
0a).
Tlrner syndrome is the mosf important cause of ptimary amenorrhea@t
ee)
may be glucose intolerance, obesity and insulin resistance. Mental retardation is not
,3). There seen4l '

carrying
Turner syndrome showing shield chest with widely spaced nipple, webbed short neck, and increased
angle.

I
\prER L 5 Genetics & Genetic Disorders

r Noonan syndrome is an autosomal dominant(Ar 03)


disorder which affects both females and males@NB ri). The most
phenotypic features resemble females of turner syndrome, but they have normal sex chromosome and normal karyotyps@Nn
13)
(46 xX or 46 XY), i.e. male or
female with turner phenotype.Itis due to mutation on chromosome 12.
c Important features are short stature@re7'ArrMSe4), webbed neck, low set ears, cubitus valgus, ptosis, micrognathia,
hypertelorism, down-slanting palpebral fissure (antimongoloid slan{Al ss, AIIMS s4)), clinodactyly, cryptorchidism,
i.e. undescended small testisql 03' e7' AIIMS e4), pectus carinatum or pectus excavatum, sensotineural hearing Ioss(DNB
r3), congenital
heart diseases (supravalvular pulmonary stenosis\t sT,AIIMS s4) : most common, ASD, hypertrophic
cardiomyopathyqtos), and defect in conduction/rhythm), and delayed puberty: Testosteronelevels are oftenlow@NBt3).

. Note: Noonan syndrome is not a chromosomal disordey'conedo6).lt is an autoso mal dominont (Mendelian
inheritonce) disorder. lt has been explained here because it has features similar to Turner syndrome.

Klinefelter syndrome
r Klinefeltersyndromeisthemostcommonchromosomaldisorderassociatedwirhmalehypogonadismandinfertility.
It is defined classically by a 47,XXY(PGr0') karyotype with variants demonstrating additional X and y chromosomes.
(Other variants can have 48 XXXX rarely 49 XXXXY or mosaics can be there with some cells containing normal
46' XY and others 47, XXY). Classically, it results from meiotic non-dysjunction of sex chromosomes (40% during
spermatogenesis and 60% during oogenesis). Mostly, non-dysjunction occur during 1,t meiotic division.
o The patient has male phenoQpe with feminizing features due to extra X-chromosome (note : presence of one y
chromosome is sufficient for male phenotlpe. Thus XY, XXY, XXXY all are males). Extra inactive chromosome appears
as Barr body.
o Important clinical features include microorchidism with normal external genitalia, mental retardation@Gl 0t),
gynecomastia, lack of secondary sexual characteristics with eunuchoidecl 01) body habits, disproportionately long
arms and legs, hypogonadism(Pcl01), increased incidence of tumors (breast carcinoma, germ cell tumors), increased
incidence of autoimmune disorders (e.g. SLE), and cardiac problems (most common is mitral valve prolapsea'o se)
,)
Testosterone levels are decreased, whereaslevels of gonadotropins (FSH/LH) are elevated@Gt01),

B) DISORDERS DUE TO ABNORMAL STRUCTURE OF CHROMOSOMES


r Chromosomal structural changes usually result from chromosome breakage followed by loss or rearrangement.
I Beside these chromosomal structural abnormalities some autosomal recessive genetic disorders are associated
with such a high level of chromosomal instability that they are known as chromosomal breakage syndrome -+
Ataxia - telangiectasis, bloom syndrome, and fanconi anemia. There is significantly increased risk ofcancer in
these syndromes.

* PedlOree ala.lVsis chart is used for: lllustrating the inheritance of genes u tuily:
'a AhA$ ,4i!; ,,
in . . ..
dbobywhenb.othparinti'ielve'auiosonil:domini,ant,aiieasei25ge.'.1;,:.:.
* Chances of being affected if one parents has autosomal dominant disorder:50o/o.

* if one parent is having autosomal recessive disorder and other parent is c.arrier:5oo/o:.
e Chances o! hau!.ng diseases.if b3th parents are carrier of autosomal recessiue disease :25o/o.

* enanlai.At'kiiiin*dijrcAiiif both parents are affectea,bi:i*tqiowt:rii iiuel:tiiearer 10o%i.:,..,,:,


o Chances of having disease if one parent is carrier for AR disease and other is normal : Oo/o (50olo will be carrier).

*
I Only males are affected:X-linked recessive.
* Fpthe$'
t,
l:, rynimifr;l.iraa*i:use'rajm : iiJinkedi :,

3 Mothers not transmitt the disease to daughters: X-linked recessive.


* .,,':Father:'*.s.nsmiitiigdiieqsg to atl daughter: x-finieddorCIna*L.,,.:
t Mother transmitting disease to all sons : X-linked recessive.
a "'lrlOlhai::+rans*ittlw,djseasela halliohlir X:linke-d'dominanti,.::, ,,:
Genetics & Genetic Disorders

Noonan syndrome
r Noonan syndrome is an autosomal dominant(Ar 03)
disorder which affects both females and males@NB r3). The most
phenotypic features resemble females of turner syndrome, but they have normal sex chromosome and normal karyotyps(oNs
li)(46 XX or 46 XY), i.e. male or
female with turner phenotype.It is due to mutation on chromosome 12.
o Important features arc short statureqle7'AIIMSe4), webbed neck, low set ears, cubitus valgus, ptosis, micrognathia,
hypertelorism, down-slanting palpebral fissure (antimongoloid slan{Al ee' AIIMS e4)), clinodactyly, cryptorchidism,
i.e. undescended small fssfisqt os' e7' AIIM9 e4), pectus carinatum or pectus excavatum, sensor ineural hearing Ioss(DNB
i3), congenitai heart diseases (supravalvular pulmonary
stenosis\I eT,AIIMS e4) : most common, ASD, hypertrophic
cardiomyopathy(Al03),anddefectinconduction/rhithm),anddelayedpuberty: Testosteronelevelsareoftenlow@N813).

r Note: Noonan syndrome is not q chromosomal disordey'comedo6).lt is an autosomol dominant (Mendetian


inheritonce)disorder. lt has been explained here because it has features similar to Turn", ,yndror".

Kfinefelter syndrome
o Klinefelter syndrome is the most common chromosomal disorder associated with m ale hypogonadism and infertility.
It is delined classically by a 47,XXY(PGI0'I) karyotype with variants demonstrating additional X and Y chromosomes.
(Other variants can have 48 XXXY, rarely 49 XXXXY or mosaics can be there with some cells containing normal
46, XY and others 47,XXY). Classically, it results from meiotic non-dysjunction of sex chromosomes (40%o during
spermatogenesis and 60% during oogenesis). Mostly, non-dysjunction occur during 1,t meiotic division.
r The patient has male phenotype with feminizing features due to extra X-chromosome (note : presence of one Y
chromosome is sufficient for male phenotlpe. Thus XY, XXY, XXXY all are males). Extra inactive chromosome appears
as Barr body.
r Important clinical features include microorchidism with normal external genitalia, mental retardation@Glo,),
gynecomastia, lack of secondary sexual characteristics with eunuchoid@Gl 01)
body habits, disproportionately long
arms and legs, hypogonadism(PGl01), increased incidence of tumors (breasi carcinoma, germ cell tumors), increased
incidence of autoimmune disorders (e.g. SLE), and cardiac problems (most common is mitral valve prolapseeqer.)
Testosterone levels are decreased, whereas levels of gonadotropins (FSH/LH) are elevated@cl0t).

B) DISORDERS DUE TO ABNORMAL STRUCTURE OF CHROMOSOMES


o Chromosomal structural changes usually result from chromosome breakage follou.ed by loss or rearrangement.
r Beside these chromosomal structural abnormalities some autosomal recessive genetic disorders are associated
with such a high level of chromosomal instability that they are known as chromosomal breakage syndrome
-+
Ataxia - telangiectasis, bloom syndrome, andfanconi anemia. There is significantly increased risk ofcancer in
these syndromes.

c Pedigree onalysis chart is used for: lllustrating the inheritance of genes in a family.
t Chances of having unoffe:cied baby when both porents have autosomal dominant disease ..25o/o.
o Chances of being aiected if onte porents has autosomal dominornl disorder'. 5Oo/o.

i;,,,:,ehqnces af.haviig disea5e if one parent is havlng autg;amAi iiiessiive Aliarder,,and,it:bsrp.atent is,iarrii*: 50%.. :

* Chances of having diseases if both parents are carrier of autosomal recessive disease :25o/o.
3.,, ekaqeei.of htiviaeircaseifbott!..$fenfs,dre affiitid, eu.tasomilrc-cessil*ii,ssr.ll00%:',r,,r"',
t Chances of having disease if one parent is carrier for AR disease ind other is norlmol: 0olo (50olo will be carrier).
::$.: ' Natii n a ata* 6,nt eL do tiiin ait :gis o r.dcc Tab g dise.ir$
a Anly.malesare7ff11yd:X-liyked recesllve, .

:t;lb 1
Fatiers not transmitt the disease to son: X-linked.
a Mothers not transmitt the disease to daughters : X-linked recessive.
::*.:

s Mother transmitting disease to all sons: X-linked recessive.


CH.A.prrn t s Genetics & Genetic Disorders

* Only males are affected and females act as carrier: X-linked recessive'
x Differentio! expression of same gene depending on parent of orgin: Genomic imprinting
a Functional gene is inherited from one Parenf : Genomic imprinting'
c Paternal 15 deletion or Maternal 15 disomy'. Prader Willi syndrome'
w Maternal 15 deletion or Paternal 15 disomy: Angleman syndrome'
e Obesity with mental retardation:Prader will syndrome'
w Mother transmitting disease to all children : Mitochondrial disorders.
* Both males snd females ale affected but ma[es do not transmitt the disease : Mitochondrial disorders'
a Common ocular findin g in trisomy 13 (Patau syndrome):Bilaleral microphthalmos
'l
& Maltiple defects with cleft tip, cleft palate, microcepholy, microphthalmos and scalp defect;Trisomy 3.

:tr:1:::;;:,;:::':;;"IJ ji,,,,,,.mv23(D.wnsvndr.me,
e Not seen in Down's syndrome : Decreased nuchal thickness, hypertonicity, Pigmented birth mark.
,losaic 21.
w Downsyn:dromeisdueto..Trisomy2l (mostcommon); Robertsoniantranslocation13-15/21,22121;!
e Most common cause of trisomy 21 in down syndrome : Maternal nondisjunction in
meiosis 1 .

e Most cornmon CHD in Down syndrome'. lndocardial cushion, ASD VSD'


a Triple test for Down's syndrome includes: Estriol (decreased), alpha fetoprotein
(decreased), HCG (raised).

w Qudriple test for Down's syndrome :\ri?le test + inhibin A'


q* PAPA levels in Down syndrome : Decreased'
x Coimmonest cAuse an syndrome
of intestinal obstruction in Dawn : Duodenal atresia'
syndrome:Duodenar a1

* ChromosomesinTurner syndrome :45 (45XO).


* Features of turne:r syndrome with normol katyotype I Noonan syndrome.

w Edema of hands and feet in an infant'.f urner syndrome'


e Most common cardiac anomaly in Turner syndrome: Bituspid aortic valve (mc) and COA (2'd mc).
e Not seen in Noonan syndrome'.lnfertility in females'
e Not seen in fragile X-syndrome : Large nose, pigmented'nevi

:';:,#l::, ;;;1';":,,:ili,
e Down syndrome:TrisomY 21.
* Klinefetter syndrome is diagnosed by: Karyotyping'

:i:;:i,'i;::;:,:::,::,i:::;,J::::,.*)me,+sxo,
w Male with present Barr body,, sv.ndrome (47XXY)'
1,'":t:,,:l
* Most cammon birth defect in North lndra : Neura[ tube defect (spina bifida).
s Most common birth defect in rest of lndia: Musculoskeletal defects.
c Skull in Down's syndrome: Brachiocephaly,
a Distinguishing feature of Edward syndrome (Trisomy 78) : Rocker bottom fect'
* Not seen in Turner syndrome : Mental retardation.
w Not a feature of Noonan syndrome: Chrolos.omal abnormality'
w Jeune syndrome {asphyxiating thoracic aplasia) is : Autosomal recessive disorder'
w pierre Robin syndrome consists: Micrognathia (mandibular hypoplasia), cleft palate, CHDs, respiratory obstruction, foreshortened
floor of mouth, normal size tongue.
* Mutation in a sinqle gene produces effect on mare than one choracteristic: Pleiotropy.
* Child resemembles grand-parents: Atavism.

&&x
QUESTIONS

Pedigree analysis chart- (All lndia Dec15 Pattern) is normal and the other is carrier and the child is
a) Used for growth monitoring also affected. What is the reason - (At 07)
b) To assess side effect during chemotherapy a) Germ line mosaicism
c) Used to see gentic transmission b) Genomic imprinting
d) To assess developmental delay in infant c) Penetrattion
2. A child with a small head, minor anomalies of d) Uniparental disomy
the face including a thin upper lip, growth delay, 9. An affected male infant born to normal parents
and developmental disability can have all of the could be an example of all of the following, except -
following, except - (At 06) (Ar 06)
a) A chromosomal syndrome a) An Autosomal dominant disorder
b) A teratogenic syndrome b) An Autosomal recessive disorder
c) A mendelian slmdrome c) A polygenic disorder
d) A polygenic syndrome d) A vertically transmitted disorder

AUTOSOMAL DOMINANT X.LINKED

J. All of the following conditions have autosomal dom- 10. True statement about inheritence of an X linked
inant inheritance except - (All India Dec.14 Pattern) recessive trait is - (Ar e7)
a) Fabry disease a) of boys of carrier mother are affected
50o/o
b) Marfan's syndrome b) 50% ofgirls ofdiseased father arecarrier
c) Osteogenesis imperfecta c) Father transmits disease to the son
d) Ehlers Danlos slmdrome d) Mother transmits the disease to the daughter
4. The chances ofhaving an unaffected baby, when 11. Kinky hair desease is disorder where an affected
both parents have achondroplasia, are - (Al 0s, AIIMS child has peculiar white stubby hair, does not grow;
May 04) brain degeneration is seen and dies by age oftwo
a) 0% years. Mrs A is hesitant about having children
b) 25Vo because her two sisters had sons who had died form
c) 50o/o kinky hair disease. Her mother's brother also died
d) 100% of the same condition. Which of the following is the
5. In family, the father has widely spaced eyes,
a possiblemodeofinheritenceinherfamily - @t04)
increased facial hair and deafness. One ofthe three a) X-linked recessive
children has deafness with similar facial features. b) X-linked dominant
The mother is normal. Which one of the following is c) Autosomal recessive
least likely pattern of inheritance in this case- (AI 06) d) Autosomal dominant
a) Autosomal dominant 12. Study the following carefully -
b) Autosomal recessive
c) X-linked dominant
d) X-linked recessive

AUTOSOMAL RECESSIVE

6. A parent is homozygous and a parent heterozygous for Read the pedigree. Inheritance pattern ofthe disease
an autosomal recessive gene. What will be the out- in the family is - (At os)
come - a) Autosomal recessive
a) 75o/o children affected (AIIMS May 9a) b) Autosomal dominant
b) No child affected, but all are carriers c) X-Linked dominant
c) 50o/o children affected, rest are carriers d) X-Linked recessive
d) 25Vo children affected, rest are carriers 13. Which of the following is the most likely inheritance
7. For a normal husband and wife the first child was pattern in the pedigree given below- (AttMS May 04)
diagnosed to have cystic fibrosis. What is the per-
centage ofchances for the second child be afiected -
a) 25 b) so (PGl Nov 14)
c) 0 d) 75
e) 100
8. In an Autosomal Recessive (en) disorder, one parent
a) Autosomal dominant lip, cleft palate, microcephaly, small eyes, scalp
b) Mitochondrial defect and polydactyly, seen in which syndrome-
(AIIMSNov
c) Autosomal recessive 06, MaY 08)

d) X-linked dominant a) Trisomy 13 b) TrisomY 18

14. Pedigree analysis - Analyze the following pedigree


c) Trisomy2l d) MonosomY2
't1
and give the mode of inheritance - (AttMS Mav 12, AI 07) Patau syndrome feature include A1E -
a) Autosomal recessive a) Cleft lip (NEET Dec.12 Pattern)

b) Autosomal dominant b) Hlpotelorism


c) Mitochondrialinheritance c) Holoprosencephaly
d) X linked dominant d) Rockerbottom foot
23. Common ocular manifestation in Trisomy 13 is -
a) Capillaryhemangioma (AilMS 03)
b) Bilateral microphthalmos
c) Neurofibroma
d) Dermoid cyst
24. Trisomy 13 is seen in - (NEET Dec.12 Pattern)

a) Edward Syndrome
GENOMIC IMPRINTING b) Patau Syndrome
c) Down Syndrome
15. Diflerential expression of same gene depending on d) Turner Syndrome
parent of origin is referred to as - (AI 0s,06)
25. Cat eye syndrome is- (All India Dec.14 Pattern)
a) Genomic imPrinting a) Partial trisomy 18
b) Mosaicism b) Partial trisomy 13
c) Anticipation c) Partial trisomy 21
d) Nonpenetrance d) Partial trisomy 22
16. The process underlying differences in expression of
a gene, according to which parent has transmitted, is
called (At06)
DowN sYNDR0ME
a) Anticipationb) Masaicism
c) Non-penetranced) Genomic imprinting
26. Increased nuchal fold thickness is a feature of -
t7. Maternal disomy of chromosome 15 is seen in - a) Paul-Bunnel syndrome @I 99)
a) Prader - Willi spdrome (AttMSNov 10) b) De-pan syndrome
b) Klinefelter's syndrome c) Downs syndrome
c) Angelman syndrome d) Cri-duchat syndrome
d) Ttrnert syndrome zz, Most common presentation of down syndrome is-
18. Which of the following hormones are raised in a) Cognitiveimpairment (All lndia Dec15 Pattern)

Prader- Willi sYndrome - @112) b) Delayed dentition


a) Growth Hormone (GH) c) Recurrent chest infection
b) Leutinizing Hormone (LH) d) Constipation
c) Follicle Stimulating Hormone (FSH) LO. All the following are features of Down's syndrome
d) Ghrelin exc€pt - (CET Nov.14 Pattern)

a) Brushfield's spots in iris


PLEIOTROPY & ATAVISM b) Simian crease
c) Mental retardation
19. Single gene defect causing rnultiple unrelated prob- d) Hlpertonicity
lems - (AttMSNov 06) 29. All of the following may occur in Down's syndrome
a) Pleiotropismb) Pseudodominance except - 6I 06)
c) Penetranced) Anticipation a) Hlpothyroidism
20. Atavism means child resembles with his - (AtlMS Nov b) Undescended testis
ee)
c) Ventricular septal defect
a) Father b) Siblings d) Brushfield's spots
c) Grand parents d) Neighbour 30. All are common in Down syndrome, except -
a) Simian crease (AllMS May 9j)
b) Clinodactyly
TRISOMIES c) Mother's age > 35 years
d) Respiratory tract infection uncommon
21. A baby presenting with multiple deformities, cleft
31. Most common carfio yascular abnormality in down b) Trisomy 17
syndrome is - (All India Decl5 Pattern) c) Trisomy 2l
a) VSD d) Trisomy 13
b) Endocardial cushion defect 42. Down Syndrome, All are $een except-
c) TOF (NEET Dec.12 Pattern)
d) coA a) t0a;21) b) Trisomy 21
7"' Most comrnon syndrome associated with A-V canal c) t(11:14) d) t (ls:21)
defect - (AlI lndia Dec15 Pattern)
a) Down syrdrome
43, A Down syndrome child is mentallyretarded. All
cytogenetic abnormalities may occurs except?
b) Klinfelter syndrome
a) Deleted 21 \trMSNov il)
c) Turner syrdrome b) Trisomy2l
d) Marfan syndrome
c) Robertsoniatranslocation
33. Most commoa cardiac lesion in Down's syndrome d) Mosaic
is?
a) Atrioventricular Septal Defect (CET lune 14 Pattern)
b) ToF TURNER'S & NOONAN SYNDROME
c) Coarctation ofaorta
44. A child presents with antimongoloid slant, pulmo-
d) Transposition ofgreatvessels
nary stenosis, short stature and undescended testis.
34. Most comrnoa CYS lesion in Do*.n's Syndrome is- The likely diagnosis is- (AIIMS Dec 94, AI 97)
(All lndia Dec15 Pattern)
a) Hlpoparathyroidism
a) VSD b) ASD b) Noonan syndrome
c) TOF d) coA c) Klinefeltersyndrome
35. LCG aew born child with Down eyadrome - d) XYY sex chromosomes
(CET July 15 Pattern) 45- All of the following are true about Noonan syn-
a) Normal b) ASD drome except - (CET Nov.l3 Pattern)
c) VSD d) ToF a) Equal incidence in boys and girls
36. All of the following are features of down's syndrome b) Testosterone is always normal leve1
except - (AIIMS Nov 07) c) Karyotype is normal
a) Increased PAPPA d) Sensorineural hearing loss is present
b) Increased free beta HCG levels 46. All of the following mayoccur in Noonan's syn-
c) Absent nasal bone drome except - (At 0i)
d) Abnormal ductus venous flow velocity a) Hlpertrophic cardiomyopathy
37. Triple test for diagnosis of down's syndrome in- b) Cryptoorchidism
cludesallofthefollowingexcept - (Atse, AIIMS0i) c) Infertility in females
a) p-HCG d) Autosomal dominant transmission
b) o-Fetoprotein 47, Wbbbing of neck, increased carying angle, low
c) Serum HPL level posterior hair line and short fourth metacarpal are
d) Serum oestriol level characteristics of - (Ar 04)
38. Prenatal diagnosis of Down Syndrome is by - a) Klinefeltersyndrome
a) Karyo typing (All India Dec15 Pattern) b) Turner sl,ndrome
b) Triple test c) Cri du chat syndrome
c) Fetalultrasonography d) Noonan syndrome
d) All of above 48. A nineteen year old female with short stature, wide
39. Commonest cause of intestinal obstruction in spread nipples and primary amenorrhoea most
dowrfs eyndrome - (AIIMS lune 2000) likely has karyotype of - (Ar 03)
a) Colomic atresia a) 47,XX+ 18 b) 46,XXXY
b) Intestinal atresia c) 47,XXv d) 4sxo
c) Duodenal atresia 49- Edema of hands & feet in Infants is characterised -
d) Oesophageal atresia a) Klinefelter's sl.ndrome (NEET Dec.12 Pattern)

40, Down's syndrome most commonly occurs due to - b) Noonam syndrome


a)Reciprocal translocation (At 10)
c) Tirrners syndrome
b)Nondysjunction in maternal meiosis d) Fragile x syndrome
c)Translocation defect 50. Most common cadiac anomaly in Tirrnert syrr&orne
d)Nondysjunction in paternal meiosis is?
a) Coarctation ofaorta (CET Nov.14 Pattern)
41, Moct common chromosomal syndrome is-
b) Bicuspid aortic valve
a) Fragile X-syndrome (AIIMS Dec 9a)
l. Ans. is t' i.e., Used to see genetic transmission lRef: Nelson 18th/e ch. 80)
2. Ans. is 'd' i.e., A polygenic syndrome [Multiple References www.som.tulane.edu/departments/peds_respcare/
genetic.htm)
A syndrome with facial anomalies including a thin upper lip, growth delay and developmental disabilities is likely
to result from either a teratogenic congenital malformation, a single gene defect (Mendelian) or a chromosomal
disorder. A trait is called polygenic if multiple genes are thought to contribute to the phenotype and includes
conditions like diabetes and hypertension. A polygenic disorder is the least likely diagnosis amongst the options
provided and the single best answer of exclusion.

AUTOSOMAL DOMINANT

3. Ans. is 'd i.e., Fabry's disease {Ref : Robbin's Vh/e p. 151)


r Skeletal disorders with autosomal dominant inheritence are Marfan sl.ndrome, EDS (some variants), osteogenosis
imperfecta and Achondroplasia.

4. Ans. is'b' i.e., 25Yo fRef: Robbin\ 7h/e p. 160, 1611


r Achondroplasia is an autosomal dominant condition.
t If both the parents are affected than the chance of having unaffected baby is 25o/o.
Aa Aa

In this situation it is tricky to calculate the percentage of having affected baby.


Most of you must be thinking 75o/o (very easy).
But, it is not the case, as already explained homozygous state is not compitable with life and fetus dies in utero.
So, the chances of having affected baby will be 5070 because 25yo (AA) will die in utero.
5. Ans. is'a'i.e., Autosomal dominant (most likely) [Ref : Readbelowl
o It is typical presentation of waardeburg syndrome type - 1. -+ Widely spaced eyes, Hearing loss, Increased facial hair
o It is an autosomal dominant disorder.
o In autosomal disordet if one parent is affected and the other is normal, 25o/o of children (1 out of 4) will manifest
disease.

AUTOSOMAL RECESSIVE

6. Ans. is t' i.e., 50% children are affected, Rest are carriers {Ref: Robbin's 7h/e p. 1611
. One parent is affected and other is carrier -+ 50% of child will be affected, 50% will be carrier.

n Ans. is t' i.e., 0 fRef: Robbin's 7h/e p. 1511


o Cystic fibrosis is an autosomal recessive disorder.
r If husband and wife are clinically normal and the child has developed the disease that means both husband & wife
are carrier.
r See below

o 25vot wittbe affected witl be nflr**nrly normal.


^lirr"
o If one child is already affected, the chance for the disease in the second offspring is zero.

8. Ans. is 'd' i.e., Uniparental disomy lRef : R.ead belowl


c If one parent is normal for an autosomal recessive disorder and the other is carrier. No child should be affected.

Aa Aa

c All should be phenotypically normal, because autosomal recessive disease manifests in Homozygous state.
o But there is exception to this rule, i.e. uniparental disomy.
Uniparental disomy
o Uniparental disomy occurs when a person receives two copies of a chromosome, from orie parent and no copies from
other.
* In this condition if the child receives ,two copies of the affected chromosome from the carrier parent, he/she will
manifest the disease.

9. .A.ns. is 'a' i"e., Autosomal dominant disorder {R"ef : Rabbinb Vh/e p. 160, 16l)
In Autosomal dominant disorders indiviiluals are affected in successive generations. The disease does not occur
in the offspring of unaffected individuals' -
e If both parents are normal, the disease cannot be autosomal dominant because it is manifested in Heterozygous
state, So, even a single gene affection amongst the allel will cause the disease to be manifested. To be normal, a
person should have normal copy of both allel. If both parents are normal, that means there is no abnormal gene.
o At least one of the parents should be affected to transmit the disease to child.

X-LIhNKED

10" Ans. is 'a'i.e., 50% of boys of carrier rnother are affected lRe! &obbins Vh/e p. 1521
o 50% boys of the carrier mother will be affected.
e All daughters of affected father will become carrier, i.e. affected father will transmit the defective X-chromosome to
all the daughters.
o Father will not transmitt the disease to son as boys do not inherit X-chromosome from father.

11. Ans. is 'a' i.e., Xlinked Recessive {Ilef: Rabbin's Tnle p. 152}
e The clues in this question are -
1) Only males are manifesting disease.
2) Females are acting as carriers -+ sisters sons had suffered from the disease.
r Both these are features ofX-linked recessive disorders.
About other options
e In X-linked dominant disorder the females will also manifest the disease.
o In autosomai dominant and autosomal recessive disorders, both males and females are affected.

tz. Ans is 'd' i"e., X Linked R.ecessive fRef: R.abbin's Vh/e p. l52l
o Similar to above question, only males are manifesting disease and females are acting as carrier.
c It will be easier to solve this question if I will change the represention of Pedigree -

XY (Normal male) XdX (Canierfemale)

XY xx d
XY xdy xx
().Jormal male) (Carrier female) Qrlormal male) (Affected male) Q.{orma1 female)

d
XY XY
d
XY XY xx
d
xx
o The Pedgree starts from normal male and carrier female.
r Carrier female transfer the mutated X-chromosome to half the male children and half of the female children.
r Second generation pedgree starts when one of the carrier female marries to normal male.
r Here also she transfer her mutated X-chromosome to half of the male children and to half of the female children.
t3. Ans. is 'd' i.e., X linked dominant lRef: Chandrasoma Taylor 3'd/e p. 232, 233]
o It is an X-linked dominant pattern of inheritance.
r The features which suggests the X-linked dominant pattern of inheritance are-
r A single abnormal X chromosome is sufficient to express the disease.
r A11 of the female offsprings of a diseased male XY will receive the abnormal X chromosome and express the
disease, whereas none of the male offspring to the diseased father will have the disease (as sons dont receive X
chromosome from their father).
: A diseased mother can transmit the abnormal X chromosome to both daughter and sons equally.
14. Ans. is t' i.e., Mitochondrial inheritance fRef: www. springerlink)
r This pediagree chart has following two characteristics : -
i) Disease is manifesting in both males and females.
ii) But, the diseaSe is transmitting to next generation only by females (mother).
r Both these are characteristics of mitochondrial diseases.
All children from affected mother will inherit the disease but it will not be transmitted from an affected father to his
children. --- Harrison's 16rh/e 374

GENOMIC IMPRINTING

15. Ans. is 'd i.e., Genomic imprinting [Rel O.P. Ghai 8*/e p. 6i7 6 7h/e p. 612; Rabbins Th/e p. 186)

The pthenomenon referred to as genomic imprinting leads to pereferential expression of an allele depending on
its parental origin. --- Harrisons

16. Ans. is t' i"e., Genomic imprinting [Refr O.P. Ghai Y'h/e p. 637 (t 7h/e p. 612; Robbin's Vh/e p. 186)

r There are functional differences between paternal and maternal genes. So the expression of gene will be different
depending on, whether it has transmitted by father or mother. This is called genomic imprinting.
\v. Ans. is'a' i.e., Prader-willi syndrome [Ref : Nelson 18th/e p. 498, 51j, 5161

t ln Prader- Willi syndrome, about 60% of cases have maternal uniparental disomy (maternal UPD- missing the paternal
chromosome l5).
18. Ans. is ?' i.e., Ghrelin [Ref NORD Gaid.e to Rase Disorders Lippincott Williams 2002 p. 2j7]
o Prader Willi Syndrome is associated with elevated circulatinglevels of Ghrelin.

PLEIOTROPY & ATVISM

19. Ans. is'd


i.e. Pleiotropy [Rel: Internet referencel
t Pleiotropy oicurs when a mutation in a single gene prcduces effects on more than one characteristic, that is
causes multiple mutant phenotypes.

20. Ans" is t'


i.e., Grand parents {Ref: Taber's Medical Dictionaty)
c Atavism means the appearance of a characterstics presumed to have been present in some remote ancestor d:ue to
chance recombination of genes or environmental conditions favourable to their expression in the embryo.

TRISOMIES

21. Ans. is'a' i.e", Trisomy f 3 [Re/ Robbin's fl/e p' 176, 177; O.P. Ghai 8il'/e p' 6j9 6 7h/e p" 615]
t Clefi lip, cleft palate, scalp defects suggest Trisomy 13 or patau syndrome.
r The highest discriminatoryt values are for ectodermal scalp d*cts and harelip and cleft Palate in trisomy 13 and
elongated skull and simple arches on all digits for trisom)t 18.
Cneerra r5
tl. Ans. is
nMqlme'
{}l.e{: Nets*w t7h/e Chap 8t Tabk 81"2]

Patau Syndrorne
o Trisomy of 13th chromosome o CIeft lip often midline a Polydacity with flexed fingers
* Ocular hypotelorism o Bulbous nose o Low set ear
c Holoprosencephaly o Microphthalmia o Cardiac mallormation
a Scalp defect o Hlpoplastic or absent ribs a Rocker bottom feet
23. ,4.ns. is 'b' i.e., Bilateral mieropthalmos [Rel Robbiw's 7h/e p" 176, ]77; A.P. Gkai &'hle p. 639 & 7h/e p. 6] 5)

x Important ophthalmological abnormalities in Trisomy 13 (Patau syndrome) are microphthalmiaand cloboma of iris.
ub'
21. Aats" is $.e., Patauz Syrx&rrau,ae {ltet': !{elson 17h/e Ch*p. 8} 'lable 8x.2"1
c Trisomy 13 - Patau Syndrome a XO - Turner Syndrome
o Trisomy 18 - Edward Syndrome o XXY - Klinefelters syndrome
* Trisomy 21 - Down Syndrome
25" Axes. is '&' i,e",War$an T'risormy 22 Ule.f: Darlawd's 2&'1'le p. 152{j}
o The supernumerary cat eye syndrome (CES) chromosome is dicentric, containing two copies 22q11.

DOW{I{ SYrllPR0ft,l8

to. Ams. is 'c' i,e", Downk Syndrorne $lef: A.P. Gkai $th/e p. 645 {r Vhle p. 5}4}
Nuchal fold thickness is inteased in Down syndrome. This feature is used in suspecting the diagnosis of Down's
syndrome prenataly.
o* iNelsow }&tt'/eChap.8t}
27. Axns, is i;.*..,A*zgxzitzw*iaxpairwzcxtt{Ilef:
* Cognitive impairement with developmental delay and mental retardation is universal.
*d'
)q Ams" is i"e., Hypertonicity [Rrf 8"P" Gbsri 9th/e p" 638 et 7h/e p" 613: l,{elsow n$th/e y:. 5{}8}
e Most striking feature in the neonate rs hypotonia and although the diagnosis is usually evident at this time, it may
some time be missed if the baby is very premature or his facial features are concealed by ventilatory apparetus.
lo Ans" is 'l)' i"e", dJndescended testis lRef : A,P. Gkai 8'h/e p. 638 dr Vh/e p. 613; Nelsaw 18tk/e p. 5811

* IJndescended testis has not been described as an association with Down's syndrome.

30. Ans" is 'd' i.e,, Respiratory tract infection uncommon [Rel O"F. Ghai 9th/e p" 638 dr 7h/e p. 613; Nclsan 18th/e p.
s081
e Chronic rhinits, conjunctivitis and periodontal disease are common in Down's syndrome.
a Lower respiratory tract infections pose a threat to life.
e Pneumonias are common in patients with Down's syndrome.
3r" is 'h' i"e,, ExaeXoeardial cushioee t{efeet [Ri,ii ]{elscm tr&Lt'le ch.6231
"&ras"
o Most common CHD in Down syndrome is ostium primum tlpe of ASD which is the simplest form of atrioventricular
septal defect also called as endocardial cushion defect.
32" Aras, is'tr' i,e., Dowm syndrome lRc.f: Na{saw }.8't'/e {:14{ep. 42b.51

o Ostium primum ASD (an incomplete atriventricular septal defect) or endocardial cushion defect is common in Down
syndrome.
Ana. is'# i.e., Atriovemtricular Septal De&et $&ef: R.*bbi*'s Ett'/e Chapter 811
o Approximately4}o/oofthepatientshavecongenitalheartdisease,mostcommonlydefectsoftheendocardialcushion
or ostium primum type atrial septal defects, atrioventricular valve malformations, and ventricular septal defects.
o Cardiac problems are responsible for the majority of the deaths in infancy and early childhood.
o Several other congenital malformation, including atresias of the esophagus and small bowel, are also common.
o#
-).+ " Am,s" isi.e., VS}} Ule, &relson 1E'hle ch.?; GkwiTt'/ep.6t3]
a In down syndrome, most common CHD is endocardial cushion defect (septum primum type of ASD).
-13. Ans. is 'd i"e", Normea[ tt"e-f: Awwa trap paedi*tric 18tl'le p.210t)]
o Narchi H, studied the value of routine neonatal ECG in l't 48 hours of life to diagnose CHD in 37 neonates with Down
syndrome :-
a, 650/o (24 newborns) had no CHD and normal ECG.
, 35o/o (13 newborns) had clinical CHD with :-
i) Half (7 newborns) of them normal ECG'
ii) Half (6 newborns) with abnormal ECG'
36" Ans. is
.a'
i.e., Increased PAPPA {Ref. Dutta */e p 492496, CGOT */e p 11; O-P- Ghai */e p. $s & */e p' 51a\

r PAPPA stands for pregnancy associated plasma protein'


r It is produced by the covering of the newly fertilized egg'
t Low levels of PAPPA are associated with Down's syndrome'
37 - Ans is t' i"e", Serum HPtr' levels {Ref: Datta &/e p" 530" 112; oP Ghoi */e P' 5a6tr

rTripletestisacombinedbiochemicaltestwhichincludes-+ MSAFP,HCG, (, Oestriol

38" Ans. is 'd i.e., All of above lRef: Ghai /'/e p' '614)
. See aext ofthe chaPter.
39. Ans. is
.b,
i.e., nntestinal atresia & t'i"e., Duodeaal atresia [Ref, 0"P" Ghai */e p- 638 & f le p' 613;
Ndsonl86/eP.5881
o Most common cause of Intestinal obstruction in Down's syndrome is duodenal atresia.
r Duodenal atresia is a tlpe of intestinal atresia'
4&. Ans. is "b, Le", Noadysjunction in materaal meiosi$ {&e$ See above explanationl

r In trisomy 21, the additional number 21 chromosome is derived from the mother
in 95o/o of cases'
I It is due to nondysiunction of 2 t st chromosome at the time of meiosis'
In nondysjunctiorL chromosomal pair fails to separate in first meiotic division during
gametogenesis'
r
4t- Ans. is i' ie." Trisomy 21. {Ref Rabbbfs *fe p. 175; o.P- Gtusi */e p- 637 & */e p' 6131

r Down syndrome (Trisomy) is the most common of the chromosomal disorder. It occurs in frequency of 1:800 to

l:1000 newborn.
42. .&ns. is t' i',e., t (X l:14) [Rei Nelson 17e/e Chap' 81)
: lo 95o/o of cases of Down Syndrome - trisomy of 21. Extra chromosome is of maternal in origin
o 10/o have mosaic with 46 chromosomes'
o 4o/o have robertsonian translocation'
t (.22 :21)
t (14 :21)
t (15 : 21)
o very rarely long arm of chromosome 21 is triplicate (Partial trisomy)

43- A.ns is 1d i.e." Deleted 2l {Ref, Ghoi *te p.637 & */e p.613, X4; Nelsan 1*/e p' 5091

TURNEN's & I{OONA}I sYI{DROME


p" 514; www'sprlng*r linkl
M" Aas" is "lJ i-e., Noonan Syndrome {Ref: OP Ghai *le p' 64$; Nsl'son 1*/e
r Antimongoloid siant, pulmonic stenosis, short stature and undescended testis (cryptorchidism) are seen in Noonan
sydnrome.
nb' norrnal Lvvel {lle! Nfdson 18'L/e Chapter 584; Thstostero"e defwiewcy
45, Aas, is i..e", Testosterone is always
in MenbY {twgJr loues P. 41)
o Testosterone levels are often low in Noonan syndrome'
r Ail other oPtions are true.
M. Ans" is t'i.e., Infertility in females {R{:Nelso* L*/e p" 5141
,Noonan that resemble
Term Syndrome' has been applied to males and females who have some phenotypic features
(45 X,O;, but themselves have apparently normal sex chromosomes (46 XX or 46 XY)'
females with turner's syndrome
r Females usualiy show normal sexual maturation but it is delayed 2years on an average.

4V. Aa* is
*S 1.7/e p. L925, I{wfisn 1*/e p" 2122; O.P. Ghai */e p' 6a0 & 7/e p' 6151
Le., Turoet's syadmme tR f: *)son
Webbing of neck, increased carrying
Noonan syndrome presents with most features similar to Turner's Syndrome.
Noonan's and Turner's Syndrome but short fourth metacatpal is
angle, low hair line are present in both
characteristic of Turner's syndrome only'
48. Ans. is 't i.e.,45X:O lRef: O.P, Ghai */e p. 540 dr tle p" 616|

49. Ans. is 'c' i.e., Turners Syndrorne lRef Ckai Vkle p. 616\

r Lymphedema of hand and feet is characteristically seen in Turner syndrome.

50. Ans. is'b'i.e., Bicuspid aortic valve lRef: Nelson 18h/e Chapter 58fi
o Turner's syndrome is commonly associated with congenitalheart diseases.
e The most comlnofl anomaly associated is bicuspid Aortic valves iru one third to one half of the patients 6A0/0).
r Other congential anomalies associated with Turner's syndrome -+ Aortic coarctation (30olo), Aortic stenosis, Mitral
valve prolapse, Anomalous pulmonary venous drainage

51. Ans. is "V Le,, Coarctation af aarblRef. Nelson ltlep. 23s7|


e Among the given options Aortic coarctation is most common.

MISCELLAHEOUS

52. Ans. is'b'i.e., SplRcl Nelson l*hle eh. &ll


Cri-du-chat syndrome
e A high-pitched cry like a cat, giving the syndrome its name.
e Genetic studies have confirmed that this characteristic cry results from the deletion at 5p15

The mainfe-atures*elrypatottitt,*r*ttstdure,drd.rfrrteistie uy,nticro*lhalywiihgrotruding


metopie suture, mooallkafaee, hygertelorism, bil*teral epieofilttc fol.d* higfu arched pal*te,
aide anil flat nas*l bridgo, anil uental reterdiltlon. Ceriliac ab*orutaliti*s incltd.ing
vantriculm septal ilefeet, *trial septal dSed, patcnt dudas itrterlosus anil F*Ilot's tctralogy.

53. Ans. is 'U i.e., Pigmented nem l\ef; Harrkan' 18h/e Chapter 61; Disarder af development and Iearning, Val-( by
MarkWotraie p.2261
c Clinical features of fragile - X syndrome
: Mental retardation : Long face with large mandible r Hlperexntesible joint r Mitral valve prolapse
r Large everted ears t Large testis (macro-orchidism) r High archedpalate
54. Ans. is''b' i.e",45 X O genome
Barr body (Sex - chromatinl
e It is a densely staining inactivated condensed X chromosome that is present in each somatic cells af female.
o It is found inthe nucleus.
e It is used as a test ofgenetic femaleness -+ it is possible to determine the genetic sex of an individual accordlng
to whether there is a chromatin mass present on the inner su;r{ace of the nuclear membrane of cells with resting
as
or intermitent nuclei. Remember following fact and the question will seem yery eacy.
e Chramatid boily (Barr body or sex chramatin) is derived from one of the two X-chromosomes which becames
inactivated.
e The nurher of Barr bodies is thus one less than the number of X-chromosomes.

ilote-
o Barr body is found in female But -
r Kleinefelter syndrome is male with Barr body.
r Turner syndrome is female without Barr body.
55. Ans. ist'i.e.,Dcwn'ssyndromelRef:Nekontcxtbookafped,iatricsl&h/eChapter8l,587tablc&1-1&51-21
c In the given karyotype, there is trisomy of 21 chromosome, which is characleristickaryotype of Down syndrome (see
above explanation).
56" Ans. is?'i.e.,Turaer'ssyndromelRef:Netsantcxtbaqkafpc.diatrtesl*leChapter8l;Harrisanblshle
Chqpter 349; Rohbins Vhlc p. 179]
r In the $venkaryotype there is single X chromosomes and ail. other pairs are normal i.e., 45XA, which is diagnostic of
turner's svndrome.
Ans. is '* i.e., Karyotypingl&ef,Gkai Sth/e p. 64a d* Vh/e p. 6151

Ans. is 'b- i.e., l{eural tube defects (Spina bifida) {R"ef Textbaok af envirorumental & Genetic Faedia{rics}
o In North India, neural tube defect is most common.
o In rest of the India, musculoskeletal defects are more common.

AT\'5WER5 OF VARIO[.!S OTI{EN EXAMINATIONS

59. Ans. is'd' i.e., Fragile X syndrorne lRef: Robbin's Vh/e p" 182)
60. Ans. is t' i.e., Brachicephalic [Rel; O,P. Gkai $tule p. 638 d? Vt'/e p. 61j]
dt. Ans. is'd'i"e., Rocker bottom feet [Rel Nelson lVhle p. 384; O.P. Ghai Sthle p. 539 d2 7h/e p. 614]
62. Ans. is'c'i.e., Chromosomal abnormality lilef Nelson 18th/e p. 5141
63" Ans" is'd i.e., AR [Rel Pediatric Urology l't/e p. 145]
a f eune syndrome, also called asphyxiating thoracic dysplasia, is a rare autosomol recessive condition character rzedby
skeletal abnormalities, along with renal, hepatic and pancreatic abnormalities.

rrt
XNFKCry'XMEJS
MXSKASK

VIRAL INFECTIONS

RUBELLA (GERMAN MEASLES)

o Rubella is caused by enveloped RNA virus belonging to TogoviridaeailMs e2).


r Rubella is transmitted either (1) By droplet infection through respiratory route, or (ii) Transplacental vertical transmission.
o The usual age of postnatally acquired rubella is 3- 10 years.

Postnatal (acquired) rubella


o Incubation period is 2-3 weeks (14-21 days)61e3),
r Clinical f,eatures are:-
l) Prodrome (fever, sore throat, coryza)
ii) Lymphadenopathy (posterior auricular, cervical, suboccipital)
iii) Rash which begins on face and spreads down the body.
o Furschheimer spotsare3) is a petechial exanthem on soft palate.
o Complications include arthralgia(K""r" ss), encephalitis and thrombocl'topenia.

Congenital rubella syndrome


o Maternal viremia associated with rubella infection during pregnancy may result in infection of the placenta and fetus.
The most imqtortant factor in the pathogenicity of rubella virus for the fetus is gestational age at the time of infection.
the damage to the fetus@r 0s' . Maximum damage to fetus
AIrMS 0s)
The earlier in pregnancy infection occurs, the greater
occurs when infection is acquireil in first trimester of pregnacy.Infection in early 2"d trimester causes only deafness.
Infection after 16 week causes no major abnormalities@ros'AllMs11'08).
o The classical triad of congenital rubella consists of cataract, deafness and CHD@Iq\) .
. Preseflce of IgM in blood at birth is diagnostilil os),
as is the persistance of IgG afier 6 months of age4r os)
.

Importart clinical features of cong€nital r$bella sydrome


) Organ of carti I Senscrineural hearlng lass fftlast comman cllnieal findlng at congenital ruheltal.
t Heart: PDAoG'$'Ha2| (Moat common CHD in congeaital rubellawri)], PS{P.ct,,}rVSDtDrtB$,PG,oe, AgDootretTOF,
pulmonary artery hypoplasia.
* Eye : CatoruffEq tans oi, ystlno?athy, microphthalmia, myopia, glaucoma.
* CNS: Met*sl retasdationw o2r, spastic diplegia, mlcrocephclfxcm, seizures, behavioral disorders.
s Ather: IDDM, thyroid disorders, inguinal hernia, cryptorchidism, iaterstitial pneumonia.
) Charcct€risticskin lesion of cequired rubellaii absenitPcxo. The skin lesion in congenital rubella I
{i)Thrombocytopenicpurpura; (il'Blueberryrnuffrn'lesions.

MUIIIPS
o Mumps is caused by a RNA Virusbelongingto famtly paramyxoviridae.
o Man is the only nafural host, and sources of infection are clinical or subclinical cases.
r Infection is acquired by air droplets through regtiratory route.
c Period of infectivity (communicability) is 4-6 days before the onset of symptam to a week thereafter {N oo.

o Secondary attackrate is 86o/oGrEEr).


o Incubation period is 2-3 weeks (14-21 ilays)(rN ee).
30-40o/o of infections are subclinical.
I The typical clinical case presents with pain and swelling of one or both parotid salivary gland @EEr). Sublingual and
submandibular glands may also be involyed@*).
o Orchitis is the most comtnon coflplication among postpubertal malesar 0o).
Overall, aseptic meningitis is the most
comrnon complication and. may develop before, during afier or in the absence of parotitis 1.106).

r MumpsvaccineisaliveattenuatedvaccineqtEar),preparedfromlerylllynnstrain?Glo6).Oneclinicalattackofmeasles
also confers life long immunity.

cHlcr(Elr Pox
r Chicken pox is caused by Varicella-Zoster Virus (HSY-III).
r Infection is transmited by air droplets.
o Incubation period is 10-21 daysqerot).
o Infectious Tteriod is 2 ilays before to 5 days afier onset of rash.
o The crusts (scabs) from chicken pox are not infective, i.e. crushed lesions are non-infective?cl 06' 01).

r Chicken-pox usually affects children of age gronp 5-9 years. Clinical features of chicken pox has two stages :-
1) Pre-eruptive stage : Characterized by fever, back pain, malaise and shivering.
2) Eruptive stage: It is characterizedby a characteristic rash with following features (which must be differentiated
from rash of small pox) :-
i) Superficial@I ts) and unilocular(Ar 1' (rash of small pox is deep seated and multilocular).
i, Symmetrical, mostly onflexor surfaceqllMslo) (rash of small-pox usually affect extensor surface).
iii) Centripetal distributional (rash of smallpox has centrifugal distribution).
13)

iv) Affects axilla but spares palms and soles (rash of small-pox affects palms and soles but spares axilla).
v) Pleomorphic(Ar 13), with appearance of all stages of rash simultaneouslyuttus to) (rash is monomorphic in nature
in small-pox).
vl) Dew-drop on rose petal apperance6IIMSe3) (rash of small-pox has umblicated apperarance).
vii) Area of inJlammation around the rash (in small-pox there is no inflammation around the rash).
vll|)Evolution of rash is rapid, scabs begin to form 4-7days after rash (evolution is slow, scabs begin 10- 14 days
after rash).
r Though the disease affects children commonly, it is more severe in adults than in children,
',,...,.'..:, r;li,,:01ii*i#c$,!$1i,.- ii
Most common complication of varicella is secondary bacterial infection of skin, usually caused by streptococci or
staphylococci. The most common extracutoneous site of involvement in children is CNS. Varicella pneumonia is the
most serious complicationfollowing chicken - pox in adults.
r Important complications of chicken pox are:-
i) PulmonarT.' Pneumonia(Arrrs86 uPsc85)
ii) CNS : Cerebellar ataxia, encephalitisarMs86 uPsc8s), meningitis, GB syndrome, Transverse myelitis.
iii) Others : Myocarditis, Nephritis, arthritis, hepatitis, corneal lesions, thrombocytop€ni4(e[arssr, wscrs), bleeding
diasthesis.
o Latent infection is established in cranial nerve, sensory ganglia and spinai dorsal root ganglia. The virus may reactivate
in immuno-suppressive conditions, resulting in herpes zoter in about 10-30% of persons(Arusto).
e Perinatal transmission of varicella occurs when VZV crosses the placenta@t8e)afid infects the fetus. It is most
threatening if transmitted during 1't trimester and can cause fetal yaricella syndrome (cicatrising skin lesions(Ar 8e),
limb hypoplasia, chorioretinitis, microcephaly).

MEASLES TRUBEOLA}

r Measles is caused by a RNA virus(pcros) of paramyxvirus family@Gr0s). So far only one serot,,pe (antigenic type) of
measles virus is lulownaro8).
o The only source of infection is a case of measles, carrier are not known to occurAilMs os' e8)
.

o Infective period (period of communicability) is 4 days before and 5 days after the appearance of rash, infectivity is
maximun during prodromal stage@roa).
t Secondary attack rate is 80o/o@108) (less than rubellaqltMs0''n') which ftas SAR 90-95%),
o Age group most commonly involved is 6 months to 3 yearsal ro), with involvement of.male and female
os, e8).
equallyarMs
a Transmission is through respiratory tract by droplets (mainly) or sometimes through conjunctiva. Measles shows
cyclic trendares) with epidemic every 2-3 years(Pcroa).
a On attack of measles confer life long immunityutas),
a Maternal antibodies provide immunity upto 6 months of age(@ce
os)
.

a Incubation period is l0- 14 days@cr ar 0a e6),


os'
i.e. 10 days from exposure to onset of fever and 14 days from exposure
to appearance of rash. Incubation period for live vaccine virus is 7 days,

Clinical features
r Clinical features are divided into following stages :-
1) Prodromal stage
r It begins 10 days after the infection and is characterized by fever, coryza with sneezing, lacrimation and
photophobia. A day or two before eruptive phase (appearance of rash), Koplik spot(Pci 04) (pathognomonic of
measles@"')) appears on buccal mucosa, opposite the l"t and 2"d lower molars(pcr05).
2) Eruptivephase
I It begins 14 days after infection. A maculopapular rash begins behind the eArseGI se) and spreads rapidly.
3) Post measle stage
r It is characterized by weight loss, growth retardation, diarrhea, cancrum oris, candidiasis, reactivation ofTB
,*
and malnutrition particularlyvitamin A deficiency (keratomalacia,ecl03,02,o0), xerophthalmia).
$
i r Complications of measles are :-
k
i)
i Respiratory: Otitis media (most common complication)rero'), pneumonia\I
pneumonia(Pcros)), bronchitis, laryngitis, croup.
07) (Giant cell or Hecht's

ii) CNS: These are most seriousoEEr) and include encephalitis, transverse myelitis and a rare complicationGr0T) SSPE
(Subacute sclerosing panencephalitis)@et oz).
iii) Gastrointestinal: Gastroentritis, hepatitis, appendicitis, diarrheaar0T), ileocolitis, mesenteric adenitis.
iv) Others : Myocarditis, glomerulonephritis, thrombocytopenia.
ERYTHE'VIA IN FECTIOS[JM {FIFTH DISEASE}

o Most common clinical manifestation of human parvovirus infection is ery'thema infectiosum.


e Most common complication is arthritis.
o Incubation period is 4 to 14 days.
a Prodromal illness is minimal.
s after defervescence@Glqs) (disappearance offever).
Rash appears 1 week
o Characteristics skin lesions appear in 3 stages :-
w fl phase: Cheeks are erlthematous giving appearnce of slapped cheeks(Pcr04).
a 2d phase: Itching erlthematous maculopapular rash on extensor surface (Palms and soles spared).
e 3'd phase: Rash begins to fade and disappears in about 2 weeks.
o parvovirus may also cause aplastic crisis in children(Aree) with chronic hemolytic anemia, e.g. in sickle cell disease.
Parvovirus B19 infection during pregnancy may casue non-immune fetal hydrops(Ar8e).

MOSEOLA I N FANTI'! M {sIXTI-I DISEASE OR EXANTH ETVI 5U BITTJ M)

ost
o It is caused by human herpes virus-6 (HSV-61oet .

o Similar illness is caused by Parvovirus B-19 and Echovirus. Age group involved is 6 mt to 3 yr.
* Incubation period --> 5 to 15 daYs'
c Clinical manifestations include abrupt onset with high grade fevet mild pharyngitis and Coryza. The temprature
01' 00' Iipnet es)
comes down rapidly in 3 to 4 ilays when a macular or maculopapular rash appear@IlMs starting from
'
trunk and sometimes extending to the extremities and face?et . Rash lasts for 24 hours, and disappear in 1-2 days
os)

quam ati o n( AItMs


1'
ith o ut p igment ati o n o r
0 0 0)
w d ese .

01' 00).
e There is occipital and post-auricular lymphadenopathyqrtMs

I [U FEeTlCIt' 5 MoN0N UCLEoSIS

e Infectious mononucleosis is the most characteristic disease produced by Epstein Barr Virus.
* Most infections are asymptomatic.
o Headache,Abdominalpain,andchillsarealsoseeninfewcases.Thereisfeveralo2'AttMse3),malaise,sorethroat,(exudative
pharyngitis), loss of appetite and generalized lymphadenopathy : Posterior and anterior cervical LN are almost always
enlarged(Ar 02, AilM e3). Splenomegaly is found in 50-75o/o cases while hepatomegaly is found only in 3|o/oat 02) cases.

Maculopapuiar rash maY be seen.

Laboratory findings :
a) Atypical Lymphocytosis$t 02' AItMs e3) ' Comprising over 10%o of total lymphocytes at sometime in the illness.
b) Paul Bunnel test is Positive
c) Monospot test is the screening test

HEPATITIS VIRUSES

Famlly Picornavirus Hepadnavirus Flavivirus Defective virus Calcivirus


(enteroviru s-72)

{& vrrotogyl,: ,. RN,A ' . ,', ', DNA ',.1 : RhlA :., ,ir:-RNA..: :. .RNA'

s lncubation I S-4skon.e6) 30-1 80 1 5-1 60 30-1 80 14-60


..,..1. .,..:..:....:.rr:a:ta-.....t.i.11

ci,qpT]ER|rf,let:

T?ansimilsion'1.'.:...
ixPit-"[$,i 1'..'''.".l':...r
ii)'fecojoial.,.,:,"- :
rili)se,Ilta*i r:.:,ir :r:: :::

irij.P.6rinq1tJ,.. "
-.,' .;, i

a Chron_fc infection No
.
'':.: '::,,:':: :. .Y'eS:''t''. ''::aa,,' ri
t ':.Fu kn inent df ssase,. ,,: :1rR5ia, lll .':,'

a Age-group Children AnY age

c Maximum perinatal transmission is seen in HBly'Atq3) '


e7)
o Most common cause of HBV transmission in children is transfusion of blood and blood proilucts(AttMs '

who are
c Most important riskfactor that determines pterinatal transmission is HBe AglAr03). HBs Ag carrier mothers
where as catrier mothers
HBe Ag positive almost invariably (> 90o/o) transmit hepatitis B infection to their offspring,
with anti HBe rarely (10 to 15%) infect their offspring'
o Infants born to Hbs Ag positive mothers should receive both HBIG and HB vaccine@t,\ee'AIIMS('') intramuscularly
at seperate sites within 12 hours of birth, followed by 2nd and 3rd dose of vaccine
at I and 6 months respectively'

PO!.IOMYELITIS

o poliomyelitis is an acute viral infection caused by Poliovirus. Poliovirus selectively damage


motor nerves and
autonomicnerves(ArMse4).Mostcommonlyaffectedareasare anteriorhorncellsattMse4)ofspinaicord,vestibularand
cerebellum . Sensory nerves are
cranial nerve nuclei, vital centers in the medulla and vermis, and nuclei in the root of
not involved.
r Most common age group affected is 6 months of 5 years'
o Incubation period ranges from 3-35 days (usually 7-14 daystAls3)). The clinical spectrum includes
:-

I) Subclinical (inapparent) infections (g5o/o) : Most commonallMsT') and play predominant role in spread of infection'
ii) Minor (abortive) illness (4.8o/o): Present with fever, sore throat, headache and maiaise.
111) Asepticmeningitisat")/non-paralyticpolio(1o/o):Therearesignsandsymptomsofmeningitis'
paralysis
") with absent teflexes\I13). Respiratoty
ts' 1-rus
iv) paralytic polio (<1%)(Ar 13) : There is flaccid paralysisLt
is the most common cause of deathate2)'
* sternous physical exercise(AilMs
Riskof paralytic polio is itrcreasedby:Tonsillectomy, tooth extraction, adenoidec tomy,
or), fatigue, intramuscular injection\llMs oz), and cortisone administration.

Paralysis in polio is characterized by:


1. Descending 4. Non progressive
2.Asymmetricol 5.No sensory involvement
3. Proximol muscles > distal muscles 6. Lower motor neuron tYPe

c Deep tendon reflexes are absent(ArtMseT),


o Most common muscle affected + Quadriceps
o Most common muscle undergoes complete paralysis -+ Tibialis anterior
o Most common muscle affected in hand -+ Opponens pollicis'
s M.C. cause of death -+ Respiratory paralysis@fiusszt '
o Following signs can be elicited :
1. Tripod sign -) Child is asked to sit up unassisted. He assumes tripod posture'
2. Kiss the knee Test -) The child cannot kiss his knees due to spine stiffness.
3. Head drop sign -) Hand is placed under the patients shoulder and the trunk is raised. The head lags
behind simply.

Treatment of poliomyelitis
o There is no specific treatment.
. Management is primary s),mptomatic and supportive.
1) Bed rest -+ Because physical activity and trauma increase the risk of paralysis.
2) Relief of pain and muscle spasm -+ Hot moist packs applied to the muscles relieve pain and spasm (sisfer
kennY's tre atm ent )( etus os ).

3) Optimum position of limbs -+ To prevent deformity.


4) Physiotherapy
5) Good nursing
6) Rehabilitation -+ Physical, emotional and phychological.

Acute faccid paralysis (AFP) surveillance


o Every case of acute falccid paralysis in any child under 15 years is to be reported pro mptly(er ozt,
o Fecal samples of all these cases are obtained (2 samples) and transported to the laboratory wi thin Z2 hrs of collection
at 4-8' C or frozen at -20'C. The process is also knwon as reverse cold chain.

HIV INFECTION IN CHILDREN

o The primary route of infection of HIV in pediatric population is ve rtical transmissionfrom mother to child. probability
of transmission from mother to infant/fetus ranges between 20-30o/oGI 13, NEED(L5-25)/o in developed countries and
25 -35o/o in developing countries).
t Mostlyvirusistransmittedduringdelivery(perinatal)@Gle7,Ketutas7). Itmayalsobetransmitted.transplacentalduring
pregnancy (in utero)(k'"t"e') and postnatal by breast feedinglK*,r"tz) (leastcommon).
o The risk factors for vertical transmission include high maternal viremia (most important
factor), preterm delivery (<
34 weeks), first-born twin, Vitamin-A deficiency, prolonged (> a h$ rupture of membrane, LBW (> 2.5 kg), and low
maternal CD4 count.
o Cesarean section reduces the risk of transmission(Pcr 08' AilMS 03)
.

o Other routes of transmission in children is transfusion of blood and blood products. Sexual transmission is seen in
adolescent population.

Clinical features of HIV infection in children


o Failure to thriyeGtlMs e4)
is universal(.4rr5). o Lymphadenopathy
c P. Carinii pneuffionia-+M.C. AIDS defining illness in 6l11l47"ntenus $). 1 Dermatological complications
r Recurrent and chronic bacterial infections (first sign). o Candidiasis
r Otitis, Sinusitis, Pneumonia ' o Lymphoid interstitial pneumonia\IlMse4)
. HeP ato sqlenom egalyAnM s e4 )
o Cardiomyopathy
o Anemia / Thrombocytopenia / Neutropenia o Hepatitis
. Kaposi sarcoma is not seen usually4ttusstl.

Diagnosis of HIV infection


o In young infants (1-6 months) :-
1) Viral detection assays are used for the diagnosis of HIV infection in infants. These are DNA-pCR(DNB 12) (most
commonly zsed), RNA-PCR, p-24 antigen assay@NB '12), immune complex dissociated p24 antigen, and. HIV
12)
cttlture@NB . These are very useful in young infants allowing a definitve diagnosis in most infected infants by 1-6
months.
2)ELISA or Western blot test are not as reliable in young infants@NB 12).
o In older infants (> 6 months), detection of anti-HIV IgA antibodies by ELISA is diagnostic.
o In children (> 18 months) demonstration of anti-HIV IgG antibodies by ELISA is used.
CHrrP,r.E.sj

Methods to prevent vertical transmission


:-
r Methods to prevent vertlcal transmission are
1) AntiretroviralProPhYtaxis
by giving antiretroviral thetapy to mother and eaily
r vertical transmision can be prevented substantially
prophylaxistonewborn.singledosenevirapineecl0s'98,Allns06,03)fomotheratonsetoflabourfollowedbya
dose to newborn within 72 hours of birth
is theprefered regimen in developing countries' Zidovudin(Pc'
single
08,e8,ArrMs06,03)is also commonly used. It is given from 2nd trimester to throughout the pregnancy and for 6 weeks

after birth to infant'


2) Elective caesarian section'
3) Breast feeding
infant formula (top
r v infected mother is usually contraindicated.rn developed countries
Breast feeding by Hr
(diarrhea' pneumonia'
i. ...o*In.nded. However in developing countries where other disease
feeiling)@Grori outweighs the
high infant martality rate, the benefit of breast feeding
malnutrition) substantially contribute to a
cauntries should breast feed theit infants
risk for HIV transmission, and HIV infected women in developing
followed by rapid weaning'
forfirst 6 months of li|strctott
to
o The vaccines that are recommended in
the nuli:L?ljTtYnil,]'.?l schedule
:11^!."-u9rn'n'stered
be given oPV and
Hrv infected children except that sfr,ii"^rliriti iifectea cniruren should not

BCG?GIB'1il.

for infants and children


RevisedWHO clinical staging of HIV/AIDS

ClinicalStage 1

r AsYmPtomatic rs)
r Persistent generalized Lymphadenopathya/
Clinical Stage 2
I HePatosPlenomegalY
r PaPular Pruritic eruPtions
r Seborrhoeic dermatitis
I Extensive human papilloma virus infection
r Extensive molluscum contagiosum
I Fungal nail infections
r Recurrent oral ulcerations
r Lineal gingival erl'thema (LGE)
I Angular cheilitis
r Parotid enlargement
r HerPes zoster
r Recurrent or chronic RTIs (otitis media' otorrhoea' sinusitis)
Clinical Stage 3
adequately responding to standard therapy
: Moderate unexplained malnutrition not
(14 days or more )
r Unexplained persistent diarrhoea
lUnexplainedpersistentfever(intermittentorconstant,forlongerthanonemonth)
r Oral candidiasis (outside neonatal period )
r Oral hairY leukoPlakia
r Acute necrotizing ulcerative gingivitis/periodontitis
r PulmonarY TB testing is necessary
con?rmatory diagnostic
r Severe recurrent presumed bacterial pneumonia conditions where
r Chronic HlV-associated lung disease including brochiectasis
r Lyrnphoid interstitial pneumonitis (LIP)
rUnexplainedanaemia(<8g/dl),andorneutropenia(<1000/mm3)andorthrombocltopenia(<50000/mm3)for
more than one month
Clinical Stage 4
r Unexplained severe wasting or severe malnutrition not adequately responding to standard therapy
I Pneumocystis pneumonia
r Recurrent severe presumed bacterial infections (e.g. empyema, pyomyositis, bone or joint infection, meningitis,
but excluding pneumonia).
r Chronic herpes simplex infection; (orolabial or cutaneous of more than one month's duration)
I Extrapulmonary TB
I Kaposi's sarcoma
T Oesophageal candidiasis
I CNS toxoplasmosis (outside the neonatal period)
r HIV encephalopathy
r Conditions where confirmatory diagnostic testing is necessary
r CMV infection (CMV retinitis or infection of organs other than liver, spleen or lymph nodes; onset at age one
month or more)
r Extrapulmonary cr)?tococcosis including meningitis
r Any disseminated endemic mycosis (e.g. extrapulmonary histoplasmosis, coccidiomycosis, penicilliosis)
r Cryptosporidiosis
r Isosporiasis
r Disseminated non-tuberculous mycobacteria infection
r Candida oftrachea, bronchi or lungs
r Visceral herpes simplex infection
r Acquired HIV associated rectal fistula
r Cerebral or B cell non-Hodgkin lymphoma
r Progressive multifocal leukoencephalopathy (PML)
r HlV-associated cardiomyopathy or HlV-associated nephropathy

BACTERIAL INFECTIONS

PERTUSSIS (WHOOPING COUGH)

o Pertussis,alsocalled'100daycough',iscausedbyBordetellapertussis(only5%casesarecausedbyB.parapertussis).
o There is no cross immunity[AflMse7) between B. pertussis and B. parapertussis.
o Source of infection is a case of pertussk@I 13' PGL'),Ihere is no subclinical case or chronic carrier t1^1"6lrMs e7' Ares).
r Period of infectivity (communicability) extends from a week after exposuse to about 3 weeks after the
onset of paroxysmal stage. Catarrhal stage is most infectiveares AIMSeT). Secondary attack rate is high, i.e.
es, PGI 12, AIMseT),
90o/o@I

. More cases occur during winter and spring months@Grl2) due to overcrowding.
e It is a disease of infantsar es) and young children (< 5 years). Infants are susceptible to infection from birth because
maternal antibody does not give them protection(ArrMs e5). Incidence and fatality are more among femalearmses) than
male children.
o Modeoftransmission-+Whoopingcoughisspreadmainlybydropletinfectionanddirectcontact.Eachtimethepatient
coughs, sneezes or talks, the bacilli are sprayed into the air. Most children contract infection from their playmates who
are in the early stages of the disease. The role of fomites in the spread of infection appears to be very small, unless
they are freshly contaminatedaliMs e5).
o Infective material -+ The bacilli occurs abundantly in the nasopharl'ngeal and bronchial secretions, which are infective.
Objects freshly contaminated by such discharges are also infective.
o Incubationperiodisl-2weeks(7-l4days)ecl12'AIIM'10).Thediseasetakesaprotractedcoursecomprisingthreestages,
each lasting for about two weeks :-
1) Catarrhal
ii) Paroxysmal
C.ulpia'n:...i5u,
i.rl) Convalescent
o Catarrhal stage is the stage of maximum infectivityremrseT,Ales) . Maximum complications occur in paroxysmal stage.
o Important complications of pertussis are atelectasis, emphysema, bronchiectasis(A[Ms e7), pneumothorax, otitis
media, encephalopathyarMseT), convulsions, ataxia (non-cerebellar\IlMssT)),hernia, rectal prolaps e, subconjunctival
Ms 7),
h em o r rh ageqrr
e
ifltr acranial hemorrhage and s evere malnutrition.
r For diagno sis, gold standard is isolation of organism in culture from nasopharyngeal secretionallMs 11' 0s, AI 11).
o Erythromycin is the drug of choice for treatment of cases(Ar") as well as for contacts. Isolation period is 4 weeks
or untii paroxysms cease.

DIPHTHERIA

r Diphtheria is caused by Corynebacterium diphtheriae, a gram-positive bacterium.


e Source of infection is either case or carrier, with carriers beingthe most common sources of infection\I13), their ratio
is estimated to be 95 carriers for 5 clinical cases. Nasal carriers are more dangerous than throat carriers(Pcree).
Inmunuzation does not prevent carrier statealMs ee).
r Diphtheria affects both sexes and children ofage group l-5 year5 (rcrse).
o Infective period (period of communicability) vary from 14-28 days from the onset of disease(ArMsrr), but it is much
longer in carriers.
o Infection is acquired by droplets through respiratory tract (mainly), from cutaneous lesions and fomites.
o A case or carrier may be considered non-communicable when at least 2 cultures prolterly obtained
from nose and
throat, 24 hours apart, are negative for diphtheria bacilli,
o A herd immunity of over 707o is considered necessary to prevent epidemic spread, but some believe that the critical
level may be as high asgOo/o@EEr).
r Clinically diphtheria is divided into :-
1) Facial (tonsillo-pharyngeal) diphtheria
a It is the commonest type with incubation period of 2-5 days. There is thick adherent pseudomembrane (grey
or white) on one or both tonsils or adjacent pharynx. Dislodging of membrane is likely to cause bleeding. A
small perecentage of patient present with massive sweiling of tonsils and uvula with cervical lymphadenopathy
(bull neckars6)) and severe toxemia, called malignant diphtheria.
2) Laryngeal diphtheria
. r It is the most severe form of diphtheria.
3) Nasal diphtheria
r It is the mildest form.
4) Cutaneous diphtheria
rIt is caused by nontoxigenic strain and patient presents with "punched out" ulcer.
5) Otherforms
I These are conjunctival, otitic, genital (vulval, vaginal or prepucial), and gastrointestinal.
e Complications of diphtheria are obstruction due to pseudomembrane, myocarditisqllMs e7), polyneuropathy, post-
diphtheric paralysisur st) and pneumonia.
o Post-diphtheric paralysis occurs in 3'd or 4'h week and characterize dby palatine and pupillary paralysis with complete
recovery after sometime. There rs isolated cranial nerve (isolated ocular) paralysis\rrus sz' et) .
e Less common complications of diphtheria are renal failure, encephalitis, cerebral infarction, pulmonary embolism
and bacteremia or endocarditis.

Control of diphtheria
o Cotrol of diphtheria requiresr-
A) Control in cases and carrier
r It includes early diagnosis and treatment, along with isolation.
r Cases are treated by diphtheria antitoxin plus penicillin or erythromycinari3).
r Carriers are treated by only erlthromycin (no antitoxin(Al13)).
r Isolationperiodisforatleast14daysoruntilprovedfreeofinfection,i.e.2consecutivenoseandthroatswabs,
taken 24 hours apart, shouid be negative before terminating isoiation.
B) Control for contacts (prophylaxis for contacts)
r Non-immunized close contacts of diphtheria should be given penicillin/erythromycin(elruse2), diphtheria
antitoxin, qnd vaccination (by toxoid).
: In immunized contact nothing is required if the booster dose was taken within previous 2years,and ifbooster
dose was taken more than 2 years before, only a booster doses oftoxoid is required.

TUBERCULOS!S IN CHILDREN

c Tuberculosis is a chronic infectious disease caused by mycobacterium tuberculosis.


. Cases of tuberculosis represent 5- I 5 precent of all tubercular casesar ,5).

Clinical manifestations of TB in children


o The majority of children with tuberculosis infection develop no signs or symptoms at any level@t 1s).
o Occasionally, infection is marked by low grade fever, mild cough, mailaise and flulike symptoms.
A) Primary pulmonary disease
t The lung is the portal of entery in > 98% of cases. Primary complex of tvberculosis includes local infection at the
portal of entry and regional lymphnodes(ArtMs e8) that drain the area.
I The combination of the a parenchymal pulmonary lesion and corresponding lymph node is called Ghon complex.
; Asymptomatic pleural effusionqrrus eB) is so frequent in primary tuberculosis that it is basically a component of
primary complex.
r Primary complex may have one of the following course : -
i) Healingbyfibrosis'nnusto and calcification.
i0 Progressive primary tuberculosis.
iii) Cavity
iv) Obstructive emphycema -+ If incomplete obstruction is caused by lymph nodes.
v) Resorption atelectesis -+ If complete obstruction is caused by lymph nodes.
vi) Tubercular bronchits 6 brionchiactasis
B) Control for contacts (prophylaxis for contacts)
r Non-immunized close contacts of diphtheria should be given penicillin/erythromycin(AltMs e2),
diphtheria
antitoxin, and vaccination (by toxoid).
r In immunized contact nothing is required if the booster dose was taken within previous 2years,and if booster
dose was taken more than 2 years before, only a booster doses oftoxoid is required.

TUBERCULOSIS IN CHILDREN

o Tirberculosis is a chronic infectious disease caused by mycobacterium tuberculosis.


r Casesoftutrerculosisrepresent5-l5precentofalltubercularcases(Aris).

Clinical manifestations of TB in children


o The maiority of children with tuberculosis infection develop no signs or symptoffis at any levelLlls).
o Occasionally, infection is marked by low grade fever, mild cough, mailaise and flulike symptoms.
A) Primary pulmonary disease
t The lung is the portal of entery in > 98o/o of cases. Primary compler of tuberculosis includes local infection at the
portal of entr y and regional lymphnodes(AltMs e8) that drain the area.
r The combination of the a parenchymal pulmonary iesion and corresponding lymph node is called Ghon complex.
t Asymptomatic pleural affusionqnus e8) is so frequent in primary tuberculosis that it is basically a component of
primary complex.
r Primary complex may have one of the following course : -
i) I{ealing by jibro5isro"*t '8) and calcification.
ii) Frogressive primary tuberculosis.
111) Cavity
iv) Obstructive emphycema -+ If incomplete obstruction is cLtused by lymph nodes.
v) Resorption atelectesis -+ If complete obstruction is caused by lymph nodes.
vi) Tubercular bronchits 6 brionchiactasis
vii) Hematogenous dissemination --+ During early bacteremia seeding firay occ;.rr at apex
-> Simoru's focus.
vtii) Mili ar y tub erculo si s
ix) Collspse consolidationquMses) (segmental lesion)-+ A, combination of pneumonitis and atelectosis.
B) Chronic pulmonary tuberculosis
r Infrequent in young children. Children above the age of seven years, especially girls are more l.ulnerable. Com-
monest site of chronic pulmonary tuberculosis is apex of lung -+ Puhl's lesion.
r Infraclavicular lesion of chronic pulmonary tuberculosis is cailed Assrz an's
focus.
t Regional lymph nodes are not involved in chronic pulmonary TB (in cottrast to primary TB).
o Most common site of extra-puimonary tuberculosis in children is lymph nodes(Ar,sr.

Diagnosis
o Following investigations are used for TB in children :-
i) Sputum smear AFB
I It is used in older children as young children do not expectorate out sputum. It is used only for presumptive
diagnosfuuttusotl '
r Most of pediatric pulmonary TB is sputum smear negative(DN8 r5r.

ii) Laryngeal swab smear AFB


r Similar to sputum AFB it is used for presumptive diagnosis,but in younger children (who do not expectorate
out sputum).
iii) AFB culture
r Definitive diagnostic of TB depends upon the culture of bacteria obtained from respiratory specimen (sputum
or laryngeal swab).
B) Control for contacts (prophylaxis for contacts)
r Non-immunized close contacts of diphtheria should be given p enicillin / ery thromycin^IlMs er), diphtheri a
antitoxin, and vaccination (by toxoid).
r In immunized contact nothing is required if the booster dose was taken within previous 2 years, and if booster
dose was taken more than2 years before, only a booster doses oftoxoid is required.

TT,BERCULOSIS I N CHILDREN

o Tuberculosis is a chronic infectious disease caused by mycobacterium tuberculosis.


c Cases of tuberculosis represent 5- 15 precent of all tubercular cases(Ar rs).

Clinical manifestations of TB in children


c The majority of children with tuberculosis infection develop na signs or symptoms at any levelhtts).
o Occasionally, infection is marked by low grade fever, mild cough, mailaise and flulike symptoms.
A) Primary pulmonary disease
t The lung is the portal of entery in > 98o/o of cases. Primary complex of tuberculosis includes local infection at the
portal ofentry andregionallymphnodesqnMses) that drain the area.
r The combination of the a parenchymal pulmonary lesion and corresponding lymph node is called Ghon complex.
t Asymptomatic pleural effusisnutrus s8) is so frequent in primary tuberculosis that it is basically a component of
primary complex.
r Prirnary complex may have one of the following course : -
i) I{ealingbyfibrasisro""'8) and calciJicaiion.
ii) Progressive primary htberculosis.
iii) Cavity
iv) Obstructive emphycema -+ If incomplete obstruction is caused by lymph nodes.
v) Resorption atelectesis -+ If complete obstruction is ca,tsed by lymph nodes.
vi) Tubercular bronchits 6 brionchiactasis
vri) Hematogenous dissemination -+ During early bacteremia seeding may occur at apex -+ Simon's focus.
viil) l,4ili ar y t ub e r cul o si s
rx) Collqpse consolidationattMses) (segmental lesion)-+ A combination of pneumonitis and atelectosis.
B) Chronic pulmonary tuberculosis
r Infrequent in young children. Children above the age of seven years, especially girls are more lrrlnerable. Com-
rnonest site of chronic pulmonary tuberculosis is apex of lung -> puhl's lesion.
r Infraclavicular lesion of chronic pulmonary tuberculosis is call,ed Assman'sfocus.
t Regional lymph nodes are not involved in chronic pulmonary TB (in contrast to primary TB).
r Most common site of extra-putrmonary tuberculosis in children is lymph nodes(/rrs).

Diagnosis
o Following investigations are used for TB in children :,
i) sputum smear AFB
r It is used in older children as young children do not expectorate out sputum. It is used only
for presumptiye
ot)
diagnosisAums .

I Most of pediatric pulmonary TB is sputum smear negative(Dnr rs).

ii) Laryngeal swab smear AFB


I Similar to sputum AFB it is used for presumptive diagnosis,but in younger children (who do not expectorate
out sputum).
iii) AFB culture
r Definitive diagnostic of TB depends upon the culture of bacteria obtained from respiratory specimen (sputum
or laryngeal swab).
w) Histopathology
r Histopatholoic specimen can be obtainedby biopsy or FNAC(DNB r:) sf fl1e lesion or lymphnocies.
v) Chest X-raY(oNs tst
r Shows specific findings.

vi) Tuberculin test (Mantoux test@NB "))


r It is of diagnostic value in infant and young children. Induration > l0 mm is considered positive.
vii) TB PCR
c Serology (ELISA or other methods) has no utility in pediah'ic TB(DNB
13).

Antitubercular drugs used in rhildren


r Firstlinedrugs:Isoniazid{PGl06,0s),RifampicinrPGr06'0s),Ethambvtol(PGI06'05),Streptomycin(PGI06'0s),Pyrazinamide(eGI
06,0s)
.

r Second line drugs : Cycloserine, Ethionamide, PAS, Capreomycin, Kanamycin


r Other drugs : Quinolones -> Ofloxacin(Pcr06), Rifamycin, Amikacin. Imipenem, Ampicillin

Drug Regimen
r Category 1 -> 2 (H3 R3 Z3 E3) + 4 (H3 R3)
r Category 2 -+ 2 (S3 H3 R3 Z3 E3) + 1 (H3 R3 23 E3) + 5 (H3 R3 E3)
r Category 3 -+ 2 (H3 R3 23) + 4 (H3 R3)

lsoniazid 600 1A-l yDNB 1'

-:..:,;}$',,ij.;.,=r.'...'r,..;.;;:'..,f 1,..' .,
55
':t:.,1;,
3,Q.,'r.:r.:r;

15

Prevention of perinatal TB
r If mother has active disease, ATT should be started for mother.
o Neonate should be given INH prophylatcisqlqo) till mother is sputum negative or at least 6 months(DrvB '5).
r Routine BCG vaccinationatqq) should be given to mother.
o Separation of mother and neonate is not required ard breast feeding should be cantinuedqlo).

Congenital tuberculosis
r Although it is rare as mother having tuberculosis primarily present with infertility.
o f'uberculous bacilli sometimes pass throgh umblical vein and may develop focus in liver (hepatic complex).
r When neonate aspirate amniotic fluid containing bacilli then develop GI tuberculosis or iung infection.
r Neonate usually present as respiratory distress, hepatosplenomegaly lymphadenopathy.
r Overall liver is most commonly involved in congenital tuberculosisarr5).

N EOilATAL T=TAN U5 {8TTI DAY DISEASf }


r Tetanus is caused by clostridium tetaniGllMs e7)
an anaerobic spore formin g gram positive bacillus.
r WHO defines neonatal tetanus as "an illness occurring in a neonate that has the normal ability to suck and cry in first
2 days of life but who loses this ability between 3-28 days of life and becomes rigid with spasm.
r '?athogenesis is due to tetanospasrnin, a tetanus toxin which is released in the wound and enters the axon of neurons
(rcr0I).
from where it is transported to brainstem and spinal c rsrdby retrograde intraneural transmissian Jslalospasmin
blocks the release of inhibitory neurotransmiter GABA. Cl. tetani itself has very little invasive proprety and is canfined
to prirnar'v site of lodgmenl(pcr01).
r Most common cause of neonatal tetanus is unsterile treatment of umbilical cord stumputms ss).
Clinical features
r Symptoms of neonatal tetanus starts by 5-15 days afierbirthbutnever infirst 2 days of lifeAilMseT). Though incubation
ot ).
Period is variable(Pcl
o Trismus is the earliest and most common symptom. Other symptoms are headache, restlessness, irritabiliry Risus
Sardonicus (spasm offacial and buccal muscles), dysplagia, opisthotonus (arched posture), seizures and respiratory
muscle spasm. Neonate commonly has excessiye ufiexplained cry refusal to feeflGtusez) and apathy.
t Mortality varies from 50-75o/o6rIMSe7).

Treatment
r If tetanus immunoglobulin is administered within time it neutralizes tetanus toxin and significantly lowers
mortality(AttMs . If a person is having serous wound and is uncertain about immunization status, combined active
ee)

(tetanus toxoid) and passive (immunoglobulin) should be given(1'uttsee).


r Metronidazole is the drug of choice (previously it was peniciilin-G), to eradicate CI. tetani. Treatment of anxiety,
com,ulsions and autonomic instability should be given (diazepam, alpha-blocker and beta-blocker).

OTHER PEDIATN!E I NFECTIONS

CONGEN ITAL CMV IN FECTION

o CMV is the most common cause of congenital infection(uPsc 10).Infections occur in fetuses born to mother who develop
primary infection during pregnancy.
c It leads to cytomegalic inclusion disease. Most infections are asymptomatic. Petechie,hepatosplenomegaly@Gte4) and
jaundice is the classicaltuiad of presentation. Other important features are microcephaly, intracerebral calcification(PGl
e'1),
IUGR, chorioretinitis, thrombocltopenia and inguinal hernia.

Diagnosis
o Virus isolation of CMV is the definitive diagnostic method. Specimens may be urine (urine culture@IlMso6'), saliva,
blood (buffy coat), bronchoalveolar washing, or amniotic fluid.
o Serology is not very helpful. IgM test lacks sensitivity and specificity. IgG tests are not diagnosticailMs 06)
as IgG
antibodies can cross the placenta (may be of maternal infection).
c PCRfor viral genome (DNA) may be usedaltMs06).
r In histopathologic examination, CMV infected cells contain large intranuclearaltMs06) and small intracl,toplasmic
inclusion bodies, which are pathognomic.

NEONATAL HSV INFECTION

o It is commonly caused by HSV-2rrctoo) (70o/o cases are caused by HSV-2 and3Oo/o caused by HSV-1 ).
o Case fatality rate exceeds 600/o in untreated neonatai herpes. '
o The infection is acquired by passage through an infected genital tract at birth. The greatest risk i.e., 50% transmission
rate occurs with a primary infection in the mother at time of delivery (because the antibodies have not yet developed,
there is more virus present and no antibody transfer). With recurrent herpes at term, the transmission rate is only
8% or less.
o Neonatal infections present on the 6th day postpartum. There are three major categories:-
r Localized skin, eye and mouth infection
r CNS infection
r
Dissenminated infection
o The hallmark of neonatal HSV infection - the vesicular, ulcerative skin lesions occur in only 30-43o/o children. If left
untreated the virus dissenmination can occur to internal organs which is the most serious complication of neonatal
herpes.
Cnlri'r&,
CONGENITAL SYPI{ILIS

o Transmission of T. pallidum across the placenta may occur at a.ny stage of pregnancy, but the lesions of
congenital syphilis generally have their onset after the fourth month of gestation.
c Earlymaternalslphilishasmorechancesoftransmission(75-90o/o) thanlatematernalsyphilis.
o Adequate treatment of mother before 16'h week of pregnancy prevents fetal damage.
o Effects of maternal slphilis on pregnancy are :-
i) Still birth > abortion
ii) Prematurity
iii) Neonatal death
iv) Non fatal congenital syphilis.

a With in 2 years of birth a After 2 yrs of age Hutchinson's teeth )


centrally
a Earllest sign -+ rhinitis or Snuffles a 600/o are subclinical notched; widely spaced, peg
a Most common menifestations are , Interstitial keratitis shaped upper central incisors
bone changes like osteochondritis, a Eight nerve deafness a Mullberry molars
periostitis, osteitis a Recurrent arthropathy a Frontal bossing
Mucocutaneous lesions - bullae, a Clutton's joints -+ B/L knee a Saddle nose
vesicle, petechie, mucous patch, effusion a Poorly developed maxillae
condylomata lata, papulosequamous a Asymptomatic neu rosyphilis a 'Saber shins' -+ Anteriortibial
lesions, superfi cial desquamation ta Gummatous periostitis bowing
I Hepatosplenomegaly 'Rhagades'-+ linear scars at
a Lymphadenopathy the angles of the mouth and
a Anemia, throm bocytopenia, nose
leukocytosis
Ja undice

common pattern of presentation of Congenital s1philia.


a Hutchinson's triad@anipot06) is a
a It consists of three phenomena > l. Interstitial keratitis, 2. Hutchinson incisor, 3. Eight nerye deafness

Diagnosis
o It involves Dark ground Microscopy and VDKLGTTMS 11' AI07),
o As IgM does not cross placenta, its presence in neonatal serum confirms congenital syphilis and helps differentiating
it from seropositivity due to passively iranfered maternal antibodies. The tests to detect IgM are -
1) FTA-ABS (195IgM FTA-ABS1{Lott.
ii)S1philis Capita M test.

EONG EN ITAL TOXOPIASMOSIS

r Congenital infections develop only when the nonimmunized mother develops infection during pregnancy or less than
6 months before pregnancy.
r The incidence of transplacental infecti on is lowest in first trimester but the disease in neonates is most seyere. The
incidence of transplacental infection is highestin third trimester (after 6 monflrs) but the infant is not severely affected.
r Most of the infections are asymptomatic. The symptomatic baby presents with characteristic triad of chorioretinitis (most
ea),
commonqte3)),hydrocephalus and intracerebral calcificatian. Other feattres are microcephalyqilMs microphthalmia,
mental retardation, deafness, blindness, epilepsy, thrombocltopenia, anemia, nephrotic syndrome, and optic atrophy.
Diagnosis
r Diagnosis is mainly dependent on serological tests. These are :-
1) IgG
t Presence of lgG in infants's blood does not confirm the diagnosis of congenital toxoplasmosis because IgG can
easily cross the placenta. So, one cannot predict whether the IgG is of maternal origin or due to fetal infection.
Transplacental IgG antibody disappears by 6-L2 months of age.
r Serological tests to detect IgG : -
i) Sabin-Fieldman dye test (Most preferred lgG test and golil standardqttMs 10)).

ii) IgG IFA test (IgG indirect Jluorscent antibody test)


ril) Differential agglutination (AC/HS) IgG test.
iv) Avidity test : - It measure the strength of antigen-antibody reaction for IgG antibodyatrus to).

2) IgM
t IgM does not cross the placenta; therefore, IgM is the test of choice for determining congenital infection(DNB 13).
r Serological tests for IgM are :-
i) IgM IFA (indirect fluorscent antibody test)
ii) Double sandwich enzyme linked immunosorbant assay : - More sensitive than IgM IFA.
rll) Immuno sorb ant agglutination as s ay (I SAGA)

3) IgA
t IgA may have greater sensitivity for neonate coffipared to IgM assay(DNB 13).

x Serological tests for IgA are : -


r) Double sandwich enzyme linked immunosorbant assay (lgA ELISA) : - It is better than double sandwich
lgM ELISA for diagnosis of congenital toxoplasmosisqllMs 10).

ii) The immunosorbant agglutination assay (ISAGA) IgA.

Prevention
o Spiramycin1ilMse4'%) is given during pregnancy to prevent vertical transmission of toxoplasma.

cYsrrcERcoSrs
r Cysticerci can be found any where in the body but are most commonly detected in - brain, skeletal muscles,
subcutaneous tissue or eye. Neurocysticerosis is the most common parastitic infection of the CNS.
r Seizures are the presenting finding in> 70o/o of cases(Pcr0'). Seizures are generalized in 80% cases but simple or
complex partial seizures@Gr") may occurs. Intraventicular neurocysticercosis is associated with hydrocephalous and
acute, subacute or intermittent signs of increased intracranial pressure. Meningeal neurocysticercosis is associated
with signs of meningeal irritation. A fulminant encephalitis@Gr03) Iike presentation also occurs most frequently in
children who have had a massive initial infection.
r Ocular neurocyticercosis presents with decreases visual acuity due to cystecerci floating in the vitreous, retinal de-
tachment or irrdocyclitis.

CONGENITAL INFECTIONS
r Congenital infections are those which are transmitted from mother to fetus or newborn child.
o Congenital infection may be :

i) Transplacental,
ii) Intrapartum (peripartum)
iii) Post partum (postnatal).
r Transplacental infections, also called vertically transmitted infections, are infections that are transmitted from
pregnant mother to fetus, through placental circulation. Important transplacental infections are : Rubellaal 11' rN e0),
CMVINe0), HPV B-19, HIV@ 11), HSVaI 11),VZV, HBVaI ir), HCV toxoplasma(N eo), treponema pallidum (syphilis),
listeria, plasmodium and TB.
r
ii
c,rprEnrr;;;#&
o Intrapartum infections are transmitted at the time of delivery. Important intrapartum infections are CMV HSV HBV
HIV E.coli, group-B Streptococci, gonococci, chlamydia and listeria.
e Post-natal infections are transmitted after birth. Important ones are CMV HBV VZV HTLV-1, HlV, HSV and
enterovirus.

* CMV is the most common congenitol infection(uPsc t0)

o Symptomatic neonatal CNS infections are more common in : HSV (76-1 00o/o), rubella (26-60Vo).
t Asymplom'a$c,neonqtal(lVS infecfion are'mare eommon in: CMV, toxoplasma, syphitis.
a lmportant causes of posterior auricular or suboccipital cervical lymphadenopathy in children: Roseola infantum, rubella, scalp infec-
tion.
Mijs{:common causeof:posterior airicular ar suboccipital lymphadenopathy in children without obvious disease or secondary infection
l
: Pediculosis capitis (lice).

& A child presenting with sore throat, fever and acute cervical lymphadenopathy, first investigation : Hemorgram.
& Mosi comman manifestation af HPV infection in children: Multiple'papillomatosis.
{t Number of pustules to diagnose a severe bacterial infection in children: > 1 0 pustules.
d Ni.mber of pustules in local Qacterial infection: Less thanl Opustules.
* Drugs used to treat resistant falciparum malaria in children -
i) < B years age : Quinine + Clindamycin or atovaquone + proguanil.

I &
li) > 8 years age : Quinine + doxycycline/clindamycin or atovaquone + proguanil,
A:line of conjunctiial inflsrnmation on lower eye lid margin: Steinei! line in measles.
& Maximum chances of neonatal chicken pox: lf maternal infection occurs near delivery (5 days before to 48 hours after delivery).
& Most severely affected age group by rubella: Fetus (Unborn child).

xxx
QUESTIONS

l. Trans-placental spread is least associatedwith? 10. Child (girl) is suffering from varicella (fever rash).

a) HBV b) Rubella (Ar 11) And child's aunt is pregnant. IMhen is it earliest that
C) HSV d) HIV the child can meet her auat - (PGI Nov 14)
a) When the lesions have crusted
b) Immediately
CONGENITAL RUBELTA c) Anytime as the child is aunt's favourite
d) After the delivery of the baby
2. .All of the following statements are true about con-
genital rubella except- (AttMS May 11,Nov 08, At 05) ll. l{hich of the following intrauterine infections is
a) It is diagnosed when the infant has IgM antibodies associated nith limb reduction defects and scar-
ring of skin - (AIIMS May 11, AI 09)
at birth
b) It is diagnosed when Ig G antibodies persist for a) Varicella virus b) Herpes virus
more than 6 months c) Rubella d) Parvovirus
c) M.C. congenital defects are deafness, cardiac
malformation and cataract MEASLES
d) Infection after 16 weeks ofgestation results in
major congenital defects 12. Measles virus is - (NEET Dec.12 Pattern)

3. Which of the following is aot a common manifesta- a) Paramyxovirus


tion of congenital Rubella - 6t 02) b) Orthomlxovirus
a) Deafness b) PDA c) Poxvirus
c) Aortic stenosis d) Mental retardation d) Picornavirus
Which of the following methods can be used to 13. Which of the following is the "Least Common"
detect rubella infection in children - @t es) Complication of measles ? @tes)
a) T4 cell count a) Diarrhoea
b) Fetal hemoglobin b) Pneumonia
c) IgM antibody in fetal blood c) Otitis media
d) IgA Antibody in fetal blood d) SSPE
Allare true *bout congenital rubell,a syndrorne, 14. Measles is iafectious during - (PGI Dec 98)

€xc€pt- a) After 4 days ofrash


a) Cardiac abnormality (All India Dec.14 Pattern) b) 4 days before and 5 days after rash
b) Renal anomalies c) Throughout disease
c) Deafness d) Only in incubation period
d) Cataract. 15. SSPX (subacutesderosingpanencephalitis) isassnci-
(PGl lune 01)
6. Rubella causes all excePt - (CET luly 15 Pattern) atedwith -
a) Microcephaly a) Mumps b) Chickenpox
b) VSD c) Herpes d) Measles
c) Conduction defect 15. Giaat cell (Hecht's) pneumonia is due to -
d) All a) CMV (All India Dec.14 Pattern)

V. Which heart diseases is raost cornraoaiy associated b) Measles


ryith rubella iafectioa - (CET Nov. 14 Pattern) c) Malaria
a) PDA b) VSD d) P. carinii
c) ASD d) Eisemenger's
syndrome
MUMPS

cHlcrcN Pox 17. Commonest complication of mumps is - (Ar oo)

a) Orchitis and oophritis b) Encephalitis


8. MC complicatioa of chicken pox in children - c) Pneumonia d) Myocarditis
a) Encephalitis (PGt June 2000)
lg. Which of the following is true of mumps -
b) Sec. bacterial infection (AILMS May 0s)
c) Pneumonia a) Salivary gland involvement is limited to the
d) Otitis media parotids
9. following arecornplications of chicken poxexcept - b) The patient is not infectious prior to clinical
(NEET Dec.12 Pattern) parotid enlargement
a) Meningitis b) Pneumonia c) Menigoencephalitis can precede parotitis
c) Enteritis d) Reyes Syndrome d) Mumps orchitis frequently leads to infertility
QUESTIONS

1. Trans-placental spread is least associatedn'ith? 10. Child (girl) is suffering from varicella (fever rash).

a) HBV b) Rubella (Ar 11) And child's aunt is pregnant. When is it earliest that
c) HSV d) HIV the child can meet her auat - (PGl Nov 14)
a) When the iesions have crusted
b) Immediately
CONGENITAL RUBETLA c) Anltime as the child is aunt's favourite
,' d) After the delivery of the baby
of the following statements are true about con-
.A.ll
genital rubella except- (AIIMS May 11, Nov 08, AI 05) ll. Which of the following intrauterine infections is
a) It is diagnosed when the infant has IgM antibodies associated with limb reduction defects and scar-
ring of skin - (AltMS May 11, At 09)
at birth
b) It is diagnosed when Ig G antibodies persist for a) Varicella virus b) Herpes virus
more than 6 months c) Rubella d) Parvovirus
c) M.C. congenital defects are deafness, cardiac
malformation and cataract MEAStES
d) Infection after 16 weeks ofgestation results in
major congenital defects 12. Nleasles virus is - (NEET Dec.12 Pattern)

3. Which of the following is not a cornmon manifesta- a) Paramlxovirus


fionofcongenitalRubella ' @to2) b) Orthomyxovirus
a) Deafness b) PDA c) Poxvirus
c) Aortic stenosis d) Mental retardation d) Picornavirus
4. Which of the following methods can be used to 13. \ryhich of the following is the'Least Commor"
detect rubella infection in children - @I es) Complication of measles ? @I e8)
a) T4 cell count a) Diarrhoea
b) Fetalhemoglobin b) Pneumonia
c) IgM antibody in fetal blood c) Otitis media
d) IgA Antibody in fetal blood d) SSPE

5. All are true about congenital rubella syndrorne, 14. Measles is infectious during - (PGI Dec 98)

€xc€pt- a) After 4 days ofrash


a) Cardiac abnormality (All India Dec.14 Pattern) b) 4 days before and 5 days after rash
b) Renal anomalies c) Throughout disease
c) Deafness d) Only in incubation period
d) Cataract. 15. SSPE (subacute sclerosing panencephalitis) is associ-
5. RubellacausesallexcePt- (CET July 15 Pattern) atedwith - (PGI lune 01)

a) Microcephaly a) Mumps b) Chickenpox


b) VSD c) Herpes d) Measles
c) Conduction defect t6. Giant cell {I{echt's) pneuraoaia is due to -
d) AII a) CMV (All India Dec.14 Pattern)

?. lllhich h€art diseases is most comrnonly associated b) Measles


with rubella infectioa - (CET Nov. 14 Pattern) c) Malaria
a) PDA . b) vsD d) P. carinii
c) ASD d) Eisemenger's
syndrome
MUMPS

CHICKTN POX t7. Commonest complication of mumps is - (A1 oo)

a) Orchitis and oophritis b) Encephalitis


8. MC comphcation of chicken pox in children - c) Pneumonia d) Myocarditis
a) Encephalitis (PGI June 2000)
18. Itrhich of the following is true of mumps -
b) Sec. bacterial infection (AIIMS May 0s)
c) Pneumonia a) Salivary gland involvement is limited to the
d) Otitis media parotids
9. Following are complicatioas of chickea pox except - b) The patient is not infectious prior to clinical
(NEET Dec.12 Pattern) parotid enlargement
a) Meningitis b) Pneumonia c) Menigoencephalitis can precede parotitis
c) Enteritis d) Reyes Syndrome d) Mumps orchitis frequently leads to infertility
FEVERWITH RASH a) Elfusion seen
b) Consolidationpresent
t9- Erythema infectiosunr is seen in - (CET lune 14 pattern) c) Fibrosis
tl.
a) Rubella b) Fifth disease d) Mediastinal lymphadenoparhy
c) Scarlet fever d) Diphtheria 29. Z0year* old Rajesft prescncewith fever amdehrtnie
20, Fever stops and rash begins i* diagnastie af - eough E$R 35 mm$Futurn eytology ir negativefar
(lipmer 9s) AFB and tubere*lin te*t show indwr*tians of lg X 23
a) Fifth disease mm. The probable diagnosis is - (AllMS Nov 0t)
b) Roseola infantum a) Fungal pneumonia
c) Measles b) Viral pneumonia
d) Toxic shock syndrome c) Pulmonary T.B.
2t. An9yr. old female child following UR?I developcd d) Bacterial pneumonia
maculopapular rash oiltbe jaw spreadingonta the 30. *,f,ost commein site of extra - pahuonary TB in
trunk whieh elearcd o* the 3d day without desquc- children is - (All tndia Ddct5 pattern)
matian alrrd ten&er po$t auricular and suboeeipital a) Abdominal b) Genitourinary
lymphadenapathy.We diagnosis is - (ArMS May 0t) c) Lymphnode d) Congenital
a) Kawasaki disease b) Erlthema infectiosum 31. Most common organ involvetl by congenital TB is-
c) Rubella d) Measles (All lndia Decl5 Pattern)
)7 A patient lr*& fever and cartqa fCIr l**t 3 days devel- a) Liver b) Pancreas
opedrwar;:.itlopapwlarerythematourrashwithwhieh c) Kidney d) Lung
Ia*ted for 48 hrs and disappearedwithautleavrng 32. Which of the following has no role in diagrrersis of
bebind pigmentatioln is most eomrnonly due tc , childhood TB ? (CET Aug.13 Pattern)
(AIIMS June 2000) a) ELISA b) Mantoux
a) Measles b) Typhoid c) CXR d) FNAC
c) Roseola infantum d) Fifth disease 33. Isoniazid child dose in DOTS regirnene (RNTCP)-
23. Which of the following is true about roseolainfan- (CET Nov 15 Pattern)
turn- @GI May t4) a) 10-15 mg/kg/dg b) 1s-20 mgikgldg
a) Defervescene follows the rash c) 20-25mglkg/dg d) s'I0 mg/kg/dg
b) Caused by HHV 6 and 7
c) Slapped check appearance is seen 34. M*nagcmewt, tf anewbormwher"Wlslhcrhas aslive
Tubcreulei*is & is takixg NEY - (Ar 2000)
d) Otitis media is common complication
a) BCG + Rifampicin + iNH + Breast Feeding
e) R.ash appear first on face and neck
b) BCG + Isolation of baby
u" Tnre about Roseola infantum - (PGI June 03) c) BCG + INH for 6 week + Breast Feeding
a) Also called 5th disease d) BCG + INH + withhold Breasr Feeding
b) Caused by HHV 5 & 7
c) Rash appear in trunk 35. Asymptomatic infant with, Hlo TB exposure, He is
3 nronth old and had takcn 3 rnonths ofcherftopro-
d) During defeverescence, rash appears
phfaxis, what is trr be done next -
,4 l"fhich cf the foltrowing is true about erFthema infec- a) Test sputum and then decide @ET luly t5 pattern)
tiosum - (PGI Dcc 04) b) Continue for 3 months
a) Slapped check appearance seen c) Tuberculin test then decide
b) Caused by parvovirus d) Immunise w-ith BCG & stop prophylasix
c) Defervescene before rash
d) Rash appears on head & neck pEaTU$$ts
26. Child with rash- wrong ir - (All India Decl5 pauern)
a) Tlphus - day 5 b) Varicella - day 1 36. Incubation period rf pertussi,s is -
c) Tlphoid - day 5 d) Measles - day 4 (All India Dec.14 Pattern)
a) 7 days b) 7-t4days
TUEERCULOSI$
c) 14-28 days d) 28 days
37. Fert&sisaffectswhiehage- (PGI ltne 2000)
27. Tuberculosis in children true is- (CET Nov 15 pattern) a) 2-3 b) <5
a) Commonly sputum negative c) 5-7 d) > 10 years
b) Incidence < 5o/o of all TB cases 3&. .&llarc fe*twres *f Verewwis exrqt- (AttMS Feb 97)
c) Clinically child does not show sign of florid TB a) Encephalopathy
d) All of above b) Cerebellar Ataxia
2&" ?:egardingp&IMARY COMPLEX in ehildren, all are c) Silbconjunctival hernorrhage
trwe except- (Arrlils eB) d) Bronchiectasis
39. Antibodies to one of the following infection is not b) Persistent cough
transmitted to child - (cET Nov. 14 Pattern) c) Failure to thrive
a) Measles b) Pertusis d) Lymphadenopathy
c) Diphtheria d) Polio 49. I{hich of the following is NOT a feature of HIY
41$, A chitdryith complaints of cough. Characteristic infectioo in childhood - (AIIMS Dec 94)
inspiratorywhoop. sample for investigation is - a) Failure to thrive
a) Nasopharyngeal swab (AIIMS Nov 09) b) Hepatomegaly
b) Tracheal aspiration c) Lymphoidinterstitialpneumonitis
c) Cough plate culture d) Kaposi sarcoma
d) Sputum culture 50" HIV in children, characteristic finding is -
4X. A child presents with recurrent bouts of severe a) Kaposi sarcoma is common (AllMS Dec 9s)
cough folloured try an audible whoop. Which of the b) Recurrent candidiasis
following is considered the best type of specimen to c) Recurrent chest infection
isolate the organism and confirm the diagnosis- d) Crlptococcal diarrhoea is common
a) Nasopharyngeal swab (AIIMS Nov 11, AI 11) 51. All of the following methods are used for the fiagno-
b) Cough plate sis of HIV infection in a 2 month old child, except -
c) Throat Swabs a) DNA-PCR (CET Nov. )2 Pattern)
d) Anterior Nasal Swab b) Viral culture
42" Theantibioticofchoiceofpertussis is c) HIV ELISA
(AtrMS 79, AP 90) $ p2a antigen assay
a) Ampicillin b) Gentamicin 52. All of the following strategies
are effective in
c) Erythromycin d) Penicillin preventing mother to child transmission of HIY
except - (AIIMS Nov 0j)
srpr{THrRtA a) Zidovudine to mother & baby
b) Vaginal cleansing before delivery
43. 'Bull neck in Diphtheria is due to - (CET lune 14 Pattern) c) Stopping breast feeding
a) Retropharyngealabscess d) Elective caesarean section
b) Laryngeal edema 53. Infant of HIY +ve mother, which should tle done -
c) Cellulitis a) BCG vaccine should not be given (PGI June 98)
d) Lymphadenopathy b) AZT therapy
M. All are cornplications of diphtheria except - c) Separation from mother
a) Myocarditis (AIIMS lune 97) d) Other than BCG, all other vaccines are given
b) Ocular muscel palsy 54. Which drug is given to prevent HIV transmission
c) Cerebellar ataxia from mother to child (AIIMS Nov 06)
d) Hepatic failure a) Nevirapine b) Lamirudine
c) Stamdine d) Abacavir
HIV'INFECTION 55. True about ItrIV in the neonate includes all the fol-
lowing except - (ArrMS 9e)
45. According to WHO the major sign of AIDS in chil-
a) Cannot be diagnosed accurately by current
dren in stage I is the following - (All India Dec15 Pattern)
methods
a) Generalisedlymphadenopathy b) Failure to thrive may be presentation
b) Not gaining weight c) Transmission rate during pregnancy exceeds 90%
c) Oral candidiasis d) Transmission vertically from mother
d) All of above
56. Vaccines contraindicated in HIV positive child -
46. Most comrnon cause of HIV infection in infant is -
a) OPV b) MMR (PGI Dec 08)
a) Perinataltransmission (PCl June 97)
c) Rabies - d) Influenza
b) Breast milk e) Hepatitis
c) Transplacental
d) Umbilical cord sepsis 57. Indications to start ART in children under NACO
all except - (Delhi PG Mar.09)
47. True statement(g regarding feeding of HlV-infected
a) <11 months infant-if CD4 Count <1500 cells/mm3
child isiare - (PGI May 14)
b) 12-35 months-if CD4 Count <500 cells/mm3
a) Breast feeding for 4-6 months then start weaning c) 36-59 months-if CD4 Count <350 cells/mm3
b) Breast feeding for 1 year then start weaning d) >5 years-if CD4 Count <200 cells/mm3
c) Exclusively top feeding
d) Breast feeding for 6 months & then rapid weaning 58. Prophylaxis with Cotrimoxazole is recommended in
the following situation except - (Delhi PG Mar.09)
48. MC symptom of AIDS in infant is -
a) A11 symptomatic HIV infected children>5 years of
a) GI infection (All India Dec15 Pattern)
C IIAPT,TE.n,::tit

age irrespective of CD4 for hepatitis B. He is otherwise asy*nptormatie" Child


b) All HIV exposed infants till HIV infection can be was given three dosec of reeomtrinanf kepatitis S
ruled out vaerine at the age af X ycar.Ilis mcther was Weateis
c) All HIV infected infants less than 1 year age for chronic hepatitis B infecticn areiune{ thc same
irrespective of symptoms or CD4 counts time. The next relevant step fcr further investig*ting
d) As secondary prophylaxis after initial treatment for the child wcwld be to - 6I 03)
pneumocystis cari ni Pneumonia a) Obtain HBeAg and anti-HBe levels
b) Obtain anti HBs levels
c) Repeat HBsAg
I"IEPATNT!5 d) Repeat another course of Hepatitis B vaccine

59. Incubation period of Hepatitis A is -


(All India Dec.1I Pattern) TETAruUS
a) 2 wks - 2 monthsb) 6wks - 6 months
c) 6days-6wksd) 2davs-2wks {}6- Al[ are feaftires of Neonatal Tetan&s ex€€frt -
a) Refusal to feed is common initial symptom
60. Which one of the following hepatiti$ Yim$e$ haYe
b) Caused by clostridium tetani (AIIMS Feb 97)
signifi cant p cfinetel tr eE\.smission - 6r 03)

a) Hepatitis E virus c) Usually occurs in lst 2 days of life

b) Hepatitis C virus d) Mortality is 50-75o/o.

Trtre abowt tctanwe - (PGl Nov 14)


c) Hepatitis B virus &7"
d) Hepatitis A virus a) Tetanus bacilli can spread through blood

61. ln * 3 year ald child, mo$t eommon ccuse efhepati- b) Spread along the nerves onlY
tisBis- (PGl June 97) c) Requires oxygen
a) Pin prick d) I.P. is variable
b) Saliva exchange e) Causes deep wound invasion
c) Perinatal
d) Blood transfusion
P6t-to
62. AaS &ay - old infant developed ictertrs and two days
later symptorn$ alld *igns of acrte liver faiI:wre ap' 6&. Mostcofflmon a*tx*e o{ &eatloiwcascolt *cwf'*po,&w'
peared. Child was fotrnd to bc poritive for !trbs Ag. myetriti* ia - (AIIMS lune 97)
The mother was also HSs Ag carmer.lbe rn*ther's a) Intercostal muscles paralysis
hepatitis ts sererlogicaf profile is likely to be - b) Convulsion
a) (At 03)
HBs Ag positive only c) Cardiac arrest
b) HBs Ag and Hbe Ag PositivitY d) Respiratory failure
c) HBsAg and anti - HBe antibody positivity
69. Pnfio virus i*€e&iar] ean rcss]t in all cxeept -
d) Mother infected with mutant HBV a) Anterior horn cell damage (AIIMS Dec 91)
63. A neonatc born to ir#ected hepatitis- & mo€hcr, b) Autonomic involvement
slrould tre treated with - (AI 09.99)
c) respiratoryinvolvement
a) Isolation d) Paralysis in > 7\oto ofcases
b) Immunoglobulins 78. Kenny Paeks were $sed irc thc treatmert trf -
c) Hepatitis-B-Vaccine a) Poliomyelitis (AIIMS 03)
d) Immunoglobulins & hepatitis-B-vaccine b) Muscular dystrophy
64. A 30-year old Nady &elivered a hcalthybaby *t37 c) Polyneuropathies
week of gestation. $he was a known case of ctrronie d) Nerve injury
hepatitis E i*fe&ian $he was pasitive for H"SsAG bwt 1t A"ewtctlaeetdparalysi* is reported im a ehifd aged -
megativefar6BeAG. r#hich of the follenring is the a)- 0-3 years (AI 02)
mo$t apfrroprtatetreatmewt for the bab,y - b) 0-5 years
(AllMS Nov 05)
c) 0-15 years
a) Both active and passive immunization soon after
d) 0-25 years
birth
b) Passiveimmunization soon afte birth and active 72. ?rtre abeitrt 6roiio - @llMS Nov 07, Mav 08)

immunization at 1 Year of age


a) Paralytic polio is most common
c) Only passive immunization soon after birth
b) Spastic paralysis
d) Only active immunization soon a{ler birth
c) IM injections and increased muscular activity
lead to increased paralysis
65. A 5 year old b ay is detecledto be HEs Ag grcsitive otr d) Polio drop given only in < 3 Year
wt separaCe oeea$iofi$ during a *ereening prograr*
co]rlcENITAL SYPltIHS 81. Congenital toxoplasmosis-False is- (AI\MS May t0)
a) Diagnosed by detection of IgM in cord blood
73, $rhich of the following STDc cause fetal abnormali- b) IgA is more sensitive than IgM for detection
ty- (Ar e4) c) Dye test is gold standard for IgG
a) Herpes b) Hepatitis d) Avidity testing must be done to differentiate
c) Gonococci d) Syphilis between IgA & IgM
74. Congenltal e'yphilis can be best diagnosed by - 82. Diagnosis of toxoplasmosis ia newborn is done by?
(All India Dec.14 Pattern) (CET Aug.lj Pattern)
a) IgM FTA- ABS b) IgG FTA-ABS a) IgG antibody against toxoplasma
c) VDRL d) TPI b) IgM antibodies to toxoplasma
75, Premature baby of 34 wts was delivered. Baby de- c) IgA antibodies to toxoplasma
veloped bullous lesion on the skln and X-ray shows d) IgE antibodies to toxoplasma
periostitis, I{hat should be the nect investigation - 83. Symptomatic neonatal CNS involvement ls most
a) VDRL for mother & baby (AIIMs Nor, 11, AI 07) commonly seen in which group of congenital intra-
b) ELiSA for HIV uterine infection - (At 08)
c) PCR for TB a) CMV and toxoplasmosis
d) Hepatitis surface antigen for mother b) Rubella and toxoplasmosis
76. Hutchisons traid is seen - (CET Nov. 14 Pattern) c) Rubella and HSV
a) Primary slphilis b) Congenital syphilis d) CMV and syphillis
c) Secondary slphilis d) Tertiary syphilis 84, In neonatal herpes, true is - (PGI lune 2000)
a) Caused by HSV-II
I lrt FECTTOUS MOtrtoil UCLEOSTS b) Hepatosplenomegalyis diagnostic
c) Spontaneous recovery occurs

77, An 8 year old boy presented with fever and bilateral


d) If not treated, disseminates
cervical lymphadenopathywith prior history of sore 85. Most common cause of postauricular lymphadenop-
throat. There was no hepatomegaly. The peripheral athy in children - @nMS May 93)
blood smear shows > 20o/alympho-plaemacytoid a) Sore throat
cells, The mo$t likely diagnosis is - (Ar 02) b) Pediculosis capitis
a) Influenza c) Pulmonary Kochs
b) Tirberculosis d) Chronic suppurative otitis media
c) Infectiousmononucleosis A child with fever and sore throat developed acute
d) Acute lymphoblastic leukemia cervical lymphadenopathy most likely investigation
78. An I year old child with acute lymphadenopathy, to be done is - (AIIMS June 2000)

feyex,20o/o atypical lymphocltosis indicates the diag- a) Openbiopsyofnode


nosis of - (AIIMS May 93) b) Radical neck dissection
a) Infectiousmononucleosis c) Neck X-ray
b) ALL d) Complete hemogram
c) Pulmonary Kocht 87. Most common manifestation of HPV infection in
d) n-hemolltic Streptococcal infection children - (PGr 2K)
a) Single papilloma
MISCELLANEOUS b) Multiple papillomatosis
c) Osteoma
79. 1.8months old childcellulitis of leg SpOr 887o, no d) Sarcoma
prior history of hospitalization or illness most prob- 88. Number of pustules to diagnose a possible severe
able organism is- (CET Nov 15 Pattern) bacterial infection in children is? (CET Aug.13 pattern)
a) MRSA a) >5 b) >10
b) streptococcuspneumonia c) >15 d) >20
c) Streptococcalpyogenes 89. Resistant plasmodium falciparum malaria in the
d) Allofabove pediatric age group should be treated with - @I0B)
80. I{hich of the following does not establisha diagno- a) Chloroquine
sis of congenital CMV infection in a neonate - b) Tetracycline
a) Urine culture of CMV (AILMS Mat 06) c) Clindamycin
b) IgG CMV antibodies in blood d) Doxycycline
c) Intra-nuclear inclusion bodies in hepatocytes
d) CMV viral DNA in blood by polymerase chain
reaction

III
CriAptEe::li

ANSWERS
1. Ans. is None > c i.e., HSV lRef : Infections : Microbialogy {r w*nagetnent 3d/e p. 3461
o Sorry guys, all the four organisms given in options can cause transplacental spread.
o Common transpiacentallytransmitted infections
l.Viral -+ CMV (most common), HBV HCY HIV Rubella, HSV Parvovirus-B19, Variceliazoster, coxsackie
virus.
2. Bacterial -+ Slphilis, TB
3.Parasites -+ Plasmodium, toxoplasma,T-cruzi.
r However, we have to choose one options. Option t' (HSV) seems best to me :-
'Although HSV infection can couse transplacental (Pre-natal) infection; it is more appropriately considered
under perinatally acquired infections (during delivory)". Infectious neonatology
"lf herpes seroconversion occurs early in pregnancy the risk of transmissiort to the newborn is very low. In women
who acquire genital herpes shortly before delivery, the risk of transmission is high". Harrison
"Despite there inclusion in table, many viruses, including HSV and hepatitis viruses, are rarely acquired as
intrauterine infections. They are typically transmitted vertically (i.e. during the delivery process".
Pediatric hospital nredicine

Congenital rubella

2. Ans. is 1d' i.e., Infection after 16 weeks of gestation results in major congenital defects lRef lawetz 23dle p.
567, 568; Harcisan l&le p" 1153)
In general, the earlier in pregnancy infection occurs, the greater the damage to the fetus. Maximum damage to the
fetus occurs when infection is acquired in the first trimester of pregnancy.
o Maternal viremia associated with rubella infection during pregnancy may result in infection of the placenta and fetus.
o The most important factor in the pathogenicity of rubella virus for the fetus is gestational age at the time of
infection, The eailier in pregnancy infection occurs, the greater the damage to thefetus.

3. Ans. is t' i.e., Aortic Stenosis [Ref Nelson 18th/e p. t j40; Myung Snle p. 2Z]
. Nerve deafness is the single most common clinicalfinding amonginfant with congenital Rubella syndrome.
t PDA is the most common CHD in congenital rubella synilrome.
o Classical triad of congenital rubella consists of --> Cataract, Deafness, CHD
r Mentil retardation is also common.
4. Ans. is t' i.e., IgM antibody in fetat blood [nef Nelson t8,hle p. 13401

Laboratory diagnosis
r Isolation ofvirus in cell cultures ofthroat samples, urine or other secretions.
t Normally, maternal rubella antibodies in the form of IgG is transfened to infants and is gradually last over a
period of 6 months, Persistent beyond 7 year in an unvaccinated child suggests the diagnosis of congenital
rubella.
o Presence of Ig M antibodies is diagnostic.
r PCR for rubeila RNA.
5. Ans. is'b'i.e., Renal anomalies ["&t' Netrsom lStt'/e p. ]34A]

6. Ans. is'c'i.e., Conduction de&et [Re/ Nelso* 18h/e Chap. 2441

7. Ans. is 'd i.e., Patent ducfus arteriosus l&ej: Internet)


c Patent ductus arteriosus is the most common congenital heart disease associated with rubella, followed by pulmonary
artery stenosis.
o Heart disease in congenital Rubella -+ PDA > PS > VSD > ASD.

CHICKEN POX

8. Ans. is 'b' i.e., Sec. bacterial infection [Rel O.P Ghai 8th/e p. 214 & 7h/e p. 186; Harcisan 17/e 110j1
e The most common infectious complication of varicella is secondary bacterial superinfection of the skin,which is
usually caused by streptococcus pyogenes or Staphylococcus aureus.
cerebellar,
. The ffiost comrnon extracutaneous site of involvement in children is cNS --> Miningitis, Encephalitis,
at axia, Rey e's syndr om e
: Varicella pneumonia is the most serious complication Jollowing chickenpox in adults,

9. Ans. is t' i.e., Enteritis lRef: O.P- Ghai 8tu/e p. 214 & Vh/e p' 186]

10. Ans. is 'a' i.e., When the lesions have crusted IRel O. P. Ghai 8&/e p. 214 6 Ple p. 186)
on the first
r Varicella Zoster (chicken pox) is a highly contagious disease, a centripetal, pleomorphic rash appearing
day of the illness and a relatively short course of illness'
r The crusts from chicken pox lesions donot contain viable virus and are therefore not
infective. Period ofinfectivity
range from a day prior to the illness and up to 5 days after the onset.
ofthe rash.
children with chicken pox are removed from school or day care centres for 6 days after appearance

11. Ans. is 'a' i.e., Yarecella virus [Ref Nelsor 1*/e p' 1j68]
reduction defect (limb
r Congenital varicella syndrome is characteristically associated with scarring of skin anil limb
hypoplasia).
Scarring of skin is the commonest feature of congenital varicella syndrome and the characteristic
cutaneous
t
lesion is called "Cicatrix".

NilEASLES

lZ. Ans.is'di.e",Paramyxo viras{Ref :Annnthanarayon7h/ep.517;O"P"Ghai8th/ep'2136*/e p- 1851

r It is paramlxo virus - enveloped, RNA(ss RNA) virus


13. Ans. is t'i.e., SSPE [Ref Ghui */e p. 214 & Harrison 1P/e p. 1215; 1&/e p. 1150, PSM Park 1*/e p.128)
develoPs many years after the
r Subacute sclerosing panencephalitis (SSPE) is a rare complication of measles, which
initial infection - PSM Park 19th/e p. 128
The most common complications are : measles- associated diarrhea, pneumonia and
r otitis media.

14. Ans. is'b' i.e., 4 days before pand 5 days after rash [Ref O.P. Ghai 8*/e p- 213)
r period of communicability for measles is 4 days before and 5 days after appearance of rash.

15. Ans. is rtl' i.e., *Ieasles tRef OP Ghai 8/e p' aa)
16. Ans. is '-o- i,e., Measles f&ef : Harrison 17&/e p' 1215; 16*/e p' 1149)
measles and is seen in
t primary giant cell or Hecht's Ttreunonia is seen in case of measles. It is a complication of
immunocompromised and malnourished patients'

fi,luMPS
l*le p. 1#/e p. */e p" 217 6 */e p'
tT" Ans. is.d i.e., Orchitis and Oophritis [Re/: Ha'7'bott 1220; 1154; O-F. Ghai

1881
,,Orchitis is the most cotnmofl comltlication of mumps among posq,ubertal males , developing in about 20%o of
" Harrison
cases
,,
Mumps virus mostly causes a milit childhood d.isease , but in adult complications including meningiti-s and
orchitis arefairly commoru" Iawetz 23th/e - 560

t, i.e., Meningn€ncephaliti$ can precede parotitis [Ref Nelsoa 1*/e p. 1342; A.P. Ghai gele p. 217 &
lg. Ans" is
frleP.1Sv-188\
meningitis may develoqt before, during, after or in the absence of parotitis!'
,Aseprtic lVh/e-1220

About other oPtions


r Mainly parotid glands are involved, but sublingual and submaxillary glands may also be involved.
r Patient is infectious 4-6 daysbefore the onset of swelling'
r Sterility after mumps orchitis is rare.
FEVERWITH RA5H

19. Ans, is 'b' i.e., Fifth disease lReJ: A.P. Ghai }t,,/e p. 216 6 6t,/e p. 2t)B)
r Erythema infectiosum also known as fifth disease is caused by parvo virus (B-19).
20. Ans. is'b'i.e., Roseala infantum tRef O.p. Ghai }th/e p. 216 (r 6th/e p. 2A8l
t In roseola infantum the temprature comes down rapidly in 3 to 4 days when a macular or maculopaytular rash
appear' starting from trunk and sometimes extending to the extremities and face.
21. Ans. is t'i.e., Rubella [Rel Nelsoa tSth/e p. 1i39; O.P. G.h*i Stu/e p. Zt6 & dh/e p. 2Ag, 586l

a Roseola 10-14days Fever (3-4 days) Pink macular rash appear- o Usually occur in
lnfantum (6th ing at the end of illness. children between
disease) caused c Resolves within 2 days 2-6 yrs. of age
by HHV-6 t It spares the face

Rubella 14-21 days Usually None t, Mild maculo papular Rash, Postauricular & sub-
Rapidly spreads and occipital adenopathy
gone by day 4 & Polyarthralgia
Erythem 13-i8 days e Slapped cheek appearence t May cause Aplasfrc
infectiosum e Reticular rashthat waxes & crisis
(Fifth disease) wanes over 3 wks
caused t ln 50%o childrenrash re-
byParovirus mains -after 1O days

KawasakiDisease Unknown , Fever o Polymorphous rash t Conjunctivitis


c Cervical 0 On trunks and extremities, r Thrombocytosis
* Lymphadenopothy red palms and soles, lips, o Coronary arteritis
tongue and pharynx
Desquamation present

,,)
Ans. is'c'i.e., Roseola infanturn lRet A. P. Ghai }th/e p. Z16 6 d'/e p.208|
e The child is having roseola infantum. It is caused by Human herpes - 6 (HHV-6).
r The most prominent feature is abrupt onset of fever often reaching 40.6"C. The fever then ceases
abruptly and a characteristic rash may appear. The rashes are maculopapular. Are non pruritic tend
to coalesce and disappears in 1-2 days without pigmentation or desquamation.
r Erythema infectiosum or fifth disease can be easily ruled out because thefirst sign of illness is the rash no fever.
The rashes are masculopapular and they coalesce to given a slapped check appearance. One half of the infected
children have some rash remainingfor 10 days fine desquamation may be present.
o In measles the rashes last for 6-7 days.
r In typhoid the rashes appear on the 7h day of illness.
23. Ans. is'b'i.e., Caused by HHV 6 & 7 [Re/ A. P, Ghai B,hle p. 216 6 6thle p. 2081
o Roseola infantum is caused by HHV-6. During defervescence (disappearance offever) rash appears (defervescence
does not follow rash rather rash follows defervescence).
r Rash starts from trunk.
o Slapped cheek appearance is seen in er1'thema infectiosum (not in roseola infantum).
. Otitis media is not a compiication.

24. .A.ns. is'b'i.e., Caused by HHV 6 & 7; t'i.e., Rash appears in trunk; t'i.e., During defeverescence> rash
appear$ tR6l O. P. Ghai 9tkle p. 216 dr #hle p. 2ASl

)q Ans. is 'a'i,e., Slapped check appearanee seen; ob' i.e", Causd by parvovirus & t' i.e., Defervescene before
rash [.eel o. P. Ghai 8.hle p. 216 & 6th/e p. 2A8l
c Er1'thema infectiosum is caused by Parvovirus and is characterized by slapped cheek appearance. Defervescence
(disappearance of fever) occurs 1 week before rash. Rash appears on cheek.

26. Ans. is t'i.e., Typhoid - day 5


r Mnemonic for Day of appearance of rash in a febrile patient is :

Very Slck Person MustTake DoubleTablets


r Very - Varicella (day 1)
r Sick - Scarlet fever (day2)
o Person - Pox-small pox (day 3)
o Must - Measles (day a)
r Take ' Typhus (days)
r Double - Dengue (day 6)
r Tablets - Typhoid (day7)

TUBERCULOSIS

27. Ans. is 'a' i.e., Commonly sputum negative & t' i.e., Clinically child does not $how sign of florid TB
lRef: Nelson 18th/e p. 1246, 12471
o Cases of pediatric tuberculosis represent 5- 15% of all TB cases.
r The most common type of pediatric TB is pulmonary TB, of which sputum smear-negative disease is most frequent.
r Most cases of pediatric TB develop no signs or symptoms at any level.
28" Ans. is'None' lRef: Nelson 18th/e p, 1246, 12471
c All may occur (See text of the chapter).

7q Ans. is t' i.e., Futrmonary T.B. tRd CMDT 2002/e p. 310; APl6h/e, p. 2441
r The history and tuberculin test strongly suggests the diagnosis of T.B.
o Do not get confused by negative sputum cytology because negative sputum cgology does not rule out pulmonary
tuberculosis.
" Sputum cytology is just a presumptivg diagnosis" - Harrison
The definitive diagnostic of T.B. depends upon the culture of bacteria obtained from respiratory
-"specimen" - Harrison
r The presence of
.induration
> 15 mm in size strongly indicates T.B. infection.
30. Ans. is t' i.e., Lymphnode lRef Ghai Vh/e p. 2121
r Most common site of extra pulmonary TB is lymphnode
31. Ans. is'a' i.e., Liver lRef: Nelson 18th/e ch. 212|{EJM 1994)
e Overall liver is most commonly involved in congenital tuberculosis.

32. Ans. is 'd i.e., ELISA fRef: o.P Ghai 6th/e p. 2351

33. Ans. is't i.e,,l0 -15 mglkgldg


o Dose of INH (in thrice a week RNTCP protocol) is 10-15 mg/kg.
34. Ans. is t' i.e., BCG + INH for 6 weeks + Breast feeding lRef: Nelson 18th/e p. 12531
r BCG is given to all new born infants
r INH therapy for newborns is considered'essential' these days. It is so effective that seperation of mother and infant
is no longdr considered mandatory. Ideally INH therapy should be continued for atleast 3 months after the mother
has been shown to be sputum negative. However, an initial course for 6 weeks, can be considered, if reevaluation is
planned at the end ofthis period.
r Breast feeding : can be continued in presence of INH therapy to the child.
In the Indian context, Breast feeding is very important and definitely deserves a place in management of this child.
r Rifampicin as a modality holds no place in the current situation as the child is not mentioned to be diseased'.
Moreover, the mother is already on ATT.

35. Ans. is 'H i.e., Continue for 3 month (WHO & RNTCP guideline) fRef: rlMR 2ila fuly edition)
"Indian academy of Pediatrics (IAP) suggests INH prophylaxis for at least 6-9 months or minimum for 3 months until mother
is culture negative".
t)iij

tl.
reH..At'p..TBR,

PERTUSSIS

-r6 -{ns. is 'tr' i.e., 7-\4 &*y* lRef: 0"P. Gfuai $tt'/e p" 243 d' Thle p. 221)

o Incubation period of pertussis is 1-2 weeks.

J) Ans. is'b'i.e., < 5 IRe/ A"P. Ghai Yft/e p. 24j & 7h/e p' 221]
r In prevaccine era, and in countries where immunization is limited the peak incidence of pertussis is in children 1-5
years ofage, infant accounts for < 15% ofcases.
r In recent years, where vaccination is employed, approximately one half of cases have occuured in infants younger
than 1 year ofage and one fourth in adolescent and adults.

38. Ans. is'H i.e.n Cerebellar ataxia [,Ref 0.P' Ghai 9th/e p. 24j & Vh/e p. 221)
a Although ataxia is seen in Pertusis, it is not of cerebellar origin There is no involveoment of cerebellum in Pertusis.
nt)'
-i9. Ans" is i.e", Fert&ssis tRel 0.p. Gkai &'t'/e p. 243 6 6il'le p" 193)
r Protective Antibodies against pertussis doe not corss plecenta.
e So, early immunization is desired.

40. Ans. is *d i.e., Nasopharyngeal swab lRe! tr{arrisan 17hle p. 9351

"Culture of nasopharyngeal secretion remains the gold stadard for diagnosis of whooping cough" - Harrison

ua,
41. Ans is i.e., Nasopharyngeal swab lRel Skerr's Meilical Micrabiolagy Sthle p. 56jl
* presenceofrecurrentboutsofseverecoughfollowedbyanauiliblewhoopsuggestsadiagnosisofpertusis(Whooping
cough).
* Nasoltharyngeal Swab is the single best specimen to isolate the organismfrom the options provided,

42. Ans. is 'c' i.e.n Erjthrornyein [Ref CI.P. Ghai *'hlc p' 24j & Vh/e p. 222)

Treatrnent of Perttlssis
DOC -) Macrolides (Ery'thromycin, Azithromycin, Clarithromycin)
Alternative -) Cotrimoxazole

DIPHTHEK'A
od' Medicine 6't'/e
.13. Ares" is i.e,, Lyurphadenopethy {Re!: APtr p^ 127)

c A small percentage of patients present with massive swelling of the tonsils and uvula with cervical lymphadenopathy
(butl neck) and severe toxemia -+ Malignant diphtheria,
M. Ans" is'd'i.e", Hepatic failure l&ef: API6hle p. 1271

Cormplications
e Obstruction of the respiratory tract by pseudomembrane
* Myocarditis
o Polyneuropathy
r Post diphtheritic paralysis
w Occurs in the 3rd or 4th week
a Palatine and pupillary paralysis is characteristic
e Spontaneous recoverY is the rule,
o Pneumonia
e Other less common complications are renal failure, encephalitis, cerebral infarction, pulmonary embolism and
bacteremia or endocarditis.

HIV- INFECTION

45. Ans, is 'a'i.e., Generalised lyrnphadenopathy lRel Ghwi Thle p, 2021

r Persistant generalizedlymphadenopathy is clinical stage 1'


46. Ans. is 'd i.e., Perinatal transmission lRef: Nelson lVh/e p. lltlJ
r HIV can be transmitted from mother to fetus/infant in the following ways :
a) In utero (During pregnancy)
b) During delivery (Perinatal) -+ Most common
c) After birth by breast feeding -+ Least common.

47. Ans. is t' i.e., Exclusively top feeding &'d' i.e., Breast feeding for 6 months & then rapid weaning [Rel.Netsom
ISth/e p. 14291

1. Developed countries Substitute top feeding for breast feeding


2. Developingcounteries Breast feed for 6 months followed by rapid weaning

48. Ans. is t'i.e., Failure to thrive lRef. wvtww.hivinckildren.org\


r Failure to thrive is universal.

49, Ans, is'd'i.e., Kaposi Sarcoma fRef: Avery's disease of Newborn 8th/e p.488)
rady in children",
"Kaposi sarcoma, commonly seen in men with advanced HIV disease, has been reported only
50. Ans. is'c' i"e., Recurrent Chest infection with atypical organism fRef: Ananthanarayan Vh/e p. j03l
a " Pneumonia caused by pneumocystis carinii is the most common AIDS defining diagnosis in children with
unre co gni ze d H IV infe cti o n"

51. Ans. is t' i.e., HIV Elisa fflef. A.F. Ghai 8'h/e p. 23,4 {r Vh/e p. 207; Nelsaw l6thle p. 1A28, C.P.D.T" Id,t,/e p. g44l

Diagnosis of HIV in Infant


r In adults HIV can be easily diagnosed by detecting IgG antibody to HIV (Elisa and western blot test). Buf
this method is not helpful in case of neonates because all neonates born to HIV infected mothers will have lgG
antibody in their blood, regardless of infection status in them. These antibodies are passively transferred to the
newborns from their mothers (IgG can cross placenta). These infants continue to test positive for IgG antibodies
for upto 18 months. At the end of 18 month theywill lose these antibodies so uptil 18 month of age this method of
diagnosis cannot be used.
o The presence of IgA or IgM anti HIV in the infant's blood can indicate HIV infection, because these classes of
antibodies do not cross the placenta. However detectable quantities of IgA antibodies are detected only after 3
months of life and detection of IgM antibodies is very unreliable (both insensitive and nonspecific).
r So directviral detection assays are used for the diagnoses ofHIV in newborn. These are :-
1) detection of HIV DNA or KNAby PCR.
2) HIV culture
j) HIV p24 antigen
4) Immune complex dissociated p24 antigen.
r These are very useful in young infants allowing a definitive diagnosis in most infected infants by 1 - 6 months
of age. Out of these method detection of viral DNA by PCR is the preferred method in developed countries.

52. Ans. is 'b' i.e., Vaginal cleansing before delivery lRef:OP Ghai 8th/e p. 235-237 6 Vh/e p.207; Williams Ob* 2l't/e
P. 15A3, 1504)

Methods to prevent vertical transmission


a) Antiretroviralprophylaxis-
b) Caesaren delivery -
c) Breast feeding -
r Because breast milk can carry the virus, breast feeding by
HIV infected mothers is contraindicated.
r However Dutta/obs writes -
"In the developing woild, where alternative forms of infant nutrition are not safe, minor risk associated with
breast feeding may be accepted. Mother is counselled as regards the risk and benefits. She is helped to make
an informed choice."
c"i*,r;a,,iot
53. Ans. is 'b' i.e., AZT therapy [Ref : a,p. Ghai Bth/e p. 232 6 6th/e p. 223, 224; Harrison tlh/e p. t 14s]
I HIV infected mother can transmit the virus to fetus/infant during pregnancy, during delivery or by breast feeding.
r Early diagnosis and antiretroviral therapy to mother and infant significantly decrease the rate of intrapartum and
perinatal transmission (Vertical transmission) of HIV infection.
r Zidomdine treatment of HIV infected pregnant women from the beginning of the second trimester through delivery
and of infant for 6 weeks following birth decreases the rate of transmissio n from 22.6 o/o to < 5o/o
r Single dose of nevirapine given to the mother at the onset of labor followed by a single dose to the newborn within
72 hours of birth decreased transmission by 5Oo/o. This is the prefered regimen now in developing countries.

54. Ans. is 'a'i.e., Neviraphine lRef: O.P. Ghai }th/e p, 2j5 6 Vh/e p, 202; Nelson tVh/e p, 11201

55. Ans. is t' i.e., Transmission rate during pregnancy exceeds gAYo
lRef: O,P, Ghai Sthle p, 234 & Vhle Xt" 203,
207)

56. Ans. is'a' i.e., OpV IRel Ghai 9th/e p. 237 6 Vh/e p. 20fl
o The vaccines that are recommended in the national immunization schedule can be administered to HIV infected
children except that symptomatic HIV infected children should not be given OpV and BCG.
5/. Ans. is'None' lRel Ghai Yth/c p. 2j4 6 6th/e p, 2A4, 205; Nelson t9th/e p. 1430-1432)

Indications to start ART in ehildren


r To know the indications of ART in children, one should know the clinical and immunological staging of HIV
infection in children :-
l. Immunologicalcategories

Cells/uL o/
/o

1. No evidence >1500 >25 >1000 >25 >500 >25


of suppression
2. Evidence of 7SO-14gg 15-24 500-9991 5-24 2oo-4gg1 5-24
moderate
suppression
3. Severe <750 <15 <500 <15 <200 <15
suppression

2. Clinicalclassification

N No (asymptomatic)
A .. .,,,.,r.',.,Mild . :;

B Moderate
c Seveie:"'. '

Clinical categories are further subdivided depending on the immunological categories for example, If a 3 years
old child has mild signs and symptoms (clinical category A) and CD4 T cells between 500-999/mL or l5-24o/o
(immunological category - 2), then it is classified as clinical category A2. Following table will help to remember
this division :-
pediatric HIV classification for children younger than 13 years

lmmunologic Categories
Clinical classification
Age-Specific CD4 + T' Lymphocyte count and
Percentage of total lYmPhocYtes

months months <'t2 months N:No sings A:Mild B:Moderate C:Severe,


or - signs and signs and signsand
< 12 < 12
lmmunologic
definitions pLL % PL % PL o/o
Symptoms Symptoms Symptoms Symptoms

1. No evidence >'1500 >-25 >1000 >25 >500 >25 A1 B1 C,1

of suppression
2. Evidence of 750-1499 15-24 500-999 15-24 200-499 15-24 A2 c2
moderate
suppression
3. Severe <750 <15 <500 <15 <200 <15 N3 A3 B3 c3
suppression

r Now see the indications of initiation of HRT in children :-


A) Regardless of Age
1)HIV infected children with symptoms (clinical category K,'B', or'C')
2)Euidence of immune dysfunction (immune category 2 and 3)
B) Infant <1 YearofAge
e All infant infected with HIV regardless of clinical or immunological category.

Coming backto question


o All the given options are indications for initiation of ART :-
r < l1 months old child with CD4 < 1500/prl falls under immunological category 2 -+ indication for ART.
s 12-35 months child with CD4 < 500/pl falls under immunological category 3 -+ indication for ART.
t 36-59 months child with CD4 cells < 350/pl falls under category 3 -) indication for HRT.
r > 5 years old child with CD4 cells < 200 cells/pl falls under immunological category 3 -+ indication for ART.

yohle
Ans. is i.c., All symptomatic HIV infeeted children >5 years of age irespective of CD4 [Ref Glraj
.a' p'
Sg.
2i5 & */e P. 2as-2071

cotrimoxazole prophylaxis in Hlv exposad/infeeted ehildren


A) Cotrimoxazole prophylaxis in HIV exposed infant is recommended for :-
l) Atl HIV."por.i infants (started at 4-6 weeks of age and continued until HIV infection can be excluded).
2) HIV exposed breast feeding children of any age (Prophylaxis should be continued until HIV infection can be
excluded at least six weeks after complete cessation of breast feeding).
B) Cotrimoxazole prophylaxis in HIV infected children is recommended for
l) All children younger than 1 year of age, regardless of symptoms or CD4 percentage.
Z) For symptomatic Jhildren (ciinical stages 2, 3 or 4) or children with CD4 <25o/o -+ In children older than one
year of age.
All HIV infected children who begin cotrimoxazole prophylaxis should continue until the age of five years
(irrespective of the age of initiation)'
At 5 year, children can be reassessed'

HEPATITIS

59" Ans. is'd i.e., 2 wk - 2 nraonths lQef: A.P. Ghai Sthle p' 22$ {r Vhle p' }91}

HAV *+ 15 - 45 days (mean 30 days)


HBV ----+30 - 180 days
HCV -4 15 - 160 days (mean 50 days)
HDV 180days
HEV -+30-
---*14 - 60 days (mean 40 days)
60. Ans. is t' i.c., Hepatitis B virus {Ref: I,{elson 18th/e p, 16801
t Maximum perinatal transmission is seen in HBV

6t" Ans. is 'd'i.e., Elood transfusion lReJ: A,p, Ghai Bth/e p, 221 & Vh/e p" 192]
"Children usually acquire infection indirectly, through parenteral route
from hepatitis B contaminated blood
transfusion".
Ans. is'b'i.e., HBs poeitive and rtrbe Ag positive l&ef: o,p, Ghai rth/e p" 221 & vhlc p. 19s)
Most important riskfactor that determines perinatal transmission is HBe Ag. HBs Ag carrier mothers
' Ag positive almost invariably (> 90%) transmit hepatitis B infection to their offspring, where as
who are HBe
carrier mothers with
anti HBe rarely (10 to 15%) infect their offspring.
63. Ans. is'd'i.e., Immunoglobulins & Hepatitis B Vaccine l%ef: A"p, Ghai Yth/e p. 222 & ?k/ep. t9S]
"Infants born to Hbs Ag positive mothers should receive both HBIG and
HB vaccine intramuscularly at seperate sites
within 12 hours of birth, followed by 2nd and 3rd dose of vaccine at 1 and 6 months respectively''
64. Ans. is '{ i,e,, Both active ond passive immunization soon aftcr birth p,
[Ref Nelson lgth/e 16g5]
uU
65. Ans. is i.e., Obtain anti HBs levels [Rel Nelsan tSthle p. 1684]
The child in question has received complete vaccination for Hepatitis B, is currently asymptomatic
but has tested
positive for HBsAg.
The aim in avestigating the child further is to establish his immune status which can
be best revealed by obtaining anti
HBsAg levels.
c If the tests shows inadequate levels of anti-HBsAg it would indicate that the vaccine had failed
to mount an
adequate immune response. Such a child should consult a pediatric hepatologist.
I If the test shows adequate levels of anti-HBs Ag, the adequacy of vaccinition is confirmed and as the child is
asymptomatic no further intervention is required.
e Positivity of HBsAg indicates that the child is a carrier of the antigen. Vaccination does not
effect the HBsAg
carrier state.

TETAIIIUS

66, Ans. is'e'i,e., Usually oecur in l't 2 days of lifc


fRef: Ghnt rth/e p, 247 & Vh/e p, 226; Nelson ty,hle p, tzzg, tzzgS
c Symptoms of neonatal tetanus starts by 5-15 days after birth, never in thefirst 2 days of lifeand rarely after the age of
2 weeks. (It is commonbt k/a as 8tu darv disease)

a./. Ans. is'd'i"e", I"P" is variable lRef : Nclson t&th/e p. t2Zg, t22g)
tIncubation period - is variable from two days to several weeks, but is commonly 6-12 days.
Note : It is tetanus toxin not bacilli that spread along the nerve.

PCILIO

68. ,dns" is nd' i"e,, Rc*piratory failurc tgh/e p, t7A; Ghat &th/e p, 2tg & Vh/e p" i90l
[F.cf: Pnrke
r Death is usually due to complications arising from respiratory dysfunction.

69. Ans. is 'd' 1.e., Faralysis in > 78 of casce l&cf: A,p" 6hal &eh/e p. 2t B & Vhle p, 19A!
c Paralltic polio is seen in less than l% ofpolio infections.

vCI, Ans, is 'd i.e,, Poliomycliti* lRef: A"p, Ghat


6th/c p, 212 & fh/e p, $al
o Relief of pain and muscle spasm --> Hot moist packs applied to the muscles relieve pain and spasm (sisfer
kenny,s
treatment).

71, Ans, is ne'i"c., 0-15 years {Rcf: A,P, Ghnt th/e p, 218 & Vk/c p, 190}
Acute faccid paralysis (AFP) surveillance
r Every case ofacute falccid paralysis in any child under 15 years is to be reported promptly.
c Fecal samples of all these cases are obtained (2 samples) and transported to the laboiatory within 72 hrs of
collection at 4-80 C or frozen at -200C. The process is also knwon as reverse cold chain.
72. Ans. is 'c' i.e., IM injecfions and increased muscular activity lead to increased paralysis [ReJ 0.P. Ghai 8*/e
p. 215 6 Vhlep. 190)
o Risk of paralytic polio is increased by:-
r Tonsillectomy r Strenous physical exercise I Tooth extraction r Cortisone administration
r Adenoidectomy r Injection (intramuscular) r Fatigue

About other options


r Most of the infected individuals (95%) develop inapparent infection and paralytic polio occurs in less than 1% of
the infected individuals.
o There is flaccid paralysis.
r pulse polio immunization is given to all children less than 5 years of age (not 3 years of age).

CONGENITAL SYPHILIS

V3. Ans. is ? i.e., Syphilis lRef Harisan lfllep' 1042, n$1


74. Ans. is'a' i.e., IgM FTA-ABS [Rel llsrrison 16'hle p. 982]
e As Ig M does not cross placenta, its presence in neonatal serum confirms congenital slphilis and helps differentiating
it from seropositivity due to passively tranfered maternal antibodies. The tests to detect IgM are
t FTA-ABS (195 IgM FTA-ABS)
r Syphilis CaPita M test
"For meysurment of Ig M in neonate in whom congenital syphilis is suspected, the syphilis capita M test and the
195 lg M FTA-ABS test are available". - Harrison 16'h/e 982
Note - 1 7trle Harrison says, "No commercially available IgM test is recommeded for evaluation
of infant with suspected

congenital syphillis". - Harrison lVh/e 1043

?5. Ans. is'd i.e., VDRL for mother and Baby lRef CPDT 18e/a p. 56)
r Bullous lesion and periostitis suggest the diagnosis of congenital syphilis.
r The only option related to slphilis is option'al
r Following evaluations are recommended: -
1. physical examiation 2. A quantitative nontreponemal test for syphilis (VDRL).
3. CSF for cell count, protein & VDRL. 4. Long bone radiograph
5. AntitrePonemal IgM.

76. Ans. is'b'i.e., Congenital syphilis


t Hutchinson's triad is a common pattern of presentation of Congenital slphilia.
o It consists of three phenomena -+ l. Interstitial keratitis, 2. Hutchinson incisor, 3. Eight nerve deafness

INFCCTIOUS MONONUCLEOSIS

7?, Ans. is.b, i.e.r lnfectious Mononucleosls [ReJ o.P, GhatSe/ep. 216 6 fllep. 18fi
r Cervical lyrnphadenopathy, fever, history of sore throat and > 20o/o atlpical lymphocytosis (lympho-plasmacytoid
cells) suggest the diagnosis of IMN. Hepatomegaly may be absent (occurs only in 30% cases)'

78. Ans. is b' i.e., Infectious mononueleiosis IR€J A. P, Ghat #lc p. 216 & flle p. 1871

MISCELLANEOUS

79. Ans. is 'c' i.e,, Streptococcal pyogenes lRef: Nelson 18th/e ch. 664)

r Most common cause of cellulitis is streptococcus Pygenes'

80. Ans. ls U t.c" IgG eiUV antibodles in blood lRef llarrlson lffle p. fi51; Nelssn lfrlc p. 10681
,,
Ig G antibody test is oflittle diagnostic value as positive results also reflects maternal antibodiesl'
Diagnosis of Congenital CMV infection

J + +
Yirus islolation Serology PCRfor viral genome (DNA)
t Definitive diagnostic r Not very useful
r Specimens r IgM test lacks sensitivity
s Urine and specificity
r Saliva r IgG tests are not diagnostic
r Blood (buffycoat)
r Bronchoalveolar washing
r Amniotic fluid

About option C
CMV infected cells contain large intranuclear and smaller intracltoplasmic inclusions which are pathognomic.

81. Ans. is? i.e., Avidity testing must be done to differentiate b/w IgA & IgM lRef Cloharty 18th/e p. 3191
c Avidity test is used to measure the strength of antigen-antibody reaction for IgG antibody (not to differentiate
between IgA & IgM)
Ans. is't) > c'i.e., IgM antibodies to toxopXasma > trgA antibodies to toxoplasma
lRef: See above explanationl
r IgA has greater sensitivity, but IgM is the test of choice.

83. Ans. is t' i.e., Rubella end HSV {Ref": Nearologlt Vu$e ?d/e p. 63)
c CNS involvement is common feature from infection with all agents of TORCH group.
e With most infections, patients remain asymptomatic in neonatal period and serious neurological consequences
become apparent after neonatal period.
r CNS manifestations that are symptomatic in neonatal period are most commonly seen in -
i) HSV (76-100o/o)
ii) Rubella (26-600/o)
o CMV toxoplasma, and syphillis become symptomatic in less than llo/o-Llo/o of cases.

84. Ans" is 'a- i"e., Caused by HSV-II [ReJr Nelsoa {frlc p. rufi, ru541
o It is commonly caused by HSV-2 (70% cases are caused by HSV-2 and 30% caused by HSV-1 )

85. Ans" is'b' 1.e., Pedieulosis eapitis {Ref: Radalph Pedtstrics 21*/e p. 1155)
o Posterior or suboccipital ceryical lymphadenopathy without obvious disease ot' secondary infection is character-
istic oflice.
86. Ans. is t'i.e., eomplete hemogram {ReJ Nelson 1*/e p. 16V7, 1678,1A62-10661
c Cervical lymphadenopathy with signs of infection such as fever and sore throat points towards any infectious
etiology.
r In this. case the most likely investigation to be done should be complete hemogram.
r Complete hemogram can provide useful datas for diagnosis of :
s Pyogenic infections t Viruses such as EBV CMV (y HIV: Leukemias (acute or chronic)
r Biopsy should be done if the patients history and physical findings suggest malignancy. These features are :-
r Solitary, hard non-tender cervical nodes in an older patient.
r Neck X-ray does not provide much information except for giving some idea about swollen soft tissues of neck
r Radical neck dissection is absurd option.
87. Ans. is'b" i.e., Multiple papillomatosis [ReJ NeJson 18e/e p. 140j)
r Multiple warts are common.
88. Ans. is 'b' i.e., > l0 [Rel Park 22a/e p. SjZ, SS21

o An infant from 1 weekto 2 months is evaluated for possible bacterial infection under IMNCI (integrated management
of neonatal & childhood illnesses) guidelines.

Fast breathing > 60 breaths/min .: :: l.m ceftriaxone


Severe chest indrawing
,'Nasa},flaringahd grUnting . :' ::',;': :,,:: '' Rectal diazepam
:Convulsions : :,:
Check and.treat plood lugqr.
Bulging fontanelle' Refer urgently to hospital
Fever (axillaiy temperature) > 37.5o C or Continue breast feeding
to*bodyfemperaturei<Siso'g ;,,' .; ,: ' Keep the child warm
Mony;'orseverepus S',1>.10) :,.

Lethargic, unconscious
tess than'normal movements :. : -;

'Nobacteiial,infeetion.' None.of,th:em :.,,,


Relevanthealth equns-el.ing

89. Ans. is t' i.e., Clindamycin [Rel: Nelson 18th/e p. 1421, 14221

Treatment of reslstant falciparum malarla ln chlldren


:i: r.ii:i:.::.l r- l:'.:::. .:r::l lr,ri:.:t.]1.1;:;1,,,,,,...,.
. :,1, 1..

, Olrei r r,n.,r e :';afii:ltiour Disease


f{s" Ayrs" is 'i3' i.e,, > \tl {kr.j': /!*rk 22,',tle p. S3:-, SS21

o An infant from 1 week to 2 months is evaluated for possible bacterial infection under IMNCI (integrated management
of neonatal & childhood illnesses) guidelines.

l.m ceftriaxone
infection Severe chest indrawing o2
Nasal flaring and grunting Rectal diazepam
Convulsions Check and treat blood sugar
Bulging fontanelle Refer urgently to hospital
Fever (axillary temperature) > 37.5o C or Continue breast feeding
low body temperature < 35.5o C Keep the child warm
Many or severe pustules (>10)
Lethargic, u nconscious
Less than normal movements

Local bacteiial Red umbilicus or draining pus


infection Skin pustules < 10 For eye infection give i/m ceftiiaxone
Pus draining from ear Advise mother on care of infection
No bacterial infection None of them Relevant health counseling

89" Ans" is t'i.e., Clindamycin [Relr Nclsan t*thle p, t4Zi, 1422]


Treatment of resistant falciparum malarla in children
i) Child < I years of age
r Quinine + Clindamycin or
r Atovaquone + Proguanil
ii) Chilil > 8 years of age
r Quinine + Doxycycline / Tetracycline or
r Atovaquone + Proguanil
Note - Clindamycine can be used in place of doxycycline or tetracycline.
eoming to the guestlon
r Chloroquine is not used for resistant falciparum malaria.
e Doxycycline and tetracycline are used in children only above the age of8 years.
c Clindamycin can be used in both, above and below 8 years
o As age of the child has not been mentioned in the question, the single best answer is clindamycin.

TUI
$PKCXAL TOPXCS

Kawasaki disease
o Kawasaki disease, also known as lymph node syndrome, mucocutaneous-lymph node syndrome, and infant
polyarteritis, is a poorly understood self-limited vasculitis that affects many organs, including the skin mucous
membrane, lymph nodes, heart and blood vessel walls.
o It is usually in children younger than 5 years.
seen
a Kawasaki disease predominantly affect medium sized vessels, but may also affect small or large vessels.
a Presentation may be -
1) Mucocutaneous, lymph node involvement l Conjunctival injection, erlthema, strawberry tongue, rash with
perineal desquamation, Beau's line (transverse grooves on nails), cervical lymphadenopathy.
2) Heart: Coronaryvas cuhlis, coronary artery aneurysm (in 25o/o patientsanus11'At08)), coronary ectasia, MI, vah,.r.riar
regurgitation.
r Treatment of choice for acute phase is intravenous immunoglobulinattMsll'At0r) along with high dose of aspirin. For
subacute phase low dose aspirin is given.

Septle shock ln ehlldren


o Septic shock is an inflammatory state resulting from systemic response to bacterial infection.
t Decrease in peripheral resistance (vasodilatation) is the hallmark of early (hyperdynamic) stage of septic shock(Pcl
0e),
this feature distinguishes it from other types of shock. Decreased peripheral resistance (vasodilatation) leads to
compensatory increase in cardiac output; therefore, cardiac output is maintained or some time increased despite of
oe).
shock@Gr
o Decreased peripheral resistance and increased cardiac output leads to increased blood flow and total oxygen delivery
to the tissue. But, still the tissues lack O, due to local microcirculatory changes produced as a result of sepsis which
causes decrease tissue extraction ofoxygen. So, increased blood flow and cardiac out are still inadequate to meet the
total metabolic needs of the tissue which are increased in sepsis.
o As sepsis progresses' hlpodynamic stage supervenes in which there is decreased cardiac output and increased
peripheral resistance@Gt oe)
.

o So, hemodynamic changes in septic shock occur in two characteristic patterns, Early hyperdynamic and late
oe),
hYPo dYnamic shock(Pct
1) Early hyperdynamic shock
1 Hypotension is a late feahffe(Pcl,e) of septic shock and early hyperdynamic stage is characteriz ed.by normal or
increaseil BP@cror). Qflrs1 features are : -
t) TachYcardia@Gloe)
ii) Normal or increased cardiac output
iii) Tachlpnea
iv) Warm extremities
v) Decreased systemic vascular resistance
vi) Increased pulmonary vascular resistance
2) Late hypodynamic stage
i) Decreased cardiac output
ii) Hypotension
iii) Increasedvascular resistance
iv) Cool, mottled and often cyanotic extremities
v) Oliguria, renal failure
vi) Hypothermia

Capillary refilland arterialpulse palpation ln <hildren


o Capillary refill is a simple test that assesses how quickly blood returns to the skin after pressure is applied.
r It is carried out by applying pressure to the pink part of the nail bed of the thumb or big toe in a child and over the
sternum@Gre8) or forehead in a young infant for 3 seconds.
r The capillary refill time is the time from release of pressure to complete return of the pink color. It should be less than
3 seconds@NB r2).
o Prologed CET is the earliest sign signiflring impairment of microcirculation.
"
c Following are the important sites to examine arterial pulse in children -
t Neonates: Umbilical artery.
s 7 months - 8 years: Brachial artery@clo,).
s > 8 years: Carotid artery.

Pedlatric traGheostomy
r In most of the cases tracheostomy is performed with general anaesthesia and the patient intubated and. paralyzed..
r Neck is extended.
t A horizontal incision is made halfway between the cricoid cartilage and the sternal notch@cl 08).

o Subcutaneous fat and tissues are reflected, and deep cervical facia, is cut to expose thyroid isthmus.
o A vertical cut is'given in 2-3 or 3-4 rings in midline@Gr\8) and no part of the tracheal wall is removed.
I The endotracheal tube is withdrawn and a suitable size tracheostomy tube is simultaneously inserted.
o Post-operatively neck & chest radiograph are obtained to evaluate the position of the tube and to identify the
subcutaneous(Pcro8) emphysema & pneumothorax that could have developed as complication.
r Tube must be cleaned at frequent intervals.
c Patient should be nursed in and atmosphere of moist air@clo9).

Retrolantal broplasla
fi
o Retinopathy ofprematurity (Previously called as retrolental fibroplasia) is a characterized by abnormal proliferation
of small retinal blood vessels due to a variety of causes.
r The complication is related to high oxygen tension in the arterial blood (Hyperoxygenemia@cree)) rather than the
concentration ofoxygen in the inspired air.
r The condition is limited to preterm babies with a birth weight of less than 1500 gmArrMso2) or gestational age of less
than32 weeks.
Lowe's syndrome
o It is also known as oculocerebrorenal syndromearrMs02) of Lowe.
o It is X-linkedqllMs 02) disorder.
e It is characterizedby:-
1) Congenital cataract@IlMs02)
ii)
Mental retardationallMso2)
lii)
Fanconi syndrome-) AminoacidurinAqilMs 02)

o The disease is caused bymutation in the OCRLI geneAuusoz), which encodes the phosphatidylinositalpolyphosphate
5 -Pha sPh at as euilMS 02 )
.

Clinical features
o Progressive growth failure e Fanconi syndrome
o Blindness o Behavioural problems
. Hypotonia o Significant proteinuria
o Renal infuficiency

Stroke bite (Salmon patch/nevus simplex)


r It is small pale, ill-defined, vascular macules('G'0e) & represent localized vascular ectasia.
r Occur most commonly on glabella, eyelids, upper iip, & Nuchal area of 30-40o/o of normal newborn infants.
r More visible during crying or changes in enyironmental temperature.
o Most stork bites on the face go away completely in about 18 months. Stork bites on the back of the neck usually do not
go away.
c No treatment is needed.If a stork bite lasts longer than 3 years, It may be removed with a laser to improve the person's
appearance.

Ataxia telangiectasia
o Ataxia Telangiectasia (AT) is an autosomal recessive disorder. It is due to mutation of AT genelocated on chromosome
II. There is defective DNA repair that results in muitiple chromosomal breaks.
o Patients present in first decase oflife. Features are : -
i) Oculocutaneoustelengiectasia
ii) I'leurolo$ical -+ Ataxia, dysarthria, extensor plantar response, myoclonic jerks, areflexia, distal sensory deficity.
iii) Immunodeficeincy -+ Thymic hlpoplasia with celluIar and humoral (IgA and IgG2) immunodeficeincy.
iv) Recurrentpulmonaryinfections
v) Endocrine disorders -+ Tye I DM
vi) Premature aging
vrl) Malignancles -+ Lymphoma, Hodgkin's disease, acute T cell leukemia, breast cancer.
c a-feto protein and carcino-embryonic antigen are increasedAl 0a) .

Sudden infant death syndrome


o SIDS is defined as the sudden death of an infant that is unexpected by history and unexplained by a through
postmortem examination.
r Most cases of are between I month to 7 year'
o Incidence is higher in winter months.

Risk factors for SIDS


r Maternal and antenatal r Infant risk factors
r Smoking and alcohol use : Age (peak 2-4 months)
r Younger age (< 20 years) t Male gender(Ketutae7)
r Low educational status t PrematuritY(Kenta e7)

r Single marital status : Side or prone sleep position


r IUGR & intrauterine hypoxia : Recent fibrile illness
r Shorter interpregnancy interval r Soft sleeping surface (bed)
I Low socioeconomic status r Cold season
Suspected causes of SIDS
o Airway obstruction and laryngeal spasm. o Anaphylactic shock due to cor.y's milk allergy{r"*r,ezt
o Infection such as clostridium botulinum. o Chronic hlpoxia due to prolonged apenic episode.

Cartilage hair hypoplasia syndrome


o Carilage hair hlpoplasia is an unusual form of short limbed dwarfism with frequent and severe infections.
o It is an autosomal recessive condition.

Feature ofCHH
o Short limb dwarfism.
: Very fine thin light hairsremaseT) ar'd eyebrows.
r Hlperextensible joints of hands and feet but an inability to extend.
o Abnormalities of spine.
r Neutropenia4lluseT)
r Defective antibody mediated immunity.
. Severe combined immunodeflciency and combined immunodeficiency.
o Decreased number of T cells and defective T cell proliferation, cell mediated immunity is also decreased.

Pentalogy of cantrell
e Pentalogy of Cantrell is named after James R. Cantrell, a pediatric surgeon llgg2-lgS3). The pentalogy of Cantrell
is an extremely rare phenomenon with an incidence estimated at around 6 per million live births.
r It encompasses the following 5 main features.
i) Pericardial or distal sternal cleft s

ii) Supraumbilicalomphalocele
iii) Diaphragmatichernia
iv) Defect in apical pericardium causing ectopia cordia (Abnormal location of heart).
v) Congenitalintracardiacdefects(VSDr"u"), ASDI@NB13),Tetralogyoffallot@NB 13),Leftventriculardiverticulum).

Cardiosplenic syndrome or Heterotaxy syndrome


r Normal arrangement of organs and vessels within the body is called 'Situs solitus'. Complete reversal of organs and
vessels within the body is called "sifus invertus",Incomplete reversal/Random reversal of body organs is called "Sifzs
ambiguous".
r Heterotaxy refers to the variation in the normal arrangement of organs and vessels within the body.
o Visceral heterotaxy (Situs ambiguous) is primarily classified on the bases of the position of spleen which is embry-
ologically destined to be a Ieft sided organ. Other normal assymetrical viscera are placed more or less symmetrically.
t Situs ambiguous or situs indeterminus is generally expressed in two major forms based on the position of spleen and
whether the body resembles the normal right side or normal left side of the body.
i) Rightisomerism (Asplenia)rArrr)
r The spleen is tlpically absent and hence either side of midlin e resembles right side.
r Therefore, it is also calledbilateralright sidedness.
r It is usually associated with severe complex cardiac abnormalities and has poor prognosis.
ii) Left isomerism (Polysplenia)
r There are multiple small spleens placed on either side of midline, therefore, either side of the midline resembles
the lefi (splenic) side.
r Thus, it is also calledbilaterallefi sideness,
r It is associated with less severe cardiac abnormalities and has relatively better prognosis.

Rule of Palm for burn


r It can be used in Children as well as in adults,It is used/orsmallburns (<10o/o of BSA).
r The palm of patients hand represents 7o/o of BSAaI 11).

r Palm is the area ofhand from wrist crease to the finger crease.
o Entire burned hand represents 2.5o/o of BSA: llo/o for palm; to/o for dorsum of the hand (excluding fingers);0. 5o/o for
the fingers.
Sandifer syndrome
I Sandifersyndromemanifestswithbrief(aboutl-3min)NEPEs(NonEpilepticParalyticEvent)ofdystonicposturing
associated with gastro-oesophageal reflux; hiatal hernia is common.
o The attacks of torticollis with arching of the back and rigid ophisthotonic posturing, mainly involving the neck,
back and upper extremities. During the attack, the infant may become very quiet, or less commonly, very restless or
uncomfortable.
o It is most commonlyconfusedwith seizures(DNB'3). However, NEPEs of Sndifert syndrome occur within t hour after
feeding, often following and imposed change of posture, and the babies frequently have a history of vomiting, failure to
thrive and repeated chest infections. The EEG is normal, barium esophagogram or pH probe may dimonstrate reflux.

Williams syndrome
o Williams syndrome is a rare neurodevelopmental disorder characterized by -
i) Unusual facial features: Elfin facial6'*t ") upp"urance, widely spaced teeth, a long philthrum along with a low
nasal bridge, an unusually cheerful demeanor and ease with strangers'
ii) Heart defects: Supravalvular aortic stenosisarrMs
11).

iii) Development delay: Mental retardationqilMs 11)


, failure to gain weighlqutrs tt) (failure to thrive), low muscle tone.
iv) Transient hypercalcemia(AttMs ) tt

r Other features include GI problems (Colic, abdominal pain, diverticulitis), nocturnal enuresis, dental irregularities,
hypothyroidism, hlperacussis and photophobia.

Neuroenteric cysts
o These cysts result from incomplete separation of the notochord from the foregut which causes a persistant
communication or adhesion between the ectoderm of spinal cord and endoderm of the foregut.
o The lining epithelium of these cysts are enilodermal in origin.
o These cysts are associated with -
i) Vertebral anomaliesal0s) -+ Hemivertebrae, spina bifida, fused vertebrae, absent vertebrae,
(mostcommon) diastematomYelia.
ii) lntestinal aiomalies -+ Duplication, malrotation, imperforate anus.
iii) Others -) Cloacal / bladder exstrophy, renal agenesis.

o Most of the neuroentgric cysts are located


i) Intraspinal -+ MC at cervicothoracic region.
ii) Mediastinum -+ MC in Posterior mediastinum on right side.

tO:E f eft Uentrld e, divertliul um.

:.Ob tv'{t- er ir 6ui1,1ts.,io,:weigh,l,Qiiir:),:rre-coc!ous:pupert!.'(therq is

fetal response to aiute hypoxra : Bradycardia.

xeiratentAt.ibrn piiitie]rj aisoiiaiedwrrhl:Pieryaturiiy and l qw,br! rlh


t Lowe's syndrome is charocterized by: Congenital cataract, mental retardation, Fanconi syndrome (aminoaciduria), defects in CNS.
a Not o feature orf.o*i**yndianre,i:unAeldndaa 5;..,.,,.i:,. .
'':
o Treatment of kawasaki disease in children : lV immunoglobulin.
a ruor;:iaaf,fe #;*a :a:boi*,llinein rhrldren : Peri p heral ;nquroparhy
I Most common fetal response to acute hypoxia: Bradycardia.
..
t O p tie, ne ryis I i arii a:ih c{li tdren : Pilocytit glioma;
I Stroke bite lesion : Macular staining (vascular macules) in infant.
?
e Chromosomal anomalies more than 200/o is associated with: Omphalocele (43olo).
I :ld*inirrraan Afairrariniiifu*prai'n"iii'iiuiis,ia.ferus:tiiniofaclal malformation{na.sal.hypoptasii}':ina,qr,,;na@it,E#,.,:,
punctata.
a Most common cause of leukocytoclastic vasculitis in children: Henoch schonlein purpura,
'*
metabolic acidosis.
* Most common cause of vaginal bleeding in children: Foreign body.
* Coimbination of atoxio, retinitis pigmentosa and ichthyosis is seen in : Refsum's disease.
& Sphenoid wing dysplasia is seen ln : Neurofibromatosis type-1.
a

ttt
QUESTIONS

Cantrell pentology include all exce?l - (CET Nov. lj Pauern) c) Bradycardia d) Arrhythmia
a) VSD b) ASD 12. In children, catecholarnine resistance shock man-
c) TOF d) Trisomy 21 aged by - (CET July 15 Pattern)
) Right Sided Isomerisrn is associated with - (Ar 1t) a) Activated protein-C
a) Asplenia b) Hydrocortisone
b) One spleen c) Nor-adrenaline
c) Two spleens d) Vasopressin
d) Polysplenia 13. Retrolental fibroplasia has association with-
-r. Cat bites in child treatment-true is - a) Prolonged labour (AIIMS May 02)
(All India Dec15 Pattern) b) Intrauterine infection
a) Cleaning the wound thoroughly c) Meconium aspiration
b) Puncture wound most common d) Low birth weight
c) May require rabies vaccination 14. Retrolental fibroplasia is due to ^ (All India Dec.14 Pattern)
d) All of above a) Hlpocapnia
4. A fiveyear old child presents to the ernergencyde- b) co2
partment with burns. The burn area corresponding c) Hlpoxia
to the size of iris palm is equal to (Ar 11) d) Hyperoxygenemia
a) 1% BSA b) 5%o BSA 1-5. Retinopathy of prematurity is cornmonly predis-
c) 10% BSA d) 20% BSA posed by - (AIIMS lune 2000)
5. Sandifer syndrome due to GARD in infants is con- a) Less gestation age
fused with ? (CET Nov. 13 Pattern) b) Low birth weight
a) Seizures c) 02 toxicity
b) Recurrent vomiting d) Carbohydrate excess
c) Acute otitis media 1$. Gliorna o{ optic nerye is usually - (NEET Dec. 12 Pattern)
d) Sinusitis a) Gemistocltic b) Pilocltic
6. All of the following about Yitamin D metabolisrn c)* Fibrillary d) Lamellar
are true except - (AIIMSNov 11) 17. n owe's syndrome is characterised try the following
a) 21-a,hydroxylation takes place in liver except - (AtrMS 02)
b) 1-<r hydroxylation takes place in kidney a) Mental retardation
c) Daily requirement in the absence of sun-light is b) Undescended testes
4s0-600lUlday c) Defect in the CNS and eyes
d) Williams syndrome is associated with obesity, d) Aminoaciduria
mental retardation, precocious puberty lo A 15 days old baby is brought to the hospital with
7. What is the protratrle diagnosis for a cyst in a child complains of seizures. Blood tests revealed serum
which is located at and associated with vertebral Ca++= 5 mg/dl, Phosphorous levels = 9mg/dl, and
defects - (Ator) PTH levels 3Opg/ml (normal range= f 0-60 pg/ml).
a) Myelocele What is the most probable diagnosis? (Ar 11)
b) Bronchogenic cyst a) Pseudohlpoparathyroidism
c) Neuroenteric cyst b) Vitamin D deficiency
d) Neuroblastoma c) Hlpoparathyroidism
8. Capilary rcfilI time ia child with shock is ? d) Hypoxic ischemic encephalopathy
(CET Aug. 12 Pattern)
19. Stroke bite lesion are seen in - (PGl lune 09)
a) >1 second b) >2 second a) Sturge weber syndrome
c) >3 second d) >4 second b) Blue rubber bleb ner,us syndrome
o Best site to demonstrate capillary filling in infants- c) Macular staining of infant
(CET June 14 Pattern) d) Craniofacial neurs
a) Nail bed b) Sternum 20. P<rsterior iliac horns are seen in - (AUMS May 02)
c) Ear lobule d) Great toe a) Fishert syndrome
10. Best artery to palpate for pulse in infants is - b) Crouzan syndrome
a) Femoral a b) Radial a @Gt Dec 2k) c) Nail patella syndrome
c) Carotid a d) Brachial a d) Pierre Robbins syndrome
11" The most common fetal response to acute hlpoxia 21. In neoaate, intra rnuscular injection given at -
ls- (All India Dec15 Pattern)
a) Tachycardia b) Tachypnea (Ar oe) a) Deltoid b) Gluteal
c) Thigh d) Abdomen c) Hypocalcemia
22. Drug dosage best decided in child by - d) Tacrolimus Toxity
a) Weight (CETluly 15Pattern) 30. Single unrblical artery is cssociated vrith-
b) Age a) NTD (All India Dec15 Pattern)
c) Height b) Hydrops fetalis
d) Investigation c) Congenital heart disease
23. Ataxia telangiectasia is characterised by all ofthe d) In utero death
followingexcept - (Ar04) 31. A study under Australian collaborative trial on
a) Chronic sinopulmonary disease steroids use in neonates was done. Which of the
b) Decreased level of q-fetoprotein following is true - (AIIMS May 11)
c) Chromosomalbreakage a) No difference between placebo & corticosteroid
d) IgA deficiency b) Corticosteroid to children causes behavioural
24. Chromosomal anomalies more than 20/o is associat- worsening
edwith - (AIIMS Nov 09) c) Corticosteroid to children causes reduction in head
a) Gastroschisis circumference
b) Omphalocele d) Corticosteroid to children causes neuro sensitivity
c) Spina Bifida degradation
d) Cleft Palate GBSin addldtreatonent- (NEET Dec.12 Pattern)
Which of the following statements regarding Kawa* a) IVIg
saki disease is true - (AIIMS Nov 11. AI 08) b) Ventilation
a) Associated with coronary artery aneurysm in up to c) Plasmapharesis
of untreated cases
250/o d) All of above
b) It is the mostcommon cause of vasculitis in Iron poisoning in 4 year child, Rx includes -
children a) Stomach lavage (PGl lune 2000)
c) IV immunoglobulins are recommended only if b) Desferrioxamine IV 100 mg
coronary artery is involved c) X-ray abdomen
d) Lymph node biopsy is used for diagnosis d) Blood transfusion
Treatment of kawasaki disease in children is - 34. Increased sweat chloride is seen in all except - (AI 9s)
a) Oral steroids (AIIMSNoy 11, AI 01) a) Ectodermaldysplasia
b) IV steroids b) Nephrogenic diabetes insipidus
c) IVIg c) Glucose 6 phosphatase deficiency
d) Mycophenolate mefentil d) Obesity
1,7 The most common leukocytoclastic vasculitis affect- A 5 yearotrdc&ildwith ge$ u&derdiaphragm is see*
ing children is - (AIIMS May 03, Al 0s) tn- (All India Decl5 Pattern)
a) Takayasu disease a) Enteric fever with intestinal perforation
b) Mucocutaneous lymph node slmdrome. (Kawasaki b) Chilaiditi'ssyndrome
disease) c) Iatrogenicpneumoperitoneum
c) Henoch Schonlein purpura d) All of above
d) Polyarteritis nodosa
36. Grisel syndromeis- (All India Dec15 Pattern)
28. A child with an obvious rash presents with recurent a) Glucosaminidasedeficiency
infections. Investigation revealed decreased platelet b) Atalanto-axial dislocation
count and reduced lgM. Which of the following is c) Hexosaminidasedeficiency
the most likely diagnosis - (Ar 12) d) L4 - L5 displacement
a) Idiopathic Thrombocytopenic Purpura 37. Thirteen pair af Ribs are seen in ? (All India Decj5 pattern)
b) Thrombotic Thrombocytopenic Purpura a) Down slmdrome
c) Wiskott-AldrichSyndrome b) Holt oram
c) Tirrner d) Fibrous dysplasia
d) Di'George anomaly
29. A sixyear old child is admitted to the Paediatric ICU
for seizures. He has been on tretment with Tacroli-
mus and Prednisolone. On investigations his blood
urea is 68 mg/dl, Serum Sodium is 136 meqll likely
cause ofhis seizures- (Ar 12)
a) Uremia
b) Hyponatremia

III
Cse..p.tE.n :r.7

ANSWERS
l. A.ns, is X'
i.e", Trisomy 21 {Ref: Rennie d* Il.aberton's {exth*ak al neawatalogy by tra*et M. Rennie Tth/e Chap 6,

Newbarn swrgery 2"d ed l3y Frew Furi Z't/e p" 24A; t{01?"L) guide ta r*re disarders 28{}3 e p. 2341
r Pentalogy of Cantrell encompasses the following 5 main features.
i) Pericardial or distal sternal cleft
ii) Supraumbilicalomphalocele
iii) Diaphragmatic hernia
iv) Defect in apical pericardium causing ectopia cordia (Abnormal location of heart).
v) Congenital intracardiac defects (VSD, ASD, Tetralogy of fallot, Left ventricular diverticulum)"
,,
Ans is 'd i.e., Asplenia lRef: Nelson 1.8'h/e p. 1929)

tuit,,:i:re*ttili
Absent Multiple
iidedness (isomerism) B,]Jatgral.right't,,.,',,,, :
Bilateral left
Lungs Bilateral trilobar with eparterial Bilateral bilobar with hyparterial
bronchi bronchi
5ex Male (657o) Female > male

Right-sided stomach Yes Less common


Symmetric liver Yes Yes l',, .:: .,,., ,.,,. ,.. :

Partial intestinal rotation Yes Yes

Dextrocardia (o/o) 30-40 30-40


Pulmonary blood flow Decreased (usually) lncreased (usually)
Severe cyanosis Yes No

Transposition of great arteries (o/o) 60-75 15

Total anomalous pulmonary 70-80 Rare


venous return (o/o)

Common atrioventricular valve (o/o) 80-90 20-40


4o-50 10-15

Absent inferior vena cava with No Characteristic


azygos continuation
Bilateral superior vena cava Yes' .' . : ' :
Yes

Other common defects Pulmonary atresia, Pulmonary Partial anomalous pulmonary


stenosis venous return, ventricle septal
defect, double-outlet right
ventricle
Risk of sepsis Yes No

Howell-Jolly and Heinz bodies Yes No


pitted erythrocytes
Absent gallbladder; biliary atresia No Yes

J. Ans" is 'd' i.e., All


qlf ahov* l[ilef: Ne/'saw tr9't'/e ck. 712]
r The most common tlpe of injury from cat and rat bites is a puncture wound. Cat bites often penetrate to deep tissue
r Infection is the most common complication of any tlpe of bite injuries.
r Treatment includes cleansing of wound, debridement, wound culture, tetanus and antirabies immunization, and
initiation of antibiotics
t Amoxicillin-clayulanate is an excellent choice for empirical oral therapy for human and animal bite wounds because of
its activity against most of the strains of bacteria that have been isolated from infected bite injuries,
Ans is 'a' i.e., f % BSA lRef: Nelson's 18th/e p. 4521

r The palm of patients hand represents 7o/o of BSA. Palm is the area of hand from wrist crease to the finger crease.

5. Ans. is '* i.e., Seizures tRel A clinical guide to epileptic syndromes and their treatmeut by C"P. Panayiotopoulos p.
1121

o It most commonly confused with seizures. However, NEPEs of Sndifer's qmdrome occur within t hour alfter feeding,
is
often following and imposed change of posture, and the babies frequently have a history of vomiting, failure to thrive
and repeated chest infections.

6. Ans is t' i.e., Williams syndrome is associated with obesity, mental retardation, precocious puberty
lRef: Chilil development by Berk 4th/e p. 6421
o In Williams syndrome there is failure to gain weight (not obesity) and delay in development (not-precocious puberty)
o 2S-hydroxylation of vitamin D occurs in liver.
o Thefinalhydroxylationinthekidneybyenzymel-ahydroxylaseistheratelimitingstep-enhancedbyparatharmone.
o 7, 25 dihydroxy Vit Dr(calcitriol) is the activeform of Vit Dr.
o Recommended dietary allowance (RDA) per day for children and adults is 600 IU (15 p g).
7. Ans. is t' i.e., Neuroenteric cyst lRef.: Sahiston 18th/e p. 1696; Internet)
o Vertebral defect in
association with intraspinal cyst suggest the diagnosis ofneuroenteric cyst.
o Any cyst in mediastinum (posterior), abdomen or intraspinal, in association with vertebral defect, suggest the diagnosis
of neuroentric cyst.
8. Ans. is'c' i.e., >3 second {Ref: hUp//ww,unicef.org/india/F8C_Participants_!4anual,pdfl
o It should be less than 3 seconds.
9. Ans. is'b- i.e., Sternum {Ref: O.P. Ghai 6h/e p. 652 6 //e p. 688)
o It is carried out by applying pressure to the pink part of the nail bed of the thumb or big toe in a child and over the
sternum or forehead in a young infant for 3 seconds.

10. Ans. is 1d' i.e., Brachial Artery fRef: Nelson 18th/e p. 1860; MyungKPark Sth/e p. 15, 16]

o Checking and Pulse :


r Neonates - Umbillical artery
r 1-8 year - Brachial artery
r > 8 year - Carotid artery

11. Ans. is t'


i.e., Bradycardia lRef: Rudolph's 21i/e p. 711
o The initial response of the normal fetus to acute hypoxia or asphyxia is bradycardia results from chemoreceptor
stimulation of vagal nerve.
t2. Ans. is 'b' i.e., Hydrocortisone {Rel Ghai Vh/e p. 700)

e In catecholamine resistance shock hydrocortisone is used.


Ans is 'a' i.e., f % BSA lRef: Nelson's 18th/e p. 4521
o The palm of patients hatd represents 7o/o of BSA. Palm is the area of hand from wrist crease to the finger crease.
5. Ans. is'a'i.e,, Seizures [Rel A clinical guide to epileptic syndrames and their treatment by C.P, Panayiotopoulos p.
1121

o It is most commonly confused with seizures. However, NEPEs of Sndifert sl.ndrome occur within t hour after feeding,
often following and imposed change of posture, and the babies frequently have a history of vomiting, failure to thrive
and repeated chest infections.

6. Ans is 'd' i.e., Williams syndrome is associated with obesitp mental retardation, precocious puberty
lRef: Chilil development by Berk 4th/e p. 6421
o In Williams syndrome there is failure to gain weight (not obesity) and delay in development (not-precocious puberty)
o 25-hydroxylation of vitamin D occurs in liver.
o Thefinalhydroxylationinthekiilneybyenzymel-ahydroxylaseistheratelimitingstep-eflhancedbyparatharmone.
o 7, 25 dihydroxy Vit Dr(calcitriol)-is the activeform of Vit Dr.
o Recommended dietary allowance (RDA) per day for children and adults is 600 IU (1S p g).
7. Ans. is t' i.e., Neuroenteric cyst fRef; Sabiston 18'h/e p. 1696; Internetl
o Vertebral defect in association with intraspinal cyst suggest the diagnosis of neuroenteric cyst.
o Any cyst in mediastinum (posterior), abdomen or intraspinal, in association with vertebral defect, suggest the diagnosis
o[ neuroenlric cyst.
8. Ans. is 'c' i.e." >3 second {ReJ: http//ww.unicef.org/india/FBC-Participants-Manual.pi$)
o It should be less than 3 seconds.
9. Ans. is'-o- i.e., Sternum {Rel O"P Ghai 6h/e p. 652 (t 7/e p. 6881
r It is carried out by applying pressure to the pink part of the nail bed of the thumb or big toe in a child and over the
sternum or forehead in a young infant for 3 seconds.

10. Ans. is 1d' i.e., Brachial Artery lRef: Nelson 18th/e p. 1860; Myung K Park Sth/e p. 15, 16j

r Checking and Pulse :

r Neonates - Umbillical artery


r 1-8 year - Brachial artery
r > 8year - Carotid artery

11. Ans. is t'i.e., Bradycardia lRef: Ruilolph's 21"t/e p. 711


r The initial response of the normal fetus to acute hlpoxia or asphyxia is bradycardia results from chemoreceptor
stimulation of vagal nerve.
t2- Ans. is'b' i.e., Hydrocortis one {Rel Ghai 7 / e p. 7 00)

o In catecholamine resistance shock hydrocortisone is used.


Guidelines for management of shock

Recognise shock
Give bolus 20 ml/kg NS opto 60 ml/kg

Start dopamine/Dobutamine
+
Dopamine/dobutamine resistance
shock

shock
Warm
++
Startnorepinephrine Startepinephrine
Cold shock

Give hydrocortisone
+
Add vasodilator or iype lll PDE
inhibitor with epinephrine/
norepinephrine
+
Persistent catecholamine
resistant shock
+
Consider ECMO

13. Ans. is 'd' i.e., Low birth weight lRef: Meharban Singh 6h/e p. 404; O.P. Ghai 8'h/e p. 666 6 7h/e p. 6451
c "Retrolental fibroplasia (retinopathl of prematurity) is generally noted some weeks after birth in premature
infants who have been given high concentration of oxygen, and is usually confineil to those under 1.7 Kg in weight at
birthl' --- Parson, l8/e

14" Ans. is 'd' i.e., Hyperoxygenemia lRef: O.P. Ghai Bth/e p. 665 b Vh/e p. 6451

15. Ans. is 'a' i.e., Less gestation age lRel O.P. Ghai 8th/e p. 665 b 7/e p. 6451
16. Ans. is'b'i.e., Pilocytic fR.ef: Nelson 18'h/e p. 26081

Optic nerve glioma


o Optic nerve glioma, also referred to as juvenile pilocltic astrocytoma, is the most frequent tumor of the optic nerve
in childhood.
o It is a developmentalhamartoma.
o It is.either stationary or only slowly progressive.
o Tirmor may arise in any portion of nerve -+ i) Intraorbital, ii) Intracanalicular iii) Intracranial

17. Ans. is'b' i.e., Undescendended testes fRef Nelson 18th/e p. 2198)
r It is characterized by : (r) Congenital cataract (i1) Mental retardation (111) Fanconi syndrome -+ Aminoacidut'ina

18. Ans. is t'i.e., Hlpoparathyroidism fRef: Reailbelowj


o This child has :-
i) Hypocalcemia
ii) Hyperphosphatemia
iii) Normal PTH levels
r Option b & d can be ruled out because :-
i) In Vitamin D deficiency :- serum calcium is normal or slightly reduced (7.5 - 8 mg/dl); serum PC :::-i or
slightly reduced; and parathormone levels are increased.
ii) hypoxic ischemic encephalopathy is not related to calcium and phosphate metabolism.
. Now, we are Ieft with Pseudohypoparathyroidism and hypoparathyroidism both of which have hypocalcemia and
hlperphosphatemia.
r In Pseudohlpoparathyroidism, there is increased concentration of parathyroid most of the times. Thus, this option is
also excluded.
o Coming to hypoparathyroidism: As the name suggests, PTH is low in hlpoparathyroidism. But, sometimes PTH may
be in normal range (10-60 Pglml) and this is inappropriately normal PTH.
However itis generallybelievedthat'Normal'levels of PTtI in apatientwithhypocalcemiaindicate'inappropriately
normal PTH' and relative Hypoparathyroidism should always be considered as the initial diagnosis.
'Serum PTH levels should be related to the concurrent serum calcium. High PTH k the normal response to hypocalcemia
and therefore'normal' PTH in a patient with llypocalcemia indicates relative Hypoparathyroidism' - Practical Algorithms
in Pediatric Endocrinology' by Hochberg (2007)/7 1.
'Because normally hypocalemia is a potent stimulus
for seuetion of PTH, patients with intact parathyroid gland function
should be expected to have high circulating levek of PTH, during hypocalcemic episodes. Low levels of serum PTH and
in some assays eren inapropriately 'normal' levels despite hypocalcemia should prompt consideration of various form of
hypoparathyroidism' - Principles and Practice of Endocrinology & Metabolism'by Beeker.
19. Ans. is t'i.e., Macular staining of infant
20. Ans is t' i.e., Nail-patella syndrome {Ref: Nelson 18th/e p. 27j1, 28821

Nail Patella syndrome is characterized by:


o Small or absent nails o Small or absent patella t Posterior iliac horns o Renal anomalies
o Overextension of joints o Skin laxity o Hyperhydrosis
21. Ans. is t'i.e., Thigh
o Anterolateral aspect ofthigh because oflack ofimportant blood vessel & nerve is preferred site upto 12 month ofage.
)) Ans. is 'a' i.e., Weight fRef: Textboak af pharmaeolagy far pecliatrics p. 785]
"Most drugs in children are dosed according to body weight (mg/kg) or body surface area (mg/m2)".

23. Ans. is 'b' i.e., Decreased levels of cr fetoproteins lRef: Nelson lVh/e p. 699; Harrisan lVh/e p. 25711
"Persistance of very high levels of oncofetal proteins, including alpha fetoproteins and carcinoembryonic antigens
is of diagnostic value" - Harrison
24. Ans. is 'b' i.e., Omphalocele fRef. Nelson 18th/e p. 776, Cloherty 6*/e p. 6271

r Chromosomal abnormalities have been reported in as many as 43% fetuses with omphalocele. The most common
being trisomy 13,18,21 and Turner syndrome.

25, Ans. is 'd i.e., Associated with coronary artery aneurysm in up to 25Yo lRef: Nelson 18th/e p. 10381
"Coronary artery aneurysm develop in up to 250/o ofuntreated patients" - Nelson
r Most common cause of vasculitis in children is HSP.
r IV immunoglobulin is the treatment of choice for Kawasaki disease.
Lymph node biopsy does not aid the diagnosis of Kawasaki disease (not used)
26. Ans. is t' i.e., I.V. immunoglobulins lRef: Nelson 18th/e p. rc40, 10411
27. Ans. is t' i.e., Henoch Schonlein purpura fRef. Rudotph's 21il/e p. 1117]
28. Ans. is t' i.e., Wiskott Aldrich Syndrome lRef. Harrison lVh/e p. 206A; Hffiman 4th/e p.B47j
o The classical triad of rash, thrombocytopenia and recurrent infection (immunodeficiency; tr tgfulS suggest a
diagnosis of Wi skott- Aldr ich Sy ndrome.

29. Ans. is t'i.e., Tacrolimus Toxity lRef Readbelow)


r Tacrolimus neurotoxicity is an established cause for 'seizures' and the most likely reason for seizures in this child.
r In hlponatremia, seizures usually develop when sodium levels decrease below L25 meqlL.
e In hlpocalcemia, seizures develop when calcium level is below 6 mg/dl.
r Uremic seizures occur late after delirium, hallucinations and agitation.

30. Ans. is'c' i.e., Congenitatr heart diseaselRef: Nelson lSth/e p. 1(|5\
o Approximately j0% of infants with a single umbilical artery have congenital abnormalities.
o Trisomy 18 is one of the more frequent abnormalities.
most common congenital anomalies in chromosomally normal
' The
1) Genitourinary (6.480/o)
fetuses and neonates were.

2) Cardiovascular (6. 25o/o )


3) Musculoskeletal (5.44o/o)
31. Ans is 'b' i.e., Corticosteroid to children causes behavioural worsening
{Ref: www.newbornwhocc.org)
r The Australian Collaborative Trial of Repeat Dose of Steroids (ACTORDS)
conducted as multicentric RC! in Australia
and New Zealand by Crowtheriv, v and colleagues enrolled 982 pregnant women less
than 32 weeks to receive either
single dose of intramuscular Betamethasone or salined placebo weeklytill 32
weeks of gestation or deliverywhich ever
was earlier' This study showed some short term benefits in the repeat corticosteroid
group with significant reduction
in RDS (33o/o vs.41%, RR = 0.82195o/o Cl 0.71-0.91)1. There was also shorter duration
of mechanical ventilation, lesser
need for oxygen therapy and less severe lung disease. The mean weight, length
and head circumference were not dif-
ferent' However the weight and head circumference Z scores were lower in the repeat
steroid dose group. In the long
term follow up these infants at 18-22 months corrected age there was no difference in growth,
malor disability or lung
disease between the two groups. There was increased occurrence of attention problems
in repeat steroid group.
32. Ans. is ?' i.e., Atrl of above {Ref: O.p. Ghai Se/e p. 591 & Vh/e p. 554)
Treatment of GB syndrome in children
r It is self limited in majority of cases.
o Intravenous immunoglobulin shows good response.
c Plasmapheresis :- Removal of autoantibodies.
t Assisted ventilation:- It patient develops respiratory muscle paralysis.
r Physiotherapy
33. Ans. is 'b'i.e., Desferrioxamine IV 100 mg &'c' i.e., X-ray abdomen p.
[Re/: KDT 6h/e 6461
lron poisoning
o Ingestion of a number of ferrous sulphate tablets may cause acute iron poisoning.
Clinical features
r Severe vomiting
r Diarrhea
o GI bleeding
r There may be severe shock, hepatic & renal failure
Treatment of lron poisoning
r Because iron is radio opaque an abdominal radiography may confirm
the ingestion. Repeat radiograph may help
with assessment of efficiency of gastric decontamination methods.
r Ipecac-induce emesis may be used to remove tablets from the stomach.
r Gastric lavage is not recommended in children because of its inefficiency,
r Activated charcoal does not adsorb iron and should not be used.
o Whole bowel irrigation may be of benefit.
r If tablets adhere to gastric mucosa, removal by endoscopy.
r Desferrioxamine is a specific shelter of iron and is the antidote of choice.
3-1. Ans. is U'i.e., Obesity [Ref : Nelson 18*/e p. 1808j
Increased sweat chloride is seen in -
i) CysticJibrosis v) Familialcholestasis ix) Untreated adrenal insuficiency (Addison clisease)
ii) Malnutrition vi) Ectodermal dysplasia x) Hereditary nephrogenic diabetes insipidus
iii) Pancreatitis vii) Fucosidosis xi) Hypoparathyroidism
iv) Hypothyroidism viii) Mucopolysaccharidosis x ii ) Glu co se - 6 - ph o sph atas e d efi c i e ncy

-1J. Ans. is 'd' i.e., All of above fRef: BMI case report 2012, I. pediatric surgery 19911
r Chilaiditi's syndrome : Condition characterised by interposition of small or large bowel between
liver and right
diaphragm. Radiologically it gives gas under diaphragm.
o Iatrogenic pneumoperitoneum : In certain procedures like peritoneal dialysis,
Iatrogenically air pushed before putting
PD cannula to avoid injury ofviscera in such case gas under diaphragm can be seen
and all cases when intestine or
viscera perforate we can get gas under diaphragm.
q
36. Ans. is'H i.e., Atalento axial dislocation [Ref Netsan l&*le Chapter 679]
Grisel's syndrome
o Spontaneous subluxation of the atlanto-axial joint following peri-pharyngeal inflammation is known as Grisel's
syndrome.
o Children are most frequently afected and classically have torticollis associated with neck stiffness or pain upon neck
movement.
37. Ans. is 'a' i"e., Down syndrome lRef Abnonnel number affetuI rtbs in USG by Liet Gtudes et.al.j

ITI

You might also like